Rosen's MCQs 10th Edition (1) .
Rosen's MCQs 10th Edition (1) .
Rosen's MCQs 10th Edition (1) .
ROSEN’S Editor-in-Chief
Ron M. Walls, MD
Senior Editors
Robert S. Hockberger, MD Marianne Gausche-Hill, MD Timothy B. Erickson, MD, Susan R. Wilcox, MD
Chair Emeritus Medical Director FACEP, FACMT, FAACT Chief, Division of Critical Care
Emergency Medicine Los Angeles County EMS Agency; Department of Emergency Medicine Department of Emergency Medicine
Harbor-UCLA Medical Center Professor of Clinical Emergency Medicine Brigham and Women’s Hospital; Massachusetts General Hospital;
Torrance, California; and Pediatrics Chief, Division of Medical Toxicology Associate Professor of Emergency Medicine
Emeritus Professor of Emergency Medicine David Geffen School of Medicine at Mass General Brigham; Harvard Medical School
David Geffen School of Medicine at UCLA University of California, Los Angeles Associate Professor of Emergency Associate Chief Medical Officer
Westwood, California Los Angeles, California; Medicine Boston MedFlight
Clinical Faculty Harvard Medical School Boston, Massachusets
Departments of Emergency Medicine and Boston, Massachusetts
Pediatrics
Harbor-UCLA Medical Center
Torrance, California
Collected by:
Dr. Sadiq Ameen
SECTION ONE Resuscitation and Analgesia
1
KEY CONCEPTS
Airway
appreciated in cardiac arrest patients. In these situations, a bulb aspi-
ration device may provide helpful information, even though this tech-
• A nticipating the clinical course of the patient’s condition and assessing the nique is generally not as reliable as ETco2 detectors. The other listed
likelihood of deterioration are crucial to the decision to intubate, especially options, traditional as they may be, are prone to failure and should not
if the patient is to leave the emergency department (ED) for a period of time be relied on for confirmation of tube placement.
(e.g., interfacility transfer, diagnostic testing). 3. During rapid sequence intubation (RSI) of a hypotensive blunt
• Although videolaryngoscopy (VL) has reduced the chance of a failed intuba- abdominal trauma patient, which of the following will mitigate the
tion attempt caused by difficult anatomic features that often thwart direct risk of circulatory collapse during emergency airway management
laryngoscopy (DL), an assessment of the patient for potential difficult intu- if performed before medications are administered?
bation, bag-mask ventilation (BMV), ventilation using an extraglottic device a. Central venous access
(EGD), and cricothyrotomy is an essential step before a neuromuscular b. Choosing propofol instead of ketamine for induction
blocking agent (NMBA) is administered. The mnemonics LEMON, ROMAN, c. Resuscitating with packed red cells
RODS, and SMART can serve as useful aids. d. Obtaining an abdominal computed tomography (CT) scan to
• Physiologic derangement can contribute to morbidity and mortality during better characterize the degree of bleeding
emergency airway management. Cardiovascular optimization with fluids, Answer: c. Before induction and paralysis, the preintubation optimiza-
blood, and pressor agents should be undertaken, when time allows, to tion step of RSI suggests that volume resuscitation with either isotonic
reduce the risk of circulatory collapse and cardiac arrest. fluids or blood in a hemodynamically unstable patient should be per-
• In the absence of a crash patient (agonal, unresponsive to laryngoscopy) or formed, if time allows, to reduce the chance of cardiac arrest during or
difficult airway, rapid sequence intubation (RSI) is the airway management immediately after intubation.
method of choice for ED patients. 4. In which of the following conditions is succinylcholine contraindi-
• Tube placement confirmation using end-tidal carbon dioxide (ETco2) is cated?
essential after intubation; failure to detect adequate quantities of exhaled a. Acute burn <5 days
CO2 is evidence of esophageal intubation until proven otherwise. b. Acute head injury secondary to motor vehicle accident
• VL increases first-attempt intubation success, even when compared with c. Acute spinal cord injury <5 days
DL combined with various optimization techniques. First- attempt suc- d. Renal failure with a serum potassium level of 4.7 mEq/L
cess (FAS) is associated with fewer adverse events and better outcomes. e. Multiple sclerosis
Emergency airway managers should learn, and adopt, VL as the method of Answer: e. Succinylcholine has been associated with severe fatal hyper-
choice for emergency intubation. kalemia when administered in specific clinical circumstances. The risk
• Cricothyrotomy is indicated in the “can’t intubate, can’t oxygenate” failed air- of succinylcholine-induced hyperkalemia in patients with denervation
way situation and should be performed once this has been identified. Delays syndromes begins with the onset of disease and continues indefinitely.
may increase the likelihood or severity of hypoxic injury to the patient. With respect to acute burns, trauma, stroke, spinal cord injury, and
• Etomidate is used in more than 90% of all RSIs. Either rocuronium or succi- intra-abdominal sepsis, the risk of hyperkalemia with succinylcholine
nylcholine is a reasonable NMBA for use during RSI. Rocuronium has less use becomes evident 5 days after the onset of injury or disease process.
potential for adverse effects but a longer duration of action. Succinylcholine is not contraindicated in renal failure; however, known
• EGDs are rarely used in ED airway management but offer additional options elevations in the potassium level may warrant use of another neuro-
for rescue oxygenation of the failed airway and are used in many prehospi- muscular blocking agent.
tal systems. 5. Which of the following provides the highest ETo2 after 3 minutes of
ambient pressure tidal volume breathing?
1. Which of the following is considered unreliable for assessing the a. Nonrebreather mask with oxygen flow at 15 L/min
need to establish an artificial airway? b High-flow nasal cannula
a. Absence of a gag reflex c. Nonrebreather facemask at flush rate oxygen (40 to 90 L/min)
b. Absence of swallowing on command d. Venturi mask
c. Level of consciousness Answer: c. Flush flow rate oxygen using a nonrebreather mask pro-
d. Patient’s ability to phonate vides an ETo2 of 85%. This is in comparison with an ETo2 level in the
e. Pooling of secretions in the oropharynx mid-50s with a nonrebreather mask at 15 L/min flow. The increased
Answer: a. The gag reflex can be absent in up to 25% of normal adults. inflow rate outcompetes room air entrainment around the margin of
Moreover, there is no evidence that the presence or absence of a gag reflex the mask and increases the fraction of inspired oxygen (Fio2) resulting
corresponds to a patient’s ability to protect his or her airway. Therefore it in better nitrogen washout.
should not be used as an indicator of the need for intubation. 6. Until how long after an acute burn is succinylcholine considered
2. Which of the following is the most reliable overall method for con- safe to use for RSI?
firmation of correct tube placement after endotracheal intubation? a. 30 minutes
a. Chest and gastric auscultation b. 12 hours
b. Chest radiography c. 24 hours
c. Detection of colorimetric or quantitative end-tidal carbon diox- d. 48 hours
ide (ETco2) e. 5 days
d. Measurement of exhaled volume Answer: e. Succinylcholine can produce severe (and fatal) elevations
Answer: c. Detection of ETco2 after endotracheal intubation is the in serum potassium levels after administration in patients with burns.
most reliable of the options listed for the confirmation of tube place- However, this vulnerability to succinylcholine-induced hyperkalemia
ment. (A fiberoptic scope passed through the endotracheal tube, with is not clinically significant until at least 5 days after the acute burn. As a
visualization of the tracheal rings, is the gold standard but is not gen- result, succinylcholine remains the paralytic of choice if rapid sequence
erally required.) Limitations of colorimetric CO2 detection should be intubation occurs less than 5 days after the burn.
2 Mechanical Ventilation and
Noninvasive Ventilatory Support
KEY CONCEPTS Answer: d. Positive-pressure ventilation (PPV) is associated with sev-
eral complications. Some of these can quickly become life-threatening.
• N oninvasive positive-pressure ventilation (NPPV) is often adequate for This reality underscores the importance of familiarization with the
reversal of impending respiratory failure due to a rapidly reversible cause, common problems that arise as a result of PPV. Most complications
and should be considered as the first-line therapy for patients with exacerba- result from changes in thoracic physiology when positive pressure is
tions of chronic obstructive pulmonary disease (COPD) and acute cardiogenic present for part or all of the respiratory cycle. Potential adverse effects
pulmonary edema (ACPE) in whom immediate intubation is not required. of PPV include an increased work of breathing because of asynchrony
• NPPV should generally be avoided for definitive management of patients or improperly set triggers, an increase in intrathoracic pressure,
with pneumonia or acute respiratory distress syndrome (ARDS) unless the decreased venous return to the heart and decreased cardiac output, an
patient is clearly improving or has a do-not-intubate status. Otherwise, increased ventilation/perfusion ratio, air trapping and intrinsic posi-
endotracheal intubation and mechanical ventilation are preferred. tive end-expiratory pressure, barotrauma, decreased renal blood flow
• For patients in need of ventilatory support, bilevel positive airway pressure and glomerular filtration rate with fluid retention, nosocomial infec-
(BiPAP) and continuous positive airway pressure (CPAP) have clinical equi- tions of the lungs and sinuses, respiratory alkalosis, and agitation and
poise. BiPAP should begin with an inspiratory pressure setting of 10 cm of increased respiratory distress.
H2O and expiratory pressure of 5 cm of H2O and be evaluated frequently for 3. What is the primary physiologic effect of applying positive end-
tolerance and need to titrate up or down. expiratory pressure (PEEP) during mechanical ventilation?
• Pressure-controlled ventilation (PC) delivers breaths at a predetermined a. Decrease cardiac output
pressure but with variable volume, while volume-controlled ventilation (VC) b. Decrease intrapulmonary shunting
delivers a predetermined inspiratory volume but variable pressures. c. Decrease ventilation/perfusion mismatch
• Assist-control ventilation (A/C) delivers a required number and volume of d. Increase functional residual capacity (FRC)
breaths per minute while synchronized intermittent-mandatory ventilation Answer: d. Although all of these effects can be attributed to the appli-
(SIMV) synchronizes mandatory breaths with spontaneous breaths. Either cation of PEEP, its primary physiologic effect is to increase FRC by
mode can be PC or VC. maintaining patency of injured or flooded alveoli that would other-
• Invasive mechanical ventilation requires dynamic, ongoing monitoring. wise collapse at the end of exhalation. Increasing the FRC may improve
After intubation, blood gas analysis should be performed to confirm appro- both oxygenation and lung compliance. PEEP increases Pao2 at a con-
priate ventilation and provide correlation with noninvasive monitoring of stant fraction of inspired oxygen (Fio2) by decreasing intrapulmonary
oxyhemoglobin saturation and end-tidal CO2. shunting and ventilation/perfusion mismatch. One of the potential
• Positive pressure can have adverse hemodynamic consequences. adverse effects to PEEP is decreased cardiac output.
• Plateau pressure should be maintained below 30 cm H2O. 4. Regarding a patient who develops acute distress on mechanical ven-
• Elevation in ventilation pressures suggests ventilator circuit obstruction, tilation, which of the following is the most accurate?
bronchospasm, mainstem intubation, tension pneumothorax or hemotho- a. Accidental extubation is a common cause of increased airway
rax, decreased chest wall compliance or increased chest wall rigidity. pressure.
• Inappropriately low ventilation pressures, particularly in conjunction with b. Anaphylaxis would cause immediate increases in both peak
hypoxia, indicates ventilator circuit leak or a faulty connection, endotra- inspiratory pressure (PIP) and plateau pressure (Pplat).
cheal tube cuff leak, accidental extubation, or esophageal intubation. c. All patients in distress should be immediately removed from the
• The Richmond Agitation Sedation Score (RASS) or a similar scoring system ventilator and bagged.
should be used to manage sedation and analgesia of the mechanically ventilated d. Compared to patients with restrictive lung disease, patients with
patient. Avoid unnecessary use of prolonged neuromuscular blockade. When obstructive lung disease and asthma are more likely to decom-
RASS is used, a target score of −2 to 0 avoids both over and under sedation. pensate without an appropriate expiratory time.
Answer: d. One of the most life-threatening complications of mechan-
1. Which of the following is the most important consideration in the deci- ical ventilation is loss of adequate cardiac output because of elevated
sion to initiate noninvasive positive-pressure ventilation (NPPV)? intrathoracic pressure from intrinsic PEEP (iPEEP). Intrinsic PEEP
a. Degree of acidosis can be precipitated by a respiratory rate that is too high, which does
b. Degree of respiratory distress not allow patients to fully exhale before the delivery of another breath.
c. Hemodynamic profile Patients with obstructive lung disease such as COPD or acute asthma
d. Level of consciousness are particularly sensitive to this phenomenon, also known as “breath
Answer: b. Although NPPV will be helpful for many patients in dis- stacking.” Although patients with restrictive lung disease may develop
tress, the need for emergent intubation is an absolute contraindication iPEEP with an inappropriate respiratory rate, this is much more likely
to NPPV. Both hemodynamic profile and level of consciousness are to occur in patients with obstructive conditions. If patients are not
important determinants in the decision to implement NPPV, but both hemodynamically unstable, they do not need to be removed from the
are relative (rather than absolute) contraindications to its use. Patients ventilator, and in hemodynamically unstable patients, pneumothorax
with chronic obstructive pulmonary disease (COPD), acute cardio- should only be presumptively treated if removing the patient from
genic pulmonary edema (ACPE), and asthma are more likely to show the ventilator does not improve the situation. Accidental extubation
benefit with NPPV than patients with pneumonia, but NPPV can be is a common cause of decreased airway pressures, and although ana-
initiated for any patient with respiratory distress. Patients with hypoxia phylaxis would cause increases in PIP, it would not typically cause
or hypercarbia may be acidemic, but the decision to initiate NPPV will increases in Pplat.
be predicated on mental status and work of breathing rather than on
degree of acidosis alone.
2. Which of the following is a potential adverse effect of intubation
and positive-pressure ventilation?
a. Decreased mean intrathoracic pressure
b. Decreased ventilation/perfusion ratio
c. Increased cardiac output
d. Increased work of breathing
3 Shock
KEY CONCEPTS
• S hock can occur with normal arterial blood pressure and not all patients
with arterial hypotension have shock.
• A base deficit more negative than −4 mEq/L or a serum lactate level greater
than 4.0 mmol/L warrants a presumptive diagnosis of shock.
• Urine output is a reliable index of vital organ perfusion in patients with sus-
pected shock. Normal urine output is 1.0 mL/kg/h. Output less than 0.5 mL/
kg/h indicates severe renal hypoperfusion in patients without preexisting
disease.
• A combination of a worsening base deficit, increasing lactate level, and low
urine output represents persistent or worsening shock.
• Hemorrhagic shock is preferentially treated with blood products, using
a balanced transfusion approach of packed red blood cells, fresh frozen
plasma, and platelets.
• Early initiation of balanced fluid resuscitation, vasopressor support, and
prompt antimicrobial therapy improve outcomes in patients with septic shock.
1. A 72-year-old male presents with crushing substernal chest pain Answer: a. Balanced crystalloids are the preferred choice of fluid
with associated diaphoresis starting 2 hours prior to arrival. Blood for patients with septic shock. Blood products may be indicated for
pressure is 72/50. ECG shows anterior ST elevation myocardial patients with a hemoglobin greater than 7 mg/dL. Synthetic colloids
infarction. Lungs demonstrate diffuse rales, and chest XR demon- such as HES cause renal failure and should be avoided. Hypertonic
strates bilateral infiltrates consistent with pulmonary edema. saline has not been proven to improve outcomes in sepsis. Normal
Bedside ultrasound demonstrates severely impaired left ven- saline can cause a hyperchloremic metabolic acidosis and acute kidney
tricular function. In addition to arranging for rapid reperfusion injury, particularly using large volumes.
with either cardiac catheterization or intravenous thrombolytics, 3. An 18-year-old unrestrained driver is transported to the emer-
which is the most appropriate initial treatment for his cardiogenic gency department (ED) after being thrown from his vehicle
shock? during a motor vehicle collision. He was intubated in the field
a. Bolus of 30 mL/kg crystalloid fluid. and received an intravascular bolus of 3 L of normal saline before
b. Initiate epinephrine at 0.1 mcg/kg/min. arrival to the ED. His initial Glasgow Coma Score (GCS) is 7, and
c. Initiate norepinephrine at 0.1 mcg/kg/min. his blood pressure on arrival is 80/50 mm Hg. Which of the fol-
d. Transfer patient for placement of an intraaortic balloon catheter. lowing would be the most appropriate to initiate immediately on
Answer: c. Norepinephrine (or inotropes such as dobutamine) are arrival to the ED?
first-line agents for cardiogenic shock. Crystalloid fluids should not a. Dobutamine.
be administrated to patients with cardiogenic shock and pulmonary b. Dopamine.
edema. Epinephrine carries an increased risk of tachydysrhythmias and c. Norepinephrine.
no better outcomes than norepinephrine or inotropes in cardiogenic d. Packed red blood cell (PRBC) transfusion.
shock. While this patient might benefit from an intraaortic balloon Answer: d. In patients with signs of hemorrhagic shock, immedi-
pump, treatment of his shock should not be delayed through transfer. ate PRBC transfusion should be initiated. This assists with volume
2. A 56-year-old female presents with fever, dyspnea, and productive expansion and oxygen delivery to the brain. Vasopressors and pos-
cough. Heart rate is 122, respiratory rate is 24, and blood pressure itive inotropes will be of little benefit before volume replacement,
is 82/46. Which of the following is the most appropriate choice for and hetastarch has no proven benefit for initial resuscitation in head
initial fluid resuscitation? injury patients.
a. Lactated Ringers.
b. Blood products.
c. Hydroxyethyl starch (HES).
d. Normal Saline.
4 Brain Resuscitation
KEY CONCEPTS c. EEG monitoring is not necessary or indicated for comatose
patients following cardiac arrest.
• N euronal injury is a dynamic process that continues for hours or days after d. Infusion of cold saline should be used to induce cooling in the
an ischemic insult to the brain. pre-hospital setting following cardiac arrest.
• Avoid hypotension and hypoperfusion by maintaining a mean arterial pres- Answer: b. The 2015 guidelines update confirms that all comatose
sure greater than 65 mm Hg and a cerebral perfusion pressure of 60 mm Hg. (lack of meaningful response to verbal commands) patients with ROSC
• Maintain normal oxygen levels or mild hyperoxemia, with a PaO2 of 80 to following cardiac arrest should receive targeted temperature manage-
120 mm Hg and oxyhemoglobin saturations in the high 90s. Avoid hypox- ment (TTM), regardless of location of arrest or rhythm type. Because
emia and significant hyperoxemia. the neurological examination is unreliable in the first few days fol-
• Intracranial pressure (ICP) elevation can further exacerbate ischemic brain lowing arrest, 72 hours is the earliest possible time that one may con-
injury. Initial management should include optimizing patient positioning sider using it to provide prognostic information to surrogate decision
while providing adequate analgesia and sedation. Management should makers. Non-convulsive seizures are common following cardiac arrest
then be escalated in a stepwise fashion to include hypertonic therapy, therefore EEG monitoring, if available, should be performed frequently
deeper sedation and ultimately barbiturates, hypothermia, and surgery as or continuously in comatose survivors of cardiac arrest. Finally, mul-
needed. tiple randomized controlled trials showed that cold intravenous fluids
• Avoid hyperventilation (target PaCO2 of 35 to 40 mm Hg) as it reduces cere- in the pre-hospital setting increased the rate of pulmonary edema and
bral blood flow. In the event of life-threatening cerebral herniation or sig- rearrest. We do not recommend this practice.
nificant ICP elevation, therapeutic hyperventilation is appropriate only as a 3. Select the best answer. Which of the following statements is false?
short-term intervention bridging to more definitive therapy (i.e., craniotomy). a. Early magnetic resonance imaging (MRI) and serum biomarkers
• Promptly treat seizures with intravenous (IV) lorazepam. Prophylactic have a clearly established role in determining the prognosis of
administration of antiepileptic drugs is not recommended. Initiate continu- patients within 48 hours after cardiac arrest.
ous electroencephalogram (EEG) monitoring if ongoing seizures are a con- b. Recent data suggest that rate of survival to hospital discharge
cern. following cardiac arrest is 12%, with 75% of survivors achieving
• Fever is an important mediator of secondary brain injury. Treat tempera- a favorable neurological outcome.
tures greater than 38°C with acetaminophen. c. In cardiac arrest survivors presenting to the emergency depart-
• Unresponsive survivors of out-of-hospital cardiac arrest should have rapid ment , the initial neurological examination is not a reliable pre-
initiation of targeted temperature management (TTM) in the emergency dictor of long-term neurological prognosis.
department and be maintained at a constant target of 33°C–36°C in an ICU d. The vast majority of 1-year survivors of cardiac arrest are able to
setting for 24 h after resuscitation. live independently without disability.
• Because withdrawal of life-sustaining treatment due to perceived poor Answer: Statement a is false. The role of early imaging, neurophys-
neurological prognosis is the most common cause of death in cardiac iologic testing, and serum biomarkers in predicting outcome has not
arrest survivors, it is important to provide accurate and evidence-based yet been clearly established at any time point, especially not within 48
prognostic information to families. The neurological examination is hours.
generally an unreliable predictor of outcome until at least 72 h from
4. Which of the following statements about neuronal damage follow-
normothermia.
ing ischemic injury is correct?
a. A patient with return of spontaneous circulation after 15 min-
1. To maximize cerebral blood flow, a patient with a normal intracra- utes of cardiac arrest has likely already suffered substantial neu-
nial pressure who is undergoing resuscitation should be ventilated ronal cell death.
to maintain a partial pressure of carbon dioxide (Paco2) within b. There is a single biological pathway by which ischemia triggers
what range? neuronal injury and eventually results in cell death.
a. 20 to 25 mm Hg. c. Cell death is delayed following cardiac arrest, suggesting that
b. 25 to 30 mm Hg. therapeutic interventions may mitigate long-term neuronal
c. 30 to 35 mm Hg. injury.
d. 35 to 40 mm Hg. d. It is not important to avoid secondary neuronal injury following
e. 40 to 45 mm Hg. cardiac arrest.
Answer: d. Carbon dioxide is a potent vasoactive agent, and lowering Answer: c. Although a cascade of cellular pathways will have been trig-
of the Paco2 by hyperventilation results in rapid reduction of cerebral gered, the resulting neuronal cell death is usually delayed by hours or
blood flow (CBF). Since reductions in CBF reduce total cerebral blood days. This suggests that therapeutic interventions, such as targeted tem-
volume, hyperventilation may transiently abort brainstem herniation perature management, can reduce cell death and improve outcomes.
in the presence of critically elevated intracranial pressure (ICP) until Careful avoidance of additional secondary insults is also important to
osmotherapy or ventriculostomy can be initiated. When ICP is not maximize the potential for favorable neurological recovery.
elevated, however, the vasoconstriction and increased cerebrovascular 5. Select the best answer. Which of the following is associated with
resistance (CVR) caused by hyperventilation can cause potentially worse neurologic outcomes in comatose survivors of cardiac arrest?
dangerous reductions in CBF. In general, ventilation to maintain a a. All of these.
Paco2 between 35 and 40 mm Hg is safe and appropriate. b. Hyperthermia.
2. Which of the following is a class 1 recommendation from the 2015 c. Hypotension.
American Heart Association Guidelines for Cardiopulmonary d. Hypoxia.
Resuscitation and Emergency Cardiovascular Care? e. Only hypotension and hyperthermia.
a. Forty-eight hours from arrest is the earliest time the clinical Answer: Statement a is the best answer. Hypotension, hypoxia, and
examination may be used for neuroprognostication. hyperthermia (as well as hyperglycemia and seizures) are all associ-
b. All comatose patients with return of spontaneous circulation ated with worse neurologic outcomes in comatose survivors of cardiac
(ROSC) after in- and out-of-hospital cardiac arrest, including arrest.
both shockable and non-shockable rhythms, should receive tar-
geted temperature management.
Adult Resuscitation 5
KEY CONCEPTS 3. Which chest compression/ventilation ratio is recommended during
adult resuscitation efforts performed by health care professionals
• C ardiopulmonary resuscitation (CPR) quality is critical to successful resus-
before placement of an advanced airway?
citation from cardiac arrest. Important benchmarks of quality CPR include
a. 10:1
compression rate 100–120 compressions/min, compression depth 5–6 cm,
b. 20:1
compression fraction at least 80%, full chest recoil between compressions,
c. 20:2
and a ventilation rate of 10 breaths/min.
d. 30:2
• Restoration of adequate cardiac function is the defining factor of return of
spontaneous circulation (ROSC). Restoration of good neurologic function is Answer: d. A 30:2 compression/ventilation ratio is currently recom-
the defining factor of a successful resuscitation. mended for health care professionals in all adult resuscitation scenarios.
• Resuscitation of a cardiac arrest victim does not end with ROSC. Rapid Although recent evidence has suggested that chest compression-only
diagnosis and proper management of the pathologic condition(s) that pre- CPR is effective when performed by bystanders in the out-of-hospital
cipitated and resulted from the arrest, as well as goal-directed post–car- setting, there is inadequate evidence to recommend this as an alter-
diac arrest care, can improve outcomes. native strategy for health care professionals, except when inadequate
• Immediate percutaneous coronary intervention is indicated in patients with personnel are present to provide compressions, ventilation, and other
ST segment elevation myocardial infarction following ROSC, independent resuscitative activities.
of neurological status. 4. Which of the following statements regarding hypothermic targeted
• Hypothermic targeted temperature management (32°C–36°C [89.6°F–96.8°F] temperature management (HTTM) in comatose survivors of car-
for 24 h) has been shown to improve survival and functional outcome of coma- diac arrest is true?
tose cardiac arrest survivors. a. Gradual rewarming should occur over 4 hours.
b. Pregnancy is an absolute contraindication.
1. Which of the following statements regarding the epidemiology of c. Prolonged pharmacologically induced paralysis without seda-
out-of-hospital cardiac arrest is true? tion is often required to control shivering.
a. Most patients have an automated external defibrillator applied d. Target core body temperature should be 32° to 36°C.
prior to emergency medical services (EMS) arrival. Answer: d. Induction of prolonged HTTM in comatose survivors
b. Most patients receive bystander CPR. of cardiac arrest has been shown to improve survival and functional
c. Most surviving to hospital discharge will not have major per- outcome. A target temperature in the range of 32°C to 36°C (89.6°F to
sistent neurologic deficits. 96.8°F) should be selected and maintained. The time to achieve this
d. Only 2% of EMS-treated out-of-hospital cardiac arrests survive temperature has not been clearly defined, and it has been suggested
to hospital discharge. there is a broad therapeutic window. In the studies showing a benefit,
Answer: c. It is estimated that 180,000 patients are treated for out-of-hospital maintenance of hypothermia occurred for 12 to 24 hours, followed
cardiac arrest each year in the United States. The number of patients receiv- by gradual rewarming over 12 to 24 hours. There are no absolute con-
ing bystander cardiopulmonary resuscitation (CPR) remains low, averag- traindications to induced hypothermia after arrest. Shivering, which
ing 40%. The proportion of emergency medical services (EMS)–treated inhibits cooling, can be prevented with sedation and pharmacologic
cardiac arrest patients with an initial shockable rhythm has declined over paralysis. However, prolonged paralysis should be avoided because
time to 18% in recent US studies. Automated external defibrillators are of the risk of unrecognized seizure activity in post–cardiac arrest
applied in a minority of cases prior to EMS arrival. Recent epidemiologic patients.
data from cardiac arrest registries indicate the survival rate to hospital dis- 5. For end-tidal pressure of carbon dioxide (PETco2) to be a reliable
charge for EMS-treated, out-of-hospital cardiac arrest is about 10%. Of indicator of cardiac output during cardiac arrest, which of the fol-
patients surviving to hospital discharge, independent of neurologic status lowing must be present?
on presentation, 79% have good neurologic function. a. Mechanical chest compressions must be performed.
2. A 75-year-old man presents with return of spontaneous circulation b. The patient must be in asystole.
(ROSC) after 2 minutes of ventricular fibrillation and successful c. The patient must be normothermic
defibrillation by EMS. The patient is unresponsive to verbal and pain- d. The patient must have relatively constant minute ventilation.
ful stimuli. Vital signs on arrival are pulse, 120 beats/min; blood pres- Answer: d. Although PETco2 will change in direct relationship to car-
sure, 130/70 mm Hg; respiratory rate, 10 breaths/min; temperature, diac output, alterations in minute ventilation will concentrate or dilute
36°C (96.8°F); and oxygen saturation, 94%. The patient has intrave- the fixed amount of expired CO2, influencing the PETco2 measured
nous access. The next most appropriate examination or procedure is: independently of cardiac output. Therefore, for PETco2 to be a reliable
a. Anteroposterior (AP) chest radiograph indicator of cardiac output, minute ventilation must be held relatively
b. Arterial blood gas (ABG) constant. The relationship of PETco2 and cardiac output during CPR
c. Comprehensive neurologic examination is not dependent on rhythm, mechanisms of chest compressions, or
d. Electrocardiography temperature. High-dose vasopressor therapy can cause a decrease in
Answer: d. Acute coronary syndrome is a common cause of out-of- cardiac output during CPR, despite increased myocardial blood flow,
hospital cardiac arrest. Electrocardiography should be performed which results in a decreased PETco2.
as soon as possible after ROSC to evaluate for ST segment elevation.
Because it is impossible to determine survival or neurologic status in
the immediate post-arrest period, ST segment elevation myocardial
infarction (STEMI) should be treated aggressively with percutaneous
coronary intervention (PCI) independently of coma or other labora-
tory values such as those provided on ABG analysis. Oxygen satura-
tions above 94% are adequate for tissue perfusion, and hyperoxia may
be harmful. AP chest radiographs may be important to evaluate venti-
latory status if the patient is unstable.
6 Pain Management
KEY CONCEPTS 1. Which of the following statements is true regarding pain trans-
mission?
• A cute pain is an urgent condition for the patient. Pain should a. Cardiac pain is transmitted via the sympathetic system.
be rapidly assessed, treated, and frequently reassessed in b. Central post-stroke neuropathic pain is associated with
tandem with diagnostic evaluations. parietal infarcts.
• Therapy for acute pain is different than for chronic pain. c. Descending modulation of pain is mediated primarily
Chronic pain treatment should be undertaken in consultation through γ-aminobutyric acid (GABA).
with the clinician(s) responsible for the patient’s long-term d. Peripheral neurotransmitters include prostaglandins,
management. In general, opioid analgesic agents should not be histamines, and substance P.
administered in the ED or prescribed for outpatient therapy Answer: A. As a general rule, all visceral pain is carried via sym-
for chronic pain patients unless the plan is agreed to by the pathetic afferents to ganglia and then to the spinal cord. Prostaglan-
responsible outpatient clinician. dins, substance P, and histamine sensitize peripheral afferents but are
• Titrated IV opioid analgesics are the primary therapeutic not neurotransmitters. The dorsal column tracts can down-modulate
approach for the treatment of moderate and severe acute pain. ascending pain signals. Central post-stroke pain is clinically seen most
When intravenous access is not indicated or attainable, SC often after thalamic strokes. Descending tracts that modulate pain pro-
administration is preferable to the IM route. cessing at the dorsal horn use norepinephrine and serotonin, with the
• Opioid therapy should be reserved for pain that has not been effect of the former being most important regarding analgesia.
or cannot adequately be treated with nonopioid therapies. 2. Which of the following analgesics is matched with the correct
• Oral oxycodone, with an onset of action similar to that of feature?
IM or SC opioids, can be used for moderate pain when the a. Fentanyl—prolonged QT interval on electrocardiography
IV route is not otherwise needed. Oxycodone and other oral b. Hydromorphone—active metabolites
opioids should be administered and prescribed as a single-drug c. Meperidine—muscle rigidity
preparation, not as part of a combination with acetaminophen. d. Oxycodone—serotonin syndrome
• Ambulatory treatment with opioids should be confined to Answer: D. Oxycodone has been associated with serotonin syn-
the period of acute pain. Most opioid prescriptions from the drome when coadministered with selective serotonin reuptake inhib-
ED for acute injury should be for 3 to 5 days, after which the itor (SSRI) medications. This is due to an active metabolite it shares
patient is transitioned to nonopioid analgesia or reevaluated by with morphine, hydromorphone, hydrocodone, and codeine. The fol-
an outpatient clinician. lowing are the other correct associations:
• Acetaminophen and NSAIDs should be added to pain therapy Meperidine—anticholinergic toxicity, active metabolites
when not contraindicated. Their analgesic effects are additive Fentanyl—muscle rigidity (chest wall)
to those of opioids and each other. Hydromorphone—inactive metabolite
• There is no evidence to support the concept that diagnosis 3. A 32-year-old male patient undergoing treatment for an ankle
based on physical examination findings will be impaired by sprain returns to the emergency department (ED) because of
administering opioid pain medications to achieve reasonable inadequate pain relief from the medicines he was prescribed. He
patient comfort. is currently taking oxycodone, 10 mg PO every 4 hours, and ibu-
• There is no evidence that morphine causes more smooth profen, 400 mg every 4 hours. What is the next most appropriate
muscle spasm than other opioids. Morphine is safe and medicine to add to his pain treatment regimen?
appropriate for patients with acute biliary or renal colic. a. Add acetaminophen, 650 mg q4h.
• Patients who are known to be diverting or abusing opioids b. Add tramadol, 50 mg PO q4h.
should not be prescribed opioids for use as outpatients. c. Increase ibuprofen to 800 mg.
Patients with chronic pain syndromes, or those with chronic d. Increase oxycodone to 15 mg.
conditions that may cause acute pain (e.g., dental caries), Answer: A. Acetaminophen provides additive analgesia to non-
should be offered alternative pain management options, and steroidal antiinflammatory drugs (NSAIDs) and opioids, with few
opioids should generally be avoided. adverse effects at low doses, and it should be incorporated in acute pain
• Topical and local anesthetics can treat pain associated with treatment when not contraindicated. The pain-relieving effects of ibu-
most ED procedures and should be considered for use in profen have not been shown to be greater when 800 mg is used versus
isolated painful conditions. 400 mg, so increasing the dose of ibuprofen is unlikely to improve pain
• Low tissue pH (5 or 6) in infected tissue impairs the relief and will increase the risk of adverse effects of the NSAIDs. An
effectiveness of local anesthesia. increased dose of oxycodone would result in improved pain relief but
increases the risk of adverse effects; it should be tried after other non-
opioid treatments have failed. In this case, acetaminophen should be
attempted first before increasing the oxycodone dose. Tramadol would
not be indicated before acetaminophen because of its high rate of diz-
ziness and nausea.
7 Procedural Sedation and Analgesia
KEY CONCEPTS 3. A 23-year-old woman presents with the sudden onset of weakness
• W eakness is a common complaint among emergency department (ED) of her face, arm, and leg 2 hours ago. On examination, she has
patients, with a preponderance in elders and those with chronic disease, weakness to her upper and lower face on the left. She cannot abduct
and therefore may require a broad approach to investigating underlying her left eye. She has 3/5 strength to her upper and lower extremities
causes. on her right side. She has a right pronator drift and an upgoing toe
• Patients may use the term weakness to reflect a variety of vague symptoms on the right. Sensation is decreased in her right upper and lower
including decreased motor strength, fatigue, poor energy, dyspnea, or even extremities as well. There is no aphasia, neglect, or visual field defi-
depression. cit. What is the most likely diagnosis?
• Global weakness is typically caused by a systemic condition while a focal a. Midbrain stroke because of cardioembolic stroke
neurologic deficit or pattern can often be traced to a specific lesion within b. Middle cerebral artery (MCA) distribution stroke
the central or peripheral nervous systems. c. Multiple sclerosis
• Although not always detectable in the acute period, lesions to the upper d. Myasthenia gravis
motor neurons (UMNs) tend to produce signs that include spasticity to e. Pons stroke because of vertebral dissection
extension in the upper extremities, spasticity to flexion in the lower extrem- Answer: e. This patient has acute onset of crossed face and extrem-
ities, hyperreflexia, pronator drift, and Hoffmann and Babinski signs. ity weakness, with upper motor neuron (UMN) signs in the extrem-
• Lesions to lower motor neurons (LMNs) typically result in flaccidity, ities. Both upper and lower extremities are affected, which makes a
decreased reflexes, fasciculations, or muscle cramps. corticospinal tract (CST) lesion more likely than a cerebral cortex
lesion. Her left-sided facial weakness is representative of a periph-
1. A 65-year-old man with a history of atrial fibrillation, on warfarin, eral cranial nerve (CN) VII that is actually due to infarction of the
with a supratherapeutic international normalized ratio (INR) of 4, CN VII nucleus in the brainstem. The CN VI deficit on that side is
presents with sudden onset right leg weakness and back pain. On due to proximity of this nucleus as well. The CST runs just anterior
examination, he is tachycardic to 108 beats/min and has 3/5 weak- to these nuclei within the brainstem. These CN nuclei lie in the
ness to the right hip flexors and extensors, knee flexors and exten- pons.
sors, as well as ankle dorsiflexion and great toe extension. However, 4. A 21-year-old man awoke this morning with weakness to his right
ankle plantar flexion is preserved. His knee reflexes are absent, but hand and right foot. He admits to drinking heavily the night before
Achilles reflexes are normal. He has a normal Babinski reflex and and falling asleep on the floor. On examination, he appears well,
no spasticity. He has sensory deficits throughout the anterior and with weakness to right wrist extension and thumb extension, as well
posterior parts of his proximal leg as well as the anterior lower leg as sensory deficits over the dorsal surface of his hand and first and
and dorsum of the foot. His posterior lower leg and plantar surface, third digits. He also has weakness to ankle dorsiflexion and great
however, have normal sensation. Rectal tone is normal, and there is toe extension on the right. He has sensory deficits over the anterior
no urinary retention. What is the most likely diagnosis? lower leg and the dorsum of his foot. Biceps and ankle reflexes are
a. Anterior cord syndrome from epidural hematoma intact, he has no pronator drift, and his toes are downgoing. What
b. Cauda equina syndrome is the most likely diagnosis?
c. Guillain-Barré syndrome a. Brainstem stroke
d. Hemorrhagic anterior cerebral artery (ACA) distribution stroke b. Brown-Séquard syndrome
e. Retroperitoneal hematoma with lumbar plexopathy c. Compressive neuropathy
Answer: e. This patient has a spontaneous retroperitoneal hematoma d. MCA distribution stroke
compressing the lumbar nerve plexus. e. Polyradiculopathy secondary to disk disease
2. A 45-year-old man has had gradual onset of progressive weakness to Answer: c. He has radial nerve and peroneal nerve palsies because
his face and trouble swallowing for 2 days. On examination, he has of compression while lying passed out on the floor for an unspecified
bilateral ptosis with dilated, poorly reactive pupils, bilateral upper time.
and lower facial muscle weakness, poor soft palate rise, and slurred 5. A 70-year-old man has had trouble swallowing and progressive
speech. His oral mucosa is dry. Arms and legs have 5/5 strength. He weakness of his hands over the past 2 months. On examination, his
has no sensory deficits. He has a palpable distended bladder. His speech is slurred, his voice is nasal, and he has fasciculations to his
symptoms have not abated since onset, and they are getting worse. face, tongue, and over his pectoralis muscles and deltoids bilater-
What is the most likely diagnosis? ally. He has 4/5 strength to shoulders, biceps, triceps, and hand grip
a. Brainstem stroke bilaterally. Stiffness to extension is present at both elbows. He has
b. Botulism bilateral pronator drift and 3+ biceps reflexes. He tends to smile
c. Muscular dystrophy inappropriately. He has no sensory symptoms. What is the most
d. Myasthenia gravis likely diagnosis?
e. Organophosphate poisoning a. Amyotrophic lateral sclerosis (ALS)
Answer: b. This patient has acute onset of progressive neuromuscu- b. Brainstem stroke
lar junction weakness. He has autonomic findings of abnormal pupil c. Chronic demyelinating polyneuropathy
response to light, dry oral mucosa from decreased salivary produc- d. Parkinsonism
tion, and a distended bladder. These imply that his problem is with the e. Polymyositis
release of acetylcholine (ACh) rather than the nicotinic receptor. The Answer: a. This patient has lower motor neuron (LMN) signs (fascicu-
latter would not have autonomic findings. The most appropriate cause lations) and UMN signs (pronator drift and increased reflexes) in sim-
of acute onset, progressive impairment in the release of ACh, as listed ilar distribution. The combination of upper and lower motor neuron
in the choices, is botulism. involvement makes ALS the leading diagnosis among those listed. His
dysphagia and inappropriate smiling are due to a release of the medulla
from upper motor neuron regulation.
10 Cyanosis
1. Which of the following statements regarding the epidemiology of Answer: b. The principal serious causes of syncope are cardiac dys-
syncope is true? rhythmias. TIA is less frequently encountered but equally serious.
a. Of athletes who die during exercise, the vast majority have had a Stroke generally does not cause isolated syncope. Toxic-metabolic
prior episode of syncope. abnormalities may induce syncope through alterations in blood
b. People younger than 65 years account for 50% of all patients pressure or cardiac rhythm. Structural cardiac lesions, such as atrial
admitted for syncope from the emergency department. myxoma, and sudden interruption of right ventricular outflow by pul-
c. Syncope in young adults is typically secondary to significant monary embolism, can also cause sudden loss of consciousness but are
pathology. less common.
d. Syncope is responsible for approximately 5% to 6% of all emer- 3. Which of the following findings most strongly suggest that a patient
gency department (ED) visits. presenting with syncope can be safely discharged from the emer-
e. There is a slight female predominance among ED patients pre- gency department (ED)?
senting with syncope. a. Age under 85 years and a normal chemistry panel
Answer: e. Syncope accounts for approximately 1% to 2% of ED visits b. A normal chest x-ray and CT scan of the head
in the United States. There is a slight female predominance among ED c. Normal heart rate and temperature in the ED
patients with syncope. Approximately 32% of these patients are hos- d. Normal ECG findings and no past medical history
pitalized, and people aged 65 years or older account for the majority e. Absence of abdominal pain and absence of headache
of such hospitalizations. Benign causes of syncope predominate in Answer: d. A normal electrocardiogram in a patient without other
adolescents and young adults. Approximately 30% of athletes who die significant risk factors (e.g., advanced age, preexisting heart disease)
during exercise have had a prior episode of syncope as a sentinel event. may be considered for outpatient disposition. The chemistry panel,
2. Syncope resulting from serious pathology is usually caused by chest x-rays, and CT head scan are commonly ordered but are of low
which of the following? diagnostic and prognostic utility. Abdominal pain, fever, and headache
a. Transient ischemic attack (TIA) are important associated signs and symptoms to assess for, but their
b. Cardiac dysrhythmias absence has little prognostic utility.
c. Pulmonary embolism
d. Atrial myxoma
e. Toxic-metabolic abnormalities
Depressed Consciousness and Coma 12
Answer: b. Using the GCS, this adult patient would receive 2 points for
KEY CONCEPTS
eye opening to pain, 2 points for a verbal response consisting of incom-
• C oma is a state of depressed consciousness in which a patient is not aware, prehensible sounds, and 4 points for withdrawing from painful stimuli.
is not awake, and does not respond to vigorous stimulation. Consciousness 3. What Glasgow Coma Scale (GCS) score would be given to a pediat-
consists of arousal (subcortical) and awareness (cortical). ric patient who opens the eyes and flexes the extremities to painful
• Damage to the dorsal brainstem, thalamus, axonal projections to the cor- stimuli and who is irritable and continually cries?
tex, or extensive injury to bilateral cortices may result in depressed con- a. 6
sciousness or coma. b. 7
• Toxicologic, metabolic, infectious, and other disorders causing diffuse brain c. 8
injury cause 65% of coma cases; of these, toxins are the most common. d. 9
Structural brain diseases account for most of the remaining cases. Answer: d. Using the GCS, this pediatric patient would receive 2 points
• A patient with depressed consciousness is unlikely to provide a reliable for eye opening to pain, 4 points for persistently being irritable, and
history. Historical information should be elicited from other available 3 points for flexion in response to painful stimuli. The pediatric GCS
sources, such as emergency medical services personnel, family members, takes into account that younger children might not yet be verbal.
the patient’s belongings, or medical records. 4. Awareness of one’s self and surroundings defines which of the fol-
• An abrupt onset of coma suggests a stroke, seizure, or cardiac event. lowing?
• The neurologic examination should include an evaluation of level of con- a. Cognition
sciousness, cranial nerves, brainstem reflexes, and motor responses. b. Consciousness
• Pinpoint pupils may represent a pontine infarct or intoxication from opioids, c. Judgment
clonidine, or cholinergic substances. d. Orientation
• Nonconvulsive status epilepticus should be suspected in cases of coma of Answer: b. Consciousness is defined as the awareness of one’s self and
undetermined cause and is diagnosed by electroencephalography. surroundings; it is made up of both arousal and awareness. Cognition
• Hypoglycemia and hypoxia are two easily identified and reversible causes involves knowing and understanding, especially through the percep-
of coma. tion of what is experienced by the senses; judgment is the ability to
• An empiric trial of naloxone will lead to rapid reversal of opioid toxicity and process input data to generate more meaningful information; and ori-
other select medication overdoses. entation is the ability to determine the relative position of oneself in
• Targeted temperature management is recommended in comatose adult relation to one’s person, place, time, and situation.
patients with return of spontaneous circulation after cardiac arrest from 5. In the evaluation of a comatose patient, which of the following
either a shockable or nonshockable rhythm, with a goal temperature range responses most clearly indicates brainstem dysfunction?
of 32°C to 36°C. In children who are comatose post arrest, it is reasonable a. A drop of saline on the cornea of one eye causes both eyes to
to target a temperature range of 36°C to 37.5°C. Hyperthermia should be blink
strictly avoided for all comatose postarrest patients. b. Infusion of cold water into the right ear causes rightward devia-
• Most patients with coma will require intensive care monitoring. If the tion of the eyes
cause of coma is not treatable in the initial facility and the patient needs a c. Stimulation of the posterior pharynx has no effect on the soft
higher level of specialty care (e.g., structural lesion requiring neurosurgery), palate of pharyngeal muscles
then a coordinated transfer should be facilitated. d. When the head is rotated from side to side in the horizontal
plane, the eyes remain in midposition and move in unison with
1. In infants, what is the most common cause of a depressed level of the head.
consciousness? Answer: d. Brainstem function is evaluated through the various brain-
a. Accidental toxic ingestion stem reflexes. The oculocephalic reflex is tested by rotating the head
b. Inborn error of metabolism from side to side in the horizontal plane. In patients with normal brain-
c. Infection stem function, the eyes should move in a direction opposite of head
d. Physical abuse movement and appear to maintain visual fixation on a single point in
Answer: c. The most common causes of depressed consciousness vary space. The corneal reflex is tested by touching the edge of the cornea
with patient age. In infants, infectious causes are most common; how- with a wisp of cotton or drop of water and observing for blinking of
ever, trauma secondary to physical abuse and metabolic derangements the eyes. The oculovestibular reflex is tested by infusing cold water into
from inborn errors of metabolism can also be seen. Accidental toxic the external auditory canal. In patients with an intact brainstem, the
ingestions are often seen in younger children but are very uncommon stimulus causes sustained conjugate deviation of the eyes toward the
in infants. Young adults and adolescents are more likely to present with ear being irrigated. The gag reflex is tested by stimulating the posterior
depressed consciousness after recreational drug use or trauma. Older pharynx and observing for brisk elevation of the soft palate and con-
adults are particularly vulnerable to infectious causes, medication traction of the pharyngeal muscles. Because the gag reflex is symmetri-
effects, and alterations in their living environments. cally reduced or absent in a subset of normal individuals, the utility of
2. What Glasgow Coma Scale (GCS) score would be given to an adult this brainstem reflex is limited.
patient who opens the eyes to painful stimuli, speaks in an incom-
prehensible manner, and withdraws from pain?
a. 7
b. 8
c. 9
d. 10
13 Confusion
KEY CONCEPTS
• C onfusion and delirium are symptoms, not a diagnosis.
• Focal cortical dysfunction, such as from tumor or stroke, typically does not
cause confusion.
• Any underlying clinical process that disrupts optimal central nervous sys-
tem (CNS) functioning can result in confusion.
• Emergent causes of confusion that need immediate detection and treat-
ment include hypoglycemia, hypoxemia, hypotension, sepsis, and toxic
ingestions.
• Assessment of attention is fundamental for the assessment of patients
with confusion as disturbances in attention are consistent with delirium
versus psychiatric illness or dementia.
• Recommended tools for identifying patients with delirium in the emer-
gency department are the Delirium Triage Screen (DTS) and brief Confusion
Assessment Method (bCAM), if indicated.
• Delirium often goes unrecognized unless a structured assessment tool is
used.
• Sedatives, including antipsychotics, are useful for managing undifferenti-
ated agitation while the diagnostic evaluation is in progress.
1. A 70-year-old man with a chief complaint of confusion is brought 3. A 30-year-old patient is brought to the emergency department for
to the emergency department by his family. Which of the following evaluation of odd behavior. Which of the following characteristics
initial assessments should be included? might suggest a psychiatric cause for the behavior?
a. Blood pressure Auditory hallucinations
b. Pulse oximetry b. Disorientation
c. Temperature c. Fever
d. All of these d. Visual hallucinations
Answer: a. Auditory hallucinations are common in psychiatric illness.
Answer: d. Confusion may result from shock states, hypoglycemia, a.
If hallucinations are present in organic causes of delirium, they are usu-
and hypoxia. Evaluation for these conditions is a priority. Confusion ally visual, tactile, or olfactory. Orientation is generally preserved with
is a symptom rather than a medical condition, and reversible causes primary psychiatric disorders unless psychosis or severe impairment
should be investigated. is present.
2. A variety of brief screening tools may aid in the detection of confu- 4. Postictal confusion is common in patients with seizures, but if
sion. All of these tests focus on which cardinal finding of confusion? improvement in consciousness does not occur within 20 to 30
a. Attention impairment minutes after seizure cessation, which of the following conditions
b. Decreased level of arousal should be considered?
c. Disorientation a. Electrolyte abnormalities
d. Long-term memory impairment b. Hypoglycemia
c. Nonconvulsive or subtle status epilepticus
Answer: a. Inattention is the critical finding for diagnosis of confusion d. All of these
and delirium. All of the screening tools (brief Confusion Assessment Answer: d. For a patient with a generalized convulsive seizure, ter-
Method [bCAM], Mini-Mental State Exam [MMSE], Quick Confusion mination of the seizure activity is usually followed by improvement
Scale [QCS]) evaluate this abnormality, although other components of of mental status within a short period of time. For the patient with
consciousness are tested in some models. persistently altered consciousness or prolonged confusion, consider
causes of provoked seizures with prolonged altered mental status or
persistence of subtle seizures.
14 Seizures
KEY CONCEPTS
• Lifetime seizure incidence reaches up to 10% in the US population and cavity are important to prevent aspiration; the use of intraoral devices may
depends upon multiple factors including age, history of epilepsy, structural lead to trauma without additional benefit.
brain or neurodegenerative disease, genetic predisposition, acuity and sever- • Pharmacologic interventions center on the timely and dose-
appropriate
ity of metabolic derangements. administration of parenteral benzodiazepines, which are first-line therapy
• Seizures are the result of an imbalance in the excitation and inhibitory neuro- (i.e., lorazepam intravenous [IV] up to 0.1 mg/kg in divided doses, or midaz-
nal synapses in the cerebral cortex or limbic system. olam IV/intramuscular [IM]/intranasal [IN] 5 to 10 mg).
• Epilepsy is a condition associated with a lower seizure threshold and an • In pregnant patients, eclampsia should be considered the underlying etiology
inherently higher risk of recurrent seizures in a lifetime. of de novo seizures after 20 weeks of gestational age and up to 8 weeks post-
• Acute symptomatic seizures occur at, or in close temporal relationship with, a partum. Clinical evaluation should focus on evaluating for associated symp-
distinct provoked event, and may occur with any toxic or metabolic, structural, toms (e.g., headache, visual abnormalities and confusion), as well as focal
ischemic, inflammatory, or infectious insult. neurologic deficits. Neuroimaging should be considered if neurologic deficits
• The initial evaluation of seizure includes the identification of seizure charac- persist, prolonged loss of consciousness or encephalopathy. IV magnesium
teristics, duration, and etiology of possible inciting events. Recognizing the 4 to 6 g is first-line therapy in the pregnant patient experiencing new-onset
occurrence of similar events in the past is key in determining triggers and seizures.
distinguishing the management priorities between patients with epilepsy or • During the postictal state, key priorities in management include ensuring air-
with acute symptomatic seizures. way patency with optimal positioning in those with alteration of conscious-
• The majority of seizures will cease spontaneously within 5 minutes; however, ness, evaluation for nonconvulsive seizures and acute brain injuries or inciting
when physiologic seizure termination pathways are overwhelmed resulting in factors that warrant further definitive treatment.
status epilepticus, characterized by the persistent and exponential refractory • Nonconvulsive status epilepticus can only be diagnosed with electroenceph-
response to abortive seizure therapies. The magnitude of treatment refracto- alography (EEG) monitoring and should be considered in any patient with
riness is time dependent; therefore a prompt stepwise escalation of therapies prolonged, unexplained altered mental status. Patients at higher risk include
aiming at seizure termination is warranted. those with poorly controlled epilepsy, acute brain injury, sepsis, and intoxica-
• Prompt identification and, whenever possible, reversal of seizure triggers are tion.
priorities. In particular, assessing for and correcting hypoglycemia should • A patient with new-onset seizures with a normal neurologic examination who
occur as early as possible. returns to baseline mental status and does not have comorbid disease does
• Initial nonpharmacologic interventions include ensuring physical safety of not require diagnostic testing beyond, serum glucose and sodium levels, a
patients with seizures to prevent traumatic injuries and aspiration. During pregnancy test (in women), and a noncontrast head computed tomography
convulsions, focusing on lateral decubitus positioning and suctioning of oral (CT); a toxicologic screen is considered in select patients.
1. A 63-year-old woman with end-stage renal disease on hemodialy- 2. You suspected posterior reversible encephalopathy syndrome
sis, diabetes, and hypertension is brought to the emergency depart- (PRES) and order a STAT electroencephalography (EEG) while
ment after losing consciousness during hemodialysis. The episode lowering the blood pressure in a stepwise approach. While the
occurred towards the end of the hemodialysis section approximately patient is waiting to be admitted to the neurology intensive care
45 minutes ago and was associated with foaming at the mouth, cya- unit and connected to EEG, the bedside registered nurse reports
nosis, and bilateral tonic-clonic movements of upper extremities new left gaze deviation, fine eyelid twitching, and more prominent
lasting nearly 3 minutes. On examination, T 36.6°C, HR 89 bpm, nystagmus that was refractory to 1 mg of lorazepam. Your index
SBP 195 mm Hg, RR 11, pulse oximetry 94%, Glasgow Coma Score of suspicion for nonconvulsive status epilepticus is high and you
(GCS) is 10 with subtle nystagmus, and no meningismus. Com- decide to treat empirically. The patient weighs 52 kg.
puted tomography of the head (CTH) demonstrated hypodensities What is the first-line therapy for status epilepticus treatment?
in bilateral parieto-occipital regions. a. Fosphenytoin 1000 mg intravenous (IV)
What is the leading diagnosis and required work up? b. Lorazepam 5 mg IV
a. Nonconvulsive status epilepticus from hypertensive emergency; c. Levetiracetam 3000 mg IV
electroencephalography (EEG) d. Lacosamide 300 mg IV
b. Cortical blindness from bilateral posterior cerebral artery stroke; Answer: b. The first-line therapy for seizures and status epilepticus is
computed tomography angiogram benzodiazepines. In the hospital setting, lorazepam IV is most com-
c. Coma from meningoencephalitis; lumbar puncture monly used. Although all the other choices are reasonable second-line
d. Coma from uremia; blood urea nitrogen (BUN) antiseizure drugs, the patient has received only 1 mg of lorazepam at
Answer: a. The description of the episode of loss of consciousness with this point, and a total of 5mg of lorazepam (0.1 mg/kg) is considered
associated cyanosis, and tonic-clonic convulsions is highly suspicious for appropriate first-line therapy.
a generalized tonic-clonic seizure. The patient has not returned to baseline 3. According to the best evidence available from randomized con-
mentation as demonstrated by GCS of 10 and is displaying subtle signs of trolled trials, which second-line antiseizure regimen is recom-
potentially ongoing seizures (nystagmus); thus the most likely diagnosis mended for benzodiazepine-resistant convulsive status epilepticus?
is nonconvulsive status epilepticus, which can be diagnosed only with an a. Fosphenytoin 20 mg phenytoin equivalent (PE)/kg
electroencephalogram. The elevated blood pressure along with hypoden- b. Valproic acid 40 mg/kg
sities in the posterior cortical region on CTH are highly suggestive of c. Levetiracetam 60 mg/kg
posterior reversible encephalopathy syndrome, a form of hypertensive d. All of the above
emergency. The patient has acute encephalopathy and the stem of the Answer: d. According to the Established Status Epilepticus Treatment
question does not display findings suggestive of cortical blindness (pre- Trial (ESETT) fosphenytoin 20 mg PE/kg (max 1500 mg), valproic
served pupillary light reflex with complete binocular vision loss). Lack acid 40 mg/kg (max 3000 mg), and levetiracetam 60 mg kg (max 4500
of prodrome of headache and confusion, fever, and meningismus make mg) were equally effective in terminating convulsive status epilepti-
meningoencephalitis less likely. The episode happened towards the end of cus in nearly half of randomized patients (fosphenytoin 45%, valproic
hemodialysis section, so profound uremia is unlikely. acid 46%, levetiracetam 47%). The ESETT trial has provided the best
CHAPTER 14 Seizures
evidence to date on the second-line treatment of convulsive status with cardiogenic syncope usually occur in the setting of hypoperfusion
epilepticus. and are brief, unlike this case.
4. A 37-year-old woman with remote history of head trauma from 6. Which additional features may assist in the diagnosis of nonepilep-
a motor vehicle collision presents to the emergency department tic spells?
with ongoing bilateral convulsions. The patient’s significant other a. Disproportionally short postictal confusional state
witnessed the onset of convulsions 45 minutes ago following an b. Normal lactic acid and creatine kinase
argument. She received a total of 10 mg of intramuscular (IM) c. Lack of reactive leukocytosis
midazolam while being transported by the emergency medical ser- d. All of the above
vices. On examination, T 36.3°C, HR 120 bpm, SBP 115 mm Hg, Answer: d. Although the only reliable way to diagnose nonepileptic
RR 22, pulse oximetry 100% on a nonrebreather mask; she has her spells is by characterizing the lack of an underlying seizure rhythm
eyes closed and resists passive eye opening of the examiner, does on electroencephalography (EEG) during spells, clinicians should be
not respond otherwise to the examiner and has ongoing asynchro- attentive to clues in the presentation and work-up that suggest a pseu-
nous, nonstereotypic, and suppressible convulsions of both upper doseizure diagnosis. Prolonged tonic-clonic convulsions (i.e., general-
extremities with associated pelvic thrust. ized convulsive status epilepticus) is invariably associated with some
What are the most important nonpharmacologic initial interven- degree of rhabdomyolysis and elevation of lactic acid. The complete
tions in patients with suspected generalized convulsive status blood count often displays reactive leukocytosis. Postictal-related con-
epilepticus? fusion or acute encephalopathy is one of the hallmarks of generalized
a. Placing an oropharyngeal airway device, lateral decubitus posi- tonic-clonic seizures and carries a direct relationship with the duration
tioning, establishing an intravenous (IV) line of seizures. Short periods of confusion following prolonged bilateral
b. Checking glucose fingerstick, establishing an IV line, lateral convulsions is suggestive of a nonepileptic spell diagnosis.
decubitus positioning 7. A 23-year-old woman currently 34 weeks’ pregnant with her first
c. Suctioning oral cavity, placing oropharyngeal airway device, lat- baby presents to the emergency department with headache and
eral decubitus positioning patient fatigue. On exam, T 36.2°C, HR 89 bpm, SBP 145 mm Hg, RR 27,
d. Checking glucose fingerstick, placing nasopharyngeal airway pulse oximetry 96%; she is fully oriented, complains of subjective
device, supine positioning blurry vision and has 2+ pitting edema of lower extremities. You
Answer: b. Initial nonpharmacologic interventions include ensuring suspect preeclampsia and initiate the work-up.
physical safety of patients with seizures to prevent traumatic injuries What is the typical period for the development of preeclampsia and
and aspiration. During convulsions, focusing on lateral decubitus posi- eclampsia?
tioning and suctioning of oral cavity are important to prevent aspira- a. 16–37 weeks of gestational age
tion; the use of intraoral devices may lead to trauma without additional b. 20 weeks of gestational age until 8 weeks postpartum
benefit. Nasopharyngeal airway devices may help with transient c. 20 weeks of gestational age until delivery
obstruction that may occur in the peri-ictal period, but oropharyngeal d. None of the above
devices should be avoided as they may induce gagging and vomiting Answer: b. In pregnant patients, the risk of eclampsia is highest from
and may damage the teeth or tongue. The serum glucose level should 20 weeks of gestational age up to 8 weeks postpartum. Clinical evalu-
be determined in every seizing or postictal patient as hypoglycemia is ation should focus on evaluating for associated symptoms (e.g., head-
a common metabolic cause of provoked seizures. Although some first- ache, visual abnormalities and confusion), as well as focal neurologic
line seizure abortive medications can be given IM, intraosseous (IO), deficits.
per rectum (PR), intranasal (IN), establishing an IV line is a helpful 8. What are the red flags that prompt neuroimaging in patients pre-
initial step to facilitate timely pharmacologic interventions. senting with eclamptic seizures?
5. An additional 6 mg of lorazepam IV (0.1 mg/kg) slows down the a. No red flags needed, all patients should get a noncontrast com-
movements but does not abort them completely. What is the most puted tomography of the head with abdominal shielding
common differential diagnosis for generalized convulsive status b. If persistent neurologic deficits, including prolonged loss of con-
epilepticus? sciousness or encephalopathy
a. Nonepileptic spells c. If more than two seizures are observed within 24 hours
b. Neuroleptic malignant syndrome d. If seizures are refractory to benzodiazepines
c. Complex migraine Answer: b. Neuroimaging should be considered if neurologic deficits
d. Cardiogenic syncope persist, prolonged loss of consciousness, or encephalopathy. Intrave-
Answer: a. Although all the answers reflect conditions that can be con- nous magnesium 4 to 6 g is first-line therapy in the pregnant patient
sidered seizure mimics, nonepileptic spells, or pseudoseizures, repre- experiencing new-onset seizures. Benzodiazepines and nonteratogenic
sent the most common differential diagnosis for generalized convulsive antiseizure medications are reasonable alternatives in magnesium-
status epilepticus with an incidence of approximately 10% of subjects refractory cases.
enrolled in clinical trials. Nonepileptic spells tend to last greater than 5 9. A 61-year-old previously healthy man presents to the emergency
minutes, thus meeting criteria for convulsive status epilepticus by dura- department with a first seizure. The episode consisted of left arm
tion of events, and be resistant to high doses of benzodiazepines. Neu- and face clonic movements with secondary generalization and post-
roleptic malignant syndrome is also associated with prolonged spells of ictal lethargy and left hemiparesis lasting 10 minutes. On examina-
abnormal movement, but the type of involuntary movement (typically tion, T 37.1°C, HR 79 bpm, SBP 105 mm Hg, RR 17, pulse oximetry
tonic extension of all extremities, opisthotonos and rigors) and associ- 96%, GCS is 15 with no focal deficits. Given the focality of seizure
ated hyperthermia, tachycardia, and diaphoresis can distinguish these onset and the patient age, you obtained a noncontrast computed
disorders. Complex migraine typically manifests as lateralizing dyses- tomography of the head, which was unremarkable. Which of the
thesia in association with severe throbbing headache with phono/pho- following factors should be considered when counseling patients on
tophobia, and not with prolonged convulsions. Convulsions associated recurrent seizure risk after a first-time seizure?
CHAPTER 14 Seizures
a. Presence of focal abnormalities on electroencephalography 13. A 24-year-old man is brought to the emergency department by
(EEG) emergency medical services (EMS). The patient’s mother reports
b. Presence of focal abnormalities on neuroimaging that she found her son seizing on the floor of her living room
c. Identification of seizure triggers approximately 30 minutes before arrival at the hospital. Two
d. All of the above months ago, the patient returned from Mexico, where he had
Answer: c. Up to 45% of adults presenting with a first unprovoked sei- been incarcerated for 6 months. The mother reports that during
zure will experience another within 2 years, most occurring in the first the past 2 months she has seen her son consistently take his sei-
year 2. The risk is higher with known brain lesions, neuroimaging or zure medicine and several other pills for a “bad lung infection”
EEG abnormalities, or nocturnal seizures 2; the latter are associated he got in Mexico. She cannot remember the names of any of the
with epilepsy syndromes. medications. Several doses of intravenous (IV) lorazepam have
10. A 71-year-old man with advanced dementia from multiple prior been administered, with no effect on the patient’s seizure activity.
strokes, atrial fibrillation on warfarin, localization related epilepsy Which of the following medications would be the most effective in
on levetiracetam presents to the emergency department with a aborting his seizure activity?
breakthrough seizure. The episode consisted of speech arrest and a. Diazepam
clonic movements of right face and arm. There was no postictal state b. Magnesium sulfate
identified. On examination, T 36.1°C, HR 109 bpm, SBP 115 mm c. Phenytoin
Hg, RR 15, pulse oximetry 97%, GCS is 15 with no focal deficits. d. Pyridoxine
Which of the following factors should be considered when deciding e. Valproic acid
on the need for neuroimaging after a breakthrough seizure? Answer: d. Several historical clues in this scenario point to tubercu-
a. No red flags needed, all patients should get a noncontrast losis being the “bad lung infection” in this patient. In patients with
CTH seizures that are refractory to benzodiazepines, isoniazid (a common
b. Young age, specifically in the pediatric population medication for tuberculosis) overdose is a possibility and should be
c. Use of vitamin K antagonist considered. Pyridoxine is the only fully effective pharmacologic treat-
d. Lack of identifiable triggers ment for toxic isoniazid seizures, although benzodiazepines have been
Answer: c. Neuroimaging should be considered if neurologic deficits shown to suppress seizure activity in some cases.
persist, prolonged loss of consciousness or coma, advanced age, immu- 14. A mother arrives with her 10-year-old daughter (41 kg) who has
nocompromised state, coagulopathy (including anticoagulant drugs) been seizing for at least 10 minutes. The patient has a history of epi-
or hypercoagulability, history of malignancy, preceding severe thun- lepsy, and a home dose of rectal diazepam has been ineffective. The
derclap headache, status epilepticus, in patients with stigmata of neu- mother states that the child has been in her usual state of good health
rocutaneous syndromes, and suspected trauma. until the seizure began, and there has been no history of trauma.
11. What is the most common metabolic cause of seizure activity? Which of the following is the most appropriate initial action?
a. Hypercalcemia a. Administer 10 mg midazolam intramuscularly.
b. Hyperglycemia b. Consult neurology to obtain a bedside electroencephalogram.
c. Hypermagnesemia c. Endotracheal intubation with vecuronium and etomidate.
d. Hypocalcemia d. Establish vascular access and administer 2 mg of lorazepam.
e. Hypoglycemia e. Obtain an immediate computed tomography scan of the head.
Answer: e. Seizure activity secondary to metabolic derangements is Answer: a. Early, aggressive benzodiazepine administration is asso-
most commonly caused by hypoglycemia. The only treatment required ciated with decreased morbidity and mortality in status epilepticus.
in this situation may be intravenous (IV) glucose. Prolonged seizure Intramuscular midazolam is superior to intravenous lorazepam; in
activity may also cause hypoglycemia so that the cause-and-effect addition, the dose of lorazepam is inadequate. Endotracheal intubation
relationship may sometimes be reversed, and further therapy may be may ultimately be required but is a secondary priority; use of a long-
required. acting neuromuscular blockade agent, such as vecuronium, should be
12. A 15-year-old girl is brought to the emergency department for avoided. Cranial computed tomography may or may not be needed in
evaluation of a recent seizure. While awaiting laboratory results, this patient, depending on the response to benzodiazepine therapy.
she begins to have further seizure activity. Which of the following Bedside electroencephalograms are most useful in diagnosing noncon-
is the optimal first-line agent to terminate her seizure activity? vulsive status epilepticus.
a. Fosphenytoin 15. Paramedics present with a 24-year-old woman with a history of
b. Lorazepam epilepsy after a seizure. She is somnolent but easily arousable and
c. Phenobarbital oriented to self and year. Her vital signs are normal, there are no
d. Phenytoin signs of trauma, and an empty expired bottle of phenytoin is found
e. Valproic acid in her purse. Her prehospital finger stick blood glucose level is 163
Answer: b. Benzodiazepines are the optimal first-line agents for stop- mg/dL. Which of the following treatment options is most correct?
ping seizure activity in patients of all ages. Available agents include a. Administer 20 mg/kg fosphenytoin intramuscularly and
lorazepam, diazepam, and midazolam. Phenytoin is recommended observe the patient until she returns to baseline.
as second-line therapy for adults with persistent seizure activity. The b. Establish vascular access, and administer 4 mg lorazepam
prodrug, fosphenytoin, can be administered more quickly, can be intravenously (IV).
given intramuscularly, and has less of a tendency to cause hypotension. c. Establish vascular access, obtain a phenytoin level, and ad-
Second-line therapy for children is phenobarbital. Third-line therapy minister 1 to 2 mg lorazepam IV if the patient begins to seize.
is pentobarbital, propofol, or a benzodiazepine infusion. Valproic acid d. Place the patient in a monitored setting, establish vascular
should be considered for patients who are on chronic valproic therapy access, and withhold diagnostic tests and treatments unless
and whose levels are subtherapeutic. patient’s condition changes.
CHAPTER 14 Seizures
Answer: c. Medication noncompliance is a frequent cause of seizures Answer: c. Adults presenting with a first seizure should undergo cra-
in adults with epilepsy. It is recommended to check phenytoin levels nial computed tomography and evaluation of serum electrolyte and
before administering additional drug. Because the patient is not in sta- glucose levels because abnormalities would likely influence disposition
tus epilepticus, a low dose of benzodiazepines can be considered for while identifying potentially life-threatening conditions. For otherwise
patients who begin to seize while undergoing a period of observation. healthy adults with normal findings after evaluation, early outpatient
16. Which of the following is not part of the routine emergency follow-up can be considered. In some cases, initiation of antiepileptic
department evaluation and treatment of a 21-year-old healthy drugs can be considered after a first seizure; however, this should be
woman with a first seizure? done in consultation with the neurologist responsible for outpatient
a. Discharge, with early outpatient neurology follow-up follow-up.
b. Evaluation of serum electrolyte levels
c. Initiation of antiepileptic drug therapy
d. Performance of cranial computed tomography
15 Dizziness and Vertigo
KEY CONCEPTS
• N eurologic complaints, such as imbalance, dysarthria, or numbness in • T he presence of auditory symptoms (e.g., hearing loss, tinnitus) suggests a
patients with dizziness/vertigo raise the likelihood of transient ischemic peripheral cause of the vertigo.
attack (TIA) or stroke as the cause. • Acute vestibular syndrome is diagnosed when dizziness develops acutely; is
• Benign paroxysmal positional vertigo (BPPV) requires head movement to constant; is accompanied by nausea or vomiting, unsteady gait, nystagmus,
elicit symptoms. Consequently, the Hallpike test should not be performed and intolerance to head motion; and persists for longer than a day.
if the patient is actively symptomatic during history taking (and the patient’s • Neck injury resulting in a vertebral artery dissection causes posterior circu-
head has not been recently moved) because such a history is inconsistent lation ischemia presenting with vertigo.
with BPPV. • Nystagmus is the cardinal sign of inner ear disease and the principal objec-
• When performing the Hallpike test, the head should be turned to the side 45 tive evidence of vestibular dysfunction.
degrees prior to laying the patient back into the head-hanging position. • HINTS (Head Impulse test, Nystagmus, Test of Skew) is a bedside oculo-
• A positive Hallpike test should elicit upbeating (fast phase towards eye- motor examination test that helps to differentiate central from peripheral
brows) nystagmus. vertigo in symptomatic patients with a first ever onset of constant vertigo
• The Epley maneuver is used to treat posterior semicircular canal BPPV, from acute vestibular syndrome.
which is the most common subtype of BPPV. • Meclizine (Antivert) has a time of onset of approximately 1 hour.
• Central causes of nystagmus are more likely when the pattern of nystagmus • Benzodiazepines are not recommended for the treatment of vestibular neu-
is purely vertical, downbeating (fast phase beating toward the nose), non- ritis or labyrinthitis in patients who are discharged home, because they can
fatigable, direction changing with gaze, or spontaneous pure torsional. interfere with the process of vestibular rehabilitation.
1. A patient presents to the emergency department with dizziness test is expected in vestibular neuritis, while torsional upbeat nystagmus
and is diagnosed with horizontal canal benign paroxysmal posi- is expected in posterior canal BPPV.
tional vertigo (BPPV). What maneuver can be used to treat the 3. Internuclear ophthalmoplegia most often suggests a diagnosis of
patient? which of the following conditions?
a. Barbecue roll a. Horizontal canal benign paroxysmal positional vertigo (BPPV)
b. Epley maneuver b. Labyrinthitis
c. Hallpike test c. Multiple sclerosis
d. Head impulse test d. Vestibular neuritis
Answer: a. The supine roll test, in which the patient lies flat on the gur- Answer: c. Internuclear ophthalmoplegia is diagnosed when, on eye
ney and the head is turned to each side, is used to diagnose horizontal movement, the adducting eye shows little to no movement while the
canal BPPV, whereas the barbecue roll maneuver is used to treat the abducting eye moves normally. In a patient presenting with vertigo,
horizontal variant of BPPV. The Epley maneuver is used to treat poste- this finding is virtually pathognomonic for multiple sclerosis.
rior canal BPPV. The Hallpike test is used to diagnose posterior canal 4. Which of the following is a central cause of vertigo?
BPPV. The head impulse test is used to diagnose vestibular neuritis and a. Labyrinthitis
labyrinthitis. b. Ménière disease
2. A 70-year-old woman presents with a first ever episode of severe ver- c. Vertebrobasilar insufficiency
tigo. Which of the following examination findings should prompt the d. Vestibular neuritis
physician to order imaging tests and/or consultation with a neurolo- Answer: c. All the other causes are peripheral.
gist? 5. Continuous vertigo of what duration is used to define acute vestib-
a. Direction changing nystagmus on change in head position ular syndrome?
b. Direction changing nystagmus on change in lateral gaze a. 1 hour
c. Positive head impulse test b. 8 hours
d. Torsional upbeat nystagmus during Hallpike test c. 24 hours
Answer: b. Direction changing nystagmus on change in gaze is con- d. 1 week
cerning for a central cause of vertigo and makes up part of the HINTS Answer: c. Acute vestibular syndrome has an arbitrary cutoff of contin-
test. Direction changing nystagmus with changes in head position is uous vertigo for at least 1 day in part to help differentiate acute vestibu-
expected in a patient with horizontal canal benign paroxysmal posi- lar syndrome from attacks of Ménière disease or prolonged migrainous
tional vertigo (BPPV) and is a benign finding. A positive head impulse vertigo.
Headache 16
KEY CONCEPTS
• P atients with a known headache disorder who present with a change in the
character of the headache should be evaluated for potential serious causes.
• The physical examination in the headache patient focuses primarily on
mental status, funduscopic exam, meningeal signs, and neurologic exam-
ination, particularly cranial nerves (CNVs) II, III, IV, and VI, as well as spe-
cific cerebral territories.
• Mild to moderate primary and nonspecific headaches are treated with non-
steroidal analgesic drugs (NSAIDs). Severe headaches can be managed
parenterally with dopamine antagonists (metoclopramide or prochlorper-
azine), migraine-specific agents (triptans), or NSAIDS (Ketorolac). Opioid
pain medications are not indicated for the treatment of primary headaches.
• Most patients with headache do not require neuroimaging. When obtained,
neuroimaging is tailored to the specific elements of concern in the differen-
tial diagnosis.
• The differential diagnosis of sudden severe headache includes subarach-
noid hemorrhage (SAH) or other intracranial hemorrhage (ICH), cerebral
venous thrombosis, and cervical artery dissection (Box 16.1).
• In patients suspected for SAH, a stepwise application of Ottawa SAH Rule,
computed tomography (CT) imaging, followed by lumbar puncture (LP) and/
or CT angiography (CTA) is appropriate.
• Antibiotics should be administered prior to performing a LP when bacterial
meningitis is suspected.
1. Of the following, which is most appropriate in the initial evaluation c. Magnetic resonance angiography (MRA)/magnetic resonance
of a patient with nontraumatic headache? venogram (MRV)
a. Computed tomography (CT) scan of brain d. No studies are indicated
b. Electroencephalogram (EEG) Answer: a. CT studies with and without contrast are able to iden-
c. Magnetic resonance imaging scan of brain tify many vascular causes of headache, for which pregnancy is a
d. Thorough neurological evaluation risk factor. Iodinated contrast is safe in pregnancy. Magnetic reso-
Answer: d. A thorough neurological examination may reveal deficits nance imaging (MRI) without contrast is another option; however,
not seen on gross evaluation, prompting expansion of the differential MRA and MRV are relatively contraindicated due to use of gado-
diagnosis to include more concerning etiologies. Depending on the linium contrast, which has not been shown to be safe in pregnancy.
history and remainder of the physical, a normal neurological examina- Inflammatory markers are nonspecific and will not confirm or rule
tion may be reassuring and obviate need for advanced imaging studies. out important emergency headaches in this population. For emer-
2. In the setting of headache, the presence of nausea and vomiting are gency headaches in pregnancy deferral of additional studies is not
diagnostic of which of the following underlying causes? appropriate.
a. Glaucoma 4. A 42-year-old male patient presents with the sudden onset of severe
b. Increased intracranial pressure headache that worsened over the 30 minutes while at rest 4 hours
c. Migraine prior to arriving in the emergency department (ED). His examina-
d. None of the above tion reveals only mild distress, no neck stiffness or meningismus,
Answer: d. Nausea and vomiting are completely nonspecific. Migraine and normal funduscopic and neurological examinations. What is
headaches, increased intracranial pressure, temporal arteritis, and the most appropriate diagnostic test?
glaucoma can all be manifested by severe nausea and vomiting, as can a. Intraocular pressure determination
some systemic viral infections with headache. Such factors may point b. Sphenopalatine block
toward the intensity of the discomfort but are not specific in establish- c. Computed tomography
ing the diagnosis. d. Lumbar puncture
3. Which of the following is the most appropriate initial diagnostic Answer: c. The patient has a sudden onset severe headache and Ottawa
study for a pregnant woman with normal vital signs and a presenta- Subarachnoid Hemorrhage Rule can be applied. However, his age being
tion concerning for a headache emergency? greater than 40, the rule is unable to exclude subarachnoid hemorrhage
a. Computed tomography (CT) (SAH). Next step is computed tomography (CT) because he is within
b. Inflammatory markers (e.g. erythrocyte sedimentation rate the 6-hour window. Lumbar puncture and or/ CT angiography can be
[ESR], C-reactive protein [CRP]) considered if initial noncontrast CT is normal.
17 Diplopia
KEY CONCEPTS
• T he diagnostic approach to diplopia is aimed at determining (1) if the diplopia • A CN VI palsy will cause the affected eye to turn inward and results in
is monocular, (2) if there is a restrictive or mechanical issue in the orbit or diplopia that worsens on lateral gaze toward the affected side.
orbital structures, (3) if there is a palsy of one or more of the oculomotor • Cavernous sinus or posterior orbit disease (orbital apex syndrome) may
cranial nerves, (4) if there is a neuroaxial process involving the brainstem and present with ipsilateral diplopia involving CN III, IV and or VI; associated
related cranial nerves, or (5) if there is a systemic neuromuscular process. symptoms may include pain, exophthalmos, chemosis, and possibly ipsilat-
• Monocular diplopia is most commonly caused by an ophthalmologic prob- eral periorbital facial numbness or dysesthesia.
lem related to refractory distortions or retinal buckling. • Multiple sclerosis or localized pathology in the brainstem, such as tumor,
• Binocular diplopia resolves when either eye is closed and is the result of a lacunar infarct, basilar artery thrombosis, vertebral artery dissection or oph-
misalignment in the visual axes. thalmoplegic migraine, may present with focal lesions presenting as diplopia.
• Important historical features in assessing diplopia includes onset, direc- • The diplopia caused by myasthenia gravis is associated with ptosis, gets
tionality and orientation of the diplopia; presence of pain, and the presence worse as the patient fatigues, and improves with rest or on placing ice over
of other associated symptoms. the eye.
• An isolated CN III palsy presents with diplopia in all directions of gaze, • Initial treatment for diplopia is directed toward imminent threats to airway
except on lateral gaze to the affected side; the eye is deviated down and and ventilation (e.g., with botulism and myasthenia gravis) and immediate
out; ptosis and a dilated pupil may also be present. or rapidly evolving threats to the central nervous system (CNS) (e.g., stroke,
• A CN IV palsy results in vertical or torsional diplopia that worsens on look- meningoencephalitis or Wernicke encephalopathy).
ing down and toward the nose.
1. A 65-year-old man with a longstanding history of diabetes and hypothyroidism if it is a presentation of or treatment complication of
hypertension presents with sudden onset of persistent diplopia that Graves disease but would not change so markedly with activity.
began a few hours before arrival. He describes left retro-orbital dis- 3. A 76-year-old man with hypertension, hypercholesterolemia, and
comfort, and his examination is notable for a left eye that is deviated diet-controlled diabetes presents with a sudden onset of diplopia
laterally and downward, with a palsy of movement medially and that developed 30 minutes before arrival. Paramedics state that the
upward. He also has a left-sided ptosis but no conjunctival injec- patient’s wife reported that he suddenly began staggering around
tion, chemosis, or proptosis. His pupils are equal in size at 4 mm, the room, unable to bear weight on his left side. On examination,
round, and equally reactive to light in both a direct and consensual the patient has normal vital signs except for mild hypertension and
reflex, and his examination is otherwise unremarkable. What is the has a right CN III palsy, with left arm and leg weakness. He has no
most likely cause of the diplopia? airway complaints and denies any pain. What is the most appropri-
a. Brain tumor ate initial response?
b. Cerebral aneurysm a. Checking blood gas levels and assessing the patient’s negative
c. Microvascular ischemia inspiratory force
d. Orbital apex syndrome b. Emergent treatment with botulinum antitoxin
Answer: c. Based on examination, this is a patient who has a pupil- c. Initiating broad-spectrum antibiotics to cover upper respiratory
sparing complete CN III (third nerve) palsy. Because his pupillary pathogens
examination is normal, with an otherwise complete CN III palsy, the d. Initiating clinical measures to address an acute ischemic stroke
palsy is very unlikely to be due to external compression from a brain Answer: d. The paroxysmal onset of the patient’s symptoms, with
tumor, aneurysm, or orbital apex process. It is a typical presentation of focal neurologic symptoms and signs, suggests an ischemic event. His
microvascular ischemia, to which the patient is predisposed, given his crossed deficits and discrete CN III palsy suggest localization in the
history of diabetes and hypertension. brainstem.
2. A 56-year-old woman presents with recurrent episodes of diplopia 4. Which constellation of symptoms is most concerning for food-
that have been ongoing for a week. She describes double vision borne botulism?
that gradually comes and goes, typically worse at the end of the a. Double vision, headache, and right leg weakness
day, with no particular direction or orientation to the diplopia. b. Double vision, left eye discomfort, and periorbital swelling
The patient’s coworker, who is present in the emergency depart- c. Double vision, nystagmus, and confusion
ment (ED) with her, states that the patient’s eyes “looked droopy” d. Double vision, slurred speech, difficulty swallowing, and dry
during an animated staff meeting they attended that afternoon but mouth
look normal now. The patient also describes waxing and waning Answer: d. Double vision, slurred speech, difficulty swallowing, and
general muscular weakness that has also been present this past dry mouth would be present with foodborne botulism.
week but denies any other symptoms and states that when she 5. A 45-year-old woman presents with progressively worsening dou-
rests, she feels better. With which entity are her symptoms most ble vision and the inability to adduct the eye on one side in the
consistent? contralateral direction during lateral gaze that resolves during
a. Botulism convergence on physical examination. What disease should be sus-
b. Hypothyroidism pected with these findings?
c. Miller-Fisher syndrome a. Multiple sclerosis
d. Myasthenia gravis b. Botulism
Answer: d. The patient and coworker are describing what appears c. Myasthenia gravis
to be an activity-related diplopia, with generalized muscle weakness d. Idiopathic intracranial hypertension
and lack of other focal symptoms, all very suggestive of a possible Answer: a. An internuclear ophthalmoplegia (INO), suggested by an
neuromuscular process (myasthenia gravis). Miller-Fisher syndrome inability to adduct the eye on one side in the contralateral direction
would not be associated with muscle weakness and would not wax and during lateral gaze that resolves during convergence, implicates a lesion
wane. Botulism would typically have a more progressive course, with in the medial longitudinal fasciculus (MLF) and is typically found in
other associated bulbar symptoms. Diplopia may be associated with patients with multiple sclerosis.
Red and Painful Eye
18
KEY CONCEPTS
• C ritical diagnoses, such as caustic injury, orbital compartment syndrome,
and narrow angle (acute angle closure) glaucoma, require immediate treat-
ment and ophthalmology consultation.
• Prompt and prolonged irrigation is essential for patients who experience
caustic injury to the eye.
• Headache and nausea may be prominent symptoms in patients with narrow
angle glaucoma.
• Complete abolition of a foreign body sensation after instillation of local
anesthetic solution indicates a high likelihood of a superficial corneal
lesion.
• Keratitis, inflammation of the cornea, is most commonly caused by a viral
infection, but may also be caused by recent ultraviolet light exposure,
chemical injury, or hypoxic injury from contact lens use.
• A localized corneal defect with edematous, inflammatory changes may sig-
nal corneal ulceration.
• A corneal dendritic pattern may signal a herpetic infection, which can prog-
ress to corneal opacification and visual loss.
• Pain, consensual photophobia, perilimbic conjunctival injection, and a
miotic pupil could signal iritis or uveitis. The cause may be trauma or an
underlying autoimmune disease. The presence of cell and flare in the ante-
rior chamber can help identify these conditions.
• Conjunctivitis is usually self-limited and rarely requires antibiotic treatment.
1. A patient who normally wears contact lenses is diagnosed with bac- Answer: c. A collection of layered pus in the dependent portion of the
terial conjunctivitis. Which of the following is the preferred treat- anterior chamber is called a hypopyon.
ment in this patient? 4. Which of the following results from inflammation of a meibomian
a. Bacitracin gland?
b. Chloramphenicol a. Blepharitis
c. Erythromycin b. Chalazion
d. Moxifloxacin c. Dacryocystitis
Answer: d. Patients who wear contact lenses are at increased risk for d. Hordeolum
infections with Pseudomonas and, in the setting of bacterial conjuncti- Answer: b. Inflammation of a meibomian gland with the subsequent
vitis, should be prescribed a quinolone, barring any contraindications. formation of a subcutaneous nodule within the eyelid is known as a
2. A 15-year-old boy presents to the emergency department (ED) after chalazion. This condition typically resolves spontaneously over several
having been shot in the face with a BB gun. He has a solitary pen- days. Authorities often recommend warm compress application and
etrating wound just inferior to his left eye. His visual acuity in the gentle massage of the nodule several times a day, although there is no
left eye is limited to light perception, but he reports having normal evidence supporting this. If complete resolution does not occur within
vision prior to the injury. He has significant proptosis of his left eye, 2 weeks, the patient should be referred to an ophthalmologist.
and his fundus is clearly seen with direct ophthalmoscopy. Intra- 5. A neonate presents 7 days after birth with a unilateral red eye. Fur-
ocular pressure (IOP) of the affected eye is 50 mm Hg. His mental ther examination reveals inflamed conjunctiva, purulent discharge
status is normal. What is the most appropriate next step in the man- and negative uptake on fluorescein examination. The child was
agement of this patient? born full term after a pregnancy complicated by poor prenatal care.
a. CT scan of the head and face Which of the following diagnoses is most likely in this patient?
b. ED observation with repeated neurologic examinations a. Chemical conjunctivitis
c. Lateral canthotomy and inferior cantholysis b. Chlamydia trachomatis
d. Plain radiography of the face c. Corneal abrasion
Answer: c. The described patient likely has a retrobulbar hematoma d. Neisseria gonorrhoeae
with visual acuity changes and an elevated IOP. The elevated IOP with Answer: b. Neonates are at high risk for serious ophthalmologic dis-
a clear fundoscopy are findings consistent with no penetration into the eases. Typically, they present with chemical conjunctivitis from anti-
globe. Although CT scan of the head and face is indicated to further bacterial prophylaxis (i.e., erythromycin ointment) at birth within 2
delineate specific injuries, lateral canthotomy and inferior cantholysis days. Gonococcal conjunctivitis is seen 2 to 5 days after birth. Chla-
is emergently necessary for orbital decompression in an attempt to sal- mydial conjunctivitis is seen 5 to 12 days after birth. Corneal abrasions,
vage vision. This sight-saving procedure should not be delayed more while common in this age group, will most likely be evident on fluores-
than 90 minutes after injury when severe findings (decreased visual cein examination of the cornea.
acuity and significantly increased IOP) are present. Ophthalmology
consultation would be indicated emergently.
3. A collection of pus in the anterior chamber of the eye is known as
which of the following?
a. Cotton wool spot
b. Dacryocystitis
c. Hyphema
d. Hypopyon
Sore Throat 19
KEY CONCEPTS
• S ore throat can indicate a range of pathology, from simple pharyngitis to
deep space infection, with or without airway compromise.
• Physical examination, supplemented by nasopharyngoscopy when appro-
priate, is key to determining threats to the airway and establishing a diag-
nosis.
• If oropharyngeal examination reveals minimal findings, lower or deeper
structures may be involved, and endoscopic examination of the upper air-
way is recommended.
• The modified Centor criteria can help identify adult patients with a higher
likelihood of Group A Streptococcus (GAS) infection. Patients with a score
of −1 to 1 are unlikely to have GAS infection (10% to 15% likelihood, 95%
confidence interval 5% to 25%), while a score of 4 or 5 corresponds to a
40% to 50% likelihood of GAS infection (95% confidence interval 31% to
51%).
• Viral pharyngitis, the most common cause of pharyngitis, is self-limiting
and antibiotics are more harmful than helpful in its management.
• For GAS-positive pharyngitis, a single injection of penicillin or a 10-day
course of oral penicillin may decrease the duration of symptoms, transmis-
sion to close contacts, and prevent rare suppurative and nonsuppurative
complications.
1. A 20-year-old female presents to the emergency department com- c. Metronidazole 30 mg/kg/day divided TID × 10 days
plaining of a sore throat. She has a temperature of 39°C, tender cer- d. Penicillin VK 50 mg/kg/day divided QID × 10 days
vical lymphadenopathy, exudates on exam and denies any history Answer: d. Penicillin VK is the first-line treatment for GAS pharyn-
of cough, rhinorrhea, nausea, vomiting or diarrhea. Her modified gitis in patients without allergies. Clindamycin and azithromycin are
Centor score is which of the following? options in penicillin allergic patients. Metronidazole is not a treatment
a. 2 of GAS pharyngitis.
b. 3 4. A 24-year-old female presents for sore throat, difficulty swallowing,
c. 4 and a muffled voice. On examination, she has fullness, swelling and
d. 5 induration of the left posterior peritonsillar area with uvular devi-
Answer: c. This patient meets four Centor criteria: fever, tender cervi- ation to the right. What is the next step in the management of this
cal lymphadenopathy, exudates, and absence of a cough. The age crite- patient?
ria is +1 point for age 3 to 14 years, and −1 point for age greater than a. Computed tomography imaging
45 years. b. Discharge with antibiotics, clindamycin 300 mg TID × 10 days
2. A 39-year-old male presents to the emergency department com- c. Drainage either by needle aspiration or incision and drainage
plaining of a sore throat. He is sitting forward on the stretcher, d. Rapid Antigen Detection Test for Group A Streptococcus
drooling, with inspiratory stridor. His oropharynx exam shows Answer: c. This patient has a peritonsillar abscess. In a clear-cut case
minimal erythema and no exudate. Which of the following condi- such as this, no imaging is needed. Definitive treatment is with either
tions is the likely diagnosis? needle aspiration or incision and drainage.
a. Epiglottitis 5. In which age group is Group A streptococcal infection most com-
b. Group A streptococcal pharyngitis mon?
c. Periodic fever, aphthous stomatitis, pharyngitis, and cervical a. Age <3 years
adenitis (PFAPA) syndrome b. Age 5–15 years
d. Viral pharyngitis c. Age 15–45 years
Answer: a. Simple pharyngitis, either bacterial or viral, and PFAPA d. Age >45 years
syndrome do not cause airway compromise. Drooling and inspiratory Answer: b. GAS pharyngitis is most common in children aged 5 to 15
stridor are symptoms of airway compromise, and epiglottitis is the only years. It is less common in young adults, and rare in children less than
diagnosis on this list that results in airway compromise. three, or adults over the age of 45.
3. In adult patients who test positive for Group A streptococcal (GAS)
pharyngitis who have no allergies, which of the following is the
most appropriate antibiotic choice?
a. Azithromycin 12 mg/kg/day (max dose, 500 mg) × 5 days
b. Clindamycin 7 mg/kg/dose (max dose, 300 mg) TID × 10 days
Hemoptysis 20
KEY CONCEPTS
• H emoptysis is caused by infection, trauma, cancer, or coagulopathy or as a Answer: b. In patients with massive hemoptysis, plain films may local-
complication of invasive pulmonary procedures. ize the site of hemorrhage in approximately 50% of patients. However,
• “Massive hemoptysis” is defined as greater than 100 mL of blood loss or high-resolution multidetector CT of the chest is the principal diag-
approximately ½ cup of blood in a 24-hour period or a bleeding rate ≥100 nostic test to detect both bronchial and nonbronchial arterial causes
mL/h. of massive hemoptysis. CT is diagnostically comparable with, but less
• Plain radiographs are the initial screening test in most cases of massive invasive than, conventional angiography, which currently is done as a
hemoptysis, although high-resolution chest computed tomography scans combined diagnostic/therapeutic modality. A chest CT scan should be
are more sensitive and can supplant plain chest x-rays as the initial diag- obtained in high-risk patients (smokers and oncology patients) or in
nostic test. any patient with moderate-to-severe bleeding even if the initial chest
• Bronchial artery embolization is highly effective, with hemostasis rates radiograph is normal. CT localization of hemorrhage can expedite
ranging from 85% to 95%. bronchoscopic evaluation or guide subsequent interventional pro-
• With massive hemoptysis, hypoxia is the more immediate concern rather cedures. Patients with “garden variety” bronchitis, normal oxygen-
than volume resuscitation, and early intubation to ensure adequate oxygen- ation, and a normal exam do not require imaging in the emergency
ation is paramount. Most causes of mortality are a result of asphyxiation. department.
• If a tracheo-innominate artery fistula (TIF) is suspected, then overinflation of 4. A 50-year-old man presents after an episode of hemoptysis. He
the tracheostomy balloon or digital pressure at the site of bleeding should describes coughing up several large clots of dark blood. During
be performed for immediate hemorrhage control. his evaluation, he coughs and expectorates approximately 5 mL of
clotted blood. The patient’s vital signs are normal, and no abnor-
malities are noted on physical examination. His chest radiograph is
1. What is the most common cause of trace hemoptysis (blood-tinged
normal. Which of the following is the most appropriate next step in
sputum)?
the management of this patient?
a. Bronchiectasis
a. Admission to an observation unit
b. Bronchitis
b. Consultation for bronchoscopy
c. Cancer
c. Consultation for percutaneous embolization
d. Congestive heart failure
d. Discharge home with follow-up in 24 hours
e. Pulmonary embolism
e. Obtain chest high-resolution computed tomography (HRCT)
Answer: b. The most common cause of small-volume hemoptysis is
scan
bronchitis.
Answer: e. Since the advent of HRCT, radiologic evaluation has
2. Disruption of which of the following vessels is responsible for the
had an integral role in the evaluation and management of patients
vast majority of cases of massive hemoptysis?
with hemoptysis. Unless the initial chest radiograph is diagnostic or
a. Aorta
the patient is hemodynamically unstable, an HRCT scan should be
b. Bronchial arteries
obtained in most cases. Further management strategy should occur in
c. Pulmonary arteries
conjunction with pulmonary and thoracic surgery consultants, guided
d. Pulmonary veins
by the HRCT results.
e. Tracheobronchial capillaries
5. A 58-year-old man with a single lung transplant presents to the
Answer: b. Massive hemoptysis almost exclusively involves one of the
emergency department (ED) with what appears to be large-volume
two sets of vessels that constitute the lung’s dual blood supply. Bron-
hemoptysis. He was just discharged from the endoscopy suite,
chial arteries, direct branches from the thoracic aorta, are responsible
where he had a number of surveillance biopsies performed. He
for supplying oxygenated blood to the lung parenchyma. Disruption of
looks pale and diaphoretic with an initial oxygen saturation of 71%.
these vessels can result in sudden and profound hemorrhage. Although
After placement of an intravenous line and supplemental oxygen,
small in caliber, the bronchial circulation is a high-pressure system and
the next most appropriate step is:
the cause in nearly 90% of the cases of massive hemoptysis requiring
a. Blood transfusion
embolization. Although they transmit large volumes of blood, pul-
b. High-resolution computed tomography scan of the chest
monary arteries are at much lower pressure and, unless affected at c. Intubation
a very central location, are less likely to cause massive hemoptysis.
d. Thoracic surgery consultation
Trace hemoptysis typically originates from tracheobronchial capillar-
Answer: c. This patient is profoundly hypoxic and will need imag-
ies that become disrupted with vigorous coughing or minor bronchial
ing outside of the ED and invasive procedures. The primary cause of
infections.
mortality in patients with hemoptysis is asphyxiation. All resuscitative
3. Which of the following statements regarding the evaluation of
and procedural efforts will be futile without intubation and adequate
hemoptysis is true?
oxygenation.
a. Chest x-ray localizes the site of bleeding in nearly 90% of cases of
patients with large volume hemoptysis.
b. Conventional angiography is the preferred diagnostic test to
detect both bronchial and nonbronchial arterial causes of mas-
sive hemoptysis.
c. High-resolution CT, even with recent advances in technology,
remains diagnostically inferior to angiography.
d. Patients with fever, cough, and blood-tinged sputum should
receive a chest x- ray even if the vital signs are normal.
21 Dyspnea
KEY CONCEPTS
3. A 34-year-old male was struck repeatedly with a pipe in the right
• D yspnea results from a variety of conditions, ranging from nonurgent to chest. He becomes acutely more dyspneic during emergency med-
life-threatening. Neither the clinical severity nor the patient’s perception ical services (EMS) transport and becomes clammy, hypotensive,
correlates well with the seriousness of underlying pathology. and more tachycardic on arrival to the emergency department
• Dyspnea is subjective and the differential diagnosis can be divided into (ED). Examination reveals tachypnea, right chest crepitus, and
acute, acute on chronic, and chronic causes, of which the majority are car- decreased breath sounds. Which of the following actions should be
diopulmonary. Other causes include metabolic, infectious, neuromuscular, done next?
traumatic, psychiatric, and hematologic conditions. a. Needle chest decompression of the right chest
• Chronic or progressive dyspnea usually denotes primary cardiac or pul- b. Perform portable chest radiograph
monary disease. Acute dyspneic spells may result from asthma exacerba- c. Provide supplemental oxygen by non-rebreather mask
tion, infection, pulmonary embolism, cardiac dysfunction, or psychogenic d. Rapid sequence intubation (RSI) and endotracheal intubation
causes. Answer: a. Needle chest decompression is indicated for management
• All patients experiencing dyspnea, regardless of etiology, should be of a likely tension pneumothorax. If ultrasound is immediately avail-
promptly evaluated. Bedside pulse oximetry readings should be obtained, able, it can be used to confirm pneumothorax, but in this patient, who
and the patient placed on a cardiac monitor. is in cardiovascular collapse, immediate intervention is necessary.
• If the pulse oximetry is less than 92% on room air, the patient should be 4. Which of the following causes of acute respiratory distress is most
placed on supplemental oxygen either by nasal cannula or mask, depending likely to be associated with decreased respiratory effort?
on the degree of desaturation. a. Pleural effusion
• If altered level of consciousness, or hypoxia cannot be improved with suc- b. Tension pneumothorax
tioning, supplemental oxygen, or airway adjuncts then breathing should be c. Cyanide poisoning
assisted with manual or mechanical ventilation, either noninvasively for the d. Multiple sclerosis
short term, or with intubation for prolonged ventilation. Answer: d. multiple sclerosis. Several different neuromuscular diseases
• Unstable patients or patients with critical diagnoses must be stabilized and are associated with decreased respiratory effort in acute distress due
may require an emergent procedure and admission to an intensive care largely to inadequate ability to create sufficient negative inspiratory
unit. Urgent patients who improve with emergency department manage- force and generate adequate tidal volume and minute ventilation.
ment may be admitted to an intermediate care unit. Patients diagnosed
with urgent conditions in danger of deterioration without proper treatment 5. When assessing a patient with acute respiratory distress, a shark fin
or patients with severe comorbidities, such as diabetes, immunosuppres- waveform on capnography suggests which of the following condi-
sion, or cancer, may also require admission for observation and treatment. tions?
a. Cardiac ischemia
b. Pneumothorax
1. Orthopnea is best defined by which of the following? c. Pulmonary embolism
a. A respiratory rate greater than normal d. Pneumonia
b. A tidal volume that exceeds metabolic demands e. Bronchospasm
c. Decreased end-tidal carbon dioxide levels Answer: e. Bronchospasm. The classic “shark fin” indicates broncho-
d. Dyspnea in a recumbent position spasm such as with significant asthma exacerbation. See Fig. 21.1 for
Answer: d. Dyspnea in a recumbent position, commonly described as assessment paradigm considerations.
the number of pillows the patient uses to rest comfortably in bed (e.g.,
two-pillow orthopnea).
2. Stridor is most likely due to which of the following conditions?
a. Bronchospasm
b. Guillain-Barré syndrome
c. Laryngeal edema
d. Malignancy
Answer: c. Stridor is an upper airway noise caused by airway narrow-
ing. Of the given options, stridor is most likely due to laryngeal edema.
22 Chest Pain
KEY CONCEPTS “Sharp” or “stabbing” pain is seen more in pulmonary and musculo-
skeletal diagnoses. Patients complaining of a burning- or indigestion-
• T ension pneumothorax is a clinical diagnosis, treated with needle decom- type of pain may initially be suspected of having a gastrointestinal
pression, followed by tube thoracostomy. cause; however, because of the visceral nature of chest pain, all causes
• Patients with suspected acute coronary syndrome (ACS) are risk-stratified of pain may present with any of the preceding descriptions.
by history, electrocardiogram (ECG), and troponin levels. Those with ST seg- 4. Uremia is a risk factor associated with which of the following causes
ment elevation myocardial infarction (STEMI) undergo revascularization via of chest pain?
fibrinolysis or percutaneous coronary intervention (PCI). Those with non–ST a. Acute coronary syndrome
segment elevation myocardial infarction (NSTEMI) do not require immedi- b. Aortic dissection
ate PCI. Those with a nondiagnostic ECG and troponin level are managed c. Pericarditis
with observation. d. Pneumothorax
• Thoracic dissection is diagnosed with CT angiography. The initial manage- e. Pulmonary embolism
ment of dissection is with urgent control of heart rate, followed by lowering Answer: c. Uremia is a risk factor for the development of pericardi-
of blood pressure. Further management, medical or surgical, depends on tis. Other risk factors associated with the development of pericarditis
the location of the dissection. include infection, autoimmune disease, acute rheumatic fever, recent
• Pulmonary embolism is diagnosed using a combination of history, serum myocardial infarction or cardiac surgery, malignancy, radiation ther-
D-dimer measurement, and imaging, usually CT angiography. Patients with apy to the mediastinum, and prior pericarditis.
a low pretest probability and normal D-dimer level do not have pulmonary 5. A narrow pulse pressure is more closely associated with which of
embolism as the cause of their chest pain presentation. the following diagnoses?
• Patients with pericardial effusions undergo echocardiography to evaluate a. Acute myocardial infarction
for cardiac tamponade. Those with signs of shock may require emergent peri- b. Aortic dissection
cardiocentesis. c. Pericarditis with effusion
1. A 25-year-old man presents with the sudden onset of unilateral d. Pulmonary embolism
chest pain, followed almost immediately by respiratory distress. He Answer: c. A narrow pulse pressure is a pivotal finding in the diagnosis
is noted to have a blood pressure of 75/45 mm Hg, pulse of 130 of pericarditis with associated pericardial effusion. Other characteris-
beats/min, and decreased breath sounds on the right side of his tic, but less specific, potential findings in the patient with pericarditis
chest. What is the most appropriate initial step in the management include hypotension, tachycardia, fever, and jugular venous distention
of this patient? (JVD). The more specific finding of pericardial rub is also heard in
a. Administer intravenous (IV) antibiotics. some patients with pericarditis.
b. Infuse a 2-L bolus of normal saline. 6. The finding of Hamman’s sign is most consistent with which of the
c. Obtain a chest radiograph. following?
d. Perform a tube thoracostomy. a. Cholecystitis
Answer: d. Tension pneumothorax is a critical diagnosis that must be b. Mediastinitis
made and remedied, if present, in the first few moments of the rapid c. Pericarditis
stabilization and assessment phase of any patient encounter. If a patient d. Pulmonary embolus
presents with chest pain, respiratory distress, shock, and unilateral Answer: b. Hamman’s sign is an audible systolic “crunch” heard on car-
reduction or absence of breath sounds, immediate intervention with diac auscultation that is produced by the heart moving against air in
needle or tube thoracostomy is required. the mediastinum. This can be heard in conditions such as esophageal
2. A 65-year-old man with a past medical history of prostate cancer rupture, mediastinitis, and pneumomediastinum.
presents with chest pain. His blood pressure is 60/40 mm Hg, and 7. Right ventricular strain on the ECG of a patient complaining of
his pulse is 145 beats/min. The ECG shows diffuse ST segment ele- chest pain would be most consistent with which of the following
vation, and cardiomegaly is seen on his chest radiograph. What is diagnoses?
the most appropriate first step in the management of this patient? a. Acute myocardial infarction
a. Administration of dobutamine b. Coronary ischemia
b. Thrombolysis c. Coronary spasm
c. Cardiac catheterization d. Pulmonary embolus
d. Cardiac ultrasonography Answer: d. In the setting of chest pain, right ventricular strain as evi-
Answer: d. Prompt bedside cardiac ultrasonography would be the most denced on the ECG is highly suspicious for pulmonary embolus.
appropriate next step in the management of this patient, who presents 8. The ECG finding of PR segment depression would be more com-
with symptoms and signs of pericardial effusion with tamponade. If monly found in which of the following causes of chest pain?
a. Pericarditis
confirmed by ultrasonography, immediate pericardiocentesis would
b. Pulmonary embolus
logically follow in an effort to reverse the signs of shock in this patient.
c. ST segment elevation myocardial infarction (STEMI)
Other signs that could accompany the presentation are low voltage on
d. Subendocardial infarction
the ECG and elevated jugular venous pressure on examination.
e. Unstable angina
3. A “tearing” sensation is classically described for which of the fol-
Answer: a. The electrocardiographic findings most commonly associ-
lowing causes of chest pain?
ated with pericarditis are diffuse ST segment elevation and PR segment
a. Aortic dissection
depression. The ECG in patients with unstable angina is most often
b. Esophageal rupture
normal or nonspecific. T wave inversion may be seen in these patients.
c. Mallory-Weiss tear
The characteristic ECG findings with subendocardial infarction are
d. Pneumothorax
T wave inversion and/or ST segment depression in concordant leads.
Answer: a. “Tearing” pain that may migrate from the front to back or Classic STEMI is manifested electrocardiographically by ST segment
back to front is classically described in aortic dissection. Descriptions elevation (>1 mm) in contiguous leads, a new left bundle branch block
such as “squeezing,” “crushing,” or “pressure” lead to the suspicion of a (LBBB), or Q waves 0.04 second or more in duration. Many possible
cardiac ischemic syndrome, although cardiac ischemia can be charac- electrocardiographic findings are associated with pulmonary embolus,
terized by nonspecific discomfort, such as “bloating” or “indigestion.”
usually manifestations of right ventricular strain.
Abdominal Pain 23
KEY CONCEPTS
• W hile the etiology of abdominal pain is frequently benign, requiring little
work-up or intervention, abdominal pain can also be the presenting symp-
tom of catastrophic illness, requiring life-saving interventions to be imple-
mented within minutes to hours (see Table 23.1).
• Patients who are pregnant, of advanced age, or immunocompromised are
at higher risk for acute pathology and require expeditious evaluation, which
generally includes imaging studies.
• Patients with prior surgery, especially bariatric surgery, have unique ana-
tomic features that put them at risk for pathology not seen in patients that
have never had abdominal surgery.
• Patients with hemodynamic instability require resuscitation in conjunction
with diagnostic evaluation, and often surgical (or gynecologic) consultation.
Bedside ultrasonography can be helpful in these patients.
• Pain medication while evaluating stable patients is appropriate and gener-
ally should not be withheld. We recommend the administration of IV mor-
phine at a dose of 0.05 to 0.10 mg/kg (usually 2 to 5 mg in adults) every 15
to 20 minutes until pain is controlled (pain score <4).
• Intra-abdominal infections are often polymicrobial and require broad spec-
trum antibiotic coverage with regimens such as: piperacillin/tazobactam
3.375 g IV, ciprofloxacin 400 mg IV + metronidazole 500 mg IV, or cefepime
2 g IV + metronidazole 500 mg IV.
• Stable patients with unrevealing evaluations whose symptoms improve
may be considered for disposition to home with close follow-up. High-risk
patients without a clear diagnosis may require in-hospital observation.
1. A 65-year-old man presents with acute abdominal pain. He has a most appropriate imaging study to confirm the suspected diag-
history of hypertension, diabetes, and an abdominal aortic aneu- nosis?
rysm. On examination, he is hypotensive and tachycardic with a. Computed tomography angiography
a distended, rigid abdomen. Which of the following is the best b. Magnetic resonance imaging
method for diagnosis? c. Plain radiograph
a. Computed tomography d. Right upper quadrant ultrasound
b. Exploratory laparotomy Answer: a. Computed tomography angiography. The patient presents
c. Plain radiograph with “pain out of proportion” to examination, multiple vascular risk fac-
d. Right upper quadrant ultrasound tors, and atrial fibrillation, which increases his risk for mesenteric isch-
Answer: b. Exploratory laparotomy. Patients with known abdominal emia. Computed tomography angiography would be the best screening
aortic aneurysm (AAA) and unstable vital signs must be assumed to test. Magnetic resonance imaging would be too time-consuming, plain
have ruptured AAA and should be brought to the operating room radiographs have limited utility, and right upper quadrant ultrasound
without delay. If the patient were stable, CT would be indicated. Plain is not warranted in this patient with diffuse, non-localized symptoms.
radiograph is not indicated in this patient, nor would right upper quad- 4. A 30-year-old man presents with abdominal pain and fever. Which
rant ultrasound be warranted. of the following would place him at higher risk for life-threatening
2. A 25-year-old woman presents with abdominal pain. She has sta- pathology?
ble vitals. On pelvic examination, she has tenderness in the right a. Drinking from a mountain spring
adnexal region. Which of the following is the most appropriate ini- b. Prednisone therapy for systemic juvenile idiopathic arthritis
tial imaging study? c. Recent cruise with norovirus outbreak
a. Computed tomography d. Use of metoprolol for essential hypertension
b. Magnetic resonance imaging Answer: b. Prednisone therapy for systemic juvenile idiopathic arthri-
c. Plain radiograph tis. Low-dose steroid therapy places patients at higher risk when
d. Pelvic ultrasound they present with abdominal pain. Drinking from a mountain spring
Answer: d. Pelvic ultrasound. Women of child-bearing age with lower increases risk for giardia. Neither this nor exposure to norovirus place
abdominal pain and adnexal tenderness on examination are best the patient at increased risk for life-threatening pathology. Hyperten-
imaged with pelvic ultrasound. CT may be warranted if the ultrasound sion increases risk for vascular conditions such as abdominal aortic
is inconclusive or suggestive of, but not diagnostic of, appendicitis. aneurysm, but this would be unlikely in a young patient controlled on
Plain radiographs are not helpful in undifferentiated lower abdominal metoprolol.
pain. MRI could be a consideration if the patient were pregnant and 5. What is a characteristic of visceral abdominal pain?
there was concern for appendicitis. a. Easily described
3. A 70-year-old man presents with abdominal pain associated b. Midline
with one loose bowel movement and an episode of non-bloody c. Sharp in nature
emesis. He has a history of hypertension, diabetes, cigarette d. Well-localized
smoking, and atrial fibrillation. He rates his pain as “10/10.” On Answer: b. Midline. Visceral pain is classically midline, poorly local-
examination, he has stable vitals. His abdomen is soft and non- ized, and less easily described than somatic pain, which is often referred
tender without signs of peritonitis. Which of the following is the to as “sharp.”
Jaundice 24
KEY CONCEPTS
• J aundice is the clinical manifestation of elevated serum bilirubin, which
arises through the metabolism of hemoglobin. Elevated bilirubin occurs
when: (1) increased bilirubin is produced due to hemolysis, (2) liver dysfunc-
tion prevents conjugation of bilirubin, (3) an obstruction prevents secretion
of bilirubin into the intestines.
• Jaundice is usually not evident on physical examination until the total
serum bilirubin concentration rises above 2.5 mg/dL. Jaundice is first
apparent in the conjunctiva, sublingually, or on the hard palate.
• Unconjugated bilirubin that is not bound to albumin can cross the blood-
brain barrier, causing adverse neurologic effects; conjugated bilirubin is not
neurotoxic.
• New-onset painless jaundice is the classic presentation for a neoplasm
involving the head of the pancreas.
• In cases of unexplained hepatocellular injury, a quantitative acetamino-
phen level may be helpful.
• If the etiology of ascites is unknown, serum ascites albumin gradient
(SAAG) determination will aid in distinguishing the cause as well as the
presence of portal hypertension.
• Ultrasonography is the preferred initial imaging modality in evaluating for
biliary obstruction, whereas CT is preferred if malignant obstruction is sus-
pected or the full abdomen needs to be evaluated.
• Elevated direct bilirubin with transaminase elevation is indicative of hepa-
tocellular inflammation or injury.
• Diagnosis of spontaneous bacterial peritonitis (SBP) is made by a peritoneal
fluid neutrophil count of greater than 250. Treatment typically consists of
cefotaxime 2 g IV and albumin 1.5 g/kg.
1. A 56-year-old male presents with fever and abdominal distention. c. Ceftriaxone alone
Bedside ultrasound reveals ascites and the results of paracentesis d. Ciprofloxacin and metronidazole
indicate possible spontaneous bacterial peritonitis (SBP). What Answer: a. The empirical antibiotic of choice is a third-generation ceph-
daily medication should be stopped upon admission? alosporin (e.g., cefotaxime). Albumin administration has been shown
a. Amlodipine to reduce risk for mortality and hepatorenal syndrome. If the patient
b. Crestor has a history of cirrhosis and is taking a nonselective beta blocker, it
c. Lactulose should be discontinued because it has been shown to increase mortal-
d. Nadolol ity in patients with SBP.
Answer: d. Beta blocker use in patients with cirrhosis with SBP should 4. A 41-year-old male with a history of cirrhosis presents with fever,
be discontinued as they have been shown to increase mortality. abdominal distension, and confusion. A paracentesis is performed
2. A 43-year-old female presents with 1 month of painless abdominal in the evaluation of spontaneous bacterial peritonitis (SBP). What
swelling. On examination she has a diffusely swollen abdomen with are the diagnostic criteria found in the ascitic fluid that confirms
a fluid wave. To help determine the etiology, you obtain the serum SBP?
ascites albumin gradient (SAAG). Which value is consistent with a. Ascitic fluid neutrophil count >100
cirrhosis? b. Ascitic fluid neutrophil count >250
a. 0.25 g/dL c. Ascitic fluid total WBC count >100
b. 0.5 g/dL d. Ascitic fluid total WBC >250
c. 1 g/dL Answer: b. The presence of more than 250 polymorphonuclear cells
d. 1.5 g/dL per cubic millimeter of ascitic fluid is diagnostic for SBP.
Answer: d. A (SAAG) value of greater than or equal to 1.1 g/dL is 5. A 55-year-old female presents with 1 month of diffuse abdominal
found in patients with portal hypertension. There are many causes of swelling and pain. She reports a long history of alcohol use. In eval-
portal hypertension, including cirrhosis, liver failure, or heart failure. uating this patient for jaundice, at what bilirubin level does jaundice
A SAAG value of less than 1.1 g/dL may be found in patients with con- typically become clinically apparent?
ditions such as lupus or pancreatitis. a. 1 mg/dL
3. A 48-year-old male with a history of cirrhosis presents with 3 days b. 2 mg/dL
of abdominal pain and fever. Abdominal paracentesis suggests c. 2.5 mg/dL
spontaneous bacterial peritonitis (SBP). Which of the following ini- d. 4 mg/dL
tial treatments is most appropriate? Answer: d. Clinical jaundice is usually not evident until the total
a. Cefotaxime, albumin, and discontinue beta blockade serum bilirubin concentration rises above 2.5 mg/dL. Jaundice is first
b. Ceftriaxone and dexamethasone apparent sublingually, in the conjunctiva or on the hard palate.
Nausea and Vomiting 25
KEY CONCEPTS
• N ausea and vomiting can result from a primary problem in the gastroin-
testinal (GI) tract but can also result from problems in the neurological,
vestibular, urogenital, cardiac, or other systems.
• In the acutely vomiting patient, associated symptoms and medication his-
tory are most helpful in narrowing the differential diagnosis.
• Laboratory studies are not necessary for all patients with vomiting. In
patients with severe or protracted vomiting, consider checking electro-
lytes and renal function, particularly when intravenous (IV) rehydration is
initiated.
• In patients with undifferentiated nausea or vomiting or vomiting due to
non-obstructive GI disease, ondansetron is the first-line antiemetic due to
its low side effect profile. However, no definitive evidence exists for the
superiority of one antiemetic agent over others.
• Although evidence is limited, metoclopramide is the antiemetic of choice
in hyperemesis gravidarum and vomiting associated with headache, and
ondansetron is the drug of choice in chemotherapy-induced vomiting.
• Ondansetron is the first-line agent for children with vomiting due to acute
gastroenteritis.
• Consider cannabinoid hyperemesis syndrome (CHS) in all patients with pro-
tracted vomiting. Emergency department (ED) treatment of CHS includes
the use of capsaicin cream as well as haloperidol or lorazepam.
1. A 50-year-old male patient presents with a complaint of vomiting. 4. A 28-year-old female presents to the emergency department with
He states the pancakes he ate 12 hours ago for breakfast came out severe vomiting. She is 7 weeks pregnant. She has experienced nau-
undigested. He admits to several similar episodes in preceding sea during this pregnancy which she had controlled with oral ginger
weeks. He is otherwise asymptomatic and has normal vital signs. and a pyridoxine/doxylamine combination pill, but for the last 3
His physical examination is normal. Which of the following condi- days she has been unable to tolerate anything by mouth. Her elec-
tions is most likely? trolytes and renal function are normal. Bedside ultrasound reveals
a. Constipation a single live intrauterine pregnancy. Her urinalysis reveals large
b. Gastric outlet obstruction ketones. In addition to intravenous fluids, which of the following
c. Hepatitis medications should be selected as the first line intravenous anti-
d. Increased intracranial pressure emetic agent?
Answer: b. Vomiting intact food eaten over 12 hours earlier is consid- a. Dexamethasone
ered pathognomonic for gastric outlet obstruction. None of the other b. Haloperidol
choices have this as a feature. c. Metoclopramide
2. Antihistamines would most effectively control the nausea and vom- d. Ondansetron
iting caused by which of the following conditions? Answer: c. Metoclopramide is a first-line agent due to its safety for
a. Chemotherapy administration b. Gastritis the developing fetus. Ondansetron should be reserved for second line
c. Gastroparesis as there is some suggestion of the capacity for fetal harm. Olanzapine
d. Labyrinthitis is indicated for chemotherapy-associated vomiting and haloperidol
Answer: d. Antihistamines are particularly useful in nausea and vom- is best used for gastroparesis or cannabinoid hyperemesis syndrome.
iting associated with labyrinthitis, motion sickness, and vestibular Dexamethasone is reserved for patients who fail first- and second-line
disorders by directly inhibiting vestibular stimulation and vestibular- therapies.
cerebellar pathways. The anticholinergic effect may also contribute to 5. What is the most common cause of nausea and vomiting in the
their effectiveness in vertigo and motion sickness. pediatric population?
3. Which of the following is the most effective for the treatment of a. Acute gastroenteritis
cannabinoid hyperemesis syndrome? b. Drug side effects
a. Capsaicin cream c. Intussusception
b. Cessation of marijuana use d. Motion sickness
c. Haloperidol
d. Lorazepam Answer: a. In pediatrics, acute gastroenteritis is the most common
Answer: b. Cessation of marijuana use has been demonstrated to cause of vomiting. Studies on management of vomiting in this popula-
decrease syndrome recurrence. The other choices are frequently uti- tion focus on children with this condition.
lized in the acute treatment of cannabinoid hyperemesis syndrome,
and have only been described in case reports or series.
26 Gastrointestinal Bleeding
KEY CONCEPTS
• R outine placement of a nasogastric tube in patients with suspected upper
gastrointestinal bleeding (UGIB) is not recommended, because this proce-
dure fails to reliably provide useful data to guide management and is asso-
ciated with unnecessary patient discomfort and potential complications.
• Patients older than 35 years with UGIB should have an electrocardiogram
(ECG) performed early in their evaluation, because patients with UGIB and
coronary disease (including occult coronary disease) may develop an acute
coronary syndrome that presents with atypical symptoms, such as nausea
and abdominal discomfort.
• A blood urea nitrogen (BUN) to creatinine (Cr) ratio of greater than 35 is
90% specific for UGIB. However, the BUN:Cr ratio is poorly sensitive and
cannot be used to rule out UGIB.
• Patients with exsanguinating hemorrhage should receive an emergent
transfusion of untyped packed red blood cells. For stable patients without
known coronary artery disease, transfusion should be initiated when the
hemoglobin level is less than 8 g/dL.
• Standard pharmacologic therapy for UGIB includes intravenous (IV) proton
pump inhibitors and prokinetic agents. Patients with cirrhosis and sus-
pected esophageal varices should receive octreotide and antibiotics.
• Based on available evidence, we do not recommend the routine use of
tranexamic acid for patients with UGIB.
• Platelet transfusion is not indicated for patients on antiplatelet medications
unless the platelet count is <50,000 per μL.
• Empirical administration of fresh frozen plasma (FFP) or prothrombin com-
plex concentrate (PCC) to patients with cirrhosis or anticoagulant use is not
indicated.
• Stable patients with lower gastrointestinal bleeding (LGIB) who can toler-
ate bowel preparation are best managed through urgent gastroenterology
consultation for colonoscopy, while unstable patients should be seen by an
interventional radiologist for possible selective embolization.
1. What is the most common cause of upper gastrointestinal (GI) c. All patients with GI bleeding on antiplatelet medications such as
hemorrhage in adults in the United States? aspirin should receive an empiric platelet transfusion
a. Esophagitis d. Antiplatelet medications do not increase the risk of acute GI
b. Gastritis bleeding
c. Liver cirrhosis Answer: a. Despite a high risk of GI bleeding while taking antiplate-
d. Peptic ulcer disease let medications, the routine use of platelet transfusion for patients on
Answer: d. Despite a decreasing prevalence with the treatment of Heli- antiplatelet medications has not been shown to be beneficial. Platelet
cobacter pylori, peptic ulcer disease (PUD) remains the most common transfusion should be reserved for patients who are thrombocytopenic
cause of upper GI bleeds in adults in the United States. <50,000 per μL or those requiring massive transfusion protocol.
2. Management of unstable patients with massive upper gastrointesti- 4. Elevations in which of the following serum markers in the setting
nal bleeding (UGIB) should include which of the following? of an acute gastrointestinal bleed is predictive of increased 30-day
a. Placement of at least two large-bore IVs. mortality?
b. Avoidance of rapid crystalloid infusion in effort to decrease a. C-reactive protein
rebleeding rates b. Lactate
c. Empiric treatment with fresh frozen plasma (FFP) c. Serum osmolality
d. Routine intubation for airway protection d. White blood cell count
Answer: a. Initial resuscitation of unstable patients with massive GI Answer: b. An initial lactate greater than 2.5 mmol/L is associated with
bleeding should include placement of two large bore IV catheters and the development of in-hospital hypotension and an increased rate of
rapid crystalloid infusion with consideration of early blood transfu- 30-day mortality.
sion. Intubation should be reserved for patients with altered mental 5. Appropriate pharmacologic therapy in the emergency department
status, respiratory distress, or high concern for aspiration. FFP should for most acute upper gastrointestinal bleeds (UGIB) includes which
be given empirically only for patients receiving a massive transfusion of the following agents?
protocol or in select patients with coagulopathy. a. Antibiotics
3. Which of the following statements is true regarding platelet admin- b. Erythromycin
istration for patients with acute gastrointestinal (GI) bleeding? c. Proton pump inhibitor (PPI)
a. Patients requiring massive transfusion should receive a bal- d. Somatostatin
anced transfusion approach of 1:1:1 packed red blood cells Answer: c. PPIs decrease rebleeding, surgery, and mortality in patients
(PRBCs):platelets:FFP. with an acute UGIB from an ulcer. Because peptic ulcer disease is the
b. Platelets should be administered to a target level of >100,000 most common cause of UGIB, it is appropriate to administer PPIs for
per μL most acute UGIB.
Diarrhea 27
Answer: d. Patient in answer D likely has a viral gastroenteritis, and C.
KEY CONCEPTS difficile testing is more likely to result in false positive result or identify
• Hospital-acquired Clostridioides difficile and norovirus infection are the chronic colonization. Risk factors for C. difficile colitis include anti-
most prevalent causes of fatal illness from diarrhea in the United States. biotic or gastric acid suppressive therapy, skilled nursing facility resi-
• Key elements of the history in the patient with diarrhea include recent dence, and chronic medical conditions.
travel, hospitalization or antibiotic use, dietary factors, immunosuppres- 3. Evaluation of stool for ova and parasites would prove least bene-
sion, fever, and presence of blood in stool. ficial in which of the following subsets of patients presenting with
• Acute diarrhea is most often viral and treated with supportive therapy. diarrhea?
• Fluids and foods with excess sugar, caffeine, or high fat content should be a. Patients concurrently taking clindamycin with diarrhea for 1 week
avoided in patients with diarrhea. b. Patients recently returning from a backpacking trip
• In selected nontoxic patients, loperamide, initial dose of 4 mg by mouth and c. Patients returning from a trip to Nepal
subsequent doses of 2 mg by mouth after each loose stool (not to exceed 16 d. Patients with chronic diarrhea
mg in 24 hours), is typically safe and effective in providing symptom control. e. Patients with human immunodeficiency virus (HIV) infection
• Empiric antibiotics are uncommonly indicated in patients well enough to go Answer: a. The assessment of stool for ova and parasites is not rou-
home. However, they should be considered in patients with fever or bloody tinely recommended in most cases of diarrheal illness. This study is
diarrhea who are immunocompromised or in travelers diarrhea. Empiric used in patients with chronic diarrhea (Entamoeba histolytica and
antibiotics are prudent whenever sepsis is suspected. Cryptosporidium); patients with a history of travel to developing
• Fluoroquinolones (ciprofloxacin 500 mg by mouth twice daily or levofloxacin countries, particularly to Nepal or areas of Russia (Cryptosporidium,
500 mg by mouth daily for 3 to 5 days) or azithromycin (500 mg by mouth Giardia, and Cyclospora); patients with exposure to infants in day-
daily for 3 days) cover the majority of enteric pathogens if empiric antibiot- care centers (Cryptosporidium and Giardia); and patients with HIV
ics are required. infection (E. histolytica and Giardia). Acute diarrhea can last up to 2
• A single dose of azithromycin, 1000 mg by mouth, is sufficient antibiotic weeks.
treatment for travelers diarrhea. 4. In which of the following patients is loperamide a safe treatment
• First line treatment for C. difficile is vancomycin 125 mg by mouth four option?
times daily for 10 days. Second line treatment is fidaxomicin 200 mg by a. Adult patient with acute watery diarrhea of unknown etiology
mouth for 10 days. b. Adult patient with sepsis and bloody diarrhea
• The efficacy of probiotics in reducing stool frequency in acute infectious c. Elderly patient with a history of bowel obstruction with watery
diarrhea is unclear. diarrhea
d. Pediatric patient with acute watery diarrhea from norovirus
e. Pediatric patient with travelers diarrhea
1. Which of the following is not a common cause of diarrhea in a Answer: a. Loperamide, a bowel-selective opioid agonist, is an effective
patient with human immunodeficiency virus (HIV) disease? tool to reduce diarrhea severity. Although generally safe, it has been
a. Antiretroviral therapy associated with toxic megacolon and obstruction in select groups, such
b. Cytomegalovirus as the elderly, pediatric patients (especially younger than age 3), and
c. Giardia patients with a severe inflammatory colitis. Adult patients in which the
d. Intestinal lymphocyte proliferation in acute HIV infection risk of obstruction or severe inflammatory colitis is low are candidates
e. Shigella for loperamide treatment.
Answer: d. Acute HIV infection is associated with depletion (not pro- 5. In which of the following patients is empiric antibiotic treatment
liferation) of intestinal lymphocytes from intestinal mucosa, which the most reasonable option?
alters the gut microbiome leading to increased incidence of viral, bac- a. Adult patient returning from Mexico with watery diarrhea.
terial, and protozoal causes of diarrhea. Gastrointestinal cytomega- b. Adult patient with bloody diarrhea during outbreak of Esche-
lovirus is associated with advanced HIV infection (CD4+ counts less richia coli O157:H7
than 50), which can lead to severe hemorrhagic colitis. In addition, c. Pediatric patient returning from Thailand with bloody diarrhea.
all classes of antiretroviral drugs may cause diarrhea, which can affect d. Pediatric patient with fever and bloody diarrhea.
more than 25% of individuals taking combined antiretroviral therapy. e. Well-appearing adult patient with suspected norovirus based on
2. In which of the following patients would Clostridioides difficile test- ill contacts.
ing be least useful? Answer: a. Travelers diarrhea generally responds well to a single
a. Patient with diarrhea who is taking omeprazole for known pep- dose of azithromycin 1000 mg PO which can decrease time to reso-
tic ulcer disease. lution of symptoms with limited side effects. The choice to initiate
b. Patient with diarrhea after recent tonsillitis treated with amoxi- antibiotics must be tailored to weigh potential benefits and harms
cillin. of the treatment for the individual patient. Empiric antibiotics are
c. Patient with diarrhea who is a long-term resident of a skilled generally contraindicated in children, as well as patients in which
nursing facility. E. coli O157:H7 is a possibility, as antibiotics increase the risk of
d. Patient with acute vomiting and watery diarrhea following con- hemolytic uremic syndrome. In cases of suspected viral-mediated
tact with a toddler with similar symptoms. diarrhea, such as a clustered norovirus outbreak, antibiotics are
e. Patient with uncontrolled HIV and acute watery diarrhea. generally not indicated.
28 Constipation
KEY CONCEPTS
• C onstipation is a variable term used by patients and may refer to straining
with defecation, hard or infrequent stools, pain with a bowel movement, a
sensation of incomplete evacuation, or abdominal bloating.
• Primary constipation is caused by functional abnormalities of the gastroin-
testinal tract.
• Secondary constipation is due to diet, related medical or psychiatric disor-
ders, or medication side effects.
• The etiology of constipation can frequently be discovered through a thor-
ough history.
• The physical examination of patients with constipation focuses on the
patient’s appearance, the presence of pain, and structural abnormalities
identified on abdominal or rectal examination.
• Lab work to evaluate constipation is typically not indicated.
• Imaging to evaluate constipation is not typically indicated when the physi-
cal examination is normal. tenderness. On rectal exam, you feel a large amount of firm stool
• The treatment of constipation for most patients is empiric and focuses on in the rectal vault. What is the appropriate next step in this patient’s
lifestyle modifications in addition to laxatives. management?
• Patients with constipation should be referred for outpatient evaluation and a. Admission for colonoscopy
testing when they fail empiric treatment. b. Analgesia and disimpaction
• Opioid-induced constipation is common and may require peripherally acting c. Glycerin suppository
μ-opioid receptor antagonists for management. d. Oral milk of magnesia
Answer: b. This is an elderly male with chronic constipation for a mul-
1. A 48-year-old female with no known past medical history presents titude of reasons. Because of his age and possibly due to his cerebrovas-
to the emergency department with the chief complaint of consti- cular accident, he has denervation of his rectum preventing him from
pation. She states she has not had a bowel movement in the past sensing rectal fullness. The large volume of stool in his rectum needs
7 days. She has no history of similar symptoms. She denies taking to be removed for his comfort, and disimpaction with analgesia will be
any medications or any past surgical history. On examination, she necessary. The use of a glycerin suppository to lubricate the stool to be
has no abdominal tenderness with palpation or masses. Her rectal expelled is unlikely to be successful as this patient has lost the ability to
examination is unremarkable for fissures, and the patient does not expel stool on his own. Because this patient does not receive medica-
have any blood or stool in her rectal vault. What is the next step in tions by mouth, milk of magnesia should not be given orally. Because
this patient’s management? his abdominal examination is benign and he is hemodynamically sta-
a. A plain film abdominal x-ray ble, the likelihood that he has a critical intra-abdominal process requir-
b. Discharge home with recommendation for laxatives and plan for ing imaging is low. Finally, this patient does not require admission to
lifestyle modification the hospital, nor does he require an emergent colonoscopy at this time.
c. Emergent outpatient gastroenterology follow-up 3. A 32-year-old female with chronic pain followed closely by a pain
d. Trial of at home docusate and strict return precautions if plan management specialist presents with constipation. She normally
fails takes a bowel regimen of bisacodyl daily in addition to consuming a
Answer: b. The patient in this question has acute constipation. Because high-fiber diet. Her physician recently increased her opioid medica-
her history and examination does not have any red flag concerns, this tion dosage. Since then, she has had difficulty having more than one
patient can be safely discharged home without any testing. This patient small, hard stool per week. She complains of abdominal discomfort
will require laxatives to help her acute constipation, but lifestyle man- but has not had any nausea or vomiting. On examination, her abdo-
agement should help to prevent similar episodes in the future. There is men is soft without focal tenderness. Her rectal examination is nor-
no utility for plain film abdominal x-rays in the emergency department mal. What is the appropriate next step in this patient’s management?
for the evaluation of constipation. Docusate has been shown to be inef- a. A trial of a peripherally acting μ-opioid receptor antagonist and
fective as first line management for constipation unless the patient has plan for close outpatient follow up with her care team
a rectal abnormality or rectal pain that prevents the passage of stool. b. Consultation with her pain management specialist to decrease
This patient is well-appearing on examination and therefore, she does her medication dosage back to previous levels
not require intravenous (IV) hydration. Because this patient’s consti- c. Lab testing to rule out an electrolyte abnormality causing this
pation is acute and she has a benign examination, she does not require patient’s symptoms
emergent gastroenterology follow-up. d. Nonopioid pain medication for her discomfort while in the
2. A 96-year-old male from a nursing home presents with the chief emergency department and plan for gastroenterology follow-up
complaint of constipation. He has a past medical history significant Answer: a. This is a patient experiencing opioid-induced constipa-
for cerebrovascular accident and now receives all of his medica- tion. Her previous bowel regimen was sufficient until her medication
tions, hydration, and feedings through a gastrostomy tube due to doses increased. Because this patient has failed her bowel regimen, it
dysphagia. The patient has been on a bowel regimen of twice daily is reasonable to try a peripherally acting μ-opioid receptor antagonist
polyethylene glycol through his gastrostomy tube with minimal, to block the effects of the opioids on the colonic μ receptor. Decreas-
watery stool output over the past week. The patient appears to be ing this patient’s pain medications back to previous doses may lead to
uncomfortable, but he is afebrile and hemodynamically stable. On withdrawal. Lab testing is unlikely to be of benefit. Providing analgesia
examination, the patient’s abdomen is soft and there is no focal for discomfort will not relieve constipation.
4. Which of the following is true regarding constipation? 5. Which of the following is true regarding treatment options for con-
a. The diagnosis of constipation requires lab testing and imaging in stipation?
the emergency department. a. Bisacodyl works by increasing water penetration and thereby
b. Most patients with constipation and a normal examination can softens the stool.
be safely discharged home with empiric treatment. b. Docusate sodium should be prescribed to all patients with
c. Constipation is less common in the elderly due to low fiber, chronic constipation given its effectiveness as a laxative.
mechanical soft diets. c. Lifestyle modification is the first line treatment for patients with
d. Chronic constipation occurs when a patient experiences acute constipation.
decreased stool output for at least 1 year. d. Sodium phosphate and magnesium hydroxide should be avoided
Answer: b. The term constipation refers to a multitude of symptoms, in patients with renal impairment.
not a medical diagnosis. Patients often have different personal defini- Answer: d. The side effect profiles for many common treatments of
tions for the term “constipation.” Chronic constipation refers constipa- constipation should be considered prior to prescribing to special
tion symptoms for 3 or more months. Constipation is more common in patient populations. Both sodium phosphate and magnesium hydrox-
the elderly due to sedentary lifestyles and low-fiber, mechanically soft ide can increase the levels of phosphorus and magnesium in the body,
diets. In the ED, lab testing and imaging is often unnecessary for patients respectively, in patients with renal insufficiency/impairment. Bisaco-
with a normal physical exam. Most well-appearing patients with consti- dyl is a laxative and works by stimulating intestinal motility. Docusate
pation and a normal examination can be discharged home with empiric sodium is a stool softener, not a laxative, that should not be used as
management. first line or solo therapy due to its marginal efficacy. Patients with acute
constipation should be prescribed both a laxative for immediate treat-
ment and lifestyle modification for longer-term management.
Acute Pelvic Pain 29
KEY CONCEPTS
• A cute pelvic pain in women is often from a gynecologic source, but urinary
Answer: a. This follows the algorithms in Figs. 29.1 and 29.2. The most
and intra-abdominal sources are also common. Less frequently, the pain may
life-threatening pathology requiring urgent or emergent intervention
arise from vascular, musculoskeletal, neurologic, or psychiatric disorders.
is hemorrhage from a ruptured ectopic pregnancy. Pelvic ultrasound
• Potentially lethal diagnoses associated with acute pelvic pain include
scan is rapid and is the first step in an evaluation of a suspected rup-
ectopic pregnancy, ovarian cyst with significant hemorrhage, and domestic
tured ectopic pregnancy. In this case, bedside ultrasonography, if avail-
violence; highly morbid conditions presenting with acute pelvic pain include
able, is strongly recommended.
pelvic inflammatory disease and ovarian torsion.
3. A 30-year-old female presents with lower abdominal pain. She
• Nearly all women of childbearing age with pelvic pain should have a preg-
is not ill appearing and has lower abdominal, uterine, bilateral
nancy test performed, and most should have a pelvic ultrasound examination.
adnexal, and cervical motion tenderness on pelvic examination.
• Ectopic pregnancy should be excluded in the pregnant patient with pelvic
She has a negative urine pregnancy test and urinalysis. What is the
pain. Bedside ultrasound is an excellent test for confirming an intrauterine
most appropriate next step in the patient’s management?
a. Await cervical culture results.
pregnancy (IUP); it excludes ectopic pregnancy with a high degree of cer-
b. Obtain a complete blood count.
tainty in patients who are not using assisted reproductive technology.
c. Obtain a CT scan.
• Pregnant patients with acute pelvic pain may also have non–pregnancy-
d. Obtain an erythrocyte sedimentation rate (ESR).
related disorders; appendicitis, nephrolithiasis, and ovarian torsion, among
e. Treat with antibiotics.
others, remain in the differential diagnosis.
Answer: e. The constellation of uterine tenderness, bilateral adnexal
• Many patients with acute pelvic pain require imaging as part of their
tenderness, and cervical motion tenderness is classically associated
assessment. If a gynecologic source is suspected, begin with an ultrasound
with pelvic inflammatory disease (PID), particularly when the pain
and then progress to computed tomography (CT) or magnetic resonance
onset is during or just after menstruation. However, the diagnosis may
imaging (MRI), if needed. The presence of an ovarian cyst on imaging does
be made without the presence of all three signs, and treatment may be
not necessarily explain the patient’s pain, and further evaluation may be
initiated with only one sign in an at-risk patient, as given in the 2015
required. Ultrasound after normal CT is unlikely to be informative. Ovarian
Centers for Disease Control and Prevention (CDC) guidelines. Ultra-
torsion may be radiographically occult.
sound is not strictly necessary in this case but should be performed if
1. Which of the following statements regarding the evaluation of there is any concern for a complication, such as tubo-ovarian abscess.
patients with pelvic pain is true? 4. A 35-year-old female using assisted reproductive technology pres-
a. Ancillary testing can be limited to a urine pregnancy test in most ents with severe lower left quadrant (LLQ) abdominal pain and ten-
patients. derness isolated to the left adnexal area on pelvic examination. The
b. Bimanual examinations have been shown to result in highly reli- urine pregnancy test is positive, and the urinalysis is unremarkable.
able findings, with substantial interobserver agreement. Rapid bedside ultrasonography reveals an intrauterine pregnancy
c. Patients typically localize visceral pain with a high degree of accuracy. with a gestational age of 6 weeks, 5 days, and moderate free pelvic
d. Thorough history taking is adequate to exclude most life- fluid. Which diagnosis should be expediently further investigated?
threatening conditions. a. Heterotopic pregnancy
e. None of the above. b. Round ligament pain
Answer: e. It is rare that any particular finding on history or physical c. Simple ovarian cyst
examination is reliable enough to make or exclude a particular diag- d. Tubo-ovarian abscess
nosis conclusively in patients presenting with pelvic pain, so ancillary e. None of the above
testing (beyond a simple pregnancy test) is commonly required in the Answer: a. Women who are actively undergoing infertility treatment
evaluation of these patients. Ultrasound is the most useful adjunct are at increased risk for ectopic pregnancy, heterotopic pregnancy,
for most patients. The bimanual examination may, at times, provide ovarian torsion, and ovarian hyperstimulation syndrome.
important information. Unfortunately, however, findings on pelvic 5. A 28-year-old female presents with sudden-onset, severe left
examination are somewhat subjective and unreliable and may serve adnexal pain that started 1 hour ago. The patient has left adnexal
more to localize the process to one side or the other rather than diag- tenderness to palpation, but there is no vaginal discharge or bleed-
nose it or even limit it to the reproductive organs. Although pelvic pain ing, and no cervical motion tenderness. Urinalysis and urine preg-
often originates from the reproductive organs, it may arise from any nancy are negative. Pelvic ultrasound is unremarkable, and you do
structures that lie adjacent to or course through the pelvis. Visceral not suspect intra-abdominal pathology. The patient has required
pain afferents supplying the pelvic organs have common innervation multiple doses of opiates for pain control in the emergency depart-
with the appendix, ureters, and colon. Their significant overlap makes ment (ED). What is the most appropriate next step in management?
accurate localization difficult for both patient and emergency clinician. a. Admit the patient for serial abdominal examinations.
2. A 26-year-old female presents with right lower quadrant (RLQ) b. Consult gynecology.
abdominal pain. She states her last menstrual period was 8 weeks c. Discharge the patient with pain control and clinic follow-up.
ago. Bimanual pelvic examination reveals tenderness in the RLQ d. Discharge the patient with treatment for PID and clinic fol-
and right adnexal area. The patient’s vital signs include a regular low-up.
pulse of 120 beats/min and a blood pressure of 110/65 mm Hg. Uri- e. Obtain a CT scan of the pelvis.
nalysis is unremarkable, and the urine pregnancy test is positive. Answer b. Ovarian torsion should be suspected based on this patient’s
What is the most appropriate next test? history and examination. Because torsion can be a radiographically
a. Pelvic ultrasonography occult diagnosis, gynecology should be consulted as laparoscopy may
b. Cervical cultures be needed for definitive diagnosis. Less emergent conditions, such as
c. Complete blood count (CBC) nephrolithiasis, could also be considered.
d. Computed tomography (CT)
e. Magnetic resonance imaging (MRI)
6. A 26-year-old female who is pregnant at 30-weeks’ gestation pres- 7. A 34-year-old female presents with severe abdominal pain. She
ents with vaginal bleeding and pelvic cramping. She states the does not allow for an abdominal examination and keeps pushing
cramping and bleeding started 2 hours ago and she has filled one your hand away any time you attempt to even mildly palpate her
pad so far. She has not had any prenatal care. Which of the follow- abdomen. Her vital signs are stable. What is the most important and
ing should be initially avoided during this patient’s evaluation? appropriate next step in this patient’s care?
a. Obstetrics consultation a. IV fluids
b. Intravenous (IV) fluids b. IV opiates
c. Pelvic examination c. Oral (PO) opiates
d. Type and Screen d. PO acetaminophen
e. Ultrasound e. Do not give any medications at this time
Answer c. Pregnant patients beyond 20-weeks’ gestation with complaints Answer b. We recommend early administration of analgesia for
of vaginal bleeding should undergo transabdominal pelvic ultrasound patients with significant pain. Analgesia greatly improves patient com-
for placental localization before the pelvic examination. The concern is fort and the reliability of the physical examination, which is otherwise
that placenta previa, which can be worsened by pelvic examination, may hampered by the patient’s extreme pain, tenderness, or both. For severe
be the cause of the bleeding. Obstetrics will need to be involved in this pain, intravenous opioids such as morphine or hydromorphone are
patient’s care, as two important potential diagnoses are placenta previa rapid and effective, titratable, and generally considered safe in preg-
and placental abruption. In any case, the patient will need obstetric obser- nancy. In a patient who may require surgery, it is recommended to keep
vation and fetal monitoring. A type and screen should be performed as the patient nil per os (NPO).
Rho(D) immune globulin is indicated if the patient is Rh negative.
Vaginal Bleeding 30
KEY CONCEPTS
• P regnancy status is the single most important determination to make in
evaluating a patient with vaginal bleeding.
• The use of the term dysfunctional uterine bleeding is no longer recom-
mended, and the term abnormal uterine bleeding is preferred.
• The etiologies of abnormal uterine bleeding can be divided into structural
and nonstructural causes using the PALM-COEIN classification. Struc-
tural causes include polyps, adenomyosis, leiomyomas, and malignancy
(PALM). Nonstructural causes include coagulopathy, ovulatory dysfunc-
tion, endometrial, iatrogenic, and not yet classified causes (COEIN).
• For nonpregnant patients, the numerous causes of abnormal uterine bleed-
ing are best categorized by patient age. The possibility of cancer should
always be considered in postmenopausal women.
• Hormonal and nonhormonal treatments can be initiated in the emergency
department to temporize an acute bleeding episode in a nonpregnant
patient until they can follow up with their gynecologist.
• Vaginal bleeding is common in early pregnancy. Most patients will be diag-
nosed with threatened miscarriage, but ectopic pregnancy should always
be considered at any level of serum β-hCG.
• Vaginal bleeding after the 20th week of pregnancy is less common and is
often associated with significant morbidity and mortality for the mother
and fetus. These patients should be managed in close consultation with an
obstetrician.
1. To avoid the potential for misdiagnosis and possible disruption of a Answer: b. Pregnancy status is the single most important determi-
desired intrauterine pregnancy, American College of Obstetricians nation to make when evaluating a patient with vaginal bleeding and
and Gynecologists (ACOG) recommends using which value of β- should be determined after assessing stability.
human chorionic gonadotropin (β-hCG) for the discriminatory 4. A 30-year-old woman with a history of polycystic ovarian syn-
zone? drome (PCOS) and prior deep vein thrombosis (DVT) presents to
a. 1500 IU/mL the emergency department (ED) with 10 days of irregular, heavy
b. 2000 IU/mL uterine bleeding. Her vital signs are normal, and a urine pregnancy
c. 3500 IU/mL test is negative. She currently takes no medication. Which of the
d. 4500 IU/mL following is the most appropriate treatment?
Answer: c. The discriminatory zone has traditionally been accepted a. Oral contraceptive taper
to be approximately 2000 IU/mL for transvaginal ultrasound (TVUS). b. Medroxyprogesterone acetate
However, viable intrauterine pregnancies may not yet be visualized on c. Oral tranexamic acid
TVUS at β-hCG levels greater than 2000 IU/mL. Therefore ACOG cur- d. Ibuprofen
rently advises that if the concept of the discriminatory zone is to be Answer: d. Patients with PCOS may have anovulatory cycles and fre-
used to help diagnose ectopic pregnancy, the value should be as high quently benefit from hormonal treatment such as oral contraceptive
as 3500 IU/mL to avoid the potential for misdiagnosis and possible pills. However, as the patient has a history of prior DVT she has a con-
disruption of a desired intrauterine pregnancy. traindication to hormonal treatments as well as oral tranexamic acid.
2. What is the most common cause of uterine bleeding in the immedi- Ibuprofen would therefore be the most appropriate treatment.
ate postpartum period? 5. A 32-year-old woman who is 26 weeks pregnant describes a
a. Uterine rupture 1-hour history of painless vaginal bleeding after tripping and
b. Vaginal lacerations falling from a standing height. Her vital signs are normal, and
c. Retained products of conception physical examination of the abdomen reveals a nontender uterus
d. Uterine atony with a fundal height 1 cm above the umbilicus. Of the following,
Answer: d. Postpartum hemorrhage is a leading cause of maternal which would be the most appropriate next step in evaluating this
mortality. Within the first 24 hours after delivery, this is usually caused patient?
by uterine atony. After 24 hours, the cause is frequently retained prod- a. Abdominal magnetic resonance imaging (MRI)
ucts of conception. b. Bimanual examination
3. A 30-year-old woman with a history of polycystic ovarian syn- c. Sterile speculum examination
drome (PCOS) and prior deep vein thrombosis (DVT) presents to d. Ultrasound
the ED with 10 days of irregular, heavy uterine bleeding. Her vital Answer: d. Painless vaginal bleeding after the 20th week of pregnancy
signs are normal. What is the first step in her ED management? is suggestive of placenta previa. Pelvic examination (speculum or
a. Check complete blood count (CBC). bimanual) should be deferred until ultrasound has excluded placenta
b. Order urine pregnancy test. previa as the cause of bleeding.
c. Administer 25 mg intravenous conjugated estrogen.
d. Prescribe a taper of oral contraceptive pills.
31 Back Pain
3. A 35-year-old man presents with severe back pain that radiates
KEY CONCEPTS down his right leg. He reports that while lifting a heavy box at work
• A cute low back pain is a common, costly, recurring and painful condition 2 weeks ago, he felt a “pop” in his lower back. He has not been able
that often has no recognizable or dangerous cause. to return to work since the injury occurred. The patient spoke with
• While most cases of back pain are non-specific and improve without lab- his lawyer and was told to come directly to the emergency depart-
oratory evaluation or imaging, the aim of the emergency department (ED) ment to get an magnetic resonance imaging (MRI). He denies hav-
assessment is to identify those serious and emergent pathologies that ing any other symptoms and reports no significant past medical
require urgent treatment. history. During the physical examination, the patient is asked to lie
• A thorough history and physical examination is performed with the goal of on his back, with his knees extended. His right leg is elevated and, at
uncovering high-risk features predisposing the patient to an emergent or 50 degrees, he reports severe pain running down the lateral aspect
life-threatening etiology. of his right leg to his foot. The patient is then asked to sit with his
• The presence of multiple key clinical findings may increase the probability knees flexed and legs hanging over the side of the bed. His legs are
of disease and is often an indication for further investigation of patients passively extended, with no production of pain. The remainder of
with back pain. However, blindly allowing the presence or absence of these the physical examination is normal. What is the most appropriate
individual findings to guide diagnostic treatment will lead to potentially next step in managing this patient?
unnecessary, misleading, and costly investigations in most patients. a. Computed tomography (CT) of the lumbar spine
• Key clinical findings should supplement, not replace, clinical judgement b. Discharge home
when selecting appropriate laboratory and radiologic testing of patients c. Emergent neurosurgical consultation
with back pain. Imaging and laboratory studies are indicated only when d. MRI of the lumbar spine
there is evidence of neurologic deficit or multiple key clinical findings sug- e. Radiography of the lumbar spine
gesting a dangerous or systemic pathologic cause. Answer: b. Several aspects of this scenario point to malingering. The
• Patients who have low back pain emergencies are generally classified into most convincing relates to the physical examination findings. The
five groups: (1) new onset back pain with neurologic findings in a patient straight leg raise (SLR) is the classic test for sciatic nerve root irritation.
with a malignancy history; (2) back pain and symptoms of epidural com- The absence of a positive result generally rules out nerve root irritation.
pression syndrome; (3) back pain with symptoms suggesting an infectious To perform the SLR, the patient is positioned supine, knee extended,
cause; (4) back pain with gross muscle weakness or paralysis; and (5) back and leg elevated until pain is elicited. A positive result is pain radiating
pain with severe or progressive neurologic deficits. down the leg below the knee in a dermatomal distribution when the
• When a critical or emergent diagnosis is strongly suspected, MRI and spine leg is elevated between 30 and 70 degrees. In a patient who may be
surgery consultation should be undertaken emergently. malingering, the SLR can be performed with the patient sitting on the
• Adherence to published guidelines will decrease improper laboratory stud- side of the bed with knees flexed. Passively straightening the legs in
ies and imaging, thereby lowering costs, improving ED throughput and this position should produce equally positive results if true nerve root
improving overall patient care. irritation exists.
• Conservative therapies are recommended for most patients with nonemer- 4. Disk herniation with involvement of the L5 nerve root will present
gent back pain given the high likelihood for recovery. with which of the following findings?
a. Decreased or absent ankle jerk
b. Decreased patellar reflex
c. Diminished sensation of the lateral small toe
1. What is the most likely cause of back pain in a 48-year-old patient d. Impaired plantar flexion
with bilateral leg pain and weakness, urinary retention, decreased e. Weakness with extension of the great toe
rectal tone, and saddle anesthesia? Answer: e. Involvement of the L5 nerve root presents with weakness,
a. Abdominal aortic aneurysm with extension of the great toe, decreased sensation in the first web space,
b. Bone metastasis and normal reflexes. An S1 radiculopathy is characterized by diminished
c. Epidural abscess sensation of the lateral small toe, impaired plantar flexion, and decreased
d. Herniated disk or absent ankle jerk. The patellar reflex is associated with L2–4.
e. Primary bone tumor 5. A history of IV drug use increases the risk for which of the follow-
Answer: d. The listed symptoms and signs describe cauda equina syn- ing causes of acute back pain?
drome, which is usually caused by a large central herniated disk. Other a. Abdominal aortic aneurysm
less common causes include tumor and infection. b. Epidural hematoma
2. What percentage of asymptomatic persons less than 60 years old c. Malignancy
will have herniated disc findings on MRI? d. Transverse myelitis
a. 10% e. Vertebral osteomyelitis
b. 25% Answer: e. Vertebral osteomyelitis and spinal epidural abscess are
c. 50% diagnosed most frequently in an at-risk population that includes
d. 75% patients with a history of diabetes, chronic renal failure, IV drug use,
Answer: b. Disc disease is a component of normal aging and is a very alcoholism, cancer, AND recent surgery or trauma.
nonspecific finding. In fact, one in four asymptomatic persons <60 years 6. Spinal epidural abscess is most commonly caused by which of the
and one in three >60 years will have MRI findings of a herniated disc. following pathogens?
a. Mycobacterium tuberculosis
b. Pseudomonas aeruginosa
c. Staphylococcus aureus
d. Staphylococcus epidermidis
e. Streptococcus pyogenes
Answer: c. S. aureus causes 70% of spinal epidural abscesses.
32
KEY CONCEPTS
Multiple Trauma
(Zyprexa) for the dementia. The patient does not have any com-
plaints but is noted to have a blood pressure of 80/50 mm Hg and
• Immediately after a trauma patient arrives the emergency department (ED), heart rate of 100 beats/min. In evaluating the patient, you should
the primary survey should be performed in a standardized fashion. The goal suspect that the asymptomatic hypotension is most likely due to
of the primary survey is to rapidly identify and initiate the treatment of which of the following?
critical, and life-threatening injuries. a. Antihypertensive medication use
• The extended Focused Assessment with Sonography for Trauma (eFAST) b. Antipsychotic medication use
examination should be performed early in the evaluation of the trauma c. Benzodiazepine use
patient (ideally as part of the primary survey). Thoracic examination of the d. Blood loss
trauma patient by ultrasound is more accurate than plain radiography. Answer: d. Lower extremity weakness, gait disturbances, decreased
• Any patient with potentially life-threatening injuries should have blood typ- visual acuity, and the use of psychotropics, antihypertensives, and
ing and screening performed. When transfusion is indicated, blood prod- sedatives have been associated with falls in older adults, resulting
ucts should be transfused in a 1:1:1 or 1:1:2 ratio of plasma to platelets to in major injury. The use of these medications, particularly antihy-
packed red blood cells. pertensives, should not be considered causative in trauma patients
• Tranexamic acid (TXA) is indicated for patients with evidence of significant with hypotension until acute hemorrhage has been ruled out. In
hemorrhage or shock and is given as a 1-g intravenous bolus followed by a addition, anticoagulants, antiplatelet drugs, and aspirin are com-
1-g infusion over 8 h. Results are best if started within an hour of injury but monly prescribed, and their effects should be suspected and reversed,
benefit may occur when it is given within 3 h. if warranted.
• Special consideration should be given to the elderly who demonstrate 4. A severely injured, hypotensive trauma patient is being considered
increased morbidity and mortality in settings where traumatic injury ini- for permissive hypotension because she has a contained retroperi-
tially appear less concerning. Patients over the age 65 with one or more of toneal hematoma and is not actively hemorrhaging. In permissive
the following comorbid conditions (coagulopathy, cirrhosis, COPD, CAD, or hypotension, the mean arterial pressure is restored to a goal of 50
DM) have more than twice the mortality risk than younger patients. mm Hg. Which of the following should help you decide against
using permissive hypotension?
1. A 33-year-old mother and her 2-year-old-son are brought in by a. Age >80 years
paramedics after they were both hit by a car moving at 15 mph. b. Age <10 years
Although mother and child had an identical mechanism of injury, c. Associated traumatic brain injury
the son would be at greater risk for all the following injuries, with d. Hemoglobin of 10 g/dL
the following exception: e. Intoxication
a. Head injury Answer: c. In the severely injured, hypotensive trauma patient, resto-
b. Hypothermia ration of normal blood pressure may be undesirable. The concept of per-
c. Intra-abdominal injury missive hypotension is based on the concern that resuscitation to normal
d. Multisystem injury blood pressures may increase bleeding from a site that is contained and
e. Posttraumatic stress disorder not actively hemorrhaging. In permissive hypotension, the mean arterial
Answer: e. Injury patterns can differ significantly between adults and pressure (MAP) is restored to a goal of about 50 mm Hg. Studies have
children subjected to similar mechanisms of trauma. The major ana- shown that this strategy leads to less blood product use, less bleeding,
tomic distinctions relate to the smaller size and surface area, larger and lower incidence of coagulopathy. However, the provider should be
head-to-body ratio, and less protected abdominal cavity of the child. aware that permissive hypotension is contraindicated in the manage-
As a result, children are more vulnerable to multisystem injury in blunt ment of traumatic brain injury because of the risk of hypoperfusion.
trauma, including significant head and intra-abdominal injuries, as 5. An 87-year-old male with history of atrial fibrillation on anti-
well as being at greater risk for hypothermia. coagulation, COPD, hypertension, and hyperlipidemia fell from a
2. Which of the following is the goal of the primary survey? standing height sustaining a pelvic fracture requiring blood trans-
a. Determine which consultations should be obtained. fusion. Which of the following should raise concern for increased
b. Do an AMPLE (allergies, medications, past medical history, last mortality in geriatric trauma patients?
meal, environments and events) history. a. Anticoagulation
c. Obtain pertinent historical data from the paramedics. b. Age >85
d. Perform a radiographic evaluation. c. Isolated pelvic fracture
e. Rapidly identify critical life-threatening diagnoses and begin d. COPD
treatment at the time of the diagnosis. e. All of the above
Answer: e. The emergency clinician should use a standardized Answer: e. All of the above can contribute to both morbidity and mor-
approach to the initial evaluation of these patients. Following the tality of geriatric trauma patients. Often times with less severe injury
Advanced Trauma Life Support (ATLS) algorithm of ABCs in the pri- patterns, this population will have increased transfusion needs, com-
mary survey allows the timely identification of critical diagnoses and plications due to hospitalizations, immobilization, and prolonged reha-
intervention without delay. The primary survey should be performed bilitation times. Patients with one or more of: coagulopathy, cirrhosis,
in a standardized fashion immediately after the patient arrives in the COPD, CAD, or DM have shown twice the risk of death in the setting of
emergency department. The goal of the primary survey is to identify trauma. Additionally, elderly patients who sustain a pelvic ring injury
critical, life-threatening diagnoses rapidly and begin treatment at the are prone to increased mortality with those >85 years at highest risk.
time of diagnosis. The goals of the secondary survey are to obtain per- 6. A 52-year-old female involved in a motor vehicle collision (MVC) at 7
tinent historical data about the patient and injury as well as evaluate pm is transferred to a Level 1 trauma center. She arrives at 11 pm and
and treat injuries not found on the primary survey. An AMPLE history despite appropriate isotonic resuscitation and remains hypotensive.
should be obtained. Transfusion protocol is initiated. What is the next best intervention?
3. An 89-year-old man who was a restrained front seat passenger a. Administer tranexamic acid (TXA) 1 g over 10 minutes
with a history of hypertension, anxiety disorder, and dementia b. Administer TXA 1 g over next 8 hours
is being evaluated after a head-on collision. His home medica- c. Repeat primary survey
tions include an angiotensin-converting enzyme (ACE) inhibitor d. None of the above
for the hypertension, lorazepam for the anxiety, and olanzapine e. All of the above
Answer: c. As with any transferred trauma patient any non-response
or change in condition should prompt rapid re-evaluation.
33 Head Trauma
2. Which of the following statements is true regarding cerebral blood
KEY CONCEPTS flow (CBF), cerebral perfusion pressure (CPP), and intracranial
• H ead trauma is a broad term describing an external trauma to the craniofacial area pressure (ICP)?
of the body from blunt, penetrating, blast, rotational, or acceleration-deceleration a. CBF autoregulation is lost below a CPP of 60 mm Hg.
forces. The term head injury refers to a clinically evident injury on physical exam- b. CPP closely parallels diastolic blood pressure.
ination, and the term brain injury indicates an injury to the brain itself. c. Normal ICP is 65 to 195 mm Hg.
• Traumatic brain injury (TBI) is often categorized into mild (Glasgow Coma d. CPP = mean arterial pressure (MAP) − ICP.
Scale score, 13–15), moderate (GCS score, 9–12), and severe (GCS score, e. The only resistance to CBF is ICP.
3–8), but this actually represents a spectrum of injury. Patients with a pre- Answer: d. CBF depends on CPP, which is the blood flow pressure gra-
sentation GCS score of 13–15 who are stable or improving are unlikely to dient. CBF resistance is provided by mean systemic venous pressures
have CT scan findings that warrant intervention. and ICP, predominantly by the latter. CPP closely parallels MAP offset
• The motor component of the GCS is the strongest predictor of outcome by ICP; thus, the formula CPP = MAP − ICP. ICP is estimated clinically
following TBI. by the central venous pressure unless a ventricular catheter is in place
• Secondary systemic insults such as hypoxia and hypotension worsen neu- and ICP can be directly determined. CBF autoregulation is lost below
rologic outcome and should be corrected as soon as detected. a CPP of 40 mm Hg. Normal ICP is 5 to 15 mm Hg or 65 to 195 mm
• A noncontrast head CT scan is the imaging modality of choice when TBI is H2O.
suspected. 3. Which of the following parameters are associated with a worsened
• Anticonvulsant prophylaxis with phenytoin or levetiracetam and broad- outcome after traumatic brain injury (TBI)?
spectrum antibiotics should be given to patients with penetrating brain a. Hematocrit (Hct) <30%
injuries for 7 days postinjury. b. Temperature >38.5°C (101.3°F)
• Patients can deteriorate from an expanding intracranial hematoma after a c. Partial pressure of oxygen (Po2) <60 mm Hg
mild traumatic brain injury (MTBI), and should undergo serial evaluations, d. A and C
including GCS scoring. e. All of the above
• An MTBI can be easily overlooked when an alert patient presents with dis- Answer: e. All of the above. The following are associated with wors-
tracting injuries. Specifically, assess for disorientation, confusion, amnesia, ened outcomes after TBI:
or disordered awareness (with or without loss of consciousness). Hematocrit (Hct) <30%
• Imaging of patients with MTBI should follow a validated guideline, such as the Temperature >38.5°C (101.3°F)
Canadian CT Head Rule and New Orleans Criteria. Emergency clinicians should Systemic blood pressure (SBP) <90 mm Hg
select the system most applicable for their setting and patient population. Po2 <60 mm Hg
• Intoxicated individuals are high-risk patients. Alcohol and drug use affect 4. A 27-year-old man presents after a motor vehicle collision (MVC)
the GCS score and may significantly obscure the neurologic examination. with a severe closed head trauma. On examination, you calculate a
• Most patients with MTBI can be discharged from the emergency depart- Glasgow Coma Scale (GCS) score of 5 and a left dilated pupil, with a
ment (ED) with a normal examination and after a reasonable period of sluggish pupillary reflex compared with the right. What other find-
observation (4–6 h) or following a negative CT scan of the head. ing will your examination likely reveal?
• Patients should be discharged with instructions describing the signs and a. Left carotid bruit
symptoms of acute and delayed complications of MTBI. All discharge b. Left foot weakness
instructions should be relayed to a responsible third party. c. Loss of controlled pain/temperature sensation
• Athletes with a concussive head injury should be immediately removed d. Right carotid bruit
from play and not return until they have been evaluated by a health care e. Right-sided hemiparesis
provider with expertise in concussion management. There should be a grad- Answer: e. Uncal herniation is the most common posttraumatic herni-
ual stepwise increase in physical activity. ation syndrome. The initial pressure compresses the third cranial nerve
• Older adults (age greater than 60 years) may have significant intracranial (CN III), causing ipsilateral pupillary sluggishness, ptosis, anisocoria,
injuries and not show signs of deterioration, especially if their baseline and impaired extraocular movements. Contralateral hemiparesis can
cognitive functioning is impaired; clinicians should therefore have a low develop early after an initial normal motor examination. In some cases,
threshold for obtaining CT scans with these patients. the contralateral uncus is compressed, resulting in ipsilateral weakness
• Falls in older adults, including low-mechanism falls, should prompt emer- (Kernohan notch syndrome).
gency clinicians to consider the possibility of brain injury.
5. A 24-year-old man presents with a closed head injury after a motor
1. The primary goal in the emergency care of a head trauma patient vehicle collision (MVC). The physical examination is remarkable
is prevention or reduction of secondary insults that are known to for a sluggish left pupil, right-sided hemiparesis, and a Glasgow
worsen outcome after traumatic brain injury (TBI). This includes Coma Scale (GCS) score of 12. What should be the most appropri-
acutely preventing: ate next step in this patient’s management?
a. Hypotension a. 3% hypertonic saline IV
b. Hypoxia b. Intubation and moderate hyperventilation
c. Malnutrition c. Mannitol, 1 g/kg IV
d. All of the above d. Methylprednisolone IV
e. Hypotension and hypoxia e. Pentobarbital IV
Answer: e. Avoiding or reversing hypotension and hypoxia is essential
in the acute management of TBI patients in the emergency department.
Hypotension and hypoxia each double the risk of mortality following
TBI.
CHAPTER 33 Head Trauma
Answer: b. The most rapid effect on intracranial pressure (ICP) Initially, in the uncal herniation process, the motor examination
reduction is achieved via intubation and moderate hyperventilation can be normal, but contralateral Babinski responses develop early.
Contralateral hemiparesis develops as the ipsilateral peduncle is com-
to a Pco2 of 30 to 35 mm Hg. The effect peaks within minutes and
should be considered a short-term intervention, with an expected pressed against the tentorium. With continued progression of the her-
ICP reduction of 25%. Prolonged hyperventilation, however, is dan- niation, bilateral decerebrate posturing eventually occurs; decorticate
gerous. Steroids may worsen outcome after traumatic brain injury posturing is not always seen with the uncal herniation syndrome. In
(TBI). Mannitol is generally efficacious and exerts an effect within some patients, the contralateral cerebral peduncle is forced against the
minutes and lasts hours. Other neuroprotective effects include opposite edge of the tentorial hiatus. Hemiparesis is then detected ipsi-
volume expansion, viscosity reduction, cerebral blood flow (CBF) lateral to the dilated pupil and mass lesion, termed Kernohan notch syn-
improvement, and free radical scavenging. Hypertonic saline data drome, and causes false-localizing motor findings. As uncal herniation
are encouraging but inconclusive; stronger data exist in the pedi- progresses, direct brainstem compression causes additional alterations
atric literature. Barbiturates exert a modest ICP-lowering effect in the LOC, respiratory pattern, and cardiovascular system. Mental sta-
because of their lowering of the cerebral metabolic rate and oxygen tus changes may initially be subtle, such as agitation, restlessness, or
demand. confusion, but soon lethargy occurs, with progression to frank coma.
6. What is the minimum time after becoming asymptomatic that an The patient’s respiratory pattern may initially be normal, followed by
individual should refrain from playing sports after a concussion if sustained hyperventilation. With continued brainstem compression,
no loss of consciousness (LOC) or prolonged posttraumatic amne- an ataxic respiratory pattern develops. The patient’s hemodynamic
sia occurred at the time of injury? status may change, with rapid fluctuations in blood pressure and car-
a. 24 hours diac conduction. Herniation that is uncontrolled progresses rapidly to
b. 48 hours brainstem failure, cardiovascular collapse, and death.
c. 1 week 8. Central pontine myelinolysis is a potential adverse event associated
d. 2 weeks with the administration of which of the following medications?
e. 1 month a. Etomidate
Answer: c. All current recommendations for return to play after a b. Hypertonic saline
sports-related concussion state that players with concussion should c. Mannitol
not return to play for at least 1 week after they have become asymp- d. Methylprednisolone
tomatic. This is usually increased to at least a symptom-free month for e. Pentobarbital
an extended LOC or prolonged posttraumatic amnesia occurring at the Answer: b. Central pontine myelinolysis is a potentially adverse event
time of the concussion. associated with hypertonic saline administration.
7. A 15-year-old boy presents after being hit in the head with a base- 9. What percentage of mild traumatic brain injury (MTBI) patients
ball. He has a Glasgow Coma Scale (GCS) score of 7 and a large discharged from the emergency department (ED) will continue to
hematoma of his scalp, anterior and superior to his right ear. In have post concussive symptoms at 3 months and 1 year after injury?
addition, he is noted to have unequal pupils and a sluggish papillary a. 5% at 3 months and 10% at 1 year
light reflex of the right eye. Which of the following is most likely in b. 40% at 3 months and 50% at 1 year
this patient? c. 30% at 3 months and 15% at 1 year
a. Central transtentorial herniation d. 5% at 3 months and 5% at 1 year
b. Cerebellotonsillar herniation e. None of the above
c. Downward transtentorial herniation Answer: c. Studies of ED patients have indicated that as many as 30%
d. Uncal herniation of patients with a discharge diagnosis of MTBI will have symptoms at
e. Upward transtentorial herniation 3 months postinjury, and up to 15% will continue to be symptomatic
Answer: d. Uncal herniation is often associated with traumatic extra- at 1-year postinjury.
axial hematomas in the lateral middle fossa of the temporal lobe. The 10. Per the Canadian Computed Tomography Head Rule (CCHR)
classic signs and symptoms are caused by compression of the ipsilateral which criteria are NOT considered “High-Risk” for intracranial
uncus of the temporal lobe on the U-shaped edge of the tentorium cere- injury and possible neurologic intervention?
belli as the brain is forced through the tentorial hiatus. As compression a. A Glasgow Coma Scale (GCS) score that is persistently <15
of the uncus begins, CN III is compressed; anisocoria, ptosis, impaired after 2 hours
extraocular movements, and a sluggish pupillary light reflex develop b. Suspected open or depressed skull fracture
on the side ipsilateral to the expanding mass lesion. This phase may last c. Vomiting 2 or more times
for minutes to hours, depending on how rapidly the expanding lesion is d. Age over 50 years
changing. As the herniation progresses, compression of the ipsilateral e. Any sign of basal skull fracture
oculomotor nerve eventually causes ipsilateral pupillary dilation and Answer: d. All are high risk except D. High-risk are patients age ≥65
nonreactivity. years. Other criteria that suggest important brain injury on CT include
amnesia before impact ≥30 min and dangerous mechanism (pedestrian
struck by vehicle, occupant ejected from vehicle, fall from elevation >3
feet [five stairs]).
Facial Trauma 34
KEY CONCEPTS
• T he face is central to the patient’s ability to breathe, eat, and communicate. • S hock from facial trauma is rare and results only from obvious external bleed-
Injuries to the face can have serious psychological and psychosocial conse- ing. Facial injuries should not distract the emergency clinician from searching
quences. for other causes of shock.
• Facial injuries may be prevented by the appropriate use of safety devices • Assertive management of the airway is indicated in a patient with severe
including motor vehicular seat belts, child restraints, air bags, helmets, and facial injuries. Surgical management (cricothyroidotomy) may be required,
mouth and face guards. particularly with penetrating gunshot wounds.
• The epidemiology of facial injury is changing, with an increasing proportion • Directed facial CT scanning is the optimal imaging technique in patients with
of injuries occurring as a result of interpersonal violence. A careful history is obvious injuries.
required, and the possibility of abuse should be considered for every patient. • Definitive facial treatment may be delayed, if necessary, to address other
serious injuries.
1. A nasal fracture complication that requires urgent treatment is: blood in the anterior chamber of the eye. Which of the following
A. Periorbital swelling diagnostic or treatment options should be avoided?
B. Septal hematoma A. CT imaging of the face
C. Laceration B. Ophthalmologic consultation
D. Epistaxis C. Prophylactic antibiotics
E. Subcutaneous edema D. Thorough eye examination including tonometry
Answer: B. A septal hematoma is a collection of blood within the nasal E. Medications for pain and nausea and place patient in semire-
septum that can occur following trauma. A septal hematoma should cumbent position
be drained promptly to avoid the risk of developing a septal abscess or Answer: D. The patient is presenting with a history of blunt trauma and
ischemia of the septal cartilage leading to a saddle nose deformity. With examination findings consistent for a globe rupture. A thorough history
the use of an anesthetic, the hematoma can be aspirated or incised, including mechanism and anticoagulant use should be obtained. CT
drained, and packed to avoid reoccurrence. imaging should be performed to look for additional traumatic injuries.
2. A patient with a mandibular fracture may complain of: The patient should be medicated for pain and nausea to avoid any increase
A. Trismus in intraocular pressures resulting in possible aqueous fluid loss. Prophy-
B. Epistaxis lactic antibiotics are initiated to prevent secondary endophthalmitis. The
C. Paresthesias of the inferior eyelid, nose, and upper lip affected eye should not be manipulated, a rigid shield should be used for
D. Ocular muscle entrapment protection, and ophthalmology should be emergently consulted.
E. Postnasal drip and sweet taste in the mouth 4. Cribriform plate fractures can be associated with:
Answer: A. Individuals with mandibular fractures may report pain A. LeFort I fracture
with chewing, abnormal bite, jaw malalignment, paresthesias of B. CSF rhinorrhea
the lip and chin area, trismus, oral bleeding, and dental injuries. An C. Dental alveolar fractures
orthopantomogram (Panorex) has 92% sensitivity in the diagnosis of D. Ageusia
a mandibular fracture; however, when there is high clinical suspicion E. Tripod fractures
or concern for associated traumatic injuries a facial CT scan should Answer: B. Cribriform plate fractures are commonly associated
be obtained. Epistaxis can be seen with isolated nasal fractures as well with severe traumatic injuries including TBI and CSI. Symptoms can
as maxillary fractures. Postnasal drip and a sweet taste in the mouth include tenderness, swelling, bleeding, CSF rhinorrhea, and anosmia.
could be related to a CSF leak, which can be seen in frontal and nasal More complex midface fractures, LeFort III, or significant nasal trauma
bone fractures. Ocular muscle entrapment can be related to orbital and affecting the NOE complex are associated with fractures of the cribri-
midface fractures. form plate. ATLS guidelines should be followed when evaluating this
3. A patient presents following a physical altercation complaining patient. CT imaging including head, C-spine, and face are obtained
of eye pain, decreased vision, and swelling. On physical exam- and a CTA to evaluate for blunt cerebrovascular injury is often per-
ination he is noted to have periorbital ecchymosis, swelling, and formed. Appropriate specialists (neurosurgery) should be consulted
tenderness, an irregular, tear-shaped pupil, and a collection of and involved in the patient’s care.
35 Spinal Trauma
KEY CONCEPTS
• N EXUS or CCR decision rules may be used to determine the need for radio- coexisting hemorrhagic shock, cardiac tamponade, or tension pneumothorax
graphic imaging in the awake, evaluable trauma patient. has been eliminated.
• CT scanning is preferred over plain radiography in the evaluation of the trauma • Because neurogenic hypotension can lead to hypoperfusion and secondary
patient with potential spinal injury, especially if axial imaging is needed for spinal cord ischemia, prolonged, severe hypotension (systolic blood pressure
evaluation of other injuries. < 70 mm Hg) should be prevented and treated.
• Suspicion for an anterior cord syndrome warrants prompt neurosurgical con- • Methylprednisolone or any other corticosteroid is not beneficial in the treat-
sultation because it is a potentially surgically correctable lesion. ment of acute spinal cord injury (SCI) and should not be used.
• Neurogenic hypotension is a diagnosis of exclusion in the trauma victim and • Emergency department management of SCI includes care to prevent further
should not be considered the cause of hypotension unless the presence of cord injury, pressure ulcers, bladder distention, and gastric distention.
1. Which of the following is a stable cervical spine fracture? 5. A 23-year-old man presents after a rollover MVC. He is brought
A. Anterior atlantoaxial dislocation without fracture by emergency medical services (EMS) from the scene fully
B. Bilateral facet dislocation restrained on a backboard, with a cervical collar. The physical
C. Flexion teardrop examination is remarkable for moderate symmetric numbness
D. Jefferson fracture below the neck, symmetric arm and leg weakness, intact reflexes,
E. Unilateral facet dislocation and diminished rectal tone. Vital signs are heart rate, 94 beats/
Answer: E. See Table 35.1 for a classification of spinal injuries accord- min, blood pressure, 80/46 mm Hg, respiratory rate, 24 breaths/
ing to stability. min, and O2 saturation, 96%. Which of the following treatment
2. A 28-year-old man is brought to the emergency department sequences is most indicated?
(ED) after a rollover motor vehicle collision (MVC). He is mod- A. Crystalloid to CT scan to phenylephrine
erately hypotensive, unable to flex his elbows, and has diffuse B. Crystalloid to focused assessment with sonography for
lower extremity paralysis. What is the likely site of the lesion? trauma (FAST) examination to transfusion
A. C4 C. Crystalloid to phenylephrine to transfusion
B. C5 D. Dopamine to crystalloid to CT scan
C. C6 E. Transfusion to phenylephrine
D. C7 Answer: B. Spinal shock should not be considered the cause of hypo-
E. C8 tension unless the patient is flaccid and areflexic. Crystalloid is the first
Answer: C. See Table 35.2 step regardless of the traumatic hypotensive cause. The possibilities of
3. Which of the following statements regarding high-dose methyl- coexisting hemorrhagic shock, cardiac tamponade, tension pneumo-
prednisolone after spinal cord injury is true? thorax, or other life-threatening injuries should first be addressed. The
A. Dexamethasone is superior to methylprednisolone if a con- absence of vasomotor activity in patients with neurogenic hypotension
current closed head injury is present. may mask the usual presentation of these life-threatening injuries. In
B. High-dose methylprednisolone should not be used to treat this case, the lack of flaccidity and presence of reflexes argues for a non-
spinal cord injury. neurogenic cause for the hypotension.
C. It is efficacious after penetrating injury if given within 4 6. Which of the following is not a characteristic of flexion-
hours. distraction injuries of the lumbar spine?
D. It is efficacious in cases of spinal shock. A. Compression of the anterior vertebral body
E. It is more efficacious after thoracic than after lumbar injuries. B. Associated intrabdominal injuries are common
Answer: B. Evidence that high-dose methylprednisolone is a clinically C. Axis of rotation anterior to the vertebral body
efficacious intervention in the management of acute, blunt, partial spi- D. Unstable fracture pattern
nal cord injury is lacking and, because of severe side effects, it should Answer: C. Flexion distraction injuries are characterized by an axis of
not be used. rotation posterior to the anterior longitudinal ligament causing com-
4. A 27-year-old man presents after a high-speed rollover MVC. pression of the anterior vertebral body and distraction of the posterior
The physical examination is remarkable for T8 motor-sensory elements with failure of all three columns. It is an unstable fracture
deficit and a moderately distended abdomen that is nontender. pattern with a high rate of associated intra-abdominal injuries.
Vital signs are heart rate, 108 beats/min, blood pressure, 88/40 7. Presence of which of the following findings warrants imaging to
mm Hg, respiratory rate, 22 breaths/min, temperature, 35° C evaluate for blunt cerebrovascular injury?
(95° F), and oxygen (O2) saturation, 96%. Which of the following A. Isolated seat belt abrasion
tests or treatments is indicated? B. LeFort I fracture
A. Baseline laboratory tests and observation C. Frontal scalp hematoma
B. Computed tomography (CT) scan of the abdomen D. Cervical bruit in 40-year-old extricated after MVA
C. Intravenous phenylephrine infusion Answer: D. A cervical bruit auscultated in a patient under the age of
D. Packed red blood cell transfusion 50 is a sign of possible blunt cerebrovascular injury and requires ded-
E. Thoracolumbar spine films icated arterial imaging. A seat belt sign with significant swelling, pain
Answer: B. Spinal cord injury often renders the abdominal exam- or altered mental status is indication for screening for BCVI, but not
ination unremarkable. CT, ultrasonography, diagnostic peritoneal an isolated seat belt abrasion. Midface fractures with instability, LeFort
lavage, or some combination is necessary to rule out intra-abdominal grades II and III, are associated with BCVI. Isolated scalp hematoma is
injury. not associated with an increased risk of BCVI.
36 Neck Trauma
KEY CONCEPTS Answer: C. Stack splint with extension of distal phalanx. The descrip-
tion of the injury is consistent with a mallet finger injury, where the
• F or general testing of the motor nerve function of the hand, have the patient distal extensor tendon has pulled off the proximal end of the distal pha-
make an “OK” sign with their thumb and index finger (testing the median lanx. The premolded Stack splint keeps the distal phalanx in extension
nerve), while spreading/abducting the third, fourth, and fifth fingers (ulnar and can remain in place for 6 weeks for healing. The position of com-
nerve) and dorsiflexing the wrist (radial nerve). fort for the finger in an aluminium volar splint is with the PIP and DIP
• For general sensory exam, test two-point discrimination on the finger tufts at approximately 10 to 15 degrees of flexion, not complete extension as
of the index finger (median nerve), the little finger (ulnar nerve), and the needed to allow the tendinous insertion to reattach. A straight alumin-
dorsum of the first webspace (radial nerve). ium splint could be used on the dorsal surface of the finger to keep the
• The most important treatment for the majority of hand injuries is applying DIP joint straight. A dorsal blocking splint is used for flexor tendon
an appropriate splint. The neutral position for general fractures of the hand lacerations or ruptures, preventing any extension in the digit and hand
or fingers is achieved by placing a volar splint ideally with the wrist at 20 to to decrease the contraction of the flexor tendon, which will increase
30 degrees extension, MCP at 70 to 90 degrees flexion, and the PIP and DIP the extensor tendon strain. The ulnar gutter splint provides much more
kept in extension. This is also known as intrinsic plus position. extensive immobilization than is necessary for this injury.
• The most common extensor tendon injury is a terminal tendon disruption 3. After cutting her hand while washing a drinking glass, a patient
from sudden flexion of the extended DIP joint resulting in a mallet injury. complains of a laceration to her finger. In which location of the pal-
• A flexor tendon injury should be considered when an injured finger does not mar hand surface is this laceration most likely to cause a lack of
assume its naturally flexed position on cascade sign testing. flexion to the index finger PIP joint?
• Traditionally, patients with acute open wounds on the hand have been a. Base of the thenar eminence
treated prophylactically with antibiotics, but there is little scientific data b. Distal end of the middle phalanx
supporting the clinical efficacy of this practice. c. Proximal end of the proximal phalanx
• For a stable finger tuft with a subungual hematoma without external disrup- d. Volar crease of the MCP joint
tion of the nail plate, trephination without antibiotics is the only treatment Answer: D. Volar crease of the MCP joint. The flexor digitorum super-
needed. ficialis (FDS) functions to flex the finger at the PIP joint. It attaches
• Small tuft avulsions defined as less than 1 cm3 can be treated conserva- at the proximal end of the middle phalanx as well as the distal end of
tively, while larger amputations should have emergent consultation with the proximal phalanx. A laceration along the volar crease of the MCP
a hand surgeon for consideration for flap coverage or reimplantation. The joint will cut the FDS tendon when the fingers are in extension and
presence of exposed bone or tendon indicates the need for surgical inter- reduce the flexion of the PIP joint. The flexor digitorum profundus
vention. (FDS) inserts at the proximal end of the distal phalanx. Cutting the
• Most hand infections are based on skin flora; treatment for outpatient finger from the mid aspect of the middle phalanx distally results in an
injuries are oral first-generation cephalosporins. With the growing emer- injury to the FDP and an inability to flex at the DIP joint. There are no
gence of community- acquired MRSA, antibiotics such as doxycycline, tendons running within the thenar webspace.
TMP-SMX, or clindamycin should be considered. For inpatient injuries, IV 4. Which of the following is the most disease-distinguishing finding of
first-generation cephalosporins or vancomycin is recommended. a patient with pyogenic flexor tenosynovitis?
• The four classic Kanavel findings for infectious flexor tenosynovitis are fusi- a. Involved finger is held in flexion
form swelling of the digit, tenderness along the tendon sheath, digit held in b. Pain on extension of the finger
flexion at rest, and pain with passive extension of the digit. c. Tenderness over the tendon sheath
d. Uniform swelling of the involved finger
1. Which of the following demonstrates the motor function of the Answer: B. Pain on extension of the finger. All of these findings are
median nerve? components of Kanavel’s findings for infectious tenosynovitis. Even
a. Extending the hand at the wrist though uniform involved swelling has been listed as the most common
b. Making an “OK” sign with the thumb and first finger sign in some studies, this is misleading as swelling is the most com-
c. Pulling the thumb across the palm to touch the little finger mon sign in many hand problems and is very nonspecific. What most
d. Spreading out the fingers of the hand commonly separates purulent flexor tenosynovitis from other finger
Answer: B. Making an “OK” sign with the thumb and first finger. The infections is pain with passive extension. Many patients with even just
median nerve controls the flexors of the thumb and index finger that cellulitis have pain with palpation over the area of infection on the fin-
allow the hand to make the “OK” sign. The ulnar nerve innervates the ger, so this does not distinguish it well. Only half of the patients diag-
adductor pollicis that pulls the thumb across the palm so it can touch nosed with pyogenic flexor tenosynovitis have all of Kanavel’s findings.
the little finger and innervates the interosseous muscles to allow the 5. A patient presents after a fist fight with a puncture wound to his
hand to spread out or abduct the fingers. The radial nerve innervates hand. Which description of the presentation has the best prognosis?
the extensor carpi radialis longus and the other wrist extensors. a. Fingertip penetration
2. A 25-year old woman comes to the emergency department com- b. Delay in treatment
plaining of hitting the tip of her finger on a basketball. She has c. Depth of the wound
difficulty extending the tip of her ring finger. What is the most d. Involvement of joint
appropriate splinting technique for this injury? Answer: A. Fingertip penetration. The three factors that have been
a. Aluminium volar splint with finger in position of function found to worsen the prognosis for a “fight bite” injury are delay in iden-
b. Dorsal blocking splint with DIP at 15 degrees of flexion tification and treatment of the wound, an increase depth of the wound
c. Stack splint with extension of distal phalanx as it is more likely to include the tendon, and intrusion into the joint
d. Ulnar gutter splint to include ring finger to the tip of the tooth. A puncture wound of the fingertip is much less likely to
create a significant infection due to the close septae that decrease the
likelihood of spreading bacteria in an open wound.
43 Wrist and Forearm Injuries
Answer: e. The radiograph shows the signet ring, or cortical ring sign,
KEY CONCEPTS which refers to the rotary subluxation of the scaphoid and oval appear-
• O n plain radiographs of the wrist, three distinct arcs, known as Gilula’s ance of the tubercle in the anteroposterior (AP) view of the wrist. On
lines, and equal spacing between carpus bones (1–2 mm), known as paral- a properly positioned radiograph, this sign is typically associated with
lelism, assist in the radiographic diagnosis of carpal injuries. scapholunate widening, suggesting ligamentous laxity or dissociation.
• In the setting of trauma, there is a high incidence of occult fractures and The signet ring sign is also used to describe pulmonary computed
soft tissue injuries of the wrist. Because of the associated risk of malunion, tomography (CT) imaging of bronchiectasis in relation to a dilated
nonunion, posttraumatic arthritis, and avascular necrosis (AVN), splint bronchus and associated pulmonary artery.
immobilization is recommended if pain persists despite normal appearing 3. A 55-year-old man complains of wrist pain after a fall onto an out-
radiographs. stretched arm. There is pain and swelling along the carpal bones.
• Routine wrist radiographs which include anteroposterior (AP), lateral, and Also noted is decreased two-point sensation distally on the tips of
oblique projections, may fail to detect scaphoid fractures. the index and middle digits. Which structure is most likely injured?
• Thumb spica immobilization is recommended for suspected scaphoid and a. Median nerve
other carpal fractures. Expedited orthopedic follow-up for repeat assess- b. Radial artery
ment, radiographs, or advanced imaging (e.g., MRI, CT, or bone scan) is c. Radial nerve
indicated. d. Ulnar artery
• Triquetral dorsal chip fractures are best seen on the standard lateral view e. Ulnar nerve
of the wrist as a small avulsion fracture fragment, although a more oblique Answer: a. The median nerve courses through the carpal tunnel on
pronated lateral view may be necessary to visualize these types of frac- the volar aspect of the wrist. It provides sensation to most of the palm
tures. and thumb, half of the ring finger and, specifically, the tips of the index
• Hamate and pisiform fractures are best visualized with a carpal tunnel or and middle digits. The median nerve is the most common neurapraxia
reverse supinated oblique radiograph. associated with Colles fractures.
• Lunate dislocations result in a characteristic triangular appearance of the 4. Which nerve is commonly associated with a Monteggia fracture?
lunate on the posteroanterior (PA) view (commonly referred to as the “piece a. Muscular branch of the radial nerve
of pie” sign) owing to rotation of the lunate in a volar direction. This rotation b. Posterior interosseous nerve
also is visible on the lateral view of the wrist, where the lunate appears like c. Deep branch of the ulnar nerve
a cup tipped forward, spilling its contents into the palm (referred to as the d. Median nerve
“spilled teacup” sign). e. Ulnar nerve
• A Colles fracture is a transverse fracture of the distal radial metaphysis, Answer: b. Injury to the posterior interosseous nerve (PIN), a deep
which is dorsally displaced and angulated. The Smith fracture is a trans- branch of the radial nerve, is commonly associated with a Monteggia
verse fracture of the metaphysis of the distal radius, with associated volar fracture. Because the PIN innervates the finger extensors along with
displacement and angulation. the supinator, associated injury is often manifested by weakness or
• Ulna fractures associated with radial head dislocation are commonly paralysis of the thumb or finger extension.
known as Monteggia fractures. Galeazzi fractures refer to fractures of the 5. A karate player presents with pain over the ulnar aspect of his hand.
middle to distal third of the radius associated with injury to and dislocation On physical examination, there is tenderness over the ulnar aspect
of the distal radial ulnar joint (DRUJ). of the wrist, distal to the volar crease. Paresthesias in the distribu-
tion of the ulnar nerve are present and the patient also complains
1. In the setting of acute trauma and negative radiographs of the wrist, of hand clumsiness. Standard X-rays are negative. What additional
which clinical examination method(s) is (are) used to detect occult view might reveal the diagnosis?
scaphoid fractures? a. Clenched fist view
a. Anatomic snuffbox tenderness b. Carpal tunnel view
b. Scaphoid tubercle tenderness c. Scaphoid view
c. Thumb metacarpal compression tenderness d. External Oblique view
d. A positive Watson’s scaphoid shift test e. Metacarpal view
e. All of the above Answer: b. Pisiform or hook of hamate fractures may cause the constel-
Answer: e. In the setting of acute trauma and negative radiographs, lation of findings in the above scenario and are better appreciated on the
tenderness to palpation within the anatomic snuffbox, on the scaphoid carpal tunnel view. The clenched fist view is best for identifying scapholu-
tubercle, with thumb metacarpal compression, or a positive Watson’s nate widening while the scaphoid view is optimal for identifying scaphoid
scaphoid shift test are all suggestive of occult scaphoid fracture. fractures. The external oblique view would not optimize imaging of these
2. A 45-year-old man complains of wrist pain after falling on an out- two bones and the metacarpal view is not an actual view.
stretched hand. What injury is shown in Figure 43.21 of the patient’s
wrist?
a. Barton’s fracture
b. Lunate dislocation
c. Perilunate dislocation
d. Scaphoid fracture
e. Scapholunate dissociation
Humerus and Elbow Injuries 44
KEY CONCEPTS
Answer: d. In children with radial head subluxation, the child, fearful
• C linical decision rules for the elbow joint have not been validated. Radio-
for eliciting a pain response, may continue to hold their arm in pro-
graphs should be obtained when there is limitation in range of motion,
tection while appearing more comfortable. Appropriate comfort and
moderate to severe pain, obvious deformity, joint effusion, or significant
discharge may be appropriate with reliable caregivers and with instruc-
tenderness or crepitus over any of the bony prominences or the radial head.
tions to return within 24 hours if the child does not return to normal.
• The threshold for radiographic imaging should be lower in pediatric patients
(with the exception of radial head subluxations), owing to the presence of 3. A 67-year-old right-hand dominant man fell onto his right arm and
open growth plates and limitations to the physical examination. suffered an injury to his upper arm, with crepitus, gross deformity,
• Injuries that result in neurovascular compromise necessitate prompt inter- and is diagnosed with a midshaft humerus fracture. 30 minutes
vention and consultation with an orthopedic specialist for reduction and after placement in a sling begins having difficulty extending the
potential operative intervention. thumb and first finger. Repeat radiography shows improvement in
• In children with a traumatic wrist injury, normal radiographs should prompt alignment of prior fracture. What is the appropriate action?
consideration of an elbow injury causing referred pain to the wrist. a. Adjust splint and discharge to home
• On lateral elbow x-ray, a small anterior fat pad, parallel to the anterior b. Do not adjust splint and discharge to home
surface of the humerus, can be a normal finding. Any convex (“sail sign”) c. Adjust splint and admit for surgery
anterior fat pad and all posterior fat pads are pathological and indicate the d. Do not adjust splint and admit for surgery
presence of joint effusion. e. Admit for compartment checks
• In the setting of trauma, patients with a radiological posterior fat pad sign Answer: c. The patient is now presenting with a radial nerve injury and
of the elbow are assumed to have an intra-articular skeletal injury. In adults, palsy, likely after movement of the fracture fragments. The offending
a posterior fat pad sign is indicative of a radial head fracture, whereas in manipulation should be undone to potentially improve nerve function,
children, a supracondylar fracture is more likely. In the absence of trauma, and the patient should then be admitted for operative repair.
inflammation and infection also cause effusions with positive fat pad signs. 4. A 57-year-old man presents with acute left upper extremity pain
• Radial nerve injury is the most common complication of humeral fractures. while suffering an injury doing pullups. The patient has visible
This is most often a benign neurapraxia that resolves spontaneously. Radial deformity of his proximal upper extremity and is splinting his arm
nerve injuries associated with penetrating trauma or open fractures are and unable to supinate and flex his forearm. The patient is neuro-
likely to represent anatomical disruption requiring operative exploration. vascularly intact. X-rays of the shoulder and humerus are unre-
• The radius and ulna, bound together firmly by the annular ligament and markable. An ultrasound diagnoses a torn biceps tendon. What is
interosseous membrane, typically displace as a unit and dislocate posteri- the most appropriate treatment plan?
orly following a traumatic injury. a. Orthopedics consultation for emergent operative repair of the
• Biceps tendon rupture is more common in men, between ages 40 to 60, ruptured tendon
resulting from an unexpected extension force applied to the arm flexed at b. Splinting and urgent orthopedic follow-up within 48 to 72 hours
90 degrees. Smoking, diabetes, chronic renal failure, systemic lupus erythe- for operative repair
matosus, rheumatoid arthritis, and steroid or fluoroquinolone therapy may c. Splinting and follow-up with PCP in 1 week
predispose to this injury. d. Splinting and discharge with instructions for physical therapy
1. A 4-year-old boy is brought to the emergency department (ED) by his Answer: b. Patients with a biceps tendon rupture should be evaluated
mother after falling from a swing set. He complains of right wrist pain for fractures with plain radiography. If negative, consultation with
and resists detailed examination of the arm. Radiographs are normal. orthopedics for urgent follow-up with possible operative repair is the
What is the most appropriate next step in this patient’s management? most appropriate follow-up plan.
a. Computed tomography (CT) scan of the wrist 5. A 46-year-old construction worker falls onto an extended arm and
b. Elbow radiograph has a sudden injury to their elbow. The patient arrives with a gross
c. Reassurance deformity to the distal humerus/elbow. Radiography reveals a pos-
d. Splinting for 3 to 6 weeks terior elbow dislocation with a wide humerus-olecranon space. The
e. Triple-phase bone scan after 7 to 10 days patient as 1+ radial pulses on the affected arm but is otherwise neu-
Answer: b. In children with wrist pain and traumatic mechanism of injury, rovascularly intact. What is the most appropriate next step?
the absence of a clear-cut explanation for the pain (e.g., normal radiograph) a. Manual reduction of the elbow dislocation and discharge to fol-
should prompt suspicion for an elbow injury producing referred pain. low up with orthopedics
2. A 3-year-old girl comes in from home with arm pain after being b. Manual reduction of the elbow and a CT angiography to evalu-
playfully swung by her parents. The child appears uncomfortable ate for brachial artery injury
and refuses the move her arm. After manipulation and a palpable c. Orthopedics consultation for emergent operative reduction
click, the child appears more comfortable however after 30 minutes d. Vascular surgery consultation for management of potential bra-
will not range her arm for the provider. What is the most appropri- chial artery injury
ate next step for this patient? Answer: b. The patient likely has a brachial artery injury. These are
a. Obtain an XR to evaluate for fracture more common in elbow dislocations with a wide separation between
b. Continue to observe the patient with frequent re-evaluations the olecranon and humerus. While there may be persistent pulses dis-
c. Observe for neurovascular check tally, the presence of pulses does not eliminate the risk for brachial
d. Discharge home with parents with strict return precautions artery injury. The injury should be reduced, and the patient should
e. Consult Orthopedics undergo CT angiography for evaluation of potential vascular injury.
45 Shoulder Injuries
KEY CONCEPTS
3. A 19-year-old skateboarder presents to the emergency department
(ED) complaining of left shoulder pain with his left arm in adduc-
• A xillary nerve function is best evaluated by testing the motor function of tion and supporting the elbow with his right hand. An obvious cla-
the deltoid muscle. vicular defect is palpable on exam. Urgent orthopedic consultation
• The three-view trauma series of the shoulder (true anteroposterior, scapular Y, is recommended for which type of clavicle fracture?
and axillary views) leads to an accurate radiographic diagnosis of most frac- a. Minimally displaced midclavicular fracture
tures and dislocations, although specialized views are sometimes necessary. b. Type I lateral clavicle fracture
Consider the presence of unfused epiphyses in adolescents and young adults. c. Type II lateral clavicle fracture
• Most shoulder girdle fractures can be treated with simple immobilization d. Type III lateral clavicle fracture
with good functional outcomes. Answer: c. Type I lateral clavicle fractures are stable and minimally
• The most important aspect of scapular fractures, scapulothoracic disso- displaced because the coracoclavicular ligaments are intact. Type II
ciation, and posterior sternoclavicular joint (SCJ) dislocations is the high fractures are associated with a torn coracoclavicular ligament and have
incidence of associated injuries to the ipsilateral lung, chest wall, medias- a tendency to displace because the proximal fragment lacks any stabi-
tinum, or shoulder girdle complex. lizing forces. Type III injuries involve the articular surface. More urgent
• Type III acromioclavicular joint (ACJ) dislocations can be treated conserva- orthopedic consultation (before 72 hours) is recommended for type II
tively (immobilization, range of motion exercises, and strengthening). lateral clavicle fractures, because these fractures have up to a 30% inci-
• Recurrence is a common complication after anterior dislocation, especially dence of nonunion and may benefit from surgical repair.
in male patients younger than 30 years old, and such patients likely benefit 4. A 75-year-old right-handed woman presents to the emergency
from arthroscopic surgical repair. department (ED) after sustaining a ground level fall onto an out-
• Posterior dislocation should be included in the differential diagnosis of any stretched right arm. She is holding her right arm in adduction and
shoulder injury, particularly in patients who report shoulder pain after a supporting the weight of the arm with her left hand. On examina-
seizure. tion she does not have an obvious palpable defect but there is sig-
• Ultrasound can be a useful technique for diagnosing tendon tears, selected nificant pain in her shoulder with any attempt to move her arm. She
soft tissue conditions, fractures, and dislocation, as well as for confirming has intact lateral deltoid sensation and full symmetric radial pulses.
successful reduction. The x-ray demonstrates a proximal humerus fracture with minimal
• Early initiation of passive shoulder range-of-motion exercises reduces the displacement. Which of the following is the most appropriate next
risk of adhesive capsulitis when the shoulder is immobilized for any reason. step in care?
a. Emergent orthopedic consultation for operative repair
1. A 27-year-old male unrestrained driver is involved in a high speed b. Pain control, immobilization, and discharge with outpatient fol-
motor vehicle collision (MVC) with direct impact of the steering low-up with orthopedics
wheel into his anterior chest wall. Which shoulder girdle injury rep- c. Orthopedics consultation for outpatient surgical repair after res-
resents a true orthopedic emergency? olution of swelling
a. Anterior glenohumeral dislocation d. Emergent vascular surgery consult due to high risk of associated
b. Anterior sternoclavicular dislocation vascular injury
c. Posterior glenohumeral dislocation Answer: b. Proximal humerus fractures with minimal displacement
d. Posterior sternoclavicular dislocation respond well to conservative care. Patients may begin physical therapy
Answer: d. Posterior sternoclavicular dislocations can be associated as soon as one week after the fracture. Surgical repair is rarely indicated
with life-threatening injuries within the superior mediastinum and for minimally displaced fractures, hence answers (a) and (c) are incor-
intrathoracic cavity; therefore, they are considered true orthopedic rect. She has good radial pulses and these fractures are not commonly
emergencies and should be reduced expeditiously. Reported compli- associated with secondary vascular injury.
cations associated with posterior sternoclavicular dislocations include 5. A 30-year-old recreational basketball player presents after an awk-
compression or lacerations of the great vessels, tracheal compression, ward fall onto an outstretched left arm during a pickup basketball
pneumothorax, thoracic outlet syndrome, tracheoesophageal fistula, game with severe left shoulder pain. He is supporting his left elbow
and injury to the brachial plexus. with his right hand and refuses to move his arm due to pain. There
2. A 30-year-old patient presents complaining of right shoulder pain and is a dimple inferior to the acromion and diffuse pain with palpa-
limited range of motion. On physical examination, you note that the tion throughout his shoulder. There is no evidence of neurovascular
shoulder has a “squared off ” appearance and is held in abduction and injury. True and standard AP radiographs do not show any fracture
external rotation. The patient is unable to adduct the arm or internally or obvious dislocation. The technician was unable to obtain an axil-
rotate without severe pain. What is the best way to manage this patient lary lateral due to significant pain with abduction. Which of the
without the aid of other emergency department (ED) personnel? following is the next best step?
a. Hippocratic method a. Obtain modified axillary lateral or trans-scapular Y view radio-
b. Kocher maneuver graph
c. Milch technique b. Upper extremity CT with intravenous contrast
d. Traction/countertraction method c. Discharge with outpatient follow-up with primary care
Answer: c. The ideal method for reduction of an anterior shoulder dis- d. Refer to orthopedics for likely complete supraspinatus tear
location should be simple, quick, and effective; require little assistance; Answer: a. Posterior dislocations glenohumeral dislocations are
and cause no additional injury to the shoulder. It is wise to be familiar uncommon but cannot be excluded with a true orthogonal view.
with several techniques of reduction because none is uniformly suc- Options to consider in this patient include a modified axillary lateral,
cessful. The Milch technique (see Table 45.3) allows for reduction by apical oblique, or transscapular Y view. A missed posterior dislocation
a single practitioner, and it can be attempted without procedural seda- can result in a locked posterior dislocation requiring orthopedic inter-
tion. Older techniques, such as the Hippocratic method (traction with vention under general anesthesia. If there is a high clinical suspicion, a
the foot in the axilla) and the Kocher maneuver (leverage, adduction, non-contrast shoulder CT scan can reveal the diagnosis; however, the
and internal rotation), are no longer recommended because of a high most appropriate next step would be to obtain orthogonal views. Con-
incidence of associated complications (axillary nerve injury, humeral trast is only necessary if there is suspicion for an associated vascular
shaft and neck fractures, and capsular damage). Traction/countertrac- injury which is uncommon with posterior GH dislocations. Although
tion requires two people to reduce the shoulder (see Table 45.3). an acute rotator cuff tear can present similarly, this patient has had
incomplete radiographic imaging and therefore a diagnosis of rotator
cuff tear represents premature closure.
46 Pelvic Injuries
2. An avulsion fracture caused by the hamstring muscles most likely
KEY CONCEPTS
involves which of the following structures?
• T he most serious pelvic ring injuries caused by high-energy impact are (1) a. Anterior inferior iliac spine
anteroposterior compression fractures (“open-book” fracture), (2) vertical b. Anterior superior iliac spine
shear fractures, and (3) fractures involving significant displacement. These c. Iliac crest
injuries are associated with major blood loss and transfusion requirements. d. Ischial tuberosity
• Pelvic fractures are a marker for serious injury to other organ systems. The e. Sacrum
vast majority of patients who die after sustaining a pelvic fracture have Answer: d. Forceful contraction of the hamstrings can result in an
multiple trauma. injury of ischial tuberosity. The anterior inferior iliac spine may be
• Careful examination of the perineum and buttocks, as well as digital rectal avulsed during strenuous contraction of the rectus femoris muscle. A
and vaginal examinations, are necessary to diagnose open fractures and portion of the iliac crest epiphysis may be avulsed by contraction of the
expanding hematomas. abdominal muscles. The anterior superior iliac spine may be avulsed by
• Computed tomography (CT) is the imaging test of choice to diagnose pelvic forcible contraction of the sartorius muscle.
fracture and concurrent intra-abdominal injuries for patients stable enough 3. A 25-year-old patient involved in a motor vehicle collision (MVC)
to undergo imaging. CT aids in establishing surgical priorities and planning presents with a suspected unstable pelvic fracture. The vital signs
of definitive orthopedic care. are blood pressure, 90/50 mm Hg; heart rate, 120 beats/min; and
• There is an increasing incidence of low-energy (e.g., fall from standing) respiratory rate, 22 breaths/min. What is the most appropriate next
pelvic fractures in older patients which often are managed medically due to step in this patient’s management?
multiple co-morbidities and poor bone quality that would make an anatomic a. Call trauma service for immediate laparotomy
reduction difficult to obtain. b. Call orthopedics for fixation
• Fragility pelvic fractures in older patients often involve the anterior column c. Chest and pelvis radiographs
of the acetabulum, are more comminuted, and result in severe impaction of d. Computed tomography (CT) scan of the abdomen and pelvis
the femur head. e. Perform serial abdominal and focused assessment with sonogra-
• In the hemodynamically unstable patient who cannot undergo CT imaging, phy in trauma (FAST) examinations
the anteroposterior radiograph usually reveals serious pelvic fractures that Answer: c. When patients are too unstable to undergo CT investiga-
cause major pelvic bleeding, which is sufficient information to undertake tion, the anterior/posterior portable radiograph is useful in screen-
pelvic stabilization. ing for pelvic injuries that are most associated with major blood loss.
• The combination of posterior arch fracture plus hypotension is associated Findings with this technique that have been reported to predict the
with a mortality rate of approximately 50%. need for transfusion include “open-book” fracture, displacement of 0.5
• Early fluid resuscitation with blood products in a 1:1:1 ratio (packed red cm or more at any fracture site in the pelvic ring, and displaced sym-
blood cells: platelets: fresh frozen plasma) is recommended for unstable physis pubis or obturator ring fracture. Chest radiographs should be
patients suspected of having active pelvic bleeding. performed to rule out associated injuries that might be the cause of
• Trauma hospitals should have institutional guidelines and mechanisms to the patient’s hypotension and tachycardia (e.g., hemothorax, tension
facilitate early decisions regarding treatment for pelvic hemorrhage. Treat- pneumothorax, widened mediastinum).
ment options include angiography and embolization, pelvic packing, inva- 4. Which of the following treatments can provide definitive control of
sive fixation, or a combination of these. hemorrhage from severe pelvic fracture?
• Unstable patients with a positive focused assessment with sonography in a. Blood transfusion
trauma (FAST) and a pelvic fracture should be treated with laparotomy with b. Commercial circumferential pelvic compression devices
pelvic stabilization and possible pelvic packing, followed by angiography. c. External fixation
• Unstable patients with a negative FAST and a pelvic fracture should be d. Transfusion of concentrated clotting factors
treated with pelvic stabilization (such as a pelvic binder) and angiography, e. Wrapping pelvis with sheet
and then a repeat FAST and laparotomy if they remain unstable. Answer: c. External fixation is an invasive strategy aimed at cessation of
• “Open-book” pelvis fractures should be internally compressed with a pelvic pelvic hemorrhage. Options A, B, and E are important for the emergency
binder or sheet to reduce the size of the pelvis, unless the fracture forces clinician to quickly stabilize a mechanically unstable pelvic fracture and
have already internally rotated the hemi-pelvis. In this case, further internal
hopefully minimize bleeding, but these options, per se, are not definitive
compression may cause an increase in the pelvic diameter.
treatments. Transfusion of blood products is important in overall resus-
• Patients with pelvic fractures are at high risk for subsequent deep venous throm-
citation of the patient but does not stop bleeding from pelvic fractures.
bosis (DVT), despite the use of mechanical and chemical thromboprophylaxis.
5. Which of the following radiographic findings necessitates further
evaluation of the pelvic ring?
1. While examining a patient with blunt abdominal trauma, you note a. Asymmetry of the pelvis caused by rotation
blood at the urethral meatus. What is the most appropriate next b. Duverney fracture
step in the patient’s management? c. Sacroiliac joint 4 mm wide on pelvic radiograph
a. Order a cystography d. Symphysis pubis 7 mm wide
b. Order a retrograde urethrogram e. Transverse fracture of the sacrum
c. Order a urinalysis Answer: d. On the anteroposterior radiograph, the symphysis pubis is
d. Order an anterograde urethrogram normally no more than 5 mm wide, and a small (1 or 2 mm) vertical
e. Order an intravenous pyelography offset of the left and right pubic rami may be normal. Normally, the
Answer: b. Blood at the urethral meatus necessitates a retrograde ure- sacroiliac joint is approximately 2 to 4 mm wide. On the anteroposte-
throgram followed by a cystogram. Gross hematuria is investigated by rior view, the physician may judge the degree of pelvic rotation caused
a combination of urethrography, intravenous pyelography, cystogra- by technique and positioning by the presence of asymmetry.
phy, and computed tomography (CT).
47 Femur and Hip Injuries
KEY CONCEPTS d. Intravenous narcotic pain medication and remove the traction
device
• A hip dislocation is an orthopedic emergency. The likelihood of avascu- e. Immediate orthopedic consult to take the patient to the operat-
lar necrosis (AVN) is related to both the initial degree of trauma and the ing room for an open reduction and internal fixation (ORIF)
amount of time the femoral head remains out of joint. Reduction of the hip Answer: b. Although the patient may ultimately need to go to the
within 6 h after dislocation significantly decreases the incidence of AVN. operating room, the patient is currently unstable and requires resus-
• When a painful hip makes ambulation difficult and plain radiographs do not citation and evaluation for other injuries. A femoral nerve block is a
reveal a fracture, computed tomography (CT) or magnetic resonance imag- valuable adjunct or alternative to systemic analgesics in a patient at
ing (MRI) should be performed. MRI is the gold standard for diagnosis— risk for hypotension and has been underused by emergency clinicians.
although this does not need to be completed in the emergency department Prolonged traction during the assessment and management of other
(ED). injuries can cause or worsen serious neurologic injury in the thigh
• In patients with intertrochanteric fractures, hemodynamic instability can by producing potentially damaging tension on the sciatic or femoral
result from dehydration and blood loss. Up to 70% of patients with these nerves.
injuries are under-resuscitated. 4. What is the most common complication of a proximal femur frac-
• It is important to identify acetabular fractures before closed reduction is ture?
attempted, because intra-articular bone fragments can interfere with effec- a. Avascular necrosis (AVN)
tive reduction. b. Myositis ossificans
• In elderly patients, the use of femoral nerve blocks should be considered, c. Osteomyelitis
due to the potential adverse effects of parenteral opioids. d. Pulmonary embolism
e. Septic arthritis
1. Which of the following fractures has the best outcome and lowest
Answer: a. AVN is the most common complication of proximal femur
rate of complications?
fractures (despite optimal treatment) because of the complex arterial
a. Femoral neck fracture
anatomy. Deep infection in the form of osteomyelitis or septic arthri-
b. Insufficiency fracture
tis is more common with femoral neck fractures because the fracture
c. Intertrochanteric fracture
line extends into the joint. Pulmonary embolism is another significant
d. Lesser trochanteric fracture
complication and is the leading cause of death 7 days after fracture in
e. Subtrochanteric fracture
orthopedic patients.
Answer: d. Treatment of a lesser or greater trochanteric fracture con-
5. A 60-year-old woman presents complaining of right hip pain after
sists of pain control and early mobilization with crutches; weight bear-
a trip and fall at home. The patient denies loss of consciousness or
ing is allowed as tolerated by pain. Outpatient management of this
other symptoms. You note that the patient’s right leg is internally
injury is possible with a satisfactory social situation. The prognosis is
rotated, and the thigh is adducted and flexed at the hip joint so that
good, and healing is generally excellent. The mortality rate during the
the ipsilateral knee is resting on the opposite thigh. Which is the
first year after a femoral neck (also known as an insufficiency fracture)
correct maneuver?
fracture is 15%. Intertrochanteric fractures have an associated mortal-
a. Apply traction and splint the leg in full extension.
ity rate of 10% to 30% in the first year. The reported mortality rate from
b. Apply traction to an extended knee and flexed hip at 90 degrees
subtrochanteric fractures ranges from 10% to 15%.
with a gentle rotational component.
2. A 40-year-old man complains of persistent thigh pain that worsens
c. Consult orthopedics immediately to take the patient to the oper-
over 2 weeks after an assault with a baseball bat. His pain is worse
ating room.
with knee extension. On physical examination, you note that there
d. Provide analgesia and Holter monitor for discharge when ambu-
is a small palpable mass at the mid-anterior thigh. What should be
latory.
your next confirmatory test?
e. Use the Allis maneuver and place the patient in a knee immobi-
a. Bone scan for fracture
lizer.
b. Bone scan for stress fracture
Answer: e. A patient with a posterior dislocation typically holds the
c. Computed tomography (CT) scan for tumor
hip flexed, adducted, and internally rotated. The knee of the affected
d. Magnetic resonance imaging (MRI) for evaluation of torn mus-
cle extremity rests on the opposite thigh. The Allis technique is usually
e. Radiograph for heterotropic calcification effective for both posterior and obturator dislocations. With the knee
Answer: e. Myositis ossificans (heterotrophic ossification) is patho- flexed, the operator applies steady traction in line with the deformity.
logic bone formation at a site where bone is not normally found. Trau- The hip is slowly brought to 90 degrees of flexion while steady upward
matic myositis ossificans results most commonly from a direct blow traction and gentle rotation are applied.
to muscle. It should be suspected when symptoms persist past 10 to 6. A mother brings her 11-year-old boy for evaluation of left knee pain
14 days or if symptoms intensify several weeks after the trauma. The that is worse after physical activity. Radiographs of the knee were
ossific mass is often palpable and may limit motion, depending on its negative. What is the most appropriate next step in the patient’s
location. management?
3. An 80-year-old woman presents complaining of pain in her right a. Hip radiograph with frog-leg views
leg after a motor vehicle collision (MVC). Emergency medical ser- b. Joint aspiration for evaluation of transient synovitis
vices (EMS) noted a swelling and a deformity to her thigh, so they c. Place a knee immobilizer and ensure follow-up in 1 week
placed a traction device on her leg. Her blood pressure is 80/40 mm d. Radiograph of right knee for comparison
Hg; heart rate, 110 beats/min; and respiratory rate, 18 breaths/min. e. Rest, ice, compression, elevation, and orthopedic follow-up
What is the most appropriate next step in the management of the urgently
patient’s fracture?
a. Femoral nerve block and leave the traction device in place
b. Femoral nerve block and remove the traction device
c. Intravenous morphine and leave the traction device in place
Answer: a. This patient may have a slipped capital femoral epiphysis 9. A 15-year-old female gymnast presents after experiencing the sud-
(SCFE), which most commonly develops in boys 10 to 17 years old den onset of severe groin pain during a dismount when she landed
during their period of rapid growth. Referred pain to the knee is a clas- in a flexed-hip position. The pain radiates into her abdomen, and
sic manifestation, and patients frequently present with groin, thigh, or flexion of the hip produces pain, but there is no deformity noted.
knee pain rather than hip pain. Initially, anteroposterior, lateral, and What is the most likely radiographic finding?
frog-leg lateral radiographs of the hip should be obtained. The frog-leg a. Avascular necrosis (AVN) of the femoral head noted on mag-
lateral projection shows the hip in a plane midway between the antero- netic resonance imaging (MRI) scan
posterior and lateral views. b. Diastasis of the pubic symphysis on anteroposterior pelvis radio-
7. A 75-year-old woman presents after a fall from standing. She has graphs
right hip pain and tenderness to palpation but no obvious deformity. c. Femoral neck fracture on dedicated anteroposterior hip radio-
Right knee and ankle examinations are normal, without tenderness, graph
deformity, or external signs of trauma. Hip and pelvis radiographs d. Iliopsoas muscle with some associated hemorrhage on com-
are negative for fracture, but the patient is unable to bear weight on puted tomography (CT) scan
her right leg. The next appropriate step in management is: Answer: d. Gymnasts and dancers are the group of athletes most likely
a. Admit to the hospital for bed rest. to experience an injury to the iliopsoas as a result of sudden force-
b. Discharge the patient home with analgesia and a walker. ful hip flexion against resistance. Severe pain often is experienced in
c. Obtain magnetic resonance imaging (MRI) of the hip to assess the groin, thigh, or low back region. Severe intra-abdominal pain is
for fracture not identified by radiographs. common at the muscle origin and may dominate the clinical picture.
d. Obtain radiographs of the rest of her right leg to ensure no occult Examination reveals groin tenderness and pain with active hip flexion.
fracture is present. Radiographs of the femur should be obtained to identify an avulsion
Answer: c. With hip injury, if radiographs do not show a fracture or fracture of the lesser trochanter. CT scan frequently will demonstrate
suggestion of injury and the patient is unable to ambulate, further a large hematoma. Bed rest with partial flexion at the knee and hip
imaging studies should be obtained to evaluate for occult fracture. Two generally is required for 7 to 10 days. With severe strains, symptoms
percent to 10% of all hip fractures are radiographically occult. Failure may persist for 2 to 3 months. Referral to a sports medicine specialist
to detect these fractures results in increased morbidity and mortality. is appropriate.
8. Which of the following injuries is appropriate for traction splinting 10. A 45-year-old male presents with a posterior hip dislocation after
by prehospital providers? a motor vehicle crash (MVC) noted on radiographs. After seda-
a. Femoral fracture with bone protrusion through the skin tion of the patient and reduction of the dislocated hip, what is the
b. Posterior hip dislocation most appropriate next step in the patient’s management?
c. Severe crush injury of leg with obvious deformity of the knee a. Have the patient ambulate to assess the stability of the joint.
d. Suspected closed mid shaft femoral fracture b. Measure the femoral compartment pressure.
Answer: d. Traction splints can provide pain relief, immobilization, c. Obtain post-reduction hip radiographs to assess for addition-
and limit blood loss when applied correctly to a femoral fracture. al injuries and adequate reduction.
However, contraindications to the use of traction splints include pel- d. Place a traction splint.
vic fractures, patellar fractures, ligamentous knee injuries, and tibia Answer: c. Obtaining post-reduction radiographs to ensure adequate
or fibula fractures. Traction in the prehospital setting should not be reduction and evaluate for associated injuries is essential. After closed
applied to any open fracture that has exposed bone. Such reduction reduction, the hip should be tested for stability, which is accomplished
pulls grossly contaminated bone fragments back into the wound by gently taking it through a full range of motion to see whether it
before adequate débridement can be undertaken in the operating will re-dislocate. After testing has ensured stability, the injured extrem-
room. ity should be placed in a knee immobilizer, and an abduction pillow
should be applied to prevent repeat dislocation.
48 Knee and Lower Leg Injuries a. High-riding patella
KEY CONCEPTS b. Mid-quadriceps muscle tenderness and deformity
• K nee dislocation often causes vascular injury to the popliteal artery. Early c. Popliteal swelling and tenderness
revascularization is crucial. Hard signs of vascular injury include absent d. Positive Lachman test
pedal pulses, cool mottled foot, expanding popliteal hematoma, or popli- e. Suprapatellar tenderness with patella baja
teal hemorrhage. When any of these are present, angiography or emergent Answer: e. This is a quadriceps tendon rupture. Patients with extensor
surgical exploration is indicated. disruption may have signs and symptoms that include an acute onset
• Soft signs of popliteal artery injury include asymmetric pedal pulses and of pain, swelling, ecchymoses over the anterior aspect of the knee, a
foot or leg paresthesias. Computed tomography (CT) angiography or duplex palpable defect in the patella, quadriceps tendon, or patella tendon loss
ultrasound study is indicated when these are present. or limited ability for active leg extension; extension lag usually is seen
• In the absence of signs of popliteal injury, the knee dislocation patient can when the last 10 degrees of extension are performed haltingly or with
be observed for 24 h, with measurement of the ankle-brachial index (ABI) difficulty. With quadriceps rupture, a low-riding patella (patella baja)
every 3–4 h. An ABI >0.9 over a 24-h period effectively excludes significant and inferior retraction may be seen.
popliteal artery injury. 3. A college football lineman presents complaining of left knee pain
• Tibial plateau fractures should be considered in patients with a traumatic knee and inability to bear weight. He was on the scrimmage line when
injury and inability to bear weight. Fractures may be radiographically occult, so CT the ball was placed in motion and, as he lunged forward, he twisted
imaging should be considered if clinical suspicion is high. In general, if a patient is his leg and heard an audible pop in his knee. He was immediately
unable to bear weight after a knee injury but no acute fracture is identified in the unable to bear weight. On physical examination, you find the left
emergency department (ED), advanced imaging should be considered. knee swollen, with a positive Lachman test. What is the most sensi-
• Extensor mechanism injuries such as patellar and quadriceps tendon inju- tive way to diagnose the injury accurately?
ries are important to recognize, as delayed diagnosis is associated with a. Arthroscopy of the knee
substantial morbidity. A palpable defect in the quadriceps tendon or patel- b. Computed tomography (CT) scan of the knee
lar tendon or over the patella and an inability to perform a straight leg raise c. History and physical examination
should raise suspicion for such injuries. Knee immobilization and urgent d. Magnetic resonance imaging (MRI) of the knee
orthopedic follow-up are advised. e. Plain radiography of the knee
• A dislocated patella will be fixed in a position superolateral to its normal Answer: a. Clinical evaluation is moderately sensitive for anterior
position. Reduction is performed by gentle knee extension with medially cruciate ligament (ACL) tears but, in the acute phase, is often inac-
directed or valgus directed pressure on the displaced patella. curate because of swelling and splinting. Acutely, only plain films
• Injuries to the cruciate and collateral ligaments may not be detectable on are indicated as long as dislocation is not suspected. Tibial plateau
initial examination because of effusion and splinting. Emergent diagnosis fractures detected on plain film may require CT scanning to deter-
is not necessary, but the patient should undergo a follow-up examination mine the need for admission for early operative repair. Arthroscopy
by an orthopedist within a week. is the gold standard for diagnosis of soft tissue injuries of the knee.
• Lipohemarthrosis is an uncommon and subtle sign of occult fracture but MRI is useful but may miss small tears and anatomic abnormalities,
should be sought on radiographs of traumatized knees. and a normal study may still lead to arthroscopy if symptoms per-
• Compartment syndrome is a serious complication of tibial shaft fracture, sist. MRI scanning of the knee is rarely indicated in the emergency
usually occurring 24–48 h after injury. Orthopedic consultation or measure- setting.
ment of compartment pressure should be obtained when the patient’s pain 4. A man presents with severe pain of his right lower leg. He was in
is increasing despite immobilization and support of the fracture. the ED the previous night for splint placement for a tibial fracture.
After removal of the splint, you see a lower leg with mild ecchy-
1. A 45-year-old woman presents after a high-impact motor vehicle mosis, healing abrasions, and palpable posterior tibial and dorsalis
collision with pain in the left knee. She thinks her knee may have pedis pulses. He has severe pain on passive movement of his first
gone out of place but isn’t sure. The distal neurovascular examina- toe and decreased sensation of the first toe web space. Which of the
tion is intact. What is the most appropriate management? following should be the next step?
a. Compression wrap and early range of motion exercises a. Admit for intravenous (IV) antibiotics and pain control.
b. Discharge if radiographs are negative with partial weight bearing b. Obtain venous Doppler scans.
c. Knee immobilizer with orthopedic follow-up c. Resplint the patient.
d. Measure ankle-brachial index and perform duplex ultrasonogra- d. Test the anterior compartment pressures.
phy if less than 0.9 e. Test the superficial posterior compartment pressures.
e. Orthopedic consultation Answer: d. The lower leg is divided into the following compartments by
Answer: d. This patient had a knee dislocation. Knee dislocation is uncom- deep partitions of the investing crural fascia—anterior, lateral, superficial
mon but should be considered in the setting of an appropriate injury posterior, and deep posterior. The anterior compartment contains the tib-
mechanism because 50% of all knee dislocations are reduced sponta- ialis anterior, long toe extensor muscles, anterior tibial artery, and deep
neously before emergency department (ED) arrival. Reduction before ED peroneal nerve, which supplies sensation to the first web space of the foot.
arrival does not lessen the likelihood of vascular injury, and vascular injury 5. A severe medial ankle sprain with no ankle fracture necessitates
should be considered in patients with severe ligamentous injuries and inju- evaluation of which of the following?
ries caused by high-energy mechanisms. Vascular injury to the popliteal a. Cervical spine
artery is the most severe complication and is the major cause of morbidity b. Femoral nerve
and limb loss. Management of suspected knee dislocation involves an algo- c. Lumbar spine
rithm designed to be noninvasive but sensitive for arterial injury. d. Popliteal artery
e. Proximal fibula
2. A 42-year-old man was at the gym performing squatting exercises. Answer: e. An important exception to fibular fractures being stable is
As he was coming to a stand, he had an immediate onset of pain a Maisonneuve fracture (see Fig. 48.13). This involves a medial ankle
in his right thigh. He presents with an inability to extend his knee. disruption (deltoid ligament tear or medial malleolar fracture), with
What is the most likely physical finding? complete tearing of the syndesmosis joining the tibia and fibula and
fracture of the proximal fibula. This results in an unstable ankle mor-
tise because the fibula now floats free relative to the tibia, and surgical
fixation is required.
Ankle and Foot Injuries 49
KEY CONCEPTS 2. A 30-year-old woman complains of pain in the ankle and difficulty
walking after attempting a jump shot while playing basketball. Her
• T he Ottawa Ankle and Foot Rules should be used to evaluate the need for past medical history includes a recent urinary tract infection, for
x-rays in ankle and foot injuries. which she is taking ciprofloxacin. On physical examination, her
• CT scans are indicated for negative x-rays of the ankle and foot when a high ankle joint is not swollen, there is a palpable defect 4 cm proximal
clinical concern of fracture exists. to the posterior ankle, and she has weakened plantar flexion. Which
• Ankle dislocations with obvious vascular compromise should be reduced of the following is true concerning her condition?
promptly, before radiographs are obtained. a. The Boehler angle should be less than 20 degrees.
• The entire fibula should be examined if a medial malleolar fracture is pres- b. In a seated position, squeezing the calf causes foot dorsiflexion.
ent to rule out a proximal fibular (Danis-Weber type C or Maisonneuve) c. It requires a stress radiograph view.
fracture. d. On the radiograph, there may be an opacification at the Kager
• Osteochondral lesions of the talar dome, nondisplaced fractures of the triangle.
lateral posterior process of the talus, and fracture of the anterior process e. Talofibular ligament laxity is present.
of the calcaneus are important to include in the differential diagnoses of Answer: D. The patient has a ruptured Achilles tendon. On examina-
presumed ankle sprains. tion, a visible and palpable tendon defect 2 to 6 cm proximal to the
• Patients discharged with a diagnosis of ankle sprain should be encouraged calcaneal insertion may be present. In some cases of complete Achilles
to seek outpatient follow-up should ankle pain and swelling persist for lon- tendon rupture, weak plantar flexion may still be present because of
ger than 2 weeks. the actions of the tibialis posterior, toe flexors, and peroneal muscles.
• Patients with Achilles tendon rupture are still capable of weak plantar flex- When the Achilles tendon is intact, squeezing the calf muscles should
ion. Ultrasound may be used to confirm the diagnosis if clinical uncertainty cause passive plantar flexion of the foot (Thompson test). Lateral radio-
exists. graphs of the ankle may confirm rupture by showing opacification of
• A Lisfranc injury should be considered with any fracture or dislocation in the triangular fatty tissue–filled space anterior to the Achilles tendon
the tarsometatarsal region, particularly with fractures of the second meta- (Kager triangle) or an irregular contour and thickening of the tendon.
tarsal base. When suspicion for a Lisfranc injury is high, even if plain radio- 3. A 24-year-old patient presents with pain in the left lateral ankle
graphs and CT scans are negative, immobilization and orthopedic referral after a twisting injury. On physical examination, you find no point
are indicated. tenderness on the medial or lateral distal 6 cm of the malleoli, no
• Fifth metatarsal tuberosity (zone 1) fractures should be carefully differenti- proximal fibular tenderness, and no navicular tenderness. Which of
ated from diaphyseal fractures. Orthopedic referral is indicated for zones 2 the following physical findings would necessitate radiographs being
and 3 fractures. taken in the emergency department?
• Stress fracture should be considered in patients with long-standing foot a. Inability to ambulate to the car
pain, particularly if symptoms are in the metatarsal region. This may be b. Laxity on anterior drawer testing
diagnosed by plain radiography, CT scanning, or radionuclide bone scan- c. Moderate swelling with ecchymosis
ning. d. Tenderness at the base of the fifth metatarsal
• Compartment syndrome may occur in any of the four major compartments e. Tenderness over the deltoid ligament
of the foot. Diagnosis is made by high clinical suspicion and measurement of Answer: D. The Ottawa Ankle Rules advise ankle radiographs when
the intracompartmental pressures. any of the following are present: tenderness at the posterior edge of the
distal 6 cm or tip of the lateral malleolus, tenderness at the posterior
1. A middle-aged farmer presents after an accident with a plow with a edge of the distal 6 cm or tip of the medial malleolus, or inability to
crushed open distal tibia fracture. What is the appropriate antibiotic bear weight for at least four steps immediately after the injury and at
choice? the time of evaluation. Foot radiographs in the setting of blunt ankle
a. Cefazolin trauma are advised when any of the following are present: tenderness
b. Cefazolin and gentamycin over the navicular, tenderness at the base of the fifth metatarsal, or
c. Cefazolin, gentamycin, and penicillin G inability to bear weight for at least four steps immediately after the
d. Ciprofloxacin injury and at the time of evaluation.
e. Pen VK and cephalexin 4. A 40-year-old female falls down some stairs and her ankle is forced
Answer: C. Because the patient has a probable soil contaminant in into sudden dorsiflexion and eversion. She reports pain over the
his open fracture, he will need the addition of penicillin G to cover lateral aspect of the ankle and reports a “snapping” sensation with
Clostridium perfringens. For low-energy injuries with mild to moderate active ankle eversion, otherwise the rest of the examination is unre-
contamination, a broad-spectrum cephalosporin is usually sufficient. markable. The plain radiograph of the ankle is negative with the
Heavily contaminated wounds require the addition of gram-negative exception of what appears to be a large avulsed fragment over the
bacterial coverage, typically with an aminoglycoside. Adding penicil- lateral malleolar region. The optimal management of this patient is:
lin G as a third antibiotic is necessary for farm- or soil-related crush a. Reassuring the patient that this is an avulsion fracture and make
injuries. her weightbearing as tolerated.
b. Order a CT scan to evaluate the origin of the avulsed fragment.
c. Orthopedic evaluation in the emergency department.
d. Outpatient dynamic ultrasound and referral to orthopedics.
e. Splinting, nonweightbearing, and outpatient referral to orthope-
dics.
Answer: D. The patient’s mechanism of injury, symptoms, and x-ray 7. A 22-year-old male football player presents with midfoot pain and
findings are consistent with a rupture of the superior peroneal retinac- difficulty weightbearing after being tackled in a college game. You
ulum with peroneal tendon dislocation. If the diagnosis is in doubt, are concerned about a Lisfranc injury, but his plain radiographs
dynamic ultrasound can be used to evaluate the peroneal tendon appear normal. What is the recommended next step in his manage-
because it will show dislocation of the tendon when the foot is dor- ment?
siflexed and everted. Rupture of the superior peroneal retinaculum a. MRI in the emergency department
invariably requires operative repair. b. Attempt weightbearing x-rays or a CT scan, to assess for liga-
5. A 32-year-old male presents after a fall in which his foot was forced mentous disruption.
into dorsiflexion. He has severe pain and swelling on his dorsal foot c. Immobilize in a nonweightbearing splint and refer for follow-up
just anterior to the ankle. In the ED, he is diagnosed with a nondis- with orthopedic surgery.
placed talar neck fracture (Hawkins type 1). His pain is controlled, d. Ultrasound to assess for disruption of the Lisfranc ligament
and he has no associated injuries. What is the disposition for this e. Orthopedics consultation for stress fluoroscopy
patient? Answer: B. Diagnosing low-energy, ligamentous Lisfranc injuries is a
a. Place in a walking boot or a cast, weightbearing as tolerated, and challenge. The American College of Radiology guidelines recommend
follow up with orthopedics in 1 to 2 weeks. an attempt at weightbearing stress views to look for diastasis of greater
b. Orthopedics consult in the emergency department is required than 2 mm between base of the first and second metatarsals, or a differ-
and the patient should be admitted. ence greater than 1 mm between the base of the first and second meta-
c. Immobilize in a padded, noncircumferential splint and arrange tarsals comparing both feet. If weightbearing x-rays are not possible, a
outpatient orthopedics within 48 hours. CT scan is recommended. If clinical concern persists, more advanced
d. Immobilize in a cast and nonweightbearing for 6 weeks, with imaging with MRI or fluoroscopic stress views may be necessary, but
outpatient orthopedics follow-up in 1 to 2 weeks. these can be arranged at the time of specialist follow-up. Ultrasound
Answer: C. Hawkins type 1 fractures are nondisplaced. These fractures has not yet demonstrated utility.
are treated conservatively and are the only talar neck fractures that are 8. A 28-year-old female suffers a twisting injury to her foot during
appropriate for outpatient orthopedic follow-up. Follow-up, however, a fall. She is tender at the base of her fifth metatarsal. Plain radio-
should be urgent in 24 to 48 hours, and the patient should certainly be graphs reveal an acute, nondisplaced fracture at the level of the
immobilized in a well-padded and noncircumferential splint. Depend- intermetatarsal articulation. Which of the following management
ing on local practice, orthopedic consultation in the ED is reasonable. options is incorrect?
6. A 44-year-old male presents after a fall from a ladder in which he a. She can be treated functionally in a supportive shoe and can bear
landed on his right foot. He has severe pain in his heel and is diag- weight as tolerated. Orthopedic follow-up should be arranged.
nosed with an intraarticular calcaneus fracture on plain x-rays. b. This type of injury may be managed surgically. She should be
Which of the following is incorrect in the emergency management referred for orthopedic assessment as an outpatient.
of this patient? c. She should be placed in a weightbearing cast or walker boot for
a. Assessment for other traumatic injuries, particularly spinal 3 months with outpatient follow-up.
injury. d. She can be provided with a walker boot but instructed to remove
b. Once pain and swelling are controlled, the patient may be immo- it when able. Orthopedic follow-up should be arranged.
bilized in a padded splint with orthopedic follow-up arranged e. She can be treated functionally in a supportive shoe if necessary.
for 48 hours. No follow-up is needed
c. The patient should get a CT scan of the foot, if available, in the Answer: E. This is a zone 2 fracture. The treatment of these fractures is
emergency department. evolving and varies according to patient factors. In high-level athletes,
d. Orthopedic surgery should be consulted to assess the patient in primary surgical management may be of benefit. Historically, these
the emergency department. injuries were thought to have a high rate of nonunion and prolonged
e. Place the patient in a well-padded splint with the foot elevated. immobilization has been the standard approach in the average patient.
Answer: B. Patients with intraarticular calcaneus fractures should have More recent data, however, suggest that the rate of symptomatic non-
an orthopedics consultation in the ED. Treatment may be conserva- union is actually quite low, and there is evidence that treating these
tive, but even when operative repair is chosen, the repair may be done patients functionally in supportive footwear, perhaps with initial use
electively at a time when the swelling has decreased. It is important to of a protective boot, is efficacious and more economical. However,
assess these patients for other associated injuries and for patients to as there remains a risk of nonunion, orthopedic follow-up is recom-
be placed in a well-padded splint with the foot elevated to the level of mended for these injuries. Fractures in zone 1 are the only base of fifth
the heart, to decrease swelling. CT imaging aids in classification and fractures that do not require orthopedic follow-up.
decisions about management.
SECTION THREE Soft Tissue Injuries
KEY CONCEPTS Answer: c. Rivulets of fluorescein tracking from the puncture (i.e.,
positive Seidel test) are helpful in identifying intraocular penetration.
• If the history and mechanism of injury are compatible with ocular pene- Ultrasound is, as always, operator dependent, and the pressure of the
tration or if a small puncture wound of the globe is noted, anteroposterior probe on the orbit may risk further injury to an open globe. Checking
and lateral radiographs of the orbit are an appropriate initial step when the for intraocular pressures with an open globe is contraindicated. Com-
foreign body is thought to be radiopaque. Computed tomography (CT) and pared with plain radiographs, CT delivers less radiation to the lens.
ultrasound are complementary diagnostic studies. Multiplanar reconstruction minimizes streak artifacts, affording better
• Although most otic and nasal foreign bodies are amenable to emergency localization of intraorbital objects.
department (ED) removal, instrumentation of these anatomical areas must 2. A 2-year-old presents with purulent drainage from the right naris
be undertaken with great care because removal attempts can cause more and was brought to the ED by a parent concerned about sinusitis.
injury than the foreign body itself. The patient’s vital signs are heart rate 110 beats/min, respiratory rate
• Most airway foreign bodies are seen in pediatric patients and may not be 15 breaths/min, and temperature 37.7°C. On physical examination,
visible on plain films. A normal radiograph does not rule out an aspirated you see a small plastic ball in the naris surrounded by swelling and
foreign body. purulent discharge. What should be the next step?
• The patient with critical airway obstruction and impending or actual respira- a. Attempt to displace the foreign body posteriorly
tory arrest requires one of three options: (1) forced expulsion of the foreign b. Blow air into the contralateral naris to help dislodge the foreign
body; (2) direct laryngoscopy with attempted manual removal with Magill body
forceps; or (3) cricothyroidotomy to bypass an obstruction, or intubation to c. Consult the otolaryngologist for removal under anesthesia
push the foreign body distally into the right mainstem bronchus. d. Make sure you have a right-angle probe, suction, and alligator
• Esophageal foreign bodies are typically found at one of the three con- forceps
striction locations: (1) proximal esophagus at the level of the cricopha- e. Treatment with topical vasodilators to facilitate removal
ryngeal muscle and thoracic inlet—radiographically, the clavicular level; Answer: d. The emergency clinician can remove most nasal foreign
(2) midesophagus at the level of the aortic arch and carina; and (3) distal bodies. Posterior movement of a nasal foreign body risks aspiration;
esophagus just proximal to the esophageal-gastric junction—radio- objects should be removed anteriorly via suction or traction. In some
graphically, a level two to four vertebral bodies cephalad to the gastric circumstances, it may be prudent to place patients in lateral decubitus,
bubble. perhaps with additional Trendelenburg positioning, to help prevent the
• Esophageal foreign bodies (e.g., coins) usually are oriented in the coronal aspiration of objects. Foreign bodies can sometimes be easily removed
plane, and airway objects usually are oriented in the sagittal plane. via positive pressure applied to the patient’s mouth (not the contralat-
• Unless caustic or sharp, foreign bodies in the stomach and bowel are eral naris, which should instead be clamped closed to increase pressure
increasingly managed with a conservative approach with watchful wait- on the involved-side naris). Pretreatment with vasoconstrictive spray
ing and tracking of the object’s progression through the gastrointestinal may improve chances of success. If positive-pressure techniques are not
tract. indicated or do not work, it is important to have necessary instruments
• In the perineal region, foreign body removal tends to be more difficult than close at hand to proceed with foreign body removal attempts. These
anticipated by emergency clinicians and be physically and psychologically instruments include a blunt-tipped right-angle probe (to maneuver
traumatic for patients. Consultation with appropriate specialists (e.g., urol- posterior to the foreign body), suction equipment, and alligator forceps.
ogy or general surgery) is indicated. 3. A 14-month-old girl presents in respiratory distress after eating a
• The most important determinant of successful soft tissue foreign body hot dog for lunch. Her mother states that she had stepped out of the
removal is an understanding of the object’s precise location. kitchen, and when she came back, her daughter was sitting forward
with noisy breathing and obvious distress. Which of the following
1. A 33-year-old construction worker presents with left eye pain. On management interventions is contraindicated?
gross inspection, you note a watery discharge and moderate ery- a. Back blows
thema. The patient tells you she was working with her coworker b. Blind finger sweep as the first step
when she felt something hit her eye. She admits to not wearing pro- c. Chest thrusts
tective goggles. Fluorescein examination of the eye reveals rivulets d. Direct laryngoscopic visualization
of dye tracking from a corneal defect. What is the next appropriate e. Intubate for respiratory distress
step in the patient’s management? Answer: b. Blind finger sweeping has resulted in the conversion of
a. Attempt ultrasound to find a foreign body partial to complete airway obstruction when objects are displaced into
b. Check intraocular pressures the subglottic space. For this reason, the technique has lost favor as
c. Obtain orbital computed tomography (CT) scan and consult an initial maneuver in pediatric and adult patients. It is recommended
ophthalmology that up to five back blows be delivered (with the patient in a head-down
d. Obtain plain radiographs of the orbits position), followed by chest thrusts. Intubation or needle cricothyroi-
e. Topical antibiotics and ophthalmology follow-up in 24 hours dotomy may be performed if other maneuvers fail and circumstances
dictate their need.
4. A 45-year-old man was at dinner when he had a choking episode.
When he recovered, he experienced new-onset wheezing and
presents with the following vital signs: blood pressure 140/90 mm
Hg, heart rate 110 beats/min, respiratory rate 20 breaths/min, and
Answer: a. If a disk battery has been ingested, its location must be
arterial oxygen saturation (Sao2) 92%. On chest radiograph, you see
ascertained with immediate removal if it has lodged in the esophagus.
a flat, fixed diaphragm on the right with a mediastinal shift to the
If the button battery has passed distal to the esophagus, the patient
left and inadequate left-sided expansion. What is the next step in
can be observed, with follow-up radiography to confirm spontaneous
the management of this patient?
passage through the gastrointestinal tract. Nifedipine is occasionally
a. Albuterol nebulizers and steroids intravenously
effective in managing food boluses but should not be used to manage
b. Consult pulmonary for bronchoscopy
nonorganic foreign bodies.
c. CT scan of the chest with intravenous contrast
6. Which of the following is an initial management option for esopha-
d. Needle decompression of the left side
geal food bolus impactions in the ED?
e. Needle decompression of the right side
a. Glucagon
Answer: b. Air trapping and atelectasis are the most common early
b. Nifedipine
clues to airway foreign body presence, with bronchiectasis and bron-
c. Papain
chial stenosis developing later. In air trapping, a comparison of inspira-
d. Sublingual nitroglycerine
tory and expiratory films shows a flat, fixed diaphragm on the involved
Answer: a. Although none of the medical therapies are as good as
side, and the heart and mediastinum shift to the uninvolved side
endoscopy, Glucagon is likely more efficacious than nitroglycerin and
during expiration. When the foreign object is distal to the oropharynx,
nifedipine. Enzymatic degradation of an impacted meat bolus using
however, subspecialty consultation is the safest and most expeditious
the proteolytic enzyme papain, should not be performed because of
means for foreign body removal. As a general rule, early bronchoscopy
risks of esophageal perforation. The gold standard intervention strat-
in any patient with a suspected foreign body is key to reducing mor-
egy for esophageal foreign body removal is endoscopy.
bidity and mortality.
7. What do you expect to see on the chest radiograph of a child who
5. A concerned father brings in his toddler, who has swallowed a but-
has swallowed a coin?
ton battery. Which of the following management strategies is most
a. Foreign body anterior to tracheal air column
appropriate for this patient?
b. Foreign body causing air trapping on the left side
a. Endoscopy is indicated if it is above the lower esophageal sphincter.
c. Visualized flat foreign body in the coronal plane
b. Endoscopy is not indicated if it is found in the esophagus.
d. Visualized round foreign body in the coronal plane
c. Expectant management is appropriate.
e. Visualized round foreign body in the transverse plane
d. If it is in the small bowel, further surveillance is not indicated.
Answer: d. Esophageal foreign objects usually align themselves in the
coronal plane and are posterior to the tracheal air column on lateral
view. Coins in the esophagus lie in the coronal position in virtually all
cases because the opening into the esophagus is much wider in this
orientation.
52 Mammalian Bites
KEY CONCEPTS
• M ammalian bites require an evaluation for both trauma and their risk of
infection. Answer: a. In vitro, P. multocida is sensitive to penicillin, ampicillin,
• Cat and human bites are at higher risk for infection than dog bites. tetracycline, fluoroquinolones, amoxicillin-clavulanate, second- and
• Most mammalian bite wound infections are polymicrobial. Pasteurella spe- third-generation cephalosporins, and trimethoprim-sulfamethoxazole.
cies are the most common pathogens in dog and cat bites. 3. A 19-year-old man presents 10 hours after sustaining a dog attack.
• Preventing infection from mammalian bites relies more on vigilant cleaning, He has multiple lacerations on his head and right hand. What is the
debridement, and irrigation than prophylactic antibiotics. appropriate management of this patient?
• Prophylactic antibiotics are most effective when started within 3 hours of a. Repair the hand and the head lacerations
the bite. The recommended duration of treatment is 5 days. b. Repair the hand laceration but not the head laceration
• Amoxicillin-clavulanate (Augmentin) is the prophylactic antibiotic of choice c. Repair the head laceration but not the hand laceration
for dog, cat, and human bites. d. Use Steri-Strips on all and give him a prescription for amoxicillin-
• The decision to close mammalian bite wounds must weigh the benefit of clavulanate (Augmentin)
improved cosmesis against the risk of wound infection. e. Wound cleansing and bandaging
• Given the risk of infection, mammalian bite wounds to the hand should not Answer: c. For dog bites, the infection rate of hand wounds is as high as
be closed primarily. Most facial bites can safely be closed if done so within 30%, regardless of suturing, whereas the infection rate of dog bites else-
24 hours of the bite. where averages 9%. Similarly, dog bites of the face and neck (including
• Clenched-fist injuries or “fight bites” have high rates of damage to deep punctures) have lower infection rates of 0% to 5% even when they are
structures and infection. sutured. Bite wounds of the face and scalp from any species that are
• Mammalian bites to the hand should receive prophylactic antibiotics. less than 12 hours old may be cleansed well and sutured. Most other
Infected human bites to the hand are treated on an inpatient basis with intra- bite wounds that are going to be closed should have this done within
venous antibiotics, operative debridement, and irrigation. 6 hours.
4. Selected monkey bites require postexposure prophylaxis with
1. A 75-year-old woman presents with a puncture mark on her hand. which of the following medications?
She reports being bitten by her cat the previous night. Which of the a. Amoxicillin clavulanate
following statements regarding this patient’s injury is true? b. Clindamycin
a. Capnocytophaga canimorsus is the organism of concern. c. Flagyl
b. Cats are very clean animals and do not carry any virulent strains. d. Tetracycline
c. Cats produce superficial infections because their teeth are not e. Valacyclovir
long enough to inoculate past the dermis. Answer: e. Valacyclovir to prevent herpes B virus infection. Other
d. Concern exists for a virulent gram-negative bacterium, which terms for this virus include herpesvirus simiae, herpesvirus B, and
can produce a rapid cellulitis. monkey B virus. This virus has serologic cross-reactivity with herpes
e. Irrigation and topical antibiotics are indicated. simplex virus (HSV) type 1 and type 2, which cause herpetic lesions
in humans. Seventy-five to 100% of monkeys of the genus Macaca
Answer: d. Cats have long, slender, pointed teeth that can penetrate
(macaques) used for biomedical research are seropositive for the
tendons, joints, and bone, inoculating bacteria deep into these tissues.
B virus. Other antibacterials may be needed if cellulitis infection
Cat bites have a substantially higher risk of infection than dog bites do.
ensues.
Another important factor in the development of wound infection after
cat bites involves the presence of Pasteurella multocida, a highly viru-
lent, gram-negative, facultatively anaerobic rod found in the oral cavity
or nasopharynx of 70% to 90% of healthy cats.
2. What is the most appropriate antibiotic prophylaxis for a cat bite?
a. Amoxicillin-clavulanate
b. Clindamycin
c. Erythromycin
d. First-generation cephalosporins
e. Vancomycin
53 Venomous Animal Injuries
KEY CONCEPTS Answer: d. The symptoms are typical for a pit viper snakebite. The
most consistent symptom associated with pit viper bites is immediate
• S nake venom causes neurotoxicity and hematotoxicity, but one usually pre- burning pain in the area of the bite, whereas pain may be minimal with
dominates, depending on the species of snake. bites of Elapidae and other exotic snakes. Tetanus prophylaxis may be
• Pit vipers have triangular or arrow-shaped heads, elliptical pupils, and char- indicated, but antibiotics are not. Fasciotomy is rarely if ever indicated
acteristic pits found bilaterally midway between the eye and the nostril. for snakebite. A constricting band may be useful for first aid in the field
• The amount of crotalid antivenom given depends the severity of the bite. with certain neurotoxic snakes, but not in North America. Immobili-
Currently, Crotalidae polyvalent immune Fab (CroFab) and F(ab′)2 (Anavip) zation may be helpful, but ice and antibiotics are not.
are the antivenoms of choice for crotalid bites. Children require the same 3. A 43-year-old woman sustained a snakebite while in her backyard
amount of antivenom as adults. in Florida. She was not able to secure the snake, but she remem-
• Envenomation from exotic international snakes such as cobras, kraits, and bered it to be colorful. She presents with ptosis, slurred speech, and
mambas often require antivenom for life-threatening neurologic and hema- nausea. What is true about the type of snake most likely involved in
tologic toxicity. this case?
• Arthropods such as hymenoptera account for more deaths from envenom- a. Death usually occurs from coagulopathies.
ation than snakes, usually as a result of allergic or anaphylactic reactions. b. Eastern species are the most venomous.
• Black widow spider bites are neurotoxic, causing severe pain and mus- c. The snake has a heat-sensitive organ between eyes and nostrils
cle spasms; an antivenom is available but used only in severe cases. Brown on both sides of the head.
recluse spider bites cause necrotizing skin wounds. d. The snake has a triangle-shaped head.
• Nematocyst (jellyfish) stings should be neutralized with vinegar or hot e. The snake has elliptical pupils.
water, and fish stings with hot water. Stings from venomous fish (e.g., lion- Answer: b. This woman sustained a bite from a coral snake. All of
fish and sting rays) are treated with circulating hot water to denature the the above except B are related to pit vipers. The coral snake can be
toxin and cause vasodilation. differentiated from the king or milk snake by two characteristics:
• Marine antivenoms are available for severe box jelly fish, stonefish, and sea the nose of the coral snake is black, and the red and yellow bands are
snake envenomation. adjacent on the coral snake. Eastern coral snake is considered more
deadly as opposed to its western relative. There are no records of
1. Which of the following venomous snakes in the United States is a fatalities caused by the western species. Ptosis is common and often
member of the neurotoxic Elapidae family? the first outward sign of envenomation. Other signs and symptoms
a. Copperheads include vertigo, paresthesias, fasciculations, slurred speech, drows-
b. Coral snakes iness, dysphagia, increased salivation, nausea, and proximal muscle
c. Rattlesnakes weakness.
d. Water moccasins 4. Which of the following is true about snake antivenom?
Answer: b. Pit vipers from the family Viperidae are the most prevalent a. Antivenom may reverse symptoms of envenomation including
venomous snakes in the United States. They are native to every state laboratory abnormalities.
except Maine, Alaska, and Hawaii. They are classified into three main b. Antivenom should be administered around the wound.
groups: true rattlesnakes (genus Crotalus), copperheads and water c. Even mild envenomations require antivenom.
moccasins (genus Agkistrodon), and pygmy and Massasauga rattle- d. Exotic snakebites require only antivenom if neurologic symp-
snakes (genus Sistrurus). Pit vipers account for 98% of all venomous toms are present.
snakebites in the United States. Other families include Colubridae, e. Pregnancy is a contraindication to receive antivenom.
Hydrophiidae, Elapidae (to which the neurotoxic coral snake belongs), Answer: a. Pregnancy is not a contraindication to antivenom ther-
and Crotalidae. apy. Any victim of a venomous snakebite with moderate or severe
2. A 30-year-old man complains of pain, burning, and swelling of envenomation is a candidate for antivenom. Antivenom may reverse
his hand after gardening. Physical examination shows two punc- the local signs, systemic symptoms, and laboratory abnormalities
ture marks on his thenar eminence, as well as moderate localized associated with envenomation. As with coral snakes, many patients
swelling. When you inquire about it, he said that he did have some do not show any early signs after envenomation by exotic snakes. The
sudden burning pain while digging around in the ground by a bush, antivenom should be administered before neurologic changes develop.
but he dismissed it as abrasions from sharp sticks. That was 2 hours All antivenom should be administered intravenously.
ago. What should you do next? 5. A 28-year-old man presents with a copperhead snakebite that
a. Apply a constricting band to impede venous and arterial flow. occurred 1 hour ago. He complains of pain and moderate swelling
b. Discharge with instructions for rest, ice, immobilization, and use of his right hand and wrist. He has no systemic symptoms and has
of oral antibiotics. a pulse of 80 bpm and a normal blood pressure. His initial coagula-
c. Give tetanus and clindamycin for prophylaxis. tion studies are normal. What should be done?
d. Obtain baseline laboratory studies and observe the patient for a. Admit for observation and antivenom
increasing symptoms. b. Discharge with elevation, ice, and pain medications
e. Test compartment pressure and prepare for fasciotomy. c. Give antivenom if swelling spreads to forearm
d. Immediate antivenom, four vials intravenously
e. Observe 12 hours for signs of increasing envenomation
Answer: e. The majority of copperhead bites do not require antivenom. Answer: a. Stonefish, a type of bony fish, may cause serious cardiac and
Some toxicologists will administer antivenom if extremity swelling respiratory symptoms, which can be prevented by early administration
is progressive and may cause long-term disability. Copperhead bites of the appropriate antivenom. The fish spines of bony fish should be
generally cause a moderate amount of swelling that may peak 24 to removed with forceps because they are thick and less likely to break off
36 hours after the bite. All venomous snakebites should be observed at the skin (like a bee stinger). In all cases, the wound should be copi-
for at least 12 hours if signs of envenomation are present. If systemic ously irrigated. Vinegar has been shown to be useful for some types
signs develop or the patient develops a coagulopathy, antivenom may of nematocyst injuries from jellyfish. Significant analgesia is achieved
be indicated. by submersion of the wound in hot water for 30 to 90 minutes or until
improvement. There is a stonefish antivenom in limited supply for
6. Which of the following patients is most likely to be the first dis-
severe envenomation.
charged home safely?
8. A patient presents with a necrotic lesion on his midthigh that started
a. A pregnant woman with black widow envenomation
as a bleb while he was working in his woodshed 3 days ago. The
b. A 3-year-old child with a scorpion sting 1 hour before arrival
wound has grown gravitationally. Her blood pressure is 110/80 mm
c. A 55-year-old man with hypertension and coronary artery dis-
Hg, respiratory rate 16 rpm, heart rate 110 bpm, and temperature
ease with black widow envenomation
38.3°C. You suspect an envenomation of which of the following?
d. An 8-year-old with a coral snake bite
e. An 18-year-old with an unknown snakebite 8 hours before a. Centruroides exilicauda
arrival b. Hapalochlaena maculosa
Answer: e. A scorpion sting in a child should be observed for at least c. Haplopelma lividum
6 hours. Symptomatic children with stings should be admitted. Most d. Latrodectus mactans
venomous snakes will show signs of envenomation with 6 hours. If e. Loxosceles reclusa
this patient is asymptomatic, it is likely that the snake was nonvenom- Answer: e. The cobalt blue tarantula, Haplopelma lividum, is an aggres-
ous or this was a “dry bite.” All children with envenomation and coral sive spider with toxic venom. The black widow spider is Latrodectus
snake bites should be admitted for observation. Pregnant patients and mactans. Centruroides exilicauda, which is found in Arizona, is a
those with coexisting medical problems should be admitted after black particularly dangerous kind of scorpion. The blue-ringed octopus is
widow envenomation. Hapalochlaena maculosa. The brown recluse spider, Loxosceles reclusa,
7. While walking in shallow water, a patient accidentally steps on a causes an initial white area of vasoconstriction at the site of the bite
stonefish. Which of the following is the least effective diagnostic within 3 or 4 hours. A bleb then forms in the center of this area, and
and treatment intervention? an erythematous ring arises on the periphery. The lesion at this stage
a. Irrigating the wound with vinegar resembles a bull’s-eye. The bleb darkens, necroses during the next sev-
b. Obtaining radiographs for retained foreign body eral hours to days, and continues to spread slowly and gravitationally.
c. Observing for cardiovascular and respiratory symptoms
d. Removing the spine with forceps
e. Using hot water to relieve the pain
Thermal Injuries 54
e. 4200 mL LR solution in the first 4 hours
KEY CONCEPTS Answer: c. The amount of LR solution required for the first hour
• A fter removing the patient from the source of injury, burns should be cooled can be rapidly estimated with the Parkland formula by multiply-
with room temperature water while avoiding hypothermia in patients with ing the estimated total body surface area (TBSA) of the partial and
large burns. full thickness burn (55%) by body weight in kilograms (80 kg) and
• Clinical signs such as facial burns, hoarseness, drooling, carbonaceous spu- dividing by 4.
tum, and singed nasal hairs indicate inhalation injury; however, they are 4. A 25-year-old woman presents with a partial thickness burn to her
poor predictors of injury severity. right forearm after a grilling accident. You note areas of gray dis-
• Confirmation of inhalation injury is best accomplished by direct visualiza- coloration with decreased blanching in the erythematous region of
tion of the glottic inlet via flexible or rigid video laryngoscopy augmented the burn. In addition to irrigation, debridement, and dressing with
by topical anesthesia and judicious sedation, when necessary. a nonadherent dressing, which of the following would be the most
• Worsening hoarseness, edema, or soot in the supraglottic region necessi- appropriate treatment?
tates immediate intubation. a. Apply silver sulfadiazine and follow-up with plastic surgery in 1
• Supplemental oxygen should be given in patients with suspected inhalation week
injury and determination of carbon monoxide levels should be performed. b. Calculate the Parkland formula and administer fluids before dis-
• Crystalloid resuscitation is required to support vital organ perfusion and is charge
guided by urine output. Care should be taken to avoid over-resuscitation. c. Educate the patient about daily dressing changes and have her
• Pain relief is required for larger burns and is accomplished through both follow-up with plastic surgery in 24 to 48 hours
pharmacological and non-pharmacological methods. Pain levels should be d. Immerse her forearm in ice water for pain control
assessed frequently. e. Unroof soft blisters and have the patient follow-up with her pri-
• Patients with large body surface area burns (>20% for adults and >10% for mary care physician in 1 week
children and the elderly) or deep burns will require admission to a burn cen- Answer: c. The distinction between superficial and deep partial thick-
ter. Most other patients presenting to the ED will have small and superficial ness burns is important in that deep partial thickness burns often do
burns that can be managed with over-the-counter or commercially available not heal within 2 or 3 weeks and may result in severe scarring and
topical agents. contractures, especially in children. As a result, deep partial thickness
burns that do not heal within 21 days may require excision and skin
1. A 30-year-old man presents with a burn to both anterior aspects of
grafting to minimize scarring. Deep partial thickness burns may also
his forearms after being burned by a radiator. He has severe pain
progress to full thickness burns during the course of several days after
and has deep (reticular) dermis extension of the burn. You call the
injury. Burns less than 20% TBSA can be treated with oral hydration.
consultant and describe the burn as which of the following?
Although there are little data to guide treatment, blisters are generally
a. Superficial burn of 4.5% body surface area
left intact initially if possible unless tense or over a joint. They may later
b. Superficial partial thickness burn of 9% body surface area
require debridement.
c. Deep partial thickness burn of 4.5% body surface area
5. Which of the following is an indication for intubating a patient who
d. Full thickness burn of 2.5% body surface area
was found in a burning house and requires transfer 2 hours away to
e. Deep full thickness burn of 9% body surface area
a burn center?
Answer: c. Deep partial thickness burns extend through the epidermis
a. Soot in the airway and singed nasal hair
into the deep (reticular) dermis. Body surface area is determined by
b. Dyspnea and hypoxia
the rule of nines. Nine percent for each upper extremity means that
c. Patient unable to handle own secretions
the forearm is approximately one fourth of 9%. Two forearms burns
d. Altered mental status
are half of 9%. Partial thickness burns are often more painful than full
e. All of the above
thickness burns, in which all of the nerve endings are destroyed.
Answer: e. Traditionally, inhalation injury was diagnosed on the
2. A 3-year-old boy presents with circumferential burns involving
basis of clinical findings, such as facial burns, singed nasal vibrissae,
both upper extremities, including his hands, from pulling a boiling
carbonaceous sputum, and a history of injury within a closed space.
pot of water off the stove. The burns are mixed partial thickness and
However, these findings are neither highly sensitive nor highly spe-
full thickness burns. Which of the following best describes the body
cific. If the patient is at a burn center and symptoms are mild, then an
surface area burned and the most appropriate disposition?
approach involving flexible laryngoscopy and close observation in an
a. 9% and admit to pediatrics with surgery consultation
ICU is reasonable. However, if the patient requires prolonged transport
b. 9% and transfer to burn unit
to a tertiary care center then intubation is recommended if signs or
c. 18% and transfer to burn unit symptoms or airway involvement are present.
d. 18% and admit to pediatric surgery
6. What is the most appropriate management for a superficial partial
e. 20% and admit to pediatric intensive care unit
thickness burn on the forearm?
Answer: c. Children with burns over 10% total body surface area
a. A clean dry dressing, such as gauze
(TBSA) should be transferred to a burn unit. In addition, hand burns
b. Petroleum gauze
should be treated at a burn unit. Circumferential burns are not con-
c. Systemic antibiotics and silver sulfadiazine
sistent with a splash injury, and child abuse should be suspected and
d. Topical antibiotic ointment
reported.
e. B and D
3. A 55-year-old, 80-kg man presents with partial thickness burns of Answer: e. Superficial partial thickness burns may be treated with a
both his legs, front torso, and groin. What is the initial fluid resusci- topical antibiotic ointment, petroleum gauze or one of several com-
tation according to the Parkland formula? mercially available burn dressings. Silver sulfadiazine, as well as dry
a. 500 mL LR solution in the first 4 hours dressings, will slow re-epithelialization. Silver sulfadiazine is appropri-
b. 1100 mL normal saline in the first hour ate for infected or heavily contaminated burns. Systemic antibiotics are
c. 1100 mL of LR solution in the first hour not indicated for non-infected burns.
d. 2400 mL normal saline in the first hour
55 Chemical Injuries
KEY CONCEPTS 2. A 20-year-old male presents with periorbital pain and swelling after
a blow to the eye by a softball. Physical examination reveals propto-
• P rophylactic topical antibiotics are not indicated for the treatment of cor- sis with significant decline in vision and limitation of ocular motion
neal abrasions, and eye patches are not recommended because they can in all planes. Tonometry reveals an intraocular pressure (IOP) of
mask a worsening infection. 45 mm Hg. Which of the following should be the first indicated
• Eyelid lacerations that may require referral to a plastic or ophthalmic sur- maneuver?
geon include those with lid margin lacerations, a canalicular laceration, or a. Acetazolamide 500 mg IV, mannitol 20 g IV, and topical timolol
levator or canthal tendon injuries. b. Computed tomography (CT) scan of the head and face
• Alkaline burns to the cornea and conjunctiva need to be irrigated until a c. Endotracheal intubation and hyperventilation
neutral pH is attained because they produce a liquefactive necrosis that d. Immediate lateral canthotomy and cantholysis
penetrates and dissolves tissue. e. Ophthalmologic consultation
• Admit hyphema patients with sickle cell trait, uncontrolled elevations in Answer: d. These findings should make one suspect retrobulbar hem-
intraocular pressures (IOPs), hyphema of greater than 50%, and when there orrhage. All of these interventions are likely indicated. Intraocular
is concern for rebleeding. hypertension may compromise central retinal artery flow. Although
• Avoid manipulation, palpation, or tonometry on a suspected globe rupture immediate ophthalmologic consultation and pressure-lowering
pending an ophthalmology consult. maneuvers are indicated, lateral canthotomy and cantholysis will pro-
• Scleritis, an autoimmune inflammatory process involving the sclera, can be vide the most rapid temporizing measure to preserve vision.
confused with episcleritis caused by inflammation in the more superficial 3. A 43-year-old male presents with acute ocular pain after a splash injury
episcleral layer of the eye. from drain cleaner. What should be the sequence of interventions?
• Episcleritis, unlike scleritis, is associated with much less discomfort, a a. Copious irrigation for 10 minutes, pH testing, cyclopentolate cyclo-
pinker and more pronounced peri-limbal injection from superficial episcleral plegia, topical antibiotics/intraocular pressure (IOP) measurement
vessels that—unlike the deeper injected scleral vessels in scleritis—will b. Intravenous (IV) analgesia, cyclopentolate cycloplegia, IOP
vasoconstrict and blanch with 10% phenylephrine. Treatment of both measurement, isotonic irrigation
involves topical corticosteroid drops. c. IOP measurement, analgesia, head-up position, cycloplegia
• Endophthalmitis is an infection of the eye itself, typically following intraoc- d. Phenylephrine cycloplegia, isotonic irrigation for 10 minutes,
ular surgery, that is treated with intravitreal antibiotics. pH testing, slit-lamp examination for foreign bodies
• Herpes zoster keratoconjunctivitis can complicate herpes zoster ophthal- e. Phenylephrine cycloplegia, slit-lamp examination for foreign
micus. Emergent ophthalmologic consultation and treatment with systemic bodies, isotonic irrigation for 10 minutes, pH testing
antiviral agents are required. Answer: a. Copious irrigation, ideally beginning at the scene, is the
• Treatment of acute angle-closure glaucoma requires both a reduction cornerstone of management. Nitrazine pH testing after 10 minutes
in the production and an increase in the outflow of aqueous humor (see should guide the need for continued irrigation. Cycloplegia, IOP mea-
Box 57.1). surement, and topical antibiotics come after pH normalization. Phen-
• With anisocoria, the following considerations help in determining which ylephrine is contraindicated for cycloplegia in these cases because of its
pupil—the larger or the smaller—is the pathological one: (1) parasym- vasoconstrictive properties.
pathetic innervation constricts a pupil in bright light, whereas sympa- 4. A 17-year-old girl who wears contact lenses presents with a 24-hour
thetic stimulation helps dilate a pupil in the dark; (2) an abnormally history of right eye pain. Physical examination reveals a right cor-
small pupil may therefore be due to either a decrease in sympathetic neal abrasion at the 6-o’clock position of the limbus. Appropriate
stimulation or an augmentation of parasympathetic stimulation—but treatment consists of which of the following?
likely the former (e.g., Horner syndrome); (3) an abnormally large pupil a. Cessation of contact lens wear, eye irrigation (qid) with isotonic
may therefore be due to either a decrease in parasympathetic stimu- saline solution, followed by instillation of undiluted topical tet-
lation or an augmentation of sympathetic stimulation—but likely the racaine for 5 days
former (e.g., partial third-nerve palsy from compression, Adie’s pupil, b. Emergent ophthalmology consultation
pharmacological mydriasis); or (4) the abnormally small pupil will usu- c. Tetanus prophylaxis, eye patching for 48 hours, antibiotic oint-
ally look worse in the dark, whereas the abnormally large pupil will ment, and a 24-hour recheck
usually look worse in the light. d. Tetanus prophylaxis, topical nonsteroidal antiinflammatory
drugs (NSAIDs), cessation of contact lens wear, and a 24-hour
1. A 23-year-old male presents with left periorbital pain after being recheck
struck with a fist. On examination, there are no globe injuries but e. Topical nonsteroidal medications, topical anti-pseudomonal
marked periorbital swelling is noted. Computed tomography (CT) antibiotic, and a 24-hour recheck
of the face reveals an orbital floor fracture. Which of the following Answer: e. Tetanus prophylaxis is not indicated for corneal abrasion
would be the most likely physical findings? unless there is corneal perforation or contamination with organic
a. Cheek anesthesia, enophthalmos, and limitation of upward gaze material. Topical NSAIDs reduce corneal abrasion pain. Antipseudo-
b. Cheek anesthesia, ptosis, and limitation of inferior gaze monas coverage with cessation of contact lens wear is appropriate. Eye
c. Forehead anesthesia and afferent papillary defect patching is not indicated. Administration of undiluted topical anes-
d. Forehead anesthesia, diplopia, and limitation of lateral gaze thetics for more than 24 hours is untested and may be dangerous. Oral
e. Ptosis, miosis, and ipsilateral anhydrosis analgesics may be needed.
Answer: a. An orbital floor fracture may entrap the inferior rectus and
inferior oblique muscles, resulting in diminished upward gaze. Other
findings may include ptosis, enophthalmos, ipsilateral cheek/lip anes-
thesia, and orbital emphysema. As many as 25% of such patients have
associated globe injuries. Option E describes Horner syndrome, which
is not a typical finding.
5. How do patients with subconjunctival hemorrhage most commonly 8. A 48-year-old woman presents with right eye pain, photophobia,
present? and decreased vision after a motor vehicle collision (MVC). Phys-
a. Asymptomatic blood in the eye, noticed in the mirror or by a ical examination reveals an irregularly shaped pupil and a small
friend hyphema. Photophobia, decreased acuity, minimal pupil reactivity,
b. Decreased visual acuity and bloody chemosis are seen on examination. What is the most
c. Foreign body sensation likely diagnosis?
d. Modest pain a. Acute angle-closure glaucoma
e. Photophobia b. Blunt ciliary injury
Answer: a. Any significant symptoms, such as pain, decreased vision, c. Iridodialysis
foreign body sensation, or photophobia, should spark the search for d. Scleral rupture
more serious pathology. Bilateral hemorrhage in the absence of a clear e. Traumatic miosis
cause (e.g., severe vomiting) should raise suspicion for coagulation Answer: d. Scleral rupture occurs either at the insertion of the extra-
issues. ocular muscles or at the limbus, where the sclera is the thinnest. A
6. A 38-year-old man presents with unilateral left-sided visual loss “teardrop” pupil is often seen and may be accompanied by bloody
after a motor vehicle collision (MVC). The only clinical finding is a chemosis or severe subconjunctival hemorrhage. Brownish-black pig-
left-sided hyphema rising to 50% of the height of the anterior cham- ment prolapse may also be seen. Intraocular pressure (IOP) may be
ber. Intraocular pressure (IOP) is 17 mm Hg in the unaffected eye low, but tonometry is generally contraindicated in cases of suspected
and 29 mm Hg in the affected eye. Appropriate management should globe injury.
include which of the following? 9. A 26-year-old man presents with a 3-day history of right eye pain,
a. Cycloplegia, intravenous (IV) mannitol, ophthalmology consul- decreased vision, and photophobia. He reports a history of left eye
tation trauma 6 weeks prior, with hyphema, traumatic iritis, and persistent
b. IV analgesia and antibiotic, immediate ophthalmologic consul- decreased vision. He is otherwise healthy. Physical examination
tation for decompression resulting from intraocular hyperten- reveals photophobia in the right eye with bilateral decreased vision.
sion Before the past 3 days, the vision in the right eye had been perfect.
c. Oral acetazolamide, shield, antiemetics, 24-hour recheck What is the most likely explanation for his right eye symptom?
d. Topical beta-blocker, shield, modest analgesia, admission a. Collagen vascular disease
e. Topical beta-blocker, topical nonsteroidal antiinflammatory b. Post-traumatic conjunctivitis
drugs (NSAIDs) for pain, shield, 24-hour recheck c. Post-traumatic retinal tear
Answer: d. Significant hyphema is an indication for admission. The d. Spontaneous vitreal hemorrhage
presence of elevated IOP requires urgent treatment (which might also e. Sympathetic ophthalmia
include topical alpha agonists or IV acetazolamide, and so on), shield, Answer: e. Sympathetic ophthalmia is an autoimmune inflammatory
elevation of the head, and cautious use of systemic analgesics. Any response in the unaffected eye, days to months after uveal trauma in the
form of platelet inhibition would be contraindicated. opposite eye. Pain, photophobia, and decreased vision are common.
7. What is the major complication of hyphema? This patient had no findings consistent with conjunctivitis or collagen
a. Detached retina vascular disease, and a retinal tear would not typically be painful.
b. Glaucoma 10. Oral antibiotics are indicated for which of the following?
c. Horner syndrome a. Blepharitis
d. Rebleeding b. Chalazion
e. Vitreous hemorrhage c. Dacryocystitis
Answer: d. Rebleeding typically occurs 2 to 5 days later as the clot d. Endophthalmitis
retracts. It is most common in patients with elevated intraocular pres- e. Hordeolum
sures (IOPs), hyphema greater than 30% of the anterior chamber, and Answer: c. Dacryocystitis is an infection of the lacrimal sac from naso-
with delayed presentation. Rebleeding may lead to glaucoma and syn- ciliary duct obstruction. Warm compresses are also recommended and
echia formation. may be helpful, although evidence is lacking. Warm compresses and
topical antibiotics are appropriate for the other conditions. Intravitreal
antibiotics are indicated for endophthalmitis.
11. Bedside ocular ultrasonography can provide useful information
for which of the following conditions?
a. Lens dislocation
b. Retinal detachment
c. Vitreous hemorrhage
d. All of the above
Answer: d. A displaced lens can be seen in the relatively hypoechoic
vitreous. Vitreous hemorrhage and retinal detachment can both be
diagnosed with emergency department bedside ultrasonography.
Otolaryngology 58
KEY CONCEPTS
Otitis Media Mastoiditis
• Otitis media is caused by eustachian tube inflammation and dysfunction • Mastoiditis is the most common suppurative complication of otitis media,
leading to fluid accumulation in the middle ear and proliferation of nasopha- though it is a relatively rare condition. It may occur in patients without known
ryngeal bacteria, most commonly Streptococcus pneumoniae, Haemophilus preceding otitis media.
influenzae, and Moraxella catarrhalis. • The most common physical findings with mastoiditis are postauricular ery-
• Diagnosis of acute otitis media (AOM) is made based on the presence of a thema and tenderness, protrusion of the auricle, and an abnormal TM.
middle ear effusion and signs of inflammation, including ear pain, tympanic • CT scanning is indicated in patients with suspected intracranial involvement
membrane erythema and bulging, and fever. from mastoiditis.
• Bullae may be seen in some cases of AOM. This is known as bullous • Mastoiditis is treated with vancomycin (15 mg/kg IV), and an agent active
myringitis and treatment for this condition does not differ from non-bul- against Pseudomonas aeruoginosa is added in those who have previously
lous AOM. received antibiotics.
• Mastoiditis is the most common suppurative complication of AOM though is • Consultation with an otolaryngologist is advised to assist in management of
rare in the modern era. mastoiditis.
• Most cases of AOM resolve spontaneously, and observation for 2 to 3 days
may be offered instead of immediate antibiotics in appropriately selected Sudden Hearing Loss
pediatric patients. Observation strategies are not validated in adult • Sudden sensorineural hearing loss (SSNHL) is defined as acute hearing loss
patients. over a period less than 3 days and is often misdiagnosed.
• Amoxicillin is the recommended initial antibiotic treatment of choice for most • SSNHL is idiopathic in 70% of cases. It may also be secondary to infection,
patients with AOM. Recurrent otitis media or treatment failures warrant otologic disease, trauma, vascular disease, hematologic disorders, or neoplasm.
selection of an alternate antibiotic regimen. • Imaging is not warranted in the ED for SSNHL unless a space-occupying
• Antibiotics alone do not provide improvement in pain in the first 24 hours of lesion is suspected.
AOM, and pain should be addressed with the use of over-the-counter analge- • SSNHL is treated with steroids (prednisone 1 mg/kg/day up to 60 mg, tapered
sics, such as acetaminophen or ibuprofen. over 10 to 14 days), and prompt surgical follow up for further evaluation.
• Otitis media with otorrhea from an acute perforation is treated in the same
manner as AOM without perforation. These perforations are typically small Epistaxis
and resolve spontaneously. • Epistaxis is distressing to patients but rarely life-threatening. Most nose-
• In patients with tympanostomy tubes presenting with acutely increased otor- bleeds arise from an anterior locus.
rhea, treatment with ototopical fluoroquinolone drops is recommended. • A causative link between hypertension and epistaxis has not been clearly
established; however, patients are often hypertensive upon presentation,
Otitis Externa which may be associated with persistent bleeding.
• Otitis externa (OE) is most commonly seen in warm climates and summer • Most cases of anterior epistaxis can be managed in the ED with a combina-
months. Pseudomonas aeruginosa and Staphylococcus aureus are the most tion of direct pressure, topical vasoconstrictive drugs, chemical cautery and
common causative pathogens. packing, as needed. Tranexamic acid is an effective adjunct to these treat-
• Patients with immunocompromised states (diabetes) and persistent OE despite ments that decreases immediate and delayed bleeding.
treatment should be evaluated for necrotizing OE, a severe, life-threatening • Posterior epistaxis requires specialized packing techniques and monitoring in
infection of the outer ear and surrounding structures. an inpatient setting.
• Topical antibiotic drops are effective for most cases of OE. Systemic antibiot- • Refractory cases of epistaxis may require embolization or surgical treatment
ics are not indicated in immunocompetent patients with infection limited to • Routine use of prophylactic antibiotics in patients treated with anterior pack-
the external ear canal. ing for epistaxis is not recommended.
• Fluoroquinolone drops are effective in most cases of OE, and they may be
Sialolithiasis
used in patients with a non-intact tympanic membrane. Aminoglycoside ear
• Sialolithiasis is most commonly seen in the submandibular salivary gland. Risk
drops are ototoxic and should not be used in patients with a tympanic mem-
factors include dehydration, anticholinergic and diuretic medications and smoking.
brane (TM) perforation.
• Sialolithiasis may be treated with manual massage of the gland, sialogogues
Necrotizing (Malignant) External Otitis and anti-inflammatory pain medications.
• Necrotizing external otitis is an aggressive from of OE with high associated • CT scan is the preferred imaging modality for suspected sialolithiasis and can
mortality. Patients with diabetes mellitus, immunocompromise, and advanced identify stones and evidence of infection.
age are at particular risk. • Sialadenitis is an associated infection of the salivary glands, and antibiotics
• Necrotizing otitis media should be considered in at-risk patients with progres- such as amoxicillin/clavulanate or clindamycin should be added to the treat-
sive OE unresponsive to treatment. ment regimen.
Neck Masses
• CT scanning can aid in diagnosis of necrotizing otitis media by demonstrating
• Neck masses in children are likely inflammatory or benign congenital abnor-
bony erosion and characteristic soft-tissue abnormalities.
malities. In adults, neck masses commonly represent malignancies.
• Necrotizing otitis media is treated with systemic antibiotics with activity
• Physical examination findings such as stridor, dysphagia, voice changes
against Pseudomonas aeruginosa. Ciprofloxacin is an appropriate initial anti- and referred pain can help localize symptoms and guide
biotic treatment. diagnostic approach.
• Patients with necrotizing otitis media should be managed in coordination with • CT scan with contrast is the preferred initial imaging modality to characterize
an otolaryngologist. Surgical intervention may be needed in refractory cases. the neck mass and involvement of surrounding structures.
• Patients with airway compromise secondary to a neck mass warrant emer-
gent airway management and ENT evaluation.
• Patients with hoarseness lasting greater than 2 weeks and patients with neck
masses unresolved within 2 weeks should be evaluated in follow-up by an
otolaryngologist.
1. Bullous myringitis is most commonly caused by which of the follow- duration. On ear examination, granulation tissue is noted in the
ing bacterial pathogens? external auditory canal. The patient has facial asymmetry, which
a. Streptococcus pneumoniae her family states developed 2 days ago. What is the most appropri-
b. Pseudomonas aeruginosa ate imaging modality for this patient in the ED?
c. Mycoplasma pneumoniae a. MRI with contrast
d. Staphylococcus aureus b. CT with contrast
Answer: a. Bullae may be seen on the TM in up to 5% of cases of AOM c. Skull x-ray
in young children. While this finding has been historically associated d. Transcranial doppler ultrasound
with M. Pneumoniae, the bacterial pathogenesis of bullous myringitis Answer: b. This patient is presenting with findings consistent with nec-
is similar to that of non-bullous AOM and is treated similarly. rotizing external otitis: granulation tissue in the EAC and facial paralysis.
2. Which of the following patients with acute otitis media would be CT scan is the most appropriate choice for imaging in the ED and will
appropriate candidates for observation in lieu of immediate antibi- demonstrate bony erosion and associated soft-tissue abnormalities. MRI
otic treatment? and nuclear medicine studies may be helpful later in the patient’s course.
a. A 6-month-old female with bilateral middle ear effusions with 4. Routine use of prophylactic antibiotics in patients treated with ante-
red bulging tympanic membranes and fever to 103.5°F rior packing for epistaxis has clearly demonstrated which of the fol-
b. A 6-year-old male with three days of ear pain, fever to 101.5°F lowing benefits?
and an erythematous, bulging TM with purulent material seen a. Prevention of toxic shock syndrome
in the external ear canal b. Reduction in incidence of acute otitis media
c. A 2-year-old male without significant medical history present- c. Prevention of acute sinusitis
ing with 1 day of right ear pain and fever to 100.5°F. His exam d. None of the above
reveals an erythematous TM with a middle ear effusion. There is Answer: d. Administration of prophylactic antibiotics in patients dis-
no otorrhea or TM perforation noted. charged with anterior nasal packing has not been shown to prevent
d. A 5-month-old female with fever to 101.2°F with an erythema- the listed infectious complications and is not recommended in patients
tous, bulging left TM seen on otoscopy. without immunocompromise, valvular disorders, or in those whose
Answer: c. Observation for 48 to 72 hours can be considered in lieu packing will be left in place for less than 48 hours.
of immediate antibiotics in pediatric patients aged 6 months to 2 years 5. Neck masses in pediatric patients are more likely to represent an
with unilateral AOM of <48 hours duration, in the absence of severe ____________ process, while neck masses in adult patients are
symptoms (including temperature <102.2°F) or otorrhea. Patients older most likely secondary to a ________________ process.
than 2 years with unilateral or bilateral AOM without otorrhea or severe a. inflammatory, inflammatory
symptoms can be appropriate for an observation strategy. Patients with b. neoplastic, inflammatory
otorrhea and those younger than 6 months should be treated with anti- c. inflammatory, neoplastic
biotic therapy and are not appropriate for an observation approach. This d. neoplastic, neoplastic
decision should be made in conjunction with caregivers, and reliable fol- Answer: c. In adults, neck masses should raise concern for malignancy,
low-up must be in place. This approach is not validated in adult patients. as 80% of non-thyroid neck masses in adults are neoplastic in etiology.
3. A 72-year-old female with a history of poorly controlled diabetes In children, the majority of neck masses are inflammatory in nature or
mellitus presents to the ED with severe left ear pain of 2 weeks secondary to benign congenital abnormalities.
59
KEY CONCEPTS
Asthma
Answer: b. No choice is contraindicated. Ketamine is the sedative of
choice because of its bronchodilatory effect. Propofol may have the
• I nhaled and systemic steroid medications are effective in controlling airway same benefit, although less profoundly. Thiopental releases histamine,
inflammation and have important roles in the management of asthma exac- and etomidate does not bronchodilate. Succinylcholine releases trace
erbations. amounts of histamine, but this is not known to cause any adverse effect.
• Inhaled bronchodilators and systemic corticosteroids remain the mainstays Rocuronium has an onset time similar to that of succinylcholine, no
of management for most acute asthma exacerbations. histamine release, and a prolonged duration of action. Either succi-
• Acute severe asthma requires rapid identification. Treatment may use nylcholine or rocuronium is acceptable for rapid sequence intubation
strategies not used in mild-to-moderate exacerbations, such as infusion (RSI) in acute asthma.
of magnesium sulfate, use of non-invasive ventilation, and endotracheal 4. Which of the following is a risk factor for death in patients present-
intubation. ing with an asthma attack?
• Ventilator management in the intubated asthmatic is critical and includes a. Currently taking theophylline
lower tidal volumes (6 to 8 mL/kg) and low respiratory rates, often less than b. Family history of asthma
10 per minute. c. Presence of symptoms for 1 week
• A normal partial pressure of carbon dioxide (Pco2) in a pregnant patient d. Use of three albuterol metered-dose inhalers (MDIs) per month
represents hypercarbia. Answer: d. The following are risk factors for death from asthma:
• Elevated lactic acid levels are common in critically ill asthmatics and do not • Past history of sudden severe exacerbations
reflect deterioration or a poor prognosis. • Prior intubation for asthma
• ED management of acute asthma is expanding (up to 24 hours) as more • Prior asthma admission to an intensive care unit (ICU)
non-critically ill asthmatics are treated in observation units. • Two or more hospitalizations for asthma in the past year
• Integration of discharged patients with acute asthma into chronic manage- • Three or more emergency department (ED) care visits for asthma in
ment strategies to prevent relapse requires physicians be familiar with con- the past year
trolling medications, such as inhaled corticosteroids and long-acting beta • Hospitalization or an ED care visit for asthma within the past month
agonists. • Use of more than two MDI short-acting beta-2 agonist canisters per
month
1. Which of the following is a risk factor for sudden death from asthma? • Current use of or recent withdrawal from systemic corticosteroids
a. A hospitalization for asthma in the past year but not within the • Difficulty perceiving severity of airflow obstruction
past 30 days • Comorbidities such as cardiovascular diseases or other systemic
b. An emergency department (ED) visit for asthma in the past year problems
but not within the past 30 days • Serious psychiatric disease or psychosocial problems
c. Current use of systemic corticosteroids • Illicit drug use, especially inhaled cocaine and heroin
d. Patient perception that the current exacerbation is very severe 5. A 25-year-old woman presents with wheezing and shortness of
Answer: c. Risk factors for sudden death are current or recent cortico- breath from asthma. She was recently exposed to cigarette smoke.
steroid use, ED or hospitalization within the past 30 days, more than She denies cough and fever. You cannot get much more of a his-
two hospitalizations for asthma in the past year, more than three ED tory from her at this time because she finds it difficult to speak in
visits for asthma in the past year, using more than two beta-agonist complete sentences. Her vital signs are blood pressure, 136/85 mm
canisters per month, previous intubation or intensive care unit (ICU) Hg; heart rate, 110 beats/min; respiratory rate, 32 breaths/min;
visit, and difficulty perceiving symptoms or their severity. and temperature, 99°F (37.2°C). Her oxygen saturation is 92%.
2. A 23-year-old man with known severe asthma presents with an Her PEF is 50% of predicted. On physical examination, you note
acute asthma flare over 2 hours. Physical examination reveals a bilateral wheezing, regular tachycardia, and accessory muscle use.
well-developed man in marked respiratory distress. Heart rate The remainder of her examination is normal. Over the course of 1
is 120 beats/min, oxygen saturation is 90%, respiratory rate is 26 hour, she receives supplemental oxygen, three doses of nebulized
breaths/min, blood pressure is 140/92 mm Hg, and oral tempera- albuterol (5 mg) mixed with ipratropium (0.5 mg), and oral pred-
ture is 98.7°F (37.2°C). Current medications are albuterol metered- nisone 50 mg. She now reports feeling somewhat better. She speaks
dose inhaler (MDI) and fluticasone inhaler 500 μg twice daily. What in longer sentences but still cannot speak in complete sentences. A
therapy is recommended for this acute flare? repeat peak flow measurement is now 60% of predicted. Otherwise,
a. Albuterol 2.5 mg nebulized, Ipratropium 0.5 mg, methylprednis- there are no changes on a repeat physical examination. You plan to
olone 125 mg intravenously, and magnesium sulfate 2 g intrave- admit her to your ED observation unit. What is an appropriate next
nously step in the management of this patient?
b. Epinephrine infusion at 5 μg/min a. Additional nebulized albuterol
c. Ipratropium 500 μg nebulized × three doses with methylprednis- b. Intravenous (IV) magnesium sulfate
olone 125 mg intravenously c. IV methylprednisolone (Solu-Medrol)
d. Methylprednisolone 125 mg intravenously with salmeterol neb- d. Oral montelukast
ulized via continuous nebulization Answer: a. This patient presents with a moderate-to-severe asthma
Answer: a. Short-acting inhaled beta-agonists with ipratropium are the exacerbation. She has responded to initial therapy but continues
cornerstone of acute asthma management. Corticosteroids are indi- to have moderate symptoms. Additional adrenergic medications
cated in any moderate or severe flare. Oral steroids are as efficacious as are indicated. Because she is tolerating nebulized medications and
intravenous (IV) steroids if the patient can take oral medications. Mag- is responding, there is no need for IV or subcutaneous adrenergic
nesium sulfate may obviate the need for intubation or decrease hospital agents, such as terbutaline. IV magnesium sulfate is a smooth muscle
length of stay in severe cases. Salmeterol is a slow-onset, long-acting relaxant, but it is generally reserved for more severe asthma exac-
beta-2 agonist (LABA) not indicated in acute asthma management. erbations. Oral and IV steroids have the same efficacy, and regard-
3. What is the medication combination of choice for the rapid less of the route, only need to be administered every 6 to 8 hours.
sequence induction of an asthmatic? Montelukast is a leukotriene-modifying drug that is used in chronic
a. Etomidate/succinylcholine management.
b. Ketamine/succinylcholine
c. Midazolam/pancuronium
d. Propofol/rocuronium
60 Chronic Obstructive Pulmonary Disease
Answer: c. The diagnosis of COPD requires spirometry-proven airflow lim-
KEY CONCEPTS
itation defined as a ratio of FEV1 divided by FVC after bronchodilators of less
• C hronic obstructive pulmonary disease (COPD) is “a common, preventable than 0.7. Patients who have normal spirometry with chronic symptoms and
and treatable disease that is characterized by persistent respiratory symp- possible emphysema on CT scan are not considered to have COPD.
toms and airflow limitation that is due to airway or alveolar abnormalities 2. In a patient with a diagnosis of chronic obstructive pulmonary dis-
usually caused by significant exposure to noxious particles or gases.” ease (COPD), increased dyspnea, sputum production, and sputum
• COPD exacerbation is a worsening of symptoms from baseline beyond day- purulence worse than baseline day-to-day variations for the past
to-day variation and requiring additional treatment. It is most commonly three days requiring evaluation in the emergency department is
caused by viral infections, bacterial infections, environmental pollutants, defined as:
particulate matter, thrombotic disease, or temperature changes. a. Chronic bronchitis
• Dyspnea, cough, increased sputum production, and sputum purulence are b. COPD exacerbation
the most common symptoms of COPD exacerbations. They are a result of c. Emphysema
airway inflammation, increased mucus production, and air trapping. d. Progression of underlying disease
• Treatment includes nebulized short-acting beta-agonists such as albuterol,
Answer: b. COPD exacerbation can be defined as a worsening of symp-
short-acting anticholinergics such as ipratropium, and glucocorticoids such
toms from baseline day-to-day variations that requires treatment. The
as prednisone.
most common symptoms include increased dyspnea, cough, increased
• Antibiotics should be given to patients with COPD exacerbations who have
sputum production and increased sputum purulence.
signs of lower respiratory tract infection, increased purulence of their spu-
3. A 64-year old male with a history of severe chronic obstructive pul-
tum, or who have respiratory failure. Antibiotics should be provided empir-
monary disease (COPD) with baseline 3 L/min oxygen requirement
ically based on local resistance patterns to Streptococcus pneumoniae,
presents to the emergency department with complaint of dyspnea.
Moraxella catarrhalis, and Haemophilus influenzae with common regimens
On physical exam, he is taking rapid shallow breaths with a respi-
including amoxicillin/clavulanate, macrolides, or tetracyclines.
ratory rate of 50, requiring 8 L/min by nasal cannula to maintain a
• Patients with COPD are at increased risk for thromboembolic disease due to
SpO2 of 88%, but is awake and following commands. A blood gas
a sedentary lifestyle and chronic inflammation. Patients admitted for COPD
reveals a pH of 7.25 and a pCO2 of 70 mm Hg. Chest x-ray reveals
exacerbation presenting a pleuritic chest pain or signs of heart failure with-
hyperinflated lungs without evidence of pneumothorax or infiltrate.
out an infectious precipitant should be screened for pulmonary embolism.
Antibiotics, nebulized short-acting beta-agonists, and glucocorti-
• Patients with COPD exacerbations commonly present with tachyarrhyth-
coids are initiated. The next step in management is:
mias including atrial fibrillation, atrial flutter, and multifocal atrial tachycar-
a. Continue medical management
dia.
b. Intubate the patient for anticipated clinical decline
• Oxygen supplementation should be administered with a saturation goal of
c. Start noninvasive bi-level positive pressure ventilation
88% to 92%.
d. Trial high flow nasal cannula
• In severe exacerbations, patients develop a rapid, shallow breathing pat-
Answer: c. This patient is presenting with respiratory failure without
tern which decreases the exhalation time, causes hyperinflation, increases
clear contraindications to noninvasive ventilation. The patient should
the proportion of dead space ventilation, and causes respiratory muscle
be trialed on noninvasive ventilation and receive inhaled bronchodi-
fatigue.
lators including albuterol and ipratropium, glucocorticoids, and anti-
• Bi-level noninvasive ventilation is the first-line therapy for patients with
biotics. Glucocorticoids have similar bioavailability and efficacy when
hypercapnic respiratory failure and acute COPD exacerbations without
given PO or IV, however in patients with respiratory distress we recom-
hemodynamic instability, severe agitation, or respiratory arrest. Implement-
mend avoiding oral medications.
ing noninvasive ventilation decreases the mortality, intubation rates, and
hospital length of stay.
4. The rapid, shallow breathing pattern that develops in a chronic
• If intubation is required, ventilation priorities are decreasing the patient
obstructive pulmonary disease (COPD) exacerbation _________
work of breathing and limiting dynamic hyperinflation primarily using a
exhalation time causing hyperinflation, and ________ the propor-
low respiratory rate of 10 to 14 breaths/min and tidal volume of ≤8 mL/kg
tion of dead space ventilation causing hypercapnia and respiratory
predicted by body weight. Respiratory acidosis with a pH greater than 7.2
muscle fatigue.
should be tolerated without manipulating the minute ventilation.
a. Decreases, decreases
• All patients being discharged from the emergency department should be
b. Decreases, increases
counseled on smoking cessation, adequate inhaler techniques, and offered
c. Increases, decreases
pneumococcal and influenza vaccination if not current.
d. Increases, Increases
Answer: b. The rapid, shallow breathing pattern that develops in a
1. A patient presents to the emergency department with 8 months COPD exacerbation decreases exhalation time causing hyperinflation
of progressive dyspnea. There is no recent change in the patient’s and increases the proportion of dead space ventilation causing hyper-
symptoms. On exam, there is a prolonged expiratory phase without capnia and respiratory muscle fatigue. If untreated, this will progress to
wheezing, no rales, no pedal edema. The electrocardiogram is nor- respiratory failure.
mal. The chest x-ray revealed hyperinflation with a flattened dia- 5. When treating a patient with a chronic obstructive pulmonary dis-
phragm. The diagnosis of chronic obstructive pulmonary disease ease (COPD) exacerbation who requires invasive mechanical ven-
(COPD): tilation, setting the initial tidal volume to 8 mL/kg predicted body
a. Can be made clinically in this patient weight and the respiratory rate to 10 to 14 breaths/min helps pre-
b. Can be made radiographically with demonstration flattening of vent the development of:
the diaphragms and hyperinflation on chest x-ray a. Acidosis
c. Requires demonstration of spirometry-proven airflow limitation b. Dynamic hyperinflation
with the ratio of forced expiratory volume in one second (FEV1) c. Hypercapnia
to forced vital capacity (FVC) after bronchodilators less than or d. Expiratory flow limitation
equal to 0.7 Answer: b. Decreasing the respiratory rate in an intubated patient with
d. Requires genetic testing and historical features including COPD allows for complete emptying and return to FRC, decreasing the
increased cough, sputum production, noxious stimuli, and dys- development of dynamic hyperinflation and auto-positive end expira-
pnea tory pressure (PEEP).
61 Upper Respiratory Tract Infections
a. Parapharyngeal abscess with invasion of the carotid sheath
KEY CONCEPTS b. Peritonsillar abscess with compression of the carotid sheath
• V iral infections cause most cases of pharyngitis. Patients should not be c. Spread of infection into the cavernous sinus causing thrombosis
treated with antibiotics based on symptoms and exam alone. Patients with and intracranial abscess
a Centor criteria score of 0 or 1 do not require further testing or treatment. d. Submandibular space infection and spread into retropharynx
Those with a score of 2 or greater should undergo rapid antigen testing with Answer: A. Complications from retropharyngeal or parapharyn-
treatment decisions based on results. Throat cultures are recommended in geal abscess include airway compromise, abscess rupture leading to
children but are not necessary in adults. aspiration of pus, and involvement of the carotid sheath. The carotid
• Antibiotics for group A Streptococcus pharyngitis are aimed at symptom sheath is surrounded by deep fascia and includes the carotid artery,
reduction, decreasing transmission, and decreasing suppurative complica- internal jugular vein, and vagus nerve, and thus infection can result
tions. Rheumatic fever is very rare and likely from a shift in streptococcal M in compromise of any of these structures including arterial erosion,
proteins rather than antibiotic use. aneurysm, Lemierre syndrome, palsy of cranial nerves IX–XII, and
• Infectious pharyngitis presents with an acute onset and resolves within mediastinitis.
days. In subacute and chronic cases of pharyngitis, consider abscess for- 3. A 34-year-old male presents with complaints of high fever, left-
mation, neoplastic causes, HIV, and autoimmune disease. sided face pain, and purulent nasal discharge for 10 days. He has
• In streptococcal pharyngitis, a single high dose of corticosteroids, such as been attempting symptom management with nasal saline irrigation
10 mg dexamethasone, is safe and reduces symptom severity. and intranasal corticosteroids, but after initial improvement reports
• Consider deep space infection and epiglottitis in patients who present with worsening symptoms. What is the next step in treatment?
neck pain, hoarseness, and have a benign oropharyngeal exam. a. CT scan of his face to evaluate for complications of rhinosinus-
• Airway edema from palatine or lingual tonsillitis, epiglottitis, or deep space itis
infection is rare, but life-threatening. b. Obtain bacterial culture via sinus puncture
• Deep space infections such as retropharyngeal and parapharyngeal abscesses c. Prescribe a 5-day course of amoxicillin
are difficult to diagnose clinically, and contrast-enhanced CT is recommended. d. Supportive care with antiinflammatory medications, rest, and
• Transcervical or intraoral ultrasound is useful in the diagnosis and treat- continued nasal saline irrigation
ment of peritonsillar abscess. Answer: C. The diagnosis of acute rhinosinusitis is clinical. The diag-
• Acute rhinosinusitis is likely viral and will resolve with supportive care, nosis of bacterial rhinosinusitis requires one of the three criteria: at
including nasal irrigation with hypertonic saline. least 10 days of persistent symptoms without improvement, three to
• Although rarely needed, first-line antibiotic treatment for acute rhinosinus- four days of severe symptoms including a fever greater than 39 degrees
itis is amoxicillin or amoxicillin-clavulanate for 5 days. Celsius with nasal discharge or facial pain without improvement, or
onset of progressive symptoms with worsening symptoms after initial
1. A previously well 18-year-old male presents with throat pain and improvement. Given the risks and lack of benefit of routine antibiotic
fever to 39 °C for the past 2 days. He denies vomiting, diarrhea, use in rhinosinusitis, antibiotics should only be used for patients who
cough, or rhinorrhea. He is not sexually active. Physical exam is meet the definition of bacterial rhinosinusitis with these criteria.
notable for tender anterior cervical lymphadenopathy and symmet- 4. Patients presenting with symptoms concerning for a retropharyn-
rically swollen and erythematous palatine tonsils with a gray-white geal abscess are best evaluated using what modality?
tonsillar exudate. The remainder of his neck and physical exam is a. Contrast-enhanced CT
benign. What is the most appropriate next step? b. Intraoral ultrasound
a. Empirically prescribe amoxicillin for 10 days c. Lateral neck x-ray
b. Obtain a complete blood count with differential d. Visualization with a fiberoptic scope
c. Obtain an intraoral tonsillar ultrasound Answer: A. Contrast-enhanced CT scan is the preferred diagnostic
d. Perform a rapid strep antigen test and prescribe antibiotics only test for the identification of retropharyngeal abscess. CT findings are
if there is a positive result characterized by a fluid collection with central hypodensity and com-
Answer: D. This patient is presenting with pharyngitis, with sore plete ring enhancement with scalloping. Fat stranding and edema
throat, fever, exudative pharyngitis, and cervical lymphadenopathy characterized by low-density thickening without peripheral enhance-
without associated cough or rhinorrhea. A majority of cases are viral ment may represent an early phase of infection before an abscess
and caused by common cold viruses. Bacterial infection is only respon- develops.
sible for approximately 5% to 10% of adult and 20% to 30% of pediatric
cases. Rapid antigen testing is recommended as a first-line diagnos- 5. This bacterium is frequently implicated in non-GAS pharyngi-
tic test for GAS pharyngitis, because it is highly sensitive and specific tis and is also most commonly implicated in suppurative jugular
when performed correctly. A positive test indicates the presence of thrombophlebitis:
GAS and does not require follow up testing. a. Arcanobacterium haemolyticum
2. A 19-year-old female patient presents to the emergency department b. Fusobacterium necrophorum
with vision changes and headaches. She reports a sore throat 10 c. Haemophilus influenzae type B
days ago started to improve, but never completely resolved. For the d. Methicillin-resistant Staphylococcus aureus
past 3 days she noted increasing left-sided neck pain and fevers. On Answer: B. Fusobacterium necrophorum is an anaerobic gram-negative
exam she appears uncomfortable with a fever and mild tachycardia. rod that is part of the normal oral flora and causes pharyngitis in patients
She is warm and diaphoretic with a dry forehead on the left. Her 15 to 45 years old with a similar presentation to GAS. Fusobacterium
left eyelid is slightly lower than the right and her left pupil is 3 mm is the primary causative agent in septic jugular vein thrombophlebitis
smaller than the right. Her extraocular movements are intact, and (Lemierre syndrome). Fusobacterium should be considered in young
her intraoral exam shows symmetric tonsils with mild erythema adults with ongoing severe symptoms.
and a midline uvula. She has mild trismus. What is most likely caus-
ing her symptoms?
Pneumonia 62
KEY CONCEPTS Answer: a. The novel coronavirus causing a worldwide pandemic is more
likely to cause severe disease in patients with comorbidities and those liv-
• E mpirical antimicrobial therapy should be started in the emergency depart- ing in nursing homes. SARS-CoV-2 often causes profound hypoxia that
ment (ED) for patients admitted with pneumonia. corrects with supplemental oxygen and causes diffuse infiltrates.
• Streptococcus pneumoniae is the most commonly encountered pathogen in 3. Which of the following causative organisms would be an indication
hospitalized patients, especially those requiring the intensive care unit. for respiratory isolation?
• No characteristic radiographic pattern is pathognomonic for a specific a. Histoplasma capsulatum
pneumonia pathogen. b. Herpes simplex virus (HSV)
• Legionella should be suspected in patients with gastrointestinal or neuro- c. Mycobacterium tuberculosis
logic symptoms presenting with pneumonia. d. Pneumocystis jiroveci
• As part of the evaluation of patients with pneumonia, the patient’s immune Answer: c. Any patient with suspected tuberculosis (TB) infection
status should be considered. Patients with HIV and other immunosuppres- should be placed in respiratory isolation and all staff should take
sive conditions are at risk for opportunistic infections, such as Pneumocys- appropriate personal protective precautions. None of the other caus-
tis jiroveci. ative agents require isolation.
• Empirical therapy should treat the most likely pathogens for the clinical 4. A 60-year-old man with a past medical history of diabetes controlled
situation, such as S. pneumoniae, Haemophilus influenzae, Mycoplasma with insulin and diagnosis of influenza 7 days prior presents with
pneumoniae, and Chlamydia pneumoniae, and should be consistent with cough, weakness, and purulent sputum production. Vital signs are
current national treatment guidelines, such as those from the American temperature 38.3°C (101°F) oral, heart rate, 130 beats/min, blood
Thoracic Society/Infectious Disease Society of America (ATS/IDSA). pressure, 80/50 mm Hg, respiratory rate, 30 breaths/min, and oxygen
• Community-acquired methicillin-resistant Staphylococcus aureus (MRSA) saturation, 92%. The chest radiograph reveals consolidation in the left
is an uncommon cause of community- acquired pneumonia (CAP), but lower lung (LLL). What is the most appropriate antibiotic therapy?
empirical coverage of MRSA should be strongly considered for patients a. Ceftriaxone plus levofloxacin plus vancomycin
with severe pneumonia and sepsis, with concomitant influenza, contact b. Ceftriaxone with a macrolide
with someone infected with MRSA, or radiographic evidence of necrotizing c. Fluoroquinolone only
pneumonia. d. Trimethoprim-sulfamethoxazole (TMP-SMX)
• Patients with prior use of intravenous antibiotics, neutropenia, or under- e. Vancomycin only
lying bronchiectasis are at increased risk of infection with Pseudomonas
Answer: a. In patients requiring hospitalization for community-
aeruginosa. Empirical therapy for high-risk or critically ill patients should
acquired pneumonia (CAP), coverage for CAP would typically be with
cover for P. aeruginosa.
a combination of a macrolide with a β-lactam. Because this patient has
• Disposition is dictated by the patient’s underlying medical conditions,
signs of septic shock, consideration should be given to the addition of
severity of illness, likelihood of clinical deterioration, and feasibility of
an agent for methicillin-resistant Staphylococcus aureus (MRSA; van-
home care and outpatient follow-up.
comycin) in addition to CAP coverage.
5. Which of the following is true regarding the use of procalcitonin in
1. Which of the following statements is true regarding the treatment
emergency department (ED) patients with pneumonia?
of community-acquired pneumonia (CAP)?
a. Amoxicillin monotherapy is not an option as it does not effec- a. Procalcitonin is not useful in determining if a pneumonia is bac-
tively cover atypical pneumonia. terial or viral.
b. Macrolide resistance almost always prohibits azithromycin b. Procalcitonin predicts mortality in ED patients.
monotherapy for CAP. c. It is helpful is predicting who can be discharged from the ED.
c. The most common atypical pneumonia in CAP is Chlamydophila. d. Procalcitonin is useful in guiding antimicrobial therapy.
d. Clinical history and prodrome often lead to the identification of Answer: a. While sometimes requested by hospitalist colleagues, the
the causative organism. ED use of procalcitonin has not been shown to be useful in determin-
Answer: b. The 2020 American Thoracic Society/Infectious Disease ing mortality, viral versus bacterial etiology, mortality, or prognosis.
Society of America (ATS/IDSA) guidelines no longer recommend
macrolide monotherapy as Streptococcus pneumoniae resistance is
greater than 30% in most settings in the United States. Amoxicillin
is now recommended as a first-line agent in the treatment of CAP in
healthy patients without comorbidities.
2. A 92-year-old male nursing home resident presents to the emer-
gency department (ED) with a history of hypertension and non-
productive cough. He has a SaO2 at triage of 82% but is speaking
full sentences and with supplemental oxygen his saturation
increases to 95%. Chest x-ray reveals diffuse infiltrates. Which of
the following viral causes of pneumonia is this patient likely to
have?
a. COVID-19
b. Respiratory syncytial virus (RSV)
c. Parainfluenza virus
d. Metapneumovirus
Pleural Disease
63
KEY CONCEPTS
• P oint of care thoracic ultrasound can be used to rule out pneumothorax with
high sensitivity.
• For healthy young patients with a small primary spontaneous pneumotho-
rax, observation with supplemental oxygen is an appropriate treatment
option. For larger symptomatic primary spontaneous pneumothorax, simple
aspiration with a catheter is often successful.
• In most cases of secondary pneumothorax, tube thoracostomy should be
considered because less invasive approaches are associated with treat-
ment failure.
• The most common causes of pleural effusion in the United States are con-
gestive heart failure, malignancy, and bacterial pneumonia. Pulmonary
embolism is an uncommon cause of pleural effusion.
• Thoracentesis should be performed under ultrasound guidance to minimize
the risk of complications.
• On a frontal (anteroposterior or posteroanterior) projection, a volume of
at least 200 mL of pleural fluid is required to detect a pleural effusion.
Ultrasound can detect as little as 50 mL of pleural fluid and can be easily
preformed at bedside.
1. What is the most common condition associated with secondary thoracostomy. If the primary spontaneous pneumothorax is small (<2
spontaneous pneumothorax in adults in the United States? cm), then observation alone is appropriate. For a larger primary spon-
a. Chronic obstructive pulmonary disease taneous pneumothorax (> 2cm) without significant symptoms, either
b. Collagen vascular disease observation alone or intervention with either simple aspiration or tube
c. Pneumocystis pneumonia thoracostomy may be appropriate and a shared decision-making dis-
d. Pulmonary malignancy cussion with your patient is appropriate to determine best course of
Answer: A. Chronic obstructive pulmonary disease is the most action.
common condition associated with secondary spontaneous pneumo- 4. Which of the following statements is true regarding the routine
thorax, although all the conditions listed may also be causes. application of suction after tube thoracostomy?
2. Which of the following are indications of a pneumothorax on bed- a. It improves the rate of lung expansion.
side ultrasound? b. It increases the risk of reexpansion pulmonary edema.
a. Identification of a lung point c. It is associated with increased rates of empyema.
b. Lack of lung sliding identified d. It is not routinely indicated.
c. When in M-mode, you see a “barcode” appearance Answer: D. Suction neither accelerates lung reexpansion nor improves
d. All of the above outcomes. It is indicated when reexpansion with a Heimlich or water
Answer: D. Pneumothorax may be difficult to identify on bedside seal device does not occur after 24 to 48 hours. A pressure of at least
ultrasound and there are several ways to help identify a pneumotho- −20 cm H2O should be used.
rax. Identifying an area of lung that lacks lung sliding and has a “bar- 5. A 68-year-old man with a history of esophageal cancer presents
code” appearance on M-mode indicates that the parietal and visceral with progressive fever, chest pain, and shortness of breath over 24
pleura are not sliding over each other and is highly suggestive of a hours. Chest radiography demonstrates a possible left lower lobe
pneumothorax. Identifying a lung point, the point where a pneumo- pneumonia and large left pleural effusion. Pleural fluid analysis
thorax transitions to a normal pleural interface, can also help identify reveals pH of 6.95, glucose level of 47 mg/dL, 11,500 white blood
a pneumothorax. cells (WBCs)/mm3 (82% neutrophils), and protein level 75% of
3. A 32-year-old male with no significant past medical history presents plasma levels. What are the indicated next steps in management?
with acute onset of right pleuritic chest pain, cough, and shortness a. Antibiotics and fluid resuscitation
of breath. The symptoms occurred at rest. Physical examination is b. Antibiotics and tube thoracostomy
remarkable for a tachycardia of 110 beats/min and respiratory rate c. Antibiotics, tube thoracostomy, and esophageal oral contrast
of 32 and use of accessory muscles to breathe. Chest radiography study
reveals an estimated 5-cm right pneumothorax. Which of the fol- d. One-time pleural aspiration for fluid analysis
lowing would be suitable management? Answer: C. This is an exudative pleural effusion as defined by Light’s
a. Admission for 100% face mask oxygen and repeat radiography criteria (see Box 63.3). A pH less than 7.0 suggests emphysema or
in 1 day esophageal rupture. This patient is at risk for both; hence, the need to
b. One-time air aspiration and repeat radiography in 6 hours assess esophageal integrity. A pH less than 7.0 with glucose level less
c. Reassurance and observation than 50 mg/dL are indications for tube thoracostomy. Normal pleural
d. Tube thoracostomy fluid has a WBC count of less than 1,000/mm3.
Answer: D. Large (2 cm) primary spontaneous cases and patients who
are showing signs of respiratory distress should be treated with tube
SECTION THREE Cardiac System
1. A 44-year old male presents after an episode of left-sided chest d. Rapid revascularization with percutaneous coronary interven-
pain without radiation that lasted for approximately 5 minutes. He tion (PCI), initiation of therapeutic hypothermia, and admission
described the pain as a “pressure” without radiation or other sig- to ICU for comprehensive postresuscitation care
nificant symptoms. Onset was at rest and the symptoms are now e. Supportive care, and admission to ICU
resolved. The patient is a non-smoker, and medical history is sig- Answer: D. A neurologic examination immediately following cardiac
nificant only for hypertension. There is no significant family history arrest is poorly prognostic of a favorable neurologic outcome with
of CAD. ECG is significant for LVH without other findings. Initial modern postresuscitation care. In the absence of multiple “unfavor-
troponin testing is negative (less than the normal limit). What is the able resuscitation features” (prolonged arrest, unwitnessed arrest,
most appropriate next step in management? nonshockable rhythm, advanced age, etc.), revascularization with per-
a. Admission for serial troponin testing cutaneous coronary intervention (PCI) and immediate application of
b. Coronary CT angiography therapeutic hypothermia is warranted. Although not contraindicated,
c. Discharge to outpatient follow-up fibrinolysis is inferior to PCI following cardiac arrest and should only
d. Stress echocardiography be used when a patient is not a candidate for PCI.
Answer: C. Utilizing the HEART score can assist in the risk stratifica- 3. Which of the following is an absolute contraindication to fibrino-
tion of this patient. Ten minutes of chest “pressure” without other typ- lytic therapy?
ical features of ACS may be considered “moderately suspicious.” Given a. Age older than 75 years
this, the patient’s heart score is 3 (1 point each for history, nonspecific b. Appendectomy performed 2 months ago
repolarization disturbance on ECG, and the presence of 1 to 2 risk fac- c. Previous coronary artery bypass grafting (CABG)
tors). This patient is at low risk for ACS and may be appropriately dis- d. Previous hemorrhagic stroke
charged from the emergency department. e. Systolic blood pressure of 175/90 mm Hg following administra-
2. A 48-year-old man with history of hypertension and hypercho- tion of vasoactive agents
lesteremia presents with chest pain and hyperacute T waves in Answer: D. Although patients older than 75 years have a higher risk of
an anterior distribution on the initial ECG. During your initial intercerebral hemorrhage, age should not be considered a contraindication
history and physical examination the patient experiences ven- to fibrinolysis. Recent major surgery or trauma is a relative contraindica-
tricular fibrillation that responds to cardiopulmonary resuscita- tion for fibrinolysis; however, the term recent is variably defined in the fibri-
tion (CPR) and defibrillation after being pulseless for a period nolytic literature and never as more than 6 weeks. Although prior CABG
of 3 minutes. Following cardiac arrest, the patient is comatose, patients should be preferentially considered for PCI, there is no contrain-
with the following vital signs: HR 110 beats/min, BP 160/98 mm dication to fibrinolytic use in these patients if PCI is not available. Systolic
Hg, RR 12 breaths/min (intubated), temperature 36.5°C, and O2 blood pressure above 150 mm Hg is a risk factor for intracerebral hem-
saturation 96%. A repeat ECG demonstrates a large, evolving orrhage. Only hypertension persistently above 200/120 mm Hg, despite
anterior STEMI. Which of the following treatment plans is most reasonable efforts, should be considered an absolute contraindication.
appropriate? 4. Which of the following drugs provides mortality benefit in the set-
a. Administer aspirin, PSY12 inhibitor, IV UFH, IV fibrinolysis, ting of AMI?
and admission to intensive care unit (ICU) a. Aspirin
b. Administer aspirin, PSY12 inhibitor, IV UFH, IV fibrinolysis, b. Intravenous beta blocker
initiation of therapeutic hypothermia, and admission to ICU c. Intravenous morphine
c. Neurologic examination for brain death and admission to palli- d. Nitroglycerin
ative care because outcome is almost universally fatal e. Oxygen
Answer: A. The ISIS-2 trial has demonstrated that aspirin inde- c. The patient should receive a fibrinolytic agent followed by appro-
pendently reduces mortality by 23% in the setting of AMI. Intravenous priate antiplatelet and anticoagulant therapies, with transfer to
morphine has not been shown to improve mortality and has been asso- the closest PCI center for immediate PCI.
ciated with mortality. Although nitroglycerin does improve symptoms d. The patient should receive a fibrinolytic agent followed by
and cause vasodilation, it has never been proven to improve mortality. appropriate antiplatelet and anticoagulant therapies, with trans-
Oxygen beyond that needed to maintain an oxygen saturation of 94% fer to the closest PCI center within 24 hours for reevaluation and
has been associated with additional mortality. The use of intravenous consideration of immediate PCI.
beta blockers does not offer significant benefit and is associated with an Answer: D. The ECG demonstrates an extensive anterolateral STEMI.
increased rate of adverse events. The patient is young and has presented early in the STEMI evolution.
5. A 42-year-old male patient presents with 45 minutes of chest pain. This patient is at extreme risk due to the extensive nature of the STEMI
The ECG is depicted below (Fig. Q64.5). You are working at a non- and yet can benefit significantly from early reperfusion therapies. A
invasive (i.e., no PCI capability) hospital; transfer time to the closest delay of more than 60 to 120 minutes in this patient is not appropriate
major medical center with PCI capability is 4.5 hours considering for the initiation of reperfusion therapies; furthermore, he is a candi-
weather and logistics. The patient has no contraindications for fibri- date for a fibrinolytic agent. The early initiation of reperfusion therapy
nolysis. Which of the following statements is most appropriate? (fibrinolysis or PCI) is vital to reduce morbidity and mortality. Such
a. The patient must be transferred rapidly to the closest PCI center, a significant delay in this case for PCI is not justified, so a fibrinolytic
with initiation of appropriate β-adrenergic blocking agents and agent is preferred. On arrival at the closest PCI center, the patient can
antiplatelet and anticoagulant therapies before transfer. be evaluated for PCI if he has not demonstrated successful reperfusion
b. The patient should receive a fibrinolytic agent followed by appro- with resolution of chest discomfort and normalization of the ST seg-
priate antiplatelet and anticoagulant therapies with admission to ment elevation.
your hospital’s ICU.
I aVR V1 V4
II aVL V2 V5
III aVF V3 V6
65 Dysrhythmias
65.4. The most frequent proarrhythmic effects are seen with which
KEY CONCEPTS class of antidysrhythmic?
• E lectrical therapy is appropriate for unstable patients in whom a dysrhyth- a. IA
mia is the cause of symptoms—pacing if the heart rate is slow, counter- b. IB
shock with sedation if fast. c. IC
• Any regular new-onset, symptomatic, wide-complex tachycardia should be d. II
assumed to be ventricular tachycardia until proven otherwise. e. III
• Type II second-degree AV block is never a normal variant and implies a Answer: c. Class IC agents, such as flecainide, encainide, and propafe-
conduction block below the AV node. When the conduction ratio is 2:1, a none, markedly slow depolarization and conduction and prolong repo-
type II block should be assumed until proven otherwise. Pacing should be larization and action potential duration. Class IB agents generally have
readily accessible. the least proarrhythmic effect.
• Any tachycardia exceeding a rate of 225 to 250 beats/min, regardless of 65.5. A 49-year-old woman presents with a sudden onset of palpita-
the QRS complex morphology, should be considered an accessory pathway tions and shortness of breath. This has happened once before.
syndrome. Nodal blocking agents should be avoided. She has no past history and takes no medications. Vital signs are
• Irregularity can be difficult to appreciate in tachycardia over 200 beats/ temperature, 36.0°C (96.8°F) oral, blood pressure, 115/69 mm
min. Atrial fibrillation can be missed if R-R intervals at fast rates are not Hg, heart rate 156 beats/min, respiratory rate 24 breaths/min,
carefully tracked. and oxygen (O2) saturation, 98%. Her electrocardiogram (ECG)
is shown in Fig. 65.28. What is the most appropriate interven-
65.1. What is the primary electrochemical difference between pace- tion?
maker and nonpacemaker cells? a. Adenosine, 6 mg IV
a. Lack of a plateau phase 3 in nonpacemaker cells b. Digoxin, 0.25 mg IV
b. Rapid phase 0 upstroke in nonpacemaker cells after stimulus c. Diltiazem, 0.4 mg/kg IV
c. Slow calcium ion influx during phase 2 for pacemaker cells d. Propranolol, 1 mg IV
d. Slow phase 4 spontaneous depolarization in pacemaker cells e. Synchronized electrical cardioversion after IV sedation with
e. Transient membrane repolarization by potassium channel midazolam
closure during phase 1 for pacemaker cells Answer: a. Adenosine causes slowing of conduction in the anterograde
Answer: d. The spontaneous return to a depolarization threshold and retrograde pathways, with no effect on ventricular contractility. It
during phase 4 (diastole) characterizes pacemaker cells. Both cell types converts a high percentage of narrow-complex tachycardias to sinus
then exhibit a rapid phase 0 upstroke resulting from sodium ion (Na+) rhythm, but with a 25% recurrence rate. Diltiazem would not be unrea-
influx, brief repolarization resulting from potassium ion (K+) efflux sonable, but the quoted dose is too high. Calcium channel blockers also
(phase 1), plateau phase resulting from balanced calcium ion (Ca2+) exert their effects only on the anterograde pathway, with little direct
entry and K+ efflux (phase 2), and then repolarization resulting from effect on accessory pathways. Contractility may be diminished. Digoxin
Ca2+ channel closure and K+ efflux (phase 3). use has been largely supplanted by adenosine and class IV agents. Its
65.2. For a reentrant tachydysrhythmia to occur, what three condi- onset of action after IV use is 1.5 to 2 hours. Cardioversion would not be
tions exist? indicated unless the patient exhibited hemodynamic instability.
a. Electrolyte disturbance, ischemia, and altered conduction in
an endogenous atrioventricular pathway
b. Electrolyte disturbance, two conduction pathways, with one
of the pathways being slower
c. Ischemia, two conduction pathways, with one of the path-
ways being slower
d. Two conduction pathways, one path being slower, and differ-
ing responsiveness
e. Two conduction pathways with equal responsiveness
Answer: d. Remember that a conducting pathway is bidirectional.
In a typical scenario, the alpha pathway of the atrioventricular
(AV) node is the anterograde conducting limb, and the beta path-
way is the retrograde conducting limb. Reentrant dysrhythmias are
almost always AV nodal and narrow complexes that start and end
abruptly.
65.3. Classic antifibrillatory effects are seen with which class of anti-
dysrhythmic?
a. IA
b. IB
c. IC
d. II
e. III
Answer: e. Class III agents, of which amiodarone is the prototype,
prolong the action potential and refractory period duration. Class I
agents have variable effects on depolarization rate and repolarization
duration.
Implantable Cardiac Devices 66
1. Which of the following conditions is an indication for placement of
KEY CONCEPTS a biventricular pacemaker?
• P acemaker malfunction soon after implantation (within 6 to 8 weeks) is a. Third-degree AV block
usually a result of a lead problem, such as a lead displacement, or a pace- b. Brugada syndrome
maker programming issue, such as a pacing rate too slow for the patient’s c. Severely depressed systolic heart failure and a left bundle branch
needs. block
• Pacemaker malfunction is categorized as failure to pace, oversensing, d. Slow atrial fibrillation
undersensing, or pacing at an inappropriate rate (too fast or too slow). Answer: C. Cardiac resynchronization therapy, or biventric-
• Pacemaker or implantable cardioverter- defibrillator (ICD) interrogation ular pacing, is indicated for patients with systolic heart failure
should be performed if a patient’s presentation (e.g., shortness of breath, with an ejection fraction less than 35% and a left bundle branch
palpitations, syncope, shock, cardiac arrest, etc.) or ECG (electrocardio- block. By ensuring appropriate synchrony between the ventricles,
gram) is suggestive of device malfunction, abnormal intrinsic rhythm, or cardiac output is improved, and multiple studies have shown a
worsening cardiac function. mortality benefit in these patients. Options A and C have an indi-
• Modern lithium-iodine batteries are highly reliable; battery-related issues cation for a permanent pacemaker but would not benefit from
are unlikely to be the cause of abrupt pacemaker failure. biventricular pacing. Option B has an indication for implantable
• Consider pacemaker lead infection and endocarditis if a patient with a cardioverter-defibrillator.
pacemaker has a fever of unclear cause. 2. A 67-year-old female presents with complaints of shortness of
• Right ventricular-paced complexes are normally conducted with a left bun- breath and a fluttering sensation in her chest. She had a dual-
dle branch block pattern. A right bundle branch pattern is abnormal and chamber pacemaker placed 10 years ago for sinus node dysfunc-
suggests lead displacement. tion. Blood pressure is 124/96 mm Hg. On examination, she
• Magnet application turns off the sensing and inhibition function, causing is anxious with clear lungs and no evidence of jugular venous
the pacemaker or ICD to fire at a preprogrammed fixed rate (typically near distention or peripheral edema. There is no tenderness over
60 beats/minute) and inhibits antitachycardia pacing in ICDs. Magnet her pacemaker insertion site. An ECG is provided as follows.
removal returns the pacemaker to its programmed pacing mode. Which of the following is the next best step in the management
• Defibrillation is safe in patients with a pacemaker or ICD. Pads should be of this patient?
placed at least 10 cm from the subcutaneous implant site of the device. a. Administration of IV amiodarone
• Most left ventricular assist device (LVADs) do not produce pulsatile flow; b. Drawing blood cultures and starting antibiotics
therefore, these patients will not have a palpable pulse. c. Immediate cardioversion
• All modern VADs tolerate external defibrillation and cardioversion. d. Placement of a magnet over the pacemaker
• Chest compressions can be performed in an unresponsive, nonperfusing
patient with a VAD, supported by typical resuscitative measures.
aVR
V1 V4
II
aVL V2 V5
III
aVF V3 V6
II
Answer: D. This patient has a pacemaker with an inappropriately fast Answer: D. This patient shows evidence of right heart failure,
rate due to a pacemaker-mediated tachycardia. Management of this condi- with poor RV systolic function and elevated jugular venous and
tion involves disruption of the circuit and can be accomplished by placing IVC pressures. Intra-aortic balloon pumps and temporary left
a magnet over the pacemaker, which sets the pacemaker into an asynchro- ventricular assist devices will support primarily the left ventricle.
nous mode with no sensing (e.g., DOO). Cardioversion and amiodarone are Veno-venous ECMO provides effective oxygenation and ventila-
less appropriate in this situation. Consultation with a cardiologist and inter- tion, but very little hemodynamic support. Veno-arterial ECMO
rogation of the pacemaker should be performed, although this can be done may be considered as temporary biventricular support, if there is
after placement of a magnet over the pacemaker. In the absence of potential expected recovery of ventricular function, or if the patient is a
for pacemaker infection, there is no role for blood cultures or antibiotics candidate for another therapy (e.g., heart transplant or intracor-
3. A 67-year-old male with a history of an ischemic cardiomyopathy poreal VAD).
and an implantable cardiac defibrillator (ICD) presents with com- 5. A 65-year-old man with an intracorporeal left ventricular assist
plaints of multiple shocks from his ICD in the past day. Interro- device (LVAD) presents to the emergency department and tells
gation of his ICD shows no evidence of any shocks in the past 2 you he has felt lethargic for the past two days. A manual blood
months. What is the most appropriate management plan? pressure cuff and Doppler ultrasound show a systolic blood pres-
a. Administration of intravenous amiodarone sure of 75 mm Hg, and a continuous “hum” is heard on ausculta-
b. Cardiology consultation for reprogramming of ICD tion. The cardiac monitor shows a wide complex tachycardia, and
c. Reassurance and follow-up with his cardiologist no pulse is palpable. Which is the most appropriate next step in
d. Temporary ICD deactivation with magnet application management?
Answer: C. This patient has experienced 2 “phantom shocks.” The a. Administer amiodarone
perception of a shock in the absence of the actual ICD firing occurs b. Begin chest compressions
in approximately 10% of patients with an ICD and is associated with c. Call heart failure specialist and continue diagnostic evaluation
significant anxiety and stress. Appropriate management includes d. Immediate cardioversion/defibrillation
reassurance and discharge with a patient’s cardiologist. There is Answer: C. A low systolic pressure is expected with a continuous-flow
no role for IV amiodarone, reprogramming of the ICD or magnet LVAD, as there is generally a very low pulse pressure. The fact that the
application. patient is awake suggests that he is adequately perfusing. His wide com-
4. A 55-year-old man presents with chest pain and hypotension. plex tachycardia may be a result of a ventricular dysrhythmia, or may be
ECG reveals ST-elevation MI in the inferior and lateral leads, and SVT or sinus tachycardia with aberrancy, and should be further investi-
bedside echocardiogram shows poor systolic function of both gated in a stable patient before starting interventions. He does not require
ventricles. He has elevated jugular venous pressure, and a ple- chest compressions or immediate defibrillation/cardioversion with evi-
thoric IVC on ultrasound. In addition to percutaneous coronary dence of adequate perfusion, despite the lack of palpable pulse. While
interventions, which of the following therapies would be most amiodarone may be an appropriate therapy, more information regarding
appropriate to consider? his rhythm and current medications is required before giving this therapy.
a. Intra-aortic balloon pump
b. Temporary LVAD (e.g., Impella or Tandem Heart)
c. Veno-venous ECMO
d. Veno-arterial ECMO
67 Heart Failure
Heart Failure
KEY CONCEPTS
• H eart failure (HF) is the clinical syndrome defined by signs and symptoms of shifts predominate over structural (ie, irreversible) ones. These patients tend
elevated intracardiac pressures or depressed cardiac output, which in turn to be hypertensive, less likely to be hypervolemic, respond favorably to intra-
are due to either functional or structural cardiovascular (CV) abnormalities. venous (IV) nitroglycerin, and have a better prognosis despite more abrupt
By definition, HF presentations managed in the emergency department (ED) symptom onset.
are acute heart failure (AHF) (ie, HF in which the signs and symptoms require • The “cardiac phenotype” of AHF involves a predominance of structural (ie,
unscheduled care). irreversible) CV abnormalities such as more severe chronic myocardial dis-
• AHF is a multiorgan, multifactorial, and multiple phenotype disease state. ease, myocardial ischemia, and more complex multiorgan interactions (eg,
Abnormalities in renal function, central venous volume and fluid shifts, arte- cardiorenal syndrome). These patients are often hypervolemic and require
rial vascular tone, neuroendocrine overactivity, microvascular dysfunction, high-dose IV diuresis. They typically present with a more indolent symptom-
respiratory failure, or myocardial ischemia have bidirectional relationships atic progression, but paradoxically are at higher risk for worse outcomes.
with AHF as both potential causes and effects. • Noninvasive positive-pressure ventilation is the first-line approach for respi-
• Central congestion without hypervolemia is typically due to intercompartmen- ratory distress in AHF and may obviate a need for intubation in most cases.
tal fluid shifts to the central circulation from venous reservoirs with reduced • The initial dose of furosemide in the ED for AHF is 1 to 2.5 times the patient’s
capacitance (increasing preload) or abrupt increases in central arterial tone total daily oral dose (or 40–80 mg if loop-diuretic naïve) and should be given
(ie, increasing afterload). IV. Although creatinine may rise after IV furosemide, it is rarely indicative of
• Emergency physician (EP) clinical gestalt based on history and physical exam iatrogenic acute kidney injury, and patients with acute cardiorenal syndromes
is inaccurate for AHF diagnosis as often as a quarter of the time. typically benefit from aggressive diuresis.
• The most useful diagnostic test for identifying lung congestion due to AHF • Inotropes are only indicated in cases of cardiogenic shock (CS), because they
is an 8-point lung ultrasound B-line scan, which is more sensitive than chest may increase mortality as a pharmacologic class.
x-ray for pulmonary edema, and has a higher positive predictive value for AHF • Resuscitation and stabilization of CS should be followed by rapid evaluation
than natriuretic peptides. from interjectional cardiology or cardiac surgery. While STEMI was tradition-
• Focused cardiac ultrasound (FOCUS) for detecting reduced ejection fraction ally the predominant cause of CS, it may only account for 30% of contempo-
(EF) misses the 40% to 50% of AHF presentations with preserved EF. It adds rary cases due to improvements in coronary intervention.
little information to risk stratification because EF does not typically change • Numerous high-moderate risk features in AHF define which patients likely
with dynamic worsening of cardiac function or improvement after treatment. require admission or observation, however, low-risk factors (ie, for discharge)
• A useful clinical classification of AHF distinguishes between a vascular phe- are less well defined. When high or moderate risk factors are absent, the
notype and a cardiac phenotype, though overlap can exist. Distinguishing patient is adherent to the appropriate guideline-directed medical therapy, and
between systolic and diastolic dysfunction is less helpful in the ED, because can secure close outpatient follow-up (≤1 week), a shared decision-making
EF is preserved in about half of AHF cases, and most AHF patients have both discussion regarding discharge from the ED may be appropriate.
systolic and diastolic dysfunction regardless of their EF.
• The “vascular phenotype” describes an AHF presentation where functional
(ie, reversible) CV abnormalities such as increased vascular tone and fluid
1. A 59-year-old female with a smoking history who has never been to Answer: A. In this variation of question two, the patient is diuretic
the doctor before presents to the ED with 2 weeks of dyspnea and naïve and has features suspicious for the vascular phenotype of AHF.
no known past medical history. Evaluation is notable for BNP 650, Furosemide should therefore be dosed at 40–80 mg IVP, and a 25%
creatinine 1.5, wheezing on lung exam, CXR with cardiomegaly but or less reduction in SBP should be targeted with repeated IV push
no pulmonary edema, and LUS demonstrating more than 3 B-lines nitroglycerin, 1–2 mg per dose. In patients with less distress or less
in 2 of 4 lung zones on each hemithorax. Which of the following is severe hypertension who still show the vascular phenotype, sublingual
the most likely/appropriate diagnosis? nitroglycerin is a useful alternative because absorption and bioavail-
a. Advanced chronic heart failure ability are similar to IV. Transdermal and PO nitrates are not currently
b. Acute decompensated heart failure thought to be useful in AHF, by contrast, because of less favorable
c. Acute exacerbation of chronic obstructive pulmonary disease kinetics.
d. De novo acute heart failure 4. A 78-year-old patient presents to the ED with hypotension and
Answer: D. The patient presents with 3 or more B-lines in 2 of 4 lung acute respiratory failure. His daughter states he complained of
zones on each hemithorax, which is highly specific for acute heart fail- severe crushing chest pain 7 days ago, but refused to go to the hos-
ure (AHF). Because the patient does not have a known past history pital. His ECG shows deep Q waves and T-wave inversions in the
of chronic heart failure, her more specific diagnosis is “de novo acute inferior leads without ST elevation, and harsh systolic murmur is
heart failure,” and not “acute decompensated heart failure.” “Advanced heard on exam. What is the best next step in management?
chronic heart failure” is a subset of chronic heart failure at the end a. Dobutamine, consult cardiac surgery for mechanical hemody-
stages of chronic pump dysfunction. Because the patient presented for namic support and repair of ruptured papillary muscle
unscheduled care due to heart failure (ie, true of essentially all patients b. Dobutamine, consult interventional cardiology for PCI
presenting to the ED for HF symptoms, as opposed to a scheduled c. Norepinephrine, consult cardiac surgery for mechanical hemo-
outpatient visit), this case is by definition acute heart failure and not dynamic support and repair of ruptured papillary muscle
chronic heart failure. Finally, although wheezes on exam may lead the d. Norepinephrine, consult interventional cardiology for PCI
clinician to think of COPD exacerbation, auscultated wheezing is com- Answer: C. This patient presents with cardiogenic shock, likely
mon in AHF and neither sensitive nor specific (likelihood ratios posi- due to papillary muscle rupture given the murmur consistent with
tive and negative = 0.6, 1.8) enough to significantly inform a diagnosis mitral regurgitation, and an ECG suggesting that a STEMI may have
of AHF versus COPD on its own. occurred several days ago. Surgical management is the definitive
2. A 68-year-old ED patient with a history of HFrEF presents with therapy. While arranging for surgical management, norepinephrine
3 weeks progressive edema and exertional dyspnea, pulmonary is the first-line vasoactive therapy for cardiogenic shock patients.
edema on portable CXR, respiratory rate 35, BP 128/67, and SpO2 Other inotropic and/or vasoactive medications such as epinephrine,
86% on room air. Last week his cardiologist increased his home dose dobutamine, milrinone, or levosimendan are reasonable second-
of furosemide from 40 mg PO daily to 40 mg BID, with unchanged line options.
urine output and no improvement in symptoms. Which of the fol- 5. In a patient with AHF and a history of HFrEF whose symptoms
lowing is the best step in management? have resolved after a single dose of IV furosemide in the ED, which
a. Nasal cannula oxygen, 160 mg IV push furosemide now of the following profiles suggests the lowest risk to be discharged
b. Nasal cannula oxygen, 60 mg IV push furosemide only if creati- from the ED?
nine is not elevated from baseline a. No renal or hepatic dysfunction, takes no home medications
c. BiPAP, 160 mg IV push furosemide now b. SBP 165 mm Hg, compliant with home meds including carve-
d. BiPAP, 60 mg IV push furosemide only if creatinine is not ele- dilol and sacubitril-valsartan
vated from baseline c. Patient’s PCP can see them for follow-up in 21 days from now
Answer: C. This patient presents with hypoxia, tachypnea, and other d. No comorbidities, de novo presentation of AHF
signs of respiratory failure, and noninvasive positive-pressure ventila-
tion (NPPV) such as CPAP or BiPAP should be the first-line resus- Answer: B. This patient who has resolution of or marked improvement
citative therapy. Although nasal cannula may sufficiently reverse the in symptoms after initial ED treatment is a good candidate for discharge
hypoxia, NPPV has beneficial effects on work of breathing and possibly from the ED. In option A, a lack of acute renal or hepatic dysfunction
on AHF hemodynamics. IV furosemide dosing should be 1 to 2.5 times would further increase the appropriateness for discharge, but having
the patient’s daily home PO dose (or 40–80 mg IV if they do not take HFrEF the patient should be on a minimum of a beta-blocker and ACE/
furosemide). Because this patient was recently increased to 80 mg, a ARB/ARNI for guideline-directed medical therapy (GDMT). Discussion
dose of 80–200 mg IV would be possible. Additionally, since the change with the patient’s physician to initiate these therapies and arrange close
resulted in no change in symptoms at home, this would suggest the follow-up could be an appropriate strategy with answer A, but in answer
patient is experiencing some aspect of acute cardiorenal dysfunction, B the patient is already on appropriate GDMT. Elevated BP in option B
loop diuretic resistance, or both. Thus, a dose at the higher end of the is also a sign of very low risk for morbidity and mortality. Patients who
acceptable range is likely advisable. Finally, elevations in creatinine due are able to follow up with their physician within 7 days of ED discharge
to acute cardiorenal syndrome are usually due to congestion, and if after AHF therapy have significantly improved outcomes, so answer C
creatinine rises further after IV loop diuresis this typically represents is incorrect. Although option D would suggest a similarly low-risk pre-
a transient response not associated with long-term renal dysfunction. sentation, with de novo AHF it is reasonable to keep a patient for further
In other words, waiting for a creatinine level is not necessary in most testing and evaluation for their cardiac dysfunction and its underlying
cases, because worsening creatinine after furosemide rarely represents etiology. Of final note, none of the answer options suggest a patient who
a true acute kidney injury. By contrast, delayed administration of furo- necessarily requires a full hospital admission. A 24-hour-or-less observa-
semide in AHF is associated with adverse outcomes. tion stay would likely be appropriate for A,C, or D.
3. If the patient in the previous question instead had a BP of 200/125,
and was diuretic naïve, which of the following would be the best
management approach?
a. BiPAP, nitroglycerin 1–2 mg IV push repeated until SBP 150 mm
Hg, furosemide 40 mg IV push
b. BiPAP, morphine 4 mg IV push, furosemide 40 mg IV push
c. BiPAP, isosorbide mononitrate 60 mg PO, furosemide 40 mg PO
d. BiPAP, nitroglycerin 1–2 mg IV push repeated until SBP <150,
furosemide 40 mg PO
Pericardial and Myocardial Disease 68
Answer: b. Both dysrhythmias and CHF are more common in patients
KEY CONCEPTS
who experience post-MI pericarditis. Large effusions are uncommon,
• P ericarditis and myocarditis have chest pain, laboratory results, and ECG and classic pericarditis ECG findings are often overshadowed by the
findings that can mimic acute myocardial infarction (AMI). When the changes of the recent or evolving MI.
diagnosis is not clear, early coronary angiography definitively identifies or 3. A 15-year-old female presents during summer break with fever,
excludes AMI. cough, and chest pain. She has no history of illnesses or medication
• Acute pericarditis is treated with aspirin or nonsteroidal antiinflammatory use. No street drug use. Her vital signs are BP 110/70, T 38.4, P 122,
drugs. Adding colchicine decreases the rate of recurrence. RR 16, SaO2 96% on RA. ECG shows sinus tachycardia with normal
• Cardiac tamponade presents many ways. Patients with dyspnea, distended intervals and low voltage. You order a troponin test and it is slightly
neck veins, hypotension, and muffled heart sounds should have the diagno- above the normal range. Which other test result would you expect?
sis made by bedside ultrasound. Pericardiocentesis is therapeutic and may a. Cardiomegaly on chest x-ray
help establish the etiology. b. Elevated C-reactive protein
• Myocarditis should be considered in any patient with viral illness symptoms c. Elevated sodium and creatinine
and chest pain. d. Regional wall motion abnormalities on POCUS
• Patients with newly diagnosed hypertrophic cardiomyopathy should avoid Answer: d. Patients with myocarditis show regional wall motion
strenuous exertion until evaluated by a cardiologist. Beta-blockers are the abnormalities and global hypokinesis on POCUS. Even though there
mainstay of therapy; nitrates should be avoided. is inflammation, WBC and CRP are neither sensitive nor specific for
• Dilated cardiomyopathy presents with symptoms of heart failure or rhythm myocarditis. Electrolyte and renal abnormalities should not occur. Car-
disturbance. diomegaly is a late finding if it occurs at all.
• Many cardiomyopathies have genetic origins. Send patients for both cardi- 4. A 55-year-old woman presents with progressive dyspnea, chest
ology and genetics follow-up. pain, and cough over 5 days. She has a past history of renal fail-
ure and is on dialysis, last 2 days ago. She does not smoke. Vital
1. A 33-year-old man presents with a 4-hour history of left anterior signs are: temperature, 100.2°F (37.9°C) oral; heart rate, 120 beats
chest pain associated with mild shortness of breath. He is other- per minute; respiratory rate, 26 breaths per minute; blood pressure,
wise healthy except for chronic tobacco use. Vital signs are: blood 100/60 mm Hg; and oxygen saturation, 96% on room air. Physical
pressure, 142/92 mm Hg; heart rate, 120 beats per minute; respira- examination is remarkable for 3-cm jugular venous distention at
tory rate, 24 breaths per minute; temperature, 100.4°F (38.0°C) oral; 45 degrees, clear lung fields on auscultation, tachycardia without a
and oxygen saturation, 97%. Physical examination is remarkable friction rub, trace pretibial edema, and weak peripheral pulses that
for tachycardia and a friction rub. The patient’s electrocardiogram disappear during expiration. Chest radiograph shows an enlarged
(ECG) is shown below. Which of the following would be the most cardiac silhouette and clear lung fields. What would be the most
appropriate therapy? appropriate initial intervention?
a. Cardiac catheterization a. Endotracheal intubation with rapid sequence induction
b. Ibuprofen 600 mg QID and colchicine b. Enoxaparin 1 mg/kg subcutaneous
c. Nitroglycerin, aspirin 324 mg oral, cardiology consultation c. Computed tomography (CT) scan of the chest
d. Oxygen, serial troponin levels d. Isotonic fluid bolus and point of care cardiac ultrasound
Answer: b. The ECG shows findings consistent with acute pericarditis. Answer: d. This patient is presenting with cardiac tamponade, presum-
Clinically, diffuse ST elevation is seen in leads I, II, II, aVL, aVF, and V2 ably secondary to an uremic pericardial effusion. Pulmonary embolus
to V6. In contrast to finding an acute myocardial infarction (AMI), ST is a consideration but less likely, given the picture of normal oxygen
elevations are concave upward in acute pericarditis rather than convex saturation and an enlarged heart. The initial intervention should be
upward as seen with a myocardial infarction (MI). PR depression is fluid loading to maintain preload and cardiac output, followed by
also a frequent finding. As the acute pericarditis resolves, ST changes ultrasound confirmation and likely pericardiocentesis. Fluid loading
revert to normal, followed by T wave flattening and later deep symmet- in renal failure may easily result in pulmonary edema, so expeditious
rical inversion, which may persist. NSAIDs are the mainstay therapy relief of tamponade is indicated. It is too early for intubation because
for uncomplicated pericarditis. Colchicine helps decrease recurrence. the patient is oxygenating adequately.
Oxygen therapy is not helpful in patients with normal oxygen satu- 5. A 44-year-old man complains of swollen legs. He just finished two
ration. Because the ECG, examination, and history point to pericar- courses of prednisone for wheezing related to asthma. The first
ditis, cardiac catheterization is not needed. There are case reports of course was prescribed 6 weeks ago in the emergency department
thrombolytics use in pericarditis leading to pericardial hemorrhage (ED), where he was diagnosed with new onset asthma and normal
and major complications. chest radiograph. The second course was prescribed by his family
2. Which of the following statements regarding post–myocardial physician 2 weeks ago. The patient denies fever and chest pain and
infarction (MI) pericarditis is true? is still mildly short of breath, which is worse at night or with exer-
a. Classic pericarditis electrocardiogram (ECG) findings are reli- tion. Examination shows bibasilar rales in his lungs, normal heart
ably seen. sounds, and 1+ edema in both legs up to his knees. What is his
b. It indicates a greater degree of myocardial damage than those diagnosis?
without pericarditis. a. Asthma exacerbation
c. Large pericardial effusions are common. b. Idiopathic dilated cardiomyopathy (DCM)
d. The incidence of congestive heart failure (CHF) is unchanged. c. Prednisone-induced edema
d. Prednisone-induced liver failure
Answer: b. The patient is unlikely to have a new diagnosis of asthma.
He most likely had a viral process leading to reactive airway disease ini-
tially and viral myocarditis later. He unfortunately now has a DCM and
symptoms of heart failure. Treatment is supportive. Prednisone-related
end-organ damage usually does not occur this quickly.
69 Infective Endocarditis and Valvular Heart Disease 3. A 32-year-old woman with a history of injection drug use presents
with persistent fevers, night sweats, and shortness of breath. Vital
KEY CONCEPTS
signs are temperature 38.2°C (100.8°F; oral), heart rate, 123 beats/
• Infectious endocarditis (IE) is more often caused by Staphylococcus species min, blood pressure, 105/60 mm Hg, respiratory rate, 26 breaths/
than Streptococcus, and increasingly occurs in elderly, recently hospital- min, and oxygen (O2) saturation, 93% on room air. The physical
ized patients, those with prosthetic valves and intra-cardiac devices, and in examination is remarkable for scattered rhonchi and a 2/6 systolic
patients who inject opioids. murmur in the left lower sternal border. Which of the following is
• IE should be considered in any patient with a predisposing condition plus the most appropriate antibiotic to start?
fever; IE frequently presents with a complication, including heart failure, a. Clindamycin
embolic stroke, or osteomyelitis. b. Vancomycin
• At least two sets of blood cultures (obtained before starting antibiotics) c. Ceftriaxone
and transthoracic echocardiography are the key diagnostic tests to order d. Cefepime
for patients with suspected IE. Answer: b. In most cases, empiric therapy in the ED should include
• Empiric antimicrobial treatment for IE should include vancomycin. vancomycin, and vancomycin alone for one or two doses is a reason-
• Consultation with a cardiothoracic surgeon is recommended for left-sided able approach for native valve and prosthetic valve infection and in
vegetations. IDUs. Ceftriaxone can be added to vancomycin to cover HACEK and
• Acute rheumatic fever is a delayed nonsuppurative complication of strepto- other gram-negative organisms, but these are responsible for less than
coccal pharyngitis characterized by arthritis, carditis, chorea, subcutaneous 6% of infections. There is no role at this time for empiric use of clinda-
nodules, and erythema marginatum. mycin or cefepime.
• In a patient with severe mitral stenosis, hypovolemia and tachycardia are 4. A 62-year-old man with presents 3 days after being admitted for an
poorly tolerated. “Slow and full” are appropriate goals. inferior ST-segment elevation myocardial infarction (STEMI). He
• In patients with critical aortic stenosis, avoid excessive preload reduction underwent a successful percutaneous coronary intervention (PCI)
with vasodilators and diuretics. and was just discharged earlier in the day. He developed sudden
• In patients with acute aortic insufficiency, classic physical findings may onset shortness of breath at home, and he arrives in extremis. Vital
be absent. Medical stabilization entails the cautious use of vasodilators and signs are temperature 37.2°C (98.8°F; oral), heart rate, 112 beats/
diuretics. Intraaortic balloon counterpulsation is contraindicated. min, blood pressure, 85/68 mm Hg, respiratory rate, 24 breaths/
• Complications of prosthetic heart valves range from structural failure and min, and oxygen (O2) saturation, 90% on room air. The physical
thrombosis to systemic embolization, hemolysis, and endocarditis. examination is remarkable for severe respiratory distress, diffuse
crackles, and a 2/6 systolic murmur in the left lower sternal border.
1. What is the most common manifestation of acute rheumatic fever Which of the following is the most appropriate intervention?
(ARF)? a. Give 2L of crystalloid over an hour.
a. Carditis b. Start dobutamine.
b. Chorea c. Initiation of dialysis for volume overload.
c. Erythema marginatum d. Initiate continuous positive airway pressure (CPAP) therapy.
d. Polyarthritis Answer: d. This patient presents with likely acute mitral regurgitation
Answer: d. Arthritis occurs early in the course of ARF. The knees, (MR) from a papillary muscle rupture after an myocardial infarction
ankles, elbows, and wrists are commonly affected, and pain can be out (MI). Giving additional fluid will likely make his pulmonary edema
of proportion to physical findings. Cardiac manifestations are subtle worse. Starting dobutamine without a vasopressor while he is hypoten-
and may reflect endocarditis, myocarditis, or pericarditis. Chorea and sive could make the hypotension worse. When the diagnosis of acute
erythema marginatum are rare. Chorea is typically a late finding. MR is suspected, initial stabilization in the ED includes treatment of
2. A 49-year-old woman presents with progressive dyspnea on exer- pulmonary edema with intravenous nitrates, diuretics, and noninva-
tion and orthopnea. Vital signs are temperature 36.7°C (98.1°F; sive positive pressure ventilation.
oral), heart rate, 110 beats/min, blood pressure, 135/80 mm Hg, 5. A 38-year-old man with presents with progressive dyspnea over
respiratory rate, 22 breaths/min, and oxygen (O2) saturation, 97% a day, starting 2 hours before arrival. Vital signs are temperature
on room air. The physical examination is remarkable for clear lung 38.9°C (102.0°F; oral), heart rate, 115 beats/min, blood pressure,
fields and an irregularly irregular rhythm with a 4/6 diastolic mur- 120/72 mm Hg, respiratory rate, 26 breaths/min, and oxygen (O2)
mur in the left anterior axillary line. She has no peripheral edema. saturation, 92% on room air. The physical examination is remark-
Which of the following would be appropriate hemodynamic man- able for diffuse crackles and a 2/6 systolic murmur in the left lower
agement of her cardiac pathophysiology? sternal border. His echocardiogram demonstrates a large, mass on
a. Aggressive diuresis the mitral valve with severe mitral regurgitation (MR). In addition
b. β1-Agonist to increase chronotropy to managing his pulmonary edema, which of the following is the
c. Beta blocker most appropriate intervention?
d. Selective arterial vasodilator a. Administer beta-blockade to reduce the risk of embolization.
Answer: c. This patient has a picture consistent with atrial fibrilla- b. Consult Cardiac Surgery.
tion and mitral stenosis. The apical diastolic murmur and left atrial c. Start a heparin infusion.
enlargement, along with progressive dyspnea, all support the diagnosis. d. Start digoxin for heart rate control.
Tachycardia is poorly tolerated because of the need for higher left atrial Answer: b. Patients with suspected infectious endocarditis (IE) and
pressures and a longer time during diastole to perfuse across the ste- acute heart failure require immediate consultation by a cardiothoracic
notic valve. Slow and full are appropriate goals. Diuresis might decrease surgeon. In general, there has been a shift toward earlier surgical treat-
venous return. Any agent producing tachycardia would decrease dias- ment in IE, with approximately one-half of patients with left-sided
tole time and left ventricular preload. An arterial vasodilator would infection undergoing surgery during the index admission. If possible,
have little effect, given the normal blood pressure and the fact that sys- patients with left-sided IE should be managed by a multidisciplinary
temic vascular dilation would not be seen at the mitral valve level as team at a center capable of cardiothoracic surgery. There is no role for
long as the aortic valve was competent. addition of beta-blockade in this patient in heart failure. Heparin and
digoxin are not indicated for IE.
Hypertension 70
2. A 27-year-old woman presents with lower back pain for 1 week. She
KEY CONCEPTS
recently moved into the community and has been busy unpacking
• E levated blood pressure with or without associated symptoms is exceed- and moving furniture. She denies any neurologic symptoms and has
ingly common in the emergency department (ED). no medical history. Her triage blood pressure is 170/110 mm Hg,
• A true hypertensive emergency is defined by the presence of acute target but vital signs are otherwise normal. With the exception of appre-
organ damage (TOD) and is distinct from other clinical presentations. ciable muscle spasm in her lower back, the physical examination is
• Commonly encountered hypertensive emergencies include ischemic and normal. You give her 10 mg of diazepam and observe her for 1 hour.
hemorrhagic stroke, myocardial infarction, acute heart failure, aortic dis- She is feeling better but her blood pressure is still elevated, at 165/90
section, and pre-eclampsia. mm Hg. At this point you should do which of the following?
• Management strategies regarding target blood pressure reduction, rate a. Give her 0.2 mg of clonidine.
of reduction, and agent choice vary widely among the discrete conditions b. Take a more extensive history to see if she is taking oral contra-
defined as hypertensive emergencies. Thus, clinical decision making in ceptives
this context requires knowledge of the physiology and an evidence-based c. Ignore the blood pressure and discharge her without follow-up.
approach to each condition. d. Order a 24-hour urine metanephrine and normetanephrine test.
• For patients without acute TOD, immediate antihypertensive therapy is not Answer: b. This patient’s blood pressure elevation is clearly not a
needed. However, emergency clinicians play an essential role in the care of reaction to her pain, and some further questioning is needed to iden-
this group, providing screening, ongoing surveillance, and linkage to care to tify a potential cause. In young women, oral contraceptive agents are
prevent secondary complications of this disease. a treatable cause of hypertension and should be considered when ele-
• Initiation of long-term oral antihypertensive therapy in the ED for those vated blood pressure is encountered in these patients. Of note, the
with elevated blood pressure but no established history of hypertension is likelihood of contraceptive-related hypertension increases with the
not recommended; however, emergency clinicians are encouraged to refill, duration of use. It would not be unreasonable to discharge the patient
reinitiate, or up-titrate antihypertensive medications in patients with ele- without further exploring the potential cause of her elevated blood
vated blood pressure who are known to have chronic hypertension. pressure, provided some provision of follow-up was sought. The
clinical picture is inconsistent with renal artery stenosis or pheochro-
1. An 85-year-old man presents with acute onset of right-sided weak- mocytoma, and an evaluation for these conditions would be inappro-
ness that began 1 hour before arrival. His family states that he was priate at this point.
recently diagnosed with dementia and has refused to take any of his 3. A 50-year-old man complains of dull chest pain that began 4 hours
medications for more than 1 week but had otherwise been relatively before arrival. The patient states he awoke this morning feeling
healthy. Emergency medical services (EMS) report a prehospital well and that the pain began while shoveling his driveway. He has
blood pressure of 240/110 mm Hg. On examination, the patient a 20-year history of hypertension and diabetes but has always been
has clear evidence of an acute stroke with severe deficit (National compliant with his medications, which include metformin, Nor-
Institutes of Health [NIH] Stroke Scale = 24), but his gag reflex is vasc, and Diovan. His initial blood pressure is 215/120 mm Hg,
intact. His initial blood pressure is 230/110 mm Hg. A head com- and his heart rate is 100 beats/min. On examination, his lungs are
puted tomography (CT) scan shows diffuse white matter ischemic clear, heart sounds are normal, pulses are bounding and symmetric,
disease but no evidence of acute infarct or hemorrhage. The most and there are no neurologic deficits. An electrocardiogram shows
appropriate next step is which of the following? nonspecific T wave inversions in the lateral leads, with no evidence
a. Repeat the blood pressure and, if 185/110 mm Hg or higher, start of ST segment elevation or depression and normal intervals. The
serum troponin level is elevated. Which of the following is the most
a nicardipine infusion at 5 mg/hour.
b. Contact neurosurgery for intracranial pressure monitoring. likely explanation for his clinical presentation?
c. Give a dose of tissue plasminogen activator (tPA) immediately. a. He has a ruptured dissecting aortic aneurysm that is leaking
d. Give sublingual nifedipine. blood into the retroperitoneal space.
Answer: a. Despite his advanced age, this is a relatively high- b. He is suffering from a massive pulmonary embolism.
functioning individual with acute onset of a severe stroke. He is within c. He strained an intercostal muscle while shoveling.
the time window for administration of tPA, but his blood pressure is d. He is suffering from subendocardial ischemia triggered by a
excessively elevated. According to the American Heart Association/ cycle of increased afterload that began while he was shoveling.
American Stroke Association guidelines, efforts should be made to
reduce his blood pressure to less than 185/110 mm Hg before tPA
administration. Other agents, such as labetalol, would also be appro-
priate to use.
Answer: d. Increased afterload can be triggered by exertional acti-
vation of the sympathetic nervous system. In the presence of long-
standing hypertension, ventricular and aortic stiffness are likely to
develop, increasing the potential for afterload-mediated effects on the
heart. When suddenly faced with increased resistance, the left ven-
tricular pressure may rise, leading to intrinsic compression of suben-
docardial myocytes and ischemia. Coupled with earlier transmission
of the reflected arterial wave form, the diastolic coronary filling time
may also be diminished, producing a clinical picture of acute coronary
syndrome. He may also have a coronary artery lesion on which this is
superimposed but, as described, the likelihood of his clinical presenta-
tion being caused by a complete right coronary occlusion is quite low.
4. Which of the following fundoscopic findings would be the earliest
indicator of acute hypertensive retinopathy in a comatose patient
whose blood pressure is 260/140 mm Hg?
a. Copper and silver wiring appearance to the retinal arterioles
b. Focal intraretinal periarteriolar transudates
c. Cotton wool spots
d. Diffuse atrioventricular (AV) nicking
Answer: b. Focal intraretinal periarteriolar transudates are the first
abnormality to appear with acute hypertensive retinopathy, preced-
ing all other findings, including cotton wool spots and disk edema.
Other findings that are listed may also be seen, but they are indicative
of chronic, not acute, retinal involvement. It is important to remem-
ber that acute retinal abnormalities may be absent in hypertensive
emergency patients and, although fundoscopy is an important tool,
the diagnosis should be based primarily on the results of the clinical
examination.
5. A 65-year-old woman presents to the emergency department (ED)
with chief complaint of elevated blood pressure, which was discov-
ered at her dentist’s office prior to a routine dental procedure. Her
blood pressure upon admission is 187/101 mm Hg. Otherwise, the
patient has no acute complaints, however, she does note chronic
low grade chronic headaches and intermittent blurry vision. She has
had previous readings of mildly elevated blood pressure at church
health fairs, although was never formally diagnosed, and does take
medication. Which of the following steps would be appropriate in
the context of and ED visit?
a. Obtain a CT of the head to rule out intracranial pathology.
b. Administer a short-acting antihypertensive to reduce her blood
pressure prior to discharge.
c. Prescribe a diuretic such as chlothalidone or hydrochlorothia-
zide, and refer to outpatient follow up with a hospital affiliated
primary care provider.
d. Discharge home with instructions to follow up with a primary
care provider.
Answer: d. Most patients who are found to have significantly ele-
vated BP on intake vital signs measurement or who come to the ED
because BP was found to be elevated in an outpatient setting or by self-
measurement do not have an acute hypertensive emergency. While
many patients have vague symptoms, such as headache or mild blurry
vision, these symptoms do not correlate with the degree of hyper-
tension. For such patients, a head CT or acute reduction of BP is not
indicated and offers no tangible outcome benefit. This patient has a
physician and is already on medication. As such, there is no indication
to start a new medication or make a new referral at this time.
Aortic Dissection 71
KEY CONCEPTS
• A ortic dissection most commonly presents as abrupt, sharp, severe pain
maximally intense at onset in the chest or back. There may be various
seemingly unconnected associated symptoms due to altered blood pres-
sure or insufficiency of disparate vascular beds.
• Definitive diagnosis is most commonly made with an imaging study such
as computed tomography of the aorta with intravenous contrast. A combi-
nation of standardized clinical assessment, chest x-ray, and serum D-dimer
testing can effectively rule out the condition in low-risk patients.
• The critical immediate therapeutic actions are medical therapy to decrease
aortic sheer force and blood pressure control. Immediate surgical consulta-
tion is indicated.
• Surgical treatments, including endovascular stent placement, are advanc-
ing, leading to decreased mortality. This increases the importance and benefit
of early diagnosis in the emergency department setting.
1. The single most important agent to treat acute aortic dissection is: better at high-volume centers. However, patient transfer incurs a delay
a Nitroprusside. in time-sensitive care. Recent data suggest regionalization of care with
b Aspirin. transfer a mean of 50 miles to high-volume centers is associated with
c Antiadrenergic. lower operative mortality.
d Enalaprilat. 4. A 60-year-old woman presents with chest pain radiating to the back
Answer: C. The initial primary goal of care for a patient with an with onset while lifting a 50-lb pot in her garden, and her ECG
aortic dissection in the emergency department setting is to minimize demonstrates an inferior STEMI. The next appropriate step is:
extension of the dissection. This is accomplished by decreasing the a MR angiogram of the chest.
force of cardiac contraction and resulting aortic sheer forces, and driv- b Aspirin and TPA.
ing the heart rate down to 60 beats per minute or less. Acceleration of c Diltiazem infusion.
aortic blood flow, dP/dt, is minimized while maintaining blood pres- d Immediate cardiac catheterization with recommendation to
sure in the low normal range. Beta-1 adrenergic blockade therapy is the interventional cardiologist for initial aortogram.
preferred treatment. Answer: D. Aortography is valuable for the patient with STEMI but
2. A 70-year-old man with a history of poorly controlled hypertension some presentation characteristics suggestive of dissection. As time is
presents with sharp central severe chest pain. ECG, chest x-ray, and essential for percutaneous transluminal coronary angioplasty (PTCA)
serum D-dimer are normal. The next appropriate step is: or stenting for STEMI patients, the patient can be brought to the car-
a Transthoracic echocardiography in the radiology suite. diac catheterization lab and aortography performed at the initiation of
b CT angiogram of the aorta. the procedure in lieu of other imaging. The emergency physician must
c Check serial troponin, and discharge with close follow up if neg- share his or her concerns with the interventional cardiologist to plan
ative. this diagnostic approach.
d Consult surgical service. 5. The connective tissue disorder responsible for the most cases of
Answer: B. This patient had a presentation very concerning for an acute aortic dissection is:
aortic dissection. In high-risk cases, chest x-ray and D-dimer cannot a Ehlers-Danlos, type I.
not be used to rule out a dissection. These patients must receive an b Ehlers-Danlos, type IV.
advanced imaging test. c Marfan syndrome.
3. You are working in a small rural hospital when you diagnose a type A d Loeys-Dietz syndrome.
aortic dissection based on CT angiogram of the aorta. Vital signs are Answer: C. Marfan syndrome is the connective tissue disease most
heart rate 85 beats per minute and blood pressure 130/85 mm Hg. The commonly associated with acute aortic dissection.
high-volume referral center is 25 minutes away by ambulance ride. 6. There are about 150 million annual ED visits in the United States.
a Arrange for immediate transfer to high-volume center. An emergency physician who sees 3000 patients per year can expect
b Stabilize with arterial and central lines and intubation, then to see an acute aortic dissection about every:
arrange transfer. a 6 months.
c Call the intensivist to arrange medical ICU admission on labeta- b 1 year.
lol infusion. c 5 years.
d Call the general surgeon on duty to organize emergent surgery at d 15 years.
your facility. Answer: C. Aortic dissection is a rare condition, with only about
Answer: A. Surgery and endovascular interventions for aortic dis- 10,000 cases a year in the United States. If an emergency physician sees
section require coordinated teams with expertise and experience to 3000 patients, and the rate of aortic dissection is 10,000 per 150,000,000
achieve success. Outcomes for type A dissection patients are generally ED visits, this results in seeing 0.20 per year, or about 1 every 5 years.
Abdominal Aortic Aneurysm
72
Answer: B. Permissive hypotension with a goal of a systolic blood pres-
KEY CONCEPTS sure of greater than 70 mm Hg but not necessarily more than 90 mm
• A ruptured abdominal aortic aneurysm (AAA) should be considered in any Hg is a reasonable target. Aggressive crystalloid resuscitation may be
patient with otherwise unexplained abdominal or back pain. The complete harmful. Lowering the blood pressure in hypertensive patients has not
triad of pain, hypotension, and a pulsatile mass may not be present. been shown to improve outcomes in patients with abdominal aortic
• In a patient with an AAA and acute symptoms such as severe abdominal or aneurysms as it has in patients with dissecting thoracic aortic aneu-
back pain or hypotension, rupture is imminent or has already occurred. rysms. Although a CT scan can be useful to better define the aneurysm
• A patient with a ruptured AAA and initially normal vital signs can suddenly prior to surgery in stable patients, bedside ultrasound may be the only
deteriorate at any time. imaging necessary in unstable patients.
• The risk of rupture increases substantially with increased aneurysm size, 4. What is the strongest predictor of mortality in patients with a rup-
and most ruptured AAAs have diameters greater than 5 cm. tured abdominal aortic aneurysm (AAA)?
• Bedside ultrasound may be used to document an AAA or free fluid and a. Hypotension
assist in the rapid diagnosis of a ruptured AAA. b. Presentation more than 12 hours after symptom onset
• The abdominal CT scan is the diagnostic test of choice in the evaluation of c. Age older than 70
the stable patient with suspected ruptured AAA; intravenous contrast is not d. Anemia on arrival
essential in emergencies. Answer: A. Although all of the options are predictors of higher mortal-
• The patient who has had endovascular repair of an AAA remains at risk for ity, preoperative hypotension is the strongest predictor. That said, there
aneurysm rupture. is little data guiding volume resuscitation, fluid choices, and target
blood pressures in hypotensive patients with a ruptured AAA. Crys-
1. What is the most common complication of abdominal aortic aneu- talloid resuscitation before surgical repair can cause dilutional coag-
rysms (AAAs)? ulopathy and clot dislodgement by raising the intravascular volume.
a. Renal artery infarction Judicious volume resuscitation with early use of blood products and a
b. Rupture target systolic pressure of 70 to 90 mm Hg is reasonable.
c. Development of an aortoenteric fistula 5. Which of the following is most common in the patient with an
d. Ureteral obstruction intact (non-ruptured) 6-cm abdominal aortic aneurysm?
Answer: B. Although all of the options are potential complications of a. Back pain
AAAs, rupture is by far the most common. Rupture typically occurs b. Abdominal pain
into the retroperitoneum, allowing possible tamponade of the rupture c. Sensation of abdominal distention
site. Only between 10 and 30% of ruptures are freely into the perito- d. Absence of any symptoms
neum, which is often rapidly fatal. Answer: D. Most intact AAAs are asymptomatic. They may be discov-
2. An 82-year-old male presents after a syncopal episode, which ered incidentally on physical examination or a radiologic study done
occurred 1 hour prior to arrival and was without preceding symp- for other reasons or may be found in an ultrasonographic aneurysm
toms. He has a past medical history significant for hypertension screening program. Pain in the abdomen or back does not usually
and a surgical repair of an abdominal aortic aneurysm (AAA) 15 develop until the aneurysm ruptures.
years ago. Physical examination reveals a pale man with a heart rate
of 112 beats per minute, a blood pressure of 86/54 mm Hg, and a 6. A 67-year-old man presents with a several-day history of inter-
respiratory rate of 24 beats per minute. There is melena on rectal mittent periumbilical abdominal pain. Physical examination and
examination and there is no palpable mass on abdominal palpa- vital signs are unremarkable except for supraumbilical tenderness.
tion. Along with appropriate volume resuscitation, what is the most Hemoglobin is 12 g/dL. Contrast-enhanced computed tomography
(CT) scan of the abdomen details a 4.2-cm infrarenal abdominal
appropriate next step in the management of this patient?
aortic aneurysm (AAA) with an apparent old contained rupture
a. CT scan of the abdomen with contrast enhancement
with no obvious new or acute blood. All of his discomfort is relieved
b. Abdominal x-rays
by a single dose of morphine sulfate. Which of the following would
c. Immediate vascular surgery consultation
be the most appropriate course of action?
d. Addition of albumin to crystalloid infusion
a. Vascular surgery clinic follow-up in 1 or 2 days
Answer: C. The presence of gastrointestinal bleeding with evidence of
b. Emergency department (ED) observation for 6 to 8 hours with
an AAA or a history of aortic surgery is an aortoenteric fistula until
serial abdominal examinations
proven otherwise. The most common site of vascular erosion is the
c. Gastrointestinal consultation for early follow-up and upper or
third or fourth portion of the duodenum, causing hematemesis or
lower endoscopy
melena. Initial bleeding may be minor, followed days to weeks later by
d. Surgical consultation
massive bleeding. Surgical intervention is necessary. Unstable patients
Answer: D. Watchful waiting is indicated only for asymptomatic aneu-
may need to be taken directly to the OR based on a strong clinical
rysms, regardless of the size (up to 5.5 cm). The majority of “stable”
presumption.
AAAs are neither painful nor tender, and the presence of both in this
3. Which of the following is true regarding the management of a
patient suggests imminent rupture. Although the CT scan did not
patient with a suspected abdominal aortic aneurysm (AAA)?
detect acute blood, the safest course of action would be surgical con-
a. Aggressive crystalloid resuscitation to normalize the blood pres-
sultation, with early or imminent rupture as the presumed source of
sure is warranted.
this patient’s symptoms
b. Permissive hypotension with a goal of a systolic blood pressure
of 70 to 90 mm Hg is a reasonable target.
c. Lowering the blood pressure in hypertensive patients has been
shown to improve outcomes.
d. Bedside ultrasound should always be followed by a CT scan to
better define the aneurysm.
73 Peripheral Arteriovascular Disease
3. A 63-year-old male presents with acute onset of left leg pain while
KEY CONCEPTS walking. He describes it as a shock-like sensation that made his
• A cute arterial occlusion is a limb-threatening emergency requiring early knee buckle. Past history is remarkable for hypertension, diabetes
anticoagulation and Fogarty catheter embolectomy. The clinical diagnosis (diet controlled), tobacco use, and a recent lateral wall myocardial
is based on some variant of the five Ps: pain, pallor, pulselessness, pares- infarction. Current medications are aspirin, metoprolol, and lisin-
thesias, and paralysis. Confirmatory tests are unnecessary and increase the opril. Vital signs are temperature, 37.0°C oral; heart rate, 98 beats
limb’s ischemic time. per minute; blood pressure, 160/105 mm Hg; respiratory rate, 20
• Atheroembolism (blue toe syndrome) is associated with cool, painful cya- breaths per minute; and oxygen (O2) saturation, 96%. Physical
notic toes in the presence of palpable distal pulses. A proximal source examination is remarkable for left lower extremity pallor with
should be localized, most often an atherosclerotic aneurysm in the aorta or decreased light touch sensation, nonpalpable left foot pulses, and
the iliac, femoral, or popliteal artery. minimal capillary refill. What would be the most appropriate next
• Popliteal aneurysms are bilateral in 60% of patients and often coexist with step in the diagnosis and management of this patient?
an abdominal aortic aneurysm. a. Abdominal ultrasonography
• The classic Raynaud attack is triphasic: the fingers become pale, blue, and b. Arteriogram
then red. Raynaud disease has no detectable underlying cause and usually c. Serum lactate level
has a benign course. Conversely, Raynaud phenomenon has an underlying d. Thoracolumbar magnetic resonance imaging (MRI) scan
disorder, usually connective tissue disease. e. Vascular surgery consultation
• The only reliable clinical test for the detection of thoracic outlet syndrome Answer: e. This patient has acute limb ischemia from an acute arterial
is the elevated arm stress test (EAST). embolus, most likely originating from his left ventricle secondary to a
• Partial arterial lacerations continue to bleed, resulting in an expanding recent myocardial infarction. Loss of light touch sensation on physical
hematoma. Complete arterial transections initially have only moderate examination indicates jeopardized tissue viability, requiring immedi-
bleeding but can result in delayed hemorrhage. Blunt arterial injury may ate vascular surgery consultation for emergent Fogarty catheter embo-
produce intimal disruption resulting in dissection, thrombosis, or obstruc- lectomy. Reliable diagnosis of an acute arterial embolism can almost
tion. Arterial vasospasm can accompany injuries adjacent to the blood ves- always be made by history and physical examination alone. Any addi-
sel, but spontaneous resolution always occurs in the absence of arterial tional diagnostic evaluation constitutes an unnecessary delay. Serum
disruption or intimal injury. lactate level, abdominal ultrasonography, and thoracolumbar magnetic
• Aneurysms and arterial stenoses are characterized by a systolic murmur. resonance imaging (MRI) scan would not provide useful information.
Pseudoaneurysms, associated with prior surgical or trauma sites, are char- An arteriogram before going to the operating room is an unnecessary
acterized by a loud systolic and possibly a separate, faint diastolic murmur. delay and may further exacerbate limb ischemia.
Arteriovenous fistulae are characterized by a harsh “to and fro” murmur 4. A supine patient is asked to raise his foot 12 inches above the esti-
associated with a palpable thrill. mated level of the right atrium and dorsiflex the foot five or six
• Intra-arterial injection of illicit drugs into the brachial or radial artery is times. He is then brought to a sitting position with his feet hanging.
associated with immediate onset of severe, burning pain, and emergency In the absence of severe advanced ischemia, venous filling of the
department presentation with patchy blue-purple skin discoloration. Iden- foot should return in less than how many seconds?
tifying the injection site helps confirm this syndrome, which can be associ- a. 1
ated with persistent ischemia and tissue loss. b. 5
c. 10
1. Where do most arterial emboli originate? d. 15
a. Abdominal aorta e. 20
b. Femoral artery Answer: e. This bedside test is Buerger sign and can provide reliable
c. Left ventricle evidence of advanced ischemia. In the absence of severe advanced
d. Left atrium ischemia, the lower extremity veins should fill within 20 seconds after
e. Thoracic aorta being placed in the dependent position.
Answer: c. Eighty-five percent of arterial emboli originate in the heart. 5. A 73-year-old man presents with acute onset of right lower extrem-
Of these, left ventricular thrombus formation after myocardial infarc- ity pain. He has a long history of tobacco use, hypertension, and a
tion (MI) accounts for 60% to 70%. Atrial thrombi account for only 5% several-year history of moderate calf claudication at 50 yards walk-
to 10% of all peripheral arterial emboli. ing. Physical examination reveals signs of chronic atherosclerotic
2. What is the most frequent site of acute arterial embolic occlusion? occlusive disease of the bilateral lower extremities, including mus-
a. Carotid artery cular atrophy, loss of hair over the toes and feet, and thickening of
b. Common femoral artery the toenails. Examination of the distal right lower extremity reveals
c. Mesenteric artery pallor, absent popliteal and foot pulses, and decreased sensation to
d. Popliteal artery light touch of the right foot. The cardiac examination is unremark-
e. Renal artery able, and the 12-lead electrocardiogram (ECG) reveals only normal
sinus rhythm. Based on the most likely diagnosis, what is the most
Answer: b. The bifurcation of the common femoral artery accounts
appropriate definitive therapy?
for 35% to 50% of acute arterial occlusion due to arterial embolism.
a. Acute hyperbaric oxygen therapy
Embolic occlusion most often occurs at major arterial bifurcations
b. Arteriogram to determine the presence of embolus versus in situ
because of the sudden change in vessel diameter at these locations.
thrombosis
c. Intra-arterial thrombolysis
d. Surgical referral for Fogarty catheter embolectomy
e. Surgical referral for Fogarty catheter embolectomy with vascular
bypass grafting
Answer: e. This patient has a history and physical examination con- 7. A 49-year-old woman presents with severe left ankle pain. She
sistent with long-standing peripheral atherosclerotic occlusive disease, describes a fairly sudden development of a left lateral malleolus
no evidence of a proximal source for embolism, but acute onset of hemorrhagic blister that transitioned to a painful superficial ulcer
ischemic symptoms and loss of light touch in the affected extremity. over 48 hours. She has no prior history of extremity ulcers, and her
The most likely diagnosis is a large, in situ thrombosis precipitating only significant past medical history is hypertension. She has a long-
acute limb-threatening ischemia. When limb-threatening ischemia is standing history of noncompliance with her hypertensive medica-
present, emergent surgical referral for Fogarty catheter embolectomy tions and smokes two packs of cigarettes per day. She has no history
is indicated, whether caused by acute in situ thrombosis or embolus. of myalgias, joint pain, fever, or systemic symptoms. Vital signs
With limb-threatening ischemia caused by in situ thrombosis, simple are: temperature, 36°C oral; heart rate, 90 beats per minute; blood
Fogarty catheter embolectomy is insufficient and usually requires addi-
pressure, 210/125 mm Hg; respiratory rate, 20 breaths per minute;
tional bypass grafting. Acute hyperbaric oxygen therapy has no role in
and O2 saturation, 96%. Physical examination reveals a thin black
the treatment of limb-threatening ischemia due to in situ thrombosis
female in distress because of pain. Cardiopulmonary examination is
or embolism. An arteriogram to determine the presence of embolus
unremarkable. Abdominal, neurologic, and extremity examinations
versus in situ thrombosis is unwarranted, represents an unnecessary
are likewise unremarkable except for a well-demarcated, shallow
delay, and may further exacerbate ischemia. Intra-arterial thrombolysis
4 × 3 cm ulcer over the left lateral malleolus. There is mild erythema
takes 6 to 72 hours to work and is contraindicated in cases of limb-
but no evidence of active infection. Distal pulses and capillary refill
threatening ischemia.
are normal. What would be the most appropriate intervention?
6. What percentage of patients presenting with arteriosclerosis oblit-
a. Analgesics and admission for vasculitis evaluation
erans are younger than 50 years old?
b. Surgical consultation for possible embolectomy
a. 1%
c. Wound care and blood pressure control
b. 5%
d. Wound care and tapering dose of prednisone
c. 10%
e. Venous Doppler scans and surgical consultation for possible
d. 20%
skin grafting
e. 40%
Answer: c. This patient has a hypertensive ulcer, which is the most
Answer: d. Peripheral arteriovascular disease can occur in younger
painful of lower extremity ulcers. They typically occur over the lateral
patients. Nineteen percent of patients presenting with atherosclerosis
malleolus, as opposed to venous stasis ulcers, which are more common
obliterans are between the ages of 30 and 50 years old. Of all arterio-
sclerosis patients, 33% have coexistent coronary artery disease, and
anteriorly and medially. Ischemic arterial ulcers are more common dis-
tally over the digits. Although vasculitis or a collagen vascular disease
70% to 90% are smokers. The non-smokers have other risk factors
are possible, the lack of any other systemic symptoms or prodrome
including significant hypertension and hyperlipidemia.
argues against this.
74 Pulmonary Embolism and Deep Vein Thrombosis
KEY CONCEPTS
• H allmarks of deep venous thrombosis (DVT) include unilateral limb pain and • P atients with low gestalt pretest probability and a negative PERC rule may
swelling, though these findings can be subtle and nonspecific. incur more harm than benefit if testing for PE (including D-dimer) is performed.
• Patients with low pretest probability (PTP) can have DVT ruled out in the ED • Patients with non-high pretest probability (e.g., Wells Score ≤ 6) can have PE
with a negative D-dimer or venous ultrasound (US), patients with high PTP can ruled out with a negative D-dimer.
have DVT ruled out in the ED with a negative D-dimer and venous US. • The D-dimer threshold can be adjusted according to the patient’s age or the
• Three-point US evaluates the leg veins proximal to the knee and has a sen- pretest probability of PE. The formula for age adjustment is Age × 10 ng/mL.
sitivity of 95% and specificity of 95% for proximal DVT when performed by Using an adjusted D-dimer threshold reduces the need for imaging by about
a certified sonographer. A negative three-point ultrasound in a patient with 10% to 15%.
a moderate or high PTP for DVT should have a D-dimer test or repeat venous • Patients with high pretest probability or a positive D-dimer require imaging.
ultrasound within 7 days. A single, whole-leg ultrasound excludes DVT in all For most patients, including pregnant women, computer tomography pulmo-
pretest probabilities. nary angiography (CTPA) is the imaging test of choice.
• Anticoagulation for DVT and pulmonary embolism (PE) can be achieved with • For pregnant women, the decision to undergo any testing for PE should be
a direct-acting oral anticoagulant (DOAC), low-molecular-weight heparin shared with the patient. To minimize fetal radiation exposure, diagnostic test-
(LMWH), unfractionated heparin or, in rare cases, an alternative anticoag- ing for PE should start with a combination of D-dimer and bilateral lower
ulant. For DVT and most PE, DOACs are as effective as and safer than the extremity venous ultrasound.
combination of heparin and warfarin. • For patients diagnosed with PE, typical resuscitative measures can be harm-
• The treatment of isolated calf vein DVT is controversial, and it is reasonable ful. Endotracheal intubation and positive-pressure ventilation can decrease
to withhold anticoagulation in favor of a repeat venous ultrasound within 7 preload and precipitate cardiac arrest. Excessive intravenous fluid adminis-
days and close follow-up. tration can lead to worsening right ventricular distention and left ventricular
• Distal superficial vein thrombophlebitis can adequately be treated with non- compression.
steroidal antiinflammatory drugs and warm compresses, but proximal superfi- • Patients with PE should be risk stratified using a combination of vital signs,
cial vein thrombophlebitis should be treated with anticoagulation. CTPA, echocardiography, and troponin. High-risk PE is defined by hemody-
• Most patients with DVT distal to the iliofemoral region can be safely treated namic instability, intermediate-risk by right ventricular dysfunction, and low-
as outpatients as long as close follow-up and access to anticoagulant medi- risk by the absence of either.
cations is assured. • Patients with high-risk PE should be treated with thrombolysis or thromboem-
• PE can present asymptomatically, with dramatic clinical symptoms, or with bolectomy unless contraindicated.
sudden cardiac death. Even the most common symptom, dyspnea, is absent in • Patients with intermediate-risk PE are usually not candidates for thrombolysis
a quarter of patients. but may be candidates for catheter-directed or low-dose systemic thromboly-
• The first step in the evaluation of possible PE is determining whether testing sis.
for PE is indicated. • About 25% of patients with low-risk PE can be treated as outpatients.
• The patient’s pretest probability for PE, as determined by clinical gestalt or
a validated score (e.g., the Wells Score) dictates the approach to objective
testing for PE.
1. A deep vein thrombosis localized to which of the following veins 4. What is the preferred anticoagulant for a patient with low-risk PE?
can safely be treated without anticoagulation and early follow-up a. A direct acting oral anticoagulants (DOAC)
with a repeat venous ultrasound? b. A nonsteroidal anti-inflammatory drug (NSAIDs)
a. Popliteal vein c. Unfractionated heparin
b. External iliac vein d. Warfarin
c. Anterior tibial vein Answer: A. See Figure 74.11. For most patients, and especially for
d. Common femoral vein patients with low-risk PE, direct-acting oral anticoagulants (DOACs)
Answer: C. See Figure 74.2. The popliteal, external iliac, common are the treatment of choice. The DOACs apixaban and rivaroxaban are
femoral, and deep femoral veins are all proximal veins. Proximal DVT approved for the initial treatment of DVT and PE and are associated
should be treated with anticoagulation. DVT isolated to a calf vein like with lower rates of major hemorrhage than the combination of heparin
the anterior tibial vein can be observed, without anticoagulation, so and warfarin. Nonsteroidal antiinflammatory drug (NSAIDs) can be
long as a follow-up ultrasound can be performed within 7 days to look used to treat superficial thrombophlebitis, but not PE.
for propagation. 5. A 46-year-old woman presents to a tertiary-care hospital with syn-
2. In which of the following patients would the D-dimer level likely be cope and hypotension. Her oxygen saturation is 88% on room air.
elevated (e.g., >500 ng/mL)? Her systolic blood pressures are between 75 to 88 mm Hg during
a. 52-year-old female with a history of breast cancer in remission her 30 minutes in the ED. Her CTPA shows acute, bilateral main
b. 18-year-old male with an acute febrile illness pulmonary artery PE and right ventricular dilatation. Three weeks
c. 79-year-old male with a history of hypertension ago, she had surgery to resect a glioma from the left cerebral hemi-
d. 24-year-old female with a history of obesity sphere. The most appropriate next step is:
Answer: C. See Box 74.2. Active cancer increases D-dimer levels, but a. 2 L normal saline bolus and endotracheal intubation.
inactive cancer (in remission) does not. Febrile illness, obesity, and b. Consult cardiac surgery to consider surgical thromboembolec-
long-haul travel do not increase D-dimer levels. While hypertension tomy.
does not increase D-dimer levels, D-dimer levels do increase with c. Low-dose alteplase, 50 mg IV over two hours.
advancing age. The specificity of the D-dimer test in a patient 79 years d. Anticoagulation with rivaroxaban.
old is about 10% to 15%. Answer: B. See Figure 74.11. This patient has high-risk, or “mas-
3. A 29-year-old woman, G2P1, 28 weeks pregnant presents with a sive,” PE. She has persistent hypotension and a large central clot
brief episode of left-sided chest pain and vague cramping of her with evidence of right ventricular dysfunction. She also has an
left leg. She agrees that she would like to pursue a workup for PE. absolute contraindication to thrombolysis having recently under-
Which combination of tests is the most appropriate first step? gone resection of a brain tumor. Patients with right ventricular dila-
a. CTPA and bilateral venous ultrasound tation from PE may clinically decompensate with excessive IV fluid
b. Ventilation/perfusion scan and echocardiogram resuscitation and intubation. Fluid boluses, when given, should
c. D-dimer and CTPA be small (250–500 mL). Intubation should be avoided if possible.
d. D-dimer and bilateral venous ultrasound DOACs like rivaroxaban have not been extensively studied in high-
Answer: D. While the radiation doses from both CTPA and ventilation risk PE and, because of their long half-life, should not be used in
perfusion scanning are lower than the minimum dose known to affect patients who may undergo thrombolysis or invasive treatment for
pregnancy outcomes, it is still best to minimize radiation exposure their PE. Because this patient is not a candidate for thrombolysis,
when possible. Therefore, a diagnostic testing strategy that starts with the best first step is to consult a cardiac surgeon to consider open
D-dimer, which can rule out PE if negative, and bilateral venous ultra- thromboembolectomy.
sound, which can rule in VTE when positive, may obviate the need for
imaging with ionizing radiation.
75 Esophagus, Stomach, and Duodenum
KEY CONCEPTS
Dysphagia administering broad-spectrum antibiotics and parenteral nutrition, and with a
• Dysphagia can be caused by obstructive lesions (e.g., esophageal neoplasm), surgeon readily available.
motility disorders (e.g., achalasia), or neuromuscular disorders that can be
vascular (e.g., cerebral vascular accident), immunologic (e.g., myasthenia gra- Esophagitis
vis, multiple sclerosis [MS]), infectious (e.g., botulism), or metabolic in nature. • Gastroesophageal reflux disease (GERD) is a diagnosis of exclusion in patients
• The incidence of achalasia increases with age, presenting insidiously with who present with chest pain. It is critical to first rule out other diagnoses, such
equal frequency for solids or liquids. as acute coronary syndrome (ACS).
• Dysphasia can be the initial presentation of myasthenia gravis. • Empirical treatment of GERD with lifestyle modifications, H2 antihistamines,
• Treatment of dysphagia is directed toward the underlying cause (e.g., myas- or proton pump inhibitors (PPIs) is appropriate, but if there is no improvement
thenia gravis, MS). with these measures, patients should be referred for further evaluation.
• An outpatient barium swallow or upper gastrointestinal endoscopy is indi- • Sucralfate (1 g qid) can be safely used in pregnant patients with GERD.
cated for most patients with dysphagia. • Eosinophilic esophagitis commonly presents as solid food dysphagia. Once
food impaction is ruled out, a PPI should be initiated and the patient referred
Upper Gastrointestinal Foreign Bodies to a gastroenterologist.
• Structural abnormalities of the esophagus are a major risk factor for foreign • Infectious esophagitis primarily occurs in immunocompromised patients, and
body obstruction; thus patients who obtain relief of foreign body sensation endoscopy may be helpful in differentiating among causal agents.
symptoms should be referred to a gastroenterologist for follow-up evaluation. • Patients with pill esophagitis typically present with sudden onset retrosternal
• Immediate intervention is indicated for button batteries, food boluses causing pain and odynophagia in the setting of taking medications without water; the
high-grade obstruction, or patients in significant distress (e.g., vomiting, gag- diagnosis can often be made on the history alone.
ging, choking, stridor, or inability to tolerate oral intake). • Medications associated with pill esophagitis include doxycycline, aspirin,
• Urgent (<24 h, and ideally <12 h) intervention is indicated for low-grade nonsteroidal antiinflammatory drugs (NSAIDs), and potassium chloride.
obstructions caused by sharp objects, coins lodged in the proximal esopha-
gus, or food boluses. Gastritis and Peptic Ulcer Disease
• Urgent intervention is also recommended for gastric foreign bodies wider • Although gastritis cannot be definitively diagnosed based on clinical features
than 2.5 cm or longer than 5 cm. alone, a clinical history such as recent NSAID use or alcohol ingestion in
• Flexible endoscopy with procedural sedation is the preferred therapeutic the setting of classic symptoms supports a presumptive clinical diagnosis of
intervention to remove most proximal foreign bodies that can be reached by gastritis.
the scope. • The most common cause of gastritis is Helicobacter pylori infection.
• First-line treatment of H. pylori infection is a PPI (e.g., omeprazole 20 mg bid),
Esophageal Perforation amoxicillin (1 g bid), and clarithromycin (500 mg bid) for 14 days.
• Iatrogenic causes, such as a complication of endoscopy, remain the most • The most serious complications of peptic ulcer disease (PUD) include hemor-
common cause of esophageal perforation. rhage, perforation, penetration, and gastric outlet obstruction.
• Water-soluble contrast should be used for initial diagnostic imaging studies
when esophageal perforation is suspected. Gastric Volvulus
• Admission with broad-spectrum antibiotics (e.g., intravenous vancomycin 15 • Acute gastric volvulus often presents with the combination of severe epigas-
mg/kg plus piperacillin 3.375 g) and early surgical consultation should occur tric pain, distention, and vomiting, followed by violent nonproductive retching.
in the majority of cases of esophageal perforation. • Volvulus has a very high morbidity and mortality. Nasogastric tube reduction
• Select stable patients with small, contained esophageal perforation may can be attempted in the emergency department, but ultimately these patients
be managed conservatively by keeping the patient nothing per os (NPO), by need emergent surgical intervention.
1. Which of the follow drug regimens is appropriate first-line treat-
ment for Helicobacter pylori infection?
5. Which of the following is an indication for urgent endoscopy?
a. Bismuth subsalicylate, famotidine, and clarithromycin
a. Button battery in the stomach
b. Metronidazole and sucralfate b. Chest pain due to foreign body
c. Omeprazole, amoxicillin, and clarithromycin c. Coin in the proximal esophagus
d. Omeprazole and bismuth subsalicylate d. Nausea and vomiting
e. Famotidine, omeprazole, and amoxicillin e. Object failing to pass out of the esophagus after 12 hours
Answer: c. The recommended triple-treatment regimen for H. pylori Answer: c. A coin that remains lodged in the proximal esophagus
infection is a proton pump inhibitor (PPI) (e.g., omeprazole, 20 mg should be removed. Other indications are inability to handle secre-
bid), amoxicillin (1 g bid), and clarithromycin (500 mg bid) for 14 days. tions, sharp objects, esophageal button battery (alkaline), and impac-
Quadruple therapy with Pepto Bismol, metronidazole, tetracycline, tions that fail to pass after 24 hours.
and a PPI is an alternative option. See Box 75.5. 6. Ulcers in which portion of the upper GI tract are most likely to
2. What percentage of esophageal foreign bodies require a nonopera- perforate?
tive intervention to facilitate removal? a. Hypopharyngeal
a. <5% b. Esophageal
b. 10% to 20% c. Gastric—antrum
c. 50% d. Gastric—body
d. 75% e. Duodenal
e. >90% Answer: e. Duodenal ulcers account for 60% of all perforations, followed
Answer: b. Most foreign bodies pass spontaneously. Approximately 10% by antral gastric ulcers (20%) and gastric body ulcers (20%). Various
to 20% require intervention, but less than 1% require surgery for removal. upper gastrointestinal (GI) symptoms may lead to esophageal ulcer-
3. A 5-year-old child is brought to the emergency department (ED) ations and severe odynophagia, although perforation is infrequent.
by his mother after a possible ingestion of a plastic Lego piece. He
7. A 32-year-old otherwise healthy man presents with acute onset of
has had no pulmonary symptoms but reports difficulty swallowing
epigastric pain radiating to his chest that woke him from sleep at 2
and declines to drink liquids that are offered. What would be the
am. It was a burning pain associated with water brash. There were
intervention of choice?
no associated pulmonary symptoms. His past medical history is
a. Contrast-enhanced CT scan of the chest
unremarkable except for tobacco use and heartburn. His electrocar-
b. Endoscopy
diogram and upright chest radiograph are normal. Vital signs and
c. Non–water-soluble barium swallow
physical examination findings are unremarkable. He is currently
d. Posteroanterior and lateral chest radiography
pain free. His troponin level is normal. What is the most appropri-
e. Water-soluble barium swallow
ate intervention?
Answer: b. Because the patient is symptomatic, endoscopy is indicated.
a. Cardiology consultation for catheterization
A CT scan of the chest is useful for organic and inorganic materials. A
b. Contrast-enhanced CT scan of the chest
basic chest radiograph cannot reliably exclude a foreign body. Barium
c. Discharge on aspirin, 325 mg once daily
swallow is difficult in pediatric patients, particularly in a child who is
d. Serial troponins
refusing oral fluids. Water-soluble media administration risks devel-
e. Trial of twice-daily proton pump inhibitor therapy
opment of pneumonitis if aspirated. Non–water-soluble materials risk
Answer: e. Peak gastric acid secretion occurs during the early morn-
increased inflammation if leakage occurs into the mediastinum. Bar-
ing hours between 1 am and 3 am, with a typical scenario of causing
ium may also obscure subsequent endoscopic visualization.
awakening from sleep. The shared afferent neural pathway makes the
4. Which of the following statements regarding the use of glucagon in pain of gastroesophageal reflux disease (GERD) similar to that of pain
esophageal obstruction from a food bolus is true? of cardiac origin. Gastric acid secretion is lowest at approximately 6
a. Glucagon should be administered by the oral route. am; thus awakening in the morning with pain from GERD is unusual.
b. Glucagon has antiemetic properties. 8. A 45-year-old woman presents several hours after an upper endos-
c. Glucagon can facilitate passage of a food bolus localized any- copy with severe chest pain and neck discomfort. She is awake and
where in the esophagus. alert, but rates pain as “10 of 10.” What is the most appropriate test
d. Glucagon is relatively contraindicated with sharp-edged foreign
to confirm the diagnosis?
bodies.
a. Abdominal x-ray
e. The success rate of glucagon approaches 90% when used with a
b. Barium contrast esophagography
few hours of food bolus impaction.
c. Gastrografin (water-soluble) contrast esophagography
Answer: d. Glucagon is a smooth muscle relaxant, so it is theoretically
d. Ultrasound
useful only for distal esophageal foreign bodies. There are only anec-
e. Upper endoscopy
dotal reports of success with glucagon, but a literature review shows
Answer: c. We recommend an initial attempt with a water-soluble
no statistical benefit of glucagon in treating esophageal foreign bodies.
agent in patients who are awake and alert and are not at risk for aspi-
There are many adverse effects, including flushing, nausea, and vom-
ration. Barium sulfate is superior for identifying small perforations;
iting, that can potentially increase the risk of aspiration. It is contrain-
however, it may incite an inflammatory response in tissue and should
dicated for use with sharp or damaging foreign bodies. There is only
be used only if no initial perforation is identified with water-soluble
low-level evidence to support the use of effervescent agents, and they
contrast. Endoscopy is generally not recommended except in cases of
are relatively contraindicated after 24 hours because of perforation
penetrating trauma because insufflation could potentially enlarge a
concerns.
minimal transmural opening. CT imaging is a reasonable alternative.
76 Liver and Biliary Tract Disorders
KEY CONCEPTS • U pper GI bleeding in patients with cirrhosis, often from esophageal varices,
Viral Hepatitis carries a 10% to 15% mortality.
• T he clinical presentation of viral hepatitis is highly variable, and many • When treating liver-associated coagulopathies in a patient with active
cases may be asymptomatic, particularly in children. bleeding, cryoprecipitate, 1 unit/10 kg body weight, is preferred over fresh-
• Hepatitis A is transmitted through oral fecal spread, whereas hepatitis B frozen plasma.
and C are spread through parenteral or intimate contact. • Angiotensin-converting enzyme inhibiting drugs or angiotensin recep-
• Highly effective immunizations exist against hepatitis A and B viruses. tor blocking drugs should be avoided in patients with decompensated
• Postexposure, passive immunization exists for hepatitis A and B viruses, cirrhosis or ascites because plasma drug concentrations may increase,
though its use is primarily in the setting of hepatitis B exposure in nonim- renal clearance may be markedly decreased, leading to impaired renal
munized individuals. function.
• Direct-acting antiviral regimens using nucleoside inhibitors have revolu- • In patients with cirrhosis, mean arterial pressure (MAP) is an independent
tionized hepatitis treatment. predictor of mortality.
• Viral hepatitis may be a reportable disease dependent on jurisdiction. • Hepatorenal syndrome is heralded by an increasing creatinine level in the
Alcoholic Hepatitis setting of liver failure, and is associated with a high rate of mortality.
• A lcohol-induced liver disease may progress from steatosis to steatohepati- Hepatic Encephalopathy
tis to cirrhosis, and finally to hepatocellular carcinoma. It is estimated that • H epatic encephalopathy is a state of cerebral and neuromuscular dysfunc-
8% to 20% of patients with steatosis will eventually progress to cirrhosis. tion secondary to an increased ammonia level and its effect on cerebral
The risk for hepatocellular carcinoma (HCC) in decompensated alcohol- metabolism. However, the severity of hepatic encephalopathy does not
induced cirrhosis approaches 1% per year. directly correlate with the measured serum ammonia level.
• With cessation of alcohol intake, steatosis may reverse within 2 weeks. • Underlying conditions that can precipitate encephalopathy in patients with
• Laboratory tests may help distinguish alcoholic hepatitis from viral hepatitis, cirrhosis include GI bleeding, hypokalemia, infection, and dehydration.
with the former associated with milder transaminase level elevations and a rel- • Management of hepatic encephalopathy includes correction of underlying
ative predominance of AST to ALT. electrolyte abnormalities, dietary guidance, administration of lactulose
• Hypoglycemia is common in patients with alcoholic hepatitis secondary to (30–60 g/day), and rifaximin (400 mg PO every 8 hours).
caloric insufficiency, chronically depleted glycogen stores, and suppressed • In addition to lactulose and rifaximin, infusion of branched chain amino
gluconeogenesis. acids has shown benefit without increased mortality in patients with
• Management of alcoholic hepatitis is guided by severity scoring; Maddrey hepatic encephalopathy.
discriminant function (MDF) is the most commonly utilized and incorporates Spontaneous Bacterial Peritonitis
coagulopathy and bilirubin levels. • S pontaneous bacterial peritonitis (SBP) is an acute infection of ascitic
• Patients with alcoholic hepatitis and a MDF over 32 should be treated with fluid that should be considered in any patient with ascites and abdomi-
prednisone, 40 mg PO daily, or methylprednisolone, 32 mg IV daily. nal pain, fever, or unexplained clinical deterioration.
Cirrhosis • SBP is most commonly caused by E. coli and Klebsiella.
• P atients with cirrhosis most often present with complications resulting • The diagnosis of SBP is dependent on obtaining ascitic fluid for analysis.
from loss of hepatocytes, including ascites, variceal bleeding, hepatorenal • An ascitic fluid granulocyte count greater than 250 cells/mm3 (100 cells/
syndrome, or hepatic encephalopathy. mm3 in peritoneal dialysis patients) is generally an indication for antibiotic
• The targeted platelet count should generally be greater than 50,000/mm3 treatment.
prior to the performance of invasive procedures in patients with cirrhosis. • Treatment of SBP includes cefotaxime 2 g IV every 8 hours for 5 days.
Management of coagulopathy may need to precede major diagnostic or Hepatic Abscesses
therapeutic interventions. • A bscesses may be amoebic or pyogenic in nature. Abdominal ultrasound or CT
• An elevated INR or thrombocytopenia is not a contraindication to paracen- are the diagnostic imaging modalities of choice.
tesis in patients with cirrhosis. • Chest radiography may reveal a right lower lobe pleural effusion
• Albumin, 8 g per L of ascitic fluid removed, should be administered when • Imaging does not distinguish pyogenic from amebic abscesses.
volumes over 5 L are removed by paracentesis. • Treatment is initiated prior to abscess drainage.
1. Which of the following statements regarding hepatitis A is true? 4. Scleral icterus becomes clinically apparent at approximately which
a. Fecal shedding and highest infectivity coincide with symptom- serum bilirubin level?
atic disease. a. 2 mg/dL
b. In the US, approximately 20% of urban-dwelling adults are sero- b. 2.5 mg/dL
positive. c. 3 mg/dL
c. Occult disease is more common in children than in adults. d. 3.5 mg/dL
d. The incidence is consistent across ethnic groups. e. 4 mg/dL
e. The most common risk factor for children is travel. Answer: b. Icterus is often first noted in the sublingual or subun-
Answer: c. Children are more likely to have occult disease (up to 70%). gual areas when the serum bilirubin level is above approximately
Adult seropositive rates approach 50% among urban-dwelling adults. 2.5 mg/dL.
The incidence varies widely across ethnic groups. In areas of pediatric 5. Which of the following statements regarding the typical laboratory
vaccinations, increasing adult cases are seen among intravenous drug profile for a patient with acute viral hepatitis is true?
users (IVDUs) and males who have sex with males. The stage of highest a. AST is generally elevated in excess of ALT.
infectivity precedes symptoms. b. Direct and indirect bilirubin levels are elevated in almost equal
2. Which of the following statements concerning hepatitis D infection proportions.
is true? c. Lactate dehydrogenase (LDH) levels are almost always normal.
a. Hepatitis D is spread primarily via the fecal-oral route. d. The alkaline phosphatase level is generally elevated 5 to 10 times
b. Infection with hepatitis D is an independent event with a course normal.
nearly identical to that of hepatitis A. e. The WBC always shows a marked polymorphonuclear leukocy-
c. It is common to see aspartate aminotransferase (AST) level ele- tosis.
vations far in excess of alanine aminotransferase (ALT) level ele- Answer: b. The alkaline phosphatase level is rarely elevated more
vations. than 2 or 3 times normal, and LDH levels are modestly elevated. The
d. Many cases are misdiagnosed as acute or reactivated hepatitis B. WBC count may range from low, with lymphocytic predominance,
e. Unconjugated bilirubin levels are 2 or 3 times higher than conju- to a polymorphonuclear (PMN)-predominant leukocytosis. ALT is
gated levels. almost always elevated in excess of AST in the setting of acute viral
Answer: d. Hepatitis D virus infection can only occur with (coinfec- hepatitis.
tion) or after (superinfection) hepatitis B infection. It is spread via the 6. A 26-year-old woman returns for follow-up after initial evaluation
parenteral route, such as by IV drug use. Many cases are misdiagnosed for possible acute hepatitis. Her hepatitis panel has returned with
as acute or reactivated hepatitis B because HBV markers will be pos- the following results:
itive. There are no unique biochemical or laboratory patterns for any
of the viral hepatitis infections. Hepatitis D appears to have a direct Hepatitis A IgM Negative
cytotoxic potential compared with other viral causes where the host Hepatitis A IgG Negative
immunologic response primarily leads to hepatopathy. Hepatitis B surface antigen Positive
3. A 26-year-old man presents with complaints of pruritus and a raised Hepatitis B surface antigen IgG Negative
rash for 7 days. The rash has been associated with nausea and painful Hepatitis B core antigen IgM Positive
symmetrical swelling of both wrists and metacarpophalangeal joints. Hepatitis B core antigen IgG Negative
He has no past medical history and takes no medications. He works Hepatitis C antigen Negative
in a retail store. Vital signs are normal, and the physical examination
is remarkable for right upper quadrant tenderness, bilateral mild wrist Which of the following is the most appropriate diagnosis?
effusion with minimal warmth and no erythema, and diffuse skin a. Acute hepatitis A
urticaria. The remainder of the examination is negative. Blood count, b. Acute hepatitis B
chemistries, and liver studies are remarkable for WBC of 11,800 cells/ c. Immunity to hepatitis B
mm3, AST of 212 IU/L, ALT of 395 IU/L, normal alkaline phosphatase d. Previous hepatitis A
level, and total bilirubin of 2.3 mg/dL. Which of the following diagnos- e. Previous hepatitis B
tic modalities would be most likely to yield the causative diagnosis? Answer: b. Acute hepatitis A is characterized by IgM to hepatitis A.
a. C-reactive protein level Prior infection is determined by IgG antibody. Acute hepatitis B is
b. Hepatitis A antigen characterized by the presence of surface antigen and IgM antibody to
c. Hepatitis B surface antigen core antigen. Surface antigen alone may be absent late in the course of
d. Joint aspiration the disease or may present chronically unrelated to the current episode.
e. Liver ultrasound IgG to the core antigen indicates previous infection. IgG to the surface
Answer: c. A small number of patients with hepatitis B develop a pro- antigen is the best marker for immunity.
drome of arthralgias and arthritis (symmetric small joints) and derma-
titis. The dermatitis is typically urticarial but may be macular, popular,
or petechial.
7. A 39-year-old man presents with a 4-day history of abdominal pain 9. A 63-year-old man with history of alcohol abuse presents with
and nausea. He has no significant past history and takes no medi- altered mental status. His family reports 3 days of decreasing ambu-
cations. Vital signs include a temperature of 37.7°C (99.9°F) oral, lation and increasingly nonsensical conversation. He has no other
pulse of 98 beats/min, respiratory rate of 20 breaths/min, and a known past medical history and takes no medications. Vital signs
blood pressure of 119/68 mm Hg. The physical examination reveals are unremarkable. Physical examination reveals a thin, unkempt
scleral icterus, a normal cardiopulmonary examination, moderate man who is oriented to person only but is cooperative and follows
right upper quadrant tenderness without rebound, and guaiac- commands. He falls asleep easily. There is no scleral icterus, and
negative stool. Laboratory assessment reveals the following: cardiopulmonary, abdominal, stool guaiac, and neurologic exam-
inations are otherwise normal. A noncontrast CT scan of the head
Total bilirubin 9.8 mg/dL is negative for acute pathology. Pertinent laboratory findings are as
Conjugated bilirubin 4.6 mg/dL follows:
Unconjugated bilirubin 5.2 mg/dL
AST 5300 IU/L Hematocrit (HCT) 34%
ALT 8400 IU/L Hemoglobin 11.4 g/dL
Alkaline phosphatase 750 IU/L Platelet count 108,000/mm3
Albumin 3.9 mg/dL WBC 9300/mm3
INR 1.2 AST 148 IU/L
Hematocrit 42% ALT 86 IU/L
Platelet count 396,000/mm3 Total bilirubin 2 mg/dL
WBC 9900/mm3 Albumin 2.2 mg/dL
Blood urea nitrogen (BUN) 53 mg/dL Alkaline phosphatase 158 IU/L
Creatinine 0.9 mg/dL INR 1.8
BUN 38 mg/dL
Which of the following courses of action is most appropriate? Creatinine 2.1 mg/dL
a. Admission for observation and GI consultation Ethanol 0 mg/dL
b. CT scan of the abdomen with contrast Bicarbonate 30 mmol/L
c. Gastrointestinal (GI) referral for interferon therapy Sodium 133 mEq/L
d. Reassurance Potassium 3.6 mEq/L
e. Tapering course of corticosteroids Chloride 95 mEq/L
Answer: a. Altered sensorium and prolongation of the PT beyond 5
seconds or INR beyond 1.5 suggests fulminant hepatic failure. Sim- What is the most appropriate intervention?
ilarly, an unexplained elevation of the BUN or creatinine level may a. Admission for lactulose, 30 to 60 g daily, titrated to modest diar-
portend hepatorenal syndrome, which is associated with significant rhea
morbidity. The laboratory derangements warrant admission or trans- b. Determination of further therapy, admission, and treatment
fer for hydration, close observation, and gastroenterology evaluation. based on serum ammonia levels
Interferon has demonstrated some success in symptomatic hepati- c. Discharge with the family; neomycin, 500 mg every 4 to 6 hours
tis B patients but does not affect the early course. There is no role for d. Oral metronidazole, 250 mg PO every 6 hours
corticosteroids. e. Oral vitamin K for 2 weeks
8. The risk of liver injury in men increases as daily consumption of Answer: a. Ammonia accumulates in severe liver disease and
alcohol exceeds which of the following? crosses the blood- brain barrier to eventually form glutamine.
a. 10 g Ammonia levels correlate poorly with encephalopathy. Lactulose
b. 20 g is an osmotic cathartic that acidifies colonic contents, causing
c. 40 g ammonia trapping. Neomycin is a poorly absorbed aminoglycoside
d. 60 g that reduces colonic bacteria but is relatively contraindicated in
e. 80 g cases of renal insufficiency. Therapies for hepatic encephalopathy
Answer: e. 80 g of alcohol consumption is equivalent to a six-pack of that have been under clinical investigation include metronidazole,
beer, four to six glasses of wine, or three or four mixed drinks. For zinc, flumazenil, and eradication of Helicobacter pylori. Vitamin
women, the risk increases with daily consumption of more than 20 g K would have modest benefit due to loss of hepatic synthetic
of alcohol. abilities.
10. A 23-year-old G2P1 woman at 35 weeks of gestation presents What is the most appropriate treatment?
with 3 days of fatigue, anorexia, nausea, and vomiting. She reports a. Clear liquids, antiemetics, and follow-up outpatient ultrasound
moderate epigastric and right upper quadrant pain. The physical in 48 hours
examination is remarkable for icteric sclerae, slightly dry mucous b. Contrast CT scan of the abdomen
membranes, and moderate tenderness in the right upper quad- c. Hydration, antiemetics, and discharge after symptom resolution
rant. She is afebrile and her uterus is not tender. Ultrasound shows d. Intensive care unit admission for monitoring for DIC
a viable fetus at 34 weeks estimated gestation, with good cardiac e. Stabilization and urgent delivery
activity, and liver and gallbladder ultrasound reveals no obvious Answer: e. Acute liver failure of pregnancy typically presents in the
gallstones or ductal dilations but moderate hepatomegaly. Labora- later third trimester. Treatment involves aggressive fluid and electrolyte
tory analysis is remarkable for the following: support, glucose administration, and immediate delivery. Liver disease
generally resolves without sequelae. The illness is more common in pri-
AST 1050 IU/L migravidas and twin gestations.
ALT 1265 IU/L 11. What is the most sensitive and specific imaging test for acute cho-
Total bilirubin 9.9 mg/dL lecystitis?
Conjugated bilirubin 4.6 mg/dL a. Contrast CT scan
Unconjugated bilirubin 5.2 mg/dL b. Hepatobiliary nuclear scintigraphy with iminodiacetic acid
Alkaline phosphatase 328 mg/dL (HIDA) scan
Glucose 62 mg/dL c. Serum alkaline phosphatase level
Hemoglobin 9.3 g/dL d. Serum bilirubin level
Platelet count 105,000/mm3 e. Ultrasonography
Serum electrolytes Normal Answer: b. Iminodiacetic acid administered IV is taken up by hepato-
Creatinine 0.6 mg/dL cytes and secreted into bile canaliculi. Visualization of the gallbladder
Prothrombin time 14.8 sec and common duct within 1 hour has a negative predictive value of 98%.
Albumin 3.1 g/dL Sensitivity of HIDA is diminished at bilirubin levels of 5 to 8 mg/dL.
77 Pancreas
KEY CONCEPTS
• A cute pancreatitis represents a wide spectrum of disease, ranging from
mild to severe life-threatening disease with a mortality rate as high as
30%.
• The most common causes of acute pancreatitis are gallstones and chronic
alcohol consumption.
• Acute pancreatitis is diagnosed by the presence two of three criteria—
characteristic abdominal pain, serum lipase or amylase levels greater than
three times the upper limit of normal, and characteristic findings on abdom-
inal imaging.
• Serum lipase level is preferred over the amylase level because of its greater
sensitivity and specificity in diagnosing acute pancreatitis.
• Computed tomography (CT) scan is not routinely recommended in the diag-
nosis of acute pancreatitis. It should be used in cases of diagnostic uncer-
tainty and assessing for complications.
• Abdominal ultrasound should be performed to evaluate for a biliary etiology
of pancreatitis.
• Treatment of acute pancreatitis is mainly supportive with fluid resuscitation
and pain management. Lactated Ringers is preferred over normal saline
because it is more physiologic and may provide antiinflammatory effects.
There is no evidence to support one analgesic agent over another.
• Prophylactic antibiotics are not indicated in the management of acute
pancreatitis but should be used in cases of infected pancreatic necrosis or
other clear evidence suggesting sepsis or infection.
• Endoscopic retrograde cholangiopancreatography (ERCP) is only indicated
in cases of cholangitis or biliary obstruction.
• Most patients with pancreatitis require hospitalization for symptomatic
control, monitoring of hydration and nutrition status, and management of
complications.
• There are several scoring systems to aid in predicting severity and out-
comes in pancreatitis, including Ranson criteria, Acute Physiology and
Chronic Health Evaluation II (APACHE II), CT severity index (CTSI), and Bed-
side Index of Severity in Acute Pancreatitis (BISAP). They are similar in their
predictive accuracy, and each has different strengths and weaknesses.
• Chronic pancreatitis is a progressive fibroinflammatory syndrome which
impairs both exocrine and endocrine pancreatic function.
• Pancreatic cancer is the seventh most common cause of death from cancer
globally with a 5-year survival rate of only 7%.
• Surgical treatment may improve survival in patients whose pancreatic can-
cer is diagnosed early without metastasis. Most patients have advanced
disease at diagnosis.
1. Which of the following statements regarding acute pancreatitis is Answer: c. Ultrasonography has a sensitivity of 94% for gallstones fur-
true? ther imaging may be necessary to rule out biliary tract pathology. CT is
a. Esophageal ultrasound is the diagnostic imaging of modality of not routinely recommended in the diagnosis of acute pancreatitis but
choice for the diagnosis of acute pancreatitis. is recommended in cases of diagnostic uncertainty, to rule out other
b. Hypertriglyceridemia is the likely etiology of acute pancreatitis diagnoses, and to assess for complications in patients who worsen or
in those with serum triglyceride levels greater than 1000 mg/dL. fail to improve. Oral contrast material does not aggravate pancreatitis.
c. The most common cause of acute pancreatitis is alcohol abuse. A non–contrast-enhanced helical scan may also be helpful if an oral
d. Acute pancreatitis is the most lethal pancreatic disease globally. contrast agent cannot be tolerated. MRI is superior to CT in visualizing
e. Pancreatic pseudocysts form within the first week of symptom the biliary tract although often more costly and less accessible. Ultraso-
onset. nography is generally not helpful in assessing for pseudocysts.
Answer: b. Serum triglyceride levels greater than 1000 mg/dL with 4. A 28-year-old woman presents with recurrent pancreatitis. She is oth-
no other clear causes of acute pancreatitis points to hypertriglyceri- erwise healthy and takes no medications. This episode of pain was pre-
demia as the etiology. Imaging is not routinely recommended in the ceded by several similar episodes of intermittent epigastric pain that
diagnosis of acute pancreatitis. CT is the diagnostic modality of choice; lasted several hours at a time. A previous ultrasound examination of
endoscopic ultrasound (EUS) is invasive and not recommended as a the liver, gallbladder, and pancreas was normal. She does not smoke,
first line imaging test. Although acute pancreatitis is the most com- drink, or use over-the-counter medications. The physical examination
mon pancreatic disease globally, pancreatic cancer carries the highest is remarkable for moderate epigastric tenderness without rebound.
mortality. Pancreatic pseudocysts generally form after 4 weeks from Vital signs are normal. Laboratory evaluation reveals an elevated lipase
symptom onset. level and moderate leukocytosis. Urinalysis and urine pregnancy test
2. Which of the following statements about serum lipase and amylase results are normal. What would be the most appropriate intervention?
levels is true? a. After stabilization, referral to a gastroenterologist for endoscopic
a. Lipase is an enzyme which aids in the digestion of carbohy- retrograde cholangiopancreatography (ERCP)
drates. b. After stabilization, referral to a gastroenterologist for upper
b. In acute pancreatitis, amylase levels rise in 1 or 2 hours and nor- endoscopy
malize in 2 days. c. CT scan of the abdomen to rule out pancreatic pseudocyst
c. Higher lipase and amylase levels are correlated with more severe d. Repeated ultrasound examination of the liver, gallbladder, and
disease. pancreas
d. Amylase is a less specific test than the serum lipase level for the e. Symptomatic treatment only unless her clinical picture worsens
diagnosis of pancreatitis. Answer: a. Many cases of presumed idiopathic pancreatitis may be due
e. Testing for both lipase and amylase enzymes improves the diag- to small stones or sludge that cannot be seen by ultrasound examina-
nostic sensitivity and specificity for acute pancreatitis. tion but may be seen by ERCP. Pancreatic pseudocyst is more likely
Answer: d. Lipase is more specific than amylase in the diagnosis of in alcoholic pancreatitis and typically occurs gradually, several months
acute pancreatitis as amylase is produced by multiple organs other than after a severe episode.
the pancreas and lipase is produced predominantly by the pancreas. 5. A 68-year-old male presents with several months of epigastric pain,
Lipase aids in the breakdown of dietary triglycerides into free fatty decreased appetite, weight loss, and weakness. Which of the follow-
acids while amylase aids in the breakdown of carbohydrates. Levels ing statements is true?
rise within 6 to 24 hours and normalize in 3 to 7 days. Higher enzyme a. CT scan will rule out the diagnosis of pancreatic cancer.
levels have not been found to correlate with disease severity. Testing for b. His pancreatic adenocarcinoma is most likely to be confined to
both enzymes has not been found to improve diagnostic sensitivity and the pancreas.
specificity in the diagnosis of acute pancreatitis. c. Following the diagnosis of pancreatic adenocarcinoma, chemo-
3. Which of the following statements regarding the use of radio- radiation will be curative.
graphic studies for the evaluation of pancreatitis is true? d. Chronic pancreatitis is a risk factor for developing pancreatic cancer.
a. Computed tomography (CT) is indicated in the evaluation of all e. Those residing in developing countries are at higher risk for pan-
cases of pancreatitis. creatic cancer.
b. Oral administration of a contrast agent for abdominal CT may Answer: d. Those with chronic pancreatitis are at higher risk of devel-
aggravate pancreatitis. oping pancreatic cancer. Abdominal CT is the initial imaging modal-
c. The initial study of choice in suspected gallstone pancreatitis is ity of choice for the diagnosis of pancreatic cancer; however, smaller
ultrasonography. tumors (<10 mm) are better detected on EUS. Less than 20% of patients
d. CT and magnetic resonance imaging (MRI) of the abdomen are are diagnosed with localized and resectable disease. Chemoradiation is
equally accurate for visualizing the biliary tract. generally focused on slowing tumor progression and palliative therapy
e. Ultrasonography may help to differentiate pancreatitis from but is unlikely to be curative without resection. The incidence of pan-
pancreatic pseudocyst. creatic cancer is higher in developed countries.
Small Intestine 78
KEY CONCEPTS Answer: d. Although rare in the general population, internal her-
• S mall bowel obstruction (SBO) is a common clinical condition accounting nias are a recognized complication of bariatric surgery, especially when
for 2% of patients presenting to the emergency department with abdomi- a Roux-en-Y type procedure has been performed. Because of the closed
nal pain, leading to an estimated 300,000 hospitalizations annually in the loop nature of an internal hernia, they are not suitable for conservative
United States. management and require surgical intervention.
• The most common cause of SBO is adhesions from prior abdominal surgery, 4. What is the length of time from acute ischemia of the intestines to
which are found in approximately 60% of cases. Tumors and abdominal completion of transmural necrosis?
hernias account for another 20% and 10% of cases, respectively. a. 15 minutes
• The most common presenting symptoms of SBO include colicky abdominal b. 60 minutes
pain, abdominal distention, nausea, or vomiting. However, no single com- c. 2 hours
ponent of the history or physical exam can reliably predict SBO. d. 6 hours
• Initial evaluation should focus on identifying SBO patients in need of e. 24 hours
prompt surgical evaluation including findings concerning for peritonitis, Answer: d. Although the mesenteric circulation is able to adapt
bowel strangulation, or ischemia. to variations in circulation, the small bowel is quickly injured after
• For patients presenting with SBO likely due to adhesions, most can be suc- acute ischemia. Within 15 minutes, structural damage to the intestinal
cessfully managed conservatively with bowel rest, pain control, intrave- villi can be seen histologically. If blood flow is not restored, mucosal
nous fluid, and electrolyte replacement. sloughing can start to occur within 3 hours and, by 6 hours, transmural
necrosis is complete.
1. What is the most common cause of small bowel obstruction in the 5. A 35-year-old woman taking oral contraceptive pills presents with
developed world? 2 days of progressively worsening diffuse abdominal pain without
a. Gallstone ileus peritoneal findings on examination. A CT scan reveals mesenteric
b. Hernias venous thrombosis. What is the next most appropriate step?
c. Intussusception a. Arrange for formal mesenteric venous angiography to confirm
d. Postoperative adhesions and treat.
e. Tumors b. Arrange for immediate exploratory laparotomy regardless of
Answer: d. In the developed world, postoperative adhesions account current clinical status, given the high risk of severe complica-
for approximately 60% of cases of small bowel obstruction. Patients with tions.
a history of intestinal or pelvic surgery are at the highest risk. c. Discharge home because this will resolve without intervention.
2. Which of the following patients is at the highest risk of developing d. Institute pain control and admit to the floor.
an obturator hernia? e. Start anticoagulation with therapeutic dosing of heparin.
a. 2-year-old boy with no known medical problems Answer: e. The treatment of mesenteric venous thrombosis is
b. 45-year-old woman with a 1-year history of hysterectomy unique in that in the absence of peritoneal findings, initial treatment
c. 67-year-old man with a history of metastatic prostate cancer with heparin alone may be adequate. In the vast majority of cases
d. 80-year-old woman with a 3-month history of rapid weight loss (>75%) an underlying inherited or acquired hypercoagulable state can
Answer: d. This type of hernia is especially common in older women be identified. Oral contraceptive use accounts for 9% to 18% of cases
who have recently lost a significant amount of weight. The female pelvis in young women.
is wider, and the obturator canal is more oblique in women. This, in
combination with a loss of preperitoneal fat, predisposes to its devel-
opment. Because an external mass is absent, diagnosis is especially
challenging and explains why it carries the highest mortality of any
abdominal hernia – nearly 70% when incarcerated.
3. A 55-year-old woman with a history of Roux-en-Y gastric bypass
surgery presents with one day of worsening colicky abdominal pain
and vomiting. A CT scan reveals an internal hernia. What is the
most appropriate disposition?
a. Administer broad-spectrum antibiotics and admit to the medi-
cine floor.
b. Arrange for barium swallow with small bowel follow-through.
c. Insert nasogastric tube and admit to the medicine floor.
d. Prompt surgical consultation and preparation for surgery.
79 Acute Appendicitis
2. Which of the following statements regarding ultrasonographic
KEY CONCEPT visualization of the appendix is true?
• A ppendicitis is a progressive illness caused by appendiceal luminal disten- a. A compressible appendix is a positive finding.
tion followed by appendiceal wall ischemia, transmural inflammation, and b. An appendiceal diameter greater than 6 or 7 mm is a positive
eventual perforation, with resulting peritonitis. finding.
• Clinical history, physical examination, and laboratory findings need to be c. The sensitivity of ultrasound for appendicitis is 94% to 98%.
combined to formulate a comprehensive assessment. No one finding can d. Ultrasonography has good reliability for detecting a retrocecal
definitively diagnose or exclude appendicitis. appendix.
• The most useful historical features in evaluating appendicitis are right e. Ultrasonography compares favorably with computed tomogra-
lower quadrant (RLQ) pain, pain preceding vomiting, and migration of pain phy (CT) scanning for the detection of appendicitis.
to the RLQ. Answer: b. A noncompressible appendix with a diameter greater
• The most useful physical findings in evaluating appendicitis are RLQ tender- than 6 or 7 mm in a setting of clinical features of appendicitis is con-
ness or rigidity. sidered a positive finding. Ultrasound sensitivities are 75% to 90%. It is
• Cervical motion tenderness is not specific for pelvic pathology and is found a less useful modality in the obese, those with peritoneal adhesions, or
in up to 28% of females with appendicitis. those with a retrocecal appendix. The sensitivity of CT is much higher
• A rectal examination contributes little and should not be routinely per- than ultrasonography.
formed in the evaluation of appendicitis.
• The white blood cell count alone is neither sensitive nor specific for appen- 3. A 27-year-old G3P2 woman at 22 weeks of gestation presents with 2
dicitis and offers little in the evaluation. days of right lower quadrant (RLQ) abdominal pain. It began mid-
• When clinicians have a low pretest probability for appendicitis, the combi- line and later became more pronounced in the RLQ. The physical
nation of a WBC count < 10,000/mm3 and CRP < 8 mg/L support the exclu- examination is remarkable for RLQ tenderness without rebound.
sion of appendicitis as a likely diagnosis. The gynecologic examination is negative except for a nontender
• A young, healthy adult patient with classic symptoms and signs of appen- gravid uterus, with good fetal movement by transabdominal
dicitis should receive graded compression ultrasound as the initial imaging ultrasound. Urinalysis shows 8 to 10 white blood cells (WBCs)/
of choice, if available. If the ultrasound is negative or inconclusive and high-power field (HPF) and occasional bacteria. Complete blood
concern for appendicitis persists, the patient should receive a CT of the count (CBC) shows a WBC count of 12,700/mm3 with 77% neutro-
abdomen and pelvis with IV contrast, which is the most sensitive test for phils. Hemoglobin level is 11 g/dL. RLQ ultrasound examination is
appendicitis. There is no need for oral or rectal contrast when ordering a CT limited, without visualization of the appendix or secondary signs of
for appendicitis. appendicitis. Transvaginal ultrasound does not show a gynecologic
• When there is a concern for acute appendicitis, pregnant women should or obstetric problem. Repeat examination shows continued RLQ
first have graded compression ultrasound followed by MRI if the ultrasound tenderness. What is the most appropriate intervention at this point?
is negative or nondiagnostic. a. Administer cephalexin for urinary tract infection and schedule a
• Nonoperative management of acute appendicitis (IV antibiotics, admission) 48-hour clinic recheck
is gaining support. The patient should not have high-risk features on imag- b. Admit for observation and serial examination
ing (e.g., presence of a fecalith, abscess, tumor, fluid collection, or appendi- c. Obtain surgical consultation for laparotomy
ceal diameter > 1.1 cm) and should be made aware of the risks of treatment d. Order a CT scan of the abdomen.
failure and recurrent appendicitis, both of which generally require surgical e. Order a magnetic resonance imaging (MRI) scan
removal of the appendix. Answer: e. MRI for appendicitis is helpful in pregnant women, and
• Because of the risk of malignancy, guidelines recommend follow-up colo- radiation exposure should be minimized. Exploratory surgery carries
noscopy for patients > 40 years old treated with nonoperative management. significant risk of premature labor and fetal loss.
• Delays of 12 to 24 hours between appendicitis diagnosis and operative
management are not associated with adverse outcomes. Many sur- 4. In young patients with classic symptoms and signs of appendicitis,
geons will perform an appendectomy the following morning if a patient what is the most appropriate initial intervention?
presents in the evening or overnight hours. Early appendectomy should a. Antibiotics and serial abdominal examinations
be considered in elderly patients, because up to 41% of patients 65 b. CT scan of the abdomen
years or older develop perforation between diagnosis of appendicitis c. MRI scan of the abdomen
and surgery. d. Surgery
e. Ultrasonography
Answer: e. Ultrasound is the most appropriate initial interven-
1. What percentage of women with acute appendicitis have accompa- tion because it uses no radiation and can often visualize and diagnose
nying cervical motion tenderness (CMT)? appendicitis without significant delay. It has become less common for
a. <5% a patient with a history and examination concerning for appendicitis
b. 15% to undergo surgery without imaging, and the use of advanced imaging
c. 25% has dramatically decreased the negative appendectomy rate. Graded
d. 50% compression ultrasound for appendicitis is specific but lacks the sen-
e. 75% sitivity of CT, so if clinical suspicion persists after a negative or non-
Answer: c. Prior to the advent of routine imaging of the appendix, diagnostic ultrasound, CT (or MRI in pregnant patients) is often the
as many as 25% of women with acute appendicitis were initially misdi- best next step.
agnosed because of the presence of CMT.
Gastroenteritis 80
KEY CONCEPTS
• G astroenteritis is usually self-limited and requires supportive care only. incidence of hemolytic uremic syndrome (HUS) or thrombotic thrombocytope-
Routine laboratory testing or stool cultures are not indicated for most nic purpura (TTP).
patients. • In patients with fever or bloody stools, if antimotility agents are to be given,
• Patients with gastroenteritis associated with fever, dysentery, bloody stools, they should be administered in combination with an antibiotic as these agents
severe dehydration, sepsis, a suspicion for Clostridium difficile, or an immu- may increase the contact time of the toxin or invasive infectious organism
nocompromised state should generally have a complete blood count, electro- within the intestinal mucosa.
lytes, and stool cultures evaluated. • Classic food poisoning usually manifests 1–6 h post ingestion of preformed
• Caution should be used in the care of the very young and elders with gastro- toxins from bacterial organisms such as Staphylococcus, Bacillus cereus, or
enteritis, because these populations have the highest morbidity and mortality. Clostridium perfringens. Food poisoning is generally short-lived (24 h), and the
• Evaluate all individuals with gastroenteritis for dehydration. Use of oral rehy- treatment is typically supportive care only.
dration therapy (ORT) with hypotonic oral rehydration solution (e.g., World • Risk factors for C. difficile colitis include recent antibiotic use (1–4 weeks),
Health Organization [WHO]-modified 245 mOsm ORT solution) is recom- recent hospitalization, living in a long-term care facility, solid organ transplant
mended. recipients, or the use of antacids.
• The use of antiemetic medications in patients with gastroenteritis may help • Evaluation with abdominal computerized tomography (CT) for toxic megacolon
with ORT. Ondansetron 0.15 mg/kg up to 8 mg given orally (PO) or intrave- and pseudomembranous colitis is recommended for patients with C. difficile
nously is safe and cost effective. colitis who are older, appear septic, have a tender distended abdomen, or
• Early continuation of normal feeding is advised, and continuation of breast- have a high leukocytosis. Consider empirical antibiotic treatment with vanco-
feeding in mothers with gastroenteritis is suggested, if possible. mycin 125 mg PO every 8 h for 10 days for suspected C. difficile.
• Campylobacter and Salmonella are the top two causes of culture-proven bac- • Scombroid poisoning results from eating spoiled dark meat fish; scombroid
terial enteritis in developed countries. Consult health authorities for local or poisoning is not an allergic reaction but results from ingestion of histamine.
regional outbreaks and pathogen prevalence. • In addition to the regular enteropathic bacterial pathogens, human immuno-
• Norovirus, previously referred to as the Norwalk-like virus, is the most com- deficiency virus (HIV)-positive patients, particularly those with a CD4+ count
mon cause of acute gastroenteritis in children and adults. less than 200/mm3, have increased susceptibility to certain viruses and para-
• Bacterial pathogens are responsible for approximately 80% of all cases of sites, such as Cytomegalovirus, Cyclospora, Cryptosporidium, Isospora, Myco-
traveler’s diarrhea, which is usually self-limited. Enterotoxigenic Escherichia bacterium avium-intracellulare complex, and Giardia.
coli (ETEC) is the most common etiology. • Consider Giardia for diarrhea lasting more than 2 weeks, with foul-smelling
• Antibiotic treatments can prolong the shedding of Salmonella non-typhi stools and symptoms of flatulence, abdominal bloating, cramping, and recent
organisms and therefore are not indicated in most gastrointestinal infections. exposure to contaminated river water.
• Avoid antibiotic treatment in patients with gastroenteritis caused by E. coli • Hand hygiene with soap and water or hand sanitizers will contain the spread
O157:H7 or Shiga toxin 2–producing organisms as treatment increases the of most infectious agents in gastroenteritis.
1. After eating undercooked ground beef, a patient develops fever, Answer: c. This patient is most likely infected with Shiga toxin–
abdominal pain, bloody diarrhea, and rectal tenesmus. Local health producing E. coli (ETEC) which often results from eating undercooked
authorities are investigating a common source from a local diner, as ground beef. Watery diarrhea that becomes bloody with significant
10 other patrons have presented with the same symptoms. Which of abdominal pain is typical. The diarrhea may be grossly bloody and mimic
the following organism is likely to produce this dysentery outbreak? inflammatory bowel disease or intestinal ischemia. Toxin assays are rec-
a. Giardia ommended. Endoscopic findings would be identical to those of any other
b. Norovirus severe colitis. Antibiotics are not effective and may increase the risk of
c. Shiga toxin–producing Escherichia coli (O157:H17) hemolytic uremic syndrome (HUS) in children. Thrombotic thrombocy-
d. Staphylococcus aureus topenic purpura (TTP) is a risk in older children and adults. Norovirus
typically causes watery non-bloody diarrhea without severe abdominal
pain. S. aureus causes the classic food poisoning with abdominal cramping
and vomiting 1 to 6 hours after ingestion the preformed toxin.
2. A patient from a resource-rich country has traveled to a resource- 4. A patient comes to the emergency department (ED) with a com-
poor developing country. He develops diarrhea, up to four times a plaint of abdominal cramping and vomiting twice; the first time was
day, with yellow watery stools. There is no fever or abdominal pain. the food ingested from a buffet 8 hours ago and the second time
Which of the following organism is most commonly the etiology of yellowish vomitus. There is no diarrhea or fever. There is contin-
Traveler’s Diarrhea? ued retching intermittently in the ED. What is the likely causative
a. Enterotoxigenic Escherichia coli (ETEC) organism?
b. Giardia a. Clostridium perfringens
c. Shigella b. Escherichia coli O157:H7—Shiga-producing toxin
d. Vibrio noncholera c. Norovirus
Answer: a. ETEC is the most common cause of traveler’s diarrhea, d. Salmonella enteritidus
especially with travel to resource poor countries. It is usually self- Answer: a. Clostridium perfringens is a common source of foodborne
limited, requiring antibiotic treatment only in severe cases. Giardia illness. Food cooked in advance, cooled, and rewarmed is the typical
causes persistent diarrhea for more than 14 days. Giardiasis is com- culprit. This is a toxin-mediated, diarrhea-predominant syndrome that
monly acquired from the ingestion of contaminated water, classically a is almost always self-limited. Ingestion of live organisms or spores is
camper who has ingested river water. The symptoms include diarrhea, required, and antibiotics are not indicated. Symptoms usually appear
abdominal cramping, and bloating, with foul-smelling stools. Shigello- within 6 to 12 hours, slightly longer than for staphylococcal food
sis usually causes a more severe illness with dysentery. poisoning.
3. A 5-year-old patient with a likely viral gastroenteritis for 3 days is 5. A patient presents with non-bloody diarrhea for 5 days, crampy
determined to be dehydrated. What is optimum mode of rehydra- abdominal pain, and bloating. The white blood cell (WBC) is
tion therapy in this patient? 16,000/mL. Which of the following types of patient and scenario is
a. High-glucose solution (e.g., apple juice) most likely to put the patient at high risk for Clostridium difficile?
b. IV normal saline (0.9%) a. A 3-year-old toddler in daycare
c. Oral hydration with clean water b. Elderly nursing home patient
d. Reduced osmolarity (245 mOsmol/kg) oral rehydration solution c. Nurse who works in the emergency department
Answer: d. Fluids containing glucose and electrolytes provide optimal d. Traveler on vacation returning from Mexico
rehydration due to the cotransport of water across the intestinal lumen. Answer: b. Elderly nursing home patients have the highest risk for
Reduced osmolarity rehydration solution (e.g., WHO ORS 245 mOsm/ acquiring C. difficile. These patients are in close quarters and have mul-
kg) is associated with a reduced need for unscheduled IV infusions, tiple daily contacts with staff members, making person to person trans-
lower stool volume, and less vomiting. High-glucose solutions have mission more likely. Antibiotics and antacids are frequently utilized
high osmolarity, and pure water has a low osmolarity; thus neither are in these patients, which are risk factors for C. difficile. The C. difficile
optimal for water resorption. IV hydration is generally not indicated or spores are highly resistant to heat, acid, and antibiotics, making them
cost effective in patients with mild or moderate dehydration. highly contagious for person to person or surface to person infections.
The WBC is generally greater than 15,000/mL.
81 Large Intestine
KEY CONCEPTS
Irritable Bowel Syndrome for use only when necessary. Refractory exacerbations for both CD and UC
• Irritable bowel syndrome (IBS) is a chronic disorder that includes both abdom- may be treated with immunomodulators or biologic agents.
inal pain and bloating and is either diarrhea-predominant, constipation- • Common complications of IBD include the formation of fistulae, strictures,
predominant, or a mixed picture. or abscesses; less common yet more severe complications include fulminant
• Treatment of IBS is challenging and commonly involves a combination of diet, colitis, toxic megacolon, or intestinal perforation.
pharmacological, and behavioral therapy, along with reassurance. • Chronic inflammation can have extraintestinal effects. Thromboembolic
events affecting the venous and arterial systems in IBD patients may be
Diverticular Disease underdiagnosed.
• Diverticular disease can consist of diverticulosis, which can cause bleeding
but is often asymptomatic, or inflammation of the involved diverticula, termed Colonic Ischemia
diverticulitis. • Colonic ischemia (CI) typically presents in elders and is the most common
• Uncomplicated diverticulitis can typically be managed on an outpatient basis, ischemia disorder of the GI tract. Local hypoperfusion and reperfusion injury
with oral antibiotics if indicated. Complicated diverticulitis may require cause crampy abdominal pain over the segment of the colon involved, fol-
admission and intravenous antibiotics, often with surgery consultation. lowed by a short course of bloody diarrhea.
• Although abdominal computed tomography (CT) is not diagnostic for CI, it is
Large Bowel Obstruction useful in supporting the clinical suspicion, assessing the extent of colon involve-
• The most common cause of large bowel obstruction in the United States is ment, diagnosing complications, and excluding other disorders. Colonoscopy
colorectal malignancy. Treatment consists of endoscopic placement of an within 48 hours of symptom onset is the most accurate diagnostic study.
intraluminal stent with surgical resection at a later date. • CI is usually self-limited, and most patients experience resolution of symp-
• Acute colonic pseudo-obstruction commonly occurs in hospitalized elders. toms with supportive medical management with bowel rest, hydration, and
Treatment involves medical management, with endoscopic decompression if pain management. Those with more severe disease, especially those with
refractory. right-sided colonic involvement, may develop peritoneal signs from transmu-
ral ischemia requiring antimicrobials and possible surgical intervention.
Volvulus
• Sigmoid volvulus often affects elders or those that are in long-term care facil- Stercoral Colitis
ities and can usually be treated by endoscopic decompression. • Stercoral colitis is a rare complication of chronic constipation that is typically
• Cecal volvulus often affects younger patients and requires surgical manage- diagnosed on CT and is associated with high mortality. The condition primarily
ment. affects the elderly, individuals living in nursing homes, young patients with
neurological impairment, and those with chronic opioid use.
Intussusception • Fecal impaction leads to increased intraluminal and colonic wall pressure,
• Intussusception is the second most common cause of an acute abdomen in which then causes inflammatory changes and pressure wall necrosis of the
the pediatric population after appendicitis, is usually idiopathic, and may be colon. It can be complicated by CI, stercoral ulcer formation, and subsequent
successfully treated with hydrostatic or pneumatic reduction. perforation.
• In adults, intussusception is often associated with neoplasm or malignancy,
typically requiring surgical management. Radiation Proctocolitis
• Radiation proctocolitis occurs commonly in those that have received pelvic
Inflammatory Bowel Disease radiation. Acute radiation proctocolitis occurs during the course of treatment
• Inflammatory bowel disease (IBD) is characterized by chronic, relapsing, and and is often self-limited. Chronic radiation proctocolitis can occur months
remitting inflammatory disease. Crohn disease’s (CD) transmural inflamma- after completion of treatment. Management typically varies based on severity
tion can affect any area of the gastrointestinal (GI) tract, whereas ulcerative and can range from stool softeners to hyperbaric therapy.
colitis (UC) inflammation is more superficial and is limited to the colon and
rectum. Neutropenic Enterocolitis
• Effective management of IBD requires prompt recognition and treatment of • Neutropenic enterocolitis, or typhlitis, occurs in those that have hematologic
acute relapses, and appropriate choice and monitoring of medications for malignancies, are undergoing chemotherapeutic regimens, or are immuno-
maintenance of remission. Most patients with CD respond to ileal-release suppressed for other reasons. It is characterized by neutropenia, fever, and
budesonide or systemic steroids; aminosalicylates are less effective. In con- abdominal pain. Treatment involves broad-spectrum antibiotics, bowel rest,
trast, aminosalicylates are first-line therapies for UC, with steroids reserved and interventions to stimulate leukocyte count recovery.
1. A 59-year-old man with a past history of diverticular disease pres- 4. A 29-year-old woman presents with a 4-month history of inter-
ents with his second episode of left lower quadrant (LLQ) abdom- mittent abdominal pain with bloating and diarrhea. The diarrhea
inal pain. He is afebrile, and laboratory examination is remarkable has been watery, nonbloody, and often nocturnal. Physical exam-
for a leukocytosis of 13,800/mm3. Physical examination reveals ination is remarkable for mild diffuse abdominal tenderness and
moderate LLQ tenderness without masses or rebound. A com- brown, guaiac-positive stool. Rectal examination also demonstrates
puted tomography (CT) scan of the abdomen reveals a small (4 cm) a small anal fissure at the 3-o’clock position. Laboratory evaluation
abscess adjacent to the sigmoid colon, with moderate diverticulitis. is remarkable only for a normocytic anemia with a hemoglobin
Which of the following would be the most appropriate treatment? level of 11.5 g/dL. The diagnosis would most likely be confirmed by
a. Admission for intravenous antibiotics which of the following?
b. Confirmation with double-contrast barium enema a. Colonoscopy
c. Discharge on oral antibiotics with 2-day follow-up b. CT scan of the abdomen
d. Radiology consultation for percutaneous drainage c. Erythrocyte sedimentation rate
e. Surgical consultation for laparotomy d. Mesenteric angiography
Answer: a. Abscesses smaller than 5 cm are typically treated with e. Response to a high-fiber diet
intravenous antibiotics followed by an outpatient oral regimen. Larger Answer: a. This presentation is typical for inflammatory bowel disease.
abscesses may be drained surgically or percutaneously. Nocturnal diarrhea, blood in the stool, and presence of an eccentric
2. What is the most common cause of large bowel obstruction in the US.? (nonposterior midline) anal fissure argue against IBS or benign diar-
a. Adhesions rhea. Ischemic colitis is unlikely in this age group. Endoscopy with
b. Colon cancer biopsy is the diagnostic intervention of choice.
c. Diverticulitis 5. Which of the following statements regarding colonic ischemia is
d. Intussusception true?
e. Volvulus a. CT scanning of the abdomen is diagnostic
Answer: b. Colon cancer accounts for 53% of cases. Volvulus and b. It is rarely associated with bloody stool.
diverticulitis account for 17% and 12% of cases, respectively. Extrinsic c. It is typically due to nonocclusive disease.
compression from malignancy accounts for 6% of cases. Adhesions, d. Isolated right-sided ischemic colitis is associated with compara-
unlike for small bowel obstructions, are a rare cause of large bowel tively lower mortality.
obstructions. e. Specific serum biomarkers may be helpful.
Answer: c. Colonic ischemia is typically due to nonocclusive micro-
3. Which of the following statements regarding intussusception is
vascular disease. It occurs due to low-flow conditions related to con-
true?
gestive heart failure (CHF), renal failure, hypovolemia, or recent illness
a. Bowel obstruction typically occurs.
or surgery. Bloody stools are predictable. It may also occur in younger
b. CT scans have a high sensitivity for detection of intussuscep-
patients with collagen vascular disease, hematologic abnormalities, or
tion.
cocaine abuse. Isolated right-sided ischemic colitis is associated with
c. Most adult cases involve the large bowel.
comparatively higher mortality. There are no sensitive or specific labo-
d. Most adult cases require surgery.
ratory tests, and although sometimes suggestive, CT scanning primar-
e. Most children have a causative lesion.
ily rules out alternative processes.
Answer: d. In contrast to children, most adult cases have a causative
pathologic lesion, usually located within the small bowel. Complete
obstruction occurs in less than 20% of cases. CT scans may miss as
many as 50% of cases of intussusception. Surgery is usually required for
adults with intussusception.
Anorectum 82
KEY CONCEPTS
• A norectal conditions can be approached using an algorithm that addresses
the presence or absence of pain, bleeding, swelling, and pruritus, in combi-
nation with an assessment of the patient’s overall health (see Fig. 82.2).
• Patients who seek treatment for nonspecific anorectal complaints should
be evaluated for the presence of underlying systemic disease (e.g., cancer,
diabetes mellitus, immunodeficiency) because disorders of the anorectum
may be the initial presentation of associated conditions.
• Patients with a sexually transmitted disease (STD) should be evaluated for
HIV infection, use of the anus for sexual purposes, and the possibility of
domestic violence or abuse.
• Most anorectal conditions can be symptomatically improved by adherence
to the WASH regimen (warm water, analgesics, stool softeners, high-fiber
diet).
• Thrombosed external hemorrhoids are covered by modified anoderm and
may be excised and drained within 48 hours of formation. After 48 hours,
the clot begins to dissolve, and conservative management with the WASH
regimen is indicated.
• Internal hemorrhoids are covered with mucosa and should be referred to a
colorectal surgeon for definitive management.
• Acutely thrombosed, gangrenous fourth- degree internal hemorrhoids
require urgent surgical consultation.
• Fistulous tracts should not be probed.
• Pilonidal abscesses should be drained with needle aspiration or with a lon-
gitudinal incision off the midline.
• Pruritis ani is caused by a variety of conditions, including infection, hemor-
rhoids, topical irritants, cutaneous conditions, cancer, and hypersensitivity
to foods and drugs (see Box 82.5).
• Sensitivity is required when managing patients with anorectal foreign
bodies. Health care provider safety is imperative when evaluating foreign
bodies with sharp edges.
• Distal anorectal foreign bodies often can often be removed in the ED,
whereas proximal or sharp ones should be removed in the operating room.
1. Which of the following modifying factors is most likely to lead to Answer: e. Making a longitudinal incision lateral to the midline spares
the development of hemorrhoids as a result of portal hypertension? the area that is most vulnerable to reaccumulation of debris and the
a. Alcoholism inflammatory response. Removal of pus, inflammatory tissue, and
b. Obesity ingrown hairs relieves the pain of the abscess and promotes healing.
c. Pediatric age group 4. A 68-year-old woman presents with hemorrhoids. On examination,
d. Pregnancy you note three protruding masses that are maroon in color and may
e. HIV infection be reduced manually into the anorectal opening. What is the most
Answer: c. Portal hypertension does not predispose to hemorrhoids, except appropriate curative therapy?
in children. Bleeding in adults is likely from rectal varices. Most hemor- a. Application of a concentrated sucrose solution to the affected area
rhoidal bleeding is from the superior rectal artery and thus is bright red. b. Emergent surgical intervention (e.g., banding, sclerosing, hem-
Approximately one-third of pregnant patients experience hemorrhoids in orrhoidectomy)
the third trimester or postpartum period. Traumatic deliveries can result c. Prescription of a 7-day course of topical corticosteroid cream
in hemorrhoid development. Painless bleeding with defecation is the most d. Referral to a surgeon for an outpatient procedure
common symptom (pain usually occurs if hemorrhoids are thrombosed). e. Removal of the hemorrhoid with an elliptic incision, including
2. What is the most common cause of sudden-onset rectal pain? overlying tissue
a. Anal fissure Answer: d. Third-degree internal hemorrhoids may be manually
b. Proctalgia fugax reduced in the emergency department (ED) but are unlikely to heal
c. Sacral radiculopathy spontaneously. Referral for operative therapy is curative. Excision of
d. Thrombosed external hemorrhoid internal hemorrhoids is contraindicated. Acutely thrombosed external
e. Thrombosed internal hemorrhoid hemorrhoids may be excised in the ED. Temporizing measures include
Answer: a. Anal fissures typically result from superficial tears in the using the WASH regimen—using warm water to encourage reduction
anoderm, usually occurring in the posterior midline. The pain is of the protruded hemorrhoids, maintaining hygiene, analgesics, stool
heightened by secondary spasm of the anal sphincter. softeners to ease passage of stool, and a high-fiber diet. Topical cortico-
3. Which of the following approaches to performing incision and steroids may be used for 1 or 2 days during an acute exacerbation, but
drainage of a pilonidal cyst is most appropriate? their continued use promotes skin breakdown and itching. A sucrose
a. Elliptic incision solution may prove helpful in reducing procidentia (rectal prolapse). A
b. Horizontal incision at the center of the affected area thorough history should be obtained and a physical examination per-
c. Horizontal incision at the lower portion of the affected area formed to learn if an underlying medical condition may be associated
d. Longitudinal incision along the midline with the hemorrhoids.
e. Longitudinal incision lateral to the midline
SECTION SIX Genitourinary and Gynecologic Systems
Renal Failure 83
KEY CONCEPTS
• T he causes of acute kidney injury (AKI) can be classified as prerenal, post-
renal, and intrinsic renal disorders. Abrupt cessation of glomerular filtration
typically results in a rise of the serum creatinine level of 1 to 2 mg/dL per
day.
• Management of AKI is directed first at potentially lethal complications such
as hyperkalemia or volume overload and then at reversal of the underlying
cause of renal dysfunction. It is important to avoid any further hemody-
namic or toxic insults to the kidneys.
• Patients with acute or chronic kidney disease have a limited ability to
handle fluid and solute loads and have altered metabolism of many drugs.
Therefore, the patient’s impaired renal function must be considered when
fluid is administered or drugs are prescribed.
• The most rapidly lethal complication of acute and chronic kidney disease is
hyperkalemia.
• The most common problems with vascular access devices used for hemo-
dialysis are thrombosis, hemorrhage, and infection. Access infection often
presents as fever without an obvious source and, if suspected, appropriate
IV antibiotics should be administered presumptively while awaiting blood
culture results.
• Peritoneal dialysis–associated peritonitis typically presents with cloudi-
ness of the peritoneal dialysis effluent. The diagnosis is made by a positive
Gram stain or finding of more than 100 WBC/mm3 in the effluent, with at
least 50% polys. It is generally treated on an outpatient basis with intraper-
itoneal antibiotics self-administered by the patient.
• Chest pain in the dialysis patient should be presumed initially to be due to
acute coronary syndrome, although other potentially serious causes may
also be responsible. Serum troponin levels tend to be elevated in patients
with poor renal function, but patients with myocardial infarction show the
typical temporal pattern of rise and fall of troponin levels.
• Hypotension in patients with chronic kidney disease (CKD) may be caused
by infection but may also be the result of rapid fluid removal during dial-
ysis. This often responds readily to fluid administration. Pericardial tam-
ponade is another cause of hypotension that should be considered for
these patients.
• Altered mental status is most commonly due to causes similar to those
seen in patients without renal disease but is sometimes the result of over-
rapid shifts in intravascular fluid and solutes during dialysis, termed dis-
equilibrium syndrome.
1. Of the following choices, which is the correct match between type 4. A 53-year-old man presents with painless gross hematuria. His only
of urine cast and pathologic condition? past medical history is an aortic valve replacement for which he
a. Fatty casts—glomerulonephritis takes warfarin. The physical examination is nonfocal. Laboratory
b. Granular casts—renal parenchymal infection evaluation is remarkable for a normal chemistry panel and blood
c. Hyaline casts—dehydration, exercise count, too numerous to count (TNTC) red blood cells (RBCs) on
d. Red cell casts—nephrotic syndrome urinalysis, and international normalized ratio (INR) of 1.8. What is
e. White cell casts—acute tubular necrosis (ATN) the next indicated step?
Answer: c. Hyaline casts are associated with dehydration, exer- a. Admission and observation
cise, and glomerular proteinuria. The following are the other correct b. Parenteral vitamin K
associations: c. Reassurance
• Fatty casts—nephrotic syndrome d. Renal imaging
• Red cell casts—glomerulonephritis e. Withholding warfarin (Coumadin) for 3 days
• White cell casts—renal parenchymal infection Answer: d. When hematuria is associated with anticoagulant use,
• Granular casts—ATN underlying disease can be identified in a significant proportion of
2. A 34-year-old man with chronic kidney disease has missed his patients.
last two hemodialysis treatments and presents with generalized 5. A 33-year-old woman presents with mild nonfocal abdominal
fatigue and dyspnea on exertion. Vital signs include a blood pain and subjective fever. Her past medical history is significant
pressure of 165/94 mm Hg but are otherwise normal. Room air for hypertension-induced renal failure, for which she is on peri-
Sao2 is 94% at rest. A portable chest x-ray shows vascular con- toneal dialysis (PD). The physical examination is remarkable for
gestion. Which of the following is an indication for immediate a temperature of 38°C (100.4°F), blood pressure 190/110 mm
dialysis? Hg, and mild nonfocal abdominal pain. Slightly cloudy perito-
a. Blood urea nitrogen (BUN) level of 87 mg/dL and serum creati- neal fluid is aspirated from the PD catheter and sent for analysis.
nine level of 10.6 mg/dL Which of the following statements regarding this patient’s condi-
b. Serum calcium level of 6.8 mg/dL and serum albumin level of tion is correct?
3.0 g/dL a. A peritoneal fluid white blood cell (WBC) count >50/mm3 is
c. Serum potassium level of 7.6 mEq/L (not hemolyzed) diagnostic.
d. Serum sodium level of 124 mEq/L b. Intravenous antibiotics should be started empirically.
e. Serum troponin level of 0.08 ng/mL c. Most cases are due to Staphylococcus.
Answer: c. Severe hyperkalemia is an indication for immediate dial- d. No organism is identified in 50% of cases.
ysis in this patient. An ECG should be obtained stat to determine the e. Polymicrobial infections suggest sample contamination.
degree of cardiotoxicity, and immediate medical treatment for hyper- Answer: c. Most PD-associated cases of peritonitis are due to Staphylo-
kalemia should be given while awaiting initiation of dialysis. Abnor- coccus aureus or Staphylococcus epidermidis. No organism is identified
malities of the serum sodium or calcium level can generally be treated in 20% of cases. A polymicrobial infection warrants GI evaluation for
without hemodialysis. A minimal troponin level elevation may be seen possible perforation or intra-abdominal abscess. A PD fluid count
in patients with chronic kidney disease, but does not generally require >100 cells/mm3, with a neutrophil count >50% or positive Gram stain,
specific treatment without other signs of cardiac ischemia; dialysis is is considered confirmatory. Treatment is typically with intraperitoneal
not required. antibiotics given for a 10- to 14-day course.
3. What is the most common cause of nontraumatic hematuria?
a. Carcinoma of the kidney or bladder
b. Cystitis
c. Kidney stones
d. Prostatic hyperplasia
e. Transfusion reaction
Answer: c. In descending frequency, the most common causes are
kidney stones, lower urinary tract infection (UTI), benign prostatic
hypertrophy (BPH), carcinoma of the kidney or bladder, urethritis, and
glomerulonephritis.
Sexually Transmitted Infections 84
KEY CONCEPTS 2. A 17-year-old female presents with complaints of pelvic pain. She
reports multiple sexual partners and inconsistent condom use. Pel-
• T he emergency department (ED) diagnosis of sexually transmitted infections vic examination reveals yellow cervical discharge, cervical motion
(STIs) is often based on clinical findings. Empirical antibiotic treatment is tenderness, and bilateral adnexal tenderness. Pregnancy test is neg-
warranted to cover the most likely infecting organisms based upon history ative. Which of the following statements regarding this scenario is
and physical examination findings. Rapidly available diagnostic tests (Gram true?
stain, darkfield microscopy, wet mount microscopy, and other point-of-care a. All adolescents with pelvic inflammatory disease require hospi-
tests) increase diagnostic sensitivity and specificity. tal admission for intravenous antibiotics.
• Confirmatory diagnostic studies (PCR, culture, serology, and others) should b. A negative nucleic acid amplification test for gonorrhea and
be considered even when results are not immediately available. A mech- chlamydia reliably excludes the diagnosis of pelvic inflammatory
anism for follow-up of test results should be established and appropriate disease.
patient contact information obtained. c. In the absence of an identifiable alternative diagnosis, the clin-
• STIs frequently coexist. Diagnosis of one STI should prompt consideration ical diagnosis and empirical treatment of pelvic inflammatory
and screening for others, including HIV. disease is warranted.
• Genital herpes, the most common ulcerating STI, is often transmitted by d. The clinical diagnosis of pelvic inflammatory disease requires
persons who are unaware that they are infected or are asymptomatic at the the presence of lower abdominal tenderness and cervical motion
time of transmission. tenderness and adnexal tenderness on physical examination.
• In a patient with a genital ulcer, visualization of spirochetes on darkfield e. Women treated as outpatients for pelvic inflammatory disease
microscopy is highly specific for the diagnosis of syphilis and provides rapid should have a follow-up evaluation in 2 weeks.
confirmatory results. Answer: C. The clinical diagnosis of pelvic inflammatory disease is
• Single-dose antibiotic therapy should be used for treatment of STIs when warranted in a sexually active woman at risk for sexually transmitted
possible. Directly observed therapy administered in the ED enhances infections (STIs) if no alternative diagnosis is identified and any one of
treatment compliance. Single-dose antibiotic therapy is recommended for the following findings is present on examination: (1) cervical motion
treatment of uncomplicated gonorrhea, primary and secondary syphilis, tenderness, (2) uterine tenderness, and/or (3) adnexal tenderness.
chancroid, and trichomoniasis in men. Additional diagnostic criteria (mucopurulent cervical discharge, fever,
• HIV, syphilis, gonorrhea, chlamydia, and chancroid are reportable diseases elevated white blood cell count, positive testing for gonorrhea or chla-
in all 50 of the United States (US). mydia, and others) improve specificity but decrease sensitivity in the
diagnosis of pelvic inflammatory disease (PID). Adolescents with PID
may be treated as outpatients using the same criteria as adult women.
1. A 30-year-old pregnant female presents for evaluation of a genital Women receiving outpatient treatment for PID should be advised to
ulcer. Darkfield microscopy reveals spirochetes. She is allergic to seek a follow-up evaluation within 48 to 72 hours.
penicillin. Which of the following statements is false? 3. Which of the following antibiotic regimens is acceptable for outpa-
a. Azithromycin is an acceptable treatment alternative for primary tient treatment of pelvic inflammatory disease in a 70-kg woman?
syphilis during pregnancy in a patient with known penicillin a. Azithromycin 1 g per os (by mouth) (PO) (single dose) and met-
allergy. ronidazole 500 mg bid for 14 days
b. Nontreponemal serologic tests for syphilis (rapid plasma reagin b. Ceftriaxone 125 mg IM (single dose) and doxycycline 100 mg
[RPR], Venereal Disease Research Laboratory [VDRL]) may bid for 7 days
yield false-negative results in primary syphilis. c. Ceftriaxone 250 mg IM (single dose) and azithromycin 1 g PO
c. Primary syphilis facilitates the transmission and acquisition of (single dose)
human immunodeficiency virus (HIV) infection. d. Ceftriaxone 500 mg IM (single dose) and doxycycline 100 mg
d. Syphilis is a reportable disease in all 50 states. PO bid for 14 days and metronidazole 500 mg PO bid for 14 days
e. The chancre of primary syphilis will heal spontaneously without e. Metronidazole 2 g PO (single dose) and doxycycline 100 mg PO
antibiotic therapy. bid for 14 days
Answer: A. Penicillin remains the drug of choice for treatment of Answer: D. Pelvic inflammatory disease is typically a polymicrobial
syphilis during pregnancy. A pregnant patient with syphilis and known infection. Neisseria gonorrhoeae and/or Chlamydia trachomatis are fre-
penicillin allergy should be admitted for desensitization and treat- quently implicated organisms, but anaerobes, enteric organisms, and
ment with penicillin. Nontreponemal serologic tests may yield false- normal vaginal flora may also be present. Empirical treatment of PID
negative results in primary syphilis when antibody titers have not yet should include adequate coverage for gonorrhea and chlamydia. A sin-
risen. False-positive serology may be seen with a variety of other med- gle dose of ceftriaxone 500 mg IM is adequate treatment for upper tract
ical conditions. Visualization of spirochetes on darkfield microscopy gonococcal infection in a patient weighing <150 kg. A 14-day course
confirms the diagnosis of syphilis. All sexually transmitted infections of antibiotics is recommended for adequate coverage of chlamydia and
(STIs) facilitate the transmission and acquisition of HIV. Reportable anaerobes in PID.
STIs in all 50 states include gonorrhea, chlamydia, syphilis, HIV, and
chancroid.
4. A 24-year-old sexually active male presents with painful genital Answer: A. Metronidazole is the drug of choice for treatment of
ulcers. Physical examination reveals a cluster of 2- to 3-mm tender symptomatic trichomoniasis during all stages of pregnancy. Tinidazole
superficial ulcers on the penile shaft. He reports a history of similar should be avoided in pregnant women due to limited data regarding
lesions in the same location sporadically in the past. Which state- safety for use in pregnancy. Visualization of flagellated protozoans on
ment regarding this clinical scenario is false? wet mount microscopy of vaginal discharge is highly specific, but only
a. Both herpes simplex virus (HSV)-1 and HSV-2 can be transmit- 50% to 65% sensitive for the diagnosis of trichomoniasis. Punctate
ted through sexual contact. hemorrhagic lesions on the cervix (so called “strawberry cervix”) is
b. Genital herpes is a lifelong viral infection. seen in up to 10% of cases. Nucleic acid amplification tests for tricho-
c. Prompt initiation of antiviral medication reduces the duration monas are highly sensitive and specific. Trichomoniasis may be asymp-
and severity of symptoms. tomatic in men and women.
d. Topical antiviral therapy is not recommended. 6. Single-dose antibiotic therapy is recommended for treatment of
e. Use of condoms is not necessary to prevent transmission in the which of the following STIs?
absence of clinically apparent lesions. a. Nongonococcal urethritis (NGU)
Answer: E. Genital herpes is a lifelong infection caused by herpes b. Vaginal trichomoniasis
simplex virus. Sexual transmission is more common with HSV-2 but c. Primary and secondary syphilis
d. Pelvic inflammatory disease
may also occur with HSV-1. Condom use is recommended during e. Chlamydia proctitis
asymptomatic periods, because viral shedding and transmission may Answer: C. Primary and secondary syphilis are treated with a sin-
occur even in the absence of clinically apparent lesions. Antiviral ther- gle dose of benzathine penicillin. Gonococcal urethritis, cervicitis,
apy is not curative. Prompt initiation of systemic antiviral medication pharyngitis, and proctitis are treated with a single dose of ceftriaxone.
within 72 hours (acyclovir, famciclovir, or valacyclovir) reduces the Single-dose metronidazole is no longer recommended for vaginal
duration and severity of symptoms, particularly at the time of primary
trichomoniasis, but is still recommended for trichomoniasis in men. A
infection. Topical antiviral therapy is not recommended.
7-day course of doxycycline is now recommended for chlamydia ure-
5. A 24-year-old female presents to the emergency department (ED)
thritis, cervicitis, pharyngitis, and proctitis in men and nonpregnant
complaining of vaginal discharge. A copious frothy whitish dis-
women. Treatment of NGU should include chlamydia coverage. Treat-
charge is noted on speculum examination. Microscopic examina-
ment of upper genitourinary tract STIs, including pelvic inflammatory
tion of a saline wet mount reveals motile flagellated organisms.
disease and epididymoorchits, requires a longer course of antibiotic
Which of the following statements regarding this patient’s treat-
therapy.
ment is true?
a. Metronidazole is the drug of choice for treatment of symptom-
atic trichomoniasis during all stages of pregnancy.
b. Punctate hemorrhagic lesions are seen on the cervix in most
cases of trichomonas vaginitis.
c. Tinidazole is a safe alternative for treatment of trichomoniasis
during pregnancy.
d. Trichomoniasis is always symptomatic in men and women.
e. Wet mount microscopy approaches 100% sensitivity in the diag-
nosis of trichomonas vaginitis.
Urologic Disorders 85
KEY CONCEPTS 2. Which of the following drugs is not an appropriate first-line choice
• U rinary obstruction should be considered in patients with a urinary tract for treatment of an uncomplicated urinary tract infection (UTI) in
infection (UTI) with risk factors for obstruction. Urinary obstruction should women?
be ruled out in patients presenting in septic shock. a. Fosfomycin
• Patients presenting with acute ureteral obstruction require a urinalysis to b. Ciprofloxacin
rule out the presence of a superimposed UTI. c. Nitrofurantoin
• Acute, uncomplicated UTIs should be treated with fosfomycin, nitrofuran- d. Trimethoprim-sulfamethoxazole
toin, or trimethoprim-sulfamethoxazole. Fluoroquinolones are not recom- e. All of the above
mended as first-line therapy for uncomplicated UTI. Answer: b. According to the Infectious Disease Society of America
• Acute bacterial prostatitis (ABP) generally affects men between the ages (IDSA) practice guidelines, fluoroquinolones should not be used as
of 20 and 40 years old with a second peak in men older than 60 years old. first-line agents for uncomplicated UTI because of increased resistance.
A very tender and swollen prostate gland is present in more than 90% of 3. Computed tomography should be undertaken in the patient sus-
patients with acute prostatitis. pected of having renal colic if which of the following is present?
• The three primary predictors of stone passage without the need for sur- a. The patient has a solitary or transplanted kidney.
gical intervention are calculus size, location, and degree of patient pain. b. The patient has gross hematuria.
The most important factor that relates to passage of a calculus though the c. The patient has had a prior history of nephrolithiasis.
genitourinary tract is its size (stone <5 mm has a 90% chance of passing d. The patient presents in severe pain.
spontaneously in 4 weeks). Answer: a. Imaging is appropriate for patients who have a history of
• Imaging is not needed in all patients with renal colic. It should be performed nephrolithiasis who do not improve with treatment, have a urinalysis
when symptoms and signs are atypical and the diagnosis is uncertain, showing infection, have a solitary or transplanted kidney, or for whom
when high-grade obstruction is suspected, or when the patient appears a diagnosis other than renal colic is suspected.
toxic or has a solitary or transplanted kidney. 4. What percentage of cases of acute bacterial prostatitis eventually
• Testicular torsion should be suspected in all patients presenting with acute develop into chronic bacterial prostatitis?
scrotal pain. There is no historical factor or physical finding that reliably a. 1%
differentiates torsion from other causes of testicular pain. b. 5%
• Approximately 10% of patients with testicular trauma have associated tor- c. 10%
sion and require prompt identification and detorsion. d. 20%
• Sexually active males should receive ceftriaxone and doxycycline to treat Answer: c. Chronic bacterial prostatitis will eventually develop in 10%
epididymitis. Patients in whom enteric organisms are likely the cause of of patients with acute bacterial prostatitis.
epididymitis should receive fluoroquinolones. Most cases of pediatric epi- 5. In testicular torsion, testicular salvage may be predicted by which of
didymitis are idiopathic, and antibiotics are not routinely recommended. the following factors?
• Acute urinary retention (AUR) is usually caused by an obstructive lesion but a. Degree of torsion
also can be the presenting manifestation of other pathologic processes. b. Duration of the ischemia
Patients with AUR and concomitant infection, pelvic mass, or neurologic c. Cremasteric reflex
deficits warrant imaging in the emergency department. d. Age of the patient
• Although generally associated with a benign process, the presence of gross e. Both A and B
or microscopic hematuria requires consideration of life-threatening causes Answer: e. Testicular torsion results in obstruction of venous out-
such as aortic abdominal aneurysms and malignancy. flow, subsequent compromised arterial flow, and testicular ischemia.
Testicular salvage hinges on the degree of torsion and duration of the
1. Which of the following medications is not an appropriate first- ischemia.
line choice for empirical treatment of uncomplicated urinary tract 6. To detorse a testicular torsion, you should first rotate the testis
infections (UTIs)? medially to laterally. If this does not produce immediate relief, you
a. Fosfomycin should do which of the following?
b. Nitrofurantoin a. Assume that the testicle is already necrotic and not amenable to
c. Trimethoprim-sulfamethoxazole further reduction attempts.
d. Ciprofloxacin b. Attempt untwisting past 360 degrees because a higher degree of
rotation may be present.
Answer: d. The options for treating uncomplicated lower UTI include c. Both A and B.
single-dose therapy with fosfomycin, 5 days of nitrofurantoin, or 3 d. Reverse the direction of reduction.
days of trimethoprim-sulfamethoxazole. Fluoroquinolones such as Answer: d. Most testes torse laterally to medially, but some may torse
ciprofloxacin or levofloxacin should not be used as first-line agents for medially to laterally. If no immediate relief is obtained by rotating
empirical treatment of uncomplicated UTIs. Instead, they should be medially to laterally, reverse the direction of the reduction attempt.
reserved for patients who have failed or have contraindications to first-
line antibiotics.
86 Gynecologic Disorders
KEY CONCEPTS
• A dnexal torsion is easily missed on initial presentation and should be con-
sidered in any patient with known risk factors, even if symptoms are subtle
or atypical.
• Doppler ultrasound is the preferred initial imaging study for suspected
adnexal torsion.
• An ultrasound examination may distinguish among the various types of
ovarian cysts and identify associated complications, such as torsion, hem-
orrhage, and malignancy. Most ovarian cysts are simple follicular cysts that
resolve without pharmacologic or surgical intervention.
• Abnormal uterine bleeding (AUB) has many structural, hormonal, and coag-
ulopathic causes. Selected imaging and laboratory testing, based on a care-
ful history and physical examination, can often lead to determination of the
cause. Combined oral contraceptive pills can help to regulate the cycle and
alleviate AUB.
• Emergency contraception is a safe, effective option to prevent an undesired
pregnancy. Levonorgestrel and ulipristal are both effective oral medications
and are associated with fewer side effects than the traditional combined
contraceptive method. Intrauterine devices should also be considered for
emergency contraception if a patient desires a long-term contraceptive
option.
87 Stroke
KEY CONCEPTS
• A nterior circulation strokes result in contralateral hemiparesis of the face and • B eyond 4.5 h, the benefit of IV thrombolysis is not clear and should be deter-
body. mined after consulting with the neurologist or stroke specialist.
• Vertebrobasilar strokes result in ipsilateral cranial nerve (CN) deficits and con- • In patients with mild nondisabling stroke symptoms, IV thrombolysis is not
tralateral hemiparesis. recommended. However, for otherwise eligible patients with mild but dis-
• Posterior cerebral artery stroke causes ipsilateral CN III palsy and contralat- abling stroke symptoms, IV thrombolysis is recommended for patients within
eral homonymous hemianopsia. 4.5 h of ischemic stroke symptom onset.
• Wallenberg syndrome (lateral medullary syndrome) causes vertigo, Horner • Patients with a hemorrhagic stroke on coumadin should be reversed using
syndrome, ipsilateral facial numbness, loss of corneal reflex, along with con- vitamin K and prothrombin complex concentrate (PCC). Fresh frozen plasma
tralateral loss of pain and temperature. (FFP) can be used if PCC is not available. However, PCC is preferred over FFP
• Cervical artery dissection is a common cause of stroke in young patients; due to more rapid correction of international normalized ratio (INR), lower risk
trans ischemic attacks (TIAs) preceding stroke in these patients are often mis- of infection, and lower administered volume.
diagnosed. • Prognosis is worse in acute stroke in the setting of fever, hypotension, hypoxia,
• The goal for eligible patients is to receive thrombolytics within 90 min of symp- and hypo/hyperglycemia. Measures should be taken to rapidly correct these.
toms onset; the dose of the tissue plasminogen activator Alteplase (t-PA) is 0.9 • Carotid Doppler, magnetic resonance angiography (MRA), or computed
mg/kg with 10% given as a bolus and the remaining 90% given over 1 h. tomography angiography (CTA) studies are recommended before discharge of
• Overly aggressive blood pressure (BP) management should be avoided in patients a patient with TIA from the emergency department (ED).
with acute ischemic stroke. If thrombolytic therapy is indicated, stringent control • TIA has moved from time-based to tissue-based definition (defined as a tran-
of BP is indicated to reduce the potential for intracranial hemorrhage. sient episode of neurologic dysfunction caused by focal brain, spinal cord, or
• Accurate identification noting the last time a patient was known to be at his retinal ischemia, without evidence of acute infarction).
or her neurologic baseline should be documented in all patients with stroke. • MRI with diffusion-weighted imaging (DWI) has a greater sensitivity than
• Acute ischemic stroke patients receiving Alteplase are at risk of developing a computed tomography (CT) for detecting small infarcts in patients with TIA.
spontaneous intracranial hemorrhage; the risk is lowest in patients with a low • Patients with TIA or nondisabling strokes are at increased risk of recurrent
National Institutes of Health Stroke Scale (NIHSS) score, no hypertension, no stroke and should be evaluated and treated immediately in order to reduce
diabetes, and age younger than 70 years. the risk of subsequent stroke.
• In acute ischemic stroke, the patient and/or their families should be informed • CTA with CT pulmonary angiogram (CTPA) or MRA with DW-MRI is useful for
of the risk and benefit of treatment with thrombolytic therapy. selecting candidates for mechanical thrombectomy between 6 and 24 h after
• The incidence of symptomatic intracerebral hemorrhage after intravenous patient was last known to be well.
(IV) thrombolysis is 2%–7%. The incidence of asymptomatic intracranial • Mechanical thrombectomy can be used in addition to treatment with intra-
hemorrhage is higher with 30%–45% of patients experiencing >10 cerebral venous thrombolytics. However, pretreatment with intravenous thrombolytics
microbleeds. (within 4.5 h of symptom onset) is not required prior to thrombectomy.
1. Which of the following statements concerning ischemic stroke is 5. A 69-year-old male presents with headache, vomiting, apha-
true? sia, a right lower facial palsy, and right upper greater than right
a. Anterior circulation strokes are more likely than posterior lower extremity weakness. The symptoms began approximately
strokes to show evidence of progression at the time of presenta- 4 hours before arrival. Vital signs are temperature 99°C, blood
tion. pressure (BP) 180/90 mm Hg, respiratory rate 18 breaths/min,
b. Anterior circulation strokes rarely present with complete loss of heart rate 92 beats/min, and oxygen saturation 96% on room air.
consciousness. Emergent computed tomography (CT) scan shows a left tempo-
c. Posterior cerebral artery strokes are associated with inconti- ral intracerebral hemorrhage (ICH). Soon after presentation, the
nence, leg weakness greater than arm weakness, and gait clumsi- patient experiences increased vomiting and a diminishing level
ness. of consciousness. What is the most likely explanation for this
d. The presence of aphasia suggests an anterior cerebral artery deterioration?
(ACA) distribution stroke, typically left sided. a. Accompanying subarachnoid hemorrhage (SAH)
e. The presence of diplopia suggests an anterior circulation stroke. b. Acute brainstem herniation
Answer: b. Forty percent of posterior and 20% of anterior circulation c. Hypoxia from neurogenic pulmonary edema
strokes present with progressive symptoms. It is rare for anterior cir- d. Increase in volume of the ICH
culation (carotid, ACA, and middle cerebral artery [MCA]) strokes e. Myocardial infarction with cardiogenic shock
to significantly alter consciousness, unless there has been a previous Answer: d. Approximately one-third of patients with ICH experience
contralateral stroke. ACA strokes primarily affect frontal lobe func- early hemorrhage volume expansion. Although brainstem herniation is
tions and may also present with primitive grasp and such reflexes. In a possibility, this is typically a delayed sequela with a more gradual pre-
addition to contralateral motor and sensory defects, MCA strokes may sentation. Acute myocardial infarction may be associated with intra-
present with expressive aphasia, agnosia, and ipsilateral hemianopsia. cranial emergencies but would not likely cause an abrupt mental status
2. Which of the following is not associated with posterior circulation change. Neurogenic pulmonary edema may accompany any condition
strokes? with elevated intracranial pressure (ICP) and would not likely cause an
a. Diplopia abrupt mental status change.
b. Homonymous cranial nerve (CN) and extremity motor deficits 6. After complete occlusion of cerebral vessels, irreversible neurologic
c. Loss of consciousness deficits are expected to reliably occur within how many hours?
d. Loss of visual object recognition a. 2
e. Nausea, vomiting, and ataxia b. 3
Answer: b. Posterior circulation (vertebrobasilar) strokes involve the ver- c. 4
tebral, basilar, and posterior cerebral arteries. Because this system supplies d. 5
the reticular activating system, cerebellum, brainstem, occipital lobe, and e. 6
brainstem vomiting centers, loss of consciousness with vomiting, visual Answer: e. As a result, ischemic stroke trials, using fibrinolytic or anti-
changes, and cerebral ataxia may be seen. Ipsilateral CN deficits (because platelet agents, have attempted to recanalize occluded arteries and re-
these nuclei largely reside in the brainstem) occur with contralateral perfuse ischemic areas of the brain within a 2-to 6-hour therapeutic
“body” deficits resulting from motor/sensory fiber decussation. window.
3. Headache, vomiting, and a decreased level of consciousness are 7. Which of the following areas of the brain is perfused by the poste-
most commonly seen with which of the following disorders? rior circulation?
a. Ischemic stroke a. Internal capsule
b. Intracranial hemorrhage b. Posterior aspect of the temporal lobe
c. Migraine headache c. Putamen
d. Subarachnoid hemorrhage (SAH) d. Speech areas of the temporal lobe
e. Tic douloureux e. Thalamus
Answer: d. The incidence of headache is highest by far in patients with Answer: e. The thalamus is perfused by the posterior circulation. The
SAH. Vomiting and depressed loss of consciousness are also generally other areas are perfused by the anterior circulation.
more common in this group. Pure migraine headache rarely, if ever, 8. Which of the following statements regarding stroke etiology is
causes a depressed loss of consciousness. Tic headaches do not cause true?
loss of consciousness. a. Lacunar strokes reliably cause a pure motor deficit.
4. What percentage of patients with hemorrhagic stroke experience b. Less than 1% of strokes occur in the 15-to 45-year-old age
clinical deterioration because of growth in hemorrhage volume group.
within the first hours? c. One-third of ischemic strokes are thrombotic.
a. 10% d. Strokes resulting from atrial fibrillation likely involve small ves-
b. 20% sels.
c. 30% e. Two-thirds of ischemic strokes are cardioembolic.
d. 50% Answer: c. One-third of ischemic strokes are thrombotic. Lacunar
e. 75% strokes may cause a pure motor, pure sensory, or ataxic/hemipare-
Answer: c. Thirty percent of patients with intracerebral hemorrhage sis stroke. Vessel occlusion resulting from atrial fibrillation–induced
(ICH) experience early hemorrhage expansion. Progression of neuro- emboli more likely involves the large vessels. Three percent to 4% of
logic deficits and decreasing mental status suggest the diagnosis. ischemic strokes occur in the 15-to 45-year-old age group.
9. A 29-year-old female presents with a left-sided headache after 12. A 66-year-old female presents with a possible transient ischemic
a moderate-speed motor vehicle collision (MVC). She suffered attack (TIA). Approximately 1 hour before her arrival, she had a
no loss of consciousness and has no other complaints or obvious 15-minute episode of strictly right arm and right leg weakness,
injuries. Physical examination is remarkable only for drooping of and her symptoms have now resolved. Her blood pressure (BP)
the left eyelid and slight miosis of the left pupil compared with the is 165/92 mm Hg. Her prior medical history is significant for
right. Which of the following would be the diagnostic test of choice? hypertension, high cholesterol, and diabetes mellitus. What is her
a. Brain magnetic resonance imaging (MRI) with gadolinium ABCD2 score?
b. Contrasted computed tomography (CT) scan of the brain a. 4
c. CT angiogram of the carotid arteries b. 5
d. Uncontrasted CT scan of the brain c. 6
e. Urine drug screen d. 7
Answer: c. Carotid or vertebral artery dissection can occur after trauma or e. 8
mild events, such as yoga, twisting, or prolonged static positions looking Answer: c. The patient’s ABCD2 score is 6 (age >60 years, BP >140/90
upward. The hallmark is unilateral neck pain, face pain, or headache, often mm Hg, unilateral weakness, symptoms lasting 10 to 59 minutes, and
with accompanying Horner syndrome. Acutely, cerebral ischemic changes history of diabetes). No speech impairment is reported.
would not be seen on brain imaging. Carotid and vertebral dissection is 13. A 72-year-old male presents with an apparent stroke. Computed
not a contraindication for thrombolytic therapy in the eligible patient. tomography (CT) imaging of the brain demonstrates only a hyper-
10. A 28-year-old G3P3 woman who is 2-week postpartum after an dense middle cerebral artery (MCA) sign and no other ischemic
uncomplicated vaginal delivery presents with acute onset of mild changes. He was last seen neurologically normal 4 hours earlier.
headache, lethargy, and double vision. Physical examination is Which of the following is a contraindication to fibrinolytic ther-
remarkable for normal vital signs and a left eye lateral gaze palsy. The apy in this patient?
most appropriate intervention is likely to be which of the following? a. His blood glucose is 372 mg/dL.
a. Computed tomography (CT) scan of the brain with possible b. His National Institutes of Health (NIH) Stroke Scale score
lumbar puncture is 24.
b. CT scan of the brain and intravenous (IV) heparin c. His platelet count is 110,000/mm3.
c. Erythrocyte sedimentation rate (ESR) and IV corticosteroids d. His systolic blood pressure (BP) is 180 mm Hg.
d. IV magnesium e. He takes warfarin daily.
e. Lumbar puncture and IV antibiotics Answer: e. Any oral anticoagulant treatment (regardless of the patient’s
Answer: b. Cerebral venous thrombosis may present with headache, international normalized ratio [INR]) is a contraindication to fibrino-
lethargy, cranial nerve (CN) deficits, seizures, or even psychiatric com- lysis in the 3-to 4.5-hour treatment window. Other contraindications
plaints. CT scan and/or magnetic resonance imaging (MRI)/magnetic include an NIH stroke scale score greater than 25, platelet count less
resonance angiography (MRA) are likely to reveal the diagnosis. Treat- than 100,000/mm3, blood glucose greater than 400 mg/dL, and systolic
ment includes heparin. Neurosurgical consultation is not useful. Sub- BP greater than 185 mm Hg.
arachnoid hemorrhage (SAH) would not be expected to cause a focal 14. A 75-year-old man is brought to the emergency department
neurologic deficit. Eclampsia and meningitis would be expected to give (ED) for altered mental status. After computed tomography
characteristic findings on history and examination. (CT) imaging of the brain is performed, he is found to have a
11. An 82-year-old male presents with an apparent stroke. He is rapidly large intracerebral hemorrhage (ICH). Which of the following
evaluated and determined to be within the 3-to 4.5-hour window for options is not an appropriate strategy for lowering intracranial
fibrinolytic therapy. Which of the following exclusion criteria applies pressure (ICP)?
only to the 3-to 4.5-hour criteria and not the 0-to 3-hour criteria? a. Barbiturate-induced coma
a. Administration of heparin within the 48 hours preceding the b. Hyperthermia induction
stroke onset c. Hypertonic saline administration
b. Age older than 80 years d. Hyperventilation
c. High clinical suspicion for subarachnoid hemorrhage (SAH) e. Mannitol administration
d. Seizure at the onset of the stroke Answer: b. Hypothermia is an experimental modality for lowering ICP.
e. Symptoms rapidly improving Hyperventilation can serve as a temporizing measure for reducing ICP.
Answer: b. In the 3-to 4.5-hour window, patients cannot exceed 80 years Mannitol and/or hypertonic saline can also be administered. Inducing
of age. Heparin administration within the 48 hours preceding stroke a barbiturate coma is also an experimental modality.
onset is a contraindication in both the 0-to 3-hour window as well as the
3-to 4.5-hour window. Similarly, high clinical suspicion for SAH, seizure
at the onset of the stroke symptoms, and rapidly improving symptoms
are contraindications to fibrinolysis in both time windows.
Seizure 88
KEY CONCEPTS
• E pilepsy is a neurologic condition associated with an intrinsically lower sei- • E merging evidence supports the consideration of secondary seizure prophy-
zure threshold and a higher risk of recurrent seizures without a clear trigger. laxis for first-time unprovoked seizures in selected patients. However, data
• The characterization of seizure semiology, duration, and etiology is important are heterogeneous, and a thorough discussion with the patient and a special-
for accurate classification of seizures and status epilepticus; these impact ist is advised before the initiation of anti-seizure drugs.
definitive treatment choices. • Alcohol withdrawal syndrome can include seizures resulting from the ces-
• There is no single test to confirm that a patient seized, and several seizure
sation or reduction of alcohol consumption leading to an unopposed excit-
mimics, including convulsive syncope, exist. A postictal alteration in mental atory sympathomimetic response. Benzodiazepines are the drug of choice and
status makes a seizure five times more likely than syncope. should be supplemented by supportive measures, including electrolyte and
• Key factors in the evaluation of epilepsy patients who present with break- thiamine supplementation. In pregnant patients, evaluations for new-onset
through seizures include: changes in anti-seizure regimen, compliance, the seizures before 20 weeks should be the same as in nonpregnant patients.
addition of new medications that may lower the seizure threshold or levels After 20 weeks, and up to 8 weeks postpartum, eclampsia is a major cause
of antiseizure drugs, presence of common infections or metabolic derange- of seizures and should be included in the differential. IV magnesium remains
ments, and recent sleep habits. first-line treatment for patients with eclamptic seizures and should not be
• Although most seizures are self-limited, the management of patients with delayed. Benzodiazepines and non-teratogenic antiseizure medications are
seizures involves a targeted search for underlying pathology, treatment of reasonable alternatives in magnesium-refractory cases.
complications associated with convulsions, and the prevention of future epi- • Post- anoxic status epilepticus, including myoclonic status, frequently
sodes. observed following cardiac arrest, was considered pathognomonic of poor
• Serious systemic complications of seizures and status epilepticus include car- neurologic outcome. However, in patients lacking factors with high predictive
diac arrest, arrhythmias, apnea, hypoxia, acute kidney injury, rhabdomyolysis, value for poor outcome post cardiac arrest, early antiseizure therapy can lead
acidosis, and death. The prognosis of status epilepticus is directly related to to improved outcomes.
the etiology of seizures. • Therapeutic approaches to nonconvulsive status epilepticus are commonly
• Primary seizure prophylaxis should only be given for 7 days following trau- extrapolated from convulsive generalized status epilepticus guidelines. How-
matic brain injury. The period is shorter, although less well-defined, in unse- ever, the presumed etiology of seizures, extent of cortical area involved (focal
cured aneurysmal subarachnoid hemorrhage. Prolonged primary prophylaxis versus diffuse or generalized), comorbidities, and response to therapy should
is not recommended because it has not been demonstrated to reduce long- be considered when selecting an antiseizure therapeutic algorithm.
term seizure risk. The duration of secondary seizure prophylaxis and the anti- • Immunomodulation remains the cornerstone of therapy for autoimmune
seizure regimen in patients with acute brain injury who had a seizure during epilepsies, in conjunction with antiseizure drugs. Transdisciplinary deci-
hospitalization should be individualized. sion making is warranted before initiating immune-targeted therapy, which
• Patients with a first-time seizure who have no known structural brain pathol- may include high-dose methylprednisolone, intravenous immunoglobulin,
ogy, normal serum glucose and sodium levels, and normal neurologic exam- plasma exchange, rituximab, cyclophosphamide, and more recently, tocili-
ination can be discharged from the ED with appropriate outpatient follow-up. zumab.
1. Which of the following statements regarding the pathophysiology 4. Which of the following statements regarding patients presenting
of seizures is true? with first-time seizures is true?
a. During a seizure, prompt loss of consciousness implies a cortical a. All patients should have basic serum electrolyte testing.
focus. b. All patients should receive a loading dose of an antiepileptic
b. Generalized seizures involve abnormal electrical activity in one drug and a prescription for an oral regimen prior to discharge.
hemisphere. c. Nondiabetic patients do not need to have serum glucose checked
c. Regarding seizure activity, acetylcholine is inhibitory. if their mental status returns to normal.
d. The clinical seizure activity typically reflects the brain focus of d. Persistent altered mental status is an indication for a lumbar
initiation. puncture.
Answer: D. Clinical seizure activity typically reflects the site of seizure Answer: D. Persistent altered mental status should trigger consideration
origin. Generalized seizures involve both brain hemispheres. Acetyl- of a lumbar puncture, serum glucose, and an electroencephalogram (EEG)
choline is an excitatory neurotransmitter, and GABA is inhibitory. among other testing. Non-contrast CT of the head is relatively high-yield
Prompt loss of consciousness with a seizure implies a subcortical focus in first-time seizure patients, and most guidelines recommend performing
involving the reticular activating system. one. Patients with no comorbidities who return to baseline mental status
2. Which of the following statements regarding epilepsy is true? do not need broad-based electrolyte testing, but hypoglycemia is a com-
a. Electrolyte disturbances are the most common cause of recur- mon cause of seizures even in nondiabetic patients and should be routinely
rent seizures in epileptic patients. tested for in first-time seizure patients. Initiation of antiepileptic drugs in
b. Epilepsy is a condition of recurrent provoked seizures. the emergency department (ED) for first-time seizure patients without a
c. Epileptic seizures are always generalized tonic-clonic. known structural brain lesion may do more harm than good.
d. Epileptic seizures can be triggered by smells or lack of sleep. 5. A 23-year-old woman is brought to the emergency department
Answer: D. Epilepsy is a condition of recurrent, unprovoked seizures. (ED) for a prolonged seizure. By emergency medical service (EMS)
Idiopathic epilepsy almost always starts in childhood. Medication non- report, the patient has no past medical history and no history of sei-
compliance is the most common cause of recurrent seizures in epileptic zures. Paramedics report tonic-clonic activity for approximately 15
patients. Epileptic seizures can be partial seizures. Some patients with minutes, refractory to diazepam 5 mg intravenously in the ambu-
epilepsy have seizures triggered by certain specific sensory stimuli, lance. Upon arrival to the ED, the patient’s seizure activity abruptly
with flashing lights being the most well-known of these. ceases, and she lucidly responds to the history and physical exam-
3. Which of the following factors does not put patients at risk for sei- ination. She is symptom free. What would be the most appropriate
zures? intervention?
a. Acute stroke a. A trial of oral phenytoin after an intravenous (IV) loading dose
b. Chronic stroke-related lesions in the ED
c. Hyperkalemia b. Confrontation
d. Hypomagnesemia c. Neurology referral for electroencephalogram (EEG) and consul-
Answer: C. Indwelling intracranial shunts and other lesions, and hypo- tation
magnesemia (as seen in malnourished alcohol-dependent patients) are d. Psychiatric consultation
all risk factors for seizures. Stroke is one of the most common causes Answer: D. No clinical criteria are 100% specific for the diagnosis of
of seizures in the elderly. Seizures occurring acutely with a stroke are nonepileptic activity. Seizures and nonepileptic activity may coexist.
often partial and reflect the affected brain area, whereas those that For many patients, these episodes may not be deliberate. The most pru-
occur chronically are often generalized. Hyperkalemia does not cause dent course of action would be to treat them as possible ictus and refer
seizures unless it precipitates hypotension and consequent hypoperfu- to a neurologist. Initiation of antiepileptic drugs in the ED is not indi-
sion of the brain. cated in first-time seizures of healthy patients who return to normal.
Headache Disorders 89
KEY CONCEPTS
• T he goals of headache evaluation in the ED are (1) to distinguish between • O pioids are not first-line therapy for the primary headaches.
benign primary headache disorders and potentially life-threatening secondary • Patients with migraine treated in the ED require a discharge “rescue plan” if
causes of headache and (2) to treat the headache pain effectively and rapidly the headache recurs.
without causing undue side effects. • High-flow oxygen will terminate the majority of cluster headaches.
• Patients with the following headache presentations are at risk for serious • The differential diagnosis of sudden severe headache includes subarachnoid
underlying disease: sudden explosive headache; new-onset headache after hemorrhage, cerebral venous thrombosis, cervical artery dissection, and idio-
the age of 50 years; headache associated with papilledema, alteration in or pathic intracranial hypertension.
loss of consciousness, or focal neurologic symptoms; subacute headache with • Cerebral venous thrombosis should be suspected in women who have a new
increasing frequency or severity; headache associated with fever, cancer, or type of headache and are pregnant or on birth control pills.
immunosuppression; and headache triggered by exertion, sexual activity, or • Carotid artery dissection may cause headache, ptosis, and miosis.
Valsalva maneuver. • Patients with post-traumatic headache should only be imaged if they have
• The need for diagnostic studies is dictated by the suspected secondary cause high-risk features on the Canadian CT Head rule.
of headache.
• Antidopaminergic agents, such as metoclopramide or prochlorperazine, are
first-line therapy for migraine.
1. A 29-year-old female presents within 1 hour of the sudden onset Answer: D. The symptoms may also include paralysis of extraocular
of a severe, diffuse headache accompanied by meningismus and movements. Ocular symptoms are most common with thrombosis of
vomiting. Emergent computed tomography (CT) scan is negative. the cavernous sinus, rather than one of the other sinuses.
Lumbar puncture (LP) reveals 50,000 red blood cells (RBCs) in tube 4. A 28-year-old man with a history of migraine and no other med-
1 and 30,000 RBCs in tube 4. Opening pressures are normal, and ical history presents with a pounding, bilateral 9/10 headache
the sample is negative for xanthochromia. What would be the most associated with nausea and vomiting that has lasted for 48 hours.
appropriate next step? The patient reports experiencing a similar headache about once
a. Admission and observation every few months. Physical exam is noteworthy for a blood pres-
b. Cerebrovascular imaging study sure of 180/110 mm HG in his right arm and 178/111 mm HG in
c. Hydration, analgesics his left arm. Physical exam, including a detailed neurologic exam,
d. Magnetic resonance imaging (MRI) scan with gadolinium is otherwise completely normal. The most appropriate first step in
Answer: B. CT scan provides a sensitivity of approximately 90% for the management of this patient is the following:
the detection of subarachnoid hemorrhage (SAH). A traumatic LP, a. Non-contrast CT scan of the head
even with diminishing RBC counts with sequential tubes, cannot b. Metoclopramide 10 mg IV
differentiate between a SAH and traumatic tap. The lack of xan- c. Labetalol 20 mg IV
thochromia is predictable, given the acute onset of headache and d. CT angiography of the head and neck
the 12 hours required for cerebrospinal fluid (CSF) xanthochromia Answer: B. For patients who present with elevated blood pressure and
to develop. A cerebrovascular imaging study, such as computed an otherwise normal physical exam in the setting of a typical migraine
tomography angiography (CTA), magnetic resonance angiography headache, treatment should first focus on relieving the acute pain.
(MRA), or standard angiography would be required next to exclude Treatment of blood pressure should commence if the blood pressure
a source of bleeding, such as an aneurysm or arteriovenous malfor- remains persistently high after adequate control of pain. Head imag-
mation (AVM). ing is not indicated in a patient who reports a typical exacerbation of
2. A 69-year-old male presents with several months of intermittent migraine. CT angiography of the head and neck is most appropriate to
left-sided headaches that have been worse at night and occasion- rule out cervical artery dissection. Sumatriptan is an appropriate treat-
ally on exposure to cold air. On several occasions, he has noted ment of acute migraine but is relatively contraindicated in a patient
increased pain while eating. He has had no other symptoms other with markedly elevated blood pressure.
than modest fatigue. Physical examination is unremarkable with 5. An obese 37- year-
old female reports worsening headache × 3
normal vital signs and ophthalmologic and neurologic survey. Lab- months, which first responded to ibuprofen but now no longer
oratory evaluation shows only a mild anemia, with a hemoglobin of does. The headache is bilateral, pulsating, associated with nausea,
11 mg/dL and normocytic indices. What would be the most appro- and improves when she assumes an upright position. She reports
priate next step? 5-second blackouts of her vision, which occur occasionally when
a. Computed tomography (CT) scan of the brain she changes position. A neurologic exam is unremarkable. Retinal
b. Electrocardiogram exam is deferred because of marked photophobia. Non-contrast
c. Erythrocyte sedimentation rate (ESR) head CT scan is normal. The most appropriate next step in the diag-
d. Neurology consultation nostic workup is:
Answer: C. Temporal arteritis may present with intermittent or con- a. MRV
tinuous symptoms, sometimes associated with fatigue, myalgias, jaw b. D-dimer
claudication, and mild anemia. The ESR is usually, but not always, diag- c. CT angiography
nostic and would confirm the diagnosis if elevated. Steroids and oph- d. Lumbar puncture
thalmology consultation would then follow. The presence of temporal Answer: D. The provided history is most consistent with idiopathic
artery tenderness may be variable because the vasculitis may affect any intracranial hypertension, which can only be confirmed by lumbar
artery. The diagnosis must be suspected in any elderly patient with puncture. Lumbar puncture will demonstrate elevated cerebrospi-
recurrent or continuous headaches. nal fluid pressure >250 mm H2O. MRV and D-dimer can be used to
3. With cavernous sinus thrombosis, the clinical picture is usually exclude the diagnosis of venous sinus thrombosis, which should be
dominated by which of the following? considered if the CSF pressure is indeed elevated. ESR is most useful to
a. Facial pain exclude giant cell arteritis, a diagnosis that does not occur in patients
b. Lethargy younger than 50 years. CT angiography can be used to diagnose cere-
c. Nausea and vomiting brovascular pathology.
d. Ocular findings such as pain and proptosis
Delirium and Dementia 90
KEY CONCEPTS
• D elirium is an acute condition characterized by an altered level of attention • T he clinician should be wary of attributing behavioral disturbances to psy-
and awareness. It develops during a short time, and symptoms tend to fluc- chiatric illness in the presence of abnormal vital signs or abnormal senso-
tuate throughout the day. rium.
• Delirium is commonly caused by medications, drug intoxication or with- • Nonpharmacologic methods of controlling agitation including reassurance,
drawal, infections, metabolic disorders, CNS and cardiovascular events, verbal de-escalation, and avoidance of environmental triggers should be
and autonomic nervous system disturbances. considered in the treatment of patients with dementia.
• Dementia is a chronic condition characterized by cognitive impairment. It • Antipsychotics and benzodiazepines are used cautiously in the manage-
is slow in onset and progressive in nature. This disorder has many causes, ment of acute agitation in delirium and dementia. The choice of agent is
some of which are reversible with treatment. It is essential to search for determined by side effect profile and etiology of delirium or acute agitation.
reversible underlying etiologies that may be worsening a cognitive impair- • Antipsychotics may cause QTc prolongation and extrapyramidal side
ment. effects, especially when given intravenously.
• Patients with either dementia or psychiatric disorders may present with • Lower doses of medications may be appropriate in older adults to decrease
superimposed delirium, often making identification of the underlying cause risk of adverse events while effectively treating acute agitation.
of their abnormal behavior difficult.
1. Which of the following characteristics helps distinguish dementia is the central component most key to the diagnosis of delirium in
from delirium? this case?
a. Acute onset a. Agitation
b. Memory impairment b. Disorientation
c. Disorientation c. Inattention
d. Delusions d. Memory dysfunction
Answer: A. Delirium is characterized by an acute onset of impaired Answer: C. Disturbance in attention is central to the diagnosis of delir-
cognition, in comparison to the slower onset of delirium. Memory ium. Disorientation often accompanies this but is not invariably pres-
impairment and disorientation can occur in either process. Delusions ent. The patient is usually disoriented to time and, less often, place.
are more common in delirium, but can occur in the later stages of The delirious patient may also experience visual, auditory, tactile, and
dementia as well. olfactory hallucinations; may lose the ability to modulate emotional
2. A 78-year-old woman presents with 3 days of decreasing ability to expression; and often exhibits fluctuating symptoms. Short-term mem-
concentrate, memory and cognition breakdown, sleep cycle disrup- ory is usually impaired, but this is also seen in dementia.
tion, and fluctuating levels of agitation. Her current medications 4. Which of the following associations is correct?
include levofloxacin (Levaquin) 500 mg/day for a bladder infection, a. Droperidol: QT prolongation
tramadol PRN for knee arthritis, and hydrochlorothiazide 25 mg/ b. Haloperidol: Dysphoria
day for essential hypertension. Her examination is normal except c. Lorazepam: Excessive half-life
for a baseline tachycardia, moderate agitation and restlessness, and d. Meperidine: Cholinergic effects
orientation to person only. Laboratory analysis shows glucose 198 Answer: A. This may also be seen with haloperidol and the phe-
mg/dL, sodium 131 mEq/L, potassium 3.8 mEq/L, creatinine 1.4 nothiazines. Meperidine causes dysphoria and possibly some anticho-
mg/dL, white blood cell (WBC) count 11,300 cells/mm3, hemoglo- linergic effects. Diazepam results in the longest terminal T½ of the
bin 12 g/dL, bicarbonate 25 mEq/L, and a normal urinalysis. What benzodiazepines.
is the most likely etiology for her delirium? 5. A 63-year-old man presents with acute-onset delirium. He is a known
a. Early sepsis alcoholic, and the family reports a cessation of alcohol intake 36 hours
b. Hyperglycemia before presentation. He has no other known medical problems. Exam-
c. Hyponatremia ination is remarkable for an acutely delirious patient who has active
d. Medication effect visual and auditory hallucinations and a tremor. Neurologic examina-
Answer: D. Medications are the most common cause of delirium in tion is otherwise negative. Finger-stick glucose is normal. Thiamine
the older adult population. Common inciting medications include 100 mg intravenously fails to improve his symptoms. Which of the
antibiotics (quinolones and macrolides), analgesics, sympathomimet- following is the intervention most likely to prevent further worsening?
ics, antiinflammatories, sedatives, and cardiovascular agents. This level a. Dextrose
of hyponatremia would not be expected to cause delirium. Likewise, b. Haloperidol
a modest hyperglycemia without associated acidosis would be an c. Lorazepam
unlikely culprit. d. Magnesium
3. An 82-year-old man presents with acute delirium. On examination, Answer: C. This patient presents with symptoms of alcohol withdrawal.
he is alert and mildly agitated. He is oriented to person and place Wernicke encephalopathy should be considered and thiamine supple-
but not time. He is easily distracted and exhibits a mild bilateral mentation may be necessary, but lorazepam will prevent progression to
upper extremity resting tremor without asterixis. His neurologic withdrawal seizures. Magnesium acts as a cofactor for thiamine, and
examination is nonfocal. His short-term memory is impaired. What should also be repleted if levels are found to be low.
Brain and Cranial Nerve Disorders 91
KEY CONCEPTS
Multiple Sclerosis in two divided doses, titrated up to 1800 mg/day); adverse side effects may
• The clinical picture of multiple sclerosis (MS); is one of marked heterogene- lead to treatment failure. While secondary therapies are available, treatment
ity. The classic clinical syndrome consists of recurring episodes of neurologic failure with sodium-channel blockers is an indication for surgical referral.
symptoms that rapidly evolve over days and slowly resolve over weeks.
• Magnetic resonance imaging (MRI) is a high-yield diagnostic test for MS. Facial Nerve Paralysis
When the emergency department (ED) MRI is an option, it should be consid- • The diagnostic dilemma related to facial nerve paralysis typically revolves
ered as it may expedite follow up and initiation of treatment. around distinguishing Bell’s palsy from other causes. While diagnostic imag-
• MS relapse is treated with high-dose methylprednisolone, typically 1000 mg ing is not needed to make a diagnosis of Bell’s palsy, it may be indicated in
IV over a course of 3 to 5 days. There is emerging evidence supporting oral cases where the etiology of facial weakness is unclear.
therapy as efficacious in the treatment of relapse. • Patients at high risk for stroke and infection or with features atypical for Bell’s
palsy (particularly intact forehead movement or bilateral facial paralysis)
Cerebral Venous Thrombosis should be considered for an alternate diagnosis and often require additional
• Cerebral venous thrombosis (CVT) should be suspected in patients (particu- diagnostic testing.
larly female patients under age 50) presenting with stroke symptoms without • Corticosteroids improve the outcome in Bell’s palsy and should be started as
risk factors, unexplained new seizures or refractory headaches, or signs of soon as possible to maximize their benefit. We recommend 5 days of predni-
intracranial hypertension. sone, 60 mg, followed by a taper over 5 days.
• The combination of MRI and magnetic resonance venography (MRV) is con- • Adding antiviral therapy to corticosteroids confers little or no benefit and
sidered the gold-standard for diagnosis of CVT, though contrast-enhanced evidence supporting its use is very weak except in cases of Ramsay Hunt
computed tomography (CT) with venous phase imaging is an alternative. Syndrome.
• Primary treatment of CVT in the ED is anticoagulation with a weak recom-
mendation for low molecular-weight heparin over unfractionated heparin. Vestibular Schwannoma
Neuro-interventional treatment may have a role in severe cases, and hospital • Vestibular schwannomas are a common cause of sensorineural hearing loss
transfer may be needed for advanced neurological care. and may lead to other distressing neurological symptoms. The primary role
of the emergency clinicians is referral for testing and treatment when the
Trigeminal Neuralgia diagnosis is suspected or confirmed.
• Trigeminal neuralgia (TN) is characterized by intermittent, unilateral, severe
and sharp facial pain precipitated by innocuous sensory stimuli and not Diabetic Cranial Mononeuropathy
explained by other local causes. • The differential diagnosis for diabetic cranial mononeuropathy includes aneu-
• Secondary TN can be associated with MS and other cranial nerve disorders rysm and stroke; a careful history and physical examination can identify cases
and neurologic conditions. A careful history and physical examination can that require neuroimaging.
help identify these causes though imaging may be needed. • Diabetic patients presenting with a CN III palsy with spared pupillary response
• Sodium-channel blockers are first line therapy, either carbamazepine (200 to and no other deficits in general do not require neuroimaging.
400 mg/day, titrated up to 1200 mg/day) or oxcarbazepine (300 to 600 mg/day
1. A 30-year old woman with no past medical history presents com- leg. These findings are noted on prior ED notes. She has a tempera-
plaining of weakness and spasms in both legs. She describes prior ture of 102.6°F and otherwise normal vital signs: What is the most
episodes of numbness in her feet that lasted for a few weeks at a time appropriate next step in this patient’s management?
before resolving. She also notes right eye pain, and unilateral papill- a. Obtain CBC, chest x-ray, and urinalysis
edema is noted on fundoscopic exam. What is the most appropriate b. CT of brain
next step in this patient’s management? c. Rapid initiation of IV antibiotics followed by lumbar puncture
a. Lumbar puncture d. Discharge home with NSAIDs for fever and pain
b. Ophthalmology consultation e. Discharge home with oral corticosteroids
c. Discharge home and referral to neurologist Answer: a. This patient is presenting with an exacerbation of MS.
d. MRI of the brain and spinal cord A relapsing and remitting pattern is characteristic. Infection is a
e. IV corticosteroids common precipitating cause of MS relapse, and investigating the
Answer: d. This patient is presenting with symptoms suggesting a source of infection can help determine the need for and selection of
new diagnosis of Multiple Sclerosis. Early diagnosis and initiation of antibiotic therapy. While there may be a need for lumbar puncture
therapy has been shown to improve outcomes. MRI is a sensitive test and imaging of the brain, these are not necessarily indicated in this
in identifying lesions consistent with MS. While MRI may not always scenario. In addition to a possible need for antibiotic therapy, IV
be an option in the ED, it should be strongly considered and pursued or oral corticosteroids can be used to manage the symptoms of MS
when available. Lumbar puncture is also a useful test in the evaluation relapse.
of possible MS, and while there are characteristic CSF findings, LP is 3. A 38-year-old woman presents 8 weeks postpartum with a 1-week
of less initial utility than MRI. Non-contrast head CT may be useful in history of severe headache and progressively altered mental status,
investigating other causes, but it will not identify MS lesions. Discharge which culminated in a seizure several minutes before presentation.
home with neurology home is appropriate, but MRI in the ED would On examination, she is normotensive, appears postictal, but has no
likely expedite diagnosis and initiation of therapy. focal neurologic findings. Ophthalmoscopic examination reveals
2. A 65-year-old woman with a history of MS presents with worsening papilledema, and non–contrast-enhanced head computed tomog-
pain, paresthesias, and weakness in her right leg over the last 3 days, raphy (CT) reveals a dense sagittal sinus and a small venous hemor-
which she describes as common symptoms during relapse. She has rhage in the occipital region. What is the most appropriate next step
4/5 motor strength and increased deep tendon reflexes in her right in this patient’s management?
a. 325 mg aspirin
b. Dexamethasone 10 mg IV push
c. Hypertonic saline 500-mL bolus
d. Mannitol 1 g/kg
e. Systemic anticoagulation with unfractionated heparin or low-
molecular-weight heparin (LMWH)
Answer: e. The patient presents with a venous sinus thrombosis. Answer: a. This patient has met diagnostic criteria for trigeminal
Although large, randomized trial data do not exist, case series and neuralgia (TN). TN can be a debilitating condition and is marked by
expert consensus strongly suggest improved outcomes with systemic episodic severe episodic facial pain brought on by innocuous stimuli.
anticoagulation, even in the setting of venous hemorrhage on head CT. TN is a clinical diagnosis, and while some suggest that MRI be part
Osmotic agents and steroids have no proven benefit in the manage- of the diagnostic process to evaluate secondary causes of TN, urgent
ment of sinus thrombosis and may cause harm. Antiplatelet agents may neuroimaging is not indicated in the absence of neurologic deficit or
be considered if absolute contraindications to anticoagulation exist but other concerns. First line therapy are sodium-channel blockers such as
probably have lower therapeutic efficacy. carbamazepine. Patients who continue to have symptoms with phar-
4. A 26-year-old man presents with a left facial droop. The symptoms macologic therapy should be referred to a surgeon for consideration of
began painlessly 3 days ago without a prodrome. Examination microvascular decompression.
reveals a left facial droop with an inability to wrinkle the forehead 7. A 58-year-old woman presents with progressive right-sided hearing
without other associated physical examination abnormalities. What loss along with a sensation of fullness in the ear and otalgia. Her otic
is the most appropriate next step in this patient’s management? canal and tympanic membrane appear normal and she has no mas-
a. Discharge home with reassurance that symptoms usually resolve toid tenderness. Her cranial nerve exam demonstrates right senso-
in a few weeks rineural hearing less, and the remainder of her neurologic exam is
b. Initiation of oral antiviral therapy normal. Which of the following is the most appropriate next step in
c. Initiation of oral corticosteroids this patient’s management?
d. Serologic testing for Lyme disease a. Admission for IV steroid therapy
e. Urgent non–contrast- enhanced computed tomography (CT) b. Discharge home with instructions for outpatient workup
scan of the head c. Head CT
Answer: c. The patient has Bell’s palsy, which is a painless left facial d. Prompt otolaryngology consultation
nerve palsy. The primary symptom is a left peripheral nerve paralysis e. Discharge home with otic hydrocortisone/neomycin/polymyxin
often with unilateral dysgeusia, hyperacusis, and external canal and for pain control
pharyngeal numbness. Facial sensation is intact. Corticosteroids have Answer: b. This patient likely has a vestibular schwannoma (VS), a
been shown to improve outcomes, and they should be initiated as soon slow-growing tumor of the vestibular nerve that is the most common
as possible. Antiviral therapy in combination with corticosteroids may cause of asymmetric sensorineural hearing loss. Patients may also com-
be considered (conflicting evidence exists) but they are less efficacious plain of headache, disequilibrium, and tinnitus. Workup of potential
than corticosteroids when given alone. Given the classic picture and VS, which includes MRI and audiometry, can generally be performed
lack of other associated neurologic findings, CT scan of the head is not as an outpatient, with subsequent referral to an otolaryngologist or
required and is not likely to provide benefit. Testing for Lyme disease neurosurgeon. CT lacks the necessary sensitivity in the posterior cra-
should be considered in Lyme endemic areas or in patients with a his- nial fossa to reliably rule out the presence of vestibular schwannoma.
tory of a tick bite. 8. A 54-year-old man with a history of poorly controlled diabetes
5. Which of the following clinical features would make a diagnosis of presents with diplopia. Physical exam reveals the inability to move
Bell’s palsy less likely? the right eye superiorly and medially, accompanied by ptosis. The
a. Increased sensitivity to sound right pupil in larger than the left, and poorly responsive to light.
b. Decreased tearing on the affected side Which of the following is the most appropriate next step in this
c. A prodromal viral-like illness patient’s management?
d. Rash a. Oral corticosteroid therapy
e. Altered sensation of taste b. Discharge with patient education on importance of blood sugar
Answer: d. In addition to the unilateral facial weakness involving control to minimize risk of stroke.
the forehead, Bell’s palsy may present with increased sound sensitiv- c. Prompt referral to an ophthalmologist
ity (hyperacusis), altered sense of taste (dysgeusia). Decreased tearing d. MRI brain
may also occur, and viral illness is known to be a precipitating cause. e. Initiation of systemic anticoagulation therapy
Rash is not characteristic of Bell’s palsy, and the presence of a herpeti- Answer: d. Cranial neuropathies are uncommon and usually a com-
form vesicular eruption should raise suspicion for Ramsey-Hunt syn- plication of diabetes. They most often affect extraocular muscles, caus-
drome. Other features that raise suspicion for an alternative diagnosis ing ophthalmoplegia. The oculomotor nerve (III) is most commonly
include vestibular symptoms, bilateral or forehead-sparing distribu- involved, leading to the inability to move the affected eye superiorly
tion, involvement of other cranial nerves, and signs of systemic illness. and medially. The etiology of diabetic mononeuropathy is thought to
6. A 62-year-old woman presents with one day of episodic sharp uni- be microvascular ischemia. Fibers controlling pupillary size and light
lateral facial pain brought on often by combing her hair. She has had reflex run superficially, and while they can be affected by microvascular
several similar episodes over the last year. Examination of the head ischemia, they are more suspectable to external compression. Ophthal-
and neck, as well as cranial nerves, is unremarkable. Which of the fol- moplegia with pupillary involvement should raise concern of a poste-
lowing is the most appropriate next step in this patient’s management? rior cerebral artery aneurysm causing compression of CN III. MRI or
a. The patient should be started on carbamazepine and referred to CT angiogram is indicated in this situation. In the absence of pupillary
a neurologist involvement or other cranial nerve abnormalities, there is not a clear
b. Neuroimaging requirement for imaging in the ED. Attention to modifiable risk factors
c. Discharge with short course of oral prednisone. is important as there may be an association of diabetic mononeuropa-
d. Timely referral to a neurosurgeon thy with subsequent stroke risk.
e. NSAIDs and reassurance that condition generally resolves with-
out therapy
92 Spinal Cord Disorders
2. Which feature most often distinguishes a conus medullaris from a
KEY CONCEPTS
cauda equina lesion?
• N ontraumatic spinal cord disorders can be intrinsic or extrinsic, some of a. Back pain
which require prompt diagnosis, advanced imaging, and specialist interven- b. Bilateral symptoms
tion to prevent or limit permanent neurologic dysfunction. c. Distal motor weakness
• The bulbocavernosus reflex is cord-mediated. Return of this reflex following d. Sacral anesthesia
a spinal injury marks the termination of spinal shock. e. Urinary incontinence
• Anterior cord syndrome is marked by symmetrical motor loss but intact pro- Answer: B. Because the conus medullaris is a small structure with
prioception and vibration sense. lumbar and sacral segments represented in a minimal area, a lesion
• In patients with sudden severe back pain, consider spinal subarachnoid will likely result in bilateral symptoms. As cauda equina is not a true
hemorrhage (SSAH) or spinal epidural hematoma (SEH), both of which are cord lesion but rather reflects dysfunction at the level of nerve roots,
diagnosed using magnetic resonance imaging (MRI). the symptoms are frequently unilateral. Both can result in urinary
• Transverse myelitis is inflammation of the spinal cord often associated with retention, fecal incontinence, saddle anesthesia, and distal motor
a prior viral infection resulting in paraplegia and a defined sensory level weakness. Conus medullaris may have upper motor neuron signs
impairment. MRI with contrast enhancement is the diagnostic modality of (hyperreflexia and increased tone), which will be absent in cauda
choice. Roughly one-third of patients have a good outcome. equina.
• Cauda equina syndrome can be difficult to differentiate from conus medul- 3. A 27-year-old woman presents with complaints of back pain and dif-
laris lesions because both can result in bladder retention, fecal inconti- ficulty walking. Her symptoms have been progressive for 2 days. She
nence, leg weakness, and sensory loss in the perineum. Conus lesions are has no significant past medical history. On history it is discovered
more typically bilateral, whereas cauda equina syndrome may be unilateral. that she recently recovered from a bout of “the flu” approximately
Upper motor neuron findings are expected with conus lesions while cauda 3 weeks ago. Physical examination is remarkable for lower extrem-
equina syndrome is associated with hypo-or areflexia. ity hyperreflexia, moderate symmetrical lower extremity weakness,
• Central cord syndrome is often due to a hyperextension injury. Physical find- moderate increased tone, a T10 level of sensory loss, and a PVR urine
ings are represented with the mnemonic MUD: Motor deficits > sensory, volume of 350 mL. What should be the next intervention?
Upper extremities > lower, Distal extremity findings > proximal. a. Antibiotics
• Brown-Séquard syndrome is due to a functional hemisection of the spine b. Complete blood count, erythrocyte sedimentation rate (ESR),
(frequently traumatic) resulting in ipsilateral loss of motor function, pro- and antinuclear antibody levels
prioception, and vibration with contralateral loss of pain and temperature c. MRI scan with gadolinium enhancement
sensation below the level of injury. d. Emergent neurosurgical intervention
• The diagnostic imaging of choice in the majority of suspected spinal disor- e. Steroids
ders is MRI with contrast. Answer: C. Transverse myelitis is postinfectious in 30% of cases and
• A syrinx is a cavitary lesion in the spinal cord that presents with a sensory also idiopathic in 30%. Other causes are autoimmune disorders and
disassociation predominately in the upper extremities. With progression, it infections. Symptoms are typically rapid in onset and progress during
can lead to upper extremity weakness and wasting. Symptom exacerbation 1 or 2 days. Symptoms are often sensory loss below the level of insult,
with cough or Valsalva is typical. extremity weakness, decreased sphincter tone, and urine retention (often
• With compressive lesions of the spinal cord, neurologic status at the time of exhibited with an elevated PVR). Back pain may accompany it. Emergent
intervention and the duration of symptoms are directly related to outcome. MRI is indicated to rule out other causes. There is no proven efficacious
• Autonomic dysreflexia is a complication of spinal cord injury that can result in treatment, although steroids have been used. There is an association with
life-threatening hypertension. Hypertension may be the result of bladder disten- multiple sclerosis (MS). Prognosis for recovery is only fair.
tion, fecal impaction, pain, or infection. Treatment focuses on blood pressure 4. A 23-year-old male patient with no past medical history pres-
management and the identification and treatment of inciting noxious stimuli. ents with longstanding, recurrent headache and neck pain that
he reports has been gradually worsening over several months. He
1. A 43-year-old man with a history of alcohol abuse and liver cirrho- denies any fevers, weakness, numbness, vision changes, back pain,
sis presents with a complaint of one week of progressive difficulty or bowel/bladder changes. He notes that he is often afraid to cough
ambulating and low thoracic back pain. He is noted to be febrile on or sneeze because that makes the symptoms worse. What is another
examination. Which of the following diagnostic modalities is most clinical feature of this patient’s most likely diagnosis?
appropriate in this patient? a. Normal gait
a. Blood cultures b. Loss of pain and temperature sensation in the upper extremities
b. CT myelography c. Lower extremity weakness
c. Lumbar puncture d. Bladder dysfunction
d. MRI with IV contrast e. Loss of proprioception and light touch
e. White blood cell (WBC) count Answer: B. Syringomyelia typically presents with headache, neck pain,
Answer: D. The patient has a presentation concerning for spinal and variable upper extremity dissociative anesthesia: symmetrical loss
epidural abscess (SEA). MRI is the imaging modality of choice and of pain and temperature sensation with preserved posterior column
should be done emergently. Lab tests may be helpful in supporting function. With progression, upper extremity weakness or wasting and
the diagnosis but are neither sensitive nor specific. CT may show lower extremity upper motor neuron changes are expected. Exacerba-
bony abnormalities concerning for SEA but should not replace MRI. tion with cough and the Valsalva maneuver is typical. There is a 90%
Blood cultures are not useful as a diagnostic tool but may reveal the association with type I Arnold-Chiari malformation (cerebellar tonsils
causative organism and help to guide antibiotic therapy. Lumbar and medulla projecting into the spinal canal—often the cause of the
puncture is relatively contraindicated in these patients because it may typical occipital headaches). MRI is diagnostic.
further seed the infection.
5. In the majority of patients with an extrinsic compressive lesion, sur- 7. A 34-year-old male presents after a motorcycle versus car collision.
gical outcome is most dependent on which of the following factors? The patient was wearing a helmet but thrown from his bike. He
a. Age reports burning discomfort and significant weakness to both arms.
b. Laterality of symptoms He also reports some mild numbness but no clear bladder symp-
c. Location of lesion toms. What additional findings would you expect to find on physi-
d. Neurologic status at the time of procedure cal examination?
e. Primary source of neoplasm a. Decreased rectal tone
Answer: D. In patients with SEH or SEA, prognosis is intimately linked b. Loss of pain and temperature below the level of the injury
with neurologic function at time of decompressive laminectomy or c. Distal muscle groups are affected more than proximal muscle
operative intervention. Patients who have had symptoms greater than groups
72 hours with SEH are unlikely to regain lost function. In SEA, neuro- d. Loss of position, vibration, and light touch sensations
logic status rarely improves if intervention is delayed longer than 12 to e. Lower extremity weakness
36 hours from onset of paralysis. Answer: C. The patient has likely suffered a cervical hyperextension
6. A 37-year-old female with a prior history of transverse myelitis at injury resulting in a central cord syndrome. Remember the pneumonic
the T4 level one year ago presents with several hours of sweating, “MUD” for central cord syndrome: motor > sensory, upper > lower,
nausea, and headache. At baseline, she is paraplegic to the lower distal > proximal. Patients with central cord syndrome have variable
extremities, performs intermittent bladder catheterization, and is sensory impairment and bladder dysfunction and these symptoms are
insensate inferiorly to the T4 level. She has never had these symp- not hallmarks of this partial cord syndrome. Brown-Séquard syndrome
toms before. Her vitals are: blood pressure 270/130, heart rate 54, is characterized by ipsilateral loss of motor function, proprioception,
respiratory rate 16, oral temperature of 97.2°F. ECG performed in and vibration with contralateral loss of pain and temperature sensation
triage shows no signs of ischemia or infarct. Which of the following below the level of injury. Anterior cord syndrome is characterized by
actions should be taken next? loss of motor function, pinprick, and light touch below the level of the
a. Manage blood pressure with nitrates and perform a thor- lesion with preservation of the posterior column modalities.
ough physical examination including bladder scan and rectal 8. An 11-month-old female presents to your emergency department
exam. with low-grade fevers and progressive refusal to crawl or pull up.
b. Administer aspirin and obtain cardiology consult. Mom reports she took the patient to her pediatrician last week but
c. Obtain an emergent CTA of chest to assess for aortic dissection. they didn’t find any abnormalities. However, her symptoms have
d. Administer benzodiazepines. continued to worsen. On exam, you see an otherwise healthy infant
e. Order an MRI with IV contrast. who now appears somewhat irritable. Vital signs are as follows: HR
Answer: A. The patient is experiencing autonomic dysreflexia, seen 110, RR 18, T 100.2°F. The patient is being held by mom and resists
frequently in patients with spinal cord injuries above the T6 level. Her movement. Most of her physical exam appears normal, including a
severe hypertension, diaphoresis, nausea, and headache can mimic nonfocal neurologic exam, but she cries as you palpate the lumbar
other diagnoses, but her history of spinal cord lesion must be taken spine. Which of the following is true of the likely diagnosis?
into consideration. Management focuses around aggressive blood pres- a. Blood cultures frequently reveal the causative organism.
sure management with nitrates and investigation into the possible nox- b. WBC is a helpful finding if elevated.
ious stimuli that is causing the syndrome. Bladder-related etiologies c. Plain radiographs are sufficient for diagnosis.
(particularly distention or detrusor overactivity) cause most episodes d. IV antibiotics are usually curative.
of autonomic dysreflexia. Other causes are fecal impaction, pain, tight e. The lack of associated neurologic deficits is unusual in this case.
clothing, and infection. Answer: D. The child has a presentation consistent with discitis. These
children may present with sepsis and, as this is an unusual diagnosis,
the provider should maintain a broad differential. Children are more
likely to develop infections in the vertebral disc and usually have devel-
oped symptoms over several weeks. A hallmark of the diagnosis is that
patients have normal neurologic exams. MRI is the imaging modality
of choice as it is diagnostic and helps to rule out other conditions. The
white blood cell (WBC) count is usually normal and blood cultures
are usually negative. IV antibiotics are effective at treating discitis and
surgery or aspiration are rarely needed.
Peripheral Nerve Disorders 93
KEY CONCEPTS
• T he diagnosis of a specific peripheral neuropathy generally requires confir- • R adial nerve mononeuropathies are characterized by wrist and finger drop
matory ancillary testing; approach in the ED should focus on identifying one and mild numbness over the skin of the first dorsal interosseus muscle.
of seven categorical patterns. • Humeral shaft fractures are associated with radial nerve injury, with “wrist
• Diagnostic approach to peripheral neuropathies involves combining three drop” being the hallmark clinical finding.
clinical features: (1) right-left symmetry or asymmetry, (2) proximal-distal • The ulnar cutaneous innervation to the hand branches from the main trunk
location, and (3) sensorimotor modalities affected. proximal to the nerve entering the Guyon canal. Thus, a lesion at the wrist
• Any patient with symmetrical weakness, distributed both proximally and should not produce sensory abnormalities, whereas one at the elbow would
distally, with loss or diminution of deep tendon reflexes (DTRs) and variable be expected to do so.
sensory abnormalities should be treated as having Guillain-Barré syndrome • The most specific finding for carpal tunnel syndrome (CTS) is splitting of the
(GBS). sensation on the fourth digit (i.e., normal sensation of the ring finger on the
• Respiratory compromise is the primary life-threatening event seen in some ulnar palmar side with abnormal sensation on the median [radial] palmar
peripheral neuropathies; GBS is by far the most common peripheral neuro- side of the same finger).
pathic cause of respiratory arrest. • Lateral femoral cutaneous mononeuropathy (meralgia paresthetica) is
• The definitive treatments for GBS are plasma exchange or intravenous caused by injury to this pure sensory nerve as it passes through or over the
immune globulin (IVIG). inguinal ligament, where it may become entrapped or kinked.
• Most polyneuropathies are characterized by a pattern of distal, symmetrical • The most striking feature of a complete common peroneal mononeuropathy
sensorimotor findings, worse in the lower than in the upper extremities, is footdrop caused by weakness of foot dorsiflexion.
with a stocking-glove distribution of sensory abnormalities that gradually • The most common neurologic abnormality in Lyme disease is unilateral or
diminishes as one moves proximally. bilateral facial nerve palsy, usually occurring within a month of exposure.
• High-level evidence supports the use of pregabalin, gabapentin, and the • ALS is the most common form of motor neuron disease (MND), and diagno-
serotonin and norepinephrine reuptake inhibitor duloxetine in the treatment sis requires the presence of both upper and lower motor neuron findings.
of diabetic distal symmetrical polyneuropathy (DSPN).
1. Which category of peripheral neuropathy tends to occur in an absent lower extremity deep tendon reflexes (DTRs). Symptom
asymmetrical, distal distribution? onset has been during 2 days. What should be the next step?
a. Autonomic neuropathy a. Emergent magnetic resonance imaging (MRI)
b. Large-fiber neuropathy b. Intravenous immune globulin (IVIG)
c. Mixed motor and sensory neuropathy c. Lumbar puncture and antibiotics
d. Pure motor neuropathy d. Pulmonary function studies
Answer: D. Pure motor and pure sensory peripheral neuropathies tend Answer: D. All patients with Guillain-Barré syndrome (GBS) are at
to occur in an asymmetrical distal pattern. risk of respiratory failure. A forced vital capacity (FVC) of less than 20
2. A 26-year-old woman presents with a chief complaint of weak- mL/kg and a negative inspiratory force of less than 30 cm H2O are asso-
ness. She notes a 1- or 2-day onset of easy fatigability and dimin- ciated with impending ventilatory failure and the need for intubation.
ished ability to navigate stairs. She has no past history and takes 4. Among patients with Guillain-Barré syndrome (GBS) who have
no medications. Vital signs are normal. Physical examination normal pulmonary function, which of the following can be mon-
reveals absent lower extremity deep tendon reflexes (DTRs); itored to predict impending ventilatory failure?
symmetrical weakness of the quadriceps, calf muscles, and foot/ a. Deltoid strength
toe dorsiflexion; and minimal sensory loss. Cranial nerve and b. Extensor neck strength
upper extremity examination is normal. Which of the following c. Hand grip strength
is likely? d. Masseter strength
a. An antecedent viral illness Answer: B. Extensor muscle strength has been shown to correlate with
b. Lack of anal sphincter tone ventilatory muscle strength.
c. Onset of ocular muscle dysfunction 5. A 53-year-old diabetic presents with a complaint of increasing
d. Sparing of the autonomic nervous system difficulty walking in the last several months. He has no other past
Answer: A. Guillain-Barré syndrome (GBS) is characterized by history but has been an insulin-dependent diabetic for 23 years.
fairly acute onset of ascending weakness, loss of deep tendon reflexes Current glucose level is 138 mg/dL, and chemistries and complete
(DTRs), and variable sensory loss. Antecedent infections often trig- blood count are otherwise unremarkable. Examination is remark-
ger, with common organisms being campylobacter, cytomegalovirus, able for bilateral lower extremity numbness extending symmetri-
Epstein-Barr virus, and mycoplasma. Rarely, symptoms begin in the cally to above the knees, loss of the Achilles reflex bilaterally with
upper extremities. Urinary retention is common, but anal tone is pre- footdrop, and steppage gait. Which of the following is true?
served. Ocular muscles are usually spared. Autonomic neuropathy is a. Autonomic neuropathy is unlikely.
common, with marked variations in heart rate and blood pressure. b. Facial numbness would necessitate MRI.
Patients with predominantly sensory symptoms tend to have less risk c. Hand numbness is expected.
of respiratory embarrassment and a more favorable prognosis. Lumbar d. Erythrocyte sedimentation rate is likely to be elevated.
puncture shows cerebrospinal fluid (CSF) pleocytosis or may be nor- Answer: C. Diabetic neuropathy is a progressive, ascending mixed
mal early on. polyneuropathy. Hand numbness and upper extremity symptoms usu-
3. A 26-year-old woman presents with lower extremity weakness and ally begin before the lower extremity symptoms ascend to the knees.
difficulty walking. Examination is remarkable for lower extremity Extensive motor loss can occur with gait and grip abnormalities. Skull
symmetrical weakness with mild symmetrical sensory loss and and face numbness can occur. Autonomic dysfunction is expected.
Neuromuscular Disorders 94
a. Bilateral ptosis
KEY CONCEPTS b. Inability to tolerate secretions
• In patients presenting with acute neuromuscular weakness, complaints of c. A forced vital capacity (FVC) of 65 mL/kg
difficulty in breathing or swallowing are signs of bulbar nerve or respiratory d. A negative inspiratory force (NIF) of 40 cm H2O
muscle compromise with potential airway or ventilatory failure. In such e. Recent initiation of a new medication for hypertension
patients, forced vital capacity (FVC) of less than 15 mL/kg or maximal neg- Answer: b. Myasthenic crisis is an emergent, life-threatening exacerba-
ative inspiratory force of less than 15 mm H2O is a potential indication for tion of myasthenia gravis that involves acute weakness of the muscles of
mechanical support, either by intubation or by noninvasive ventilation. the respiratory system. Detecting signs of impending myasthenic crisis
• Initiating new medications in patients with myasthenia gravis may precipi- and differentiating this from a regular exacerbation of myasthenia gravis
tate a crisis. Commonly prescribed cardiovascular agents, including calcium is crucial for emergency medicine providers. Pulmonary function tests,
channel blockers, beta-blockers and antidysrhythmics; commonly used anti- such as forced vital capacity (FVC) and negative inspiratory force (NIF)
biotics; and other agents, including corticosteroids, require particular vigi- can suggest impending crisis, but these tests should not be the sole fac-
lance. tors to consider. Physical exam findings of significant muscle weakness or
• Myasthenic crisis is treated with either plasma exchange or intravenous respiratory distress are the most concerning signs for impending myas-
immune globulin (IVIG). thenic crisis. Bilateral ptosis is a sign of acute myasthenia but, by itself,
• Botulism usually arises as a painless descending paralysis, often first likely does not indicate a significant exacerbation. Inability of the patient
affecting the cranial nerves (with diplopia) and bulbar innervated muscles, to tolerate oral secretions, on the other hand, is a much more concerning
without sensory deficits or alteration of consciousness. The treatment is exam finding and implies significant muscle weakness and impending
airway management with ventilatory support and administration of anti- crisis.
toxin. The previous patient continues to have worsening respiratory symp-
• Injection drug use remains an important cause of wound botulism out- toms. Both neurology and critical care teams are consulted and are
breaks. at bedside. The patient is started on pyridostigmine. Which of the
• Botulism is an important consideration in evaluation a weak and floppy following findings would indicate worsening myasthenic crises as
infant. opposed to overtreatment with pyridostigmine?
• Hypokalemic periodic paralysis is caused by intracellular shift of potassium a. An electrocardiogram showing a heart rate of 35
and so requires frequent serum potassium measurement to avoid overcor- b. Pupil size of 7 mm bilaterally
rection during treatment with potassium infusion. c. Increasing respiratory fatigue
• Patients with newly diagnosed hypokalemic periodic paralysis should be d. Significant abdominal discomfort
evaluated for hyperthyroidism. e. Excessive sweating
Answer: b. When treating myasthenia gravis, worsening of the
1. Which of the following is not found in a neuromuscular junction? patient’s condition can be caused by worsening myasthenic crisis or
a. Myelin sheaths by cholinergic crisis from overtreatment of the patient with anticho-
b. Presynaptic receptors linergic agents. These two conditions can present similarly but have
c. Postsynaptic receptors different causes and thus different treatments. Worsening myasthenic
d. The synaptic cleft crisis occurs from undertreatment of the initial crisis by anticholiner-
e. Acetylcholine gic agents where cholinergic crisis occurs from overtreatment of the
Answer: a. The neuromuscular junction is an important unit in regu- initial crisis. Patients in cholinergic crisis tend to present with brady-
lating muscle function. It consists of both presynaptic and postsynaptic cardia, abdominal pain, and increased secretions. In both cholinergic
receptors that release and receive the neurotransmitter acetylcholine crises and myasthenic crises, increased weakness and respiratory dis-
as it travels through the synaptic cleft. Myelin sheaths are important in tress can be present. However, mydriasis is much more likely to be seen
quickening the transmission of signals along an axon but do not extend in myasthenic crisis.
all the way to the neuromuscular junction. An 8-month-old infant presents to the emergency department with
2. Which of these features in a patient’s history suggest a diagnosis of increasing weakness and poor feeding. Physical exam reveals an
myasthenia gravis? ill-appearing infant with decreased muscle tone. What is the most
a. History of small cell lung carcinoma DEFINITIVE treatment?
b. Worsening of symptoms following a large meal a. Administration of antibiotics against Clostridium species
c. Ascending paralysis b. Administration of corticosteroids
d. An erythematous periorbital rash c. Contacting the Centers for Disease Control and Prevention
e. Improvement of symptoms following plasma exchange (CDC) for antitoxin
Answer: e. Myasthenia gravis, along with several other neuromuscular dis- d. Contacting the California Department of Health for antitoxin
orders, can be treated with plasma exchange. History of small cell lung car- e. Serum and stool testing for botulinum toxin
cinoma is associated with Lambert-Eaton syndrome. Ascending paralysis Answer: d. This patient likely has infantile botulism and the definitive
is seen more commonly in Guillain-Barré syndrome and tick paralysis. An treatment is obtaining IV human-derived botulism immunoglobulin,
erythematous periorbital, or heliotrope, rash is seen in dermatomyositis. or BabyBIG, from the California Department of Health. The CDC has a
3. A 40-year-old female patient with a history of myasthenia gra- heptavalent botulinum toxin for noninfantile cases. IV corticosteroids
vis presents to the emergency department with mild shortness of and antibiotics are not recommended for cases of infantile botulism.
breath upon exertion. You are concerned for an impending myas- Serum and stool testing can diagnose botulism but often have a pro-
thenic crisis. Which of the following patient factors would most longed turnaround time and so are not useful in emergency depart-
suggest this diagnosis? ment management.
6. Which of the following is TRUE about wound botulism? 8. An 8-year-old patient presents to the emergency department with
a. It is the most common form of botulism. diffuse weakness and myalgias. His parents note that he recently
b. Antibiotics do not have a role in wound botulism management. had a muscle biopsy which confirmed an unspecified illness. You
c. Compared with other forms of botulism, wound botulism also note a rash on the patient’s chest. What would you likely find
patients more often have abdominal complaints. on the patient’s physical exam?
d. It is associated with enteroviruses. a. Difficulty in sitting upright from a supine position
e. It is associated with topical tobramycin treatment. b. Difficulty in holding a pencil or pen
Answer: c. Wound botulism is caused by infection of a wound with c. Fasciculations of proximal muscle groups
Clostridium botulinum which then causes systemic infection via hema- d. Atrophy of proximal muscle groups
togenous spread. It is an infrequent presentation of C. botulinum e. Increase responsiveness of the patellar reflex
infection; infantile botulism is the most common form of botulism. Answer: a. This patient likely has dermatomyositis. Dermatomyositis
Antibiotics as well as surgical consultation are first line in treatment. is an inflammatory myopathy that most commonly affects the proximal
Patients with wound botulism often present with fever and less often muscle groups and spares the distal muscle groups. This patient likely
present with abdominal symptoms compared with other forms of bot- would have trouble sitting up as it uses the proximal hip flexor muscles.
ulism. Enteroviruses are not associated with botulism but are associ- Distal muscles, such as those in the hand, are less commonly affected.
ated with acute flaccid myelitis (AFM). Tobramycin treatment can be a Fasciculations are not commonly seen. Atrophy is generally a much
trigger for acute myasthenia gravis but not for wound botulism. later finding in inflammatory myopathies. Patients also tend to present
7. A patient presents with an episode of flaccid lower extremity paral- with decreased patellar reflexes.
ysis. He notes that he frequently gets these episodes and the condi- 9. Which of the following pairs is correct regarding a neuromuscular
tion runs in his family. His serum potassium concentration is 5.9 disease and its pathophysiology?
mEq/L. What is the next best treatment step? a. Botulism—concomitant gastroenteritis is almost always present
a. Intravenous (IV) immunoglobulin b. Myasthenia gravis—autoantibodies attack postsynaptic acetyl-
b. IV hydration choline (ACh) receptors
c. Thyroid function testing c. Periodic paralysis—hyperkalemic form is most common
d. Magnesium supplementation d. Polymyositis—distal muscle weakness precedes proximal mus-
e. High-dose insulin and dextrose cle weakness
Answer: b. This patient likely is having an episode of hyperkalemic e. Tick paralysis—symptoms resolve quickly after the tick is removed
periodic paralysis. Hyperkalemic periodic paralysis is due to an imbal- Answer: b. Myasthenia gravis is caused by autoantibodies that attack
ance in the distribution of potassium at a cellular level. IV hydration postsynaptic ACh receptors at the neuromuscular junction. Botulism
helps to rebalance the body’s potassium stores. Testing of thyroid func- usually does not present with gastroenteritis. Periodic paralysis is most
tion and high-dose insulin therapy may be indicated but are not the commonly associated with hypokalemia. Polymyositis usually presents
immediate next steps. IV immunoglobulin and magnesium supple- first in proximal muscles. Tick paralysis may take days to weeks to
mentation are not indicated in this case. resolve after removal of an offending tick.
95 Central Nervous System Infections
KEY CONCEPTS infection. Treatment should not be delayed obtaining neuroimaging
first (if necessary), or until the lumbar puncture is completed. How-
• C NS infection should be considered in all patients with headache, nuchal ever, lumbar puncture should still be obtained expeditiously, because
rigidity, fever, altered sensorium, or diffuse or focal neurologic findings. culture yields begin to decrease after 1 hour.
• Patients with suspected CNS infection should be asked about history 3. Which of the following antimicrobial regimens would be appropri-
focused on risk factors for CNS disease and infection and receive a full ate coverage for suspected meningitis in a 40-year-old patient with
neurologic examination. no comorbidities in a country with high antibiotic use?
• CSF testing is the most reliable method of assessing the presence of men- a. Ceftriaxone
ingitis and should be obtained in all patients in which there is a suspicion b. Ceftriaxone and vancomycin
for CNS infection. c. Acyclovir and ceftriaxone
• Early initiation of empirical antimicrobial therapy is recommended in cases d. Cefotaxime, vancomycin, and ampicillin
of suspected acute CNS infection. This should occur before imaging or lum- Answer: B. Ceftriaxone and vancomycin are the appropriate initial
bar puncture. antibiotics for suspected meningitis or encephalitis in an adult patient.
• Patients without focal neurologic symptoms, altered level of consciousness Ceftriaxone alone may be appropriate in lower income countries with
or signs of increased intracranial pressure do not require imaging prior to less antibiotic use and resistance. Ampicillin is added for patients older
lumbar puncture. than 50 years and neonates due to risk of Listeria. Acyclovir should
• We recommend steroid treatment before or alongside treatment with anti- be added for immunocompromised patients with suspected HSV or
biotics in all cases of suspected meningoencephalitis. VZV infections. Cefepime should be used instead of ceftriaxone in
• First-line treatment for healthy adults with suspected bacterial meningitis patients at his risk for pseudomonas infections (e.g., recent surgical
is ceftriaxone or cefotaxime plus vancomycin in most countries given high procedures).
antibiotic resistance.
4. What is the most appropriate imaging test for a patient with low
• Acyclovir is recommended for patients with suspected encephalitis because
back pain, low-grade fever, and a history of IV drug use?
HSV is a common pathogen.
a. CT lumbar spine without contrast
• Clinical course and risk factors, such as immunosuppression, dictate the
b. CT myelogram of the lumbar spine
need to test and treat for fungal and tuberculous meningoencephalitis.
c. MRI of the lumbar spine with contrast
• In the absence of a clear clinical picture of viral meningitis, patients should
d. MRI of the cervical, thoracic and lumbar spine with contrast
generally be empirically covered for bacterial meningitis and admitted to
Answer: D. Because exam findings can be inaccurate, patients with
the hospital.
suspected epidural abscess should receive an MRI with contrast of their
• Antibiotic chemoprophylaxis should be given for close contacts of patients
entire spine. Presentations can be subtle and may not include all of the
with meningitis resulting from N. meningitidis and contacts of patients with
classical features such as fever and back pain; patients must be que-
H. influenzae living with immunocompromised or unvaccinated individuals.
ried for potential risk factors. CT myelogram of the entire spine is an
• CT or MRI can be used in patients with a suspected brain abscess.
acceptable alternative for patients unable to undergo an MRI.
• MRI with gadolinium contrast of the entire spine is the definitive imaging
test for patients with suspected spinal epidural abscess. 5. Contacts with exposure to which of the following types of meningo-
encephalitis should receive prophylactic antibiotics?
a. Pneumococcal
1. Which of the following presentations should receive a lumbar b. Tuberculous
puncture to rule out CNS infection: c. Meningococcal
a. 23-year-old healthy female with fever and stiff neck but minimal d. West Nile
headache. Answer: C. Close contacts with exposure to patients diagnosed with
b. 32-year-old male with diabetes and unexplained altered mental meningococcal meningitis or encephalitis should receive chemopro-
status. phylaxis, most commonly with oral rifampin. Chemoprophylaxis is
c. 67-year-old male with fever and new right arm weakness. also recommended for immunocompromised close contacts of patients
d. All of the above with H. influenzae CNS meningitis.
Answer: D. All patients with fever, headache, altered mental status or
new neurologic deficits should be considered for possible meningitis
or encephalitis. Lumbar puncture and CSF analysis is the most reliable
method for assessing for CNS infection; imaging and blood tests are
inadequate alone.
2. Patients with suspected bacterial meningitis should receive which
of the following interventions FIRST?
a. Non-contrast CT of the head
b. MRI with contrast of the head
c. Lumbar puncture to evaluate for bacterial cause
d. Steroids and antibiotics for presumed bacterial infection
Answer: D. Patients with suspected bacterial meningitis should receive
steroids and antibiotics first to cover for the possibility of bacterial
SECTION EIGHT Behavioral Disorders
Thought Disorders 96
KEY CONCEPTS 4. What is the most common adverse effect seen with neuroleptic agents?
a. Akinesia
• T hought disorder symptoms can be precipitated by psychiatric, underlying b. Dystonia
medical, and toxicologic etiologies. c. Orthostatic hypotension
• Diagnostic testing should be patient specific and based on the particular d. Pseudoparkinsonism
medical processes that the clinician feels may be causing or exacerbating e. Tardive dyskinesia
the thought disorder, rather than panels of routine tests. Answer: B. Dystonia occurs in 1% to 5% of this patient population.
• Consider nonphysical interventions first when appropriate, but chemical The reaction occurs because of a dopaminergic pathway disruption
sedation or physical restraints may become immediately necessary for with a resulting cholinergic predominance. Anticholinergics should be
patients who demonstrate life- threatening aggressive and dangerous administered parenterally (Benadryl 25 to 50 mg intravenous [IV] or
behaviors. Cogentin 1 or 2 mg IV), followed by 48 to 72 hours of oral follow-up
• Appropriate disposition depends on the etiology of the underlying psycho- treatment to prevent recurrence. Patients may experience tongue pro-
sis and response to treatment while addressing patient and community safety trusion (buccolingual crisis), upward eye deviation (oculogyric crisis),
considerations. Psychiatric consultation is often required. back arching (opisthotonus), and, rarely, laryngospasm. Symptoms
may lessen with voluntary muscle action and increase with stress.
1. Which of the following pharmacologic agents have been implicated 5. A 27-year-old known schizophrenic is brought to the ED for altered
in causing acute psychosis? mental status. His only known medication is clozapine, which he
a. Aripiprazole, hydralazine, nitroglycerin started 4 weeks ago with subsequent dose increases. He has no other
b. Diazepam, rifampin, captopril past history. Physical examination reveals a muscular man who is
c. Hydrochlorothiazide, acetaminophen, albuterol somnolent and diaphoretic. He withdraws all extremities stiffly and
d. Lorazepam, salsalate, rocuronium grimaces in pain. Vital signs are temperature, 40.5°C; heart rate, 146
e. Penicillin, ceftriaxone, risperidone beats per minute; blood pressure, 205/125 mm Hg; and respiratory
Answer: B. Box 96.2 provides an extensive list of other agents that may rate 28 breaths per minute. Rectal examination is guaiac positive.
cause psychosis. Foley placement shows brown urine. What should be the next diag-
2. Rapid tranquilization using a neuroleptic agent would be indicated nostic maneuver?
in which of the following cases? a. Creatine kinase level
a. An intoxicated schizophrenic b. Head computed tomography (CT) scan
b. Anticholinergic psychosis c. Lumbar puncture
c. A lactating schizophrenic d. Thyroid hormone levels
d. A phencyclidine overdose e. Urine drug screen
e. A pregnant schizophrenic Answer: A. Neuroleptic malignant syndrome is an idiopathic condi-
Answer: A. Neuroleptics are contraindicated in choices B to E. They tion clinically similar to serotonin syndrome and malignant hyperther-
should not be the sole agent for alcohol withdrawal but would be useful mia. Milder cases may be confused with serotonin syndrome. Severe
for acute psychotic agitation. cases, related to possible hypothalamic dysfunction, present with fever,
3. A 45-year-old woman presents to the ED complaining of severe anxi- rigidity, altered mental status, autonomic instability, elevated cre-
ety and unrest. She has a history of moderate schizophrenia, for which atine phosphokinase (CPK), and possibly rhabdomyolysis. It is seen
she was placed on olanzapine 2 months prior. She has been compliant. with both typical and atypical antipsychotics and generally occurs in
Physical examination is remarkable for the presence of anxiety, clear the first few weeks of treatment. Complications may include hepatic/
sensorium and orientation, and normal speech. She is restlessly pac- renal failure, gastrointestinal (GI) hemorrhage, and respiratory failure.
ing the room and reports being compelled to keep moving. Urine drug Severe cases may require intravenous dantrolene or dopamine agonists
screen is negative. What would be the most appropriate therapy? (e.g., bromocriptine).
a. Benztropine orally
b. Lorazepam orally
c. Olanzapine intravenously
d. Psychiatry consultation
e. Ziprasidone intravenously
Answer: A. Akathisia is a state of motor restlessness characterized by a
physical need to be constantly moving. The patient does not want to do
so but feels compelled. It is most commonly seen in middle-aged patients
within the first few months of starting treatment. It may be mistaken for
an acute deterioration, but psychotic features are not increased. Treat-
ment is with oral beta-blockers and anticholinergics (benztropine).
97 Mood Disorders
KEY CONCEPTS
• P atients with apparent mood disorders should be evaluated for medical
disorders, medication effects, or substance abuse or withdrawal because
these conditions can mimic both depression and mania.
• Mood disorders should be suspected in patients with multiple, vague,
nonspecific complaints and in patients who are frequent, heavy users of
medical care.
• Patients with mood disorders should be assessed for their suicide potential.
• Pharmacologic treatment and linkage to care after discharge are important
parts of managing mood disorders in the emergency setting.
1. What is the lifetime suicide risk for people with major untreated Answer: b. Depression in children and adolescents can be misdiag-
depression? nosed as ADD. Somatic complaints are a common feature of chil-
a. 5% dren and adolescents presenting with depression, but the diagnostic
b. 10% criteria are not different. Geriatric depression may be manifested
c. 15% in a manner similar to dementia (pseudodementia), but unlike
d. 20% dementia, the depression is highly treatable and reversible once it
e. 25% is recognized.
Answer: c. Patients with major depression have a high lifetime suicide 4. A 31-year-old attorney is brought to the emergency department
risk, and although episodes of acute decompensation with even higher (ED) by his family for a chief complaint of agitation and a behavioral
risk can be identified and treated, a certain number of patients succeed change. He has no past medical history and takes no medications.
in committing suicide. The family reports decreased sleep, increased talkativeness, marked
2. Which of the following abnormalities of central nervous system increased time and involvement at work, and an uncharacteristic
neurotransmitters is associated with clinical depression? buying spree. Your examination is remarkable for distractibility,
a. Decreased hypothalamic-pituitary-adrenal (HPA) activity gregarious and pressured speech, flight of ideas, and mild psycho-
b. Depressed serotonin levels motor agitation. Laboratory examination and urine drug screen
c. Elevated gamma-aminobutyric acid (GABA) levels results are negative. The patient is adamant that he has important
d. Elevated norepinephrine levels things to do and needs to leave. Which of the following statements
e. Unchanged dopamine levels regarding this patient’s most likely diagnosis is true?
Answer: b. The central biochemical features toward which pharmaco- a. Antipsychotic agents are not effective.
logic management is directed are depressed levels of norepinephrine b. Hallucinations would be atypical.
and serotonin. Data are also emerging that suggest decreased dopamine c. If treated, intravenous valproic acid is indicated.
levels. The HPA axis may also be altered with elevated cortisol levels. d. Initiating treatment in the ED is not indicated.
3. Which of the following statements regarding depression in children e. Multiple antibiotics can cause this clinical picture.
and elders is true? Answer: e. This patient has a fairly classic presentation for acute
a. Children with depression rarely present with somatic com- mania with pressured speech, distractibility, grandiosity, increased
plaints. involvement (in this case with work), and decreased need for sleep.
b. Depression in children may be misdiagnosed as attention deficit Multiple drugs may precipitate this, including acyclovir, isoniazid, sul-
disorder (ADD). fonamides, the floxins, and chloroquine. An acute manic episode may
c. Depression presents differently from dementia in elders. be manifested with hallucinations and mimic an acute psychosis. ED
d. Diagnostic criteria for depression in children are different. treatment is usually indicated for this disorder. Acute stabilization is
e. Serious depression in elders is generally refractory to treatment. generally effective with major tranquilizers, such as haloperidol.
Anxiety Disorders
98
KEY CONCEPTS
• P atients who present with predominant symptoms of anxiety may be suf-
fering from medical disorders, medication effects, or substance abuse or
withdrawal.
• Anxiety may accompany the onset of serious medical disease, cause signifi-
cant metabolic demands, and stress a marginally compensated organ system.
• Anxiety caused by non-psychiatric illness is usually suggested by the
patient’s physical examination findings but may require testing to further
delineate the cause.
• Oral, intravenous, or intramuscular medication may be necessary for
patients who are a significant threat to themselves or others and for anx-
ious patients with significant medical illness.
• Limited benzodiazepine therapy may be helpful for select patients.
1. Which of the following is the most common mental health disor- 4. Which of the following statements regarding anxiety and endocrine
der? disorders is true?
a. Anxiety a. Anxiety can often be traced to reactive hypoglycemia.
b. Bipolar b. Anxiety is not a manifestation of hypothyroidism.
c. Depression c. Diabetics treated with antianxiety agents have improved hemo-
d. Schizophrenia globin A1c levels.
e. Substance abuse d. Less than 5% of diabetics experience anxiety.
Answer: A. Many of these patients never receive appropriate care, in e. Patterns of diaphoresis in pheochromocytoma mimic those of a
part because they choose to present with a physical complaint and dis- panic attack.
guise their anxiety. Patients with chronic illnesses have higher rates of Answer: C. Approximately 15% of diabetics have an anxiety disorder.
anxiety and depression than the rest of the population. Treatment improves hemoglobin A1c levels. Anxiety due to reactive
2. What is the most common cause of organic anxiety, anxiety that hypoglycemia is rare despite the common perception among patients.
results from a physiologic origin? Pheochromocytoma causes whole-body diaphoresis, whereas panic
a. Adrenal disorders disorders primarily cause sweaty palms. Hyperthyroidism or hypothy-
b. Alcohol and drug use roidism can cause significant anxiety manifestations. It is more related
c. Cardiac disease to the rate of change than the level of thyroid hormones.
d. Hyperthyroidism 5. A 23-year-old woman with a history of asthma presents with
e. Pulmonary embolus increasingly frequent episodes of panic attacks. Her medications
Answer: B. This may be from intoxication or withdrawal states. are an inhaled beta-agonist and an intermittent steroid inhaler. She
3. A 52-year-old woman presents with 2 months of recurrent episodes reports subjective increasing asthma severity as her panic episodes
of anxiety, mild chest pain, subjective palpitations, hand paresthe- have worsened. When counseling the patient, which of the follow-
sias, and occasional muscle spasms. They have occurred weekly in ing statements is most correct?
the past but are now increasing in frequency. Her only past history a. An anxiety disorder in an asthmatic patient does not increase
is a thyroidectomy 4 months prior. She is taking levothyroxine (Syn- morbidity.
throid) and had normal thyroid levels 2 weeks ago. Her vital signs, b. Anxiety does not precipitate asthma attacks.
physical examination, and electrocardiogram are normal. Labora- c. Anxiety does not worsen airflow.
tory evaluation shows sodium 141 mEq/L, potassium 4.1 mEq/L, d. Asthmatics are more likely to have an anxiety disorder.
creatinine 1.0 mg/dL, bicarbonate 26 mEq/L, chloride 100 mEq/L, e. It is difficult to differentiate dyspnea related to asthma from anx-
and calcium 7.1 mg/dL; a complete blood count is normal. Which iety.
of the following should be the next step in her management? Answer: D. Anxiety can precipitate and prolong an asthma attack.
a. Outpatient clonazepam Morbidity and mortality are increased in asthmatic patients who have
b. Parathyroid hormone level a coexisting anxiety disorder. Patients who have asthma are twice as
c. Psychiatry consultation likely to have an anxiety disorder and five times as likely to have a pho-
d. Thyroid hormone levels bia. Acute dyspnea from “panic” dyspnea can be differentiated from
e. Urine drug screen asthma by clear lungs on auscultation.
Answer: B. Anxiety is the predominant symptom in 20% of patients 6. Which of the following syndromes is not associated with anxiety?
with hypoparathyroidism. Other symptoms include paresthesia, mus- a. Left hemispheric strokes
cle cramps, and spasms. Most cases are idiopathic or due to inadver- b. Multiple sclerosis
tent parathyroid gland harvest during thyroidectomy. The diagnosis is c. Right hemispheric strokes
suggested by a low serum calcium and an elevated phosphate and is d. Transient ischemia attack
confirmed by a depressed parathyroid level. e. All of the above can be associated with anxiety.
Answer: E. Anxiety may be a component of seizures, tumors, arte- c. Have her try dosing the benzodiazepine at bedtime, because
riovenous malformations, and ischemic events. It may be the only this will likely continue to control her anxiety and limit daytime
manifestation of some disorders (e.g., right hemispheric strokes and somnolence.
transient ischemic attacks [TIAs]). The coexistence of anxiety plays an d. Switch the patient to a selective serotonin reuptake inhibitor
important role in the prognosis and impairment of stroke patients. (SSRI) and refer her back to her psychiatrist.
7. A 38-year-old woman with a long history of anxiety and panic dis- e. Switch the patient to a shorter-acting benzodiazepine.
order presents with anhedonia, melancholy, sleep disruption, cry- Answer: C. Instituting an SSRI should be reserved for primary care
ing episodes, and some hostility feelings. She has no current anxiety physicians or psychiatrists who can monitor the patient more closely,
symptoms. Her only medication is clonazepam. She has no known because the response will be delayed. Some patients do adapt to the
medical illness. Which of the following statements regarding this sedative effects of benzodiazepines but usually only after long-term
patient’s symptoms is true? use. Stopping the benzodiazepine may ultimately be necessary but
a. Approximately 50% of patients with panic disorder develop at the risk of recurrent anxiety. Dosing benzodiazepines at bedtime
major depression. may minimize daytime sedation and still provide an anxiolytic effect.
b. Depression with anxiety and hostility is usually refractory to Shorter- acting benzodiazepines produce a more severe abstinence
treatment. syndrome when stopped abruptly, and thus most prescribers prefer
c. The first diagnostic step should be a thyroid panel. longer-acting agents.
d. The majority of patients with depression have panic attacks. 10. A 52-year-old male construction worker presents with chest pain.
e. This is likely a drug-induced depression. He states his symptoms began early this morning and have pro-
Answer: A. Approximately 50% of patients with a primary panic dis- gressively worsened throughout the day. His symptoms include
order will later develop major depression. Twenty percent of patients nervousness, tremors, chest pain, shortness of breath, and pal-
with depression have panic attacks. Depression with panic attacks is pitations. He states that he has had anxiety for 30 years but has
less responsive to treatment, but depression with anxiety and hostility controlled it with the consumption of alcohol. He became unem-
responds well to antidepressants. Although benzodiazepines can exac- ployed 1 week ago, and his daily alcohol use has diminished sig-
erbate symptoms of depression, there is already a high spontaneous nificantly. His vital signs are blood pressure (BP) 185/95 mm Hg,
rate of depression with anxiety disorders. heart rate 123 beats per minute, respiratory rate of 20 breaths per
8. Which of the following statements regarding benzodiazepine use minute, and temperature of 98.9°F. His physical examination is
and anxiety is true? remarkable for diaphoresis, tongue fasciculation, both resting and
a. Benzodiazepines are first-line agents for anxiety disorders. intention tremors, and mild psychomotor agitation while main-
b. Several weeks of treatment are indicated after initial diagnosis. taining orientation with a congruent anxious mood and affect.
c. Short-acting benzodiazepines produce a more severe abstinence What is the most likely etiology of this patient’s symptoms?
syndrome. a. Acute alcohol withdrawal syndrome
d. They are particularly useful in patients with alcohol abuse. b. Exacerbation of endogenous anxiety secondary to diminished
e. Withdrawal rebound is less common than with selective sero- alcohol intake
tonin reuptake inhibitors (SSRIs). c. Exacerbation of exogenous anxiety secondary to change in
Answer: C. SSRIs are the first-line agents for anxiety and panic dis- employment status
orders, but the primary disadvantage is the several-week lag needed d. Hypertensive emergency with acute coronary syndrome
for maximal clinical benefit. Benzodiazepines work best for moti- e. Reactive anxiety secondary to the onset of chest pain
vated, dependable patients when an immediate reduction of symptoms Answer: A. Hypertensive emergency is unlikely given the level of this
is indicated or a short-term treatment is necessary. Patients who do patient’s BP. On the basis of the history alone, it may be difficult to dif-
not benefit from benzodiazepines within a week are unlikely to do so. ferentiate organic versus functional anxiety or identify an exogenous
Patients with a history of alcoholism or drug abuse, who are exces- trigger, but the abnormal vital signs and physical examination associ-
sively/emotionally dependent, or who become anxious from normal ated with a recent cessation of long-term alcohol consumption makes
stress are at greater risk for dependency. Rebound withdrawal is more acute alcohol withdrawal the most likely cause. Given the significant
likely after short-acting agents. morbidity associated with withdrawal states, this must be addressed
9. A 29-year-old Caucasian female presents with excessive daytime somno- acutely. Appropriate diagnosis and management of underlying psychi-
lence. She states that she had been suffering from anxiety associated with atric disease will be a secondary concern after the patient’s withdrawal
her paralegal occupation, and 1 week ago her psychiatrist had started is managed.
her on a 2-week course of once-daily benzodiazepine therapy, which
she takes in the morning. Her anxiety symptoms are well controlled. She
asks if you can change her to a new medication because the somnolence
is significantly affecting her job performance. What would be the most
appropriate course of action?
a. Counsel the patient that she should continue the medication as
prescribed because she will soon adapt and the somnolence will
likely subside.
b. Discontinue the benzodiazepine and refer her back to her psy-
chiatrist.
99 Somatic Symptoms and Related Disorders
KEY CONCEPTS 3. A patient presents multiple, nonspecific symptoms and believes that
she has cancer despite multiple previous evaluations in emergency, pri-
• S everal conditions previously classified in the fourth edition of the Diag- mary care, and specialist settings. She says she is certain of this diagno-
nostic and Statistical Manual of Mental Disorders (DSM-IV) as somato- sis, that it causes a significant amount of anxiety and that she spends
form disorders are now classified under DSM-V as Somatic Symptom several hours each day researching the subject. Your standard work-up
and Related Disorders (SSRD). These include somatic symptom disorder for her presenting symptoms is negative. What should you do?
(SSD), illness anxiety disorder (IAD), and conversion disorder. They share a a. Perform a few more tests to reassure the patient.
common feature of patients’ maladaptive and inappropriate psychological b. Tell the patient that this is “all in your head” and promptly discharge.
response to somatic (bodily) symptoms. c. Place the patient under an involuntary psychiatric hold for self-harm.
• SSRD patients have approximately twice the rate of medical disease seen d. Seek therapeutic alliance, provide reassurance, and refer the
in the general population. It is unclear whether this is the consequence patient to primary care or psychiatry for management.
of more frequent health care use or whether an increased disease burden Answer: d. This patient’s presentation is consistent with illness anxiety
prompts these patients to have a greater concern for bodily sensations. disorder, formerly known as hypochondriasis. Note that patients with
• SSD is characterized by disproportionate or persistent health- related this disorder may have few or no somatic symptoms.
thoughts, anxiety, and time and energy devoted to somatic (bodily) symp- 4. A predominance of pain is a subtype of which of the somatic symp-
toms, resulting in disruption of daily life. tom and related disorders?
• Patients with IAD, formerly known as hypochondriasis, have excessive anx- a. Somatic symptom disorder.
iety regarding the possibly having or acquiring a serious medical illness in b. Illness anxiety disorder.
the presence of minimal or absent somatic symptoms. c. Conversion disorder.
• Conversion disorder, also known as functional neurologic symptom disor- d. Psychological factors affecting medical illness.
der, is characterized by abnormal sensory or voluntary motor function that Answer: a. Predominant pain is a subtype of somatic symptom disor-
is found to be incompatible with known neurologic or medical conditions, der in which pain is the defining symptom. This may be due to prior
and that causes significant distress or life impairment. neural sensitization or genetic predisposition.
• The differential diagnoses for SSRD may be broadly divided between 5. Which of the following is considered the gold standard for diagnos-
(a) psychiatric disorders that manifest somatic symptoms and (b) medical ing somatic symptom and related disorders (SSRD)?
conditions with signs or symptoms that might be attributed to psychiatric a. Elimination of other possible medical conditions via extensive
disorders. medical testing.
• A “positive review of systems” in an emergency department (ED) evaluation b. Structured clinical interview.
is similar to a high score on SSRD symptoms severity scales and thus may c. A positive review of symptoms.
serve as an inadvertent screen for SSRD. Further research is needed to d. Emergency physician gestalt.
ascertain whether this is clinically reliable. Answer: b. SSRD diagnoses are suggested by overall clinical picture
• ED care goals for patients with SSRD include establishment of rapport, and may be supported by self-report questionnaires. However, final
building a therapeutic alliance, legitimizing the patient’s distress, and enhanc- diagnosis is typically based upon formal structured clinical interview
ing the patient’s ability to function despite the symptoms. in psychiatric or primary care settings. Note that a “positive ROS” is
• Multiple care modalities are available for SSRD treatment. These are typi- similar to high scores on SSRD symptom severity questionnaires.
cally managed by the patient’s primary care physician or psychiatrist.
102 Arthritis
septic arthritis with gram stain showing gram positive cocci. What
KEY CONCEPTS
is true about the treatment of this patient with septic arthritis?
1. A rthritis can be considered in three broad classes: degenerative, infec- a. Intra-articular antibiotic treatment is the gold standard and
tious, and inflammatory. The number of joints involved, time course, and should be given immediately.
presence of other systemic symptoms can help in classification. b. Septic arthritis can safely be treated with oral antibiotics as the
2. Septic (acute infectious) arthritis should be considered in any patient pre- first line of treatment as long as the patient does not have a fever.
senting with acute monoarticular arthritis. c. Synovial fluid aspiration is rarely helpful in the treatment of sep-
3. No singular physical examination finding, serum test, or synovial fluid test tic arthritis. Blood cultures are sufficient.
in isolation can diagnose or exclude the diagnosis of a septic joint. d. Vancomycin is a good choice for initial antibiotic treatment in
4. Arthrocentesis can aid in diagnosis and should be considered if the diag- this patient with septic arthritis.
nosis is unclear or any time infection is suspected. Answer: d. A is incorrect because intraarticular antibiotics do not play
5. Crystal arthritis and septic arthritis can and do frequently coexist. The pres-
a role in the treatment of septic arthritis. IV antibiotics are the gold
ence of crystals in joint aspirate should not be used to exclude infection.
standard. B is incorrect because IV antibiotics are the first line of treat-
6. Gram stain of synovial fluid is positive only in 50% of patients with con-
ment. Fever is not always present with septic arthritis. C is incorrect
firmed infection and should not be used in isolation to exclude infection.
7. Many common arthritis conditions, including gout, rheumatoid arthritis, and
because synovial fluid aspiration is extremely important in the treat-
osteoarthritis, can be managed in the outpatient setting. Analgesics, spe- ment of septic arthritis. Drainage of the joint is important, but it can
cifically antiinflammatory medications, are the mainstay of initial therapy. also guide your antibiotic choice. D is correct because vancomycin
8. A systematic history and physical examination should be performed to 15–20 mg/kg/dose every 8 to 12 hours is a good starting antibiotic,
evaluate for multiple joint involvement or signs of systemic disease. with a positive gram stain showing a gram positive organism. Ceftri-
9. Intra-
articular corticosteroid injections can be considered in select axone 2 g IV (or other cephalosporin, see Table 102.8) can be added if
patient populations where systemic therapy is either contraindicated or gram stain is inconclusive or if results are not readily available.
presents a significant risk to the patient. 3. Which of the following statements regarding septic arthritis is true?
10. Consider ultrasound evaluation of acutely painful joints to better evaluate a. Streptococcus pneumonia is the most common cause of bacterial
for the presence of joint effusion requiring aspiration. arthritis
b. Older patients are more likely to present with fever
1. A 72-year-old male with history of gout presents to the emergency c. Synovial fluid C-reactive protein (CRP) is more accurate than
department (ED) with worsening knee pain and swelling. He states serum CRP
this pain is worse than previous gout episodes. A diagnostic arthro- d. Blood cultures should be obtained when bacterial arthritis is
centesis is performed. Which of the following synovial fluid results suspected
would be most consistent with a diagnosis of septic arthritis? Answer: d. Blood cultures are recommended in all suspected septic
a. Gram stain negative, positive crystals (needle-shaped), synovial arthritis cases because bacteremia is a relatively common finding. The
white blood cell (WBC) count 25K, polymorphonuclear (PMN) most common pathogen is Staphylococcus aureus. Older patients are
70%, turbid appearance less likely to present with a fever. Synovial and serum CRP levels are
b. Gram stain negative, negative crystals, synovial WBC count similar in diagnostic value.
12K, red blood cell (RBC) 1K, fat globules in aspirate
c. Gram stain negative, positive crystals (needle-shaped), synovial
WBC count 40K, PMN 92%, turbid appearance
d. Gram stain negative, positive crystals (rhomboid-shaped), syno-
vial WBC count 35K, PMN 60%
Answer: c. A synovial fluid sample with PMN greater than 90% is pre-
sumed to be consistent with septic arthritis regardless of total synovial
WBC count. Answer choices a and d may represent septic arthritis
in certain scenarios, but neither is diagnostic. These represent acute
gout and pseudogout, respectively. Answer b is representative of a lipo-
hemarthrosis consistent with intra-articular fracture. A positive Gram
stain would also be diagnostic of septic arthritis, although approxi-
mately 50% of cases of septic arthritis are associated with a negative
Gram stain.
2. A 38-year-old previously healthy female presents to the emergency
department with a complaint of worsening elbow pain and swelling.
She plays recreational soccer and fell on her elbow one week ago.
There was a small laceration over the elbow, but she has been trying
to keep it covered with gauze. Over the last two days, the elbow has
become more swollen, red, and painful. Joint aspiration confirms
4. A previously healthy 19-year-old female presents with an acutely 5. A 54-year-old male with poorly controlled hypertension and cor--
swollen and painful left knee starting a few days ago. She denies onary artery disease on daily aspirin presents with a swollen first
trauma or any previous history of similar episodes. She denies fever metatarsophalangeal (MTP) joint on the right foot. You make the
or chills but has had minor pelvic discomfort and a vaginal dis- diagnosis of gout. Which of the following is the most appropriate ini-
charge, as well as bilateral wrist pain that resolved spontaneously. tial treatment for this patient with a presumptive diagnosis of gout?
In addition to performing diagnostic joint aspiration, what tests a. Naproxen 550 mg twice daily
should be ordered on this patient? b. Naproxen 550 mg twice daily coupled with daily proton pump
a. Oral and genital swabs for polymerase chain reaction (PCR) inhibitor (PPI) therapy
b. C-reactive protein (CRP) and sedimentation rate c. Colchicine
c. Serum uric acid levels d. Allopurinol and colchicine
d. Lyme titers e. Febuxostat and naproxen
Answer: a. This patient has gonococcal arthritis. This bacterium is Answer: b. This patient is moderate risk for gastrointestinal (GI) injury
notoriously difficult to find in synovial samples, and it is recommended due to history of poorly controlled hypertension and daily aspirin use;
to obtain oral and genital samples, in addition to samples of any skin thus an oral nonsteroidal antiinflammatory drug (NSAID) coupled
lesions that may be present, to maximize diagnostic possibilities. CRP with daily PPI use is reasonable. If the patient had no cardiac or GI
or sedimentation rate may aid in diagnosis of a septic joint but are not risk factors, use of NSAID alone would be appropriate. Colchicine is
helpful in distinguishing the actual causative agent. Uric acid level has an alternative therapy, although given its side effect profile, another
limited utility in this setting. Lyme disease is less likely in this patient NSAID is generally preferred. Allopurinol and febuxostat are medica-
who also has complaints of vaginal discharge, but this may be consid tions that may be used to prevent gout attacks but should generally not
ered in endemic areas with known or possible tick exposure. be started during an acute flair and are not typically initiated in the ED
setting.
Tendinopathy and Bursitis 103
KEY CONCEPTS
Tendinopathy • O
perative treatment may be indicated for selected cases of tendon injury that
• Mechanical overload and repetitive microtrauma are key underlying mecha- require primary repair (e.g., rupture of the Achilles tendon) or that have failed
nisms in the development of tendinopathy. Patients most often present with to respond to conservative treatment (e.g., rotator cuff tendinopathy).
a history of progressively worsening localized pain after repetitive work-or
sports-related activities. Bursitis
• Tendinopathy may also be associated with nonmechanical causes, including • Consider the possibility of an infectious cause in all cases of acute bursitis.
systemic manifestations of diseases, infectious etiologies, and the use of flu- • The definitive diagnosis of bursitis is made by aspiration of the bursa and
oroquinolones or statins. evaluation of the fluid.
• Most patients with tendinopathy can initially be treated with conservative • Septic bursitis is most commonly caused by Staphylococcus aureus.
measures, such as activity modification/protection, icing, medications (e.g., • Nonseptic bursitis may be traumatic, rheumatologic (e.g., gout and pseudo-
short-term use of nonsteroidal antiinflammatory drugs [NSAIDs] or nitroglyc- gout), or idiopathic in nature. It is prudent to consider other conditions, such
erin patches for certain tendinopathies), bracing/splinting, ergonomic modifi- as septic arthritis, osteomyelitis, or an underlying fracture, in the differential
cations, and graduated exercises. diagnosis of bursitis.
• Overuse syndromes can take at least 6 to 12 weeks to heal. Advise patients • The management of bursitis includes treatment with appropriate medication
accordingly and provide appropriate referral for follow-up to a musculoskel- (antibiotics for septic bursitis, NSAIDs for nonseptic bursitis), rest, application
etal specialist (e.g., a sports medicine, orthopedic, or physical medicine and of ice, compression, elevation, and prompt referral for appropriate follow-up.
rehabilitation specialist). Hospitalization should be considered for severe local infections, for patients
• Emergent imaging may be indicated in the ED when fracture or a condition who are immunosuppressed, and in the presence of systemic toxicity.
such as calcific tendinopathy is suspected. The use of point-of-care ultrasound
to evaluate tendinopathy can help to identify tendon disruption/rupture.
1. What is the etiology of most tendinopathies? Answer: B. This patient meets the diagnostic criteria for an Achilles
a. Direct blow to the tendon tendon rupture. Although MRI can be a useful adjunct when evaluat-
b. Recent or previous complete tendon rupture ing tendon injuries, obtaining such imaging in the acute care setting is
c. High-grade inflammation of the tendon not essential when the clinical diagnosis is obvious. The same can be
d. Mechanical overload and repetitive microtrauma stated for point-of-care ultrasound. Treatment consists of splinting the
Answer: D. Tendinopathies are thought to be precipitated by repetitive leg in 20° of equinus (plantar flexion) and close follow-up with a mus-
microtrauma and mechanical overload rather than true inflammation culoskeletal specialist (e.g., sports medicine or orthopaedist) for either
of the tendon operative or nonoperative treatment. Implementing non-weightbearing
2. Which of the following interventions should routinely be used status without proper splinting is insufficient in most cases.
when treating patients with tendinopathies? 4. What is the most important predisposing factor in septic bursitis?
a. Activity modification a. Chronic obstructive pulmonary disease
b. NSAIDs b. Diabetes mellitus
c. Intratendinous corticosteroid injection c. Human immunodeficiency virus (HIV)
d. Immobilization of the affected tendon/joint d. Trauma
Answer: A. Patients with tendinopathy should be advised to modify Answer: D. Trauma is the most common predisposing factor in
their activities (including ergonomic workplace adjustments, when patients who develop septic bursitis.
pertinent) to avoid exacerbating pain and further tendon injury. Some 5. What is the causative organism in the majority of cases of septic
patients may benefit in the near term from a short course (e.g., one bursitis?
week) of NSAIDs, although this can result in impaired tendon healing a. Beta-hemolytic streptococci
in the long term. Thus, their use should be dependent on the circum- b. Pseudomonas aeruginosa
stances of presentation, such as the severity of pain or comorbid con- c. Staphylococcus aureus
ditions. Intratendinous corticosteroid injections and immobilization d. Prototheca wickerhamii
should not be routinely utilized as they may prove detrimental in the Answer: C. S. aureus is the most common cause of infectious bursitis.
long term. 6. A 42-year-old female without significant past medical history pres-
3. A 38-year-old male presents after feeling a “pop,” followed by pain ents with 4 days of edema, mild warmth, and mild erythema in the
to the back of his leg while playing basketball. You note a defect to area of the right olecranon bursa. She is afebrile and has minimal
his Achilles tendon, decreased resting plantar flexion of the ankle, tenderness to palpation of the bursa. Aspiration of the bursa reveals
and a positive Thompson test on examination. What should be the a white blood cell count of 1500/μL.3 What is the most appropriate
next step in the patient’s management? next step in the patient’s management?
a. Provide crutches and advise the patient to remain non- a. Admit for intravenous (IV) antibiotics.
weightbearing until follow-up. b. Apply a compression dressing and give ibuprofen.
b. Splint the affected leg in an equinus position and refer for prompt c. Discharge the patient to home with oral antibiotics.
follow-up. d. Obtain magnetic resonance imaging (MRI) for further evalua-
c. Obtain emergent magnetic resonance imaging (MRI). tion.
d. Perform a steroid injection to the affected tendon. Answer: B. The clinical presentation and results of the fluid analysis
highly suggest a nonseptic bursitis. Cases of nonseptic bursitis can be
managed with a compressive dressing, nonsteroidal antiinflammatory
drugs (NSAIDs), avoidance of local trauma, and close follow-up.
104 Musculoskeletal Back Pain
KEY CONCEPTS
• M ost patients presenting to the emergency department (ED) with back pain
have uncomplicated musculoskeletal pain that is self-resolving with con-
servative therapy and does not require imaging.
• Indications for emergent imaging include “red flags” such as an acute neu-
rologic deficit, bowel or bladder dysfunction, or saddle anesthesia.
• Risk factors for compressive myelopathy include immunocompromised
patients with a history of malignancy, injection drug use, fever, chronic
steroid or anticoagulant use.
• Back pain due to metastatic disease is more common than primary tumors
in the spine, and thoracic metastases are more common than lumbar
metastases.
• Epidural abscess or hematoma, cauda equina syndrome (CES), spinal malig-
nancy with compressive symptoms, and spinal osteomyelitis are all indi-
cations for emergent surgical consultation or transfer to a center where
surgical spine consultation is available.
• Empirical parenteral antibiotics active against staphylococci, streptococci,
and gram-negative bacilli should be administered for suspected epidural
abscess. Specific antibiotics should be directed against the known patho-
gen if the culture or Gram stain of the aspirate is positive.
• Corticosteroids given as a single dose in the ED (10 mg dexamethasone)
or as a 15-day tapering course after discharge (prednisone 60 mg, 40 mg,
20 mg daily for 5 days each) may improve functional ability but does not
improve nerve root pain secondary to disc herniation.
1. A 55-year-old man presents with low back pain for one month. The 3. To avoid injuring the adult spinal cord, at what level should a lum-
pain is worse at night and is associated with a 10-pound weight loss. bar puncture be performed?
He denies any radicular symptoms. Which of the following is the a. L1 to L2
most likely cause of the back pain? b. L3 to L4
a. Discitis c. L5 to S1
b. Muscle spasm d. S2 to S3
c. Osteosarcoma e. Coccyx
d. Prostate cancer Answer: B. The spinal cord ends at around L2 in adults, lower in chil-
e. Sciatica dren. When performing a lumbar puncture, to avoid injuring the spinal
Answer: D. The patient’s subacute time course of back pain and worri- cord, insert the needle below the conus medullaris at level L3 to L4.
some finding of weight loss suggest a malignancy. Primary and meta- Remember that between individuals, there may be significant anatomic
static bone neoplasms can cause back pain from tumor infiltration into variance.
the bone. Primary bone tumors, such as multiple myeloma, chordoma, 4. A 55-year-old man complains of low back pain when walking
Ewing sarcoma, and osteosarcoma, are 25 times less frequent than met- downhill that is relieved with walking uphill. His neurovascular
astatic disease. Of the neoplasms, breast, lung, prostate, thyroid, lym- examination is unremarkable except for decreased bilateral Achilles
phoma, and kidney are the most likely to metastasize to bone. reflexes. In addition to analgesia, what is the appropriate manage-
2. Which one of the following findings, in conjunction with low back ment of this patient?
pain, indicates a benign cause of pain? a. Bedrest
a. Age of six years b. Emergent spine surgery consultation
b. Fever and night sweats c. Lumbosacral plain radiographs
c. Negative sitting but positive supine straight leg raise test d. Magnetic resonance imaging
d. Post-void residual of 500 mL e. Surgical referral for pseudoclaudication
e. Saddle anesthesia Answer: E. This patient presents with typical complaints of spinal ste-
Answer: C. A positive supine SLR test but a negative sitting SLR test nosis. Patients with spinal stenosis should be managed conservatively
suggests a nonphysiologic cause for the pain. Low back pain and with pain medications. In the absence of alarming red flag findings,
fever suggest an epidural abscess or osteomyelitis. Saddle anesthesia these patients do not require emergent laboratory or radiographic
and post-void residual greater than 100 mL are indicative of cauda studies. These patients may be candidates for surgery if they show any
equina syndrome (CES). Children complaining of back pain must be of the following conditions: progressive neurologic deficit, progressive
investigated. They may have spondylolysis with varying degrees of reduction in ability to walk secondary to pseudoclaudication, evidence
spondylolisthesis, Scheuermann disease (kyphosis and osteochondritis of cauda equina syndrome (CES), or intractable pain.
of the vertebral end plates), infectious diseases, or neoplastic etiologies.
5. Which of the following findings is most consistent with cauda gentle stretching. It is very common for a patient with lumbar radicu-
equina syndrome (CES)? lopathy to have no clear motor or sensory deficit but have exacerbation
a. Absent patellar reflexes of leg pain with SLR testing. The SLR has high sensitivity but low spec-
b. Bilateral buttock pain ificity. In contrast, the CSLR test has high specificity but low sensitivity.
c. Decreased rectal tone Given this, it is common for the patient with lumbar disk herniation to
d. Saddle anesthesia have a positive SLR but negative CSLR. The reverse is very uncommon.
e. Urinary retention Diagnoses (such as spinal epidural abscess and spinal malignancy)
Answer: E. The most consistent examination finding in CES is urinary usually present with prominent low back pain that is more significant
retention. With a high sensitivity of 90%, the patient is unlikely to have than extremity pain.
this disease process if his or her post-void residual urine volume is less 8. A 68-year-old man presents with a 5-week history of worsening low
than 100 to 200 mL. Saddle anesthesia (sensory deficit over the but- back pain. He reports mostly midline spinal pain with occasional
tocks, upper posterior thighs, and perineal area) is frequently an asso- radiation into both lower extremities. Two weeks before the onset
ciated finding, with a sensitivity of 75%. In 60% to 80% of cases, the of his pain, he was discharged from the hospital after an inpatient
rectal examination reveals a decreased sphincter tone. admission for pneumonia. On examination, he has intact lower
6. A 42-year-old man presents to the emergency department (ED) extremity motor and sensory function but tenderness to percussion
with a four-day history of low back pain after an episode of heavy over the lumbar spine. What diagnostic evaluation will confirm the
lifting at work. He reports bilateral low back pain at the level of the diagnosis?
iliac crests. He denies sensory or motor symptoms. He also denies a. Complete blood count
bowel or bladder dysfunction. His neurologic examination is nor- b. Contralateral straight leg raise test
mal. What management is indicated? c. Erythrocyte sedimentation rate (ESR)
a. Erythrocyte sedimentation rate (ESR) d. Lumbar computed tomography (CT)
b. Lumbar spine MRI e. Lumbar magnetic resonance imaging (MRI)
c. NSAIDs and gentle stretching Answer: E. The patient’s history is suspicious for spinal epidural
d. Oxycodone for three weeks abscess. He is at higher risk because of his age and recent infection.
e. Strict bedrest for two weeks In addition, the patient has tenderness with percussion of his spine.
Answer: C. The patient most likely suffers from uncomplicated muscu- The optimal diagnostic evaluation to confirm this diagnosis is MRI.
loskeletal low back pain. This is also commonly called acute lumbosa- Patients with epidural abscess usually have an ESR elevated above 20
cral strain. Most patients with this injury should not be placed on bed mm/hr. However, it is not uncommon for them to have a normal or
rest and should be allowed to return to normal activity, possibly with only mildly elevated WBC count.
some restrictions. The patient has a relatively short history of low back 9. A 63-year-old man presents with a 9-month history of progressive
pain with clear onset around an episode of lifting. Given a lack of con- low back pain with ambulation. He reports significant pain in his
cerning historical or examination findings, the patient does not require buttocks and posterior thighs when he walks distances greater than
imaging at this time. Blood work would not be of help in evaluating 25 meters. He says the pain is partially relieved when he flexes for-
the patient, because he lacks history or examination findings consistent ward and completely relieved by recumbency. He reports the pain
with spinal infection. is not relieved if he stops walking but remains standing. On neu-
7. A 35-year-old woman presents with a 3-day history of severe right rologic examination, he has intact lower extremity strength but
lower extremity pain associated with mild low back pain. Her neu- diminished Achilles reflexes bilaterally. What is the expected find-
rologic examination is normal except for a positive straight leg raise ing on diagnostic imaging?
(SLR) test on the right and a negative contralateral straight leg raise a. Diminished Doppler flow in the tibial arteries
(CSLR) test on the left. What therapy is indicated? b. Lumbar disc herniation at L5 to S1
a. Anxiolytic medication c. Normal lumbar MRI
b. High-dose corticosteroids d. Post-void bladder volume of 500 mL
c. NSAIDs and gentle stretching e. Spinal stenosis at L4 through S1
d. Oxycodone for three weeks Answer: E. The patient presents with classic findings of spinal stenosis
e. Strict bedrest for two weeks and neurogenic claudication or “pseudoclaudication,” including relief
Answer: C. Patients with herniated lumbar disks often present with with flexing forward and recumbency. Persistence of pain with stand-
radicular leg pain that overshadows the complaint of back pain. The ing despite having stopped ambulating is also indicative of neurogenic
treatment of radicular pain syndromes without neurologic deficit is claudication, as are diminished Achilles reflexes. Diminished pulses
similar to that of nonspecific uncomplicated back pain: NSAIDs and are indicative of vascular claudication. Pain from vascular claudication
is generally relieved if a patient stops walking but remains standing.
Unilateral disk herniation does not usually present with bilateral lower
extremity symptoms.
Systemic Lupus Erythematosus
and the Vasculitides 105
KEY CONCEPTS 3. Which of the following symptoms is least likely to be seen in a
patient presenting with giant cell arteritis?
• S ystemic lupus erythematosus (SLE) is a chronic autoimmune disorder that a. Amaurosis fugax
can affect any organ system. A general approach to determine whether b. Constitutional symptoms
SLE is the cause of a nonspecific or single-organ symptom is to search for c. Dysphagia
evidence of other organ involvement or systemic inflammation, which is d. Jaw claudication
expected in SLE-mediated presentations. e. Scalp tenderness
• Patients with SLE are at significantly higher risk of coronary artery or throm- Answer: c. Patients presenting with giant cell arteritis (GCA) com-
boembolic disease, which should prompt more thorough evaluation for monly have constitutional symptoms. Other presenting symptoms
these etiologies even in otherwise low-risk patients, such as young women. include headache and scalp tenderness. Jaw claudication is relatively
• SLE itself, as well as its treatment, may lead to immunosuppression; thus, it is specific for GCA, and visual symptoms (e.g., amaurosis fugax) may
important to remain vigilant for the possibility of infection in patients with SLE. herald impending blindness from the disease if not treated. Dysphagia
• Glucocorticoids are the mainstay for the initial management of the major- is not a presenting feature of GCA.
ity of conditions that are associated with increased SLE disease activity, 4. A patient with a history of SLE presents with acute bilateral arm weak-
including musculoskeletal, cutaneous, renal, pleural, or pericardial disease. ness and decreased sensation below the clavicles starting 90 minutes
• Antiphospholipid antibody syndrome (APS) is common in patients with SLE prior to arrival. A CT of the head is normal, and an MRI of the brain and
and carries with it an increased risk of venous and arterial thromboembolic spine reveals T2 hyperintensity in the cervical spine. Which of the fol-
disease. lowing is the most appropriate next step in management for this patient?
• Rheumatology input may be helpful in diagnostic, management, or disposi- a. Intravenous ceftriaxone and vancomycin
tion decisions for select patients with SLE. b. Intravenous lorazepam
c. Intravenous methotrexate
1. Which of the following findings would decrease the likelihood that d. Intravenous methylprednisolone
a patient’s pleurisy is related to a lupus flare? e. Intravenous tissue plasminogen activator
a. Elevated erythrocyte sedimentation rate Answer: d. This patient is presenting with transverse myelitis. The
b. Low complement levels first line of treatment for this condition is high-dose glucocorticoids,
c. Negative review of systems such as methylprednisolone, and since this patient has a history of
d. Polyarthritis SLE, cyclophosphamide. Refractory cases may be treated with plasma
e. PR depression on electrocardiogram exchange and additional immunosuppressive agents.
Answer: c. The general approach to assessing whether a symptom is 5. A patient with a history of SLE presents with shortness of breath.
caused by SLE disease activity is to look for signs and symptoms of the CT angiography reveals two segmental pulmonary emboli. Labora-
disease in other organs or systemically. Patients experiencing an SLE tory testing and bedside echocardiography are unremarkable. Chart
flare are likely to have other signs or symptoms of disease (e.g., arthral- review reveals positive anticardiolipin and β2-glycoprotein I titers.
gias, fever, malaise, elevated ESR, low complement levels) beyond an Which of the following is the most appropriate next step in therapy
isolated chief complaint (in this case, pleurisy). PR depression can be for this patient?
suggestive of pericarditis, which may occur during flares of SLE. a. Aspirin
2. A patient with a history of SLE presents actively seizing. In addition b. Enoxaparin
to respiratory support, what is the most appropriate initial step in c. Rivaroxaban
management? d. Tissue plasminogen activator
a. Intravenous ceftriaxone and vancomycin e. Warfarin
b. Intravenous cyclophosphamide Answer: b. This patient has SLE and antiphospholipid syndrome (APS)
c. Intravenous lorazepam based on the history of thromboembolic disease and positive antiphos-
d. Intravenous methylprednisolone pholipid (aPL) antibodies. The most appropriate anticoagulation for
e. Intravenous tissue plasminogen activator patients with APS is a vitamin K antagonist (e.g., warfarin) or a hepa-
Answer: c. In nearly all cases (including seizure), the initial resus- rin analog (e.g., enoxaparin). In the acute setting, however, a heparin
citation and stabilization of the patient with SLE are the same as for analog should be used due to the delayed therapeutic effect of vitamin
patients without SLE. While this patient could have meningitis, lupus K antagonists. Aspirin is insufficient to treat acute thromboembolic
cerebritis, or a stroke, which could warrant some of the other treat- disease in APS, and direct oral anticoagulants have decreased efficacy
ments listed, the patient should be stabilized by treatment of the sei- when compared to vitamin K antagonists and heparin analogs. This
zure first. patient does not have an indication for tissue plasminogen activator.
106 Allergy, Anaphylaxis, and Angioedema
1. Which of the following statements regarding tinea capitis is TRUE? Answer: c. Clindamycin or trimethoprim-sulfamethoxazole are recom-
a. It is contagious. mended first-line treatment choices for suspected methicillin-resistant
b. It is not transmitted by household pets. Streptococcus aureus (MRSA) infection. Macrolides and penicillins are
c. Tinea capitis presents with alopecia with normal underlying often ineffective against MRSA. Linezolid, although effective, is expen-
scalp. sive and is generally not recommended as a first-line treatment.
d. Topical treatment is effective. 3. What is the causative organism of erythema migrans?
e. Treatment should be instituted for 1 to 2 weeks. a. Borrelia burgdorferi
Answer: a. Tinea capitis is the dermatophytosis that is contagious. Sys- b. Group A Streptococcus
temic treatment for 4 to 6 weeks is the minimum. It may be transmitted c. Methicillin-resistant Staphylococcus aureus
by pets. The underlying scalp is typically inflamed. d. Neisseria meningitides
2. A 16-year-old boy presents with cellulitis of his left forearm. What e. Parvovirus B-19
is the appropriate initial antibiotic? Answer: a. Borrelia burgdorferi is the causative agent of erythema
a. Azithromycin migrans, or Lyme disease. Treatment should be instituted with dox-
b. Ceftriaxone ycycline for 10 to 21 days, or as alternates, cefuroxime, ceftriaxone,
c. Clindamycin or penicillin G. Group A Streptococcus is the causative organism of
d. Linezolid scarlet fever. Neisseria meningitides is the causative agent of Menin-
e. Penicillin VK gococcemia. Parvovirus B-19 is the causative agent of erythema
infectiosum.
4. A 26-year-old man presents with an erythematous maculopapular Answer: c. Women are affected primarily. The lesions have a predilection
eruption of his torso, palms, and soles. He had a painless lesion on for distal joint skin. The lesions are often multiple and have a predilection
his penis 1 month earlier. What is the treatment of choice? for periarticular regions of the distal extremities. The lesions begin as
a. Azithromycin erythematous or hemorrhagic papules that evolve into pustules and vesi-
b. Benzathine penicillin G cles with an erythematous halo. They may be tender and may have a gray
c. Ceftriaxone necrotic or hemorrhagic center. The organism may be cultured from the
d. Doxycycline cutaneous lesions. Gram stain only occasionally reveals the organisms.
e. Trimethoprim-sulfamethoxazole 7. A 30-year-old man presents with headache, nausea and vomiting,
Answer: b. Secondary syphilis is treated with benzathine penicillin G myalgias, fever, and a petechial rash on his extremities and trunk.
in a dose of 2.4 million units IM. Lesions are clustered on the palms and soles. What is the best treat-
5. A 25-year-old female presents with fever, migratory polyarthral- ment?
gias, and hemorrhagic papules on her fingers and wrists. What is a. Cephalexin.
the best treatment? b. Doxycycline.
a. Ceftriaxone c. Erythromycin.
b. Ciprofloxacin d. Penicillin VK.
c. Doxycycline e. Trimethoprim-sulfamethoxazole.
d. Ofloxacin Answer: b. Patients with Rocky Mountain spotted fever present
e. Vancomycin with headache, nausea and vomiting, myalgias, chills, and fever.
Answer: a. Treatment of disseminated gonococcal infection is with The disease may last 3 weeks and may be severe with prominent
parenteral ceftriaxone, or ceftizoxime or cefotaxime. Patients allergic involvement of the central nervous system, cardiac, pulmonary, gas-
to β-lactam antibiotics or those with severe penicillin allergies may be trointestinal and renal systems, disseminated intravascular coagu-
treated with spectinomycin. Ciprofloxacin and ofloxacin are not rec- lation, or shock. The rash begins with erythematous macules that
ommended because of increasing resistance patterns. Hospitalization blanch on pressure, appearing first on the wrists and ankles. These
is recommended for patients with disseminated gonococcal infection. macules spread up the extremities and to the trunk and face. They
6. Which of the following statements regarding gonococcal dermatitis may become petechial or hemorrhagic. Lesions on the palms and
is TRUE? soles are particularly characteristic. Doxycycline is the antibiotic of
a. Gonococci can usually be seen on gram stain from the lesions. choice. Chloramphenicol may be used for patients allergic to tet-
b. It affects primarily men. racyclines and in children younger than 9 years of age. Sulfa drugs
c. It occurs in 1% or 2% of patients with gonorrhea. should be avoided because they may exacerbate the illness. Rickett-
d. The lesions have a predilection for the knees and elbows. siae are routinely resistant to penicillins, cephalosporins, aminogly-
e. The skin lesions are not tender. cosides, and erythromycin.
108 Blood and Blood Components
108
KEY CONCEPTS
• R ed blood cell transfusion is indicated only to increase oxygen delivery at
the tissue level.
• One unit of packed red blood cells (PRBCs) can be expected to raise an
adult’s hemoglobin level by 1 g/dL. A similar increase is expected in chil-
dren following the transfusion of 10 mL/kg of PRBCs.
• Controlled trials have supported newer, restrictive, red cell transfusion
strategies. Pending further trials, a transfusion trigger of a hemoglobin
level below 7 to 8 g/dL is appropriate for most stable hospitalized patients.
• Platelet transfusions are typically used prophylactically for counts less than
10 K/μL in adults without bleeding. For patients undergoing central venous
catheter placement, a level of 20 K/μL is recommended. Patients undergo-
ing lumbar puncture and non-neuroaxial surgery should be prophylactically
transfused to a level of 50 K/μL.
• Prospective and retrospective reports have suggested a benefit to massive
transfusion protocols, with most advocating a 1 : 1 : 1 ratio of fresh frozen
plasma (FFP) to platelets to PRBCs.
• When available, low-titer whole blood is safe and effective for transfusion
and provides a physiologic mix of blood products.
• When available, prothrombin complex concentrate (PCC) should be given
over FFP for reversal of vitamin K antagonism in the setting of a life-
threatening bleeding. When PCC is not available, FFP can be used for this
purpose, but is considered a second-line therapy.
• Transfusion reactions can vary from minor symptoms to fatal systemic reac-
tions. If any transfusion reaction is suspected, the transfusion should be
stopped while the cause and extent of the reaction is investigated.
• An intravascular hemolytic transfusion reaction is usually the result of
ABO incompatibility and typically results in immediate symptoms that
can include fever, chills, headache, nausea, vomiting, sensation of chest
restriction, severe joint or low back pain, burning sensation at the site of
the infusion, and feeling of impending doom. Treatment involves stopping
the transfusion, fluid resuscitation, and monitoring for the development of
renal failure and disseminated intravascular coagulation (DIC).
• Transfusion-related acute lung injury (TRALI) is now the leading cause of
reported transfusion-related mortality. Treatment involves stopping the
transfusion and providing supportive respiratory care, which may include
noninvasive positive- pressure ventilation (NIPPV) or intubation and
mechanical ventilation.
• Improved techniques for selecting and testing blood donors has dramati-
cally reduced the risk of viral transmission of disease by transfusion.
KEY CONCEPTS
• C hronic lymphocytic leukemia (CLL) is the most common leukemia in the
elderly, and acute lymphocytic leukemia (ALL) is the most common leuke-
mia in children.
• Splenomegaly is a common finding in leukemias.
• Leukostasis is usually not accompanied by clinical sequelae until the white
blood cell (WBC) count is more than 500,000.
• Neutropenia plus a fever should be treated as a potentially life-threatening
infection until proven otherwise.
• Infection is the most common cause of neutropenia in children.
• WBC count determination in the emergency department (ED) has poor sen-
sitivity and specificity for any specific disease process.
• Inflammatory markers, absolute neutrophil count, bandemia, or immature
granulocyte count may be more useful for identifying infection than the abso-
lute WBC count.
1. Which of the following statements best describes the use of the d. The classic physical findings associated with infection will be
WBC count in emergency medicine decision making? obvious owing to the inflammatory response.
a. The absence of leukocytosis excludes the presence of significant e. The review of systems focuses on bleeding problems, generalized
disease. symptoms such as fatigue, sweats, and weight loss, or autoim-
b. The absolute neutrophil count, presence of bandemia, and mune symptoms.
immature granulocyte count may be more helpful than the total Answer: D. The classic physical signs of infection may be blunted in
WBC count in identifying bacterial infection. severe neutropenia, because the inflammatory or purulent response
c. The total WBC count is both specific and sensitive for serious may be hampered by the limited numbers of neutrophils.
bacterial infection. 4. A patient with a known history of chronic myeloid leukemia (CML)
d. The WBC count is highly discerning in screening for illness in presents with shortness of breath. The WBC count is 25,000 cells/
the acute care setting. mm³. Which of the following conditions is LEAST likely to be the
Answer: B. The WBC count and accompanying differential count are cause of the patient’s dyspnea:
among the most common laboratory tests ordered in the ED. The WBC a. Angina
count has not proved to be a highly sensitive or specific test, and the b. Heart failure
absence of leukocytosis does not exclude the presence of significant c. Hyperleukocytosis resulting in pulmonary ventilation-perfusion
disease. The WBC count has limited screening value in the acute care abnormalities
setting. The absolute neutrophil count or an elevated band count may d. Renal failure and fluid overload
be more helpful than the total WBC count in identifying bacterial e. Severe anemia
infection. The immature granulocyte count can be used to help identify Answer: C. Hyperleukocytosis can occur in CML, though the compar-
patients with or without sepsis. atively more mature, “less sticky” cells found in CML rarely cause prob-
2. Which of the following can affect the “normal” WBC count? lems unless the count exceeds 500,000 cells/mm3. A higher cell count
a. Age and ethnicity may cause leukostasis and result in deafness, visual impairment, pul-
b. Exercise monary ventilation-perfusion abnormalities, or priapism. Treatment
c. Gender involves hydration, leukapheresis, transfusion as necessary, allopuri-
d. Tobacco use nol to prevent severe hyperuricemia, and specific chemotherapy. The
e. All of the above need for urgent therapy in CML is usually related to hyperuricemia and
Answer: E. All of the statements are true. One unique problem in WBC renal injury, or severe anemia and subsequent angina or heart failure.
disorders is the wide variability in normal values and the multiple fac- 5. When applied to lymphocytic neoplasms, the terms acute and
tors influencing them. Normal values for the WBC count are listed in chronic describe all of the following except:
Table 110.1. The “normal” WBC count range is age-dependent and may a. Aggressiveness of therapy required in treatment
be shifted upward by exercise, female gender, smoking, or pregnancy. b. Cell maturity
Decreases in the total WBC count range by 1000 to 1200 cells/mm3 c. Patient survival time
have been noted in the Black population. d. Rapidity of onset
3. In evaluating a patient with severe neutropenia, which of the fol- Answer: C. Historically, acute and chronic were descriptive terms
lowing is false? applied to lymphocytic neoplasms with respect to patient survival
a. If the patient is febrile, basic isolation techniques and specific time. The terms acute and chronic are currently utilized to describe cell
therapies should be initiated after cultures are completed. maturity, rapidity of onset, and aggressiveness of therapy required in
b. If the patient is febrile, urine and blood cultures should be treatment. Chronic lymphocytic leukemia (CLL) is primarily a B-cell
obtained. disorder and is the most common form of leukemia in patients 50
c. The clinician should ask about medications, history of neutrope- years or older. Acute lymphocytic leukemia (ALL) is most commonly
nia, family history, and also review information regarding recent diagnosed in children younger than 10 years and is the most frequent
infections. malignant neoplasm in children younger than 15 years of age.
111 Disorders of Hemostasis
KEY CONCEPTS
• A lthough hemostatic disorders are confirmed through laboratory testing, a
careful history and thorough physical examination may provide clues to the
diagnosis.
• The use of antithrombotic agents remains widespread, whereas diseases
such as hemophilia or disseminated intravascular coagulation (DIC) are
encountered infrequently in the emergency department (ED) setting.
• Critical thrombocytopenia increases the risk of bleeding, particularly with
trauma or invasive procedures. Platelet dysfunction may occur with platelet
levels outside of or within the normal range. Aspirin therapy or renal dis-
ease, for instance, can cause platelet dysfunction in the setting of a normal
platelet count.
• Patients with suspected new diagnosis of immune thrombocytopenic pur-
pura (ITP) should typically be admitted for further management; glucocorti-
coid therapy is the mainstay of treatment.
• In patients with possible heparin-induced thrombocytopenia (HIT), clinical
scoring systems are helpful in risk stratifying the possibility of HIT, prompt-
ing early cessation of heparin, alternative anticoagulation, and hematology
consultation. Spontaneous HIT should be considered in patients following
major surgery (typically an orthopedic procedure), or with recent serious
infection.
• Thrombotic thrombocytopenic purpura (TTP) should be suspected if both
thrombocytopenia and microangiopathic hemolytic anemia (MAHA) are
identified. Early treatment includes plasma exchange therapy.
• Platelet transfusion is rarely indicated unless platelet counts are below
10,000/mm3 or severe life-threatening bleeding occurs. Platelet transfusion
should be avoided in the setting of thrombotic microangiopathies, such as
TTP, hemolytic uremic syndrome (HUS), the “hemolysis, elevated liver func-
tion tests, and low platelets” (HELLP) syndrome complicating pregnancy, or
HIT.
• Hemophiliacs are often highly informed about their disease. It is impera-
tive that prompt intervention with replacement therapy occurs early when
bleeding or the potential for bleeding is suspected. As a general rule, 1 U/
kg of factor VIII will increase the circulating factor VIII level by 2%.
• Hemophilia with inhibitors creates a challenge for emergency resuscita-
tion, and a treatment option is recombinant Factor VIIa.
1. What platelet count is generally recommended prior to perfor- Answer: A. Early identification of TTP is imperative. This patient pres-
mance of a lumbar puncture? ents with classic signs and symptoms (neurologic symptoms, fatigue,
a. 100,000 mm3 thrombocytopenia, and microangiopathic hemolytic anemia). A val-
b. 50,000 mm3 idated score such as the PLASMIC score should be used to rapidly
c. 40,000 mm3 assess the likelihood of TTP, as early plasma exchange is the treatment
d. 20,000 mm3 of choice. A platelet transfusion will more than likely exacerbate symp-
e. 10,000 mm3 toms. Tranexamic acid prevents fibrinolysis, and therefore would likely
Answer: B. A platelet count above 50,000 mm3 is recommended prior cause harm in the setting of microthrombi in TTP. IVIG has not been
to performance of invasive spinal procedures such as lumbar procedure. shown to be the front-line treatment for TTP. Heparin will further
A platelet count above 20,000 mm3 is recommended for central line increase this patient’s bleeding risk. The nonspecific T wave changes
placement. A platelet count above 100,000 mm3 is recommended prior are likely due to TTP.
to neurosurgical procedures or invasive ophthalmologic interventions 5. A 22-year-old patient with hemophilia A presents to the ED with a
2. An isolated elevation in prothrombin time (PT) or INR indicates an headache. He has a history of migraines but this feels different from
abnormality or deficiency in which of the following? his previous headaches. He denies any recent significant closed
a. Calcium head injury, though states he fell while skateboarding 1 week prior.
b. Factor VII He is concerned about the possibility of bleeding. Neurologic exam-
c. Factor XI ination is unremarkable. Upon further review of available records,
d. Fibrinogen the patient has a history of severe hemophilia A with inhibitors. His
e. Platelets headache is worsening. What would be the best next step in man-
Answer: B. The PT or INR tests the components of the extrinsic path- agement?
way, and abnormalities or deficiency in factor VII will cause elevation a. Obtain imaging of brain and cervical spine.
in PT and INR. Calcium derangements do not typically cause aber- b. Consult neurosurgery for possible intracranial bleed.
rations in PT, INR, or PTT. Factor XI is a component of the intrinsic c. Administer factor VII as expeditiously as possible.
pathway reflected in the PTT. Fibrinogen function is reflected in both d. Administer factor VIII as expeditiously as possible.
prothrombin time (PT) and PTT values. Platelet deficiencies do not e. Administer a migraine “cocktail” for his headache and reevalu-
prolong the PT or INR. ate in 30 minutes.
3. If initial treatment of ITP with glucocorticoids fails, which of the Answer: C. Patients with known hemophilia should be treated empir-
following is most reasonable additional treatment for severe throm- ically with factor replacement when intracranial bleeding is first sus-
bocytopenia? pected. An atypical headache, particularly in the setting of trauma (even
a. Splenectomy seemingly minor or incidental trauma), should be treated with factor
b. Four factor-PCC replacement as expeditiously as possible. Neurosurgery may need to be
c. IVIG contacted, though consultation should not delay factor administration.
d. Tranexamic acid Although this patient has hemophilia A (factor VIII deficiency), the
e. Thrombopoietin receptor agonist presence of inhibitors (antibodies toward factor VIII) limit the utility
Answer: C. IVIG has been shown to improve platelet counts in patients of factor VIII replacement. In the setting of hemophilia with known
with severe thrombocytopenia secondary to ITP. Splenectomy is con- inhibitors and bleeding, recombinant factor VIIa is recommended.
sidered the last line of treatment. Tranexamic acid prevents fibrinolysis. Medications for treatment of routine migrainous symptoms may be
Thrombopoietin receptor agonists are typically reserved for patients considered secondary to exclusion of life-or disability-threatening
who are not responsive to glucocorticoids and IVIG. intracranial hemorrhage.
4. A 52 year-old-female presents to the emergency department with
confusion, shortness of breath, and fatigue. Laboratory evaluation
reveals a platelet count of 14,000 mm3 and hemoglobin of 7.2. mg/
dL, a low MCV, indirect bilirubinemia, and a creatinine of 1.4 mg/
dL. Electrocardiogram shows non-specific T wave abnormalities in
the lateral leads. She denies chest pain and vitals are normal. What
would be the most reasonable next step in management?
a. Obtain a blood smear and utilize a validated risk score for possi-
ble TTP.
b. Immediate platelet transfusion
c. Administer tranexamic acid
d. Start IVIG
e. Administer heparin
112 Oncologic Emergencies
1. A patient presents to the emergency department with severe meta- Answer: B. Plasma is maintained as electrically neutral, meaning equal
bolic acidosis. In addition to initial stabilization, what should be a charges from plasma cations and anions. Sodium [Na+] is the major
priority for the clinician? cation routinely measured in clinical laboratories. Chloride [Cl−] and
a. Infuse sodium bicarbonate to normalize pH to approximately bicarbonate [HCO3−] are the major anions routinely measured in
7.40. clinical laboratories. The anion gap is calculated as: [Na+] − ([Cl−] +
b. Induce a respiratory alkalosis to normalize pH to approximately [HCO3−]). In normal physiological states, “unmeasured” anions are
7.40. more plentiful than “unmeasured” cations, and hence the anion gap
c. Identify the cause of metabolic acidosis and target therapy at that is positive (typically between about 9 mmol/L and 15 mmol/L). Met-
cause. abolic acidosis is defined by low [HCO3−]. A normal anion gap meta-
d. Infuse saline with the goal of decreasing the strong ion differ- bolic acidosis occurs when a decrease in [HCO3−] is accompanied by
ence. an increase of [Cl−] by the same amount to maintain electroneutrality.
Answer: C. The key steps to managing acid-base disorders are: (1) A wide anion gap metabolic acidosis occurs when a decrease is bicar-
identifying the type of acid-base disorder(s) present; (2) identifying the bonate is accompanied by an increase in anions other than chloride (so
underlying cause of the acid-base disorder; and (3) initiating therapy called “unmeasured anions”).
to reverse the underlying cause. Optimal therapy varies based on the 3. A patient in the emergency department has the following labo-
underlying cause. For example, in the case of metabolic acidosis, pri- ratory values. Select the term that best characterizes this patient’s
mary treatment for diabetic ketoacidosis is insulin, for lactic acidosis acid-base status. pH = 7.36; Paco2 = 36 mm Hg; Pao2 = 135 mm Hg;
from sepsis is increasing tissue perfusion with vasopressors and intra- Na = 142 mmol/L; Cl = 100 mmol/L; HCO3 = 21 mmol/L.
venous fluids, and for severe toxic alcohol exposure is a specific rever- a. Acidemia
sal agent. In some scenarios with severe metabolic acidosis, sodium b. Alkalemia
bicarbonate infusion is a reasonable adjunctive therapy, especially c. Respiratory alkalosis with metabolic compensation
those caused by loss of bicarbonate or renal failure, while the underly- d. Metabolic acidosis with respiratory compensation
ing cause is being addressed.
2. What causes an increase in the anion gap?
a. A decrease in plasma bicarbonate concentration accompanied
by an increase in plasma chloride concentration of the same
magnitude
b. An increase in the concentration of unmeasured anions in the
plasma
c. Saline infusion
d. Hypoventilation
Answer: D. The patient’s pH is in the normal range of 7.35 to 7.45. 5. A patient presents to the emergency department with short-
Therefore, neither acidemia nor alkalemia is present. However, the ness of breath. Plasma bicarbonate concentration is 44 mmol/L.
patient may have an acid-base disorder with compensation into a What is a potential explanation for this laboratory value?
normal pH range, or multiple primary acid-base disorders that when a. Administration of a large volume of saline
combined result in a pH in the normal range. To classify an acid-base b. Acute respiratory failure from a spontaneous pneumothorax
disorder, initially interpret the basic metabolic panel findings and then sustained 2 hours prior to presentation
the arterial blood gas findings. This patient’s bicarbonate is low at 21 c. Chronic respiratory failure from chronic obstructive pulmonary
mmol/L, indicating a metabolic acidosis. The anion gap is 21 mmol/L, disease
indicating a wide anion gap metabolic acidosis. According to the rule d. Diabetic ketoacidosis
of 15, respiratory compensation for this metabolic acidosis is expected Answer: C. A plasma bicarbonate concentration of 44 mmol/L indi-
to result in a Paco2 of 36 mm Hg and a pH of 7.36, consistent with the cates either a primary metabolic alkalosis or metabolic compensation
patient’s measured values. Looking at the ABG results, the pH is less for a respiratory acidosis. Metabolic compensation requires alteration
than 7.40, indicating a primary acidosis, and Paco2 is less than 40 mm of renal excretion of acid, which usually takes at least 2 days to develop.
Hg, indicating the respiratory findings are a compensatory effect rather Therefore, metabolic compensation is typically not present with acute
than a primary disorder. respiratory failure but is present with chronic respiratory failure.
4. How can calculation of an osmolar gap be used clinically? Administration of a large volume of saline increases plasma chloride
a. To rule out toxic alcohol ingestion concentration and decreases plasma bicarbonate concentration to
b. To assist with understanding potential causes of a wide anion maintain electroneutrality. Respiratory acidosis from a pneumothorax
gap metabolic acidosis sustained 2 hours previously would not be associated with metabolic
c. To differentiate primary lung pathology from respiratory muscle compensation because not enough time would have elapsed for the
weakness in a patient with respiratory acidosis metabolic compensatory mechanisms to have an effect. Respiratory
d. To understand if metabolic compensation has occurred after acidosis from chronic respiratory failure is associated with metabolic
respiratory alkalosis compensation and thus an elevated bicarbonate concentration. Dia-
Answer: B. Calculation of the osmolar gap is a method to help under- betic ketoacidosis results in accumulation of “unmeasured” anions
stand the cause of a wide anion gap metabolic acidosis. It can help (acetoacetic acid and beta-hydroxybutyric acid) and a decrease in
identify the presence of a large amount of osmotically active solutes plasma bicarbonate to maintain electroneutrality.
in the blood other than the primary osmotically active solutes that are
commonly measured (Na, counter anions to Na, glucose, urea, and
ethanol). An elevated osmolar gap occurs when measured osmolality
is higher than predicated based on a calculation accounting for the
commonly measured osmotically active solutes. An important cause
of wide anion gap metabolic acidosis and elevated osmolar gap is toxic
alcohol poisoning (e.g., methanol or ethylene glycol). Although an ele-
vated anion gap with an elevated osmolar gap can suggest toxic alcohol
poisoning in the right clinical setting, a normal osmolar gap does not
rule out toxic alcohol poisoning because the osmolar gap decreases as
alcohols are metabolized and calculation of the osmolar gap assumes
normal osmolality before the poisoning, which may not be true, espe-
cially in chronically ill patients.
KEY CONCEPTS
Hyperthyroidism Box 117.6). It usually occurs in patients with untreated or undertreated hypo-
• Hyperthyroidism induces a hypermetabolic state and increases β-adrenergic thyroidism. Treatment with thyroid hormone replacement must be initiated,
activity. The resulting clinical manifestations range from vague constitutional often based solely on clinical findings.
symptoms to more organ-specific symptoms (see Box 117.1).
• Hyperthyroidism in elders may be asymptomatic or may manifest with sub- Adrenal Excess States
tle nonspecific symptoms such as weight loss, shortness of breath, and/or • Adrenal excess states run the spectrum from Cushing syndrome, to primary
dementia. and secondary hyperadrenalism, to pheochromocytoma.
• The laboratory test of choice for suspected hyperthyroidism is the thyroid • Symptoms of adrenal excess will vary, depending on the etiology, with
stimulating hormone (TSH) concentration with free T4 and T3 levels. chronic, nonspecific symptoms that arise from Cushing syndrome (general-
• Thyroid storm is a life-threatening thyrotoxic crisis that often presents with ized weakness, fatigue, menstrual irregularities, and weight gain), to simple
fever, extreme tachycardia, and/or altered sensorium. It requires prompt rec- refractory hypertension with hyperaldosteronism, to acute, refractory hyper-
ognition and therapy, as well as identification and treatment of any precipi- tension and hyperadrenergic symptoms with pheochromocytoma.
tating cause, such as infection. • Diagnosis of hyperaldosteronism is typically clinical, and confirmed by labo-
• The order of medication administration in thyroid storm is critical. Iodine can ratory testing and imaging studies, depending on the etiology. In most cases,
precipitate thyroid storm and must be given a minimum of 1 hour after thion- acute stabilization of the presenting complaint is paramount, and definitive
amide therapy (PTU or methimazole). As such, the typical order is beta blocker diagnosis will occur outside the ED. Any adrenal incidentaloma discovered on
(propranolol), PTU or methimazole, and then iodine (SSKI, Lugol solution). imaging obtained in the ED, especially in the setting of hypertension, should
prompt further evaluation for adrenal excess.
Hypothyroidism
• Hypothyroidism results from lack of stimulation of the thyroid gland (central Adrenal Insufficiency
or secondary hypothyroidism) or intrinsic gland dysfunction limiting hormone • Clinical manifestations of secondary adrenal insufficiency are often vague
production (primary hypothyroidism). and nonspecific, including fatigue, weakness, dizziness, nausea, vomiting,
• Signs and symptoms of hypothyroidism range from asymptomatic to overt and other nonspecific GI symptoms. Patients with primary adrenal insuffi-
organ failure, which can lead to death (see Box 117.5). ciency characteristically have more pronounced clinical manifestations and
• Determination of an elevated TSH level is the most sensitive and single best skin hyperpigmentation.
screening test to confirm the diagnosis of primary hypothyroidism. • The ACTH stimulation test and measurements of cortisol levels is the most con-
• Replacement with levothyroxine (T4) remains the treatment of choice and venient method and is considered the criterion standard to make the diagnosis.
resolves physical and psychological signs and symptoms in most patients. • Refractory hypotension in the acutely ill patient may be the only clue to adre-
• Myxedema coma is a life-threatening event that presents with altered mental nal insufficiency and is readily treated with IV administration of glucocorti-
status and hypothermia, along with a concomitant precipitating event (see coids (hydrocortisone, 100 mg).
1 What is the most common presenting sign of thyrotoxicosis? Answer: B. Iodine can precipitate thyroid storm and must be given
a. Atrial fibrillation a minimum of 1 hour after thionamide therapy (PTU or methima-
b. Means-Lerman scratch zole). As such, the typical order is beta blocker (propranolol), PTU, or
c. Tachycardia methimazole, and then iodine (SSKI, Lugol solution). In addition, it is
d. Wide pulse pressure important to identify and treat the precipitating cause of thyroid storm.
Answer: C. The most common cardiac sign of thyrotoxicosis is 3. A 32-year-old female with a history of bipolar mood disorder on
tachycardia. Widened pulse pressure, a Means-Lerman scratch (a sys- lithium presents with asthenia, fatigue, cold intolerance, and dry
tolic friction rub), dilated cardiomyopathy, and pulmonary hyperten- skin. A TSH test confirms your suspected diagnosis. How should
sion are all cardiac findings in patients with thyrotoxicosis, with atrial you counsel the patient regarding her lithium use?
fibrillation being more common in patients over the age of 65. a. Instruct the patient to continue the lithium, no other interven-
2. In thyroid storm, which of the following is the proper sequence of tion is necessary.
drug administration? b. Instruct the patient to continue the lithium, while initiating
a. All three together (antithyroid drugs, sodium or potassium therapy with levothyroxine.
iodide, steroid) at the same time c. Instruct the patient to stop her lithium, while initiating therapy
b. Antithyroid drugs, sodium or potassium iodide, steroid with levothyroxine.
c. Sodium or potassium iodide, steroid, antithyroid drugs d. Instruct the patient to stop her lithium immediately and follow up
d. Steroid, sodium or potassium iodide, antithyroid drugs with her primary care physician for an alternative medication.
Answer: B This patient presents with symptoms of overt hypo- of acute illness has resulted in adrenal crisis. Additional fluids and
thyroidism, confirmed by a high TSH on laboratory testing. Instruct pressors will be ineffective until appropriate glucocorticoid treatment
the patient that she may continue her lithium for her chronic bipolar is initiated, hydrocortisone, 100 mg IV. ACTH administration is used
mood disorder, while initiating levothyroxine therapy. A well-known for evaluation of adrenal insufficiency and is not appropriate for thera-
cause of hypothyroidism, lithium causes overt hypothyroidism in 14% peutic administration during the acute crisis.
to 17% of patients, and subclinical hypothyroidism in 19% to 35% of 5. A 37-year-old male presents to the ED with complaints of palpi-
patients. Most of lithium’s effects on the thyroid is due to the inhibition tations, abdominal pain, nausea, vomiting, and diaphoresis. He
of T4 and T3 release. Lithium also increases thyroid autoimmunity if reports episodes similar to this that have been increasing in fre-
it is present before the initiation of lithium treatment. Treatment with quency and severity over several months. His vital signs are P 127,
exogenous thyroid hormone is effective, and lithium therapy need not BP 174/90, RR 28, T 37.1°C, and Sao2 96%. Once stabilized, what is
be discontinued. the most appropriate oral treatment for this patient?
4. A 52-year-old man is brought in by ambulance from his home with a. Amlodipine 5 mg PO daily
altered mental status and hypotension. EMS reports that he became b. Metoprolol, 100 mg PO BID
ill with an influenza-like illness 4 days ago and stopped taking all c. Phenoxybenzamine 10 mg PO BID
of his medications secondary to vomiting. Despite multiple fluid d. Propranolol ER 80 mg PO daily
boluses, he remains hypotensive. Laboratory results demonstrate Answer: C. This patient presents with episodic, or paroxysmal
severe acidosis, hyponatremia and hyperkalemia. What is the most symptoms that are consistent with pheochromocytoma. Following ini-
appropriate treatment for this patient? tial stabilization and appropriate workup, the patient will require con-
a. Adrenocorticotropic hormone (ACTH) trol of his vital sign abnormalities and symptoms leading up to surgery.
b. Continued normal saline (0.9%) fluid boluses The first-line treatment for pheochromocytoma is alpha-blockade, so
c. Hydrocortisone in this question, phenoxybenzamine is correct. In patients with cate-
d. Norepinephrine cholamine excess, beta blockade alone can result in hypertensive cri-
Answer: C. This patient presents with adrenal crisis, as evidenced sis because of unopposed alpha-adrenergic receptor stimulation. Beta
by refractory hypotension and electrolyte abnormalities typical for blockers can be used, but this should only be done following alpha-
hypoadrenalism. While refractory hypotension in the acutely ill blockade; as such, propranolol and metoprolol are incorrect. Calcium
patient may be the only clue to adrenal insufficiency, the paramedics channel blockers may be used in mild cases or in patients with unde-
also report that the patient has been unable to take his usual medica- sirable alpha-blockade side effects, but are not considered first line; as
tions, presumably for primary adrenal insufficiency, and in the setting such, amlodipine is also incorrect.
118 Bacteria
KEY CONCEPTS 3. After which of the following circumstances has tetanus been
118
reported?
• A ll septic patients should be treated with antibiotics as soon as possible, a. Childbirth
even before a definitive diagnosis is made. Patients with pneumococcemia, b. Chronic skin ulcers
meningococcemia, and aggressive soft tissue infections can decompensate c. Corneal abrasion
rapidly. d. All of the above
• The source of sepsis should be identified as soon as possible, and surgical Answer: D. It has also been reported after dental procedures, abor-
causes should be addressed. A surgeon should be consulted as soon as tions, and intestinal operations. In these cases, the source of the bac-
possible for patients with sepsis and a débridable source of infection. teria is endogenous because up to 10% of humans harbor Clostridium
• Immunity to diphtheria, tetanus, and pertussis wanes significantly in adults. tetani in the colon.
Pertussis should be considered a cause of persistent cough in adults. A 4. Which of the following statements concerning the tetanus neuro-
tetanus vaccination history should always be obtained from patients toxin (tetanospasmin) is true?
with trauma or infection. When there is doubt about the history, the age- a. A shorter incubation period portends a better prognosis.
appropriate vaccine according to CDC guidelines should be administered. b. It blocks presynaptic release of gamma-aminobutyric acid
• Consider botulism in the differential diagnosis for the infant with failure (GABA).
to thrive, constipation, or decreased muscle tone and for the patient who c. Sensory nerves are blocked, followed by motor nerves.
injects drugs with neurologic symptoms. d. Synaptic binding is reversible and overcome by acetylcholines-
terase therapy.
1. An 18-year-old male recent immigrant presents with acute-onset Answer: B. The toxin migrates in the motor nerve from the injury
sore throat, fever, and weakness. He has no past medical history. site to the central nervous system (CNS). Presynaptic release of the
Examination is remarkable for a grayish white exudative coat over inhibitory neurotransmitter GABA and glycine is diminished. The
both tonsils and the posterior pharynx. There is modest neck swell- unopposed excitatory action results in muscle spasm. Presynaptic inhi-
ing with cough, low-grade fever, and hoarseness. Rapid strep testing bition of the autonomic nervous system is also lost, and wide heart rate
and monospot results are negative. Which of the following state- and blood pressure fluctuations are seen. The toxin binding is irrevers-
ments regarding this patient’s disease is true? ible and diminishes only with axon synaptic regeneration. A shorter
a. Antibiotics and corticosteroids are the main therapy. incubation period is a harbinger of a worse outcome.
b. Neuromuscular examination is critical. 5. A 62-year-old male farmer with no medical problems presents with
c. Rapid treatment will obviate the need for intubation. leg pain and muscle spasms. He reports moderate to severe pain
d. The electrocardiogram (ECG) is a sensitive indicator of myocar- and muscle spasms in the calf that have occurred and worsened
ditis. during 3 days. Two weeks prior, he suffered a puncture wound to
Answer: B. Diphtheria can be a more benign upper respiratory/nasal his ankle just above his boot top with a piece of metal. Examination
infection or a “malignant” process with airway obstruction, myocar- is remarkable for a heart rate of 115 beats/minute, blood pressure
ditis, and neuropathy (palate is involved first; the only cells spared are of 170/110 mm Hg, and a healing clean wound above the medial
cortical). Antibiotics stop further organism growth, but diphtheria malleolus with no evidence of active infection. The calf is in active
equine antitoxin is the key to therapy. Death is usually from myocar- spasm with some increased tone in the peroneal musculature also.
ditis or airway obstruction, with the highest mortality seen in patients Which of the following statements regarding this patient’s disease is
with the “bull neck” appearance. The ECG is an insensitive indicator true?
of myocarditis. a. A radiograph should be obtained to assess for subcutaneous air.
2. An 11-year-old child presents with coughing paroxysms for 10 b. Admission for intravenous (IV) antibiotics is indicated.
days. This was preceded by a mild upper respiratory tract infection. c. Outpatient management is indicated.
The paroxysms occur multiple times per day and have occasionally d. Tetanus immune globulin is indicated.
caused vomiting. The child is relatively well between paroxysms. Answer: D. Localized tetanus reflects a local neuromuscular process
Vaccination history is unknown. Which of the following statements with pain and spasm. It is likely due to a partial immunity. Immune
regarding this patient’s disease is true? globulin is indicated. Although mortality is lower, it can progress to
a. Antibiotics will shorten illness duration. generalized tetanus, and admission is warranted.
b. Erythromycin should be given to unimmunized contacts.
c. Fever is expected.
d. Thoracic petechiae should prompt a septic evaluation.
Answer: B. Pertussis, caused by a gram-negative bacterium, is a three-
phase illness: catarrhal (upper respiratory tract infection), paroxysmal,
and convalescence of weeks to months. The coughing phase is charac-
terized primarily by complications related to the paroxysms, such as
subconjunctival hemorrhage, pneumomediastinum, headache, rectal
prolapse, chest wall petechiae, and even seizures. Fever is rare unless
there is secondary infection. Antibiotics only reduce the carrier state.
Corticosteroids may help younger children. Antibiotic prophylaxis
is indicated for nonimmunized contacts. Nasal culture is diagnostic.
Standard cough suppressants are ineffective.
119 Viruses
KEY CONCEPTS
• Recent outbreaks of vaccine- preventable childhood infections have
occurred secondary to unvaccinated individuals and travel to areas where
disease is still endemic. Emergency clinicians should recognize the possi-
bility of these once rare diseases.
• Herpes simplex encephalitis is fatal if untreated. Clinicians should suspect
this diagnosis when evaluating severely ill patients for suspected meningi-
tis or encephalitis and promptly institute empirical therapy with IV acyclovir
while awaiting diagnostic results.
• Primary varicella can be dangerous in select populations, including older
children, adults, and pregnant patients. These patients require treatment
with acyclovir.
• Zoster patients should be treated with acyclovir if they present within 72
hours of symptoms onset or if they are immunocompromised regardless of
duration of illness. Disseminated zoster should be treated with IV acyclovir.
• In healthy patients with influenza infection, the duration of illness can be
shortened by almost 1 day if antiviral treatment is administered within 48
hours of symptom onset. Hospitalized patients with influenza infection
should be treated with antiviral medication regardless of duration of symp-
toms, because it may decrease mortality and influenza complications.
• Rabies has the highest case fatality rate of any recognized infectious disease.
• Almost 60,000 deaths per year worldwide are caused by dog-mediated
human rabies, and this burden falls disproportionately on children and the
poor in rural areas.
• Globally, human rabies results from bites by infected dogs. In North Amer-
ica human rabies results predominantly from wildlife exposures (bat, rac-
coon, and skunks).
• Treatment is rarely effective once symptoms of human rabies occur.
• Rabies postexposure prophylaxis (PEP) given strictly according to the World
Health Organization (WHO) or US Centers for Disease Prevention and Control
(CDC) guidelines is extremely effective. Discussion with public health officials
is recommended to guide decisions regarding when PEP should be consid-
ered. The CDC clinician information line is 877-554-4625 or 800-CDC-INFO.
• Many emerging viral infections, including SARS-CoV2 and Ebola, should
be considered in febrile patients. It is important to identify patients at risk
by determining travel history and exposure history to individuals with con-
firmed infection. Once a patient is deemed at risk, the patient should be
promptly isolated according to established guidelines while further investi-
gation occurs. It is also important to immediately inform the hospital infec-
tion control program and public health agencies.
1. In which of the following patients is antiviral treatment not recom- Answer: B. In spite of little evidence for a bite or scratch, the intern
mended by the CDC for confirmed influenza infection? and 18-month-old fulfill requirements for PEP. Per CDC recommen-
a. 1-year-old male with 24 hours of symptoms dations, based on developmental staging, the 18-month-old girl cannot
b. 22-year-old otherwise healthy female with symptoms for 3 days know whether an exposure may have happened. The intern does not
c. 65-year-old male with history of asthma, coronary artery dis- explicitly meet the requirements for PEP because she is not a child,
ease, and congestive heart failure with 2 days of mild symptoms developmentally challenged, or intoxicated but, after a 24-hour shift,
d. 35-year-old male with no significant past medical history with she was likely in a state of deep sleep and would not have been aware
symptoms for 5 days, intubated with severe hypoxemic respira- of her surroundings. Initiating PEP for her should be seriously con-
tory failure, admitted to the ICU sidered. Although there is no contraindication for PEP during preg-
Answer: B. The 22-year-old female with no risk factors for influenza- nancy, at 36 weeks, her spouse probably was not sleeping well and was
related complications does not require antiviral treatment because her certainly more aware of her surroundings than the intern. PEP should
symptoms have been present for 3 days. The CDC recommends treating all be considered for her but, if she is adamant that no bite or scratch
patients as early as possible who are hospitalized, have severe illness, or are occurred, it may be held.
at risk for influenza-related complications. The greatest efficacy for antivi- 4. An 18-year-old male who arrived from Sierra Leone 2 weeks ago
ral treatment is within 48 hours of symptom onset, but admitted patients presents with fever, headache, vomiting, and rash. He has a tem-
with severe disease should be treated regardless of symptom onset. perature of 39.5°C and erythematous maculopapular rash over
2. A 19-year-old female presents to the ED with fever, altered mental his trunk, back, and arms. He appears ill and severely dehydrated.
status, and seizures. She is a college student and lives in a dormitory. Which of the following is immediately indicated?
Her only past medical history is occasional cold sores. She had been a. Isolation and contact the public health department
in her usual state of health until 2 days ago when she developed b. Thin and thick smear to check for malaria
fevers up to 38.5°C and headache. Today she became more lethargic c. Ribavirin
and had two generalized seizures prior to ED arrival. What is the d. Intravenous acyclovir for 10 days
next best course of action? Answer: A. This is a classic presentation for Ebola virus disease (EVD).
a. Administer 1000 mg of intravenous acetaminophen, 2 liters of The exposure history is important. Patients should be asked if they
intravenous crystalloid fluids, and admit to the observation unit have lived or travelled to a country with Ebola or had contact with a
with the diagnosis of viral syndrome. confirmed EVD patient in the past 21 days. If the answer is yes, then
b. Prescribe 1 g valacyclovir twice daily for 10 days and 650 mg the next task is to assess for any signs or symptoms compatible with
acetaminophen every 4 to 6 hours as needed and discharge Ebola, including fever, headache, weakness, myalgias, vomiting, diar-
home. rhea, abdominal pain or hemorrhage. Once a patient has been screened
c. Administer 2 g intravenous ceftriaxone, 1 g intravenous vanco- positive as a potential EVD patient, he or she should be immediately
mycin, intravenous acyclovir 10 mg/kg every 8 hours, order CT isolated and hospital infection control and the health department
scan of head, and then perform lumbar puncture. should be notified at once.
d. Order MRI of the brain, administer 1 g phenytoin, and consult 5. Which of the following viruses is not transmitted by a mosquito
neurology. vector?
Answer: C. This patient has HSV encephalitis. Given the high mortal- a. Dengue virus
ity associated with this condition, antiviral therapy should be started b. Chikungunya virus
as soon as the diagnosis is suspected. There is significant overlap in the c. West Nile virus
clinical presentation of bacterial meningitis and HSV encephalitis, so d. Lassa fever virus
patients should empirically be treated for both diagnoses while await- Answer: D. The reservoir for the Lassa virus is an African rodent,
ing CSF results. HSV encephalitis cannot be treated as an outpatient Mastomys natalensis. Humans contract the disease by exposure to
and requires 14 to 21 days of treatment with intravenous acyclovir. An urine or feces of Mastomys natalensis. Human-to-human transmission
MRI may be necessary eventually to look for temporal lobe involve- can occur via contact with blood or bodily secretions from infected
ment but is not necessary at the time of ED presentation and can be humans. The other viruses are all arboviruses transmitted via a mos-
negative early in the course of disease. quito vector.
3. A 32-year-old emergency medicine intern crawls into bed after a 24-hour
shift in the medical intensive care unit. Her 36-week pregnant spouse
and 18-month-old daughter are asleep as she climbs into their bed. She
is awakened from a dead sleep 3 hours later when her wife asks, “What is
that brown thing on the ceiling?” Using a tennis racket, the bat is success-
fully encouraged out of the window. The intern calls you and asks if they
need to be vaccinated. Your recommendations should include which of
the following?
a. PEP (postexposure prophylaxis) is contraindicated in the preg-
nant spouse.
b. The 18-month-old and intern need PEP.
c. The spouse needs PEP.
d. They need better screens on their windows.
Coronaviruses 120
KEY CONCEPTS
• C oronaviruses infect humans and animals and have the ability for recombi-
nation, generating novel viruses.
• Most human coronaviruses cause mild disease. Three coronaviruses,
severe acute respiratory syndrome (SARS) associated virus, Middle East
respiratory syndrome (MERS), and severe acute respiratory syndrome coro-
navirus 2 (SARS-CoV-2), have caused severe disease in humans.
• The hallmark presentation of severe disease in coronavirus patients is
respiratory failure, usually manifesting as acute respiratory distress syn-
drome (ARDS).
• Public health measures, including social distancing, wearing face masks,
limiting social gatherings, and vaccinations, remain integral to controlling
the COVID-19 pandemic.
• The vast majority of children have a benign disease course. However, a
small number may develop a severe hyperinflammatory illness called multi-
system inflammatory syndrome in children (MIS-C).
• Treatment for COVID-19 is rapidly evolving with the available data. Man-
agement of nonsevere illness is largely supportive and can occur at home.
Treatment of moderate to severe disease occurs in the hospital and focuses
on supporting oxygenation and ventilation. These patients may benefit from
antiviral and corticosteroid therapies. At this point, the remainder of thera-
peutics remain mostly investigational.
1. Which of the following viruses are not known to be currently circu- Answer: A. Ground-glass opacities, particularly in the lower lobes,
lating in the wild? are the most common findings of COVID-19 on CXR and on CT scans.
a. Middle East respiratory syndrome–related coronavirus (MERS- In fact, early diagnosis of COVID-19 in many areas relied on radio-
CoV) graphs because the results of PCR testing were often delayed.
b. Severe acute respiratory syndrome coronavirus (SARS-CoV) 3. Which of the following medications has proven to reduce mortality
c. Severe acute respiratory syndrome coronavirus 2 (SARS-CoV2) in those with severe COVID-19?
d. Measles a. Dexamethasone
Answer: B. The last reported case of SARS was in 2003. There are b. Hydroxychloroquine
still sporadic cases of MERS in the Middle East. Currently there is a c. Lopinavir/ritonavir
global pandemic caused by SARS-CoV-2. SARS carried a high mor- d. Low-molecular-weight heparin
tality and similarities of SARS-CoV-2 to SARS informed some of the Answer: A. Dexamethasone was the first agent to show a mortality
original pandemic response to COVID-19. benefit in those who were hospitalized with COVID-19. Hydroxychlo-
2. What are classic CXR findings of SARS-CoV-2 (COVID-19)? roquine and many other medications showed some in vitro activity
a. Patchy, ground-glass opacities against the virus, but well controlled studies did not show any benefit.
b. Pneumothorax
c. Lobar consolidation
d. Pleural effusion
HIV 121
KEY CONCEPTS
• H IV/AIDS can affect any organ system, and the nonspecific complaints seen
with viral illness are common. Consider acute HIV infection in the evalua-
tion of patients with mononucleosis-like syndromes in the presence of risk
factors.
• The presenting illness may originate from acute HIV infection, opportunistic
infections, medication side effects, inflammation, and immune reconstitu-
tion inflammatory syndrome (IRIS).
• Patients with CD4+ count above 500 cells/μL tend to have illnesses similar
to those without HIV infection.
• Opportunistic infections are more frequent as the CD4+ count declines but
can occur at any stage of HIV infection.
• The patient’s immune status should guide formulation of the differential
diagnosis, considering the CD4+ count, viral load, current medications, and
prior opportunistic infections.
• The current medications used for the treatment of HIV infection (especially
ART) can interact with many commonly prescribed drugs.
• The role of the emergency physician has expanded beyond the manage-
ment of acute HIV-related presentations, but also the facilitation of PreP,
PEP, ART initiation, and linkage to care.
1. A 52-year-old man with acquired immunodeficiency syndrome Answer: D. Toxoplasma encephalitis presents over days to weeks with
(AIDS) presents with dysphagia. The pain radiates to his chest. fever, headache, altered mental status, focal neurologic findings, and/or
He does not appear toxic. There is white plaque on his tongue seizures. In 90% of cases, there are ring-enhancing CNS lesions. the CD4+
and oropharynx, which is easily scraped off. His CD4+ count was count is typically less than 100 cells/μL, and often the count is much lower.
52 cells/μL. He was just recently started on highly active antiviral The main differential diagnosis in developed countries is CNS lymphoma.
therapy (HAART) and is taking atazanavir-ritonavir, tenofovir, and Patients with ring-enhancing CNS lesions are often empirically treated for
emtricitabine. What is the most appropriate treatment? Toxoplasma, and then a repeat CT scan is performed. A brain biopsy is diag-
a. Acyclovir nostic. Tuberculoma must be considered in patients with an exposure to
b. Fluconazole tuberculosis and in those who live in areas highly endemic for tuberculosis.
c. Ganciclovir 4. A 36-year-old woman presents with a warm, red, painful lower
d. Immediate endoscopy leg. She has multiple other dermatologic concerns, including flesh-
Answer: C. Odynophagia with a CD4+ count less than 100 cells/μL and colored, dome-shaped lesions on her face, a new dark pigmented
oral thrush suggests esophageal candidiasis. If the patient can tolerate lesion on her arm, cold sores, and facial erythema. Which of her
oral therapy, fluconazole is first-line treatment. Endoscopy is reserved cutaneous findings suggests HIV disease?
for treatment failure or those with an atypical presentation. Cytomeg- a. Cellulitis
alovirus and herpes simplex virus can cause esophagitis but are less b. Facial molluscum
common. They are diagnosed with endoscopy, often after treatment c. Melanoma
failure with fluconazole. Patients with HIV can have esophagitis due d. Oral herpes
to reflux but, in this setting, empirical treatment for candidiasis is the Answer: B. HIV has many cutaneous manifestations. In this case, her
most appropriate. Proton pump inhibitors can decrease the absorption facial molluscum is highly suggestive of HIV disease.
of atazanavir and should not be prescribed without first discussing the 5. A 35-year-old man with AIDS presents with fever and a productive
case with the patient’s human immunodeficiency (HIV) physician. cough for 1 day. His last known CD4+ count 1 month ago was 538
2. Which of the following is not an AIDS-defining condition? cells/μL. He has a lobar pneumonia in the left lower lobe on his
a. Cellulitis chest radiograph. There is no evidence of lymphadenopathy. What
b. Extrapulmonary Cryptococcus is the most likely culprit pathogen in this case?
c. Invasive cervical cancer a. Cryptococcus neoformans pneumonia
d. Recurrent pneumonia b. Pneumocystis jiroveci pneumonia (PCP)
Answer: A. All the answers except A are AIDS-defining conditions. c. Pulmonary tuberculosis
3. A 52-year-old male patient presents with 3 days of confusion, fever, d. Streptococcus pneumoniae
and headache. He was diagnosed with HIV 3 weeks ago and is not Answer: D. It is critical to have an understanding of diseases relative to
currently taking antiretroviral medications. His CD4+ count at that the absolute CD4+ T cell count. Patients with CD4 counts higher than
time was 32 cells/μL. He has lived in the United States all his life. 500 cells/μL typically develop illnesses similar to the general population.
A head CT scan with contrast on this visit reveals multiple ring- This patient presents with a lobar pneumonia, and S. pneumoniae is the
enhancing lesions. Which of the following is the most likely cause most common cause. PCP is the most common opportunistic pathogen
of this central nervous system (CNS) lesion? in AIDS patients, usually occurring in those with a CD4+ T cell count
a. Cytomegalovirus encephalitis lower than 200 cells/μL. It would be extremely unusual for PCP to cause a
b. HIV encephalopathy lobar pneumonia in someone who is immune-reconstituted. Tuberculo-
c. Mycobacterium tuberculosis sis can occur at any stage of HIV infection. However, the symptoms tend
d. Toxoplasma gondii to be more gradual in onset, and a lobar pneumonia is unlikely.
122 Parasites
KEY CONCEPTS 2. Parasite-induced loss of vision would be suggested by which of the
following?
• P arasitic diseases may manifest with almost any constellation of signs and a. Cardiomegaly
symptoms. b. Edematous and pruritic skin
• The combination of presenting signs and symptoms and a history of recent c. Fever
travel to specific geographic regions can lead to early diagnosis and the d. Hepatosplenomegaly
initiation of pharmacotherapy, decreasing morbidity and mortality and e. Iron deficiency anemia
increasing the probability of eradication of the infection. Answer: B. Onchocerciasis is a major cause of blindness worldwide.
• Parasitic coinfections are particularly common in patients with HIV infec- Ninety-five percent of cases occur in Africa. The biting flies are found
tion and AIDS. A travel history is essential because the clinical presentation near rivers, and humans are the only host for the parasite. It occupies
may be atypical, morbidity and mortality are more severe, and treatment is the skin, resulting in pruritus, edema, and later atrophy with redundant
often prolonged. skin folds. The following are other causes of parasite-induced visual
• Acute malaria should be suspected in patients with irregular high fevers loss: Toxoplasma can cause retinal hemorrhages, Toxocara can cause
associated with headache, abdominal pain, or respiratory symptoms. Fal- inflammatory retinal granulomas, and Acanthamoeba may cause a ker-
ciparum malaria, which has a unique morphology easily identifiable on the atitis in contact lens wearers.
peripheral blood smear, is the predominant species of malaria that causes 3. A macrocytic anemia would suggest infection from which parasite?
coma and death. P. falciparum is the most highly resistant to chemotherapy, a. Ancylostoma duodenale
demanding close observation and clinical follow-up of patients. Patients b. Diphyllobothrium latum
who are clinically ill or who are suspected of having falciparum malaria c. Falciparum malaria
should be hospitalized. d. Necator americanus
• Cysticercosis should be considered in the differential diagnosis of the Answer: B. The fish tapeworm is associated with pernicious anemia.
patient with new-onset seizures, especially in patients who have been liv- Hookworm and whipworm are associated with gastrointestinal iron
ing in Central and South America. loss and microcytic anemia. Malaria causes hemolytic anemia.
• Giardiasis should be suspected in patients with diarrhea who have recently 4. A 42-year-old man from Ethiopia presents with complaints of
been camping or drinking unfiltered mountain spring water. Patients may skin nodules and skin ulcerations. He has no known past illnesses,
have tolerated several weeks of severe bloating, flatulence, eructation, and exposures, medication use, or systemic symptoms. Examination is
weight loss without fever before seeking medical attention. remarkable for four 2- or 3-cm cutaneous ulcers on the arms and
• Trypanosoma cruzi infection results in Chagas disease, most notable for legs and scattered 1-cm nodules. Vital signs are normal, and phys-
the development of acute and chronic myocarditis. Cardiomyopathy can be ical examination is otherwise unrevealing. Which of the following
severe, at times even necessitating heart transplant. statements regarding this infection is TRUE?
a. Respiratory tract symptoms would suggest an alternative diag-
nosis.
1. A 33-year-old man presents with irregular fevers, shaking chills, b. The lesions always require treatment.
intermittent abdominal pain, and fatigue. The fever comes in cycles c. The lesions are likely painful to touch.
during approximately 2 or 3 days. He has no medical history and d. The skin pattern may be confused with leprosy.
takes no medications. He works as a baggage handler in Miami, Answer: D. Leishmaniasis is transmitted by the sandfly bite. Skin pap-
Florida. Physical examination reveals a low-grade fever and mildly ules and macules develop at bite sites. These may ulcerate into painless
tender hepatosplenomegaly. Laboratory evaluation is remarkable ulcers. A microcutaneous variant may be seen, and the inflammatory
for hemoglobin 9.6 g/dL, leukocytosis, lactate dehydrogenase 1850 process may involve the larynx and trachea. Disseminated cutaneous
IU/dL, elevated bilirubin, and urine dipstick “blood positive” but leishmaniasis may resemble lepromatous leprosy.
no red blood cells or white blood cells. He has had no international 5. Which of the following is the correct association between the type
travel. Peripheral smear reveals few possible parasites with frag- of parasitic infection and pulmonary symptoms?
mented red blood cells. What is this patient’s most likely infection? a. Hookworm—positive PPD response
a. Babesiosis b. Leishmaniasis—pulmonary nodules
b. Early sepsis c. Löffler syndrome—ascariasis
c. Leishmaniasis d. Pneumocystis—90% of opportunistic infections in Africa
d. Malaria Answer: C. Ascariasis and hookworm may cause Löffler syndrome of
Answer: D. Airport malaria has been reported in people who have chest pain, fever, rales, wheezing, and eosinophilia. The following are
never been in endemic areas but who work in or live near an interna- the other correct associations:
tional airport. The infected mosquito is transported from the endemic • Pneumocystis—less than 10% of pulmonary opportunistic infec-
region and released when the plane arrives. Babesiosis is a parasitic tions in Africa
illness with a clinical picture like that of malaria. It is tickborne and is • Paragonimus westermani—positive tuberculin skin test response
endemic in the northeastern United States. and chest radiograph resembling tuberculosis
• Echinococcus—anaphylaxis from leakage of cystic contents
• Schistosomiasis—diffuse pulmonary nodules (Katayama fever)
123 Tickborne Illnesses
KEY CONCEPTS
• T ickborne illnesses frequently are misdiagnosed as common viral or bac-
terial infections. Diagnosis can be facilitated by considering tickborne ill-
nesses in patients who recently have been in endemic areas and routinely
asking for a history of recent tick or insect bites in patients with febrile
illnesses.
• Lyme disease should be suspected in patients who present with signs of a
viral illness, monarticular arthritis, meningitis, multiple neurologic abnor-
malities, or heart block. Diagnosis can be confirmed with serologic testing
of acute and convalescent serum samples.
• Normal physiologic changes from tick bites should not be confused with
erythema migrans.
• Relapsing fever should be suspected in patients who present with recurrent
viral-like illness associated with high fever. The diagnosis can be confirmed
by identifying spirochetes on a blood smear obtained during a period of
rising temperature.
• Ulceroglandular tularemia should be suspected in patients with slow-
healing extremity ulcers associated with large lesions of regional adenopa-
thy (buboes). The diagnosis can be confirmed with serologic testing.
• Rocky Mountain spotted fever should be considered in patients who pres-
ent with an unexplained febrile illness, even in the absence of a rash or
known tick exposure. Delayed diagnosis and late initiation of specific anti-
rickettsial therapy may lead to a fatal outcome. Treatment never should be
delayed pending laboratory diagnosis.
1. What is the most common vector-borne disease in the United 4. A 26-year-old woman presents complaining of muscle and joint
States? aches. She has no past history and takes no medications except
a. Babesiosis over-the-counter analgesics. She describes a pattern of migratory
b. Cysticercosis and intermittent muscle aches, which she reports as lasting only
c. Lyme disease hours at any single location and then migrating. The physical exam-
d. Malaria ination reveals a mild pharyngitis and conjunctivitis and is other-
Answer: c. Lyme disease accounts for 95% of cases. wise normal. Which of the following statements is true?
2. Which of the following statements regarding erythema migrans a. A history of tick bite should be sought.
(EM) is true? b. A rheumatoid factor (RF) level should be determined.
a. The incidence with Lyme disease is 70%. c. An erythrocyte sedimentation rate (ESR) would be confirma-
b. The lesion center may become necrotic. tory.
c. The lesion is tender and warm to touch. d. Creatine phosphokinase (CPK) levels are likely elevated.
d. The rash does not begin at the tick bite site. Answer: a. The migratory, short-lived, and intermittent nature of the
Answer: b. EM occurs in 90% of cases. It begins at the bite site Lyme-related arthralgias is sometimes the best clue to the diagnosis.
and expands. There may be central clearing, skipped areas, The ESR, RF, and CPK values will likely be normal. Both rheumatoid
central necrosis, vesicles, and marked erythema. It is warm but not arthritis and fibromyalgia present with progressive and usually sym-
tender. Satellite and secondary lesions may occur due to hematogenous metrical symptoms. Conjunctivitis, pharyngitis, meningitis, and hepa-
seeding by spirochetes. They are smaller, nonmigratory, and titis pictures may be part of the Lyme presentation.
spare the palms and soles. All lesions fade after approximately 5. Which of the following statements regarding Lyme carditis is true?
1 month. a. Cardiac involvement is uncommon.
3. Which of the following clinical pictures is not seen with Lyme dis- b. Electrocardiographic changes are stable and persistent.
ease? c. Onset time from initial illness is 2 or 3 months.
a. Conjunctivitis d. The most common manifestation is bundle branch block.
b. Hepatitis Answer: a. Cardiac involvement is uncommon. The most common
c. Meningitis manifestation is atrioventricular block, which may fluctuate signifi-
d. Pleuritis cantly but often resolves as the cardiac inflammation recedes. Onset
Answer: e. Clinical pictures consistent with hepatitis, conjunctivitis, time from illness is an average of 3 to 5 weeks.
and meningitis may be seen with Lyme disease.
124 Tuberculosis
KEY CONCEPTS
• E arly recognition of patients with risk for TB should begin at ED triage.
Patients with possible active pulmonary TB should be placed in respiratory
isolation promptly.
• TB should be considered in the differential diagnosis of patients who pres-
ent with fever, cough, and weight loss.
• Risk factors for TB include HIV infection, immunosuppression, age older
than 60 years, being from an endemic country, being undomiciled, and close
contact with known cases.
• In addition to pulmonary manifestations, a variety of extrapulmonary mani-
festations may occur, including involvement of lymph nodes, pleura, bones,
joints, genitourinary, gastrointestinal, and central nervous systems.
• Therapy should be determined based on consultation with infectious dis-
ease specialists. The most commonly used agents are isoniazid (INH),
rifampin (RIF), pyrazinamide (PZA), and ethambutol (EMB). Resistant
strains, including multidrug-resistant TB (MDR-TB) and extensively drug-
resistant TB (XDR), have been increasing in frequency.
1. Which of the following is not a typical vehicle for tuberculosis 4. Which of the following statements regarding chest radiography
transmission? findings in pulmonary tuberculosis is true?
a. Coughing a. Adenopathy may distinguish TB from bacterial pneumonia.
b. Eating utensils b. Primary TB typically appears as an upper lobe cavity lesion.
c. Parenteral transmission c. The false-negative rate is 10% in immunocompetent patients.
d. Sneezing d. The false-negative rate is 40% in HIV-infected patients.
Answer: B. Bedclothes and eating utensils do not require special Answer: A. Hilar adenopathy may be the only way to suspect TB as
decontamination. Parenteral transmission is largely a risk of health opposed to a typical bacterial pneumonia. Primary TB is usually a
care workers (e.g., skin ulcers, draining wounds). Talking, coughing, single-lobe infiltrate with a homogenous appearance. It is postpri-
and sneezing all may produce significant droplet spread. mary TB that has a predilection for the upper lobes, with or without
2. Which of the following anatomic areas is not a site of preferential cavitation. The false-negative rate for chest radiography in immuno-
organism spread during stage 2 spread of TB? competent patients is very low, 1%. The rate increases to 7% to 15% in
a. Kidneys HIV-infected patients and those with endobronchial disease.
b. Long-bone epiphyses 5. Which of the following statements regarding the radiographic
c. Lung bases appearance of tuberculosis and infectivity is true?
d. Lymph nodes a. Chronic fibrotic changes are unlikely to be infective.
Answer: C. Lung apices are favored rather than the bases. These sites b. Normal radiographs are uncommon with HIV infection.
are preferred, possibly because of high oxygen tension. c. Only the presence of adenopathy can determine active disease.
3. Clinically active TB develops in what percentage of immunocom- d. The presence of cavitation implies high infectivity.
petent purified protein derivative (PPD) converters? Answer: D. Cavitation suggests high infectivity. Chronic fibrotic
a. 5% changes cannot differentiate old versus active disease, and many of
b. 10% these patients will have positive sputum. Active disease can only be
c. 15% determined radiographically by serial radiographs. Patients with late
d. 20% HIV infection are more likely to have mediastinal adenopathy and less
Answer: B. TB develops in 10%: 3% to 5% in the first 2 years (acute likely to show cavitations. Normal radiographs are common with HIV
primary) and 5% later in life (reactivation). Patients with HIV infection infection.
develop primary TB at a rate of 37% within 6 months and then develop
active TB at a rate of 7% to 10%/year.
Bone and Joint Infections
125
KEY CONCEPTS
• S keletal infection should be considered in the differential diagnosis of all
patients who present with bone or joint pain.
• Laboratory evaluation is of little value in the diagnosis of bone and joint
infections, with the exception of the ESR and CRP level, which are elevated
in approximately 90% of cases of bone and joint infections.
• Joint aspiration is the definitive diagnostic procedure, and intraoperative
synovial culture is the only reliable joint fluid test for establishing a diagno-
sis. When limited fluid is available, it should be sent for a cell count.
• With suspected septic arthritis, joint fluid and blood culture specimens are
obtained before IV antibiotics are administered. With suspected osteo-
myelitis, blood culture specimens are obtained, and IV antibiotics are
administered while plans are made for further imaging studies, surgical
aspiration, or resection of bone.
• The most important aspect of antibiotic treatment of suspected bone and
joint infections is to provide potent bactericidal activity against S. aureus
with additional empirical antibiotic coverage aimed at suspected organ-
isms on the basis of age, risk factors, and regional variability.
1. Which of the following associations between osteomyelitis and 4. Which of the following statements regarding vertebral osteomyelitis
pathogenic organism is true? is true?
a. Dog bite—Pasteurella a. Children are less prone to isolated diskitis.
b. Freshwater wounds—Pseudomonas b. Of the cases of epidural abscesses, 30% are due to osteomyelitis.
c. Human bite—Aeromonas c. The diagnostic procedure of choice is a magnetic resonance
d. Intravenous drug use—Fusobacterium imaging scan.
Answer: A. The following are correct associations: freshwater—Aer- d. The most common location is the lumbar spine.
omonas hydrophila; intravenous drug use (IVDU)—Staphylococcus Answer: D. The incidence of associated epidural abscess is 15%. The
aureus; sickle cell—Pseudomonas and Salmonella; cat or dog bite—Pas- most common location of vertebral osteomyelitis is the thoracic, lum-
teurella multocida; human bite—mixed with Fusobacterium, Eikenella, bar, and cervical spine. The diagnostic procedure of choice is needle
and Streptococcus anginosus. biopsy. The disease usually involves two vertebrae and the disk in
2. Which of the following statements regarding septic arthritis is true? between. Children are more prone to isolated diskitis, although it may
a. Despite vaccination, Haemophilus influenzae remains a frequent also occur in adults.
pathogen in children. 5. An 18-year-old woman with known sickle cell disease presents with
b. Pseudomonas aeruginosa is associated with IVDU-related osteo- leg pain of 2 days’ duration. Her typical pain syndrome is lower
myelitis. extremity tibial and femur pain. Today’s episode is primarily right
c. Pseudomonas aeruginosa is not associated with prosthetic device tibial. She complains bitterly of pain, but there are no gross findings
joint infection. other than trace bilateral anterior tibial swelling, with no discern-
d. The most common organism in neonates is Staphylococcus ible warmth or erythema. Vital signs are remarkable for a low-grade
aureus. fever and heart rate of 110 beats/min. Which of the following state-
Answer: B. Pseudomonas is associated with IVDU-related cervical ments regarding this patient’s condition is true?
osteomyelitis and lumbar osteomyelitis in cases of prolonged uri- a. Bony infection would be expected in the bony metaphysis.
nary catheterization. H. influenzae has largely disappeared as a joint b. Plain radiography can differentiate bony infarction from infec-
pathogen in vaccinated children. The most common neonatal joint tion.
pathogens are group B streptococci, Escherichia coli, and Staphylococ- d. Staphylococcus aureus would be the most likely infectious cause.
cus epidermidis. The most common cause of septic arthritis in people d. MRI is not necessary as it will not differentiate infection from
younger than 30 years is gonococcal. infarction.
3. Which of the following statements regarding acute hematogenous Answer: A. Sickle cell osteomyelitis is more typically seen in the diaph-
osteomyelitis in children is not true? ysis than in non–sickle cell situations, which more often involve the
a. Blood cultures are not usually positive. metaphysis. Fever, toxicity, and an elevated ESR suggest infection. The
b. Skeletal scintigraphy is indicated in neonates. most likely infectious cause is now Salmonella, followed by Staphylo-
c. The child usually appears toxic. coccus aureus. Often, observation and response to therapy (e.g., analge-
d. The most common site is the long-bone epiphysis. sics, hydration) ultimately help differentiate the two. When in question,
Answer: C. Children may be ill but not usually toxic. There is a 2 : 1 an MRI should be pursued.
or 3 : 1 male predominance, with the most likely site being the distal
metaphysis. Blood cultures are positive in 60% of cases. Scintigraphy is
not useful in neonates due to a limited inflammatory response. Radio-
graphs are more useful and sensitive early on than in adults.
126 Skin and Soft Tissue Infections
KEY CONCEPTS
• B acterial skin infections such as cellulitis and abscess are common and are
rarely life-threatening.
• Necrotizing infection is suggested by pain out of proportion to physical
findings, crepitance, gas seen on imaging studies, or clinical instabil-
ity. Suspected necrotizing infection should be managed with prompt
broad-spectrum antibiotics and surgical consultation.
• Emergency clinicians should be familiar with toxic shock syndrome and
Rocky Mountain spotted fever, which are rare, life-threatening, systemic
infections with skin manifestations.
• Antibiotics reduce treatment failure after surgical drainage of uncompli-
cated abscesses.
• Current recommendations for the treatment of uncomplicated cellulitis
suggest selection of an agent effective against streptococci and methicil-
lin-sensitive Staphylococcus aureus (MSSA) (e.g., cephalexin at maximal
doses). The addition of community-associated methicillin-resistant Staph-
ylococcus aureus (CA-MRSA) coverage does not improve outcomes for
uncomplicated cellulitis.
• Clindamycin is no longer recommended for routine treatment of purulent
SSTIs due to increasing rates of both MSSA and CA-MRSA resistance.
• White blood cell count should not be routinely measured in patients with
uncomplicated skin infections.
• Blood cultures are not necessary to evaluate skin infections, except in
cases of sepsis, necrotizing infections, immunocompromised hosts, and
multifocal infections suggesting hematogenous seeding.
• Mimics such as venous stasis dermatitis are often misdiagnosed as celluli-
tis. These conditions are termed pseudocellulitis and novel tools are being
developed that may assist clinicians improve diagnostic accuracy (e.g.,
ALT-70 and surface thermal imaging).
4. A 7-year-old boy presents with a left leg rash. The mother describes
1. Which of the following statements regarding cellulitis is true?
an initial sequence of a patch of small red papules that rapidly
a. Bilateral cellulitis of the lower extremities is a common bacterial
became vesicular, then pustular, and then crusted over. The emer-
infection.
gency physician observes a 2- × 3-cm area on the left thigh with
b. Computed tomography (CT) can rule out necrotizing fasciitis.
heavily crusted erythematous macules. There is moderate left ingui-
c. Fever and leukocytosis are key to the diagnosis.
nal lymphadenopathy. The lesions are not tender, and the child is
d. Needle aspiration of the uncomplicated cellulitis is unlikely to
not toxic-appearing. Which of the following statements regarding
identify the causative organism.
this patient’s condition is true?
e. Ultrasound evaluation cannot rule out a large abscess.
a. Acute rheumatic fever is a risk.
Answer: D. Needle aspiration and even biopsy are rarely able to iden-
b. Corticosteroids are indicated.
tify a causative organism in nonpurulent cellulitis. Necrotizing fasciitis is
c. Systemic antibiotics are necessary.
a clinical diagnosis and cannot be ruled out with a CT. Fever and leuko-
d. The streptozyme test is highly reliable.
cytosis are often absent, and measurement of the white blood cell count
e. Topical mupirocin is indicated.
is not indicated. Ultrasound evaluation is sensitive and specific and can
Answer: E. Impetigo involves a blistering eruption with a hon-
rule out a large abscess, although distinguishing small abscesses from the
ey-colored crust. It may be caused by streptococci or staphylococci.
cobblestoning of cellulitis can be difficult. Bilateral lower extremity inflam-
For limited disease, topical mupirocin is the treatment of choice.
mation is usually venous stasis dermatitis, which can be confused with cel-
Oral or systemic antibiotics are only indicated for more severe cases.
lulitis. Bilateral cellulitis is rare and suggests hematogenous dissemination.
Corticosteroids are not indicated. No laboratory test is useful. Acute
2. A 53-year-old woman presents with fever and painful swelling of rheumatic fever does not occur after impetigo, but poststreptococcal
the left side of her face. The physical examination is remarkable for glomerulonephritis may occur.
a toxic-appearing woman with a sharply demarcated, raised, bright 5. Which of the following statements about antibiotic therapy for skin
red, and extremely tender eruption involving the left side of her infections is true?
face. Which of the following statements regarding this patient’s con- a. Dual coverage of CA-MRSA and Streptococcus spp. reduces
dition is true? treatment failure for uncomplicated cellulitis.
a. Echocardiography is indicated. b. Uncomplicated abscesses should always be treated with dual
b. Fluoroquinolones are first-line antibiotics. antibiotic therapy covering CA-MRSA and Streptococcus spp.
c. Penicillin G monotherapy is the correct treatment. after incision and drainage.
d. The face is the most commonly involved site. c. Single-dose intravenous antibiotics are recommended prior to
e. There is an association with glomerulonephritis. discharge for uncomplicated cellulitis to reduce treatment fail-
Answer: E. Erysipelas is an acute cellulitis typically caused by group A ure.
streptococci. It presents with an angry red area of inflammation that is d. Clindamycin should be included in the antibiotic regimen for
well-demarcated from the surrounding skin and has a raised border. necrotizing infections.
Like other group A streptococcal skin infections, erysipelas can give e. Mammalian bites should routinely receive CA-MRSA coverage.
rise to poststreptococcal glomerulonephritis. Penicillin G is probably Answer: D. Multiple trials suggest that dual antibiotic therapy does
the ideal therapy, but any toxic-appearing patient should be treated not improve outcomes for uncomplicated cellulitis. Recent data sug-
more aggressively, with goal-directed therapy and broad-spectrum gest a margin of benefit for systemic antibiotic after incision and drain-
antibiotic coverage. H. influenzae is a classic cause of facial cellulitis but age but there is no support for routine dual antibiotic therapy. Single
is more common in children, is rare in the era of HiB vaccination, and intravenous doses prior to ED discharge have not been demonstrated
typically causes transdermal cellulitis, rather than the more superficial to reduce outpatient treatment failure. Given its potential to medi-
form of cellulitis known as erysipelas. The lower extremities are usually ate toxin production, clindamycin should be included in the initial
involved. antibiotic regimen for necrotizing infections, along with agents pro-
3. A 5-month-old girl presents with fever and a diffuse dermatitis viding broad-spectrum gram-positive and gram-negative coverage.
characterized by bulla formation, with surrounding vesicles leading CA-MRSA is not considered a common pathogen in mammalian bites
to the loss of large sheets of epidermis. She has no past medical and amoxicillin–clavulanic acid is first-line therapy.
history and has been on no medications. The areas of desquamation 6. A young woman presents complaining of a painful itchy rash. She
are tender and red. Which of the following statements regarding has just returned from a vacation in the Caribbean, where she was
this patient’s condition is true? snorkeling. Examination reveals streaks of erythema with slight
a. Antibiotics are not indicated. vesiculation along the lateral aspect of the right leg. The lesions are
b. Corticosteroids will help prevent progression. oriented diagonally, not along vascular lines, and are about 15 cm
c. Culture of the bullae is not indicated. long. What is the most appropriate next step in this patient’s man-
d. Mortality is greater than 30%. agement?
e. Mucous membranes are likely affected. a. Obtain a CT scan to assess for necrotizing fasciitis.
Answer: C. Staphylococcal scalded skin syndrome is a toxin-me- b. Perform a Tzanck smear on an unroofed vesicle.
diated process occurring in the very young (6 months–6 years) and c. Treat with acyclovir.
older adults. The cornerstone of treatment is resuscitation and prompt d. Treat with corticosteroids.
administration of antistaphylococcal antibiotics. Culture of bulla fluid e. Treat with trimethoprim-sulfamethoxazole.
is generally negative. Mucous membranes are spared. This condition is Answer: D. This is an example of contact dermatitis, likely from sea-
rarely fatal. weed or jellyfish stings. Nothing in the presentation is suggestive of
an infection. Trimethoprim-sulfamethoxazole monotherapy is rarely
appropriate because abscesses generally do not require antibiotics,
and streptococci are thought to be covered poorly by this antibiotic. A
Tzanck smear and acyclovir treatment would be appropriate if herpetic
infection were suspected, but neither herpes simplex nor varicella zos-
ter would present with 15-cm streaks.
127 Sepsis Syndrome
KEY CONCEPTS
• S epsis is a progressive disease due to a dysregulated inflammatory cas-
cade, leading to organ dysfunction and circulatory compromise in severe
cases.
• Older adults, immunocompromised and neutropenic patients, and patients
with multiple comorbidities are at increased risk for the development of
sepsis syndromes.
• A thorough history, physical examination, and laboratory testing should
guide the diagnostic evaluation.
• Early treatment should focus on appropriate identification, improvement
of tissue perfusion (through the administration of fluids and vasopressor
medications), improvement of tissue oxygenation (through administration
of oxygen and positive-pressure ventilation), administration of antibiotics,
and early identification of infections requiring surgical management.
• Prompt administration of antibiotics is essential and should be based on the
suspected source of infection.
1. Which of the following patients meets the criteria for systemic 4. What are the two most common sources of infection in cases of
inflammatory response syndrome (SIRS)? sepsis?
a. 6-year-old boy with pneumonia, temperature 39.0°C (102.2°F) a. Genitourinary > respiratory
b. 53-year-old man with respirations 30 breaths/min, white blood b. Musculoskeletal > genitourinary
cell (WBC) count 16,000 cells/mm3 c. Respiratory > gastrointestinal
c. 74-year-old woman with chest pain, heart rate 130 beats/min d. Respiratory > genitourinary
d. 81-year-old man with WBC count, 2700 cells/mm3, heart rate, Answer: C. Epidemiology studies show that pneumonia is the most
73 beats/min common cause of sepsis, followed by an intra-abdominal source. How-
Answer: B. SIRS is defined as two or more of the following—tachy- ever, a careful investigation to identify the source of infection should
cardia, tachypnea, temperature higher than 38°C (100.4°F) or lower occur.
than 35°C (95°F), high or low WBC count, or bandemia. Sepsis is SIRS 5. In most chemotherapy patients, which neutrophil count should
with infection. Severe sepsis includes organ dysfunction. Septic shock prompt admission, isolation, and empirical antibiotics?
involves systolic blood pressure below 90 mm Hg. a. <250 cells/mm3
2. Sepsis is characterized by which of the following? b. <500 cells/mm3
a. Depression of tumor necrosis factor levels c. <750 cells/mm3
b. Increased endogenous anticoagulant levels d. <1000 cells/mm3
c. Prolonged suppression of nitric oxide levels Answer: B. Patients with an ANC <500 cells/mm3 are at increased risk
d. Increased inflammatory cytokine levels of infection; thus, a conservative approach should be taken in these
Answer: D. Clinical sepsis is induced by sustained levels of proinflam- patients.
matory and procoagulant mediators. Cytokines (interleukin-1, inter- 6. Among patients with clinical septic shock, which percentage will
leukin-6, and tumor necrosis factor-α) and prostaglandins are primary have positive blood cultures?
mediators. Nitric oxide synthase is upregulated, resulting in sustained a. 0%–30%
elevations of the serum nitric oxide level, with subsequent vasodila- b. 30%–60%
tions. Sustained suppression of vasopressin adds to this sometimes c. 60%–90%
refractory vasodilated state. d. 90%–100%
3. Which of the following statements regarding septic shock is true? Answer: B. Although blood cultures are perhaps a gold standard for
a. Cardiac output is always decreased. identification and isolation of bacteria, they may be negative even
b. Much of the cardiac decompensation is reversible. when the etiology of illness is clearly infectious. Empirical antibiotic
c. Systemic vascular resistance is high. treatment in the ED remains a standard approach.
d. The ejection fraction is always increased.
Answer: B. Sepsis affects myocardial function and peripheral vascular
tone. The systemic vascular resistance is usually markedly depressed.
Cardiac output is generally increased because of a compensatory
tachycardia that can at least partially overcome the ventricular dilation
and depressed ejection fraction. The myocardial effects are typically
reversible.
128 Hypothermia, Frostbite, and
Nonfreezing Cold Injuries
KEY CONCEPTS 3. A 30-year-old woman in cardiac arrest is brought to the ED by
emergency medical services. She was intubated in the field, and
• P atients with hypothermia should be actively rewarmed whenever possi- chest compressions have been continuously performed. Her tem-
ble. Specific indications for active rather than passive rewarming include perature is 25°C (77°F). When placed on the cardiac monitor, she
trauma, cardiovascular instability, temperature below 32°C (89.6°F), poor is noted to be in ventricular fibrillation. A defibrillation attempt
rate of passive rewarming, and endocrine insufficiency. is made at the appropriate setting, but she remains in ventricular
• Rewarming methods should be chosen to minimize core temperature afterdrop. fibrillation. A nurse resumes providing chest compressions and asks
• If tachycardia is out of proportion to core temperature then hypoglycemia, for further instructions. What should be done next?
hypovolemia, or an overdose should be considered. a. IV amiodarone
• The effects of most medications are temperature dependent. Overmedi- b. IV lidocaine
cation to achieve an effect when the patient is cold could cause toxicity c. IV procainamide
during rewarming. d. Warm the patient
• Laboratory coagulation tests are performed at 37°C (98.6°F). Despite clin- Answer: D. Warm the patient. This severely hypothermic patient
ically obvious coagulopathy, measures of coagulation will be deceptively needs further rewarming before subsequent defibrillation attempts are
normal. Treatment for coagulopathy is to rewarm the patient. made to treat her ventricular fibrillation. Most hypothermia-induced
• There are no safe predictors of serum electrolyte levels. Hypothermia dysrhythmias convert spontaneously during rewarming. Asystole that
enhances the cardiac toxicity of hyperkalemia and obscures premonitory develops during rewarming is not as ominous as asystole in normo-
electrocardiographic changes. thermic patients. For VF, defibrillation should be attempted at the
• Failure to rewarm despite good technique should suggest infection, endo- usual energy level. Successful defibrillation has been reported at 20°C
crine insufficiency, or a futile resuscitation. (68°F) but attempted defibrillation is often unsuccessful until the core
temperature is above 30°C (86°F). If a defibrillation attempt is unsuc-
1. Emergency medical services (EMS) notifies your emergency cessful, active rewarming should be initiated while continuing CPR.
department (ED) that an unknown male who was “found down” is Defibrillation can be attempted occasionally during rewarming. Once
being transported. No history is available. Paramedics report that the core temperature is above 30°C (86°F), further attempts can be
the patient’s pulse is 42 beats/min and blood pressure is difficult to made.
obtain. Spontaneous respirations are present at a rate of 10 breaths/ 4. A 27-year-old homeless man presents complaining of numb feet.
min. The electrocardiogram (ECG) showing sinus bradycardia He reports sleeping outdoors overnight. The temperature was con-
is faxed before the patient’s arrival. What treatment should you sistently below −20°C (−4°F). The feet appear white and waxy. They
administer when the patient arrives? are cold and hard to the touch. What is the best way to rewarm the
a. Intravenous epinephrine feet?
b. Normal saline bolus a. Room temperature air convection
c. Synchronized cardioversion b. Room temperature water immersion
d. Warm the patient c. Warm air convection
Answer: D. Warm the patient. When this hypothermic patient arrives d. Warm water immersion
in the ED, after assessment of the airway, the top priority is rapid Answer: D. This patient is suffering from frostbite of the feet. The treat-
rewarming. Patients with hypothermia should be actively rewarmed ment of choice in this scenario is rapid warm water immersion. Rapidly
whenever possible. Specific indications for active rather than passive rewarm completely frozen or partially thawed tissue by immersion in
rewarming include trauma, cardiovascular instability, temperature gently circulating water that is carefully maintained at a temperature
below 32°C (89.6°F), poor rate of passive rewarming, and endocrine of 37°C to 39°C (98.6°F to 102.2°F). A whirlpool is ideal, but any large
insufficiency. container can be used for the hands or feet. Water warmer than 39°C
2. Rapid rewarming is the mainstay of treatment for hypothermia. (102.2°F) does not thaw significantly faster but causes more pain. Tis-
However, rewarming can cause complications. Which of the fol- sue can suffer thermal injury when the water temperature exceeds
lowing complications should be anticipated and prevented when 42°C (107.6°F). Rewarming should be continued until distal erythema
rewarming a hypothermic patient? is noted. The part should have return of color and feel pliable, which
a. Hyperkalemia usually requires 15 to 30 minutes of submersion.
b. Hyponatremia 5. What is the most common error made when treating frostbite
c. Hypotension injury?
d. Rhabdomyolysis a. Débridement of broken blisters
Answer C: Hypotension should be anticipated and prevented when b. Premature termination of thawing
rewarming a hypothermic patient. Patients with moderate or severe c. Use of nonsteroidal antiinflammatory drugs (NSAIDs) for anal-
hypothermia are usually volume depleted. During rewarming, the total gesia
plasma volume is usually high, but the circulatory plasma volume is d. Use of thrombolytic agents to restore blood flow
usually low due to increased peripheral vascular resistance. Rapid vol- Answer: B. Premature termination of thawing in 37°C to 39°C (98.6°F
ume expansion can be lifesaving, especially in hypothermic neonates. to 102.2°F) water is a common error. Reperfusion of completely frozen
Adult patients with moderate or severe hypothermia should initially tissue may be painful, requiring parenteral analgesia.
receive a 500-mL fluid challenge of warmed normal saline. Avoid lac-
tated Ringers solution because the cold liver metabolizes lactate poorly.
Fluids administered via the intravenous (IV) route should be warmed
to 40°C to 42°C (104°F to 107.6°F).
Heat Illness 129
Answer: c. Heat exhaustion is a clinical syndrome. Whereas there are
KEY CONCEPTS typically two types of heat exhaustion, water depletion and salt deple-
• C lassic heatstroke is generally diagnosed in older patients with comorbidi- tion, pure forms of either type are rare. Most cases of heat exhaus-
ties during heat waves, whereas exertional heatstroke is more common in tion involve mixed salt and water depletion. In salt depletion heat
young athletic patients or military personnel. exhaustion, the syndrome is characterized by hyponatremia, hypo-
• Heat exhaustion and heatstroke are a continuum of the same pathophys- chloremia, and low urinary sodium and chloride concentrations.
iologic process. Neurologic dysfunction is a hallmark of heatstroke, and The symptoms and signs associated are variable and nonspecific but
cerebral edema is common. usually systemic, such as weakness, fatigue, frontal headache, vertigo,
• In heat stroke, morbidity and mortality are directly related to the duration nausea, and vomiting. The body temperature usually remains nearly
of core temperature elevation. Rapid cooling should be initiated without normal.
delay. 3. Despite cooling measures, poor outcomes are seen in heatstroke
• The evaporative technique and ice water immersion are the 2 forms of pri- patients with which of the following?
mary cooling measures. Other cooling measures are secondary and consid- a. Altered coagulation status
ered adjunct therapy. b. History of schizophrenia
• Antipyretics are ineffective and should not be used to control environmen- c. Need for supplemental oxygen
tal hyperthermia. d. Presentation with acute renal failure
• Heatstroke can cause right-sided cardiac dilation and elevated central e. Presentation with acute rhabdomyolysis
venous pressure and clinically resemble pulmonary edema but may still Answer: a. Factors such as advanced age, hypotension, altered coagu-
require crystalloid resuscitation. lation status, and the necessity for endotracheal intubation on arrival at
the emergency department predict a poor outcome, despite successful
cooling measures.
1. An 18-year-old female marathon runner presents to the medical
4. The usual characteristics of classic heatstroke include which of the
aid station during a hot summer race. She is extremely irritable
following?
and diaphoretic. She is complaining of generalized weakness, diz-
a. Diaphoresis
ziness, nausea, and headache. The physical examination reveals an
b. Disseminated intravascular coagulation
oral temperature of 40.5°C (105°F), heart rate of 120 beats/min,
c. Hypoglycemia
muscle twitching, and ataxia. What is the most appropriate man-
d. Marked lactic acidosis
agement?
e. Usual occurrence during heat waves
a. Assess her volume status and immediately start normal saline to
Answer: e. Usual characteristics of classic heatstroke include predis-
replete volume loss before transfer to a hospital.
posing factors or medication, older population, sedentary lifestyle,
b. Encourage her to drink cold water to replace her free water defi-
anhidrosis, normoglycemia, mild coagulopathy, mild elevation in cre-
cit rapidly.
atine kinase level, oliguria, mild acidosis, and occurrence during heat
c. Immediately remove her from the hot environment and begin
waves. Diaphoresis, hypoglycemia, disseminated intravascular coag-
cooling before transfer to a hospital.
ulation, and marked lactic acidosis are characteristics of exertional
d. Prescribe immediate rest, after which she may be allowed to fin-
heatstroke.
ish the race.
5. Which of the following is associated with heat cramps?
Answer: c. The onset of heatstroke is sudden. Prodromal symptoms
a. Excessive salt intake
lasting minutes to hours can occur that are nonspecific and similar to
b. Drinking copious amounts of hypertonic fluids
those of heat exhaustion. Signs and symptoms may include weakness,
c. Carpopedal spasms with distal extremity paresthesias
dizziness, nausea, frontal headaches, confusion, muscle twitching,
d. Hyponatremia and hypochloremia
ataxia and signs of cerebellar dysfunction, and psychiatric symptoms,
e. Rhabdomyolysis
ranging from anxiety and irritability to psychosis. Heat exhaustion can
progress to heatstroke if it is untreated. If the patient is evaluated as this Answer: d. Heat cramps are brief, intermittent, and often severe
is occurring, differentiation between heat exhaustion and heatstroke is muscle cramps occurring typically in muscles that are fatigued by
difficult. If heatstroke cannot be excluded, begin cooling the patient heavy work. Heat cramps appear to be related to a salt deficiency.
immediately. Heat cramps are occasionally confused with hyperventilation tetany.
Whereas rest is part of the treatment for heat exhaustion, it is not Hyperventilation tetany can be distinguished by the presence of car-
the only treatment. She must be removed from the hot environment, popedal spasm and paresthesias in the distal extremities and perioral
not be allowed to finish the race, and assessed for her volume status. area. Heat cramps accompanied by systemic symptoms may be part
Normal saline is used to replete volume if the patient is orthostatic; free of salt depletion heat exhaustion. Heat cramp victims exhibit hypo-
water deficits are replaced slowly to avoid cerebral edema. natremia and hypochloremia, so serum electrolyte levels should be
2. Which of the following statements regarding heat exhaustion is measured. Rhabdomyolysis or resultant renal damage is not present
true? with isolated heat cramps.
a. It causes body temperatures that often exceed 40.5°C (105°F).
b. It only exists in two discrete forms, either salt depletion or water
depletion.
c. It is associated with systemic symptoms.
d. It is characterized by hyponatremia and hyperchloremia.
e. It occurs when muscles are fatigued by heavy work.
Electrical and Lightning Injuries
130
KEY CONCEPTS
• E lectrical current follows the path of least resistance, which is often along
neurovascular bundles. Deep tissue injuries and organ damage are often
more extensive than indicated by examination of the overlying skin.
• Testing is not indicated for victims of low-voltage electrical injuries who
are asymptomatic or have minimal local symptoms and physical evidence
of burn injury. These patients may be discharged home after emergency
department (ED) evaluation.
• Patients exposed to high-voltage sources or lightning strikes who present
with syncope, altered mentation, focal neurologic abnormality, significant
burns, entry and exit wounds, or persistent symptoms should have test-
ing including electrocardiography, complete blood count, basic chemistry
panel, myoglobin and troponin level determination, and urinalysis. Addi-
tional testing is directed at suspected areas of injuries.
• Patients with electrocardiographic signs of cardiac injury or dysrhythmias,
or with evidence of significant local injury, should be monitored for 12 to 24
hours in the ED, observation unit, or inpatient setting.
• The enormous current of a lightning strike may cause critical injury or
death, or the current may be directed superficially over the patient to the
ground, resulting in no injury and minor burns.
• Electrocardiography is indicated for all patients evaluated for lightning
strike. Additional testing should be based on specific signs and symptoms.
• Lightning strike patients who present without symptoms or signs of injury,
or with only minor first-degree burns, and with a normal ECG can be dis-
charged home from the ED after evaluation.
• Lightning strike can cause fixed, dilated pupils in the absence of irreversible 2. Which of the following patients presenting to the emergency
brain injury. department should have consultation by the burn surgery service?
a. 22-year-old female who presents after receiving a taser shock
who is awake and alert. She has barb wounds in her abdomen
that were removed by law enforcement prior to arrival and has
1. A 37-year-old woman presents with blurred vision. Her vision has mild erythema at the barb sites, but no other injuries.
been progressively worsening for approximately 2 months. She b. 47-year-old male who presents after a lightning strike, who com-
denies eye pain or direct eye trauma. She reports that approximately plains of chest pain and EKG shows sinus tachycardia.
2 months ago, just before the beginning of her vision problems, she c. 34-year-old male who presents after an occupational accident,
was struck by lightning while playing golf. She states that she went in which he received an electrical shock to the right hand from
to an emergency department (ED) at that time, had no major inju- a high-voltage source. He is awake and alert, with normal vital
ries, and seems to have recovered well. What is the most likely cause signs and no complaints other than a small area of hyperemia to
of her decreased vision? his right palm and pain to his right arm.
a. Cataracts d. 3-year-old female, brought in by parents for concern that she put
b. Glaucoma her finger into an electrical socket. On exam, she is awake and
c. Iritis alert, has normal behavior, and has no skin or soft tissue findings.
d. Macular degeneration
e. Retinal detachment e. 67-year-old female who presented unresponsive after a light-
Answer: A. Cataracts are a well-known complication of electrical ning strike, but is now awake and alert, but reports hearing
injury, either from artificial electrical sources or from lightning. They difficulties.
may occur immediately or have a delayed presentation. Glaucoma is Answer: C. Electrical injuries, especially high- voltage injuries, can
due to elevated intraocular pressure and is often painful. Iritis and ret- cause deep tissue burns and damage that may not be readily apparent
inal detachment can be caused by electrical injury but are manifested on the surface, due to decreased resistance of neurovascular bundles.
acutely. Macular degeneration is a chronic condition of older adults In severe cases, tissue edema and decreased perfusion may require
with central vision loss and is not associated with electrical injury. fasciotomy. Often, fluid resuscitation calculations using the Parkland
Macular holes may occur acutely with lightning injury. formula are underestimated, as visible injury may be insignificant
compared to damage beneath the surface. Patients who receive a taser
shock do not require further management beyond local wound care.
The victims of lightning strikes may require further evaluation for
cardiac and HEENT abnormalities, but skin and soft tissue damage in
these patients is less likely as the electrical energy often passes over
rather than through them. The 3-year-old does not require further
management beyond reassurance.
3. A 42-year-old man suffers an electrical injury while working on Answer: E. Delayed labial artery bleeding usually occurs two days after
power transmission lines near your hospital. Paramedics report the initial injury, when the initial eschar separates. When this occurs,
that he is unresponsive and initially in ventricular fibrillation but there can be significant bleeding from the labial artery. Traditionally,
spontaneously converted to sinus tachycardia before treatment. His admission for observation has been advocated for these patients but, if
initial electrocardiogram (ECG) from the field shows ST segment there is good social support, discharge with specific return instructions
elevation in the inferior leads. An ECG repeated in the ED shows and close follow-up are appropriate. These types of electrical injury
continued inferior ST segment elevation but with decreased mag- have only local effects, so dysrhythmias, cataracts, and rhabdomyoly-
nitude. Creatine kinase (CK) and CK-MB levels are markedly ele- sis are not complications. Contractures are common and may require
vated and troponin levels are slightly above normal range. While he reconstructive surgery, but this complication is not life-threatening.
does have pulses, he continues to be unresponsive, has no sponta- 5. A 53-year-old male is brought to the emergency department by law
neous respirations, and has bilateral fixed and dilated pupils. What enforcement after they incapacitated him with a taser. There are no
is the appropriate next action? visible signs of traumatic injury, but he is not responsive. The most
a. Brain death testing appropriate next step is:
b. Cardiac catheterization a. Supportive care as most patients quickly arouse after a taser
c. Mannitol infusion shock.
d. Intubation and supportive care b. Evaluate for burns around the taser barb site.
e. Thrombolytic administration c. Perform EKG to evaluate for STEMI.
Answer: D. Electrical injury can cause a variety of cardiac manifes- d. Evaluate the patient for other etiologies of altered mental status.
tations, including multiple dysrhythmias, transient ST elevation, and e. Discharge the patient with law enforcement to be taken to jail.
conduction blocks. Myocardial infarction, although reported, is rare. Answer. D. There have been a number of high-profile deaths associ-
Extensive skeletal muscle damage can be seen in electrical injuries and ated with taser shocks by law enforcement. However, the cause of these
results in marked elevations of CK and CK-MB levels. Respiratory deaths has not been linked to the electrical effect of the taser shock
muscle paralysis and ocular injuries often occur. This patient is proba- itself, and patients who present after a taser shock with altered men-
bly not brain-dead, nor has he suffered a myocardial infarction. Man- tal status should be evaluated for other etiologies, including but not
nitol is used in cases of impending cerebral herniation, which is not limited to infection, toxic/metabolic etiologies, and traumatic injuries.
expected in this patient. Supportive care, including mechanical venti- Burns evaluation and EKG alone would not be sufficient evaluation for
lation and treatment of dysrhythmias per ACLS protocols as needed is this patient.
most appropriate.
4. A 22-month-old female is brought to the emergency department by
parents after she was found chewing on an electrical cord. She has
minor burns to the lips and oral commissure. Upon discharge, what
condition should the parents be warned about as a reason to return
to the ED?
a. Facial compartment syndrome
b. Cardiac dysrhythmias
c. Delayed dental eruption
d. Rhabdomyolysis
e. Delayed labial artery bleeding
131 Scuba Diving and Dysbarism
KEY CONCEPTS
• T he majority of dive injuries are diagnosed on the basis of the focused dive
history and physical examination and are best differentiated into disorders
of descent, disorders of depth, and disorders of ascent.
• The U.S. Navy Diving Manual and the Divers Alert Network (DAN) are valu-
able resources for the clinician presented with a diving emergency. DAN
provides a 24-hour medical emergency hotline at 1-919-684-9111 (see
Table 131.4).
• Treatment with 100% oxygen is the initial therapy for all diving emergen-
cies until the diagnoses can be determined. It has been demonstrated to
reduce the morbidity and mortality related to decompression illness and
can be helpful in patients with pneumothorax.
• Diagnostic imaging and laboratory studies are generally not useful for rul-
ing in or out decompression illness and should not delay transfer for recom-
pression therapy.
• Recompression treatment is recommended for patients with decompres-
sion sickness (DCS) and arterial gas embolism (AGE).
1. When is a diver most likely to suffer barotrauma? 3. Which of the following individuals should be advised not to dive?
a. When the diver also suffers from decompression sickness (DCS) a. A 10-year-old boy with no known medical problems
b. When the diver is at extreme depth b. A 19-year-old woman with no medical problems but who just
c. When the diver is closest the surface landed from an intercontinental commercial flight
d. When the diver stays at depth for an extended period of time c. A 20-year-old man with well-controlled asthma
e. When the diver uses specialized gas mixtures with decreased d. A 27-year-old woman who is 20 weeks pregnant
partial pressure of nitrogen e. A 70-year-old man with asymptomatic coronary artery disease
Answer: C. Barotrauma refers to injuries due to pressure changes. Answer: D. Upon surfacing, small nitrogen bubbles form in the circu-
Boyle’s law states that pressure and volume are inversely proportional lation of all divers. If proper technique is followed, these bubbles are
(P × V = k or P1V1 = P2V2). As the pressure decreases, the volume small and asymptomatic. They form in the venous circulation and are
of gas increases and can damage gas-filled structures (sinuses, inner/ eliminated in the lungs. However, if these bubbles are transmitted into
middle ears, lungs, and intestines). The incremental changes in pres- the arterial system, embolization with serious consequences can occur.
sure (and therefore volume) are greatest at the surface, so barotrauma The fetus has a patent foramen ovale (PFO) and ductus arteriosus, both
most commonly occurs near the surface (either at the beginning or at of which would allow air to bypass this filtering effect of the lungs and
the end of a dive). Long or deep dives are not required for barotrauma proceed into the systemic circulation. Asthmatics can dive as long as
but predispose to decompression illness due to the build-up of tissue symptoms are well controlled. There are no definite age recommen-
nitrogen which comes out of solution during ascent. dations for diving. Flying immediately after diving should be avoided;
2. A 55-year-old man presents with the acute onset of left-sided flying prior to diving is of less concern.
weakness and confusion. Family members report that they are on 4. A 24-year-old woman presents with complaints of severe chest pain
vacation and had just finished scuba diving when the patient com- and shortness of breath. The symptoms started approximately 1 hour
plained of chest pain, became confused, and stopped moving his ago while surfacing from a dive. She denies loss of consciousness
left arm and leg. This happened 30 minutes ago. The patient has no or other symptoms. Physical examination reveals decreased breath
known medical history and takes no medications. Physical exam- sounds in the right chest and crepitus in her neck; otherwise, it is
ination reveals a drowsy and confused male who follows commands normal. Vital signs are normal except for a slight tachypnea. A chest
but does not move his left arm or leg. His blood pressure is 162/98 radiograph shows a moderate right-sided pneumothorax and a pneu-
mm Hg, and his other vital signs are within normal limits. What is momediastinum. She has no other symptoms or signs. You place a
the appropriate treatment? chest tube on her right side. What is the next course of action?
a. Computed tomography (CT) of the brain a. Computed tomography (CT) of the brain
b. Endotracheal intubation b. CT of the chest
c. Intravenous labetalol c. Observation and supportive care
d. Intravenous tissue plasminogen activator d. Pericardiocentesis
e. Recompression in a dive chamber e. Recompression in a dive chamber
Answer: E. This patient has suffered an arterial gas embolism (AGE). Answer: C. This patient has suffered from pulmonary barotrauma.
In AGE, air bubbles gain access to the arterial circulation and can cause Although this injury does predispose one to arterial gas embolism
mechanical obstruction of the artery. Symptoms can mirror an acute (AGE), the diagnosis of AGE is clinical. The treatment for a pneumo-
thrombotic or embolic event including myocardial infarction or stroke. thorax from pulmonary barotrauma is the same as for a pneumotho-
Treatment with 100% oxygen should be instituted immediately, but the rax from another cause—that is, aspiration, catheter insertion, or chest
only definitive treatment is emergent recompression. tube placement. Management of pneumomediastinum is supportive.
Recompression is only necessary for decompression sickness (DCS) or
AGE, not for barotrauma.
5. A 32-year-old man presents with bilateral lower extremity numb- 6. You decide that a 27-year-old patient with an arterial gas embo-
ness and weakness. The patient reports several days of scuba diving lism (AGE) following scuba diving requires recompression therapy.
without incident until after his most recent dive, when the symp- Which of the following treatments should be initiated before hyper-
toms started. Physical examination reveals bilateral lower extremity baric therapy?
weakness and decreased sensation to pinprick and light touch. Pri- a. Administer 40% oxygen.
apism is also noted. The remainder of the physical examination and b. Administer steroids.
all vital signs are within normal limits. You decide that recompres- c. Ensure that the endotracheal tube and urinary catheter balloons
sion therapy is indicated, but your emergency department does not are filled with water, rather than air.
have a dive chamber and the nearest chamber is 50 miles away. The d. Place prophylactic thoracostomy tubes bilaterally.
patient has been accepted in transfer. What is the most appropriate e. Place the patient in the Trendelenburg position.
way to transfer this patient? Answer: C. 100% normobaric oxygen is standard of care during eval-
a. Air ambulance, lying flat uation, treatment and transport. Inflate endotracheal tube and urinary
b. Air ambulance, Trendelenburg position catheter balloons with sterile saline. There is no evidence that steroids
c. Ground ambulance, lying flat are effective. The Trendelenburg position increases intracranial pres-
d. Ground ambulance, Trendelenburg position sure and facilitates coronary gas embolization.
e. This patient has not yet been stabilized for transport; he should
remain at the current hospital.
Answer: C. This patient is suffering from spinal decompression sick-
ness (DCS II). Recompression therapy is the treatment of choice.
Unless an inordinate delay to the recompression chamber would result,
anything that further decreases ambient pressure should be avoided
(e.g., flying). If a patient must be flown, the aircraft should fly at the
lowest possible altitude or pressurize the cabin at the highest possi-
ble pressure. Traditionally, the Trendelenburg position was advocated
because it was believed that it would decrease the incidence of AGE
to the brain. This is not the case. In addition, the Trendelenburg posi-
tion increases cerebral edema and the incidence of coronary artery air
embolism. Therefore, the Trendelenburg position should not be used
for patients with diving injuries.
High-Altitude Medicine 132
3. Which of the following medications used to treat acute mountain
KEY CONCEPTS sickness (AMS) actually accelerates acclimatization?
• A ll forms of altitude illness can be treated with oxygen and rapid descent. a. Acetazolamide
• The diagnosis of acute mountain sickness requires the presence of head- b. Ibuprofen
ache in the setting of a recent elevation change to greater than 8,000 c. Dexamethasone
feet. Additional nonspecific symptoms may include nausea, anorexia, and d. Oxygen
fatigue. Those with significant symptoms should not ascend further until e. Prochlorperazine
symptoms improve. Answer: A. Acetazolamide is a carbonic anhydrase inhibitor and
• Patients with mild high-altitude pulmonary edema may be treated in place induces a metabolic acidosis that results in increased ventilation. Ibu-
if experienced providers and treatment options exist. Patients with moder- profen is an appropriate treatment for the headache associated with
ate high-altitude pulmonary edema or high-altitude cerebral edema should AMS. Dexamethasone and oxygen both improve all symptoms of AMS
descend immediately. but do not aid acclimatization. Prochlorperazine is appropriate treat-
• Dyspnea at rest is an early symptom of high-altitude pulmonary edema. ment for the gastrointestinal upset caused by AMS and is preferred
More advanced findings include rest tachypnea, cough productive of frothy over promethazine because it does not depress the respiratory centers.
sputum, and altered mentation. Treatment involves oxygen and descent. 4. Which of the following conditions occurs more commonly at high
• Altered consciousness and cerebellar ataxia are early signs of high-altitude altitude?
cerebral edema. Failure to initiate immediate treatment with oxygen, a. Congestive heart failure (CHF)
descent, and dexamethasone can result in permanent disability or death. b. Myocardial infarction
• Acute mountain sickness may be prevented by using acetazolamide or c. Pericarditis
dexamethasone. Symptoms can be controlled with analgesia and anti- d. Pneumothorax
emetics. High-altitude pulmonary edema may be prevented using nifedip- e. Pulmonary embolism (PE)
ine. In patients unable to use nifedipine, consider inhaled salmeterol, and Answer: E. Altitude may contribute to hypercoagulability from
oral phosphodiesterase type 5 inhibitors (sildenafil [50 mg every 8 hours] hyperviscosity due to elevated hematocrit and dehydration. Venous
and tadalafil [10 mg every 12 hours]). Temazepam (7.5 mg qhs) can safely stasis is also more common at altitude due to the relative immobility
improve sleep quality. of sleeping in confined spaces (sleeping bags and small tents) and long
flights that tend to precede vacations at altitude. Together, these facts
lead to an increased risk of PE. Occasionally, PE may be misdiagnosed
1. Which of the following individuals has the highest risk of devel- as high-altitude pulmonary edema (HAPE) and treated inappropriately.
oping high-altitude pulmonary edema (HAPE), assuming all are 5. The acute hypoxic stresses of high altitude can cause a range of symp-
generally healthy, live at the same altitude, and travel to the same toms suggestive of both cardiovascular and CNS dysfunction. Although
altitude? high-altitude pulmonary edema (HAPE) and acute mountain sickness
a. A 16-month-old boy (AMS) share the same pathophysiologic trigger (decreased available
b. A 19-year-old man oxygen), the treatment and prognosis are different. Which of the fol-
c. A 19-year-old woman lowing features is indicative of HAPE as opposed to AMS?
d. A 42-year-old man a. Frequent urination
e. A 42-year-old woman b. Dyspnea at rest
Answer: B. Many factors contribute to the development of HAPE c. Headache
including total altitude, sleeping altitude, time at altitude, previous d. Lower-extremity edema
acclimatization, and the presence of comorbidities. However, in gen- e. Symptom onset immediately after arrival at altitude.
eral, both females and the elderly have been found to be less likely to Answer: B. Dyspnea on exertion is nearly universal at altitude and
develop HAPE. Also, whereas younger males are more at risk, HAPE is not indicative of a pathologic process. Dyspnea at rest is not normal
is extremely rare in individuals younger than 2 years old regardless of and is an early symptom of HAPE. Cough is common at altitude or any
gender. time when one breathes cold dry air. Fluid retention and peripheral
2. Which of the following symptoms must be present to establish a edema are common manifestations of AMS, whereas an alkaline diure-
diagnosis of acute mountain sickness (AMS)? sis can be a sign of acclimatization. Neither AMS nor HAPE commonly
a. Dizziness occurs immediately after arrival at altitude. AMS usually occurs within
b. Fatigue 24 hours and HAPE usually occurs after 2 to 4 days.
c. Headache
d. Dyspnea 6. The definitive treatment for high-altitude pulmonary edema
e. Nausea (HAPE) is oxygen and descent to lower altitudes. Which of the fol-
Answer: C. To diagnose AMS, a patient must have a recent gain in lowing medications is also useful in the treatment of HAPE?
altitude, be at that altitude for several hours, and have a headache. In a. Hydrochlorothiazide
addition, at least one of the following must also be present: gastrointes- b. Metoprolol
tinal upset, fatigue, dizziness or light-headedness. Exertional dyspnea c. Nifedipine
is common at high altitude. Resting dyspnea is concerning for potential d. Nitroglycerin
HAPE. e. Furosemide
Answer: C. Nifedipine is a nonselective pulmonary vasodilator
and is useful in the treatment and prevention of HAPE. Sildenafil and
tadalafil may also be useful in the prevention of HAPE. These phos- Answer: C. This patient has an acute neurologic emergency,
phodiesterase–5 inhibitors increase cyclic guanosine monophosphate most likely high- altitude cerebral edema (HACE) until proven
availability and result in pulmonary vasodilation. As a non-cardiac otherwise. Any hard neurologic finding (ataxia, slurred speech,
form of pulmonary edema, neither nitroglycerin or furosemide has a focal neurologic deficits, seizure, or altered mentation) at altitude
role in the treatment of HAPE. is suggestive of HACE, but acute CVA and other malignant etiol-
7. Which of the following signs or symptoms is specific for high- ogies must be considered. Unlike acute mountain sickness (AMS)
altitude cerebral edema (HACE) as opposed to acute mountain or high-altitude pulmonary edema (HAPE), which can occasion-
sickness (AMS) or high-altitude pulmonary edema (HAPE)? ally be treated without descent, descent is mandatory for all cases
a. Ataxia of HACE. Dexamethasone should also be given to all patients with
b. Dizziness HACE.
c. Dyspnea at rest 9. You are the doctor on a group expedition to Denali and are cur-
d. Headache rently at an elevation of 10,000 feet. One of the members of the
e. Vomiting group experiences headache, lightheadedness, and nausea. You
Answer: A. Ataxia, seizures, slurred speech, focal neurologic defi- diagnose him with acute mountain sickness (AMS) and treat him
cits, and altered mentation are all concerning to HACE and require with acetazolamide and oxygen. He wants to know when he can
immediate descent and treatment. Dizziness, headache, and vomiting continue climbing the mountain. What do you tell him?
can all be seen in AMS. Dyspnea at rest is nearly universal in HAPE. a. You may ascend if your oxygen saturation (as measured by pulse
8. You are the doctor on a group expedition to Denali. Recently, some oximetry) is higher than 90%.
of the climbers have complained of headaches, dyspnea, nausea, b. You may ascend when all your symptoms resolve.
and difficulty sleeping. At breakfast this morning, one of the climb- c. You may never ascend any higher than this altitude.
ers, a 28-year-old female, is noted to have difficulty speaking and d. You may not ascend any higher on this expedition but may try
some slurred speech, which she attributes to a restless night’s sleep. again in 1 or 2 months.
What treatment is mandated for this patient? e. You may only sleep 1000 feet higher than your current altitude.
a. 24 hours of rest before further ascent Answer: B. Management of AMS must adhere to the axiom, “After
b. Acetazolamide the symptoms of altitude illness occur, further ascent to a higher sleep-
c. Descent to lower altitude ing altitude is contraindicated.”
d. Nifedipine
e. Ibuprofen
Drowning 133
decision to cease care must be made on a case-by-case basis, but initial
KEY CONCEPTS
treatment of asystole should be begun in drowning victims following
• D rowning is a leading cause of death and loss of years of life with over 90% standard advanced cardiac life support (ACLS) protocols and as in this
of cases occurring in lower-and middle-income countries. Cost-effective case, administration of intravenous epinephrine. Experiments with
prevention strategies have been developed for settings where resources induction of hypothermia are ongoing in drowning victims, but the
limit treatment for drowning victims. results are not definitive.
• Significant drowning injuries induce pulmonary injury and hypoxia in pro- 3. A 16-year-old boy presents with tachypnea and coughing. Family
portion to the amount of water aspirated and the duration of submersion. members state that they were swimming in a nearby lake when
• Pulmonary and neurologic support is essential to optimize the victim’s they noticed the patient with the previously mentioned symptoms.
chance of a favorable outcome. The patient is awake and alert but in obvious distress. He has a
• Electroencephalography is used in obtunded drowning victims to assess for respiratory rate of 35 breaths per minute, with the remainder of
subclinical seizures. his vital signs within normal limits. His oxygen saturation as mea-
• No prognostic scale or clinical presentation accurately predicts long-term sured by pulse oximetry is 84% while on high-flow oxygen. Pul-
neurologic outcome after significant drowning injuries. Normal neurologic monary examination reveals diffuse rales. There is no evidence of
recovery is documented in patients despite prolonged submersions, per- trauma. In addition to intubation, which of the following is indi-
sistent coma, cardiovascular instability, and fixed and dilated pupils on cated?
presentation. a. Administer intravenous diuretics.
• Hyperventilation, corticosteroids, diuresis, barbiturate coma, and neuro- b. Administer intravenous corticosteroids.
muscular blockade do not improve outcome following resuscitation of a c. Obtain an electrocardiogram (ECG).
drowning victim. d. Perform maneuvers to remove fluid from the lungs.
• Comatose patients who have been resuscitated after reasonable submer- e. Place the patient in cervical spine precautions.
sion time regardless of rhythm should not be rewarmed above 34°C. Answer: C. Dysrhythmias are frequently associated with drown-
ing, especially when no obvious cause is found. ECGs should be
1. Which of the following is the greatest risk factor for death from performed on all drowning patients. Antibiotics, steroids, and
drowning? diuretics have all been studied in drowning victims, and none show
a. Ethanol any benefit. Cervical spine precautions are often initiated in all
b. Male gender drowning victims but are not needed unless the patient has clin-
c. Poor swimming ability ical signs of trauma or a history of motor vehicle crash, fall from
d. Saltwater drowning height, or diving into the water. Maneuvers to remove fluid from the
e. Warm water lungs (Heimlich, Patrick maneuvers) are ineffective and potentially
Answer: A. Ethanol is a major risk factor for drowning and for dangerous.
death from drowning. Ethanol is a contributing factor in up to 50% of
4. An 18-year-old man is brought to the emergency department
drowning incidents among adolescents and adults. More males drown
(ED) after submersion in his swimming pool. Per witnesses, once
than females, but male gender alone does not increase the risk of death
the patient was brought out from the water, he initially had severe
from drowning. No direct evidence exists relating swimming ability to
coughing and complained of shortness of breath. On arrival to the
risk of death from drowning. Although frequently cited in older liter-
ED, the patient denies shortness of breath and is not coughing. Vital
ature, there is no significant difference between saltwater and fresh-
signs as well as oxygen saturation are all within normal limits. Elec-
water drowning. Although case reports of survival from cold-water
trocardiogram (ECG) and chest radiographs are normal. What is
drowning exist, overall mortality is greater in cold water than in warm
the appropriate disposition of this patient?
water.
a. Admission to telemetry for 23-hour observation
2. A 13-year-old boy is brought to the emergency department (ED)
b. Admission to the general hospital for 23-hour observation
after an apparent drowning. Family and paramedics report the
c. Admission to the intensive care unit for 23-hour observation
patient was submerged for approximately 4 minutes. Bystanders d. Discharge home after observation in the ED for 8 hours
began cardiopulmonary resuscitation (CPR) immediately. Para- e. Discharge home now
medics intubated the patient and continued CPR, which has now Answer: D. Any symptomatic patients or patients with a history
been ongoing for 10 minutes. On arrival in the ED, the patient has of apnea, unconsciousness, or hypoxia should be admitted. Likewise,
a Glasgow Coma Score (GCS) of 3, unreactive pupils, no pulse, and patients with dysrhythmias or abnormal chest radiographs should be
is in asystole on the cardiac monitor. What is the most appropriate admitted. Some effects of drowning can be delayed, so asymptomatic
next step? patients should be observed for 8 hours in the ED but can be safely dis-
a. Cessation of treatment/pronounce death charged if they continue to be asymptomatic and can maintain normal
b. External cardiac pacing room air oxygen saturation. They should be discharged in the care of a
c. Induction of hypothermia responsible family member or friend.
d. Intravenous epinephrine
e. Unsynchronized defibrillation
Answer: D. Although this patient will likely have a poor outcome,
asystolic victims of drowning have a higher incidence of full neurologic
recovery than patients with asystole from other causes. Poor prognostic
indicators are age younger than 3 years old, submersion greater than
5 minutes, delay in initiation of CPR greater than 10 minutes, hypo-
thermia, severe acidosis, unreactive pupils, GCS of 3, and asystole. The
Radiation Injuries 134
KEY CONCEPTS
• P atients contaminated with radiation pose minimal risk to health care pro-
viders when appropriate precautions and decontamination procedures are
employed.
• Decontamination should not delay or impede emergency stabilization of
patients with radiation exposure.
• Tissues with greater rates of cellular division, particularly the hematopoi-
etic and gastrointestinal systems, are most radiosensitive. Children are
more radiosensitive than adults.
• Vomiting, diarrhea, and skin burns occurring shortly following radiation
exposure are predictors of severe radiation injury.
• The 48-hour absolute lymphocyte count is the most important prognostic
indicator and should be drawn on suspected radiation exposure patients.
• Most therapy is supportive with symptomatic measures except for expo-
sures involving the ingestion or inhalation of radioactive material, when
directed therapy with blocking or chelating agents may be indicated.
• Formal consultation at the hospital, regional, and national levels is avail-
able 24 hours a day and should be used for assistance when managing
patients with radiation injuries (Table 134.4).
1. Your Emergency Department receives a report of a “dirty bomb” Answer: D. In radiation incidents, the immediate risk to patients and
explosion with multiple injuries and deaths on scene. Which inju- providers from radiation injury is low. The most immediate risks are
ries would you be most likely to see? from traditional ABCD compromises. Thus, mass casualty triage and
a. Cutaneous (local) radiation injuries emergency stabilization take precedence, followed by an assessment
b. Gastrointestinal radiation injuries for radiologic contamination, and decontamination if needed. This is
c. Hematopoietic radiation injuries in contrast to chemical or biologic incidents, where decontamination
d. Neurologic radiation injuries takes precedence.
e. Soft tissue and orthopedic injuries 3. A worker in your hospital’s blood bank comes into the emer-
Answer: E. “Dirty bomb” is a term for a traditional explosive device to gency department (ED) complaining that he has been exposed
which an amount of radioactive material has been added. The greatest to radiation on an ongoing basis for approximately 1 month. He
risk of injury is from traditional blast/explosive effects. The risk of radi- reports his job duties include irradiating blood products to be
ation injury and contamination from a “dirty bomb” is extremely low. used in the hospital and that he has not been wearing protective
The small amount of radioactive material that survives the explosion clothing. He reports a burning sensation in his hands. His phys-
is concentrated in the area immediately around the explosion, where ical examination is normal. What is the appropriate course of
victims are likely killed by the blast itself. action?
2. Which of the following is the proper sequence for caring for patients a. Admit for observation
from a mass casualty incident involving potential radiologic con- b. Decontaminate the patient, admit for observation
tamination? c. Decontaminate the patient, discharge home
a. Decontamination, assess with radiation meter, emergency stabi- d. Decontaminate the patient, place on isolation, admit for obser-
lization, mass casualty triage vation
b. Assess with radiation meter, decontamination, mass casualty tri- e. Educate patient and arrange appropriate follow-up
age, emergency stabilization Answer: E. This patient has been irradiated but not contaminated.
c. Assess with radiation meter, decontamination, emergency stabi- Patients who are exposed to external beam radiation are not made
lization, mass casualty triage radioactive and are not a hazard to others. Decontamination and
d. Mass casualty triage, emergency stabilization, assess with radia- isolation are not necessary. With a normal physical examination
tion meter, decontamination and minimal symptoms, this patient can be followed as an outpa-
e. Decontamination, emergency stabilization, mass casualty triage, tient. The most important action in this instance is to educate the
assess with radiation meter patient.
4. What is the most important laboratory test to be performed 48 paralysis, dizziness, ataxia, seizures) within 24 hours of exposure indi-
hours after exposure in determining the prognosis of a patient cates a lethal dose of radiation. The hematopoietic system is the most
exposed to a significant radiation source? radiosensitive (can be affected by as little as 0.5 to 1 Gy), followed
a. Absolute lymphocyte count by the gastrointestinal system (more than 5–7 Gy). The skin will be
b. Absolute neutrophil count affected by local doses of radiation greater than 3 to 5 Gy.
c. Genetic karyotype 6. A 45-year-old man is inadvertently exposed to a radiation field
d. Platelet count of 850 mSv/hour for 4 hours while working on the clean-up of a
e. Aspartate aminotransferase nuclear power plant. In addition to general supportive care, which
Answer: A. Monitoring the kinetics of lymphocyte depletion during of the following medications may improve his survival?
the first 48 hours following a radiation exposure is very useful at esti- a. Acyclovir
mating the radiation dose received. The 48-hour absolute lymphocyte b. Colony-stimulating factor (cytokines)
count is the most important prognostic indicator. Levels greater than c. Diethylenetriaminepentaacetate (DTPA)
1200/μL indicate a clinically insignificant dose of radiation and an d. Erythropoietin
excellent prognosis. Levels less than 500/μL indicate a significant and e. Potassium iodide
possibly lethal exposure. None of the other answer choices carry prog- Answer: B. This patient’s radiation dose is sufficient to produce the
nostic significance. hematopoietic component of acute radiation syndrome (ARS). Cyto-
5. Which of the following symptoms within the first 24 hours after a kines or colony- stimulating factors have shown modest effects in
significant radiation exposure would indicate a lethal dose of radia- improving survival. Irradiated patients may develop oral herpes, and
tion? acyclovir can be used for treatment but does not affect survival. Ane-
a. Bruising mia is present in most irradiated patients but is typically not clinically
b. Gait ataxia significant, and erythropoietin has no role. DTPA is a chelating agent
c. Nausea and vomiting that can be used for patients with internal exposure to plutonium
d. Skin burns or transuranics. Potassium iodine is useful if a patient is exposed to
e. Shortness of breath radioactive iodine, because it will compete with the radioactive iodine
Answer: B. The neurologic system is the most resistant to radiation for uptake into the thyroid gland, resulting in overall less radioactive
injury, requiring at least a 10 Gy dose to be affected. The development uptake.
of neurologic symptoms (such as altered mental status, focal weakness/
SECTION TWO Toxicology
KEY CONCEPTS muscarinic receptor blockade are utilized for clinical purposes includ-
ing pupillary dilation, antispasmodics, sleep aids, treatment of motion
• A nticholinergic (antimuscarinic) intoxication is common and can occur as a sickness, allergic reactions, drying of airway secretions, reactive air-
result of ingestion of a variety of plants and drugs, both prescription and way disease, treatment of bradycardia, treatment of Parkinsonism,
over-the-counter. and the management of urinary incontinence and bladder spasm. The
• Most patients with antimuscarinic toxicity do well with supportive care and agents that most commonly precipitate antimuscarinic toxicity, such
observation. as H1 antihistamines and some antipsychotics, often affect several
• Antimuscarinic syndrome varies in clinical presentation with either periph- neurotransmitters and receptor systems in addition to antagonism at
eral or central manifestations predominating. muscarinic receptors. This may complicate the clinical presentation
• Central antimuscarinic delirium is most effectively treated with physostig- and some clinical symptoms may be unique to the specific etiologic
mine. agent (Box 140.1).
• Contraindications to physostigmine administration include narrow angle Antimuscarinic toxicity has both central and peripheral manifes-
glaucoma, atrioventricular (AV) blockade, bradycardia, and seizure precipi- tations (Fig. 140.1). Peripheral muscarinic antagonism causes tachy-
tated by overdose. cardia, hypertension, hyperthermia, mydriasis, dry mouth, lack of
sweating, skin flushing, decreased bowel motility, and urinary reten-
tion. Central nervous system blockade of muscarinic receptors may
PRINCIPLES OF TOXICOLOGY
produce delirium characterized by confusion, mumbling speech, agi-
Overview tation, hallucinations, hand picking gestures, myoclonus, tremor, and
Anticholinergic agents cause toxicity through inhibition of muscarinic, coma. Manifestations of the toxidrome are frequently incomplete and
nicotinic, parasympathetic, or sympathetic acetylcholine receptors. either peripheral or central components may predominate depend-
Nicotinic receptor inhibition and ganglionic acetylcholine inhibition ing upon which antimuscarinic agent is involved, the dose, and the
at parasympathetic and sympathetic locations are covered in Chap- individual patient (Table 140.1). In one large series of antimuscarinic
ter 152. This chapter will focus on antimuscarinic effects and toxicity. poisoning, only 28% of patients had all three classic manifestations of
The terms anticholinergic and antimuscarinic are used synonymously tachycardia, dry skin/axilla, and mydriasis. Therefore, most patients
though the mechanism of toxicity is more accurately described by the will not present with all of these features.1 Duration of toxicity may be
term “antimuscarinic,” and thus that term will be used in this chapter. prolonged (18–72 hours) depending on the specific agent, dose, and
Antimuscarinic effects are due to competitive inhibition of acetyl- the pharmacologic effect of delayed gastric emptying.1
choline at muscarinic receptors. Muscarinic receptors are found on
peripheral postganglionic cholinergic nerves in smooth muscle (intes- DIFFERENTIAL DIAGNOSES
tinal, bronchial, and cardiac), the secretory glands (salivary and sweat),
the ciliary body of the eye, and the central nervous system (CNS). The differential diagnosis of altered mental status is broad. Consider
Antimuscarinic agents have been used medicinally from antiquity to antimuscarinic toxicity when there is a history of exposure or if there
the present day. Mandrake plant remains were found in the coffin of Tut- are physical exam findings consistent with the antimuscarinic tox-
ankhamen, the Old Testament of the Bible references its use as an aphro- idrome (Box 140.2).
disiac, and antimuscarinic plants were used as anesthetics in Greek and
Roman settlements in the 1st century. Atropine, hyoscyamine, and scopol- DIAGNOSTIC TESTING
amine are naturally occurring tertiary amine antimuscarinic agents that
remain in wide clinical use today. The tertiary amine structure allows the Laboratory
agent to cross the blood-brain barrier; therefore, these agents may precip- Patients with mild toxicity, a reliable history of exposure, and symp-
itate CNS toxicity. Quaternary amine antimuscarinic agents, such as the toms consistent with antimuscarinic toxicity do not require specific
anti-sialagogue glycopyrrolate, have been developed to mitigate CNS side laboratory testing. Patients with an unclear history of exposure, other
effects due to their limited ability to cross the blood-brain barrier, though potential etiologies, moderate to severe toxicity or hyperthermia
mild delirium may occur in the setting of a large overdose. should be evaluated for causes of altered mental status and end-organ
toxicity including: serum glucose, electrolytes, cardiac biomarkers,
renal function, creatinine kinase to evaluate for rhabdomyolysis, and
CLINICAL FEATURES
acid-base status. Patients with an overdose of unclear history should
Over 600 compounds contain antimuscarinic activity, including pre- be evaluated for co-ingestion since antimuscarinic agents are often for-
scription drugs, over-the-counter drugs, and plants. The effects of mulated with other potentially toxic agents. Serum acetaminophen and
1872
REFERENCES delirium: a prospective study from a regional poison center. Clin Toxicol.
2019;57:50–55.
1. Dawson AH, Buckley NA. Pharmacological management of anticholinergic 4. Watkins JW, Schwarz ES, Arroyo-Plasencia AM, Mullins ME. Toxicology
delirium -theory, evidence and practice. Br J Clin Pharmacol. 2016;81: investigators consortium i. The use of physostigmine by toxicologists in
516–524. anticholinergic toxicity. J Med Toxicol. 2015;11:179–184.
2. Arens AM, Shah K, Al-Abri S, Olson KR, Kearney T. Safety and 5. Nguyen TT, Armengol C, Wilhoite G, Cumpston KL, Wills BK. Adverse
effectiveness of physostigmine: a 10-year retrospective review. Clin Toxicol. events from physostigmine: an observational study. Am J Emerg Med.
2018;56:101–107. 2018;36:141–142.
3. Boley SP, Olives TD, Bangh SA, Fahrner S, Cole JB. Physostigmine is
superior to non-antidote therapy in the management of antimuscarinic
C H A P T E R 1 4 0 : Q U E S T I O N S A N D A N S W E R S
1. A 21-year-old man presents after drinking an “herbal tea” with some Answer: A. This patient is experiencing many of the signs and symp-
friends. He reports visual hallucinations. He has a resting tachycar- toms of antimuscarinic syndrome. However, pure antimuscarinics
dia and a mildly elevated temperature. On physical examination, rarely if ever cause cardiac dysrhythmias (other than sinus tachycar-
he is noted to have dry mucous membranes, dry and flushed skin, dia). Tricyclic antidepressants (TCAs) frequently cause antimuscarinic
and absent bowel sounds. In addition to certain plants, which of the signs and symptoms but also cause dysrhythmias. Although selective
following medications can also cause these symptoms? serotonin reuptake inhibitors, stimulants, and lithium can all cause
a. Amiodarone similar signs and symptoms, electrocardiographic abnormalities such
b. Clonidine as those described here are rare.
c. Diphenhydramine 4. Which diagnostic test should be performed in almost all patients
d. Lidocaine presenting with the antimuscarinic syndrome?
e. Morphine a. Arterial blood gas analysis
Answer: C. This patient is experiencing the antimuscarinic toxidrome. b. Computed tomography (CT) scan of the brain
In addition to the signs and symptoms described here, patients may also c. Electrocardiography
have mydriasis and bladder distention. Mental status can be agitated or d. Electroencephalography
depressed. Myoclonus or choreoathetoid movements can also be seen. e. Urine drug screen
Amiodarone can cause hypothyroidism or hyperthyroidism and skin Answer: C. Patients with a clear presentation and mild symptoms do
discoloration, as well as several other long-term effects. Clonidine can not necessarily require any diagnostic evaluation. However, patients
cause dry mouth, drowsiness, bradycardia, and hypotension. Lidocaine with more severe symptoms should have measurements of serum elec-
can cause headaches, dizziness, confusion, tinnitus, and tremor, as well trolytes, renal function, creatine kinase, and glucose concentration
as bradycardia and hypotension. Morphine can cause respiratory cere- performed. Electrocardiography is most helpful because cyclic antide-
bral depression, as well as miosis, bradycardia, and hypotension. pressants are a common cause of antimuscarinic symptoms and can
2. Many signs and symptoms of the antimuscarinic syndrome are simi- cause fatal cardiac dysrhythmias. Arterial blood gas analysis might
lar to those of other toxic syndromes, including the sympathomimetic be helpful if the patient has respiratory depression. Head CT might
syndrome, serotonin syndrome, and neuroleptic malignant syndrome. be indicated in patients with altered mental status of unknown cause.
Which of the following antimuscarinic findings is most likely to distin- Electroencephalography would be indicated only if there is a suspicion
guish the antimuscarinic syndrome from the other syndromes listed? of unrecognized seizures. Urine drug screens are almost never helpful
a. Altered mental status in determining treatment, especially in the case of antimuscarinic syn-
b. Altered movements drome because they will not detect most of the medications responsible
c. Dry skin for this syndrome.
d. Fever 5. What is the best initial treatment of hyperthermia in patients with
e. Mydriasis antimuscarinic syndrome?
Answer: C. All the other syndromes often have some degree of diapho- a. Acetaminophen
resis. Hyperthermia, altered mental status, and mydriasis can occur in b. Cooling blankets
all the named syndromes. Myoclonus can occur in the antimuscarinic c. Dantrolene
syndrome, tremor in the serotonin syndrome, and rigidity in the neu- d. Evaporative cooling
roleptic malignant syndrome. e. Physical restraints
3. A 31-year-old woman presents with altered mental status after ingest- Answer: D. Evaporative cooling is the most effective and noninva-
ing an unknown quantity of an unknown medication. Her vital signs sive way to decrease temperature. Death has occurred because of
are significant for tachycardia and hyperthermia. Her physical exam- untreated hyperthermia in patients with antimuscarinic syndrome.
ination reveals mydriasis, dry mucous membranes, dry skin, decreased Antipyretics such as acetaminophen are ineffective at reducing tem-
bowel sounds, and hypotension. Her electrocardiogram (ECG) reveals perature because hyperthermia is not “fever.” Dantrolene is useful in
a wide QRS complex, at 130 msec, and prolonged QT interval. Which malignant hyperthermia but has no role in hyperthermia of other
of the following medications is associated with this toxidrome? causes. Cooling blankets are ineffective. Physical restraints are likely
a. Amitriptyline to worsen the problem and to increase the risk of rhabdomyolysis
b. Dextroamphetamine and myoglobinuric renal failure. If a patient is dangerously agitated
c. Diphenhydramine from antimuscarinic toxicity, physostigmine is the agent of choice to
d. Fluoxetine decrease agitation, muscle activity, and related metabolic activity that
e. Lithium contribute to hyperthermia.
1875.e1
1875.e2 PART IV Environment and Toxicology
C H A P T E R 1 4 0 : Q U E S T I O N S A N D A N S W E R S — c o n t ’ d
6. Which of the following medications crosses the blood-brain barrier 7. Which of the following is a contraindication to physostigmine use
and is potentially useful in the treatment of antimuscarinic syn- in a patient with antimuscarinic syndrome?
drome? a. Altered mental status
a. Edrophonium b. Bradycardia and atrioventricular (AV) blockade
b. Metoclopramide c. Coexisting myasthenia gravis
c. Neostigmine d. Hyperthermia
d. Physostigmine e. Bladder distention and urinary retention
e. Pyridostigmine Answer: B. Physostigmine is an acetylcholinesterase inhibitor that is
Answer: D. Metoclopramide is an antiemetic and prokinetic medi- useful to reverse the effects of antimuscarinic medications. However, it is
cation that has no role in antimuscarinic syndrome. All of the other contradicted with narrow angle glaucoma, AV blockade, bradycardia, and
agents are acetylcholinesterase inhibitors, but only physostigmine seizures due to the causal overdose. The main benefit of physostigmine is
crosses the blood-brain barrier and so it is the only drug that can to reverse the altered mental status and agitation caused by the antimusca-
reverse the central and peripheral effects of antimuscarinic medi- rinic medication. Physostigmine is occasionally used to treat myasthenia
cations. However, physostigmine can cause cardiac dysrhythmias gravis. Hyperthermia and urinary retention can occur as symptoms of the
and thus should be used carefully in patients with bradycardia or antimuscarinic syndrome, and although neither is directly treated with
AV block. physostigmine, they are not a contraindication to its use.
141 Antidepressants
KEY CONCEPTS
• A lthough rarely used for depression, MAOIs are used in the treatment of
Parkinson disease.
• Because serious symptoms can occur after a lengthy latent period, patients
with reported MAOI overdose should be admitted for 24 hours, regardless
of symptoms. Toxicity is characterized by tachycardia, hypertension, and
CNS changes, and later cardiovascular collapse.
• The primary manifestations of TCA toxicity are seizures, tachycardia,
hypotension, and intraventricular conduction delay. IV sodium bicarbonate
should be administered for QRS prolongation.
• SSRIs are relatively benign in overdose and generally managed with sup-
portive care alone.
• SNRI ingestions can result in seizures, tachycardia, and occasionally intra-
ventricular conduction delay.
• The hallmark feature of serotonin syndrome is lower extremity rigidity with
spontaneous or inducible clonus, especially at the ankles.
• Serotonin syndrome is primarily treated with supportive care, including dis-
continuation of the offending agent, and benzodiazepines.
1. A 42-year-old woman presents after ingesting an unknown num- occur within 30 to 60 minutes of ingestion. If the patient has tachy-
ber of pills in a suicide attempt. She does not know the name of cardia or seizure or decreased level of consciousness at 6 hours, she
the medication she ingested but knows it is an antidepressant. She should be admitted for medical observation. If any of these pills were a
denies taking any co-ingestants. Currently, she has no complaints; monoamine oxidase inhibitor (MAOI), however, the patient should be
her vital signs, physical examination findings, and electrocardio- admitted to the hospital for 24 hours.
gram (ECG) are normal. To receive a psychiatric evaluation, she 2. After sinus tachycardia, what is the most common electrocardio-
must be transferred off-site. How long should she be observed in graphic abnormality seen in cyclic antidepressant overdose?
the emergency department before transfer to a psychiatric facility? a. Left bundle branch block
a. 1 hour b. PR prolongation
b. 2 hours c. QRS prolongation
c. 6 hours d. QT prolongation
d. 12 hours e. Right bundle branch block
Answer: C. The most dangerous class of antidepressants is the cyclic Answer: C. After sinus tachycardia, QRS prolongation of more than
antidepressants. Typically, peak plasma concentrations and therefore 100 milliseconds is the most common specific finding and results from
peak effect occur in 2 to 4 hours. However, in overdose, the anticholin- sodium channel–blocking effects of the cyclic antidepressant. Pro-
ergic effects of these agents may result in delayed gastric emptying and longed PR and QT intervals, as well as a right bundle branch block, can
delay peak absorption to 6 hours. Most serious signs and symptoms also occur but are less common.
3. Which of the following treatment options would be the most appro- overdoses; however, myoclonus is unique to the serotonin syndrome.
priate to consider in the awake, asymptomatic patient who pres- Additional historical features consistent with the serotonin syndrome
ents within 1 hour of a large overdose of a tricyclic antidepressant are the fact that the patient is taking an antidepressant, possibly a selec-
(TCA)? tive serotonin reuptake inhibitor (SSRI), and has likely added an over-
a. Activated charcoal the-counter “cold” medication. Many of these medications contain
b. Endotracheal intubation dextromethorphan, which also decreases serotonin reuptake and can
c. Gastric lavage precipitate the serotonin syndrome in patients taking SSRIs.
d. Physostigmine 6. A 24-year-old man presents after taking an overdose of his antide-
e. Sodium bicarbonate pressant. He does not know the name of the drug. He has no com-
Answer: A. Physostigmine should never be prophylactically admin- plaints, and his vital signs and physical examination findings are
istered and its use in TCA overdose is generally considered contraindi- normal. Soon after completing your evaluation, the patient experi-
cated, particularly in patients with bradycardia, AV block, and seizures ences a tonic-clonic seizure. Which of the following antidepressants
related to the overdose. Sodium bicarbonate should not be prophylac- is most likely to produce seizures without other symptoms of severe
tically administered either, but it should be administered as an intra- toxicity?
venous (IV) bolus for patients with intraventricular conduction delay a. Amitriptyline
and QRS widening. Endotracheal intubation is indicated for patients b. Bupropion
with significant central nervous system (CNS) depression who are not c. Fluoxetine
able to protect their airway. Gastric lavage is not indicated. Activated d. Phenelzine
charcoal can be considered in an awake individual presenting within e. Trazodone
one hour of ingestion. It should not be “forced” on someone who will Answer: B. Bupropion can induce seizures even at therapeutic lev-
not voluntarily drink it (e.g., by placement of a nasogastric tube for els. Other adverse effects include tachycardia, tremulousness, halluci-
the purpose of administering charcoal) because of the increased risk of nations, and QRS prolongation. Cyclic antidepressants (amitriptyline),
aspiration and subsequent charcoal pneumonitis. selective serotonin reuptake inhibitors (SSRIs; fluoxetine), and mono-
4. A 23-year-old man presents after an ingestion of a cyclic antide- amine oxidase inhibitors (MAOIs; phenelzine) can also cause seizures
pressant. His initial vital signs are normal. During your initial but less frequently than bupropion and usually with other symptoms of
evaluation, the patient begins to seize. Which agent should be serious intoxication such as central nervous system (CNS) depression
administered first? or QRS prolongation.
a. Lorazepam 7. A patient is prescribed a new medication that he takes each night.
b. Phenobarbital After 4 days, he is drowsy and noted to have orthostatic hypoten-
c. Phenytoin sion, nausea, vomiting, and priapism. What medication is most
d. Propofol likely involved?
e. Valproic acid a. Amitriptyline
Answer: A. Lorazepam or any other benzodiazepine is the first- b. Bupropion
line treatment of toxin-induced seizures. Intravenous phenytoin can c. Fluoxetine
increase the incidence of ventricular tachycardia and is not generally d. Phenelzine
effective in controlling toxin- induced seizures. Phenobarbital and e. Trazodone
propofol are effective but take longer to give and typically are used after Answer: E. Trazodone and nefazodone may cause orthostatic
benzodiazepines. Valproic acid is not likely to be effective. hypotension and lethargy. Priapism is a relatively unique complication
5. A 57-year-old woman presents with altered mental status. A friend of trazodone and is more common with therapeutic use rather than in
states that the patient takes antidepressant medications and has acute overdose.
recently been complaining of symptoms of an upper respiratory 8. A patient presents after a substantial accidental overdose of her
tract infection. The patient is noted to have a temperature of 39.2°C monoamine oxidase inhibitor (MAOI), which she confused with
and a pulse of 135 beats/minute. Otherwise, her vital signs are her megavitamin therapy. There is no need for psychiatric evalua-
within normal limits. On examination, she has a tremor, myoclo- tion. She is asymptomatic and has normal vital signs and a normal
nus, and diaphoresis. Which of the following is the most consistent physical examination. What is the appropriate disposition?
with this presentation? a. Admit to intensive care unit for a minimum of 24 hours of
a. Anticholinergic syndrome observation
b. Cocaine intoxication b. Admit to ward for a minimum of 24 hours of observation
c. Cyclic antidepressant overdose c. Discharge home
d. Neuroleptic malignant syndrome (NMS) d. Observe for 6 hours, then discharge home
e. Serotonin syndrome e. Observe for 12 hours, then discharge home
Answer: E. Symptoms of serotonin syndrome include altered Answer: A. All patients who overdose on MAOIs should be admit-
mental status, agitation, ataxia, diaphoresis, diarrhea, hyperreflexia, ted for at least 24 hours of observation because symptom onset is often
hyperthermia, myoclonus, shivering, and tremor. Many of the symp- delayed. In addition, the effects of overdose may be severe and require
toms are similar to symptoms caused by NMS and sympathomimetic aggressive therapy.
142 Cardiovascular Drugs
KEY CONCEPTS
• D igoxin toxicity is often occult and should be considered in any patient who
is on digoxin and presents with gastrointestinal or visual disturbance and a
new dysrhythmia, conduction disturbance or hemodynamic instability, par-
ticularly in the setting of ingestion of a natural weight-loss supplement.
• Digitalis Fab is the specific antidote for digoxin toxicity and is dosed based
on chronicity of poisoning; most patients require only 1 (chronic poisoning)
or 2 (acute poisoning) vials. If full reversal is needed for patients in extre-
mis, dosing is based on total body load of digoxin, not by patient weight.
• Indications for digitalis Fab include progressive and hemodynamically sig-
nificant bradydysrhythmias and serum potassium >5.0 mEq/L, as summa-
rized in Box 142.4. Of note, Fab therapy should be used before pacing or
antidysrhythmic drugs.
• Hyperkalemia in acute digoxin toxicity is best treated with Fab fragments.
Conventional treatment as for any other cause of hyperkalemia is also
appropriate when Fab fragments are not immediately available. Hyperkale-
mia in chronic poisoning is likely multifactorial and should be treated with
Fab fragments followed by usual hyperkalemia treatments as needed.
1. A 72-year-old man presents with a chief complaint of nausea. He 2. In a patient with a known digoxin overdose, which of the following
also complains of blurred vision and general weakness. His vital is an indication for administration of digitalis antibody fragments?
signs and physical examination are normal. He has a past medi- a. Acute ingestion of 10 mg of digoxin
cal history significant for myocardial infarction, congestive heart b. Atrial fibrillation with rapid ventricular response
failure, hypertension, and diabetes. He states that he takes all pre- c. First-degree heart block
scribed medications regularly and just took all of his medications d. Ingestion of oleander (Nerium oleander)
as prescribed approximately 1 hour ago. Reviewing his medication e. Serum potassium greater than 5 mEq/L
list, you note that one of his medications is digoxin. Which of the Answer: E. Hyperkalemia is an indication for treatment with digitalis anti-
following should be done? body fragments. Other indications include severe ventricular dysrhyth-
a. Administer oral potassium. mias; hemodynamically significant bradydysrhythmias unresponsive to
b. Give digoxin-specific Fab fragment antibodies now. atropine; rapidly progressive rhythm disturbances or rising potassium
c. Give digoxin-specific Fab fragment antibodies only if he has an level; co-ingestion of cardiotoxic drugs such as beta-blockers, calcium
abnormal electrocardiogram (ECG). channel blockers, or cyclic antidepressants; or ingestion of a plant known
d. Obtain a serum digoxin level in 6 hours. to contain cardiac glycosides in the setting of severe dysrhythmias.
e. Obtain a serum digoxin level now. 3. A 27-year-old man presents after ingesting 8 mg of digoxin in a
Answer: D. The steady-state (as opposed to peak) digoxin level is most suicide attempt. He has ventricular tachycardia on his electrocar-
closely correlated with toxicity. Peak level is reached in 1.5 to 2 hours, diogram (ECG). His blood pressure is 93/54 mm Hg. Laboratory
whereas steady-state levels are not reached until 6 to 8 hours after results are not available. You decide to treat with digitalis antibody
ingestion. As long as a patient is stable and a chronic ingestion is sus- fragments. How many vials would bind the entire ingestion?
pected, digoxin levels taken soon after ingestion are not helpful. Other a. 2 vials
diagnostic studies should be undertaken while waiting for the digoxin b. 4 vials
level, including, but not limited to, an electrocardiogram (ECG) and c. 8 vials
serum electrolyte determination. d. 16 vials
e. 20 vials
Answer: D. Each vial contains enough digitalis antibody fragments
to neutralize 0.5 mg of digoxin; therefore, the patient should be given
16 vials of Fab fragments to fully treat his ingested dose.
4. A 5-year-old girl presents after taking some of her grandmother’s 6. A patient presents after an apparent overdose of diltiazem. Which
heart medications. She has sinus bradycardia and a slightly low of the following findings would lead you to suspect a co-ingestant
blood pressure. The remainder of her vital signs and physical in addition to the calcium channel blocker?
examination are nonspecific. A bedside glucose test reveals hyper- a. Atrioventricular (AV) block on electrocardiogram (ECG)
glycemia. The child is not responsive to calcium therapy. The next b. Hyperglycemia
therapeutic intervention that is likely to treat both the endocrine c. Metabolic acidosis
and cardiovascular pathology would be: d. Prolonged QRS complex on ECG
a. Dobutamine e. Pulmonary edema
b. Epinephrine Answer: D. Prolonged QRS and QT intervals are generally not seen in
c. High-dose insulin (HDI) calcium channel blocker overdose and should prompt the search for
d. Intravenous fat emulsion (IFE) co-ingestants. One exception is the calcium channel blocker bepridil,
e. Glucagon which can cause QRS or QT prolongation, though bepridil is no longer
Answer: C. Calcium channel blocker toxicity may result in bradycardia sold in the United States. All the other listed abnormalities are typical
and hypotension, much like beta-blocker toxicity. Unlike beta-blocker of calcium channel blocker overdose.
toxicity, which can cause hypoglycemia, calcium channel blockers can 7. Treatment for beta-blocker overdose is most similar to the treat-
cause hyperglycemia via blockade of calcium channels on beta islet ment for overdose from which other class of medications?
cells. HDI therapy treats calcium channel blocker toxicity via increas- a. Anticholinergics
ing inotropy, as well as normalizing glucose. b. Calcium channel blockers
5. A 60-year-old man has taken an overdose of his beta-blocker med- c. Cyclic antidepressants
ication. He is awake but lightheaded and has a pulse of 50 beats per d. Digoxin
minute and a blood pressure of 92/60 mm Hg. Which of the follow- e. Nitrates
ing is the most appropriate initial treatment? Answer: B. Symptoms from overdose of calcium channel blockers
a. Atropine, crystalloid fluids, calcium salts, glucagon and beta-blockers are similar, as are the therapeutic strategies. Severe
b. Atropine, crystalloid fluids, insulin anticholinergic toxicity may respond to physostigmine administration,
c. Atropine, norepinephrine, glucagon cyclic antidepressants to intravenous (IV) sodium bicarbonate boluses,
d. Norepinephrine, crystalloid fluids, insulin digoxin poisoning to antibody Fab fragments, and nitrate toxicity to
e. Norepinephrine, glucagon, insulin methylene blue therapy.
Answer: A. Atropine, crystalloid fluids, calcium salts, and glucagon are 8. Which of the following drugs is most likely to kill a toddler with
the initial treatments of choice for beta-blocker toxicity. If these agents ingestion of a single tablet?
are ineffective, high-dose insulin (HDI) is reasonable as next-line ther- a. Atenolol
apy for cardiogenic shock. If HDI is ineffective, catecholamines are a b. Chlorthalidone
reasonable option. The catecholamine choice may vary depending on c. Lisinopril
the type of shock the patient is in, and the dose needed for a response d. Nitroglycerin
may be greater than for other conditions. e. Verapamil
Answer: E. Verapamil and other calcium channel blockers can kill
a toddler with the ingestion of a single tablet. Appropriate medical
treatment, however, is extremely effective, and very few children die
from calcium channel blocker overdose. Although beta-blockers have
the potential to cause serious toxicity, propranolol is the beta-blocker
most likely to cause serious toxicity. Educating parents and caregivers
of young children about the potential effects of these medications is
important.
143 Caustics
KEY CONCEPTS
• H ealth care workers caring for patients with caustic exposures should
adhere to universal precautions to prevent additional exposure.
• All symptomatic patients should undergo endoscopy and be should be
observed for at least 24 hours.
• Asymptomatic patients can undergo endoscopy in the emergency depart-
ment or be discharged with close follow-up monitoring.
• Gastric emptying or GI decontamination is not indicated for the majority of
caustic ingestions.
• Concentration and pH are the most important characteristics of a substance
to predict esophageal and gastric injury.
• Button batteries lodged in the airway or esophagus require endoscopic
retrieval.
1. A patient presents after the intentional ingestion of hydrochloric 3. A patient presents after an intentional caustic ingestion. She com-
acid (HCl). He complains of mouth, throat, and chest pain, as well plains of hoarseness, with mouth, throat, and chest pain. Burns are
as painful swallowing and nausea. His vital signs are normal. Phys- present on her lips and oral mucosa and she is drooling. Her vital
ical examination reveals oral burns without edema. The remainder signs are normal, as is the remainder of her physical examination.
of the examination is normal. You decide that in addition to psychi- Which of the following is the most appropriate treatment?
atric consultation, the patient should have upper endoscopy. What a. Administer 500 mL water orally
is the best time for the patient to have the endoscopy? b. Administer intravenous Solu-Medrol
a. Immediately c. Endotracheal intubation
b. In 2 to 4 hours d. Obtain electrocardiogram
c. In 4 to 12 hours e. Upper endoscopy
d. In 12 to 24 hours Answer: C. Early intubation is indicated if there is any evidence
e. In 2 or 3 days of airway compromise, such as hoarseness, throat pain, drooling,
Answer: D. The ideal time for endoscopy is 12 to 24 hours. Endos- or edema. Because edema and secretions can both increase rapidly
copy done too soon may miss the extent or depth of injury, whereas and can make intubation difficult or even impossible, preparations
endoscopy after 24 hours is actually more likely to cause perforation should be made for a difficult airway. Fluids for oral dilution should
because the wounds have softened. All patients with signs or symptoms not be given if the patient has difficulty swallowing. Corticosteroids
of strong acid ingestion as well as patients with intentional ingestion have been studied to decrease the incidence of stricture formation,
should have endoscopy performed. but evidence for their benefit is lacking and serious side effects can
2. A patient presents immediately after the ingestion of bleach. The occur. With the exception of hydrofluoric acid, an electrocardiogram
patient is awake and alert and complaining only of mouth pain. His is not routinely needed for caustic ingestions, and endoscopy should
vital signs and physical examination findings are normal. You con- be performed 12 to 24 hours after the ingestion and after the airway
sider having the patient drink fluids to dilute the bleach. Which of has been secured.
the following statements regarding this therapy is true? 4. What empirical treatment is indicated to prevent systemic toxicity
a. Dilution is beneficial only if it is done very soon after ingestion. from hydrofluoric acid ingestions?
b. In cases of alkali ingestions, dilution with a mild acid such as a. Calcium chloride
acetic acid is best. b. Magnesium chloride
c. Large volumes of fluid should be used. c. Potassium chloride
d. Milk should always be used instead of water. d. Sodium bicarbonate
e. Patients should also be encouraged to eat solids. e. Sodium chloride
Answer: A. Dilution, if it is done at all, should be done early because Answer: A. Calcium chloride is indicated in significant hydroflu-
injuries from caustics occur almost immediately. Water and milk are oric acid exposures. Although hydrofluoric acid is a weak acid, the
equally beneficial and are the agents of choice. Weak acids or alkalis fluoride ion is extremely electronegative and will bind with multi-
should never be used for dilution, because they can cause thermal reac- ple cations, specifically calcium and magnesium. Profound hypo-
tions that worsen the injury. Small volumes up to approximately 500 calcemia is responsible for most deaths from hydrofluoric acid
mL should be used. Solids are not beneficial and can complicate the exposure and can occur before a serum calcium concentration can
situation and increase the risk of aspiration. be measured.
5. A 3-year-old boy presents after swallowing a button battery. What is d. Obtain electrocardiogram
the most appropriate management? e. Upper endoscopy
a. Endoscopic removal Answer: E. This patient is high risk for significant esophageal or
b. Inpatient observation gastric injury due to the self-harm attempt. Several studies evaluating
c. Radiograph to assess anatomic location for the presence or absence of oropharyngeal burns as a predictor of
d. Surgical removal distal esophageal of gastric injury had found these finding to be poorly
e. Whole-bowel irrigation predictive. Thus, endoscopy should be a standard diagnostic tool used
Answer: C. Outpatient observation is warranted for button batteries in the management of intentional caustic ingestions.
that are located in the stomach or intestines, which can be assessed 7. A 45-year-old female presents after an intentional caustic ingestion.
by plain radiographs. Batteries lodged in the esophagus require endo- She has severe chest pain, and is tachycardic, stridulous, drooling,
scopic removal. Examination of the stool for passage of the battery is and hoarse. She has a normal blood pressure. Due to concerns about
recommended. If it is not passed in one week, repeated radiographs her airway, she is intubated without difficulty. Which of the following
should be obtained. Inpatient observation is not needed as long as close diagnostic imaging modalities is most appropriate for this patient?
follow-up can be ensured. Surgical removal and whole-bowel irrigation a. Plain chest radiograph
are not beneficial and are potentially deleterious. b. Ultrasound
6. A patient presents after a self-harm attempt with 3% household c. MRI chest
bleach (sodium hypochlorite). The patient has normal vital signs d. CT chest
and is asymptomatic. His physical examination is unremarkable. e. Contrast esophagram
Which is the most appropriate treatment? Answer: D. CT scanning is considerably more sensitive than radiogra-
a. Administer 500 mL water orally phy and ultrasound for detecting hollow viscus perforation, for which
b. Administer intravenous Solu-Medrol the patient is at high risk. MRI does not add significant benefit over CT
c. Endotracheal intubation and typically takes longer to obtain.
144 Cocaine and Other Sympathomimetics
KEY CONCEPTS
• E xcessive use of stimulants can lead to sympathomimetic toxicity, mani-
festing as tachycardia, hypertension, mydriasis, diaphoresis, hyperthermia,
hyperreflexia, and agitation. If untreated, sympathomimetic toxicity can
lead to seizures, coma, and death.
• Administration of benzodiazepines is the key therapeutic intervention for
sympathomimetic toxicity. Ketamine, olanzapine, and butyrophenones are
alternative adjuncts if the cause of severe agitation is not clearly sympath-
omimetic toxicity.
• Worsening hyperthermia portends imminent death. Reduce body tempera-
ture rapidly by external cooling, sedation, and, if needed, paralysis.
• Anti-hypertensives are adjuncts to benzodiazepines. Antihypertensives do
not treat as many aspects of sympathomimetic toxicity as do benzodiaz-
epines, however, and short-acting antihypertensive agents (e.g., phentol-
amine, nitroglycerin, nicardipine, clevidipine) are preferred.
• Wide-complex rhythms secondary to stimulants (cocaine, bupropion) may
respond to intravenous sodium bicarbonate administration.
• Cocaine body packers who develop toxicity need emergent surgical inter-
vention to limit bowel necrosis and life-threatening sequelae caused by
leaking packets.
• Novel psychoactive substances (e.g., MDMA, bath salts) combine stim-
ulant, hallucinogenic, and psychedelic effects. They generally produce
longer-lasting and more intense effects than cocaine or amphetamine.
• Screen for hyponatremia in patients with sympathomimetic or serotonin
toxicity.
• If the clinical presentation does not fit the history (e.g., ingested cocaine
but hypoxic and somnolent), consider a contaminant or alternate cause. Con-
taminants change rapidly and vary by geographic region. Consult a medical
toxicologist or regional poison control center.
1. Which unique life-threatening electrolyte abnormality is seen with Answer: e. Hyponatremia can occur with MDMA use and can be severe and
the use of N-methyl-3,4-methylenedioxyamphetamine (MDMA)? life-threatening. MDMA alters the release of vasopressin and will induce a
a. Hyperkalemia clinical syndrome resembling the syndrome of inappropriate secretion of
b. Hypernatremia antidiuretic hormone (SIADH). The patients have concentrated urine with
c. Hypokalemia high urine sodium content. Seizures can occur and should be treated with
d. Hypomagnesemia hypertonic saline. In the absence of seizures or other life-threatening condi-
e. Hyponatremia tions, general supportive care with water restriction is adequate treatment.
2. How long does a typical urine drug screen result remain positive ultrasound examination while contacting the cardiothoracic sur-
after use of cocaine? geon. In the meantime, what should you use to lower her blood
a. 1 day pressure?
b. 3-7 days a. Labetalol
c. 2 weeks b. Nitroglycerine
d. 1 month c. Nitroprusside
Answer: b. Urine drug screens typically detect the cocaine metabolite d. Phentolamine
benzoylecgonine, which is typically present in the urine for 3 to 7 days e. Treatment should be withheld until a definitive diagnosis is
after the last use. made.
3. A 24-year-old man is seen by police to be smoking crack cocaine Answer: d. Phentolamine is a direct alpha-adrenergic antagonist and
on a street corner. On the way to jail, the patient reports a severe is the drug of choice for sympathomimetic-induced hypertension with
headache, so the police bring him to the emergency department end-organ damage. Labetalol and beta-blockers are not recommended,
(ED) for medical clearance. The patient is awake with normal men- because they have little clinical benefit in this setting. Nitroprusside
tal status but continues to complain of diffuse head pain. His vital and nitroglycerine are acceptable agents if phentolamine is not avail-
signs are blood pressure (BP), 195/99 mm Hg; heart rate (HR), 102 able. Treatment should not be withheld because rapid blood pressure
bpm; respiratory rate (RR), 18 rpm; and temperature, 37.2°C. His control could be lifesaving.
physical examination is normal. How should his headache be eval- 6. A 35-year-old man presents with chest pain that started approxi-
uated? mately 2 hours ago, soon after he smoked crack cocaine. The pain
a. Acetaminophen administration, discharge if headache resolves occurred with exertion and has not resolved. He has no previous
b. Blood pressure reduction, discharge if headache resolves medical history. His vital signs are blood pressure (BP), 182/99 mm
c. Head computed tomography (CT) scan, discharge if normal Hg; heart rate (HR), 122 bpm; respiratory rate (RR) 18 rpm; and
d. Head CT scan, lumbar puncture if normal, discharge if both are temperature, 38.2°C. His electrocardiogram (ECG) shows sinus
normal tachycardia with ST depression in the anterior leads. The chest
e. No evaluation needed; cocaine is not associated with headache radiograph is normal. Laboratory results are significant for eleva-
Answer: d. Cocaine is associated with subarachnoid hemorrhage tions in troponin I. What is the likely explanation for his elevated
(SAH). Patients complaining of a severe headache after cocaine use serum cardiac markers?
should receive a complete evaluation for SAH, including head CT and a. He has a completely occluded coronary artery causing ischemia.
lumbar puncture. In addition, elevated blood pressure should be low- b. He has coronary artery spasm without ischemia.
ered while the evaluation is being performed. c. He has coronary stenosis causing ischemia.
4. What is the preferred primary method for controlling a combative d. He has fever and rhabdomyolysis but no cardiac disease.
patient suffering from a sympathomimetic overdose? e. His elevated blood pressure is causing cardiac strain but not
a. Chlorpromazine ischemia.
b. Diazepam Answer: c. Although cocaine can cause coronary spasm with resultant
c. Droperidol chest pain, patients with positive serum markers are likely to have sig-
d. Haloperidol nificant angiographic stenosis. Cocaine users who have complete coro-
e. Physical restraints nary occlusion typically develop ST elevation just as non-cocaine users
Answer: b. Diazepam and other benzodiazepines are the best agents to do. His elevated blood pressure and fever are certainly increasing the
sedate and establish control of an agitated patient. They cause sedation workload on the heart but are not causing his elevated cardiac markers.
as well as decrease muscle tone, which can ameliorate hyperthermia. In Cocaine patients may suffer from rhabdomyolysis, and although the
addition, they can lower the acutely increased blood pressure often seen resultant renal failure can slow the clearance of the cardiac markers, it
with sympathomimetic use. In the agitated patient when intravenous does not cause the elevation.
(IV) access may be difficult to establish, lorazepam is more predictably 7. A 24-year-old man is brought to the emergency department (ED)
absorbed than diazepam. Chlorpromazine can be given intramuscu- in police custody after he admitted to swallowing multiple pack-
larly to provide rapid sedation but is associated with anticholinergic ets of cocaine to smuggle them through an airport. The patient has
effects that may exacerbate hyperthermia. Butyrophenone agents (e.g., no complaints. His physical examination is normal. An abdominal
haloperidol and droperidol) are reserved for more severe agitation and radiograph shows multiple slightly radiopaque areas consistent
are considered secondary methods of treatment after adequate doses of with packets of cocaine. An electrocardiogram (ECG) is normal.
benzodiazepines. Also, they may have associated dysrhythmic effects What is the most appropriate management of this patient?
that are additive to those of cocaine. Physical restraints may be neces- a. Endoscopic removal of packets
sary initially but are not desirable, because they can also increase the b. No therapy needed; the patient may be discharged
risk of hyperthermia as well as increase agitation, which is associated c. Observation alone to watch for elimination of packets
with sudden death. d. Surgical removal of packets
5. A 27-year-old woman presents with severe chest pain that began e. Whole bowel irrigation to remove packets
soon after use of a large amount of cocaine. She describes the pain Answer: e. Whole bowel irrigation with polyethylene glycol facilitates
as “tearing from my chest to my back.” Her vital signs are blood passage of the packets and is safe and effective. Endoscopic removal
pressure (BP), 210/112 mm Hg; heart rate (HR), 142 bpm; respira- should be avoided, because there is increased risk of packet rupture.
tory rate (RR) 22 rpm; and temperature, 38.0°C. A BP measurement Emergent surgical removal is indicated if there is evidence of packet
taken in the other arm is 147/86 mm Hg. An electrocardiogram leak and the patient becomes symptomatic. The patient needs to be
(ECG) is normal. You are highly suspicious that the patient is observed with cardiac monitoring at a center capable of emergent sur-
suffering from an aortic dissection and order a transesophageal gery until all packets have passed.
THC and Hallucinogens 145
KEY CONCEPTS Answer: A. Dextromethorphan presents with more of a dissociative
• H allucinogens include many types of drugs and chemicals with different clinical pattern than opioids. “Skittles, red hots, and triple Cs” are street
associated effects, including action at serotonin receptors, dopamine names for over-the-counter tablets of dextromethorphan. Most immu-
receptors, and glutamate N-methyl-d-aspartate receptors. noassays for PCP cross-react with dextromethorphan to give a false-
• Diagnosis and management are based primarily on the history and physical positive PCP report. Ketamine can also give a similar clinical picture,
examination, with hallmarks of therapy including supportive care, a calm so the history is also necessary to determine which is the most likely
quiet environment, and sedation with benzodiazepines such as diazepam intoxicant in the differential.
or lorazepam. Severely agitated patients may benefit from butyrophenone 3. Which of the following is likely to be detected on a typical urine
antipsychotic agents such as haloperidol and droperidol. immunoassay drug screen?
• Screening tests for hallucinogenic drugs of abuse are of limited value in the a. LSD
acute management of intoxicated patients. b. Kratom
• Novel synthetic hallucinogens continue to emerge and may have effects c. Salvia
from hallucinogenic, serotonergic, and dissociative toxidromes. These d. Cocaine
drugs are rarely detected by screening tests, and cases of toxicity may e. Synthetic cannabinoids
occur in regional outbreaks. Answer: D. The immunoassay screen for cocaine detects its metab-
• Patients with phencyclidine (PCP) toxicity can have unpredictable, violent olite, benzoylecgonine. Unlike immunoassays for amphetamines,
behavior, and may sustain traumatic injuries. Extreme agitation, although benzodiazepines, phencyclidine, and THC, which may be highly
possible, is less common with abuse of ketamine and methoxetamine. unreliable, inconsistent, or nonspecific, the immunoassay for ben-
• Extremely agitated, violent PCP-intoxicated patients may require rapid zoylecgonine does not typically result in false-positives or false-
sedation to decrease danger to the patient and health care providers. negatives. Detection of benzoylecgonine is a specific indication of
For hyperthermic patients, sufficient sedation to decrease neuromuscular cocaine use in the past 3 days. LSD, kratom, salvia, and synthetic can-
hyperactivity may require intubation, paralytics, and active external cooling nabinoids do not typically cross-react with immunoassays that are
to decrease the risk of multiorgan failure and mortality. commonly used.
• The care of patients intoxicated from cannabis and synthetic cannabinoids
consists of prevention of injury and reassurance for those who have panic 4. Which of the following mushroom species has hepatotoxic but not
reactions. An extremely agitated patient can be sedated with oral or par- hallucinogenic properties?
enteral administration of benzodiazepines or antipsychotics. High doses of a. Amanita muscaria
antiemetics may be necessary to treat the nausea and vomiting associated b. Amanita pantherina
with synthetic cannabinoids and heavy daily cannabis use, referred to as c. Amanita phalloides
the “cannabinoid hyperemesis syndrome.” d. Conocybe smithii
• The central nervous system and physiologic effects of mescaline use are e. Psilocybe cubensis
similar to those of lysergic acid diethylamide (LSD) derivatives, but more Answer: C. A. phalloides mushrooms do not have hallucinogenic
vivid hallucinations can occur. Nausea and vomiting are pronounced and properties. Their toxicity is mainly hepatic due to a number of cyclo-
almost always precede the hallucinogenic effects. peptide toxins contained in all parts of the mushroom. C. smithii and
P. cubensis contain psilocybin and psilocin tryptamines. A. muscaria
1. Intoxication with morning glory seeds will mimic intoxication with and A. pantherina contain the isoxazole compounds ibotenic acid
which of the following hallucinogens? and muscimol, which are analogues of glutamic acid (excitatory)
a. Ecstasy and γ-aminobutyric acid (GABA; inhibitory) neurotransmitters,
b. Ketamine respectively.
c. LSD 5. Which of the following agents have been used to treat acute with-
d. Cannabis drawal syndromes and chronic drug dependence?
e. PCP a. Kratom
Answer: C. Several plants contain alkaloids similar to LSD, including b. Ibogaine
morning glory and Hawaiian baby wood rose. c. Buprenorphine
2. A 14-year-old boy presents with altered mental status. On examina- d. Clonidine
tion he has mydriasis, nystagmus, and lethargy. There is a history e. All of the above
of exposure to “skittles.” His urine drug screen for drugs of abuse is Answer: E. Kratom, ibogaine, buprenorphine, and clonidine have all
presumptively positive for PCP. Which toxicant did he ingest? been used to treat acute opioid withdrawal and chronic drug depen-
a. Dextromethorphan dency with varying effects and long-term outcomes.
b. Ecstasy
c. LSD
d. Psilocybin
e. Synthetic cannabinoids
146 Iron and Heavy Metals
KEY CONCEPTS
• A symptomatic patients seeking emergency department (ED) care for an
abnormal metal test need follow-up evaluation arranged with a medical
toxicologist.
• Metal testing in the ED should be ordered in consultation with a medical
toxicologist or regional poison center.
• Acute ingestion of the salts of most metals causes rapid severe gastroin-
testinal pain and emesis.
• Abnormal neurologic signs in a patient with any metal exposure warrants
admission for further evaluation and chelation therapy.
• Acute iron poisoning can result in gastrointestinal symptoms, metabolic
acidosis, and hepatoxicity. Serum iron levels at 3 and 6 hours after inges-
tion determine toxicity and need for therapy.
• The chelation agent of choice for severe iron poisoning is deferoxamine,
which is indicated for peak serum iron concentrations greater than 500 μg/
dL and in patients with severe signs and symptoms regardless of the iron
level.
• The most important intervention for lead poisoning is removal from the
source of exposure.
• The gastrointestinal decontamination method of choice for iron and lead
toxicity with radiographic presence of pills or paint chips is whole bowel
irrigation (WBI).
• The chelation agent of choice for acute arsenic poisoning is intramuscular
British antilewisite (BAL) or oral succimer.
• Elemental mercury is nontoxic to the gastrointestinal tract but may cause
pulmonary and central nervous system (CNS) toxicity from inhalation of vola-
tilized vapors.
1. An 18-month-old female presents to the emergency department a relative) or the child should be admitted to the hospital until the local
with her mother after they were informed of an elevated venous public health entities can inspect and remediate the home. Discharging
blood lead level of 38 mg/dL by their pediatrician. The child is home with primary care follow-up is not appropriate because the child
asymptomatic and has a normal physical exam. She is an only child, will continue to be exposed. Parenteral chelation is required for chil-
does not attend daycare, and spends her time in the family home dren with a lead level of 69 mg/dL and above or for severely symptom-
which was built in 1942. Which of the following would be the most atic patients. Oral chelation is for children with blood lead levels above
appropriate initial management? 44 mg/dL. This asymptomatic child does not meet criteria for chelation
a. Admit for parenteral chelation with a level of 38 mg/dL.
b. Start oral chelation and discharge home 2. A 20-year-old man presents after an iron overdose. An abdominal radio-
c. Admit or secure alternative housing to limit exposure graph shows many radiopaque objects in his stomach, consistent with
d. Discharge home to follow-up with their pediatrician iron tablets. You decide to try to decrease the gastrointestinal absorption
e. Obtain an abdominal x-ray to determine if leaded foreign bodies of the iron. Which of the following methods is most effective?
are present a. Activated charcoal
Answer: E. The most important treatment for all heavy metal poison- b. Gastric lavage
ings is to limit further exposure. The source of lead in this case is likely c. Surgical removal
from leaded paint in the home. Evaluating if the child currently has d. Syrup of ipecac
paint chips or other leaded foreign bodies in her gastrointestinal tract e. Whole bowel irrigation (WBI)
is important. If present, she will require decontamination to prevent Answer: E. WBI is the preferred method to minimize iron absorption.
absorption. After confirming the absence or presence of leaded for- WBI should not be routinely used in small overdoses or with mild
eign bodies, environmental control is paramount. To limit any further symptoms, but it should be considered when a significant ingestion is
exposure, parents should ensure alternative housing (i.e., staying with suspected or when multiple tablets are identified by radiography. Iron
is not adsorbed to oral activated charcoal. Gastric lavage and syrup Answer: C. Iron toxicity typically presents with gastrointestinal symp-
of ipecac do not remove significant amounts of iron. Open surgical toms including occasional bleeding soon after ingestion. The previously
removal of tablets has been used in the past and is effective, but it is mentioned laboratory findings are also typical. Digoxin toxicity can
invasive and has a higher rate of adverse outcomes than WBI. also be manifested with gastrointestinal symptoms, but there is usually
3. A healthy 26-year-old female was helping her great aunt clean out no gastrointestinal bleeding and there are also typically ECG changes.
her home when she spilled a large tray of old mercury thermome- Beta-blocker toxicity, such as with metoprolol, is manifested with
ters on the floor. She then attempted to vacuum up the mercury. She hypotension and bradycardia, as well as with hypoglycemia. Calcium
is now presenting to the emergency department 3 hours later with channel blocker toxicity, such as with diltiazem, can be manifested with
cough, fever, and shortness of breath. Her oxygen saturation is 93% gastrointestinal symptoms, acidosis, and hyperglycemia. However,
on room air and her respiratory rate is 24 breaths per minute. A patients typically have bradycardia, and gastrointestinal bleeding is not
chest x-ray shows bilateral patchy infiltrates. Which of the following expected. Metformin toxicity is manifested with gastrointestinal upset,
would be the most appropriate initial management? typically without bleeding. Lactic acidosis can result, but the vital sign
a. Provide supplemental oxygen and intravenous fluid resuscita- abnormalities and other laboratory results are not expected.
tion and admit for further monitoring. 5. A 57-year-old woman with hypothyroidism presents to the emer-
b. Give oral activated charcoal. gency department with painful, burning circumferential bilateral
c. Consult nephrology for potential acute kidney injury. lower leg pain. She states that over the last 2 weeks the pain has
d. Administer antibiotics and admit for further monitoring. gotten progressively worse so that she feels as though she cannot
e. Call the department of public health to evaluate the home for even wear socks. She lives in a rural community, works part-time at
hazardous material clean-up. the public library, and denies alcohol use or taking any herbal sup-
Answer: A. This patient is exhibiting acute inhalation elemental mer- plements. Her only medication is levothyroxine. She denies fevers
cury toxicity. After an inhalation exposure where mercury is volatilized or chills, polyuria, chest pain or shortness of breath. Her exam is
and inhaled, patients can have fever, shortness of breath, and cough remarkable for hyperalgesia in the bilateral lower extremities. Look-
that can rapidly progress to ARDS. This may occur hours after expo- ing through her chart you notice that she was in the ED 2 weeks ago
sure. Later, as the mercury is absorbed patients are at risk for inorganic for gastroenteritis, was given IV fluids and then discharged home.
mercury poisoning resulting in nephrotoxicity. The best approach to As you leave her room, she tells you she is convinced her husband
these patients is supportive care and close monitoring of their respi- is poisoning her. You then decide to order a complete blood count,
ratory status. There are no roles for activated charcoal or antibiotics as a basic metabolic panel, and a:
activated charcoal does not bind heavy metals and this is a pneumoni- a. Neurology consult
tis. Nephrology may ultimately need to be consulted but now the focus b. Venous blood lead level
should be on the patient’s cardiorespiratory status. Answer choice E c. Vitamin B12
is also reasonable to prevent further exposure at the home given the d. 24-hour urine collection for arsenic testing
large amount of elemental mercury spilled, however it is not the most e. Ethylene glycol level
emergent management step. Answer: D. Acute inorganic arsenic poisoning is characterized by
4. A 5-year-old boy presents with abdominal pain, nausea, vomiting, cholera-like diarrhea followed weeks to months later by peripheral
and bloody diarrhea. The boy was found rummaging through his neuropathy and nail and skin changes. Arsenic was commonly used
grandmother’s medicine cabinet and says he swallowed some of as a rodenticide until the mid-19th century when it was no longer pro-
her medications. His vital signs are significant for hypotension and duced. It can still be found in rural areas, stored in old barns. A 24-
tachycardia. An electrocardiogram (ECG) shows sinus tachycardia. hour speciated urine arsenic test to determine the amount of inorganic
Laboratory tests are significant for an anion gap acidosis, hypergly- and organic arsenic is the gold standard for determining the need for
cemia, and moderate leukocytosis. Which of the following medica- chelation. Although lead poisoning can cause peripheral neuropathy,
tions did he most likely ingest? this is typically motor in nature and not preceded by nausea, vomiting,
a. Digoxin and diarrhea. Constipation and colicky abdominal pain are character-
b. Diltiazem istic for lead poisoning. Vitamin B12 deficiency can lead to a peripheral
c. Iron neuropathy however in this patient it is not the most likely diagnosis.
d. Metformin
e. Metoprolol
Hydrocarbons
147
KEY CONCEPTS
• A spiration is the major toxic risk of hydrocarbon poisoning.
• Hydrocarbons may cause systemic toxicity, burns, cardiac dysrhythmias,
altered mentation, and seizures depending upon the specific agent, dose,
and duration of exposure.
• Gastrointestinal decontamination is potentially harmful in cases of hydro-
carbon ingestion and is contraindicated.
• Hydrocarbon inhalant abuse can cause central nervous system (CNS) and
cardiotoxic effects.
• In most cases of hydrocarbon ingestion or inhalation, symptomatic care
with observation and monitoring are the cornerstones of management.
There are no specific antidotes for hydrocarbons. Patients with pulmonary
symptoms should have a chest radiograph.
• Symptoms of toxicity, especially aspiration, can be delayed, so asymptom-
atic patients should be observed for 6 hours and given instructions to return if
symptoms develop after discharge.
1. Hydrocarbons can affect many organ systems, but derangements in 3. A 4-year-old boy with a history of asthma is brought to the emer-
which organ system most commonly lead to death after hydrocar- gency department (ED) following an accidental hydrocarbon inges-
bon exposure? tion 2 hours ago. He initially was noted to be coughing. His vital
a. Cardiac signs are normal. His physical examination reveals mild bilateral
b. Gastrointestinal wheezing but with good air movement. His oxygen saturation is
c. Nervous 99% on room air. A chest radiograph is normal. The child is placed
d. Pulmonary on a cardiac monitor and intravenous (IV) access is obtained. What
e. Renal is the most appropriate treatment plan?
Answer: D. Most fatalities from hydrocarbon ingestion occur a. Admission and observation
because of aspiration. Hydrocarbons cause direct lung injury, as b. Endotracheal intubation
well as displace oxygen and disrupt surfactant. Hydrocarbons can c. Intravenous sodium bicarbonate
sensitize the myocardium to catecholamines, which can result in d. Racemic epinephrine
ventricular dysrhythmias and sudden death. Although an import- e. Oral activated charcoal
ant consideration, death is more frequently caused by pulmonary Answer: A. Patient is symptomatic after a hydrocarbon exposure.
complications. Gastrointestinal complications from hydrocarbon Hydrocarbon pneumonitis can worsen over the next 24hours. Typi-
exposure are rare unless related to aspiration potential. Chronic cally, they will cause few symptoms when ingested. Any therapy that
hydrocarbon abuse or exposure causes nervous system dysfunction, risks inducing vomiting should be avoided (i.e., gastric lavage and oral
including peripheral neuropathy, cerebellar degeneration, and neu- activated charcoal). Patient is not ill enough to get intubated. Racemic
ropsychiatric disorders, but typically does not result in death. Sev- epinephrine may be helpful if stridor or upper airway swelling is evi-
eral hydrocarbons cause renal failure, particularly toluene, but rarely dent. Sodium bicarbonate has no role in hydrocarbon toxicity.
result in death. 4. A 16-year-old male presents by emergency medical services (EMS)
2. A healthy 20-year-old man presents after accidentally ingesting paramedics with sudden cardiopulmonary arrest after being found
furniture polish that was stored in a plastic container that he unresponsive by friends at a high school beach party. In the emer-
mistook for a beverage. The ingestion occurred approximately gency department (ED), he is unresponsive with no gag reflex.
1 hour ago. He has no complaints. His vital signs and physical When you attempt to orally intubate the patient, you note gold
examination are normal. Which is the most appropriate course metallic colorations inside his oral cavity. What else are you imme-
of action? diately concerned about clinically?
a. Carbon monoxide level a. Airway obstruction from anaphylactic shock
b. Careful gastric decontamination using nasogastric tube b. Cardiac arrest from ventricular dysrhythmias
c. Drug screen c. Sedative hypnotic toxicity and respiratory arrest
d. Observe for 6 hours and discharge if condition does not change d. Toxic alcohol poisoning and acute renal failure
e. Serum electrolytes e. Traumatic intracranial hemorrhage
Answer: D. Furniture polish is typically a mixture of nontoxic chem- Answer: B. This adolescent has likely abused a hydrocarbon (such as
icals with a hydrocarbon base. The primary toxicity from hydrocar- toluene from gold spray paint) for recreational and euphoric reasons.
bon is from aspiration. Gastric decontamination is not recommended. Cardiac arrest could occur from primary asphyxia if they huffed using
Serum electrolytes and drug screens are rarely helpful in acute asymp- a bag over their head or possible direct cardiotoxicity from the inhaled
tomatic hydrocarbon exposures. Wood strippers often contain meth- hydrocarbon resulting in ventricular dysrhythmias.
ylene chloride, which can cause carbon monoxide poisoning, but this
chemical is not found in furniture polish.
148 Inhaled Toxins
KEY CONCEPTS
• A n asphyxiant is any gas that displaces sufficient oxygen from the breath-
able air. Treatment consists of removal from exposure, supplemental oxy-
gen, and supportive care.
• Highly water-soluble gases produce rapid irritation and predominantly upper
respiratory tract effects, such as airway irritation. Poorly water-soluble gases,
like phosgene, often produce delayed lower respiratory tract findings, such as
bronchospasm or acute respiratory distress syndrome (ARDS).
• Carbon monoxide (CO) poisoning is confirmed by co-oximetry measurement.
Cyanide poisoning is treated empirically when cardiovascular instability (e.g.,
hypotension), altered mental status, or a serum lactate greater than 10 mmol/L
are present in a patient with a concerning history, such as a fire victim.
• Hydroxocobalamin is the preferred antidote for most cyanide-poisoned
patients due to its efficacy, ease of use, and safety in patient with concom-
itant CO poisoning. Sodium thiosulfate may be administered concomitantly
and may provide additional benefits.
• Patients with hydrogen sulfide poisoning generally respond to removal from
exposure and ventilatory support.
• Normobaric oxygen therapy is sufficient to many patients with CO poison-
ing, but we recommend consultation with a hyperbaric (HBO) facility, poi-
son control center, or medical toxicologist for consideration of HBO therapy
under specific conditions. Indications for consultation and HBO treatment
include patients with a carboxyhemoglobin (COHb) greater than 25% in the
absence of clinical findings, a COHb greater than 15% or signs of fetal
distress in pregnancy, or an elevated COHb level with one or more of the
following: syncope, coma, altered mental status, abnormal cerebellar func-
tion, or a prolonged CO exposure with minor clinical findings.
1. A laboratory worker is brought to the emergency department a. Arterial blood gas (ABG)
(ED) after being found unconscious. His colleague reports that the b. Chest radiograph
patient was found in his vehicle with its windows closed which he c. Computed tomography of the neck
was using to transport dry ice. The patient is now awake but reports d. Fiberoptic laryngoscopy
feeling tired and confused. He has no other complaints. His vital e. Soft tissue neck radiograph
signs and physical examination are normal. What toxin-specific Answer: d. Fiberoptic or direct laryngoscopy is the preferred method to
diagnostic test should be ordered for this patient? evaluate upper airway symptoms after exposures to irritant gases. Radio-
a. Carboxyhemoglobin (COHb) graphs and laboratory tests have no role and should not influence the
b. Chest radiograph decision to provide a definitive airway. Symptoms can progress rapidly,
c. Electrocardiogram so patients with upper airway symptoms require either placement of a
d. Methemoglobin (MetHb) definitive airway or close observation with frequent serial examinations.
e. No tests are indicated 3. A 52-year-old man is brought to the emergency department (ED) after
Answer: e. Carbon dioxide is primarily a simple asphyxiant, meaning being rescued from a house fire. He has not suffered any cutaneous
that its major consequential adverse effects stem from its displacement burns. He complains of a sore throat, hoarse voice, and cough. Vital
of oxygen in the lungs. Once patients are removed from the source, signs are normal. Physical examination reveals soot in his oropharynx
they generally recover completely. Patients should be observed until and carbonaceous sputum. What therapy should be instituted first?
this time. COHb measurement would be indicated if there is a suspi- a. Endotracheal intubation
cion of carbon monoxide (CO) exposure. Chest radiographs should be b. Intravenous (IV) methylprednisolone
ordered if patients have pulmonary complaints after an unknown expo- c. Nebulized albuterol
sure. Electrocardiograms should be ordered if patients are exposed to d. Nebulized sodium bicarbonate
known cardiac toxins. MetHb levels should be checked when there is e. Saline bronchoalveolar lavage
suspicion for oxidative stress on the red blood cells. Answer: a. Endotracheal intubation should be performed early in
2. A 32-year-old woman presents following exposure to an irritant patients with signs and symptoms of significant airway burns (as with
gas at her job site. She reports cough, burning eyes, and short- this patient). Corticosteroids are not beneficial and can worsen associ-
ness of breath. She has mild tachypnea, with the remainder of her ated injuries. Inhaled beta-agonists are commonly used, but there is no
vital signs within normal limits. Her oxygen saturation is 96% on evidence of improved outcome. Inhaled sodium bicarbonate plays no
room air. She is noted to have stridor on physical examination. role in the management of smoke inhalation. Bronchoalveolar lavage
What is the preferred method to evaluate her upper airway symp- can be performed if there is suspicion of inhaled debris or toxins, but
toms? the airway should first be secured.
4. A 47-year-old woman is brought to the emergency department Answer: c. This patient was exposed to chlorine gas which is an inter-
(ED) after being rescued from a house fire. She was found uncon- mediate water-soluble gas that combines with water in the respira-
scious at the scene and intubated before arrival. Her vital signs are tory tract to form hydrochloric acid. Patients may have improvement
significant for hypotension and tachycardia. Physical examination of their symptoms with the administration of nebulized 2% sodium
is significant for soot in the oropharynx. No cutaneous burns are bicarbonate solution. Systemic corticosteroids have not shown a ben-
noted. You suspect that she is suffering from cyanide poisoning. efit in these patients. Antibiotics should not be administered routinely
What is the most appropriate immediate therapy? in these patients unless an overlying infectious process is suspected.
a. Hyperbaric oxygen (HBO) 7. What is the major benefit of hyperbaric oxygen (HBO) therapy for
b. Intravenous (IV) hydroxocobalamin patients suffering from carbon monoxide (CO) poisoning?
c. IV methylene blue a. Decreased rate of hospitalization
d. IV sodium nitrite b. Improvement of 24-hour mortality
e. Observation and supportive care c. Improvement of 30-day mortality
Answer: b. One of the two major treatments for cyanide poisoning is d. Prevention of delayed cardiovascular complications
hydroxocobalamin (the other is the traditional cyanide antidote kit). e. Prevention of delayed neuropsychiatric complications
The nitrite compounds in the cyanide antidote kit convert hemoglobin Answer: e. There is controversy regarding the role of HBO therapy for
to methemoglobin (MetHb), which in turn binds to cyanide. However, patients with CO poisoning, but the best evidence suggests that it can
nitrites produce hypotension and the MetHb prevents proper oxygen significantly decrease the incidence of delayed neuropsychiatric com-
deliver, which may compound the reduction in oxygen delivery asso- plications. There is no change in rate of hospitalization, nor on overall
ciated with carbon monoxide (CO) poisoning. HBO has no role in mortality, either short term or long term. There are no delayed cardio-
acute cyanide poisoning. Methylene blue has been used for cyanide vascular symptoms associated with CO poisoning.
poisoning in the past but is not as useful as the cyanide kit. Its primary 8. Assuming that all patients have similar vital signs and complaints
use is in treating methemoglobinemia. General supportive care is not of headache and nausea, which of the following patients suffering
appropriate because there is an effective antidote for this patient’s from carbon monoxide (CO) poisoning should be considered high-
poisoning. est priority for hyperbaric oxygen (HBO) therapy?
5. A 22-year-old man is brought to the emergency department (ED) a. A 22-year-old otherwise healthy man with a carboxyhemoglobin
after being found unconscious in a car with an intentionally promi- (COHb) level of 30%
nent suicide note visible in the window. By the time he arrives in the b. A 25-year-old otherwise healthy pregnant woman with a COHb
ED, he has regained consciousness and is complaining of headache level of 25%
and nausea. Paramedics report that the car engine was not running c. A 30-year-old otherwise healthy man also suffering from cya-
when the patient was discovered. His vital signs and physical exam- nide poisoning with a COHb level of 15%
ination are normal. Which of the following therapies should be d. A 35-year-old otherwise healthy woman with second-degree
instituted? burns to 20% of her body and with a COHb level of 20%
a. Hyperbaric oxygen (HBO) e. A 67-year-old asymptomatic woman with coronary artery dis-
b. Intravenous (IV) methylene blue ease and with a COHb level of 25%
c. IV sodium nitrite Answer: b. Pregnant patients should be considered for HBO therapy.
d. IV sodium thiosulfate CO binds more strongly to fetal hemoglobin than to adult hemoglobin
e. Observation and supportive care and can cause severe hypoxia to the fetus. There is controversy about an
Answer: e. This patient has been exposed to hydrogen sulfide (a com- absolute level of COHb that requires HBO therapy. HBO does not ben-
mon form of suicide in some parts of the world), which has similar efit cyanide victims, nor is it indicated in uncomplicated burn patients
effects on the mitochondria as cyanide. However, hydrogen sulfide is or those with stable comorbidities.
rapidly removed from the body; and as long as patients are recovering, 9. You successfully treat a patient for cyanide poisoning. Several hours
removal from the source is usually all that is necessary. HBO and meth- after improvement of the patient’s hemodynamics, you are alerted
ylene blue have no role in hydrogen sulfide poisoning. Sodium nitrite that the patient has developed a bright red skin discoloration.
can be used in patients who are not recovering once removed from Which of the following is the most likely explanation for this new
the source or for severe exposures. Sodium thiosulfate is not necessary physical exam finding?
because hydrogen sulfide is detoxified by a different pathway than cya- a. Administration of hydroxocobalamin
nide and does not need a sulfur donor. b. Concomitant carbon monoxide toxicity
6. A worker at a chemical plant was exposed to a cloud of chlorine c. An anaphylactic reaction
gas (Cl2). On examination in the emergency department you note a d. Development of hyperthermia
normal oxygen saturation, increased respiratory rate, and wheezing e. Peripheral vasodilation
on lung auscultation. In addition to administration of inhaled beta- Answer: a. Patient presented with cyanide toxicity for which the anti-
adrenergic agonists, which of the following can be administered for dote, hydroxocobalamin, was given. Hydroxocobalamin has only mild
symptomatic relief? adverse effects, although it can cause hypertension in those without
a. Nebulized hypertonic saline cyanide toxicity and it can cause a bright red discoloration of the
b. Systemic corticosteroids patient’s skin. This discoloration can often be mistaken as an allergic
c. Nebulized 2% sodium bicarbonate solution reaction and it can interfere with spectrophotometric laboratory assays
d. Antibiotics including COHb level. Cherry red skin discoloration in the setting of
e. Intravenous magnesium sulfate CN or CO poisoning is a postmortem finding.
149 Lithium
KEY CONCEPTS
• T he clinical pattern of acute and chronic lithium toxicity is different. Gas-
trointestinal symptoms occur early, and neurologic toxicity manifests late
in acute toxicity. Neurologic findings (e.g., tremors, altered mental status
and seizures) often are presenting signs of chronic lithium toxicity, whereas
gastrointestinal symptoms often are absent.
• Neither activated charcoal nor whole bowel irrigation (WBI) is indicated in
the routine management of acute or chronic lithium toxicity.
• Serial lithium concentrations should be obtained every 2 to 4 hours initially
to determine the peak level and the need for hemodialysis.
• Fluid hydration with crystalloid is essential to enhance renal excretion of
lithium. Diuretics are contraindicated.
• We recommend hemodialysis for acute lithium concentrations greater than
5 mEq/L or for patients with signs of severe neurologic lithium toxicity,
particularly tremor, clonus, altered mental status, and seizure, regardless
of the serum concentration.
1. What are the two most common electrolyte abnormalities seen in Answer: b. Hydrochlorothiazide, other diuretics, angiotensin-
chronic lithium toxicity? converting enzyme (ACE) inhibitors, and nonsteroidal antiinflam-
a. Hyperkalemia and hypernatremia matory drugs (NSAIDs) can increase lithium levels and consequently
b. Hypernatremia and hypercalcemia cause chronic lithium toxicity by interfering with renal elimination.
c. Hypokalemia and hypercalcemia Lithium has also been implicated in serotonin syndrome when com-
d. Hypomagnesemia and hyponatremia bined with other serotonergic drugs, such as monoamine oxidase
e. Hyponatremia and hypercalcemia (MAO) inhibitors, selective serotonin reuptake inhibitors (SSRIs), and
Answer: b. Chronic lithium use can cause hypernatremia due to dextromethorphan. None of the other drugs listed causes significant
the development of diabetes insipidus. Prolonged therapy with lithium interactions with lithium.
alters cellular calcium sensing and leads to elevated parathyroid hor- 4. A 26-year-old woman presents following a witnessed seizure. She
mone levels and hypercalcemia. Magnesium and potassium levels are was found with an empty bottle of lithium and a suicide note.
typically normal, and abnormalities are not caused by lithium. Hypo- Her vital signs reveal hypotension and tachycardia. She is lethar-
natremia and hypercalcemia are frequently seen in cases of acute lith- gic and her only response is to withdraw from painful stimuli. The
ium overdose. remainder of her physical examination is normal. Laboratory tests
2. Which of the following conditions is most frequently seen in including a serum lithium level are pending. Which of the following
patients with chronic lithium use? treatments is the most appropriate?
a. Anticholinergic syndrome a. Activated charcoal
b. Diabetes insipidus b. Forced diuresis
c. Hyperthyroidism c. Hemodialysis
d. Hypoparathyroidism d. Urinary alkalinization
e. Syndrome of inappropriate antidiuretic hormone e. Whole bowel irrigation (WBI)
Answer: b. Diabetes insipidus commonly occurs in patients on Answer: c. Hemodialysis is the most effective way to remove lith-
chronic lithium therapy or with chronic overdose. Diabetes insipidus ium. It can remove lithium at a rate five to seven times the rate of
can cause dehydration and a further increase in lithium concentration typical renal elimination. Common indications for dialysis include
and is a frequent contributory cause to chronic lithium toxicity. Rarely, decreased level of consciousness and seizures. Activated charcoal does
hypothyroidism can also develop. Lithium use is also associated with not adsorb lithium. Lithium overdose patients are often dehydrated
neuroleptic malignant syndrome and serotonin syndrome but none of and occasionally hypernatremic and should be fluid resuscitated, but
the other listed conditions. once dehydration is corrected, forced diuresis is of no benefit and can
3. Which of the following drugs should be used with caution in be detrimental. Urinary alkalinization has no role in lithium toxicity.
patients taking lithium? Because this patient has already experienced a seizure, the administra-
a. Acetaminophen tion of WBI is not advised. In addition, the seizure suggests that the
b. Hydrochlorothiazide brain concentration of lithium is already toxic and this is not improved
c. Metformin by WBI.
d. Metoprolol
e. Penicillin
Antipsychotics
150
KEY CONCEPTS
• A ntipsychotics are commonly categorized into typical, or first-generation • Q T prolongation and torsades de pointes are potential complications of
antipsychotics (FGAs) with primary antagonism to dopamine receptors, and antipsychotic overdose but may also occur with therapeutic use.
atypical, or second-generation antipsychotics (SGAs) which include sero- • Clozapine is associated with potentially life-threatening agranulocytosis.
tonin receptors as a target. Aripiprazole is an example of a third type of Treatment includes cessation of the medication, treating potential infec-
antipsychotic that acts as a partial agonist at dopamine sites. tions, and supportive care.
• Extrapyramidal symptoms are common side effects of antipsychotics. First • Neuroleptic malignant syndrome (NMS) is characterized by altered mental
line treatment is benztropine or diphenhydramine. Lorazepam may be used status, hyperthermia, muscle rigidity, and autonomic instability. Treatment
in refractory cases. includes supportive care with airway management, benzodiazepines, mus-
• The most common presentation of an antipsychotic overdose is central cular rigidity management, and evaporative cooling methods for hyperther-
nervous system (CNS) depression. Treatment focuses on supportive care, mia.
airway management, and cardiac monitoring.
1. Which of the following is a second-generation atypical antipsy- significantly improve acute dystonia. Morphine may help but does not
chotic? resolve the underlying problem.
a. Aripiprazole 4. A 52-year-old man presents to the emergency department with a
b. Haloperidol cough. He is tachycardic with a temperature of 38.6°C. His chest
c. Lumateperone x-ray has a left lower lobe infiltrate. His lab work is significant for
d. Olanzapine a leukopenia with an absolute neutrophil count of 250 cells/μL. His
Answer: d. Olanzapine is an atypical second-generation antipsychotic. past medical history is significant for psychiatric illness. Which of
Aripiprazole is an atypical psychotic but would be considered a third the following medications is he most likely to be taking?
generation. Haloperidol is a first-generation, high-potency antipsy- a. Clonidine
chotic. Lumateperone is a newer unique antipsychotic that affects mul- b. Clozapine
tiple systems including serotonergic, dopaminergic, and glutamatergic. c. Fluoxetine
2. A 19-year-old woman presents with urinary frequency, urgency, d. Lithium
and dysuria. She had a urinalysis which demonstrated 20 to 40 Answer: b. Clozapine is an antipsychotic associated with agranulo-
white blood cells per high-power field. She has schizophrenia for cytosis and neutropenia. Clonidine is an alpha-adrenergic agonist
which she takes haloperidol. You diagnose her with an uncompli- with no known association with this condition. Fluoxetine is a selec-
cated urinary tract infection. Which of the following antibiotics tive serotonin reuptake inhibitor commonly used in the treatment of
could cause a life-threatening arrhythmia if administered to this depression and is not known to cause neutropenia. Lithium is used
patient? as a mood stabilizing medication such as in bipolar disorder and not
a. Amoxicillin/clavulanic acid known to cause an alteration in cell counts.
b. Ciprofloxacin 5. A 23-year-old woman is brought to the emergency department by
c. Cephalexin her boyfriend with a change in mental status. He reports that the
d. Nitrofurantoin patient had her psychiatric medications adjusted 3 days ago and has
Answer: b. Antipsychotics can cause QT prolongation, so other drugs been confused since then. The patient’s vital signs are: blood pres-
that cause QT prolongation should be avoided. Macrolides, fluoro- sure: 162/100 mm Hg; heart rate: 140 beats/min; respiratory rate:
quinolones, and trimethoprim-sulfamethoxazole are common antibi- 22 breaths per minute; and temperature: 40.2°C. On physical exam-
otics that can all cause QT prolongation. ination, the patient is noted to have muscle rigidity. Laboratory
3. A 32-year-old man presents complaining of neck pain. His neck is tests are pending. Intravenous (IV) access is obtained, and balanced
turned to the right and he states that he has difficulty moving it. crystalloid fluids are started. Cool mist and fans are applied to the
He denies any trauma and has not had a fever. He has no previous patient. What therapy is indicated next?
similar episodes. He has schizophrenia but cannot recall the names a. Acetaminophen
of his medications. His vital signs are normal and his examination b. Cyproheptadine
reveals a palpable spasm of the right trapezius and sternocleido- c. Dantrolene
mastoid muscles. What is the most appropriate treatment? d. Lorazepam
a. Bromocriptine Answer: d. The patient has neuroleptic malignant syndrome (NMS)
b. Cyclobenzaprine and the best first line treatment option is a benzodiazepine, such as
c. Diphenhydramine lorazepam. Acetaminophen is ineffective in treating the hyperthermia,
d. Morphine which should be treated with active cooling measures. Cyprohepta-
Answer: c. This patient has an acute dystonic reaction to an antipsy- dine can be used in serotonin syndrome but does not improve NMS.
chotic medication best treated with an anticholinergic medication, Dantrolene can be used in malignant hyperthermia and for refractory,
such as diphenhydramine or benztropine. Bromocriptine is a dopa- severe muscle rigidity but has no mechanistic indication in NMS, and
mine agonist used to treat pituitary disorders and can worsen psycho- evidence for its use is of low quality. Diphenhydramine is used for acute
sis. Cyclobenzaprine can be used for typical muscle spasms but will not dystonia and does not affect the course of NMS.
151 Opioids
KEY CONCEPTS
• T he opioid toxidrome includes three prominent findings—central nervous
system depression, miosis, and, most importantly, respiratory depres-
sion—but presentations may be variable.
• A negative urine screen is often unreliable, and absence of detection
should not deter a diagnosis of opioid intoxication when clinical findings
support it.
• Airway protection, oxygenation, ventilation, and early administration of the
reversal agent naloxone are the cornerstones for management of patients
with opioid toxicity.
• The duration of action of many opioids, especially after overdose, is signifi-
cantly longer than that of naloxone. Patients responsive to naloxone should
be observed for recurrence of respiratory depression because they may
require additional doses of naloxone.
• Naloxone distribution, prescription drug monitoring programs, and initia-
tion of buprenorphine with a referral to addiction treatment programs are
ways in which the medical profession is trying to combat the epidemic of
opioid-related deaths.
• Opioid withdrawal syndrome does not include altered cognition. Patients
with known or suspected opioid withdrawal who also have altered mental
status should be evaluated for an alternative cause of altered cognition.
1. A 26-year-old female is brought to the emergency department (ED) Answer: a. An abdominal radiograph would likely reveal multiple
from the local airport by law enforcement. She is sleepy, with small packets of illicit opioid in the gastrointestinal tract of this body packer.
pupils, and mumbles incoherently in a foreign language. Vital signs One or more of the packets has leaked, producing the opioid toxicity.
include the following: blood pressure, 104/66; respiratory rate, 14 A urine drug screen may not identify an opioid but would not identify
breaths/min; and temperature, 98.6°F. Which of the following tests the internal packets. A head computed tomography (CT) scan would
might identify the cause of this patient’s symptoms? not be helpful unless associated head trauma is suspected. An EEG and
a. Abdominal radiograph ECG would not provide specific information to identify the internal
b. Electrocardiogram (ECG) packets.
c. Electroencephalogram (EEG)
d. Head computed tomography (CT)
e. Urine drug screen
1961.e2 PART IV Environment and Toxicology
C H A P T E R 1 5 1 : Q U E S T I O N S A N D A N S W E R S
2. Most opioids cause mild hypotension related to histamine release is suspected. Lactate and a urine drug screen would not change patient
and bradycardia. Which of the following opioids can also cause management.
sodium channel blockade and QRS widening? 6. Which of the following medications can be used to treat opioid
a. Hydromorphone withdrawal?
b. Hydrocodone a. Clonidine
c. Morphine b. Dextromethorphan
d. Oxycodone c. Diphenhydramine
e. Propoxyphene d. Nalmefene
Answer: e. Propoxyphene and its metabolite norpropoxyphene can e. Valproic acid
cause QRS widening. None of the other listed opioids has significant Answer: a. Clonidine suppresses the sympathetic hyperactivity of opioid
effects on the cardiac conduction system. withdrawal. Dextromethorphan is an opioid derivative used as a cough
3. Which of the following laboratory abnormalities is most commonly suppressant, but it does not treat the symptoms of opioid withdrawal.
seen in opioid overdose? Diphenhydramine is not the most commonly used antihistamine for a
a. Hypocalcemia patient in withdrawal. Valproic acid has no role in opioid withdrawal.
b. Hypochloremia Nalmefene is an opioid antagonist similar to naloxone but with a longer
c. Hypoglycemia duration of action. Administration of nalmefene is longer-acting than
d. Hypokalemia naloxone and would worsen opioid withdrawal symptoms.
e. Hyponatremia 7. A 14-month-old child is brought to the emergency department
Answer: c. Hypoglycemia is the only consistent laboratory abnormal- (ED) 4 hours after he was found with his grandmother’s antidiar-
ity found in opioid overdose. It is generally mild but can contribute to rheal medication bottle. A pill count identifies that only one Lomotil
the decreased level of consciousness seen in opioid overdose. tablet is missing. The child is playful, has a normal respiratory rate
4. A 32-year-old man presents with confusion, nausea, vomiting, diar- and pattern, and has a soft abdomen with normal bowel sounds.
rhea, and abdominal pain. His friends report that he is withdraw- Appropriate management includes which of the following?
ing from heroin. Vital signs reveal mild hypertension, tachycardia, a. Administration of activated charcoal
and tachypnea. Physical examination is significant for confusion, b. Administration of naloxone
mydriasis, diaphoresis, lacrimation, piloerection, and mild diffuse c. Admission to a monitored unit
abdominal tenderness. Which of the following signs and symptoms d. Discharge home
makes you concerned that this may not be a simple opioid with- e. Gastric lavage
drawal case? Answer: c. Activated charcoal and gastric lavage are means of gastro-
a. Confusion intestinal decontamination and are not routinely recommended in opi-
b. Diarrhea oid toxicity. Opioid intoxicated patients with central nervous system
c. Mydriasis (CNS) and respiratory depression should be treated with naloxone,
d. Piloerection but asymptomatic patients do not require antidote administration.
e. Tachycardia Asymptomatic patients with known or suspected Lomotil (diphenox-
Answer: a. Opioid withdrawal almost always causes restlessness, agita- ylate/atropine) ingestion should be observed in a monitored setting for
tion, and anxiety. Cognition and mental status are not affected in sim- delayed onset of toxicity from the metabolite of diphenoxylate.
ple opioid withdrawal and, if present, should prompt the clinician to 8. An 18-year-old male is driven to the emergency department (ED)
search for other causes instead of, or in addition to, withdrawal. by friends and carried into the triage area. He has agonal respira-
5. A 20-year-old woman is brought to the emergency department tions and is cyanotic. Immediate resuscitative measures include
(ED) after being found with decreased mental status at a club. Vital bag-valve-
mask (BVM) ventilation, establishment of an intra-
signs indicate mild hypotension and bradycardia. She is drowsy but venous (IV) line, and administration of 0.4 mg of naloxone. The
arousable, and she has an otherwise normal physical examination. patient’s respiratory status improves and, although sleepy, he is
Upon receiving naloxone 2 mg IV, her mental status immediately able to answer some questions. During subsequent monitoring,
improves and soon thereafter she vomits. She now reports nausea the patient’s respiratory status again declines, and he requires two
but has no other complaints. She states that she took some “pain additional doses of naloxone. Additional treatment should include
pills” to get high but does not know what they were. What diagnos- which of the following?
tic tests should be performed? a. Nalmefene
a. Acetaminophen b. Naloxone infusion
b. Arterial blood gas c. Hemodialysis
c. Chest radiograph d. Naltrexone
d. Lactate e. Narcan nasal spray
e. Urine drug screen Answer: b. This patient likely has toxicity from a long-acting opioid
Answer: a. Because many prescription opioid medications are combi- agent, and a continuous infusion of naloxone will be necessary for
nations of an opioid and acetaminophen, ibuprofen, or salicylate, con- ongoing reversal of toxicity. Nalmefene and Naltrexone are a lon-
centrations of acetaminophen and salicylate should also be ordered. ger-acting opioid antagonists but are not preferred over naloxone infu-
Acetaminophen overdose might otherwise remain undiagnosed but, sion because naloxone allows for dose titration. Once multiple doses of
if identified, can be treated with an existing antidote, N acetylcysteine. IV narcan are given, there is no role for narcan nasal spray. Opioids are
The chest radiograph is not indicated unless a pulmonary complication not dialyzable due to large volumes of distribution.
152 Pesticides
KEY CONCEPTS Answer: d. This patient has symptoms of the cholinergic syndrome
(SLUDGE symptoms) and was likely exposed to an organophosphate
• O rganophosphates cause symptoms by accumulation of acetylcholine. or carbamate pesticide. Patients poisoned with organophosphate or
• Treat cholinergic symptoms with atropine. carbamate pesticides that are acidotic have a higher mortality than
• Reverse the inhibition of acetylcholinesterase with oximes. those with normal acid-base status. Elevated and decreased glucose
• Aging, which results in prolonged toxicity, occurs with organophosphate concentrations have both been associated with increased morbidity
poisoning, but not with carbamates. and mortality. As for PSS and GCS, these vary by type of poisoning and
• Chlorinated hydrocarbons can present with seizures and cardiac toxicity. cannot reliably be used to determine risk of fatality.
• Substituted phenols are found in weight loss products and exert their tox- 3. A 4-year-old boy arrives at the emergency department (ED) after
icity by uncoupling oxidative phosphorylation. These products can cause drinking a medication that was being used to treat his sister’s head
cardiac, liver, and renal injury. lice. An unknown amount was consumed, and the bottle is not
• Chlorophenoxy compounds cause muscular injury. Measure creatinine available. The patient’s only complaint is of nausea. Vital signs and
kinase; assess for acute rhabdomyolysis, kidney injury, and liver injury. physical examination findings are normal. Which of the following
• Bipyridyl compounds (paraquat and diquat) cause pulmonary and renal injury. symptoms should be anticipated?
• Paraquat concentrates in lungs; supplemental oxygen therapy should be a. Gastrointestinal hemorrhage
limited, since this will exacerbate pulmonary toxicity. b. Hallucinations
• Diquat causes renal injury. c. Hypotension
• Pyrethrins and pyrethroids cause local dermatologic symptoms. d. Paralysis
• Acute toxicity with glyphosate is likely related to the surfactant included in e. Seizure
the product. Answer: e. Lindane is a chlorinated hydrocarbon insecticide that is
• DEET (N, N-Diethyl-meta-toluamide) should not be used in infants younger used for the topical treatment of head lice and scabies. It is rapidly
than 2 months old. DEET in concentrations of more than 30% should not absorbed and can result in difficult-to-control seizures requiring
be used in children and may result in neurotoxicity and self-limited seizure high doses of benzodiazepines or barbiturates and that may require
activity if used in excessive amounts. sedation, paralysis, and intubation. Because lindane is a hydrocar-
• Most rodenticide exposures will be superwarfarin compounds. bon, it can sensitize the cardiac membrane and predispose to ven-
• For large exposures, International Normalized Ratio (INR) should be tricular dysrhythmias. It can also cause pulmonary compromise if it
checked at a minimum of 2 days after ingestion. is aspirated.
• Vitamin K should be used for reversal; blood products should be used for 4. An 18-year-old woman arrives at the emergency department (ED)
active bleeding. complaining of feeling generally weak. She reports reading online
about a pesticide that can help in weight loss, and she has recently
1. A 37-year-old man arrives at the emergency department (ED) after tried this. Her vital signs are blood pressure, 110/70 mm Hg; heart
exposure to an organophosphate. He is severely symptomatic, and rate, 121 bpm; respiratory rate, 22 rpm; and temperature, 104°F
atropine is given. When should atropine treatment be discontin- (40.0°C). Her physical examination reveals dry mucous membranes
ued? and yellow staining on her abdomen. Which of the following labo-
a. After 100 mg has been given ratory findings can you anticipate?
b. When fasciculations stop a. Hypoglycemia
c. When mydriasis occurs b. Hypokalemia
d. When secretions have stopped c. Hyponatremia
e. When tachycardia occurs d. Hypoxia
Answer: d. Patients with organophosphate poisoning may require very e. Methemoglobinemia
large doses of atropine (up to 500 mg). Proper dosing is 1 or 2 mg intra- Answer: a. Substituted phenols, such as dinitrophenol, are pesticides
venously, with doubling of the dose every 5 minutes until the drying that uncouple oxidative phosphorylation. This causes an increased
of secretions. Atropine has no effect at the neuromuscular junction. metabolism, which in turn consumes glucose and generates heat, often
Mydriasis may occur before secretions have dried. Tachycardia is likely, causing an increased body temperature. For this reason, they have been
but it is not a contraindication for continued atropine treatment. The used as diet aids. They can be applied topically, and the yellow skin
tachycardia often improves as the pulmonary status improves. staining is considered pathognomonic for nitrogen compounds. Treat-
2. A 30-year-old agricultural worker arrives at the emergency depart- ment is supportive and should be aimed at stopping further exposure;
ment (ED) with confusion, abdominal pain, nausea, vomiting, and providing the needed substrates of oxygen, water, and glucose; and
shortness of breath. His coworkers report that he was spraying applying active cooling measures.
plants with an insecticide. The patient’s vital signs are significant 5. What organ system is most affected by paraquat ingestion?
for hypotension, bradycardia, and tachypnea. Physical examination a. Cardiac
reveals miosis, wheezing, vomiting, diarrhea, and urinary inconti- b. Gastrointestinal
nence. Which clinical finding is associated with higher mortality? c. Nervous
a. Hyperglycemia d. Pulmonary
b. Hypoglycemia e. Renal
c. Glasgow Coma Scale (GCS) ≤3
d. Acidosis
e. Poison Severity Score (PSS) 4
Answer: d. Paraquat is concentrated in the lungs and directly damages Answer: d. DEET absorption and toxicity increase with repeated appli-
the alveolar capillary membrane. This results in surfactant loss, adult cations, with increased ambient temperatures, with sweating, when it is
respiratory distress syndrome, pulmonary fibrosis, and respiratory fail- applied to abraded or thin skin, and when it is covered with tight-fitting
ure and is accelerated with supplemental oxygen. All the other organ clothing. Oils or lipophilic substances applied to the skin also increase
systems listed are affected but to a much lesser degree. absorption of DEET.
6. A patient arrives at the emergency department (ED) after an inten- 9. A patient has long-standing depression that was exacerbated by the
tional paraquat ingestion complaining of severe mouth, throat, and recent COVID-19 pandemic. During the isolation of the pandemic,
chest pain. What potentially fatal complication of paraquat inges- the patient was cleaning their attic and found an old box of rat poi-
tion should be suspected? son. During a particularly challenging time, the patient attempted
a. Aortic dissection suicide by consuming the contents of the rat poison container. The
b. Esophageal rupture patient woke up the next day and was asymptomatic. Two days later,
c. Myocardial infarction the patient noted bruising and hematuria. Which class of pesticides
d. Pneumothorax likely represents the overdose in this situation?
e. Pulmonary embolism a. Bipyridal compounds
Answer: b. Paraquat is extremely corrosive and can cause severe burns b. Organophosphates
to the oropharynx, as well as to the esophagus. Frequently, esophageal c. Pyrethrins
rupture occurs, leading to mediastinitis and death. None of the other d. Superwarfarins
listed conditions occurs with any frequency in paraquat poisoning. e. Chlorophenoxy compounds
7. How do pyrethrins cause toxicity? Answer: d. Superwarfarins were developed from warfarin for rodenti-
a. Sensitivity or allergic reactions cides, after increasing numbers of rats developed resistance to warfarin.
b. Bone marrow suppression They act by inhibiting vitamin K, which is a cofactor in the synthesis
c. Cardiovascular instability of clotting factors II, VII, IX, and X. The therapeutic and toxic effects
d. Inhibition of coagulation require significant reduction of these clotting factor concentrations,
e. Uncoupling of oxidative phosphorylation thus taking at least 15 hours to present. While warfarin has a half-life
Answer: a. Pyrethrins are naturally occurring substances from the yel- of 35 hours, the duration of action can be up to 5 days. Superwarfarins
low chrysanthemum and commonly cause allergic reactions in humans. are 100 times more potent than warfarin, leading to increased and pro-
These reactions can be mild or life-threatening, with bronchoconstric- longed toxicity, which can manifest as superficial, as well as internal,
tion and laryngeal edema. They also affect gamma-aminobutyric acid bleeding (such as bruising and hematuria). Bipyridal compounds, such
(GABA)-mediated chloride channels in the nervous system, but this as paraquat and diquat, are pesticides that are not used in the United
typically results in only a mild headache and paresthesias. States. Organophosphates are insecticides that are highly regulated in
8. DEET is a commonly used insect repellant that can cause contact der- the United States. Pyrethrins are insecticides derived from chrysanthe-
matitis and more severe neurologic complications, including seizures mums, commonly used to treat scabies. Chlorophenoxy compounds
with high doses that are absorbed through the skin. Which of the fol- are herbicides, and not used as rodenticides.
lowing methods will minimize DEET absorption through the skin?
a. Apply to skin at night
b. Cover skin with clothing
c. Expose skin to direct sunlight
d. Keep skin dry
e. Remove from skin with oil-based products
153 Plants, Herbal Medications, and Mushrooms
KEY CONCEPTS
• M ost botanical exposures result in minimal toxicity and management is
largely supportive.
• Most serious toxicities result from exposure to plants with anticholinergic,
antimitotic, cardiovascular or convulsive properties.
• Most cases of mushroom ingestion in which gastrointestinal (GI) symp-
toms begin within the first 2 h will prove to involve a non–life-threatening
substance.
• Delayed GI symptoms with an onset of more than 6–8 h after exposure
suggest a potentially life-threatening ingestion, such as the cyclopeptide
and gyromitrin mushroom groups.
• Regional poison control centers, mycologists, and botanists can assist in
identifying potentially toxic plants and mushrooms. We recommend smart
phone digital photography with expert consultation.
• Herbal medications are largely unregulated and may have inherent toxicity,
herb-drug interactions, or contaminants. Clinicians should advise against
the routine use of herbal medications.
1. A 52-year-old man presents after ingestion of water hemlock com- Answer: a. The vast majority of severely toxic mushrooms have delayed
plaining of nausea but no other symptoms. He reports that he ate onset of symptoms (>6 hours from ingestion). Occasionally, people
the water hemlock in a suicide attempt but has since changed his will ingest two different types of mushrooms—one benign and causing
mind. His vital signs and physical examination are normal. What is early onset of symptoms and one more toxic. Therefore, patients should
the most appropriate initial treatment of this patient? be observed and rechecked.
a. General seizure precautions 5. What is the most common cause of death after mushroom inges-
b. Hemodialysis tion?
c. Neutralization with milk a. Gastrointestinal hemorrhage
d. Supportive care only b. Heart failure
e. Whole bowel irrigation c. Liver failure
Answer: a. Water hemlock is very toxic, with fatality rates as high as d. Renal failure
70%. Because this patient is at risk for seizures, oral activated char- e. Respiratory failure
coal and gastric lavage are contraindicated. Hemodialysis, whole bowel Answer: c. The Amanita species of mushrooms are very toxic and cause
irrigation, and milk are not beneficial. Supportive care is not adequate fulminant liver failure often necessitating liver transplant. Hepatotoxic
with water hemlock ingestion. Most deaths are caused by intractable mushrooms account for about 90% of all mushroom fatalities. Although
seizures, so this complication should be anticipated. certain mushrooms (Cortinarius spp. and Amanita smithiana) can cause
2. A family of four presents to the emergency department feeling ill renal failure, death is uncommon with these mushrooms. Mushrooms
after consuming a homemade stew containing Datura stramonium generally do not affect the other listed organ systems.
(Jimsonweed). Which of the following can be expected upon phys- 6. A 46-year-old man presents after waking in the night with headache
ical examination? and severe nausea and vomiting. For dinner the night before, he
a. Bradycardia consumed some mushrooms that he had gathered from a nearby
b. Confusion forest. He felt fine immediately after dinner. His vital signs are
c. Diaphoresis normal. While conducting your physical examination, he begins
d. Miosis to seize. Benzodiazepines are administered without improvement.
e. Respiratory depression What treatment is indicated?
Answer: b. Jimsonweed contains atropine, hyoscyamine, and scopol- a. Phenobarbital
amine, all of which have anticholinergic properties. Anticholinergic b. Phenytoin
signs include confusion, dry skin and mucous membranes, mydriasis, c. Propofol
tachycardia, urinary retention, confusion. In severe cases, physostig- d. Pyridoxine
mine can be used to reverse effects. e. Vecuronium
3. Oleander contains cardiac glycosides similar to digoxin. Which of Answer: d. Most mushrooms do not cause seizures, but gyromitrin-
the following should be considered when treating oleander inges- containing mushrooms are an exception. They are commonly mis-
tion? taken for edible morels because they look quite similar. They contain
a. Activated charcoal should be avoided. an isoniazid-like toxin that causes seizures. Traditional seizure medi-
b. Serum digoxin levels correlate with degree of toxicity. cations can be used (because they are typically more readily available),
c. Treatment is supportive. but seizures can be intractable until pyridoxine is given.
d. Treatment with digoxin-specific Fab fragments is beneficial, but 7. A 54-year-old male with a history of depression presents to the
a higher dose may be necessary. emergency department with confusion. He is prescribed nortripty-
e. Treatment with digoxin-specific Fab fragments is not beneficial. line only, but his wife reports that he has recently started taking an
Answer: d. Treatment is with digoxin-specific Fab fragments, but herbal supplement. His vital signs are temperature 100.3°F, blood
much higher doses are necessary compared with those needed to treat pressure 130/75, pulse 124, respiratory rate 20, oxygen saturation
digoxin toxicity. Activated charcoal is useful if the ingestion was recent. 96%, You note spontaneous clonus to his lower extremities and
A serum digoxin level should be measured, but it is not a reliable indi- diagnose him with serotonin syndrome. Which of the following
cator of level of toxicity. An elevated level confirms oleander ingestion, herbal supplements did he most likely start taking?
but a negative level cannot rule out ingestion. a. Ephedra
4. A husband and wife present to the emergency department (ED) b. Hypericum perforatum
complaining of nausea and vomiting after eating mushrooms that c. Gingko biloba
they picked while hiking. They report eating the mushrooms on a d. Piper methisticum
salad approximately 1 hour ago. Each is complaining of severe nau- e. Saw palmetto
sea, vomiting, and diffuse abdominal pain. There is no hemateme- Answer: b. Hypericum peforatum (St. John’s wort) is a non-selective sero-
sis. Vitals signs and physical examination are normal. You inform tonin reuptake inhibitor and is thought to also upregulate post-synaptic
them that they will be symptomatically treated and observed, but serotonin receptors. Combination therapy with other anti-depressants
there is likely nothing to worry about. Why are you not concerned that increase serotonin has been reported to cause serotonin syndrome.
about their ingestion? Ephedra is no longer commercially available as it was banned by the FDA
a. Early onset of gastrointestinal symptoms for its association with cardiovascular disease. Gingko biloba may have
b. No hematemesis antioxidant properties and inhibit platelet aggregation. Piper methisti-
c. No right upper quadrant tenderness cum can increases GABA-mediated CNS depression when combined
d. Normal mental status with alcohol. Saw palmetto is a remedy for benign prostatic hypertrophy
e. Normal vital signs and is thought to work by inhibiting 5-α-reductase.
154 Sedative-Hypnotics
KEY CONCEPTS
• S upportive care, with a focus on respiratory depression, is the foundation
of management of all sedative-hypnotic ingestions.
• Benzodiazepines are commonly used medications both medically and recre-
ationally. Coingestions with other sedative-hypnotic agents can potentiate
their neurologic and respiratory effects.
• Barbiturate medications are less commonly prescribed, and intoxications
are infrequent. Most patients recover with supportive care alone.
• A positive urine toxicology screen for benzodiazepines or barbiturates does
not prove a causal linkage between the drug and the current clinical condi-
tion.
• We do not recommend routine use of flumazenil for benzodiazepine tox-
icity, particularly in chronic benzodiazepine users in whom flumazenil can
precipitate seizures. Due to the short duration of action of flumazenil, when
administered, patients should be monitored closely for recurrence of respi-
ratory depression.
• Chloral hydrate toxicity may result in sedation and cardiotoxicity, principally
in the form of supraventricular tachycardias, which are best treated with a
short-acting beta blocker.
• Withdrawal from sedative-hypnotic use, including benzodiazepines, barbi-
turates and gamma hydroxybutyrate (GHB), can be life threatening. Man-
agement often requires high doses of benzodiazepines or barbiturates.
1. All of the following patients have depressed mental status, decreased Answer: a. Flumazenil variably reverses the effects of benzodiazepines
respiratory drive, and mild hypotension. Supportive care has been but can cause dysrhythmias and intractable seizures. Contraindications
initiated. All of the patients have a confirmed benzodiazepine over- for flumazenil include history or suspected chronic benzodiazepine
dose by a reliable historian. For which patient is it most reasonable use (which are the vast majority of benzodiazepine overdose patients),
to consider the use of flumazenil? coingestants that lower the seizure threshold, tricyclic antidepressants,
a. A 5-year-old boy with no known medical problems who took his cocaine, history of seizure disorder, chronic alcoholism, and head trauma.
mother’s medication Its use should be for a known non-habituated patient (e.g., the pediatric
b. A 22-year-old woman with no known medical problems patient described here). Because flumazenil’s duration of action is short
c. A 30-year-old man who is known to abuse cocaine compared with the benzodiazepine and it has variability to reverse respi-
d. A 43-year-old man with depression and anxiety ratory depression, patients require close, monitored observation, repeated
e. A 79-year-old woman with congestive heart failure, coronary dosing or continuous infusion, and preparations to manage the airway.
artery disease, and diabetes who takes benzodiazepines for Because benzodiazepine overdose is rarely lethal and patients recover with
insomnia supportive care, using an antidote requires a careful benefit/risk analysis.
2. How do barbiturates affect peripheral and cerebral blood flow? d. Serum myoglobin
a. Decrease peripheral pressure and have no effect on intracranial e. Urine drug screen
pressure Answer: a. Many over-the-counter (OTC) sleep aids contain acet-
b. Decrease peripheral pressure and increase intracranial pressure aminophen or salicylates. The active ingredient in OTC sleep aids is an
c. Decrease peripheral and intracranial pressure antihistamine, typically diphenhydramine or doxylamine. Both have
d. Increase peripheral and intracranial pressure anticholinergic effects in addition to causing sedation. Toxicity is typ-
e. Increase peripheral pressure and decrease intracranial pressure ically mild and can be successfully managed with supportive care. It is
Answer: c. Barbiturates cause direct cardiac depression with decreased rare that an antihistamine overdose results in rhabdomyolysis, and a
cardiac output. In addition, they cause venous pooling and blunt the serum creatine phosphokinase level, urinalysis, and urinary myoglobin
normal compensatory increase in system vascular resistance that level can be checked if there is concern. Myoglobin is cleared rapidly
would occur with decreased cardiac output. They also decrease cere- from the serum and is not a useful diagnostic test. Arterial blood gases,
bral blood flow and lower intracranial pressure. These effects are par- serum chemistries, and urine drug screens can be ordered if there is a
ticularly pronounced in individuals with an already low cardiac output, complicated clinical course—but only rarely alter treatment.
such as in congestive heart failure or hypovolemic shock. 6. A 21-year-old woman is brought to the emergency department (ED)
3. Other than general supportive care, which of the following is most by her friends after they found her unconscious in the bathroom of
useful in the management of a severe phenobarbital overdose with a local bar. The friends report that she had only had one or two alco-
hemodynamic instability? holic drinks. The patient’s vital signs show bradycardia, hypotension,
a. Flumazenil and bradypnea. Physical examination reveals short periods of apnea
b. Hemodialysis with hypoxia. She wakens briefly to noxious stimuli. You suspect that
c. Multidose activated charcoal she may be suffering from gamma-hydroxybutyrate (GHB) poison-
d. Urinary alkalinization ing. What is the most appropriate next step in management?
e. Whole bowel irrigation a. Atropine
Answer: b. Although rarely indicated, hemodialysis is effective for b. Electrocardiogram
acute phenobarbital toxicity (persistent serum levels over 100 mcg/mL) c. Endotracheal intubation
in the presence of prolonged coma, refractory hypotension, renal or d. Flumazenil
cardiac failure, metabolic acidosis, or inadequate response to conven- e. Naloxone
tional measures and ventilatory support. These patients often require Answer: c. GHB frequently causes emesis and aspiration because
mechanical ventilation, so the patient’s airway should be secured prior. patients are unable to protect their airways. Patients with significant
Although there is evidence that multi-dose activated charcoal (MDAC) GHB overdose should be intubated, but supportive care alone is all
increases clearance of phenobarbital and may shorten duration of clin- that is required for most GHB overdoses. The half-life is short, and
ical toxicity, there is no evidence it results in improved outcomes over many patients will return to baseline mental status in several hours.
supportive care alone. Flumazenil will reverse the effect of benzodi- Bradycardia is common, usually mild, and does not typically require
azepines but has no effect on barbiturates. Urinary alkalinization has atropine, but this can be used if bradycardia is severe. GHB does not
traditionally been used to “trap” the acidic barbiturates in the urine to generally cause dysrhythmias. Neither flumazenil nor naloxone has any
increase excretion, However, a recent comprehensive review concluded effect on GHB.
that there is little clinical role for urine alkalinization in acute barbitu- 7. A 30-year-old man presents with anxiety and insomnia. He is agi-
rate poisoning. Whole bowel irrigation is not beneficial. tated and has a tremor on physical examination. His vital signs are
4. Which of the following sedative-hypnotics is most likely to cause as follows: blood pressure, 200/124 mm Hg; heart rate, 123 beats/
fatal cardiac dysrhythmias in overdose? min; respiratory rate, 22 breaths/min; and temperature, 37.5°C. On
a. Buspirone further questioning, he reports that he chronically uses gamma-
b. Chloral hydrate hydroxybutyrate (GHB) for muscle building but has not been able
c. Eszopiclone to take any for approximately 24 hours. Lorazepam is given in two
d. Flunitrazepam escalating doses without response. Which of the following should
e. Zolpidem be given next?
Answer: b. Chloral hydrate sensitizes the myocardium to catechol- a. A barbiturate
amines and also independently induces dysrhythmias. Treatment of b. Additional lorazepam
choice of dysrhythmias from chloral hydrate is beta-blockers. None of c. Clonidine
the other listed medications cause significant cardiac toxicity. d. GHB
5. A 53-year-old woman presents with decreased mental status and e. Labetalol
mild hypotension. She is arousable to pain. Her husband reports Answer: a. GHB withdrawal is severe, and deaths have been reported.
that approximately 4 hours ago she had been complaining of insom- GHB acts as a central nervous system (CNS) depressant, so withdrawal
nia and said she was going to take something to help her sleep. He causes CNS excitement and autonomic instability. Chronic GHB abuse
then found her in her present state with an empty bottle of “sleep- can lead to GABA depletion in the CNS. Because benzodiazepines
ing pills” on the counter. He does not remember the name of the require the presence of GABA, they may not be effective. Barbiturates
sleeping pills. Supportive care is initiated. Which of the following act independently of GABA and are therefore more effective in GHB
laboratory tests would be the most helpful to manage this patient? withdrawal. Clonidine can be used in opioid withdrawal but is not
a. Acetaminophen level useful in GHB withdrawal. Labetalol would improve the vital signs
b. Arterial blood gas but would have no effect on the underlying cause of the autonomic
c. Serum chemistry instability.
SECTION ONE The Pediatric Patient
KEY CONCEPTS
• P atterns of illness and injury vary by age, and a number of anatomic and
physiologic characteristics affect the presentation and management of
pediatric emergencies.
• A basic understanding of normal development will aid the emergency clini-
cian in assessment of the pediatric patient.
• The pediatric assessment triangle (PAT) can be used as a tool for rapid
evaluation of the patient’s overall status.
• Tachypnea in children should be evaluated relative to age norms and is
often a sign of increased metabolic demands. A child with tachypnea
despite normothermia should be evaluated for respiratory and nonrespira-
tory causes (e.g., hypoperfusion, acidemia).
• Maintenance of a neutral thermal environment is necessary for critically ill
infants.
• Child abuse should be considered when injuries are inconsistent with his-
tory, when details of the history change, or with certain injury patterns.
• A joint guideline of the American College of Emergency Physicians, Ameri-
can Academy of Pediatrics, and Emergency Nurses Association summarizes
the role of pediatric emergency care coordinators, development of pediat-
ric policies, and recommended equipment, supplies, and medications for
emergency departments (EDs).
• EDs that comply with the national guidelines for pediatric readiness have
improved outcomes for care of critically ill children.
• The family’s presence should be encouraged for pediatric procedures and
resuscitations.
• A variety of pharmacologic and nonpharmacologic techniques are available
to decrease procedural pain and anxiety.
1. Which of the following respiratory signs is characteristic of a child instead of kilograms. The use of length-based resuscitation tapes and
in compensated or decompensated shock? resuscitation calculators and pharmacy review of medication orders
a. Grunting with abnormal breath sounds have also been suggested to reduce medication dosing errors, but the
b. Tachypnea with nasal flaring most effective measure is to weigh and record the weight in kilograms.
c. Tachypnea with clear breath sounds on auscultation 3. You have written an order for a blood draw and placement of an
d. Tripoding with stridor intravenous line in a nervous 3-year-old boy. Which of the follow-
e. Wheezing with retractions ing is least likely to be helpful in decreasing the patient’s procedure-
Answer: c. Effortless tachypnea, or rapid respirations with clear related distress?
breath sounds, is characteristic of a child in compensated or decom- a. Application of a lidocaine-epinephrine-tetracaine mixture
pensated shock. Grunting, nasal flaring, tripoding, stridor, wheezing, b. Having the patient blow bubbles prior to the needle stick
and retractions are all signs of increased work of breathing and would c. Needle-free jet injection of local anesthetic
be more indicative of pulmonary or airway disease. d. Use of a vapocoolant
2. As the pediatric physician coordinator for your emergency depart- Answer: a. The formulation of lidocaine-epinephrine-tetracaine
ment, you decide to institute new policies to prevent pediatric only works on broken skin (i.e., lacerations). Unlike eutectic mixture
medication errors. Which of the following strategies will be most of local anesthetic (EMLA) and 4% liposomal lidocaine preparations
effective in decreasing risk of dosing errors? which are effective on intact skin, it will not work on intact skin. The
a. Pharmacy review of medication orders use of vapocoolants and a needle-free jet infection of local anesthetic
b. Use of length-based resuscitation tapes may decrease the patient’s pain. Distraction techniques such as blowing
c. Use of resuscitation calculators bubbles, singing a song, or watching a video may relieve procedure-
d. Weighing and recording the weight in kilograms related anxiety.
Answer: d. Weights should be measured and recorded in kilograms,
not pounds, to avoid inadvertent dosing calculations using pounds
Pediatric Airway Management 156
c. The neck should be placed in slight flexion and the shoulders
KEY CONCEPTS
extended.
• P ediatric advanced airway management is a relatively rare skill to perform d. The patient should be positioned so that a line drawn through
in most emergency departments (EDs), and skill maintenance is difficult the external auditory canal and the anterior shoulder is parallel
based solely on clinical practice. to the bed.
• There are several anatomic differences that impact pediatric airway manage- Answer: D. The relatively large head and occiput of the infant
ment, and these occur mostly in the very young child (<2 years of age). Infants results in slight flexion at the neck when supine, impeding the abil-
have a large occiput and a high, anterior airway, which impacts positioning ity to visualize the glottis. The infant is correctly positioned so that
during intubation. The narrowest portion of the pediatric airway is at the level of a line drawn through the external auditory canal and the anterior
the cricothyroid membrane which means a foreign body could be lodged below shoulder is horizontal and parallel to the bed. In the infant (<6
the cords. They are also more dependent on diaphragmatic excursion for venti- months old), this is accomplished by placing a towel roll under the
lation, thus gastric insufflation can result in difficulty with rescue ventilation. patient’s shoulders, elevating the body and overcoming the flexion
• Children are prone to desaturation due to their high metabolic rate and associated with their large occiput. A head tilt–chin lift may open
their lungs’ small functional residual capacity, making preoxygenation and the airway.
maintenance of oxygenation during intubation attempts crucial. 3. Relative to rapid sequence intubation (RSI) in adults, children
• The cognitive burden inherent in dealing with the large age/size spectrum in undergoing RSI:
pediatrics can be overcome with reference aids that organize equipment selec- a. Are less affected by stomach pressures.
tion and drug dosing based on length/age/size. Formulas have been developed to b. Are more likely to develop bradycardia in response to hypoxia.
aid in selection of the correct endotracheal tube (ETT) size and determine appro- c. Have less missed intubations.
priate depth of ETT insertion. For estimation of uncuffed tube sizes in children d. Have less pliable airways.
older than 1 year old: ETT size = 4 + (age in years/4). Subtract 0.5 in size for cuffed e. Maintain their oxygen saturations longer after paralytics are
tubes. To estimate the depth of ETT insertion (the so-called “lip to tip” distance), administered.
multiply the ETT size × 3 (e.g., a 5.0 ETT would be inserted to 15 cm at the lip). Answer: B. Children have a higher metabolic rate and thus a higher
• Rapid sequence intubation (RSI) is the preferred method of airway manage- oxygen demand. In the absence of respiratory support, children will
ment in the vast majority of pediatric cases in the ED. drop their oxygen saturations more quickly than adults due to their rel-
• Compared to adults, children are more prone to desaturation over the time atively greater consumption of oxygen. Hypoxia is the most common
it takes for a neuromuscular blocking agent (NMBA) to take effect. Use of reason for bradycardia during intubation in children. The more pliable
high-flow nasal cannula during the apneic period of RSI has not been well pediatric thorax combined with children's lower ventilatory reserve
studied in children in the emergency setting, but we recommend its use at make stominflation a significant impediment to effective respiratory
1 to 2 L/min/year of age to a maximum of 15 Lpm. Because children desat- support. Pediatric intubations are infrequent and multiple pediatric
urate more rapidly than adults, we recommend that assisted ventilation intubation attempts are more common in children than in adults.
(coordinated with the child’s respiratory efforts if not yet fully paralyzed) be 4. Which of the following does not characterize the pediatric airway
initiated if oxygen saturation drops below 95%. relative to adults?
• Video laryngoscopy is an evolving technology for use in pediatrics and a. A straight blade is used to pick up the floppy epiglottis.
assists in visualization of the airway but may prolong time to intubation. b. Infants may require a shoulder roll to align airway axes.
• Surgical airway techniques differ in infants and young children, necessitat- c. The airway is more pliable.
ing a needle technique that is different from the older child or adult. This d. The airway is more superior and posterior.
technique provides a mechanism to oxygenate the “can’t intubate, can’t e. The epiglottis is larger and omega-shaped.
ventilate” child, but should not be relied on as a definitive airway.
Answer: D. Small infants have a large occiput and a high, anterior air-
1. Which of the following is a correct formula for use in airway man- way, which impacts positioning during intubation, often requiring a
agement of children? shoulder roll to align airway axes. The airway is also more pliable and the
a. Cuffed ETT size (1 to 10 years old) = 4 + (age in years/2) membranes thinner, necessitating needle cricothyroidotomy for surgical
b. ETT depth = 3 × ETT size airways in children younger than 10 years old. Blade choice is age depen-
c. ETT depth = 4 × ETT size dent, and a straight blade is recommended in young children; this blade
d. Uncuffed ETT size (1 to 10 years old) = (16/age in years) + 4 lifts the relatively larger epiglottis to visualize the vocal cords.
Answer: B. Correct formulas are as follows: 5. Following the administration of succinylcholine in a 7-year-old boy
Uncuffed ETT size = 4 + (age in years/4) with respiratory distress, severe masseter spasm is noted. Which of
Cuffed ETT size = subtract 0.5 from formula for uncuffed ETT; (4 + the following medications should be administered to terminate this
(age in years/4)) − 0.5 spasm?
ETT depth = 3 × uncuffed tube size a. Diazepam
2. Which of the following should be used for proper positioning of the b. Fentanyl
pediatric patient during endotracheal intubation? c. Repeated dose of succinylcholine
a. The chin should be tilted and the head lifted. d. Rocuronium
b. The infant younger than 6 months old should have a towel roll e. Thiopental
placed under the occiput to align the airway axes. Answer: D. Masseter muscle spasm is a rare side effect of succinyl-
choline administration. It primarily occurs in pediatric patients and
is typically terminated by the administration of a competitive neuro-
muscular blocking agent (NMBA), such as rocuronium, vecuronium,
or pancuronium. Failure to respond to such therapy should prompt
consideration of malignant hyperthermia.
157 Pediatric Sedation and Analgesia
KEY CONCEPTS
• P atients of all ages experience pain, including infants, neonates, and pre-
mature babies.
• Oligoanalgesia, the inadequate treatment of pain, has many short-term and
long-term consequences: worse patient outcomes, increase in patient’s
pain threshold, and development of chronic pain.
• Pain management may include a combination of techniques: analgesics,
topical anesthetics, local anesthetic injections, oral sucrose in infants, and
nonpharmacologic interventions.
• Nonpharmacologic interventions to decrease pain or anxiety include paren-
tal presence; physical measures, such as heat or cold therapy and splinting
for musculoskeletal injuries; and behavioral or cognitive measures, such as
distraction and play therapy.
• Topical anesthetics are recommended to decrease the pain of minor proce-
dures, such as venipuncture or IV cannulation.
• Techniques for decreasing the pain of intradermal injections include topical
agent prior to the intradermal injection; slowly injecting warmed, buffered
local anesthetic solution from within the wound with the smallest gauge
needle possible; and limiting the number of needle punctures.
• When using large amounts of local anesthetics in small children or infants,
calculate the drug dose to avoid toxicity; a 1% solution = 1 g/100 mL or 10
mg/mL.
• Procedural sedation and analgesia (PSA) requires pre- sedation evalu-
ation; sufficient monitoring (during and after the procedure) by qualified
individuals capable of dealing with any adverse events that may occur;
age-appropriate equipment (including airway equipment) and medications
(including reversal agents and advance life support drugs); and discharge
criteria for when the patient is fully awake, returns to baseline with normal
vital signs, and is able to be discharged in the care of a responsible adult.
• Overall, preprocedural fasting is not necessary for most emergency patients,
because large studies show no clinically significant differences with airway
complications, emesis, or other adverse effects between groups of patients
stratified by their preprocedural fasting status.
• Choice of sedative and analgesic for PSA depends on many variables
including patient factors and the procedure to be done. Slow titration of
medications can achieve the desired level of sedation and analgesia while
minimizing risk of adverse events.
1. When do most adverse events associated with emergency depart- Answer: c. The patient’s ability to follow commands in response to
ment (ED) procedural sedation occur? varied levels of stimulation and direct observation of the ventilatory
a. During the manipulation or intervention status have been the most reliably documented methods of assessing
b. 5 to 20 minutes after the last sedative dose the level of consciousness during procedural sedation. Pulse oxime-
c. 20 to 30 minutes after the last sedative dose try is a reliable adjunct, but it identifies hypoventilation late, espe-
d. 30 to 60 minutes after the last sedative dose cially when used with supplemental oxygen. Cardiac monitoring has
e. 60 to 90 minutes after the last sedative dose been shown to be helpful in older patients or in those with a history
Answer: b. High-risk times are 5 to 20 minutes after the last med- of cardiac disease, but there is no evidence that it is of any benefit in
ication administration and at the completion of the procedure when young healthy patients. End-tidal carbon dioxide monitoring has been
there is no longer a painful stimulus, but the patient remains sedated. shown to be useful to detect inadequate ventilation earlier than oxim-
2. Which of the following modalities has proven most effective for etry, especially when direct observation of the patient is difficult, but
monitoring patients undergoing procedural sedation? no studies have demonstrated an effect on clinical outcome to date.
a. Capnometry or capnography Recently, the American Society of Anesthesiologists (ASA) updated its
b. Cardiac rhythm monitoring procedural sedation standards to include capnography during moder-
c. Continual direct visual observation of qualitative clinical signs ate or deep sedation, in addition to the continual observation of quali-
d. Documented respiratory rate tative clinical signs. Respiratory rate alone is an insensitive indicator of
e. Pulse oximetry adequacy of ventilation.
3. Which of the following agents is matched with the correct associ- properties. Its adverse effects include dose- dependent respiratory
ated side effect? depression, apnea, hypotension, and pain on injection. Preload-
a. Etomidate—longer (>30-minute) duration of sedation dependent patients are particularly susceptible to hypotension. Its
b. Ketamine—laryngospasm combined use with ketamine is common. The two agents are felt to
c. Methohexital—venoirritation have synergistic effects that balance each other’s deficits. The combined
d. Pentobarbital—seizures use has been shown to improve provider satisfaction, sedation quality,
e. Propofol—myoclonus and decrease emesis but has not been shown to be clinically superior to
Answer: b. Ketamine has been associated with laryngospasm in either agent used alone regarding respiratory depression, airway com-
children younger than 3 months old and those with a respiratory infec- plications, or improved recovery times.
tion. The following are the other correct associations: 6. Which of the following statements is true regarding the need for
• Etomidate—myoclonus fasting before procedural sedation?
• Methohexital—seizures a. A 6-hour period of fasting is required after the ingestion of liq-
• Pentobarbital—longer (>30-minute) duration uids or solids before procedural sedation.
• Propofol—venoirritation b. Preprocedural fasting is required in all circumstances.
4. Which of the following statements regarding the use of ketamine is c. The recommendation for preprocedural fasting is based on con-
false? trolled trials involving patients undergoing procedural sedation.
a. Benzodiazepine administration may be useful for emergence d. The risk of vomiting and the loss of the airway protective reflexes
phenomenon in children that are not improved by removing is an extremely rare occurrence during procedural sedation.
stimulation and providing calming interventions. e. There is an increased risk of aspiration during procedural seda-
b. Despite increased secretions, airway reflexes are generally well tion after a liquid or solid meal.
maintained. Answer: d. The American College of Emergency Physicians orga-
c. Hypotension is common. nized a multidisciplinary Consensus Statement on Unscheduled
d. Profound analgesic and sedative effects occur with minimal Sedation that recommends the assessment of the timing and nature
respiratory depression. of recent oral intake. The urgency of the procedure will dictate the
e. Repeat doses are well tolerated in longer procedures. necessity of providing sedation without delay, regardless of fasting
Answer: c. Ketamine increases the release of catecholamines upon status. For patients with established risk factors for aspiration (e.g.,
administration and supports blood pressure well. It also decreases serious underlying illness, obstructive sleep apnea, obesity, age
smooth muscle tone in the bronchial tree and may have a benefit in less than 12 months, upper endoscopy as the procedure, or bowel
patients with reactive airways disease. Several studies have failed to show obstruction), consider the risks versus benefits of delaying proce-
benefit with the concurrent administration of low-to-moderate dos- dural sedation after recent ingestion of a substantial meal. American
ages of benzodiazepines in preventing emergence phenomenon in chil- Society of Anesthesiologists (ASA) currently recommends a period
dren. These studies have shown a slightly increased risk of side effects. of 2 hours after ingestion of clear liquids, a period of 4 hours after
Their routine use is discouraged and should be reserved for the actual ingestion of breast milk, and a period of 6 hours after ingestion of
treatment of severe emergency phenomenon that are not improved by other liquids or solids before the performance of procedural sedation.
removing stimulation and providing calming interventions. This recommendation is based on expert consensus and extrapolated
5. Which of the following statements regarding the use of propofol is true? from data on patients receiving general anesthesia and manipulation
a. Propofol has a long duration of action and provides significant of the airway during intubation and extubation. There are no pub-
analgesia. lished studies showing increased risk of aspiration after a liquid or
b. Propofol has significant antiemetic properties. solid meal, nor benefits of fasting before procedural sedation. There
c. Propofol can be easily reversed with a reversal agent. are large studies showing no clinically significant differences with air-
d. Propofol is well tolerated in volume-depleted patients. way complications, emesis, or other adverse effects between groups of
e. The use of “ketofol” (ketamine in combination with propofol) is patients stratified by their preprocedural fasting status. Adherence to
clinically superior to the use of propofol alone. the ASA preoperative fasting guidelines for procedures is not neces-
Answer: b. Propofol is an ultra–short-acting, sedative-hypnotic, sary in emergency department (ED) patients undergoing procedural
cerebroprotective agent with no analgesic but profound antiemetic sedation and analgesia (PSA).
158 Pediatric Resuscitation
KEY CONCEPTS
• U nlike adults, most cardiac arrests in children arise from respiratory etiolo- • K nowledge of the child’s approximate weight is necessary to administer the
gies. Therefore, emphasis is first on oxygenation and ventilation. appropriate medication doses and to utilize the appropriate equipment. Deter-
• Detection of a child’s pulse may be difficult; if a brachial pulse is not defini- mine actual weight in kilogram and estimate weight based on parental report
tively present after 10 seconds of palpation, initiate cardiopulmonary resusci- or the child’s length.
tation (CPR). • Prompt vascular access is critical in a resuscitation. Intraosseous access
• Hypotension is a late finding in pediatric shock and requires immediate inter- tends to be the easiest and fastest, but care should be taken that the needle
vention to prevent cardiac arrest. Progression from tachycardia to bradycardia is not residing in the subcutaneous tissue. Central venous access is resource
in a child is often a harbinger of imminent arrest. intensive and not necessary in the first few hours of a resuscitation.
• Although few studies support recommendations for exact rate, depths, and • Except in specific circumstances, resuscitations greater than 30 minutes are
ratios of compressions to ventilations in pediatric CPR, emergency clinicians unlikely to yield favorable outcomes for pediatric arrest.
should perform rapid compressions over the lower sternum with minimal • After return of circulation, maintain normothermia and avoid hypotension.
interruptions. The hand encircling technique is recommended for infants • Current definitions of sepsis are based on consensus and primarily for
requiring chest compressions and the two-hand approach in mid sternum for research purposes.
older children. In the emergency department (ED), we recommend 15 com- • Systemic inflammatory response syndrome (SIRS) criteria are sensitive but
pressions to 2 ventilations for children less than 8 years and 30 compressions not specific for identifying sepsis in children.
to 2 ventilations for older children. • Hypotension should prompt administration of 20 to 60 mL/kg fluids and likely
• Verbal and quantitative feedback improves compressions and ventilations. vasopressors.
• Advanced airway management (i.e., endotracheal intubation) may be harmful • Most children with septic shock have cold shock and therefore epinephrine
for children in arrest in the ED. If attempted, physicians should strive for min- (0.05 mcg/kg/min) is the first line vasopressor.
imal interruptions in compressions and to ventilate at 8 to 10 breaths/min. • Protocols bundling intravenous fluids, antibiotics, and blood culture acquisi-
High quality bag-mask ventilation is a reasonable alternative to advanced tion in septic children improve outcomes.
airway maneuvers. • Brief resolved unexplained events (BRUEs) involve alterations in breathing,
• If ventricular fibrillation or pulseless ventricular tachycardia arise, defibrillate tone, or behavior that last less than 1 minute before resolving spontaneously.
at 2 J/kg as soon as possible. While preparing the equipment and charging, Children appear well on presentation to the ED, and there are no elements of
continue to perform high quality compressions. Administer subsequent the history that suggest a particular etiology for the event.
defibrillations at escalating energy doses of 4 J/kg and then 10 J/kg. • Low-risk BRUEs occur in children that were born greater than 32 weeks,
• Early administration of epinephrine for non-perfusing rhythms may improve are greater than 60 days of age, have had no prior BRUEs, and CPR was not
survival. required during the event. These children require an assessment and brief
• Empiric administration of medications to children in arrest worsens outcomes. observation in the ED, little or no diagnostic testing, and may be discharged
Reserve medications for specific indications, for example, bicarbonate admin- with close follow-up.
istration for hyperkalemia.
1. You are treating a 4-month-old with bronchiolitis. While being Answer: D. This child is in septic shock. He received an initial fluid
observed in the ED, the child becomes bradycardic and then loses bolus but has ongoing signs of decreased perfusion. In the absence
pulses. What should be the primary focus of your initial minutes of of signs of pulmonary edema, the next step should be to administer
the resuscitation? another bolus of isotonic fluids. He may benefit from epinephrine and
a. Preparing to insert an endotracheal tube to improve oxygen- vancomycin but after a second bolus is initiated.
ation and ventilation 4. A 4-month-old girl presents after changing color and tone at home.
b. Correction of potential acidosis with bicarbonate She was feeding and started to choke. She turned red-purple in her
c. Vascular access to administer epinephrine face and upper body while her arms stiffened. After 30 seconds, she
d. Uninterrupted compressions with ventilations via bag- mask spit up some formula, and the episode resolved. She was born full-
device term and has no other medical issues. In the ED she is asymptom-
Answer: d. At the beginning of the arrest, the most important elements atic and has a normal physical examination. What is your next step
are restoration of perfusion and ventilations with compressions and in the management of this patient?
ventilations. Early insertion of an endotracheal tube is likely to impede a. Discharge after 2 hours of observation
compressions, and initial ventilations should occur with a bag valve b. Classify the event as a high-risk BRUE and admit to the hospital
mask and oral airway. Vascular access will be needed but is not as time c. Obtain a pH probe to assess for GERD
sensitive. Bicarbonate should be given only if there is a clear indication. d. Obtain an ECG and blood glucose
2. After return of circulation, which action is most critical to improve Answer: a. Choking in the setting of feeding with complete resolution
a child’s prognosis? of the symptoms after vomiting suggests reflux. As a potential etiology
a. Empiric amiodarone to prevent arrhythmias is apparent based on the history, this event should not be classified as a
b. Avoidance of hypotension BRUE. Diagnostic testing is not necessary; pH probes are not specific
c. Induced hypothermia to 33° for GERD and are rarely helpful in the ED.
d. Strict glucose control less than 110 mg/dL with an insulin drip 5. Which of the following medications have shown a mortality benefit
e. Maintenance of oxygen saturation at 100% for children in arrest without a clear etiology?
Answer: b. A single episode of hypotension increases mortality and a. Calcium chloride 20 mg/kg
therefore efforts should be made to avoid it. Induced hypothermia does b. Sodium bicarbonate 1 mEq/kg
not appear to improve outcomes in pediatric arrest. Strict glucose con- c. Epinephrine 0.01 mg/kg
trol, hyperoxemia, and empiric amiodarone are not beneficial and may d. Normal Saline 20 mL/kg
induce harm. Answer: c. There are no randomized control trials of medications in
3. A 6-year-old male with Trisomy 21 presents with decreased respon- pediatric arrest. However, observational studies suggest that epineph-
siveness and not eating. His temperature is 39°C, heart rate is 150, rine is beneficial. Other medications are only beneficial for specific
blood pressure is 80/55, respiratory rate is 24, oxygen saturation is indications, such as glucose for hypoglycemia.
98%, and glucose is 90 mg/dL. He receives cefepime and 20 mL/
kg of lactated Ringers but his vital sign abnormalities persist. On
examination, he is lethargic with a neck that is supple, lungs that
are clear to auscultation, cool extremities, and capillary refill of 3
seconds. What is the next best step in the management?
a. Hydrocortisone 50 mg/m2
b. Vancomycin 50 mg/kg
c. Epinephrine 0.05 mcg/kg/min
d. Lactated Ringers 20 mL/kg
159 Neonatal Resuscitation
KEY CONCEPTS
• T rauma is the leading cause of death in children in the United States.
• Avoid hypoxia and hypotension by early administration of oxygen and
assisted ventilation, and fluid resuscitation with crystalloid at 20 mL/
kg increments. Initiate transfusion of 10 mL/kg of packed red blood cells
(pRBCs) if hypotensive or signs of hypovolemic shock after 40 mL/kg of
crystalloid is infused.
• Key pediatric anatomic and physiologic differences include:
• Children are smaller, so bodily force is more widely distributed, making
multi-system injuries more likely.
• The infant’s head-to-body ratio is greater, creating a relatively higher
center of gravity. This, combined with a less myelinated brain and thin-
ner cranial bones, predisposes infants to more serious head injury.
• Children have a higher anatomic fulcrum in the cervical spine (C2 to C3
in children <8 years old), resulting in higher cervical spine injuries.
• Children have greater laxity of the cervical column ligaments, leading to
a greater risk of spinal cord injury without radiographic abnormalities
(SCIWORA).
• Children have more horizontally positioned ribs, resulting in a more
upward movement during inspiration; this leads to a limited ability to
increase tidal volume and risk for respiratory failure with chest or dia-
phragmatic injury.
• Children have more elastic ribs, allowing for pulmonary injury without
skeletal injury.
• Children have thinner abdominal walls and a more anterior location of
the liver and spleen; this results in a greater chance of injury to those
organs.
• Despite large intravascular volume loss, children are able to substan-
tially increase their systemic vascular resistance to remain normotensive;
hypotension is a very late sign.
• Most minor head trauma may be managed with observation and without
computed tomography (CT) imaging. When applied, clinical decision rules
may reduce imaging and radiation exposure.
• In major trauma patients, indications for intubation include respiratory fail-
ure or a Glasgow Coma Scale ≤8.
• The diagnostic test of choice for the evaluation of intra-abdominal injury in
a stable patient with high suspicion for injury is abdominal CT.
• Solid organ injuries are generally treated nonoperatively in children. 2. Which of the following statements regarding imaging of a multi-
trauma pediatric patient is correct?
1. A 12-year-old male fell while climbing over a 12-foot barbed-wire a. A negative computed tomography (CT) scan of the cervical spine
fence and sustained a deep 10-cm laceration to his medial left thigh. rules out spinal cord injury, and if normal, immobilization can be dis-
Parents bring him to the ED directly from the scene. There is active continued.
oozing from the laceration. What is the first step in the manage- b. A negative focused assessment with sonography in trauma
ment of this patient? (FAST) examination rules out traumatic intra-abdominal injury,
a. Apply a tourniquet to the leg. making a CT scan unnecessary.
b. Begin with a primary survey and assess the patient’s airway and c. In a hemodynamically stable pediatric trauma patient, CT imag-
breathing. ing should be completed before transfer to a pediatric trauma
c. Obtain intravenous (IV) access and begin blood transfusion facility, even if it delays transfer.
immediately. d. In a hemodynamically stable pediatric patient with a high level
d. Pack the wound to decrease hemorrhage. of concern for intra-abdominal trauma, CT scan is the imaging
Answer: b. The primary survey should quickly assess the airway, breathing, test of choice.
and circulation (ABCs). Approaching trauma patients in a systemic fash- Answer: d. CT scan is the diagnostic test of choice for evaluation of
ion will ensure that large, obvious injuries do not distract from the detec- intra-abdominal trauma in children. Spinal cord injury without radio-
tion of other injuries. Oftentimes multiple interventions would be done logic abnormality (SCIWORA) is more common in pediatric patients,
simultaneously. Jaw thrust to open an airway and then another health care and a normal cervical spine CT scan does not rule out ligamentous
providers quickly applying pressure on the wound for hemorrhage control. injuries or spinal cord edema. Although often a useful adjunct, a FAST
The child in the question fell from a fence and may have other fall-related examination does not rule out intra-abdominal injury. In a hemody-
injuries that should be identified during the secondary survey. namically stable patient, CT imaging does not need to be completed
before transfer to a pediatric trauma center and definitely should not
delay transfer.
3. A 5-year-old male is struck by a car while riding a bicycle with no 6. A 4-year-old female is brought to your emergency department (ED)
helmet, presents with heart rate of 92 beats/min, blood pressure from the scene of a motor vehicle collision. Paramedics intubated
60/30 mm Hg, cervical spine immobilized with a collar, and quadri- the child in the field because she was unresponsive at the scene. On
plegia. The patient has no evidence of hemorrhage on examination arrival, her vital signs are within normal limits. Before transport to
or diagnostic testing. You suspect neurogenic shock and attempt to the computed tomography (CT) scanner, however, you note that
correct the hypotension with fluids but are not successful. Which of she is becoming mildly bradycardic and hypertensive. Her left pupil
the following should be your first choice for a vasopressor? becomes dilated and nonreactive. You elevate the head of bed 30
a. Dobutamine degrees and maintain ETCO2 at 35 mm Hg with controlled ventila-
b. Dopamine tion. Which of the following should be your next immediate action?
c. Epinephrine a. Mannitol or 3% hypertonic saline IV
d. Norepinephrine b. Nicardipine infusion
Answer: d. Patients with neurogenic shock lose their sympathetic c. Phenytoin administration
tone, manifested as the triad of hypotension, bradycardia and periph- d. Proceed immediately to the CT scanner
eral vasodilation. Treatment includes fluid administration, parasym- Answer: a. The sudden onset of a dilated, nonreactive pupil, along
pathetic receptor blocking agents (e.g., atropine), and vasopressors with bradycardia and hypertension, is indicative of acute brain herni-
with chronotropic, vasoactive, and inotropic characteristics (e.g., ation from raised intracranial pressure. Controlled ventilation to keep
norepinephrine). ETCO2 greater than 30 mm Hg, a hyperosmolar agent, such as man-
4. A 6-month-old infant is brought to your emergency department nitol or hypertonic saline given intravenously, and raising the head of
(ED) by his mother after he reportedly fell 3 feet from his changing the bed 30 degrees are recommended strategies. Phenytoin can be con-
table approximately 45 minutes ago. His mother reports that the sidered for seizure prophylaxis in this patient, but not before the other
child did not lose consciousness but is fussy and has a large scalp measures are instituted. Nicardipine and other antihypertensive agents
hematoma on the right side of head. On exam, the Glasgow Coma have no place in the management of a head-injured child. Hyperten-
Scale (GCS) is 14 (spontaneous eye and motor, irritable cries). The sion is a physiologic response to brain herniation, where increasing the
patient has a right sided 4 cm scalp hematoma in the temporal mean arterial pressure (MAP) preserves cerebral perfusion pressure
area but otherwise the examination is normal. What is your most (CPP).
important intervention at this point? 7. A 7-year-old girl is brought into the emergency department (ED)
a. Consulting a social worker to help screen for child abuse by emergency medical service (EMS) personnel from the scene of
b. Cranial computed tomography (CT) scan a motor vehicle collision (MVC). On your primary survey, she is
c. Ice packs for the scalp hematoma noted to have a patent airway, decreased breath sounds in the right
d. Observation in the ED for 4 hours lung field, and subcutaneous emphysema. She has hypotension and
e. Skull radiographs weak distal pulses. What is your next step in stabilizing this patient?
Answer: b. A 6-month-old infant with a GCS 14 has a 4% risk of clin- a. Airway control with endotracheal intubation (ETI)
ically important traumatic brain injury (TBI) by the Pediatric Emer- b. Bag-mask ventilation (BMV)
gency Care Applied Research Network rules. Additionally, the temporal c. Immediate needle thoracostomy in the second midclavicular
scalp hematoma is associated with a 1.6% risk of clinically important space on the right
TBI while a severe mechanism of injury carries a 0.5% risk of clini- d. Portable chest radiograph to confirm diagnosis
cally important TBI. Solely on the GCS 14 score, the infant needs a Answer: c. This patient is presenting with signs of tension pneumo-
non-contrast cranial CT to rule out TBI. If the infant had a GCS 15, thorax. In this case, immediate decompression with a needle thoracos-
and was acting normally, based on the severe mechanism alone, shared tomy, followed by the placement of an appropriately sized chest tube
decision-making with the parents to observe the infant in the ED is a or immediate chest tube placement is required to avoid cardiovascular
reasonable alternative to an immediate cranial CT scan. collapse. A portable chest radiograph should be performed to confirm
5. Which of the following is the most commonly injured solid organ in chest tube placement.
pediatric patients with abdominal trauma? 8. Which of the following statements regarding chest injuries in chil-
a. Bladder dren is correct?
b. Duodenum a. Aortic transection is more likely in a pediatric patient than in an
c. Kidney adult patient.
d. Liver b. Multiple rib fractures without significant underlying lung injury
e. Spleen are common in children.
Answer: e. Injuries to the spleen are the most common injuries in c. Penetrating chest trauma is more common than blunt chest
pediatric abdominal trauma. Children involved in motor vehicle colli- trauma in pediatric patients.
sions (MVCs), sudden deceleration injuries, and contact sports–related d. Significant pulmonary contusions may be present in the absence
injuries may sustain splenic trauma. Clinical exam findings include left of rib fractures in children.
upper quadrant abdominal pain that may radiate to the left shoulder. Answer: d. The pediatric rib cage has more compliance than an adult
Splenic injuries are best identified and characterized by abdominal rib cage. Thus children are predisposed to pulmonary injury in the
computed tomography (CT) scans. Treatment includes fluid resuscita- absence of rib fractures. Blunt chest trauma is more common than pen-
tion and blood transfusion as needed. Splenic salvage is important for etrating chest trauma in pediatric patients and concurrent chest and
immunocompetence in children and operative intervention is avoided abdominal injuries are common. Aortic transection is more common
as long as the patient can be stabilized with fluid resuscitation. in adults.
Pediatric Fever 161
KEY CONCEPTS
• F ever is the most common complaint among pediatric patients presenting to evaluation. Because UTI is still common in this population, a urinalysis should
the emergency department (ED). Although rates of bacterial illness are lower be obtained.
since the advent of universal vaccination for Haemophilus influenzae type b • There are numerous risk-stratification strategies (e.g., Boston, Rochester, and
and Streptococcus pneumoniae, serious bacterial infection (SBI) should be Philadelphia criteria) reported in the literature that have similar performance
considered in the under-vaccinated or unvaccinated child. characteristics. All involve a laboratory evaluation designed to identify a sub-
• Viruses cause the vast majority of childhood febrile illnesses and are gener- set of febrile infants younger than 3 months old that can safely be managed
ally self-limited and benign. as outpatients with or without antibiotics.
• SBI is the growth of pathogenic bacteria in a previously sterile site, such as • Standardization and adoption of a clinical practice guideline for the evalua-
urinary tract infection (UTI), bacteremia, meningitis, osteomyelitis, bacterial tion of the febrile infant have been shown to reduce variation and cost.
gastroenteritis, bacterial pneumonia, cellulitis, or septic arthritis. • Due to universal vaccination against pneumococcus, the evaluation of highly
• The rate of SBI in infants younger than 3 months old with fever is between 6% febrile children 3 to 36 months old has evolved from one of universal screen-
and 10%. ing for occult bacteremia to one where clinical assessment determines the
• Infants 28 days old and younger are at much higher risk for bacterial illness need for bloodwork.
with fever because of their immature immune systems and incomplete vacci- • Inflammatory markers, such as C-reactive protein (CRP) and procalcitonin,
nation status. have been shown to predict bacterial illness in febrile children more accu-
• Empirical treatment of a febrile neonate is indicated, and appropriate antibi- rately than the white blood cell (WBC) count but does not solely rule out SBI.
otic regimens include ampicillin plus either gentamicin or cefotaxime, which • Traumatic lumbar punctures occur relatively commonly in young infants and
cover the bacterial organisms likely in this age group. can make interpretation of cell counts difficult. The use of various formulas
• Empiric treatment for herpes simplex virus (HSV) in neonates with a maternal to account for the protein and WBCs in the cerebrospinal fluid (CSF) after a
history of genital herpes, ill appearance, fever and seizure, cutaneous vesicles traumatic tap should be used with caution.
on physical examination, transaminitis, or evidence of coagulopathy is recom- • As the risk of meningitis is exceedingly low, we do not recommend lumbar
mended. puncture in well-appearing children with a simple febrile seizure.
• The most common cause of SBI in children continues to be UTI. In infants • Children presenting with fever and petechiae are at risk for infection with
who are not toilet-trained, bladder catheterization is the best method for meningococcus; blood should include a complete blood count (CBC) and
collecting uncontaminated urine. However, for select patients, bagged urine culture and, if available, CRP and procalcitonin (>0.5 ng/mL). Children with
may be obtained, and if results do not indicate infection, no further studies an abnormal CRP, WBC count (<5000/mm3 or >15,000/mm3), or bandemia
are needed. Suprapubic aspiration may be considered when a urine sample should be treated with parenteral antibiotics and admitted. Lower risk, well-
cannot otherwise be obtained. appearing children with normal laboratory parameters can be considered for
• Bacterial meningitis can occur at any age, but most commonly presents in close outpatient follow-up.
a relatively small proportion of febrile infants younger than 3 months old • Children with fever who also are receiving cytotoxic chemotherapy for cancer
(3/1000). are at high risk for bacteremia and sepsis and should receive prompt broad-
• Respiratory syncytial virus (RSV) and influenza are common viral causes of spectrum antibiotic therapy after appropriate diagnostic evaluation (at mini-
fever and respiratory distress in infants, but the presence of viral infection mum, a CBC and blood culture).
does not lower the risk of concomitant SBI in children younger than 28 days • Patients with fever and a history of sickle cell disease are at risk for bacte-
old. remia from encapsulated organisms due to functional asplenia and should be
• In older infants and children, the documented presence of RSV or influenza considered high risk, admitted, and treated.
significantly reduces the incidence of SBI and can be used to modify the
1. Which of the following are appropriate methods to obtain urine as 2. A 38-day old male infant presents to the emergency department
part of a fever evaluation in a non-toilet-trained child? (ED) after parents noted a fever at home today. In the ED, the tem-
a. Catheterized specimen perature was noted to be 40.0°C. The child was born at 35 weeks
b. Clean catch, midstream gestation and 1 week ago was diagnosed with otitis media by his
c. Suprapubic aspiration primary care physician and has been taking amoxicillin as treat-
d. A and C ment. Which of the following historical aspects places this child at
Answer: d. The only reliable method to obtain sterile urine in a higher risk for invasive bacterial infection?
non-toilet-trained child is bladder catheterization or suprapubic aspi- a. Fever responsive to acetaminophen
ration if a catheter specimen cannot otherwise be obtained. Bag col- b. Temperature of 40.0°C
lection has a high rate of false-positive results and is not preferred in a c. Prior treatment with antibiotics
child who is not toilet trained. d. Prematurity
e. B, C, and D
Answer: e. Risk for serious bacterial infection (SBI) varies with age. has an associated maculopapular rash. On examination, you find
Children less than 3 months are at high risk for SBI, with those less the patient also has bilateral conjunctival injection, a strawberry
than 28 days of age at highest risk. Hyperpyrexia, prior treatment with tongue, and swelling of his hands and feet. Which of the following
antibiotics, and prematurity have also been identified as risk factors for medications should be included in the treatment of this patient?
SBI. Response to antipyretics has not been linked to SBI risk. a. Aspirin
3. Which of the following statements regarding occult bacteremia in b. Decadron
children younger than 36 months old is true? c. Penicillin G
a. Children with no obvious source of fever and a temperature d. Ceftriaxone
higher than 102.2°F (39°C) have an incidence of bacteremia of 5%. Answer: a. This patient has Kawasaki disease. Goals of treatment
b. Most patients appear toxic. include amelioration of symptoms and prevention of coronary aneu-
c. The most common pathogen is Neisseria meningitidis. rysms, which are normally accomplished with aspirin and immuno-
d. There has been a marked decrease in the incidence of occult bac- globulin. There are no indications for antibiotics or steroids.
teremia since the advent of universal vaccination against pneu- 5. A 2-year-old presents with a high fever and vomiting. On examina-
mococcus and Haemophilus influenzae type B. tion, you find an irritable child with a rectal temperature of 102°F
e. With pneumococcal bacteremia, most patients remain febrile rectal and a stiff neck. The patient’s past medical history is significant
until antibiotic therapy is initiated. for hydrocephalus with a ventriculoperitoneal shunt placement. You
Answer: d. Children younger than 36 months old with a fever higher suspect the patient has a ventriculoperitoneal shunt infection. Which
than 102.2°F (39°C) and no obvious source have an incidence of occult of the following is the most likely bacterial pathogen?
bacteremia of less than 1%. Most patients appear nontoxic. The most a. Haemophilus influenzae
common pathogen in positive cultures is Streptococcus pneumoniae, and b. Neisseria meningitidis
the incidence of infection has dropped dramatically since the advent of c. Staphylococcus aureus
universal vaccination. With pneumococcal bacteremia, most patients d. Staphylococcus epidermidis
become afebrile in 3 or 4 days with or without antibiotic therapy. Answer: d. Patients with ventriculoperitoneal shunts and fever
4. A 3- year-old boy presents with a fever of 103°F. His mother should be evaluated for shunt infection. The most common bacterial
reports that the fever started approximately 5 days ago, and he pathogen is S. epidermidis.
162 Pediatric Upper Airway Obstruction and Infections
KEY CONCEPTS
Respiratory arrest precedes most pediatric cardiac arrests. Quick recognition of • T reatment of moderate to severe croup includes vaporized epinephrine in
an airway problem and intervention in potentially life-threatening upper airway addition to glucocorticoids. Patients can be discharged from the ED after a
obstruction in children are critical. posttreatment observation period. We recommend discharge after a period of
observation, if the child is free of resting stridor and distress and has access
Retropharyngeal Abscess to follow-up care.
• This is a potentially life-threatening emergency in young children with signs
of upper airway obstruction or meningismus; a retropharyngeal abscess is Bacterial Tracheitis
often related to oral trauma. • Suspect bacterial tracheitis when an upper respiratory infection (URI) pro-
• Retropharyngeal abscess is most frequently caused by Staphylococcus gresses to acute toxicity and marked respiratory distress and stridor. Standard
aureus, group A streptococci, and anaerobes. Treatment is admission, IV anti- treatment for croup may be tried, but usually does not significantly improve
biotics, and for more severe cases, surgical drainage. the patient’s symptoms. Antibiotic therapy should include a cephalosporin
plus coverage for S. aureus, which is the most common cause of this infec-
Epiglottitis tion.
• Epiglottitis may be caused by many bacteria or local injury. In the post–Hae- • Bronchoscopy is diagnostic and therapeutic and should be emergently per-
mophilus influenzae type b vaccine era, the incidence of pediatric epiglottitis formed.
has decreased, and epiglottitis is now more likely in older patients.
• Clinical features of epiglottitis are often subtle, such as in the older adoles- Airway Foreign Body
cent, (e.g., sore throat out of proportion to physical findings, anterior neck • Complete obstruction due to an airway foreign body requires emergent life
tenderness), but may also be dramatic, as in infants and young children (i.e., support procedures for removal of the foreign body.
drooling, stridor, toxicity, severe respiratory distress). • Plain films may be negative in aspirated foreign bodies. Bronchoscopy should
be performed with a clinical suspicion of aspiration.
Croup • Emergency cricothyroidotomy may be required for obstructed patients who
• Viral croup is the most common infection of the upper airway in young chil- cannot be intubated or ventilated as a lifesaving temporizing measure; needle
dren. cricothyroidotomy is preferred for infants and young children because of the
• Glucocorticoids (usually given as a single oral dose of dexamethasone) reduce challenges in identifying landmarks and associated complications of surgical
symptoms, hospitalizations, and length of stay in the emergency department (ED). cricothyrotomy.
1. Which of the following is the most common cause of upper respira- Which of the following would be the most appropriate next step in
tory obstruction in childhood? management?
a. Airway foreign body a. Administer dexamethasone, observe the patient for 2 to 3 hours,
b. Bacterial tracheitis and discharge if well.
c. Croup b. Admit the patient for overnight observation.
d. Epiglottitis c. Allow the patient to go home.
Answer: c. Although all the choices may lead to symptoms of air- d. Allow the patient to go home with a prescription of steroids.
way obstruction in children, the most common cause of upper airway Answer: a. The child has croup and, after being treated with neb-
obstruction is viral croup. ulized epinephrine, should be observed for recurrent stridor after 2
2. A 3-year-old girl presents at 2 am with complaints of a barky hours. If the child is well, is in no respiratory distress, has a good hydra-
cough, which started abruptly overnight. Vital signs are heart rate, tion status, and is able to access emergency and follow-up care, he or
140 beats/min, respiratory rate, 40 breaths/min, and temperature, she can then be safely discharged.
100.1°F (38°C). She has no history of asthma or wheezing. She 4. What is the ideal head position to assess a pediatric soft tissue
appears to be in moderate distress and has audible stridor. Indica- radiograph of the neck for upper airway pathology?
tions for admission include which of the following? a. Extension during inspiration
a. Low-grade fever b. Flexion
b. Prior history of croup c. Flexion during inspiration
c. Stridor at rest d. Neutral
d. Severe dehydration Answer: a. Gentle extension of the head gives the most accurate images,
Answer: d. Indications for admission of patients with croup include avoiding the artificial soft tissue widening that can be seen in flexion. Inspi-
severe respiratory distress or failure, unusual symptoms (hypoxia and ration, if possible, allows maximal distention of the pharynx and the best
hyperpyrexia), anything but mild dehydration, persistence of stridor viewing of soft tissue structures defined by an air–soft tissue interface.
at rest after aerosolized epinephrine and steroids, persistence of tachy- 5. Which of the following factors is least consistent with the diagnosis
cardia or tachypnea, and complex medical history (e.g., prematurity, of peritonsillar abscess?
pulmonary or cardiac disease). a. Muffled, hot potato voice
3. A 5-year-old immunized boy presents with severe stridor at rest, b. Pain radiating to the ear
low-grade fever, and nasal congestion. His family reports a barky- c. Patient 3 years of age
sounding cough. After initiating vaporized epinephrine, he appears d. Patient 13 years old
well and is in no distress. The parents are asking about discharge. e. Trismus
Answer: c. Peritonsillar abscess more commonly occurs in older results are most likely to grow which of the following organ-
children and teenagers, whereas retropharyngeal abscess is more com- isms?
mon in a younger population. All the other signs or symptoms listed a. Bacteroides fragilis
are consistent with peritonsillar abscess, along with deviation of the b. Candida albicans
uvula away from the abscess side. c. Parainfluenza
6. A 3-year-old immunized girl presents after a brief viral illness d. Staphylococcus aureus
with progressive dyspnea, ill appearance, and high fever. The e. Streptococcus pneumoniae
child is relatively still, appearing as if she is trying not to cough. Answer: d. The case described is consistent with bacterial tra-
Stridor is heard, and she does not respond to croup therapy. cheitis. Although Candida, parainfluenza, and Streptococcus have all
You notify the operating room, where the patient undergoes been reported, S. aureus is most common. Broad-spectrum antibiotics
bronchoscopy, with suctioning and airway placement. Culture are appropriate, with an emphasis on covering S. aureus.
163 Pediatric Lower Airway Obstruction
KEY CONCEPTS
achieve the goal of three treatments within the first hour of care. Per the
• N o single asthma score has been universally adopted to assess the degree
2007 National Heart Lung and Blood Institute guidelines, a high dose (4–12
of illness or treatment responses. However, most scores include some com-
puffs) of a short-acting beta-agonists (SABAs) metered-dose inhaler (MDI)
bination of respiratory rate, degree of wheezing, inspiratory-to-expiratory
with a spacer has “equivalent bronchodilation” to nebulized treatment.
ratio, use of accessory muscles, and oxygen saturation.
• Levalbuterol does not lead to better emergency department (ED) outcomes
• Chest x-ray (CXR) is not required for wheezing children, even for those who
compared with racemic albuterol. Racemic albuterol, at a substantially
are febrile, are wheezing for the first time, or require hospitalization. CXR is
lower cost, should remain the drug of choice for children with acute asthma
indicated for those with a history of choking, focal chest findings, extreme
exacerbations.
distress, subcutaneous emphysema, diagnostic uncertainty relative to
• Dexamethasone is as effective as prednisone in the ED treatment of acute
respiratory illness, or with clinical findings suggestive of a cardiac etiology.
asthma. Dexamethasone is associated with fewer doses, less vomiting,
• Albuterol delivered by metered-dose inhalers with spacers (MDI-S) is as
and greater compliance.
effective as that delivered by nebulizers for children with acute asthma.
• Continuously nebulized albuterol, corticosteroids, magnesium sulfate, and
The mode of delivery is largely chosen on the basis of cost and ability to
parenteral SABA are cornerstones of therapy for moderately to severely ill
children with asthma.
1. A 19-month-old girl with a history of asthma presents with severe Answer: e. Most patients who receive a magnesium infusion at the
respiratory distress marked by wheezing, tachypnea, deep retrac- recommended dose will experience a clinically insignificant decrease
tions, and an oxygen saturation of 92% in room air. She fails to in blood pressure. This may be minimized by infusing the medication
improve 10 minutes after nebulized short-acting β2-agonist (SABA) over 20 minutes and by concurrently administering normal saline
therapy and repeatedly tries to pull off her face mask. Which strat- solution.
egy is likely to produce the most rapid clinical benefits in this set- 4. You are about to treat a child with acute asthma with albuterol.
ting? Which of the following factors is most important when considering
a. Administering IM terbutaline the method of aerosolized drug delivery?
b. Administering IV methylprednisolone a. About 20% to 30% of nebulized drug reaches the alveoli.
c. Administering the SABA via metered-dose inhaler (MDI) b. Children receiving β-agonists by nebulizer (NEB) have similar
instead of by nebulization outcomes compared with those using a metered-dose inhaler
d. Doubling the dose of SABA in the nebulizer reservoir (MDI-S).
e. Taking off her face mask and holding it close to her face to c. Children weighing >20 kg should receive a maximum of four
reduce her agitation puffs of albuterol via MDI-S.
Answer: a. This patient’s degree of bronchospasm and lack of coop- d. MDI-S therapy is more costly than NEB therapy.
eration makes it difficult to deliver aerosolized medication effectively. e. The use of MDI-S for acute asthma is not supported by national
In this setting, IM terbutaline is most likely to result in rapid broncho- guidelines.
dilation, making subsequent aerosolized therapy more effective. Answer: b. Clinical trials and systematic reviews have repeatedly
2. A 6-month-old male infant presents to the ED with his father, who shown these two forms of therapy to be equivalent. With NEB treat-
reports that the patient has been wheezing. His vital signs are a ment, less than 10% of nebulized drug reaches the alveoli. A recom-
temperature of 39.7°C, respiratory rate 62 breaths/min, and oxygen mended MDI-S dose for older children is eight puffs. Most studies
saturation of 90%. On examination, you find diffuse wheezing and demonstrate that MDI-S therapy is more cost-effective because of a
copious nasal secretions. While in the ED, the patient does not have slightly reduced need for hospitalization. National guidelines support
a wet diaper; his father reports that he has had decreased oral intake the use of MDI-S to deliver albuterol to children with acute asthma.
for the past 2 days. Which of the following findings with bronchiol- 5. A 7-month-old male infant was diagnosed 2 days ago with bronchi-
itis is not associated with the need for admission? olitis. He continues to have wheezing and increased work of breath-
a. Age ing, prompting his parents to bring him into the ED because of a
b. Decreased oral intake new-onset fever. Which of the following is the most likely second-
c. Oxygen saturation ary bacterial infection in this infant with bronchiolitis?
d. Respiratory rate a. Pneumonia
e. Temperature b. Acute otitis media
Answer: a. Patients often in need of admission for bronchiolitis c. Meningitis
include those with respiratory rates more than 70 breaths/min, O2 sat- d. Urinary tract infection
uration of 95% or less, age younger than 3 months, and poor feeding. Answer: b. Bacterial acute otitis media (AOM) is the most common
3. Which of the following side effects are seen with the use of intrave- condition associated with bronchiolitis, with a prevalence of up to 60%.
nous magnesium in the treatment of asthma in children? The bacterial pathogens are similar to those recovered in other children
a. Change in serum pH with AOM; thus, it should be treated according to standard recommen-
b. Development of a prolonged QT interval on the electrocardio- dations. Other concurrent bacterial infections are rare.
gram
c. Hypercalcemia
d. Hyperkalemia
e. Hypotension
Pediatric Lung Disease
164
KEY CONCEPTS
• D etermining the causative agent of pneumonia by clinical presentation and
radiographic findings is not reliable; empirical treatment is based on guide-
line recommendations and likely pathogens.
• Causative agents vary by age; viral agents predominate, especially in
younger children, and Streptococcus pneumonia is the leading bacterial cause
outside of the neonatal period.
• Infants and younger children with pneumonia may have subtle or nonspe-
cific symptoms and signs on presentation, and fever may be the only sign of
disease.
• First line therapy for the treatment of bacterial pneumonia in children is
amoxicillin for an outpatient and ceftriaxone or ampicillin for an inpatient.
• Pertussis should be considered in a young infant with a staccato cough or a chest radiograph by themselves would not be contraindications to
episodes of cyanosis. outpatient treatment.
• In patients with cystic fibrosis, defects in chloride transport across the 3. Which of the following statements best describes the epidemiology
airway epithelium result in reduced ciliary clearance of thickened mucus, of pneumonia in children?
which leads to an increased likelihood for pneumonia, especially that a. Bordetella pertussis is the most common cause in infants.
caused by Pseudomonas aeruginosa. b. Haemophilus influenzae type b is still an important pathogen.
• Cystic fibrosis may respond favorably to bronchodilator therapy and mucol- c. Listeria monocytogenes may cause illness in children younger
ytics, such as inhaled N-acetylcysteine. than the age of 5 years.
• Patients with bronchopulmonary dysplasia have increased airway resis- d. Viral agents are the most common cause of pneumonia in chil-
tance, decreased lung compliance, and obstructive lung disease; reactive air- dren overall.
way disease and pneumonia are common in these patients. Answer: D. Viral agents are the most common cause of pneumonia in
children. Bacteria predominate in neonates but are less common caus-
ative agents in toddlers and older children. Although L. monocytogenes
1. A 5-month-old boy presents with a cough. His parents report that may cause pneumonia in infants, it is rare after 3 months of age. H.
for the past several weeks he has had mild respiratory tract symp- influenzae type B has decreased by 90% since the onset of immuniza-
toms and cough; however, during the past day, he has developed a tion of infants and young children.
severe paroxysm of staccato cough followed by posttussive emesis. 4. What is the most common viral agent causing pneumonia in infants
What is the most appropriate antibiotic choice for this patient? younger than 1 year?
a. Amoxicillin a. Adenovirus
b. Ampicillin b. Enterovirus
c. Azithromycin c. Epstein-Barr virus
d. Trimethoprim-sulfamethoxazole d. Respiratory syncytial virus (RSV)
Answer: C. This patient likely has pneumonia caused by Bordetella
Answer: D. RSV and parainfluenza are the most common viral agents
pertussis. All children younger than 6 months with presumed pertus-
in infants younger than 1 year. Viruses that may be responsible for the
sis should be observed in the hospital for monitoring and supportive
other agents may also cause viral pneumonia but are not as common
care. First line treatment is with azithromycin, and erythromycin is a
possible alternative. Amoxicillin and ceftriaxone are used to treat other as RSV.
types of pneumonia. 5. A 2-year-old presents with fever, cough, and rales. Chest radio-
2. Which of the following findings is an indication for admission to graph reveals right middle lobe pneumonia. Which of the following
the hospital in children with a diagnosis of pneumonia? antibiotics would be recommended for outpatient treatment for this
a. Abnormal chest radiograph showing pulmonary infiltrates toddler who is not penicillin allergic?
b. Decreased breath sounds in a lower lung field a. Amoxicillin
c. Dehydration and vomiting b. Azithromycin
d. Diffuse rales and mild tachypnea c. Ceftriaxone
Answer: C. Many children with pneumonia can be managed as an d. Cephalexin
outpatient with appropriate therapy. Clinical dehydration or inability Answer: A. High-dose amoxicillin is the treatment of choice for chil-
to orally maintain hydration should prompt admission for ongoing dren with pneumonia younger than 5 years. Once the child reaches
treatment and monitoring. Abnormal lung auscultation or findings on school age, azithromycin is suggested as empirical therapy because of
the increased risk of Mycoplasma pneumoniae infection.
Pediatric Cardiac Disorders 165
KEY CONCEPTS
• A cute bacterial endocarditis should always be considered in a child with a
• C onsider a congenital heart defect in an infant with central cyanosis who
known congenital heart defect or an acquired cardiac defect who presents
does not respond to 100% supplemental oxygen (hyperoxia challenge).
with fever of unknown origin, acute neurologic deficits, new-onset micro-
• Neonates with ductal-dependent cardiac lesions typically present within
scopic hematuria, myalgias, splenomegaly, petechiae, or other signs of
the first 2 to 3 weeks of life with either acute cyanosis or shock. Prosta-
systemic embolization.
glandin E1 (alprostadil, 0.05 to 0.1 μg/kg/min) can maintain a patent ductus
• Oxygen, positive-pressure ventilation (noninvasive or invasive), diuretics,
arteriosus to supply mixed blood and temporize the patient.
and possibly inotropes are the main emergency department (ED) treatment
• Treatment of a hypoxic “tet spell” first includes the placement of an infant
of infants and children who present with congestive heart failure (CHF).
in the knee-to-chest position (or an older child in a squatting position)
Nitroglycerin can cause profound hypotension in children and is not a first-
to increase systemic vascular resistance (SVR) and the provision of sup-
line therapy.
plemental oxygen. Sedative agents can be used to decrease hyperpnea.
• If vagal maneuvers fail to convert stable paroxysmal supraventricular tachy-
Various medications can be used as adjunctive treatment to increase the
cardia in children, rapid adenosine administration (0.1 mg/kg for the first
SVR and thereby decrease the degree of right-to-left shunting across the
dose, followed by 0.2 mg/kg on repeated doses) is the treatment of choice.
ventricular septal defect (VSD).
Verapamil should be avoided in children because of its profound hypoten-
• Prompt recognition of the clinical findings and symptoms of Kawasaki dis-
sive effects.
ease along with the rapid initiation of high-dose aspirin and intravenous
• Young athletes with a positive family history of sudden unexplained death
immune globulin (IVIG) infusion may prevent the formation of coronary
or exertion-induced symptoms (such as, chest pain, dyspnea, palpitations,
aneurysms.
and syncope) should be evaluated by a cardiologist before their resumption
of vigorous activity.
KEY CONCEPTS
• P hysiologic jaundice of the newborn and breast milk jaundice are the most • C hildren with intussusception may present atypically, with an altered level of
common causes of jaundice in the neonatal period. consciousness (e.g., lethargy) rather than abdominal pain.
• Direct hyperbilirubinemia in infants is always pathologic and requires a • Hirschsprung disease is a pathologic cause of constipation in the neonate and
detailed evaluation. usually manifests as delayed passage of meconium. Occasionally, children
• Hypertrophic pyloric stenosis is associated with gradually progressive nonbil- may present later in life with symptoms of chronic constipation. Toxic mega-
ious emesis that becomes projectile. colon is the most serious complication.
• Hypochloremic-hypokalemic metabolic alkalosis is the classic electrolyte • Meckel diverticulum classically manifests in children younger than 5 years
derangement associated with hypertrophic pyloric stenosis. with massive, painless, “brick red” colored rectal bleeding.
• Bilious vomiting in the neonate is an ominous sign and should initiate a diag- • More than 90% of GI foreign bodies pass without complications.
nostic evaluation for possible malrotation with volvulus or other intestinal • Lithium button batteries lodged in the esophagus may cause serious burns,
obstructive pathology. erosions, and perforations within as little as two hours. Of all foreign bodies,
• Infants with bilious emesis should receive an emergent surgical consultation, button batteries require the most expeditious removal, usually by endoscopy.
particularly if ill-appearing. • Appendicitis is the most common surgical disease in children. Diagnosis
• Necrotizing enterocolitis (NEC) occurs more commonly in premature infants, depends on a combination of clinical factors, including history, physical exam-
but 10% of affected infants are full term. Pneumatosis intestinalis in neo- ination, laboratory values, and imaging studies.
nates—intramural air seen on x-ray—is pathognomonic for NEC. • Effective imaging strategies for children with suspected appendicitis include
• Gastroesophageal reflux (GERD) is very common in infants and is usually initial ultrasound examination followed by CT scanning of the abdomen for
benign and self-limited. Occasionally, GERD may cause more severe symp- those with equivocal findings.
toms, including irritability, respiratory distress, and failure to thrive. GERD • Causes of pancreatitis in children include viruses, trauma, drugs, and toxins.
usually responds to conservative measures (e.g., positioning, thickening of • Biliary disease in children is more commonly caused by cholestasis rather
formula, smaller and more frequent feedings); pharmacologic interventions than biliary obstruction.
are seldom needed. • Pigment gallstones are more common than cholesterol stones in children. Bil-
• The classic clinical triad of intussusception includes colicky, intermittent iary tract disease is usually diagnosed with right upper quadrant ultrasound
abdominal pain, a palpable sausage-shaped abdominal mass, and bloody imaging; management strategies are similar to those for adults.
“currant jelly” stools; however, this triad occurs in less than one-third of
patients.
1. What is the most common cause of jaundice in the newborn? 3. An 11-month-old infant presents with vomiting. The patient’s
a. Breast milk jaundice mother reports that he has been crying out in pain intermittently
b. Crigler-Najjar syndrome throughout the day, at which times he brings his knees to his abdo-
c. Gilbert syndrome men. In between these episodes, the patient acts normally and
d. Physiologic jaundice of the newborn plays. He has not had a fever, but the mother complains that his
Answer: D. Although each of these may be a cause of hyperbilirubin- stool earlier looked like currant jelly. On examination, you find a
emia in the newborn, the most common cause of jaundice is physio- playful afebrile patient, with a soft nontender abdomen. Which of
logic jaundice of the newborn. the following may be used as an initial screening examination?
2. A 4-week-old white infant presents with projectile vomiting. The a. Air-contrast enema
mother denies that the patient has a history of fevers, irritability, b. Barium enema
or signs suggestive of abdominal pain. On physical examination, c. Computed tomography scan of the abdomen-pelvis
you palpate an olive in the patient’s right epigastrium. Which of the d. Ultrasound
following laboratory abnormalities would you expect to find? Answer: D. The initial screening examination for intussusception
a. Hyperchloremia and hypokalemia is abdominal ultrasonography. Although each of the other imaging
b. Hyperglycemia and hypokalemia modalities may be useful to exclude other diagnoses or identify intus-
c. Hypernatremia and hyperkalemia susception, ultrasonography has high sensitivity and specificity and is
d. Hypochloremia and hypokalemia the screening modality of choice.
Answer: D. This patient likely has pyloric stenosis. As vomiting con-
tinues, the infant loses hydrogen and chloride ions through emesis
of gastric juices. As this metabolic derangement worsens, the kidney
attempts to retain hydrogen ions by substituting potassium, resulting
in a hypochloremic-hypokalemic metabolic alkalosis.
Pediatric Infectious Diarrheal
Disease and Dehydration 167
KEY CONCEPTS
Identification of Pathogen subsides. Routine fasting with infectious diarrhea is not
• Stool studies are not indicated in most uncomplicated cases of recommended.
acute gastroenteritis (AGE). Exceptions are those cases in which Dehydration Assessment
specific treatment, specific prophylaxis, or health precautions • The degree of volume depletion is estimated from the history
are required, or in which the patient has systemic involvement, and physical examination findings. The desired volume of oral
underlying medical complications, or dysenteric features. rehydration solution is calculated as 30 to 50 mL/kg for mild
• Antibiotics are not required for most cases of uncomplicated dehydration and 60 to 80 mL/kg for moderate dehydration;
acute bacterial enteritis. Antibiotics are recommended routinely 25% of the volume of oral rehydration solution is to be replaced
for C. difficile, Giardia intestinalis, and E. histolytica. Antibiotics every hour (100% over 4 hours). Continue to replace ongoing
can be considered for Campylobacter, Cryptosporidium, traveler’s losses with 10 mL/kg for each diarrheal stool and 2 mL/kg for
diarrhea, and Shigella (because antibiotics have been shown to each vomiting episode. Patients who fail an oral rehydration trial
decrease diarrhea and eradicate organisms in the stool). of 4 to 8 hours in the emergency department (ED) should be
• Patients with Shiga toxin–producing E. coli (STEC) should not admitted for intravenous hydration.
empirically receive antibiotics, because they may increase the Severe Dehydration
risk of hemolytic-uremic syndrome (HUS). • In severe dehydration, 20 mL/kg of 0.9% saline (or other
• Testing for fecal leukocytes is a useful initial test because it appropriate isotonic crystalloid solution) given intravenously
may support a diagnosis of invasive disease. This test should or intraosseously should reverse signs of shock within 5 to 15
be considered in children with diarrhea who are febrile or have minutes. Repeated boluses of 20 mL/kg are indicated until clinical
mucus or blood in the stool. If the test result is positive, stool improvement occurs, but volume requirements greater than 60
culture is indicated to further guide management. mL/kg without signs of improvement suggest other conditions,
Oral Rehydration such as septic shock, hemorrhage, capillary leak with third-space
• Most patients with mild to moderate dehydration can be treated fluid sequestration, and adrenal insufficiency. Rapid correction of
with oral rehydration therapy (ORT). Resumption of feeding sodium derangements in dehydration can lead to central nervous
with age-appropriate diets should begin as soon as vomiting system complications.
1. For which of the following patients should the emergency clinician within 5 to 7 days; bacteremia; focal infection in the central nervous
not obtain diagnostic testing of the stool? system (CNS), bone, joint, kidney, or pericardium; or those with immu-
a. 1-month-old with fever to 38°C with bloody diarrhea nosuppressive conditions, hemoglobinopathies, malignant neoplasms,
b. 8-month-old with sickle cell disease with fever and diarrhea human immunodeficiency virus (HIV), or chronic gastrointestinal
c. 12-month-old, well-appearing child with 2 days of watery diar- disease. The recommended antibiotics are for unknown susceptibility
rhea or in areas of high resistance to use ceftriaxone or cefotaxime until sus-
d. 2-year-old with nephrotic syndrome on prednisone with diar- ceptibility is known. Ampicillin and trimethoprim-sulfamethoxazole
rhea and fever for 3 days (TMP-SMX) may also be effective, but they should be used only in
e. Community outbreak of diarrhea high-risk individuals once susceptibility is known.
Answer: C. Most children with uncomplicated gastroenteritis or 4. A 5-month-old infant presents with persistent vomiting and diarrhea
acute diarrhea do not need laboratory studies. Stool cultures are useful for 3 days. The parents came to the emergency department because
and important to obtain in febrile infants and children with blood in their child suddenly seemed to be passing out. Parents have noted
their stools, in community outbreaks, and in the immunosuppressed. significant weight loss and poor breastfeeding. Exam shows an
2. A 4-year-old child presents with moderate dehydration from gas- unconscious infant with doughy skin and dry mucous membranes.
troenteritis. He is still actively vomiting. Which of the following IV access is obtained, the airway is secured, and a normal saline infu-
statements regarding this patient’s rehydration is correct? sion is started. What central nervous system complication of dehy-
a. A nasogastric tube should be placed for rehydration. dration is most likely in this patient?
b. Oral rehydration should not be started. a. Cerebral edema
c. Oral rehydration success can be determined in 2 or 3 hours. b. Central pontine myelinolysis
d. Oral rehydration target volume is 80 mL/kg. c. Hypoxic ischemic encephalopathy
e. Target oral rehydration volume should be administered over 2 d. Primary epilepsy
hours. e. Subdural hematoma
Answer: D. Oral rehydration can be effective even in the face of Answer E: The clinical picture suggests hypernatremic dehydra-
vomiting in cases of mild or moderate dehydration. Target oral volumes tion (infant with poor breastfeeding, doughy skin, significant weight
are 60 and 80 mL/kg, respectively. Nasogastric tubes may occasionally loss). Hypernatremia leads to dehydration and shrinking of the brain,
be useful but should not be used unless oral rehydration therapy (ORT) which increases the risk of injury to bridging veins, subdural hema-
fails. The target oral volume should be given in doses of 25% of total toma, and CNS thrombi. Cerebral edema is a complication of hypona-
replacement per hour over 4 hours. tremic dehydration (more often in older children with skin tenting on
3. A 9-year-old child with a history of sickle cell disease presents for examination). Central pontine myelinolysis is a complication of rapid
evaluation of fevers, diarrhea, and vomiting. The parents say the sodium correction in hyponatremic dehydration. HIE and epilepsy are
patient has not had a sickle cell crisis in almost a year. On examina- not complications of dehydration alone.
tion, the vital signs are stable and the child is nontoxic-appearing 5. A 4-year-old girl presents with acute gastroenteritis. On history and
with mild dehydration. His abdominal examination is benign. In examination the child has lost 12% of her weight from her last pri-
addition to rehydration, which of the following would be the most mary care appointment 3 weeks ago. She appears lethargic and has
appropriate management and disposition for this child? mottled skin with a capillary refill of 5 seconds. Multiple attempts
a. Obtain blood cultures and admit for observation. at IV access are unsuccessful. What is the next best approach for
b. Obtain cultures (stool and blood), begin antibiotics, continue rehydration?
hydration, and admit for observation. a. Hypodermoclysis (subcutaneous rehydration)
c. Obtain electrolytes; and if he has a normal bicarbonate level, b. Obtain intraosseous access
then discharge home. c. Oral ondansetron
d. Perform a stat Rotazyme assay; and if negative and patient toler- d. Nasogastric tube insertion
ates oral intake, then discharge home. e. Transfer to a pediatric tertiary hospital
e. Provide oral rehydration therapy (ORT); and if successful, then Answer B: Patients with severe dehydration (>10 % body weight
discharge home. loss) or evidence of decreased perfusion (poor capillary refill, lethargy)
Answer: B. Patients with sickle cell disease are more susceptible to should have emergent infusion of isotonic crystalloid at a rapid rate to
Salmonella infections and are at increased risk for complications. It has restore perfusion. In this patient, the fastest method to achieve this is
been shown that giving antibiotics to patients with nontyphoidal Sal- with intraosseous access. In a conscious patient, the line can be primed
monella has been ineffective in shortening the duration of symptoms with lidocaine for 1 to 2 minutes before the infusion to help with the
and prolonging the carrier state. Therefore, antibiotics are generally pain of the infusion. Hypodermoclysis, nasogastric tube rehydration,
not recommended for asymptomatic cases or for uncomplicated cases. and oral rehydration are all appropriate management options for mild-
However, antibiotic treatment is indicated in infants younger than 3 moderate dehydration. Transferring the patient before IV rehydration
months old or those with complications, such as failure to improve would delay her care and increase her mortality risk.
168
Pediatric Genitourinary
and Renal Tract Disorders
KEY CONCEPTS
• Ischemic priapism is a urologic emergency due to compartment syndrome
of the penis and is managed with local analgesia with or without sedation,
cavernosal aspiration, irrigation, and possibly injection with phenylephrine.
1. A 12-year-old male presents due to right-sided scrotal pain for the 3. You are caring for a 3-year-old boy who was brought in by his
last 2 days. On examination you note a swollen right testicle with mother for a possible abdominal mass palpated during his bath
normal cremasteric reflex that has a “bag of worms” consistency on tonight. As part of your workup you obtain a renal bladder ultra-
examination. What is your next step in this patient’s workup? sound that demonstrated a right-sided renal mass. What is the most
a. Further imaging of the abdomen likely etiology of his renal mass?
b. Further imaging of the testicles a. Congenital mesoblastic nephroma
c. Mumps titers b. Neuroblastoma
d. Urine to assess for Neisseria gonorrhea and Chlamydia tracho- c. Renal cell carcinoma
matis d. Wilms tumor
Answer: A. Further imaging of the abdomen. The patient described Answer: D. Wilms tumor. The most common renal cancer in chil-
has a right-sided varicocele. Most patients have a left-sided varico- dren is Wilms tumor. The second most common cancer in patients <12
cele due to the angle of drainage of the left spermatic vein to the left months is congenital mesoblastic nephroma and <5 years is renal cell
renal vein; however, a right-sided and abrupt onset varicocele warrants carcinoma.
abdominal imaging to investigate for an intra-abdominal mass. Imag- 4. Patients with nephrotic syndrome are at risk for which complica-
ing of the testicle would be appropriate to investigate for testicular tor- tion?
sion, but abdominal imaging is still necessary to rule out mass in this a. Henoch-Schönlein purpura
presentation. Mumps may present with orchitis, but not usually with b. Supraventricular tachycardia
varicocele. STIs do not usually present as varicoceles and in absence of c. Thrombosis
other symptoms testing is not necessary at this time. d. Vascular malformations
2. A family brings their 10-day-old girl into the ED for a newly devel- Answer: C. Thrombosis. Due to increased permeability of the base-
oped bulge in the right inguinal area which appeared about an hour ment membrane, patients lose antithrombin 3 molecules which pre-
ago and is not reducible. Patient has otherwise been feeding well disposes them to thrombosis. Patients are also at risk for infections,
and afebrile prior to this. On examination, the infant is fussy but fluid overload, and electrolyte abnormalities. In general, patients do
consolable and has a palpable lump in the right inguinal region not have increased risk of Henoch-Schönlein purpura, supraventricu-
which is not reducible and has no overlying erythema. What is your lar tachycardia, or vascular malformations.
next step in care? 5. A 5-year-old male presents with 5 days of diarrhea followed by
a. Full septic workup with CBC, UA, blood culture, and CSF stud- 1 day of bloody diarrhea, fever, and abdominal pain. Labs reveal
ies acute kidney injury, thrombocytopenia, and schistocytes on smear.
b. Pain control and outpatient follow up Further workup most likely reveals which pathogen?
c. Surgery consult a. Campylobacter
d. Ultrasound with Doppler b. E. coli 0157
Answer: D. Ultrasound with Doppler. Inguinal hernia in females c. Salmonella
may contain ovary in the hernia sack due to failure of closure of the d. Shigella
canal of Nuck, whereas hernias in males are more likely to have intes- Answer: B. E. Coli 0157. Most cases of HUS occur secondary to
tines. All females with inguinal hernia should have an ultrasound with Shiga toxin–producing E. coli 0157 and less frequently due to Shigella
Doppler to assess contents and assess for ovarian viability because or Salmonella. Though Campylobacter can result in bloody diarrhea, it
torsion could occur. These cases do require surgical consultation after does not usually cause hemolytic uremic syndrome as described with
ultrasound as ovarian hernia will require more urgent surgical man- acute kidney injury, thrombocytopenia, and microangiopathic hemo-
agement. Sending the patient home with incarceration would not be lytic anemia.
appropriate in this case. Full septic workup is not indicated in an afe-
brile otherwise well child without signs of infection.
169 Pediatric Neurologic Disorders
KEY CONCEPTS
• A ltered mental status in children has a varied spectrum of clinical presenta- • A pproximately 45% to 60% of all childhood brain tumors arise in the brain-
tions, and may include any of the following: altered level of consciousness, stem or cerebellum and can manifest with slowly progressive ataxia.
excessive sleepiness, irritability, lethargy, and abnormal behavior. • When assessing an infant or child with motor weakness, it is important to
• A careful and detailed history is instrumental in determining whether an distinguish presentations consistent with upper motor neuron pathology from
event was a seizure. lower motor neuron processes.
• Status epilepticus constitutes a neurologic emergency that carries high mor- • Strokes represent a pediatric neurologic emergency and may be hemorrhagic
bidity and mortality rates. Initial treatment is typically with IV benzodiaze- or ischemic in nature. Imaging with CT or MRI can help confirm the diagnosis
pines, followed by fosphenytoin or levetiracetam. If the seizure continues to of stroke. Children with stroke may present with less specific signs such as
be refractory after a second-line agent, the patient may require airway man- headache, seizure, or alteration level of consciousness.
agement. • Children presenting with suspected or confirmed strokes should be emer-
• A simple febrile seizure is generalized, lasts less than 15 minutes, and occurs gently transferred to a pediatric stroke center for timely consideration of ther-
in a neurologically and developmentally normal child between 6 months and apies.
60 months of age. • Spinal cord compression is a medical emergency and requires prompt diag-
• Breath-holding spells occur in children 6 months to 6 years of age, and are nosis and treatment. It may arise from trauma, infection and inflammation, or
triggered by pain or emotional upset. After a trigger, the child becomes pale malignancy.
or cyanotic and may lose consciousness, sometimes with a brief period of • The diagnosis of Guillain-Barré syndrome (GBS) is largely clinical, although
clonic movements or opisthotonos that may mimic a seizure. lumbar puncture (LP) may be helpful in confirming the diagnosis. Patients with
• Warning signs of secondary headaches include sudden onset, occurrence GBS are at risk for respiratory compromise and should be admitted to the
with straining or exertion, association with neurologic symptoms, change in hospital for observation and supportive care.
headache pattern, nocturnal awakening, worsening in a recumbent position, • The diagnosis of infant botulism is largely clinical. If there is high clinical sus-
and bilateral occipital headaches. picion, treatment should be initiated promptly, without awaiting laboratory
• If there are red flags on the history or physical exam, radiologic evaluation by confirmation. Given the risk of respiratory compromise, infants with botulism
computed tomography (CT), magnetic resonance imaging (MRI), or both may should be admitted to the hospital for observation and supportive care.
be necessary to rule out secondary causes of headache, such as intracranial • Diagnosis of myasthenia gravis is often not confirmed in the ED. The disor-
hemorrhage, subarachnoid hemorrhage, brain tumor, or brain abscess. der can often be treated on an outpatient basis, but patients with truncal
• A toxicology screen is the test with the highest diagnostic yield for acute- involvement and concern for respiratory compromise should be admitted to
onset ataxia in children. the hospital for observation and supportive care.
• In children, 40% of ataxia cases are caused by acute cerebellar ataxia.
1. Which of the following clinical findings of bacterial meningitis is Answer: E. Additional diagnostic testing should be obtained because
more common in newborns compared with infants and children? this child has exhibited a prolonged postictal period and has not
a. Hypothermia returned to baseline.
b. Neck ache 3. What is the appropriate treatment for a 12-year-old boy with a
c. Rash history of epilepsy and brought from home by emergency medical
d. Shaking chills services to the emergency department (ED) after multiple seizures
e. Tachycardia within the 15 minutes. The child is still unconscious and seizing.
Answer: A. Hypothermia is more commonly found in newborn infants Which of the following is the next step in the management of this
with bacterial meningitis compared with older infants and children child on arrival at the ED?
with a similar disease. a. Benzodiazepine
2. A 15-month-old child is seen with a history of one 10-minute tonic- b. Electroencephalography (EEG) and CT scan of the head
clonic event, which was nonfocal and accompanied by a tempera- c. Intubation using etomidate and vecuronium
ture of 39°C (102.2°F) and period of illness preceding this event. d. Methohexital
Two hours after the event, the child is still unresponsive to external e. Propofol
stimulation. Which of the following is the next step in the manage- Answer: A. If the airway cannot otherwise be maintained, there is
ment of this child? respiratory failure, or there is evidence of increased intracranial pres-
a. Admit to the inpatient service for antibiotics after blood cultures sure, the patient should be intubated. Do not paralyze the patient
are drawn. unless absolutely necessary. If needed, consider short-acting neuro-
b. Admit to the inpatient service for continued observation. muscular blockers such as succinylcholine and vecuronium. The three
c. Carry out imaging by obtaining an MRI of the brain. most commonly used agents to treat convulsive status epilepticus are
d. Obtain a complete blood count (CBC) and urinalysis if the child benzodiazepines, fosphenytoin, and levetiricam.
does not improve in 1 hour.
e. Order neuroimaging, a lumbar puncture, complete blood count,
and blood and urine cultures to evaluate for infectious causes.
4. A 3-year-old boy is seen for seizure-like activity at home. He has Answer: A. Most children with ataxia are seen in the first few days after
no prior history of epilepsy and no family history of seizures. His onset, usually because of a refusal to walk, unsteadiness of arm move-
fever has been as high as 39°C (102.2°F), and his parents have been ments, or the sudden development of a wide-based “drunken” gait.
watching him at home. Seizure activity has ceased and the history History should include recent infection, injury, inadvertent drug inges-
and physical examination are normal; the child is fully immunized tion, or other family members with the same problem. Children with
for age. Which of the following statements is not consistent with ataxia usually need admission for a complete evaluation. Consultation
appropriate counseling of parents regarding their 3-year-old? with a pediatric neurologist should be sought for patients in whom the
a. Although acetaminophen may make him feel better, it will not cause of the ataxia is not evident on ED evaluation.
prevent recurrence. 7. A 14-year-old girl with a history of migraines presents to the ED
b. He is at a substantially higher risk for developing epilepsy later in with the complaint of being unable to move her left side following
life. a severe right-sided headache. Which of the following statements
c. It is safe for him to go home with the parents. best describes her management?
d. Seizures with fever are common in young children and do not a. Imaging, analgesics, and admit to the hospital because symp-
lead to brain damage. toms may last for days.
e. There is no need to start any antiepileptic medication. b. Immediate evaluation with neuroimaging for a stroke
Answer: B. In general, a child between 6 months and 5 years who has c. Inpatient vasculitis evaluation is indicated.
a normal neurologic examination and has had a brief seizure in the d. She can be safely discharged home, and the symptoms will
setting of fever can be assumed to have a simple febrile seizure. Future resolve in an hour.
epilepsy is not predicted and is unlikely in children with simple febrile Answer: B. Hemiplegic migraine is characterized by the sudden onset
seizures. of hemiparesis or hemisensory loss followed by headache in the con-
5. Which one of the following statements best describes the signs and tralateral hemisphere. This type of migraine is seen more frequently in
symptoms associated with pediatric brain tumors? children than adults, but imaging should be obtained to rule out the
a. At the time of diagnosis, most patients with brain tumors have diagnosis of stroke. Although symptoms usually last for hours or days,
associated symptoms, such as nausea, vomiting, visual effects, affected individuals are rarely left with permanent deficits.
problems with walking, weakness, changes in personality or 8. A mother brings in her 4-year-old son for periods of lip smacking
school performance, or speech changes. that occur multiple times per week and last several minutes. During
b. Headaches are never the first symptom of a pediatric brain these episodes, he may turn his head to her when she calls, but he
tumor. does not speak. You suspect which of the following conditions?
c. Most pediatric brain tumors are diagnosed within the first a. Complex partial seizure
month after symptom onset. b. Generalized seizure
d. Papilledema is one of the least common neurologic findings in c. Infantile spasm
children with brain tumors. d. Lennox-Gastaut syndrome
Answer: A. Of patients with brain tumors, most have some of the asso- e. Simple partial seizure
ciated signs or symptoms noted in choice A. Headaches may be the first Answer: A. In complex partial seizures, the patient has a change in
symptom of a pediatric brain tumor, although the frequency of this level of awareness and may exhibit bizarre behaviors including star-
presentation increases with age. Papilledema is one of the more com- ing, lip smacking, wandering, or picking at clothing. In simple partial
mon neurologic findings, along with abnormal eye movements, ataxia, seizures, the patient has no change in mentation. Lennox-Gastaut syn-
abnormal tendon reflexes, and abnormalities on the visual examina- drome is characterized by mental retardation, multiple seizure types,
tion. CT scanning is usually not performed with the first sign of a head- and a classic EEG pattern of slow spike and wave. Infantile spasms
ache unless clinically indicated. manifest during the first year of life and consist of rapid, jackknife
6. A 10-year-old boy is brought in by his mother for gait imbalance. flexor or extensor spasms that appear in clusters.
He is alert and oriented to person, place, and time, and he has a
negative drug and alcohol screen. He has a persistent wide-based
unsteady gait. What should be the next step in management?
a. Head imaging, admission, and neurologic evaluation
b. Discharge home if CT scan is negative
c. Perform a lumbar puncture
d. Reassurance and primary care referral
e. Start sepsis evaluation
Pediatric Musculoskeletal Disorders 170
KEY CONCEPTS
• T he pediatric physis is the weakest part of the bone and more likely to sep- limited range of motion, but pain varies and patients can usually still ambu-
arate before adjacent tendon or ligament tears, occurring more frequently late with a limp. Approximately 50% to 60% of patients not treated for
during periods of rapid growth. early stages of Lyme will go on to develop Lyme arthritis, which can be their
• Displaced supracondylar fractures are at greater risk for neurovascular presenting symptom of the disease.
injury and compartment syndrome. A lateral elbow radiograph with an ele- • Little League elbow describes a group of elbow injuries, including apoph-
vated fat pad is suspicious for occult fracture. ysitis, medical epicondylitis, and osteochondritis dissecans of the radial
• Transient synovitis has a peak presentation between 3 and 6 years of age. head and capitellum. The pediatric elbow is vulnerable to overuse injury,
History, physical examination, radiographs, and laboratory values can help because it has multiple muscle and ligamentous attachments as well as six
distinguish from septic arthritis and Lyme arthritis. ossification centers that close at different ages of skeletal maturity.
• Slipped capital femoral epiphysis (SCFE) is the posterior and inferior • Apophyseal injuries of the hip occur at the multiple sites of muscle origina-
slippage of the proximal femoral epiphysis on the metaphysis, occurring tion or insertion including on the pelvis. Athletes most at risk are dancers,
through the epiphyseal plate. This condition affects boys at twice the rate distance runners, and those participating in kicking sports.
of girls, occurring more commonly between 8 and 15 years of age. This • Gymnast wrist is a chronic wrist pain affecting almost 80% of pediatric
prevalence is changing due to the increasing obesity rate. gymnasts at some point. Compressive loading and shearing forces cause
• Lyme arthritis typically presents as mono-arthritis in two-thirds of cases, physeal microfractures at the hypertrophic zone, causing physeal widening
involving the knees 90% of the time. The knee tends to be swollen with and metaphyseal irregularity in almost three-fourths of patients.
1. An 8-year-old boy presents with left wrist pain after fall off a tram- 3. Which of the following statements regarding Lyme arthritis is true?
poline. Wrist radiograph shows a fracture at the distal left radius a. Lyme arthritis is an early localized sign of Lyme disease.
extending through the epiphysis, physis, and metaphysis. Accord- b. Synovial fluid WBC count ranges between 50,000 and 100,000
ing to the Salter-Harris classification, what fracture pattern is cells per microliter.
described? c. Azithromycin is the treatment of choice.
a. Salter-Harris type I d. Monoarthritis is noted in two-thirds of cases, involving the knee
b. Salter-Harris type II 90% of time.
c. Salter-Harris type III Answer: D. Lyme disease is caused by the spirochete Borrelia
d. Salter-Harris type IV burgdorferi and is now considered the most common vector-borne
e. Salter-Harris type V illness in North America and Europe. The primary vector is the Ixo-
Answer: D. Salter-Harris type IV fractures involve the growth plate des tick. Lyme arthritis is a finding in late Lyme disease. Synovial fluid
extending from the physis into both the epiphysis and metaphysis. The WBC count ranges are typically 10,000 to 50,000. Treatment recom-
risk of growth plate damage is higher in type IV and requires orthope- mendations depend on prior treatment failures and additional Lyme
dic consult and commonly open reduction to stabilize. manifestations.
2. A 13-year-old overweight male presents with a complaint of insid- 4. Which of the following statements regarding gymnast wrist are
ious left hip pain that now radiates down to his knee. No trauma is true?
noted on history and the patient is not athletic. On examination the a. Initial radiographs of the wrist demonstrate a distal radius
patient walks with a limp and is unable to fully range his left hip due Salter-Harris II fracture.
to pain but has full range of motion of the knee. Radiograph of the b. Gymnast wrist only affects about 10% of gymnasts.
left hip is likely to show what finding? c. Radiographic evidence is seen in almost three-fourths of
a. Inferiorly and posteriorly displaced femoral epiphysis relative to patient’s with the clinical diagnosis.
the metaphysis on a cross-table lateral view of the hip d. The weight load to the athlete’s wrist is generally greater than 30
b. Femoral neck fracture times their body weight.
c. Avulsion fracture at the ischial tuberosity Answer: C. Distal radial epiphysitis, also known as “gymnast wrist,”
d. Femoral head fragmentation is a chronic wrist pain affecting almost 80% of pediatric gymnasts at
Answer: A. The patient’s history and examination are consistent some point. Radiographs demonstrate widening of the distal radial
with a slipped capital femoral epiphysis (SCFE). The diagnosis of SCFE physis. This condition is commonly seen in gymnasts who bear weight
is made with AP and lateral radiographs of both hips as it can be pres- through their upper extremities, increasing the weight load to their
ent in 18% to 50% of patients. Femoral head fragmentation is consis- wrist from 2 to 16 times their body weight. Treatment is generally con-
tent with Legg-Calvé-Perthes. Medial joint space widening can be seen servative with cessation from weightbearing activities.
with a joint effusion in the setting of septic arthritis, transient synovitis,
or Lyme arthritis.
Pediatric Drug Therapy 171
KEY CONCEPTS
• A wareness of differences in pediatric pharmacokinetics and specific drug tox- • P erform a risk assessment (for prescription drug abuse and diversion) prior
icities is of critical significance for the safe and effective use of medications to prescribing opioid analgesics and, when indicated, limit prescribing to the
in children. lowest duration and amount possible.
• Avoid prescription and over-the-counter (OTC) cough and cold medications • A multifaceted approach using clinical support systems and readily available
in children because these agents have limited efficacy data and may cause reference tools is essential for the delivery of optimal emergent pediatric care.
harm.
• Counsel parents about the management of fever and appropriate indications
and proper use of antipyretics.
1. A 12-year-old girl is being discharged from the emergency depart- Answer: B. Young children can have significant differences in the
ment (ED) after having her displaced forearm fracture reduced. pharmacokinetic properties based on their physiology. Absorption
Which one of the following actions would be considered potentially can be impaired and delayed based on gut pH and motility. Metab-
dangerous? olizing enzymes mature at different rates during the first year of life.
a. Prescribe codeine. Young children have a higher total body water and larger volumes
b. Consult state prescription drug monitoring programs (PDMP) of distribution and extracellular fluid. There are several factors to
and prior medical and pharmacy records. consider when assessing the risk/benefit profile of taking medica-
c. Use a screening tool, such as CRAFFT, to evaluate for high-risk tions while breast-feeding, including limited human data, maternal
behaviors. comorbidities, therapeutic alternatives, properties of the drug, and
d. Prescribe a minimum number of days to decrease risk for misuse the benefits of continued breast-feeding, when possible. LactMed
and diversion. is an excellent resource to determine the safety of the drug during
Answer: A. Codeine is no longer recommended for pain in pediat- breast-feeding.
ric patients. This is due to the variation in pharmacogenomics (CYP 3. A healthy, fully vaccinated 11-month-old male infant presents to
2D6) and metabolism to morphine, which can lead to either poor your department for evaluation of a fever of 38.4°C (101.1°F) this
pain control or unintentional opioid toxicity. In adolescent patients, morning that responded to a dose of acetaminophen. The physi-
it is recommended that the patient be evaluated for high-risk factors cal examination reveals a well-appearing, afebrile infant with clear
that may lead to opioid use, misuse or diversion. This includes check- lungs. The patient has an unremarkable evaluation and, during the
ing the state’s PDMP and screening to evaluate the risk for drug abuse discharge process, his parents ask for advice regarding fever man-
and addiction. There are currently no pediatric-specific guidelines for agement. Which of the following statements is correct?
prescribing opioids, but it is generally recommended not to prescribe a. Aspirin should not be used in children younger than 15 years
opioids for acute pain more than 3 to 7 days in duration. Providers because of the risk of Reye syndrome.
should also emphasize a nonopioid, multimodal approach to pain con- b. Cool water baths and creams should be used to supplement anti-
trol, including nonpharmacologic methods and adjunct medications pyretics, even if they cause some discomfort to the patient.
such as nonsteroidal antiinflammatory drugs, acetaminophen, or neu- c. Ibuprofen cannot be used in this age group because of ongoing
roactive agents. renal development.
2. What is a unique pharmacokinetic property observed in the pediat- d. Over-the-counter (OTC) antipyretics are standardized, contain
ric population? similar products and formulations, and are thus interchangeable.
a. Infants have decreased absorption of topically applied medica- Answer: A. Counsel parents and caregivers about the management of
tions. fever and appropriate indications for and proper use of antipyretics.
b. Neonates and infants have higher total body water and larger There is no need to cause discomfort with external cooling methods for
volumes of distribution and extracellular fluid. fever control. Ibuprofen should not be used in children younger than 6
c. All drug metabolizing enzymes are at full capacity at birth. months because of ongoing renal development. The formulations and
d. All maternal drugs should be considered not safe when breast dosing of OTC antipyretics are varied and cannot be used interchange-
feeding. ably. The correct answer is A because of the risk of Reye syndrome with
aspirin administration during a viral illness in children younger than barrier can result in kernicterus from bilirubin displacement by cef-
15 years. triaxone. The correct answer is that a thinner stratum corneum and
4. Which of the following statements regarding pediatric pharmacoki- increased body surface area contribute to a greater risk for systemic
netics is correct? toxicity from dermally administered drugs.
a. A thinner stratum corneum and increased body surface area 5. Which of the following steps can be taken to reduce pediatric dos-
contribute to a greater risk for systemic toxicity from dermally ing errors?
administered drugs. A. Adoption of electronic health records with clinical support tools
b. Ceftriaxone administration in neonates results in increased bili- to decrease weight-based dosing errors
rubin production. B. Calculation of weight-based dosing for all emergent medications
c. Because of minimal differences in the volume of distribution and administered in code situations as opposed to using a validated
renal development, weight-based dosing of gentamicin without quick reference guide
attention to age is sufficient. C. Limiting hospital pharmacist presence in the emergency depart-
d. Quicker gastric emptying and decreased gastric pH in neonates ment to avoid delays in bedside care
increase systemic absorption of enterally administered medica- D. Medication reconciliation should occur with just the patient
tions. present to limit primary caregiver influence
Answer: A. Awareness of differences in pediatric pharmacokinetics Answer: A. A multifaceted approach using clinical support systems and
and specific drug toxicities is of critical significance for the safe and readily available reference tools are essential for the delivery of optimal
effective use of medications in children. The dosing of gentamicin emergent pediatric care. Caregivers are an integral part of medication
needs to account for age-based differences in renal development in reconciliation. ED pharmacists have been proven to increase depart-
addition to weight-based differences in distribution volume. Decreased mental accuracy in pediatric medication management. Validated quick
gastric emptying times and an increased pH can prolong exposure to reference guides for code drug administration have been shown to
medications before they pass the pylorus. An immature blood-brain decrease errors and improve efficiency.
172 Child Abuse
KEY CONCEPTS
• T he ultimate determination of whether abuse has occurred can take days
or weeks. Emergency clinicians should focus on recognizing possible abuse,
treating medical injuries, and establishing a safe disposition for the child.
• Completely undress infants and preverbal children for the physical exam-
ination; pay particular attention to the skin, ears, mouth and oral cavity,
scalp, fontanel, and genitalia.
• Consider abuse routinely for sentinel injuries in young children without an
independently witnessed traumatic mechanism. Sentinel injuries include
the following: bruising in children younger than 6 months old; bruising on
the torso, ears, neck, jaw, cheek, or eyelid; oral injuries; patterned cuta-
neous injuries; subdural hematoma; long-bone fractures in infants; intra-
abdominal injuries; and rib fractures.
• Consider abuse for children when family violence (child abuse, intimate
partner violence, elder abuse, or animal abuse) is recognized in the child’s
home.
• The goal of diagnostic testing is to identify additional clinically or forensi-
cally significant injuries or medical entities that may present similarly to
abuse.
• Use objective, nonaccusatory, matter-of-fact statements to communicate
concern for abuse.
• Emergency clinicians in the United States, Canada, and many other coun-
tries are legally mandated to report reasonable concerns for abuse to public
child protective services (CPS) agencies.
1. A 12-month-old girl is referred to the emergency department (ED) Answer: C. Periosteal reaction and other signs of fracture healing are
because her daycare noticed that she cries whenever she is picked not visualized until approximately 1 week after injury, and persist for
up and there was crepitus in the left side of the chest. They deny any weeks or months, whereas sudden coma implies an acute, traumatic
witnessed trauma in the daycare facility. The mother arrives and event. Mixed-density subdural hematomas can be observed in the
appears concerned. On physical examination, you find some cir- acute, subacute, and chronic phases. The color of bruises is affected by
cular contusions on the inner aspects of both her upper arms. The several factors in addition to the age of a bruise, including the child’s
mother states this occurred from a fall a week ago. Which of the complexion, depth of the bruise, natural variation, and even the per-
following should be the next step(s) in the patient’s management? ceptions of the observer. Too numerous to count intraretinal hemor-
a. Conduct a skeletal survey, and report to child protective services rhages do not persist beyond a few days, but this could be consistent
(CPS). with a hepatic laceration.
b. Determine whether the arm contusions are of the same age. 4. Which of the following is very concerning for sexual abuse in a pre-
c. Perform a dedicated retinal examination to determine if the pubertal child?
child has retinal hemorrhages and, if she does, perform a head a. A new history of constipation and dilated anal sphincter (>1 cm)
computed tomography (CT) scan. b. An 8-year-old girl whose anogenital exam is completely normal
d. Perform a humerus x-ray to determine if there is a humerus reports that her camp counselor “put his wee-wee in my butt.”
fracture. c. Erythema of the hymen and posterior fourchette.
Answer: A. Contusions on the inner aspect of the upper arm are atypical d. Thin, fragile epithelium surrounding the labia with petechiae
injuries in light of the mother’s claim of a fall for the child. This pattern and subcutaneous bleeding.
of injury is often related to being held tightly around the arm. Chest wall Answer: B. A child’s report of sexual abuse is highly concerning and is
pain in a child could indicate a rib fracture. Chest x-ray is insensitive often the strongest evidence of abuse. A normal exam neither confirms
for rib fractures, which are very uncommon at this age unless caused nor excludes the possibility of sexual abuse, even with penetration.
by inflicted trauma. This should raise your suspicion of child physical Dilation of the anal sphincter is nonspecific, especially in the setting of
abuse. In general, children younger than 2 or 3 years old with suspected constipation. Like many mucosal tissues, erythema can occur for many
inflicted injuries should be evaluated with a skeletal survey. reasons, including trauma, foreign bodies, lack of hygiene or even nor-
2. A 6-year-old, pre-pubertal girl presents to the emergency depart- mal variation. “Cigarette-paper” skin with petechiae or hemorrhage
ment (ED) at midnight after her parents noticed malodorous vag- suggests the presence of lichen sclerosis et atrophicus.
inal discharge over the last 3 days after visiting her grandfather 2 5. In a 2-week-old infant that is not yet crawling, which of the follow-
weeks ago. The patient is tired and irritable at the time of presenta- ing is most highly concerning for abuse?
tion. Which of the following are indicated in the ED? a. Clavicle fracture with callous
a. Empiric treatment for gonorrhea and chlamydia b. Lateral rib fracture with callous
b. Obtain a forensic interview. c. Parietal skull fracture without callous
c. Post-exposure prophylaxis for HIV. d. Torn lingual frenulum with scant bleeding
d. Report to child protective services (CPS) Answer: D. Lingual frenulum tears are very concerning for abuse in
Answer: D. Genital discharge can be caused by both sexually transmit- pre-mobile infants. These injuries heal quickly, and the presence of
ted and non-sexually transmitted infections, as well as other inflamma- bleeding excludes the possibility of birth injury. Several self-limited
tory processes. Several days of discharge implies a subacute timeframe injuries can result from the birth process, even in deliveries that are not
for a potential abusive exposure. For subacute sexual abuse, ED man- recognized as particularly traumatic. These injuries can be clinically
agement should include fulfilling the mandate to report to CPS and subtle and are not-uncommonly missed in the newborn nursery. Rib
ensuring that the child has a safe disposition. A genital examination fractures are uncommonly seen from birth and are more common in
may be attempted in a cooperative patient and can assist in assuring the larger infants or difficult deliveries. The presence of obvious callous on
child that they are normal and healthy, but the examination should not the rib fractures is consistent with a fracture that is at least 7 to 14 days
be forced on the child if they are reluctant; it is unlikely to identify tran- old. Clavicle and skull fractures are not uncommon parturitional inju-
sient evidence of sexual contact. The detailed forensic interview should ries. Unlike other fractures, skull fractures do not develop callous and
be conducted by a trained interviewer and may be deferred until the their age cannot be estimated.
patient is rested. HIV PEP is unlikely to be effective 2 weeks after the
assault. Empiric treatment for gonorrhea or chlamydia should not be
given to pre-pubertal children until a sexually transmitted illness is
definitively confirmed.
3. Which of the following implies multiple episodes of trauma at dif-
ferent times?
a. Bruises that are different colors.
b. Mixed density subdural hematoma on head CT.
c. Rib fractures with robust periosteal reaction are identified the
day after a child presents with sudden coma and subdural hema-
toma.
d. Too numerous to count intraretinal hemorrhages and liver lacer-
ation with elevated hepatic transaminases
SECTION TWO The Pregnant Patient
KEY CONCEPTS
Asthma • H ypoglycemia is most common in the first trimester, and up to 40% of preg-
• The treatment goal for a pregnant woman with an acute asthma exacerbation nancies are marked with at least 1 episode of severe hypoglycemia.
is to prevent fetal hypoxia by keeping maternal oxygen saturation above 95%. • Comorbid obesity increases the risk of cesarean section and venous thrombo-
Inhaled beta-agonists and corticosteroids are first-line emergency depart- embolism.
ment treatment and are considered safe for use in pregnancy. • Graves disease commonly rebounds in the immediate postpartum period with
thyrotoxicosis.
Cardiac Disease • Radioiodine is strongly contraindicated for the treatment of hyperthyroidism
• Hypertensive emergency in pregnancy is defined as acute-onset persistent in pregnancy.
hypertension with systolic blood pressure greater than 160 mm Hg or diastolic • All gravid patients with new-onset nephrolithiasis should be screened for
blood pressure greater than 110 mm Hg that is persistent for greater than 15 hypercalcemia. Treatment of hypercalcemia with bisphosphonates is contra-
minutes. In these cases, antihypertensive therapy should be administered as indicated in pregnancy.
soon as reasonably possible, and no later than 30 to 60 minutes after diagno-
sis with a target blood pressure of 140 to 150 mm Hg systolic and 90 to 100 Psychiatric Disorders
mm Hg diastolic. The drugs of choice are oral nifedipine, IV hydralazine, and • Prenatal discontinuation of methamphetamines and other stimulants,
IV labetalol. although desirable, can cause depression and psychosis.
• The risk for acute coronary syndrome and acute myocardial infarction is • Maternal and neonatal outcomes in women on opioid agnostic therapy show
increased in pregnant women compared to age-matched controls. The most decreased rates of neonatal abstinence syndrome (neonatal opioid with-
common cause of AMI in the pregnant women is spontaneous coronary artery drawal syndrome).
dissection. Treatment is similar to the nonpregnant agent, although P2Y12
receptor inhibitors should be avoided and fibrinolytic agent use should be Inflammatory Disorders
carefully considered in women close to term. • Antiphospholipid syndrome (APS) in lupus patients is characterized by deep
vessel clotting, pregnancy-related morbidity, and positive anticoagulant
Anemia serum markers. Catastrophic APS has rapid-onset small vessel thrombosis,
• Anemia in pregnancy is defined as a hemoglobin less than 11. Serum ferritin multiorgan dysfunction, and a high maternal mortality rate.
is the most accurate lab value for diagnosing iron deficiency anemia in preg-
nancy. Women with mild iron deficiency anemia can be started on daily iron Renal Disease
supplementation while those with severe iron deficiency in the second and • Management of chronic kidney disease (CKD) in pregnancy can be treated
third trimester can be referred for IV iron infusion. with intensified hemodialysis (increased length of treatment time or increased
• Sickle cell disease causes maternal complications including more frequent frequency) to improve fetal outcome.
pain crises, and increased risk of venous thromboembolism and preeclampsia. • Patients post renal transplantation have fertility rates that return to normal
Treatment of pain crises is the same as in nonpregnant patients with the within 6 months.
exception that hydroxyurea is contraindicated due to known teratogenicity.
Infectious Disorders
Epilepsy • Moderate to severe anemia in pregnancy in an HIV-infected mother should
• Gravid patients with epilepsy have a tenfold risk of death compared to preg- prompt a workup for tuberculosis.
nant women without epilepsy. Many antiepileptic drugs (AEDs) have known • During pregnancy, penicillin is the only known effective treatment for congen-
teratogenicity and levetiracetam and lamotrigine are considered the safest ital syphilis, and pregnant patients with penicillin allergy should be desensi-
agents during pregnancy. Treatment of status epilepticus is with benzodiaze- tized and treated with penicillin.
pines followed by a phenytoin as first-line AED and levetiracetam as second- • Lamivudine given in late pregnancy to women with high viral loads of HBV
line AED. DNA reduces viral transmission when given in conjunction with HBV vaccine and
immune globulin.
Endocrine • Obstetric hemorrhagic complications, including DIC and shock and subse-
• Pregnant patients with type 1 diabetes mellitus (T1DM) are recommended to quent need for transfusion, are more common with HCV infection.
transition to insulin during gestation to achieve HgbA1C values <6%.
1. A 28-year-old G2P1 woman at 26 weeks of gestation presents with a normal vital signs and has normal fetal heart tones. Her labs show
recurrent asthma flare. Vital signs are temperature 36°C, heart rate a WBC of 10.1, Hgb 6.2, Hct 18, and platelets of 60,000. You should
110 beats/min, blood pressure 120/60 mm Hg, respiratory rate 28 consider what other comorbid condition?
breaths per minute and O2 saturation 96%. She has diffuse expi- a. Antiphospholipid syndrome
ratory wheezes. Arterial blood gas reveals Po2 90 mm Hg, Pco2 b. Hepatitis B infection
40 mm Hg, and pH 7.34. Which of the following statements best c. Thyrotoxicosis
describes the issues in management with this patient? d. Tuberculosis
a. Corticosteroids are contraindicated. Answer: D. The diagnosis of new infection or activation of latent tuber-
b. Most patients with asthma improve during pregnancy. culosis in pregnancy can be challenging, given that common symp-
c. She has a metabolic acidosis. toms can be mimicked in normal pregnancy including tachypnea and
d. Treatment and discharge are likely. fatigue. Moderate to severe anemia in an HIV-infected mother should
e. Inhaled beta-agonists are first-line therapy. prompt a workup for TB.
Answer: E. Beta-agonists followed by corticosteroids are the mainstay 4. A 24-year-old G1P0 woman at 8 weeks’ gestation presents with anx-
of asthma therapy during pregnancy. During pregnancy, one third of iety, sweating, and palpitations. She has a history of Graves disease.
asthmatics worsen, one third improve, and one third stay the same. She is tachycardic and hypertensive. Her TSH is <0.02 and her free
Blood gas interpretation must take into account the “normal” alke- T4 level is 9.2 ng/dl (normal 0.8 to 2.8 ng/dL). Which of the follow-
mia of pregnancy with a Pco2 of 30 to 32 mm Hg and a compensatory ing treatments is most appropriate?
HCO3 level of 18 to 20 mEq/L. This patient has a relative “hypoventila- a. Alpha-adrenergic blockade
tion” and may indeed need admission for close observation. b. Methimazole
2. A pregnant woman in the second trimester presents to the ED with c. Propylthiouracil
a complaint of a 5/10 headache and has a blood pressure of 165/100. d. Radioiodine
She reports regular prenatal follow-up. She reports she has a history Answer: C. Radioiodine is strongly contraindicated in pregnancy
of mild hypertension dating to before her pregnancy and her blood because it will destroy the fetal thyroid gland. PTU is the treatment
pressures have been in the 150/90 range during this pregnancy of choice in the first trimester because of the risk of adverse congen-
and that she is not on any anti-hypertensive medications. A repeat ital drug effects of methimazole during organogenesis. Beta blockade
blood pressure after 15 minutes is 165/100. Which of the following remains the treatment of choice for abnormal vital signs, not alpha-
best describes the issues in management of this patient? adrenergic blockade.
a. The patient should have been placed on an anti-hypertensive 5. A 20-year-old G2P1 woman at 20 weeks’ gestation with a known
medication by her OBGYN for her blood pressures of 150/90. history of SLE presents with acute onset of R hemiplegia and dys-
b. A full workup should be completed before treating the patient’s arthria. One hour post onset of symptoms, her head CT is normal.
hypertension. Her symptoms are most likely caused by which of the following?
c. An IV must be established quickly in order to administer IV a. Catastrophic antiphospholipid syndrome (CAPS)
anti-hypertensives. b. Cerebritis
d. Oral nifedipine can be used to lower the patient’s blood pressure. c. Eclampsia
e. ACE inhibitors can be used to lower the patient’s blood pressure. d. Stroke
Answer: D. ACOG states that mild chronic hypertension in pregnancy Answer: D. The most common arterial presentation of neurologic
defined as less than a systolic blood pressure of 160 or a diastolic blood symptoms in lupus patients is stroke; positive aPL results are found
pressure of 110 does not need to be treated. Hypertensive emergency in up to 20% of ischemic stroke patients younger than 50 years of age.
in pregnancy is defined as acute-onset persistent hypertension with CAPS is characterized by rapid-onset, small vessel thrombosis, multi-
systolic blood pressure greater than 160 mm Hg or diastolic blood organ dysfunction or failure (renal, hepatic), and a high maternal mor-
pressure greater than 110 mm Hg that is persistent for greater than 15 tality rate, but not as a large vessel occlusion. Patients with lupus are not
minutes. In these cases, antihypertensive therapy should be adminis- at increased risk of cerebritis during pregnancy, and eclampsia does not
tered as soon as reasonably possible, and no later than 30 to 60 minutes present as a focal hemiplegic deficit.
after diagnosis with a goal blood pressure of 140 to 150 mm Hg sys- 6. A 41-year-old G4P3 woman with known hypertension presents at
tolic and 90 to 100 mm Hg diastolic. Oral nifedipine, IV hydralazine, 36 weeks with chest pain and the ECG that follows. Which of the
and IV labetalol are the drugs of choice in this situation, and IV access conditions listed below is the most likely diagnosis?
attempts should not delay treatment as oral nifedipine can be given. a. Hypertensive emergency
ACE inhibitors are contraindicated in pregnancy. b. Normal physiologic ECG changes of pregnancy
3. An HIV-positive woman G3P2 at 15 weeks gestation presents with c. Spontaneous coronary artery dissection
increasing fatigue. She is mildly tachycardia but otherwise with d. Stanford type B dissection
English: 12 lead ECG showing
inferior and right ventricular
infarct
Date 19 May 2014, 06:20:17
Source Own work
Author James Heilman, MD
Answer: C. ECG changes in normal pregnancy include T wave flatten- 8. A patient with poorly controlled epilepsy is at highest risk of break-
ing, T wave inversion (mainly in lead III), and nonspecific ST changes through seizures during which time period of pregnancy?
during pregnancy, as well as ST depression during labor induction for a. 1 to 2 weeks postpartum
cesarean section. This ECG demonstrates an inferior MI. Although b. During delivery
aortic dissection is more common than ACS in pregnant patients, a c. Post 32 weeks
type B dissection would not involve the coronary outflow tract. The d. She is not at increased risk of seizing
most common cause of ACS in pregnancy is spontaneous coronary Answer: B. Delivery and the first 24 hours postpartum are the most
artery dissection. likely times for a seizure to occur, with a nine fold greater incidence of
7. Perinatal maternal complications in women with sickle cell disease seizure than during pregnancy in general.
include which of the following? 9. Which of the following autoimmune disorders most consistently
a. Deep vein thrombosis worsens during pregnancy?
b. Maternal infections a. Addison disease
c. Pain crises b. Graves disease
d. Preeclampsia c. Multiple sclerosis
e. All of the above. d. Rheumatoid arthritis
Answer: E. Maternal complications are common in patients with SCD; Answer: A. Pregnancy has an immunosuppressive effect. Because of
these include preterm labor, premature rupture of membranes, mater- this, pregnancy can have a variable effect on autoimmune disorders.
nal infections, more frequent pain crises, thrombosis, preeclampsia, Rheumatoid arthritis consistently improves, Graves disease has a tran-
and increased need for cesarean delivery.38,39 Due to these compli- sient worsening in the first trimester but improves later in pregnancy.
cations, pregnant women with SCD have a sixfold increased risk of Patients with MS see a decrease in the frequency and severity of exacer-
maternal death compared with controls. bations. Addison disease is exacerbated, especially in the third trimes-
ter, because of the biggest need for cortisol replacement.
175 Drug Therapy in Pregnancy
KEY CONCEPTS
• C hemically induced birth defects are responsible for approximately 1% to 3% • In general, the health of the fetus is directly related to the health of the
of anomalous births. mother, and drugs should be given when the maternal benefits outweigh the
• Gestational age is crucial in determination of the impact of any given expo- risks to the fetus.
sure, especially during organogenesis (days 21–56 of fetal life), when major • Certain medications should be avoided during pregnancy because they are
body organs are formed. known teratogens or cause potential toxic effects in the newborn; these include
• Human data on teratogenicity and fetal toxicity of medications are often lim- anticonvulsants, warfarin derivatives, NSAIDs, sulfonamides, fluoroquinolones,
ited, and causal associations are difficult to determine, especially with newer and ACE inhibitors. If there are no alternatives to these agents, it is recom-
medications. mended to use the lowest dose for the shortest duration possible.
1. Which of the following statements is true regarding the teratogenic Answer: E Several antiepileptic drugs (AEDs) are known teratogens,
risk of medications administered during pregnancy? including valproic acid, carbamazepine, ethosuximide, phenobarbital,
a. A drug should be given when maternal benefits outweigh the phenytoin, and topiramate. Valproic acid is associated with the most
risks to the fetus. frequent serious adverse effects on the pregnancy and fetus compared
b. The FDA assigns one of five letters (A, B, C, D, and X) to classify with other AEDs. Newer-generation AEDs, such as lamotrigine and
teratogenic risk. levetiracetam, have not been associated with increased risk of congenital
c. The fetus is at greatest risk of anatomic defects during the third malformations compared to controls but the data are limited. Maternal
trimester. mortality is higher in women with a seizure disorder and control of sei-
d. Most drugs do not enter the breast milk due to protein binding. zures is felt to be important. Use of monotherapy in the lowest effective
e. Most drugs have epidemiologic studies confirming the risks in anticonvulsant dose is optimal. The other drugs are safe in pregnancy.
pregnancy. 3. A 28-year-old woman who is 7 months pregnant was well until she
Answer: A. In general, the health of the fetus is directly correlated experienced sudden onset of shortness of breath and left-sided sharp
with the health of the mother and necessary medications should not chest pain one hour prior to arrival. The pain is worse on inspiration
be withheld for fear of harming the fetus. The FDA no longer uses let- and is not related to exertion, although her dyspnea becomes worse
ter categories to describe teratogenic risk, which was a flawed system. with any activity. She is alert and appears in mild distress. Her pulse
The Pregnancy and Lactation Labeling Rule (PLLR), effective in 2015, is 112 beats/min, RR is 24 breaths/min, and BP is 110/68 mm Hg.
changed the content and format for prescription drug labeling to bet- Her temperature is 100.8°F (32°C), and O2 saturation is 92%. Lung
ter summarize the risks of drugs in pregnancy, lactation, and repro- examination reveals diminished breath sounds on the left base with
duction. The fetus is at greatest risk during organogenesis in the first scattered wheezes. Cardiac examination reveals tachycardia. Chest
trimester and functional development of the CNS in the 10th to 17th radiograph reveals elevated right hemidiaphragm but no infiltrates.
weeks of pregnancy. Unfortunately, current studies are insufficient to ECG reveals generalized T wave inversions. Which of the following
determine reliable and accurate risks in the use of medications in preg- treatments is most appropriate?
nancy. Most drugs passively diffuse into the breast milk. a. Administer a low-molecular-weight heparin subcutaneously
2. Which of the following drugs is a known teratogen? and start oral warfarin.
a. Albuterol b. Administer intravenous ceftriaxone and azithromycin.
b. Hydroxycobalamin c. Administer nebulized bronchodilators and start prednisone.
c. Penicillin VK d. Administer oxygen and start low-molecular weight heparin
d. Pyridoxine e. Place a central line, and start early goal-directed therapy for sep-
e. Valproic acid sis.
Answer: D On the basis of the information provided, the patient most for the outpatient management of this patient once her symptoms
likely has a pulmonary embolus. The patient is hypoxic and requires are controlled and she is tolerating fluids well?
oxygen administration and anticoagulation. This may be accom- a. Doxylamine 10 mg and pyridoxine 10 mg once per day
plished with heparin, low-molecular-weight heparin (LMWH), or b. Haloperidol 2 mg twice a day
warfarin. Warfarin, a vitamin K antagonist, is teratogenic and is con- c. Omeprazole 20 mg every morning
traindicated in pregnancy. The treatment of pulmonary embolism is d. Ondansetron orally disintegrating tablets 4 mg every 8 hours
LMWH preferentially or heparin. Heparin is preferred if the patient e. Prochlorperazine 10 mg three times per day
is hemodynamically unstable due to its rapid onset and short half- Answer: A. This patient’s symptoms likely represent nausea and vom-
life. The patient has no symptoms of an infectious process; choices B iting of pregnancy or possible hyperemesis gravidarum. The preferred
and E are therefore not appropriate. Bronchodilators and steroids are initial home treatment is Diclegis, which is a combination drug of dox-
indicated for asthma but not for thromboembolic disease. Choice C ylamine and pyridoxine (10 mg/10 mg). The non-combination version
is therefore not appropriate. is: half-tab (21.5mg) of OTC Unisom and 10 mg pyridoxine (Vitamin
4. Which of the following drugs may be associated with complications B6). There are some inconsistent studies suggesting a possible link with
in the newborn when used at term? long term use of Zofran (Ondansetron) and cardiac abnormalities and
a. All of these cleft palates. Omeprazole is not first-line treatment for hyperemesis
b. Nitrofurantoin gravidarum, it is for the treatment of GERD. Prochlorperazine is tera-
c. Nonsteroidal antiinflammatory drugs togenic in animals and is not considered first-line treatment. Haloperi-
d. Propranolol dol is an antipsychotic and not indicated in this patient.
e. Sulfonamides 6. In determining a causal link between a specific drug and congenital
Answer: A. All the choices have been associated with complications malformations, which of the following may be viewed as confound-
in the newborn when used at term. Sulfonamides compete with bili- ing factors?
rubin for protein-binding sites, leaving large amounts of free biliru- a. Genetic background of the fetus
bin to diffuse freely into the brain. This results in bilirubin deposition b. Maternal illicit drug use
in the infant’s brain, thus causing kernicterus. Nitrofurantoin at term c. Presence of maternal illness
has been associated with hemolytic disease of the newborn. Nonste- d. Presence of nutritional deficits
roidal antiinflammatory drugs are associated with premature closure e. All of the above.
of the ductus arteriosus. Propranolol at term has been associated Answer: E. The process of establishing teratogenicity of a substance
with neonatal hypoglycemia, respiratory depression, and neonatal is often flawed. Much of our current knowledge on teratogenicity has
jaundice. been derived from case reports, case-controlled studies, and cohort
5. A 7-week pregnant (by LMP) patient arrives in the emergency studies, which are inherently weak in establishing a causal link. These
department. She is complaining of severe nausea and vomiting. reports are often complicated by a number of confounding factors,
She is unable to keep any solids or liquids down. Her heart rate is which make a causal link difficult to establish. The presence of any of
115, BP 110/75, Temp 98.8°F, RR 15, and glucose is 98. The patient the listed choices may confound results. In the presence of maternal
is given IV fluids and antiemetics with improvement of her symp- illness, for example, the outcome of pregnancy may be related to the
toms. Which of the following medications is the most appropriate medical condition and not the medication.
176 Labor and Delivery
KEY CONCEPT
• M ost ED deliveries require only basic equipment to cut and clamp the umbil- • D eliveries complicated by dystocia, malpresentation, or multiple gestations
ical cord and dry and suction the infant. However, the ED should have addi- are life-threatening emergencies. The emergency clinician should develop
tional equipment and trained staff should be available to care for a newborn strategies to treat each of these potential complications of delivery. Please
requiring further resuscitation. see the following link for a video demonstration of maneuvers used to treat
• Women in labor who present to the ED are generally best cared for in the shoulder dystocia: https://www.hopkinsmedicine.org/gynecology_obstet-
obstetric suite. Women with the urge to push or with the head of the infant rics/education/training/shoulder-dystocia.
crowning are at imminent risk of delivery, which should take place in the ED. • When a prolapsed cord occurs with a viable infant, cesarean section is the
• The Braxton Hicks contractions of false labor do not escalate in frequency or delivery method of choice. If surgical delivery is available, maneuvers to pre-
duration like the contractions of true labor. When in doubt, external electrical serve umbilical circulation should be instituted immediately. The mother is
monitoring of uterine activity can rule out true labor. placed in the knee-chest position, with the bed in the Trendelenburg position,
• Preterm labor is defined as uterine contractions with cervical changes before and instructed to refrain from pushing to avoid further compression of the
37 weeks of gestation. Treatment includes tocolytics and fetal maturation cord. The presenting part is then manually elevated off the cord. Elevation is
therapy combined with bed rest and hydration. maintained until the baby can be delivered surgically.
• Premature rupture of membranes (PROM) occurs after 37 weeks’ gestation. • Uterine atony accounts for 75% to 90% of cases of postpartum hemorrhage.
Its management depends on several factors, including gestational age and Administration of a uterotonic, such as oxytocin in conjunction with massage
fetal maturity, presence of active labor, presence of infection or placental usually provide enough stimuli to control bleeding.
abruption, and degree of fetal well-being or distress. • Approximately 10% of postpartum hemorrhage cases are due to retained pla-
• Preterm PROM should be treated with antibiotics to prevent infection (chorio- cental tissue. Treatment requires manual removal of the remnant placental
amnionitis). tissue.
• The first stage of labor averages 8 hours in nulliparous women and 5 hours • Pelvic bleeding postpartum can be difficult to control without hysterectomy.
in multiparous women. Throughout labor, ongoing assessment of fetal well- When available, embolization of bleeding vessels by an interventional radiol-
being is important, and continuous external electrical monitoring helps iden- ogist has reported success rates of 95% to 100%.
tify fetal distress. • Maternal complications of labor and delivery include obstetric trauma, uterine
• Ultrasonography provides crucial information regarding pending delivery, inversion and rupture, amniotic fluid embolism, coagulation disorders, and
including fetal viability, lie, and presentation. infections. Many of these problems can initially be managed in the ED while
• The fourth stage of labor refers to the first hour after delivery of the placenta awaiting obstetric consultation.
and is a critical period during which postpartum hemorrhage is most likely to
occur.
1. After episiotomy, which is the first maneuver that a clinician should 3. Which of the following medications is effective at reducing postpar-
attempt when treating shoulder dystocia? tum uterine bleeding?
a. McRoberts maneuver a. Tranexamic acid
b. Fracture the clavicle b. Terbutaline
c. Deliver the posterior shoulder c. Magnesium sulfate
d. Cesarean section d. Gentamycin
Answer: A. The most important first step is to use McRoberts maneu- Answer: A. Tranexamic acid (1 gr IV) is effective at reducing bleeding.
ver (see Fig. 176.10). Maternal leg flexion to a knee-chest position Oxytocin and fundal massage should also be used.
may disengage the anterior shoulder. This maneuver “walks” the 4. Which antibiotic combination provides the best initial coverage for
pubic symphysis over the anterior shoulder and flattens the sacrum, postpartum endometritis?
helping the fetus pass through the birth canal, one shoulder at a. Cephalexin and vancomycin
a time. b. Ciprofloxacin and metronidazole
2. Vaginal delivery of a frank breech is best accomplished by which of c. Clindamycin and gentamicin
the following actions? d. Amoxicillin and nitrofurantoin
a. Pulling hard on the presenting part Answer: C. Antibiotics should include anaerobic coverage because
b. Pushing the presenting part back through the cervix endometritis is usually caused by genital flora.
c. Supporting the presenting part and giving the cervix time to 5. The first stage of labor ends with which of the following?
dilate. a. Delivery
d. Administering oxytocin b. Ruptured membranes
Answer: C. The head is the widest part of the fetus. Pulling on the c. A fully dilated cervix
presenting part causes the head to become trapped behind an incom- d. Placental delivery
pletely dilated cervix. Answer: C. The first stage of labor is the cervical stage, ending with a
completely dilated, fully effaced cervix.
Trauma in Pregnancy
177
KEY CONCEPTS
• F emale trauma patients of reproductive potential should be screened for
pregnancy and assumed to be pregnant until proven otherwise.
• Management of life-and limb-threatening injury in the mother comes first.
Saving the mother provides the best chance of saving the baby.
• Even in the stable pregnant trauma patient, the fetus is at increased risk of
morbidity and mortality; therefore, the fetus should be monitored by cardio-
tocography continuously for a minimum of 4 hours after any trauma.
• The fetus is considered viable between 22 to 24 weeks’ gestation (>500
grams). This usually corresponds to when the fundus is at or above the umbili-
cus.
• Alterations in anatomy and physiology that occur during pregnancy alter the
pattern of injuries in trauma and the clinical findings related to blood loss,
which may mask injuries, making a systematic approach essential.
• Keeping the mother tilted 15 to 30 degrees to the left, or manually displac-
ing the uterus leftward, in any pregnancy of 20 weeks’ gestation or greater
is recommended to alleviate hypotension due to aortocaval compression in
order to improve maternal and fetal perfusion.
• Resuscitative hysterotomy should be initiated for a potentially viable fetus
(fundus above the umbilicus) within 4 minutes and completed in 5 minutes
after the onset of cardiopulmonary arrest and no return of spontaneous
circulation (ROSC) for optimal maternal and fetal benefits.
• The use of ionizing radiation to the pregnant patient, including CT and plain
radiography, should be minimized, but imaging should not be withheld if
it may provide significant diagnostic information. In certain circumstances
ultrasound, MRI, or a period of observation can preclude the need for ioniz-
ing radiation.
1. A 30-year-old female who is 28 weeks’ gestation presents to the and only an 18% increase in red blood cells, results in hematocrits of
emergency department after being a restrained passenger in a low- 32% to 34% by gestational age of 32 to 34 weeks. Electrocardiographic
speed MVC. Which of the following is the best indicator of fetal changes include a left-axis shift averaging 15 degrees, caused by dia-
outcome in this scenario? phragm elevation. Consequently, flattened T waves or Q waves in leads
a. Abdominal tenderness III and aVF may be seen.
b. Cardiotocographic monitoring for 4 hours 3. A 26-week gravid woman presents to the emergency department
c. Maternal blood count and arterial blood gas results (ED) after a moderate-speed motor vehicle collision (MVC). The
d. Maternal vital signs patient is without complaints, and her vital signs are as follows:
e. Ultrasonography blood pressure, 100/60 mm Hg; heart rate, 100 beats per minute;
Answer: B. For women with mild blunt trauma, fetal outcome is not and respiratory rate, 18 breaths per minute. Ultrasound examina-
predicted by maternal vital signs, abdominal tenderness, blood tests, or tion shows good fetal movement, with a fetal heart rate of 150 beats
ultrasound results. Only cardiotocographic monitoring for a minimum per minute. What is the appropriate disposition for this patient?
of 4 hours is useful to predict fetal outcome. Fetal monitoring can iden- a. Consult obstetrics for a minimum of 4 hours of cardiotoco-
tify fetal distress and the need for further intervention. graphic monitoring.
2. Which of the following factors is most concerning in the presenta- b. Perform a FAST examination and if negative discharge home.
tion of a pregnant trauma patient? c. Consult trauma and obstetrics for admission and serial exam-
a. Diastasis of the symphysis pubis inations.
b. Electrocardiogram (ECG) findings of Q waves in III and aVF d. Consult trauma surgery for exploratory laparotomy.
c. Hematocrit 34% in third trimester e. Discharge the patient with close follow-up with obstetrics.
d. Hypotension in the third trimester Answer: A. Placental abruption results when the inelastic placenta
e. Respiratory alkalosis in the third trimester shears away from the elastic uterus during sudden deformation of the
Answer: D. Blood pressure declines in the first trimester, levels out in uterus. Because deceleration forces can be as damaging to the placenta
the second trimester, and then returns to nonpregnant levels during as direct uterine trauma, abruption can occur with little or no external
the third trimester. In pregnancy, minute ventilation increases, leading sign of injury to the abdominal wall. For the viable fetus (more than 22
to hypocapnia. Therefore, a partial pressure of arterial carbon dioxide to 24 weeks’ gestation), monitoring is the next step.
(Paco2) of 35 to 40 mm Hg may indicate inadequate ventilation and
impending respiratory decompensation. The physiologic anemia of
pregnancy, resulting from a 48% to 58% increase in plasma volume
4. A 26-year-old, 30-week gestation woman presents unresponsive 5. A 28-week pregnant patient comes in after a gunshot wound to the
with cardiopulmonary resuscitation (CPR) in progress after a high- epigastrium. Her blood pressure is 95/30 and her heart rate is 125
speed motor vehicle collision (MVC). The patient lost her vital signs after receiving 1 liter IV fluids by EMS en route. What is the optimal
3 minutes before arrival in the emergency department (ED). What management for this patient?
is the most appropriate next step in the management of this patient? a. Explore the wound locally for entrance and exit wounds.
a. Transfer to the operating room for emergency cesarean section b. Expectant management with serial abdominal examinations.
by obstetrics c. Intraoperative exploratory laparotomy or laparoscopy by a
b. Resuscitative hysterotomy with vertical midline incision trauma surgeon.
c. Thoracotomy with cardiac massage d. Fetal monitoring for a minimum of 4 hours
d. Administer epinephrine and continue ACLS guidelines for car- Answer: C. The gravid uterus affects the injury pattern seen with pen-
diac arrest etrating trauma to the upper abdomen with the probability of harm
e. Defibrillate immediately at 300 J to the bowel, liver, or spleen at almost 100%. The gravid uterus tends
Answer: B. In the event of maternal cardiopulmonary arrest, resus- to protect abdominal organs from trauma but substantially increases
citative hysterotomy is indicated in any pregnancy of greater than the likelihood of bowel injury from penetrating trauma to the upper
20 weeks’ gestation (above the umbilicus) to improve both maternal abdomen. A trauma surgeon/trauma service should be activated or
and fetal survival. It should ideally be performed within 4 minutes consulted. With persistent hypotension after an initial 1-liter IV fluid
of loss of spontaneous circulation and completed by 5 minutes of no bolus, the massive transfusion protocol should be initiated in anticipa-
ROSC. The most experienced physician available should perform the tion of the expected course of action. The patient will need to go to the
procedure. operating room for an exploratory laparotomy.
SECTION THREE The Geriatric Patient
1. Which of the following pharmacokinetic change in older adults is concentrations. Liver blood flow, glomerular filtration rate, total body
associated with increased plasma concentration of drug? water, and absorptive capacity decrease with age. However, gastric pH
a. Decrease in absorptive capacity in an orally administered medi- increases.
cation 2. Which of the following is the most appropriate strategy for analge-
b. Decreased glomerular filtration rate in a renally eliminated med- sic provision in an older patient with renal insufficiency and severe
ication abdominal pain in the emergency department?
c. Increase in gastric pH in a medication that is a weak base a. Hydromorphone 0.5 mg IV, followed by the same dose in 15
d. Increased liver blood flow in a high–extraction ratio medication minutes, if needed
e. Increased total body water in hydrophilic medications b. Ketorolac, 30 mg IV qid
Answer: B. Decrease in glomerular filtration rate is common with age. c. Meperidine, 50 μg IV every 2 hours
This leads to decreased elimination and increased plasma concentration d. Morphine, 4 mg IV every 2 hours
of a drug. All other options listed would lead to decreased plasma drug e. Oxycodone/acetaminophen, 5/235 mg orally every 2–4 hours
Answer: A. One strategy that has been used successfully in older adults c. Insulin NPH for hyperglycemia
is a two-step hydromorphone protocol. Patients are given 0.5 mg IV d. Oxycodone for fracture pain
hydromorphone, which is repeated in 15 minutes if the patient desires e. Promethazine for nausea
another dose when asked, “Do you want more pain medication?” Answer: B. Studies of harmful drug interactions in older adults have
Hydromorphone does not have active metabolites that would accu- shown that the initiation of antibiotics in patients taking warfarin can
mulate with renal insufficiency, so it would be an appropriate option lead to bleeding. Ciprofloxacin can lead to an elevation in the inter-
for severe pain. Ketorolac should be avoided in older adults with renal national normalized ratio (INR) due to disruption of vitamin K syn-
insufficiency because the inhibition of prostaglandins can decrease thesis in patients taking warfarin. Thus, more frequent monitoring is
renal blood flow. Meperidine use is no longer recommended because of required when ciprofloxacin is initiated. The other options listed do
a neurotoxic metabolite and accumulation in renal insufficiency. Mor- not interact with warfarin. However, oxycodone and promethazine
phine has an active metabolite that also accumulates in renal insuffi- can be harmful because of central nervous system effects, potentially
ciency. Oxycodone-acetaminophen given orally is not appropriate for leading to falls.
severe acute pain due to delayed onset of effect. Bioavailability of oral 5. Which of the following statements is true regarding neurologic
medications may further be delayed in older adults. emergencies in older adults?
3. Which of the following agents should be avoided for outpatient use a. Ampicillin is added for the treatment of bacterial meningitis for
in an older patient being discharged from the emergency depart- additional coverage against Enterococcus faecalis.
ment? b. Anticholinergic medication use is not associated with delirium.
a. Diphenhydramine c. Haloperidol, 10 mg IV, is recommended for acute agitation due
b. Loratadine to underlying psychiatric illness.
c. Acetaminophen d. In patients with ischemic stroke who are older than 80 years,
d. Clindamycin alteplase should not be administered if the patient presents 4
e. Metronidazole hours after the onset of symptoms, according to guidelines.
Answer: A. Diphenhydramine and other first-generation antihista- e. Large doses of haloperidol given via the IV route have been asso-
mines should be avoided in older patients given the potential for anti- ciated with QTc interval prolongation
cholinergic toxicity. The use of diphenhydramine in the emergency Answer: E. There is a US Food and Drug Administration (FDA) black
department for severe allergic reactions is appropriate; however, lon- box warning regarding the use of haloperidol and QTc interval pro-
ger term outpatient use is not recommended. This effect is much less longation. This risk is increased with large doses given intravenously.
common with second-generation antihistamines such as loratadine. Polypharmacy in older adults also puts them at risk for drug interac-
Acetaminophen is generally considered safe in older patients. Anti- tions due to concomitant QTc-prolonging agents. Doses of haloperidol
biotics such as clindamycin and metronidazole have the potential to should be less than 5 mg in most older adults. Ampicillin is added for
cause drug-drug interactions with other home medications; however, bacterial meningitis to cover Listeria monocytogenes. Although recent
by themselves they are not typically problematic. evidence has extended the 3-hour time window to 4.5 hours for receiv-
4. Which of the following medications prescribed on discharge is ing alteplase after the onset of stroke in a small subset of patients, those
most likely to interact in an older patient taking warfarin? older than 80 years are not eligible, according to guidelines. Anticho-
a. Albuterol inhaler for asthma linergic medications are known to be associated with delirium and
b. Ciprofloxacin for urinary tract infection sedation.
Geriatric Abuse and Neglect 181
2. A 72-year-old female is brought to the emergency department (ED)
KEY CONCEPTS
by paramedics for “not eating for 5 days,” according to her family.
• E lder mistreatment, which includes physical abuse, sexual abuse, neglect, The paramedics noted that she was covered in urine and feces and
emotional/psychological abuse, abandonment, financial/material exploita- that the house was filthy, and she was found lying on the floor. On
tion, and self-neglect, is common and may have serious medical and social examination, she is noted to have a stage 4 pressure ulcer on her
consequences. sacrum, which appears infected. Which of the following are con-
• Elder mistreatment is under-recognized by emergency clinicians and under- cerning for neglect?
reported to the authorities. a. Delay in seeking care
• Signs suggestive of potential elder abuse and neglect that should be rec- b. Evidence of dehydration
ognized by emergency clinicians may exist in the medical history, physical c. Poor hygiene
examination, and medical/laboratory markers. d. Pressure ulcer
• Emergency clinicians should be vigilant in assessing for the possibility of e. All of the above
elder abuse or neglect and routinely ask elderly patients about mistreat- Answer: E. All of the above signs are potential markers for neglect.
ment, even in the absence of signs and symptoms. Screening protocols may 3. A 69-year-old man with dementia presents to the emergency depart-
be helpful. ment (ED) with pneumonia and delirium. He is very confused, and
• Using a team-based approach including social workers and other emer- his cognition is markedly worse than his baseline. You are called to
gency department (ED)- based professionals may improve elder abuse the bedside because his adult daughter is at the bedside asking him to
detection, and Emergency Medical Services can play a critical role. sign a legal form giving her the deed to his house. What are the next
• ED management of elder abuse should include the following: treating steps in advocating for your patient?
acute medical and psychological issues, ensuring patient safety, and a. Perform an assessment of cognition and capacity and document
proper reporting to the authorities. Trauma-informed care should be it on the chart.
provided. b. Request that the hospital lawyer and notary be present.
• Emergency clinicians should hospitalize elderly patients who are in imme- c. Tell her that there must be two witnesses present for the form to
diate danger or implement a care plan that prevents them from having any be legal.
contact with the suspected abuser(s) but must respect the wishes of an d. Tell the daughter that if there are other children, they must be
older adult with decision-making capacity who refuses interventions and present for the form to be valid.
desires to return to an abusive situation. Trauma-informed care should be Answer: A. When an elderly patient has delirium or is confused
provided. because of another medical condition, he or she may lack the capacity
• Emergency clinicians should document completely and accurately the to enter into a legal agreement or sign a legal document. The patient
history and all physical findings in cases of suspected elder abuse or described is clearly too confused to understand all of the ramifications
neglect, as this documentation may be critical to ensure justice for the of signing his or her home over to another person. Confusion resulting
victim. from delirium or dementia is a risk factor for elder financial abuse.
4. An 85-year-old female with dementia, who lives in a skilled nurs-
ing facility, presents with bruises on her bilateral inner thighs. The
1. A 70-year-old man who lives alone but has 12 hours per day of a facility staff states that she sustained these while being lifted from
home attendant is brought to the emergency department (ED) by her bed to a chair. Who should you notify?
his daughter, who found him in his caregiver’s home with severe a. Adult protective services (APS)
bruising. On examination, he had multiple bruises at varying stages b. Law enforcement
of healing on the chest and arms, as well as a possible pattern inju- c. Sexual assault response team
ry—a bruise in the shape of a rectangle—across his left chest. What d. The long-term care ombudsman
should you do? e. All of the above
a. Ask the daughter to make a report to adult protective services Answer: E. The physical findings are concerning for sexual abuse,
(APS) because she is the one who found him. therefore law enforcement and APS should be called, as well as the
b. Contact the long-term care ombudsman. sexual assault response team to collect forensic evidence. Because the
c. Do not make a report because he has a long history of falls. patient lives in a facility, the long-term care ombudsman should also be
d. File a report with APS or law enforcement if your suspicion for called to investigate the possible abuse.
abuse is high.
Answer: D. The injuries noted are of significant concern for elder
abuse. The patient should be asked questions in a nonthreatening
atmosphere, separate from the caregiver. If the emergency clinician
still has high suspicion, a report should be made to APS and/or law
enforcement. The physician has the obligation to make the report. The
long-term care ombudsman should be contacted only if the suspected
abuse occurs in a facility, such as a nursing home.
SECTION FOUR Special Clinical Circumstances
KEY CONCEPTS
• Immunocompromised patients who present with acute infections, especially • W hen pneumonia develops in patients with febrile neutropenia, purulent
those that are neutropenic, may appear deceptively benign initially. Their sputum may be absent, and the initial chest radiograph may not show an
symptoms and signs often mimic noninfectious complications. infiltrate.
• Broad-spectrum antibiotics are indicated after obtaining appropriate cultures • Some low-risk febrile neutropenic patients may not require admission to the
of all potential sites of infection, especially if the patient is neutropenic. hospital. After calculating a Multinational Association for Supportive Care in
• Immunocompromised patients can have serious local or systemic infections Cancer (MASCC) risk index score and consulting with their oncologist, dis-
without fever. charge may be reasonable.
• Symptoms, signs, and findings of infection may include tachypnea, tachy- • Diabetic patients have a high incidence of MRSA infection, osteomyelitis, and
cardia, mental status change, metabolic acidosis, increased volume require- wound infections and are at higher risk of bacteremia. Severe infections may
ments, rapid changes in serum glucose or sodium concentration, or acute be more insidious in presentation.
abdominal pain. • Patients with cell-mediated immune deficiency, including those on high-dose
• The incidence and severity of febrile neutropenia are inversely proportional corticosteroids, may develop life-threatening infections with intracellular
to the absolute neutrophil count and directly proportional to the duration of bacteria (Listeria, Salmonella, tuberculosis), fungi (Cryptococcus, Coccidioi-
neutropenia. des, Histoplasma), herpes simplex virus, and varicella-zoster virus.
• In neutropenic patients, the temperature should be measured orally or tym- • Guidelines no longer support empiric antibiotic treatment of aspiration in
panically, not rectally, due to theoretical risk of bacterial translocation and alcoholic patients.
subsequent bacteremia. • In patients with cirrhosis, empirical treatment of suspected spontaneous bac-
• Febrile neutropenia is more common in hematologic malignancy (compared terial peritonitis (SBP) with antibiotics should be started regardless of ascitic
with solid malignancy) and is most likely to occur 7 to 10 days after chemo- cell count.
therapy. • Patients who require hemodialysis for end-stage renal disease have high
• Gram-positive organisms are responsible for most serious infections in neu- mortality if they develop pneumonia, C. difficile disease, or infections of the
tropenic cancer patients, but infections due to gram-negative organisms are dialysis access site.
more rapidly lethal. • Functional or surgical asplenia predisposes to fulminant infection with pneu-
• Neutropenic cancer patients with chemotherapy-induced oral mucositis can mococci and other encapsulated organisms (H. influenzae, N. meningitidis,
develop rapid onset of fever with shock. Viridans streptococci is a common and Capnocytophaga canimorsus after dog bites) and, when seen early, may
pathogen and requires Vancomycin. be misdiagnosed as a viral illness, gastroenteritis, or food poisoning.
1. What is the most common site of infection in febrile neutropenic Answer: A. This patient is at high risk for overwhelming postsplenec-
patients? tomy sepsis, usually caused by Streptococcus pneumoniae. Persons who
a. Gastrointestinal tract have undergone splenectomy for a hematologic disorder or lymphoma
b. Perineum and anorectal area are at much higher risk for overwhelming postsplenectomy infection
c. Respiratory tract than are those undergoing splenectomy for trauma. The initial prodro-
d. Skin and soft tissue mal symptoms may be misdiagnosed as a viral illness, gastroenteritis,
Answer: C. The respiratory tract is the most common site of infection, or food poisoning before there is abrupt deterioration with develop-
with 25% of infections in the lung and another 25% in the mouth or ment of septic shock with disseminated intravascular coagulation, pur-
pharynx (and an additional 5% in the nose or sinuses). Still, all neu- pura, and multiorgan dysfunction. After blood cultures are obtained,
tropenic patients with fever need to have a thorough physical exam- he should immediately receive antimicrobials active against pneumo-
ination because an undiagnosed infection can cause severe morbidity cocci, meningococci, and Haemophilus influenzae. He can be inves-
or mortality. tigated for other possible etiologies of his symptoms after this initial
2. A 49-year-old man with diabetes who has been receiving chemo- critical action is taken.
therapy for non-Hodgkin lymphoma presents with fever and facial 4. Which answer is correct regarding the diagnosis and management
pain. Six weeks ago, he was treated with broad-spectrum antibi- of patients on long-term high-dose corticosteroid therapy?
otics for fever but had negative cultures of blood and urine. He is a. Acute, short-term administration of high-dose corticosteroids,
taking ciprofloxacin as prophylaxis against infection. Examination but not long-term administration, can make the diagnosis of
is normal except for a purple-black lesion on his hard palate that peritonitis particularly difficult.
looks like dried blood. A complete blood count shows severe neu- b. After laceration repair, sutures should be left in place for 50% to
tropenia. In addition to blood cultures and chest radiography, what 100% longer than usual.
action should be taken immediately? c. Complications of chronic corticosteroid use include pancreati-
a. Administer intravenous clindamycin or ampicillin-sulbactam. tis, pseudotumor cerebri, avascular necrosis of bone, cataracts,
b. Administer intravenous levofloxacin. myopathy, spontaneous vertebral fractures, psychosis, and hypo-
c. Observe in the emergency department (ED) because he may be glycemia.
a candidate for home antibiotic therapy if other laboratory tests d. Long- term corticosteroid use is frequently accompanied by
are normal, if he appears stable after an initial dose of antibiotics, peripheral blood neutrophilia.
and if his oncologist agrees. Answer: B. Corticosteroids interfere with wound healing, so sutures
d. Obtain a computed tomography (CT) scan of the paranasal need to remain in place longer than is usual for the type of laceration.
sinuses and initiation of intravenous antifungal therapy in the Despite the profound defect in CMI that occurs with long-term cor-
hospital. ticosteroid use, infections with organisms associated with defective
Answer: D. This patient has invasive aspergillosis or mucormycosis CMI are unusual. Most serious infections in these patients are caused
until proven otherwise. These fungi invade tissues and produce life- by pyogenic bacteria, such as Staphylococcus aureus, streptococci, and
threatening necrotizing infections in cancer patients with neutrope- gram-negative bacilli. Both short-term and long-term administration
nia, as well as in diabetic patients, especially those who have received of corticosteroids interfere with the diagnosis of peritonitis. These
broad-spectrum antibiotics. CT scan will often show deep extension of patients will have poorly localized abdominal discomfort with minimal
the infection into the sinuses, orbit, or brain. The initial lesion in the findings on examination. All the conditions listed in B are associated
palate or nose is often mistaken for a benign process. The antibiotics with chronic corticosteroid use—except hypoglycemia. Corticosteroid
listed in A and B are incorrect. The most appropriate empirical antibi- use is associated with hyperglycemia, hyperosmolar nonketotic dia-
otic regimen is cefepime or piperacillin-tazobactam, with or without betic coma, and diabetic ketoacidosis.
an aminoglycoside, in addition to antifungal therapy. He is not a can- 5. A 34-year-old woman on chemotherapy for lymphoma presents
didate for home therapy. to the emergency department with severe oral pain and difficulty
3. A 27-year-old man presents to the emergency department (ED) swallowing. She has no fevers, cough, abdominal pain, or dysuria.
with acute onset of fever, chills, headache, myalgias, vomiting, On physical exam, her oral mucosa is erythematous, friable, and
mild abdominal cramping, and diarrhea for 8 hours. A splenec- edematous. Her labs reveal an ANC of 600 cells/μL. What is the
tomy was performed 15 years earlier when he was treated for lym- most important next step?
phoma, which has been in remission since then. He is not taking a. Administer 30 mL/kg of crystalloid for sepsis.
any medications and has been well. Vital signs are as follows: pulse, b. Draw blood cultures and start empirical vancomycin and
125 beats/min; blood pressure, 110/60 mm Hg; respiratory rate, 20 cefepime.
breaths/min; and temperature, 39.5°C. His mental status is normal c. Start IV analgesics to allow oral rehydration.
and he has mild generalized abdominal tenderness. What is the d. Draw blood cultures and start empiric metronidazole and
most appropriate treatment for this patient at this time? cefepime.
a. Blood cultures followed by immediate administration of ceftri- Answer: B. This patient is a neutropenic patient presenting with muco-
axone or cefotaxime, with or without vancomycin sitis. Mucositis is a frequent prelude to viridans streptococcal bacte-
b. Hydration, antipyretic, antiemetic, and observation in the ED remia, which can produce sudden onset of acute respiratory distress
c. Immediate hospital admission with observation and frequent syndrome, a toxic shock–like syndrome, rash, and pneumonia. Impor-
abdominal examinations tantly, mucositis predisposes the patient to gram-positive infections,
d. Lumbar puncture and vancomycin is appropriate in addition to a broad-spectrum gram-
negative agent.
The Solid Organ Transplant Patient 183
KEY CONCEPTS
2. A 57-year-old deceased donor kidney transplant recipient is sent to
• The solid organ transplant recipient’s altered anatomy, denervated the ED from her primary care physician’s office 3 months postop-
allograft, and immunosuppression frequently result in atypical disease pre- eratively because routine blood work revealed a creatinine of 3.45
sentations both related and unrelated to the transplanted organ. (baseline creatinine 1.0). Which of the following medications may
• An understanding of the solid organ transplant recipient’s altered anatomy, be responsible for her acute kidney injury?
including vascular and nonvascular anastomoses, is critical to evaluating a. Albuterol
early post-transplantation complications. b. Tacrolimus
• Rejection can manifest at any point post-transplantation with constitutional c. Pancrelipase
symptoms and signs of allograft insufficiency, requiring prompt recognition d. Prednisone
and augmented immunosuppression to salvage the transplanted organ. Answer: B. Tacrolimus is associated with dose-dependent nephrotox-
• Timing post-transplantation, the net state of immunosuppression, and icity. A serum level of tacrolimus should be obtained in any solid organ
ongoing antimicrobial prophylaxis should be incorporated into the evalua- transplant recipient maintained on this immunosuppressive agent pre-
tion of a solid organ transplant recipient with fever and other concerns for senting with acute kidney injury. It is important to recognize that tac-
infection. rolimus can both cause and be affected by renal insufficiency, so it may
• In addition to affecting a specific arm of the immune system, each antirejec- not have been the initial trigger but may have ultimately worsened the
tion agent is associated with unique toxicities which may be independently renal injury. Alternatively, a new drug (fluconazole, azithromycin, etc.)
responsible for a solid organ transplant recipient’s clinical condition. may have been initiated which resulted in higher serum concentrations
• Cardiac allograft vasculopathy, a form of chronic rejection and similar to cor- of tacrolimus and resultant renal injury.
onary artery disease in presentation, has emerged as a common complication
3. A 35-year-old woman with cystic fibrosis status post bilateral lung
of orthotopic heart transplantation as transplant recipients live longer.
transplantation 2 weeks ago is brought to the ED by her husband.
• Especially in lung transplant recipients with underlying cystic fibrosis or
He reports that she was discharged several days prior to presen-
bronchiectasis, prior imaging and culture data should help guide initial
tation with confusion attributed to intensive care unit delirium.
management of new respiratory symptoms.
Instead, she has developed worsening episodes of alternating agita-
• Recurrent allograft pyelonephritis post-transplantation merits thorough
tion and lethargy. She is afebrile and normotensive on presentation,
evaluation for ureteral stones and strictures, perinephric abscesses, and
with a 2-liter oxygen requirement to maintain saturations above
urinary retention to identify potentially actionable contributors to infection.
90%. No leukocytosis or abnormalities of liver or kidney function
• Hepatic artery thrombosis is an uncommon but devastating early complica-
are noted. A blood gas test does not demonstrate hypercarbia. What
tion of liver transplantation resulting in allograft dysfunction, biliary necro-
is the next best test?
sis, and sepsis if left undetected and untreated.
a. Lumbar puncture
• Allograft rejection, bacterial or viral enteritis, and altered intestinal transit
b. Magnetic resonance imaging
time can all contribute to diarrhea in the intestinal and multivisceral trans-
c. Serum ammonia
plant recipient.
d. Electroencephalography
• As chronic corticosteroids are incorporated into immunosuppression reg-
Answer: C. Hyperammonemia syndrome affects 1% to 4% of lung
imens for the majority of solid organ transplant recipients, adrenal insuf-
transplant recipients, resulting from systemic infection with Myco-
ficiency should be considered in the differential of transplant recipients
plasma hominis or Ureaplasma urealyticum. These urease-splitting
presenting with hypotension and fever.
organisms metabolize urea as an energy source and produce ammonia
• In addition to the organ-specific complications described in the sections
as a by-product, which can accumulate in tissues such as the trachea
above, solid organ transplant recipients experience graft-versus-host dis-
and lung to cause unexplained agitation, lethargy, and new-onset sei-
ease, malignancy, trauma, and psychosocial distress post-transplantation
zures. If ammonia levels are elevated, whole blood or plasma and bron-
requiring heightened awareness and assembly of multidisciplinary teams.
choalveolar samples should be obtained and tested for Mycoplasma
1. A 42-year-old man presents from home with a 3-day history of hominis and Ureaplasma urealyticum polymerase chain reaction (PCR).
fever, cough, and shortness of breath. He underwent orthotopic While awaiting these results, empirical antibiotic therapy against these
heart transplantation 5 years ago, has never experienced rejection, organisms, such as doxycycline or ciprofloxacin, should be initiated.
and is maintained on minimal doses of prednisone and tacroli- Hemodialysis, intravenous sodium benzoate, and sodium phenylace-
mus. He takes trimethoprim-sulfamethoxazole for antimicrobial tate can help to clear ammonia from the blood and tissues but will be
prophylaxis. Chest radiograph reveals a diffuse interstitial pattern. incompletely effective if the underlying infection is not addressed.
What is the most likely cause of his pneumonia? 4. A 30-year-old liver transplant recipient presents to the ED 10 days
a. Community-acquired respiratory virus post-transplantation with fevers, right upper quadrant pain, and
b. Cytomegalovirus jaundice. Laboratory evaluation reveals leukocytosis, transaminase
c. Pneumocystis jirovecii elevations, and hyperbilirubinemia, which have all worsened since
d. Nocardia hospital discharge. Which of the conditions responsible for this pre-
Answer: A. Not all transplant recipients are equivalently immunocom- sentation is most important to diagnose emergently?
promised. Timing post-transplantation, the net state of immunosup- a. Rejection
pression, and antimicrobial prophylaxis should all be considered in b. Cholangitis
developing a differential diagnosis for a transplant recipient with sus- c. Donor-derived hepatitis B infection
pected infection. His 5-year rejection-free post-transplant course and d. Hepatic artery thrombosis
minimal immunosuppression make opportunistic infections much less
likely than community-acquired respiratory viral infections. Given his
ongoing use of trimethoprim-sulfamethoxazole, Pneumocystis jirovecii
pneumonia is unlikely.
Answer: D. The hepatic artery serves as the sole blood supply to the Answer: D. Intestinal transplant recipients may develop diarrhea from
liver allograft and thus hepatic artery thrombosis has devastating con- a wide variety of causes and, in fact, dehydration is the most common
sequences to allograft survival. This complication develops within days reason for recurrent presentations to the ED and readmission in this
to weeks post-transplantation and requires immediate recognition and population. While rejection, bacterial overgrowth, and CMV infection
management in close collaboration with transplant surgeons because are common reasons for diarrhea in the first year after transplant, later
thrombectomy or urgent re-transplantation may be required. Diag- causes for diarrhea are more likely to be community-acquired. This
nosis is with Doppler ultrasound. Consideration should be given to patient’s history of an exposure to a child with a like illness suggests a
assessing the hepatic blood flow for any liver transplant recipient pre- new infection with a typical pathogen. Acute rejection can be precip-
senting to the ER in the first weeks after transplantation. itated by the inability to ingest and absorb immunosuppressive drugs
5. A 53-year-old intestinal transplant recipient with a history of Crohn and could now be a part of this patient’s disease process. Given the use
disease status post multiple bowel resections resulting in short gut of chronic corticosteroids in this population, endogenous steroid pro-
syndrome has been maintained on tacrolimus and prednisone for duction by the adrenal glands is often impaired and seemingly minor
the last 2 years since transplantation without previous significant insults such as viral infections and hypovolemia may lead to signs of
infection complications. He presents to the ED with 5 days of nau- adrenal insufficiency. CMV is unlikely to emerge as a new infection
sea, vomiting, cramping abdominal pain, and diarrhea, and states this late post-transplant.
that his 2-year-old son had been ill with similar symptoms that have
since resolved. The recipient has not been able to consistently take
his medications for the last 3 days. He is febrile and hypotensive
with blood pressures in the 80s/50s. His lab work is significant for
acute kidney injury and hyponatremia. Which of the following is
unlikely to contribute to this patient’s clinical presentation?
a. Community-acquired gastroenteritis
b. Acute rejection
c. Adrenal insufficiency
d. CMV enteritis
184 The Morbidly Obese Patient
KEY CONCEPTS
• O besity has reached epidemic proportions in the United States (US), as well as
much of the world, with nearly 40% of American adults classified as obese.
• Obesity results in several physiologic changes including aberrations to lung
physiology and predisposes patients to obstructive sleep apnea and obesity
hypoventilation syndrome.
• Drug dosages can be challenging in obese patients because some medica-
tions are lipophilic and will need to be dosed based on total body weight
while hydrophilic medications are dosed closer to ideal body weight. Many
medications require specific scalers for appropriate dosing.
• Many drugs lack large-scale trials in the obese population to guide appro-
priate dosing.
• Obese trauma patients suffer greater rates of chest and abdominal injuries
from blunt trauma than healthy-weight patients.
• Bariatric surgery is the only weight loss strategy that consistently results in
long-term significant weight loss.
• Laparoscopic gastric banding, sleeve gastrectomy, and Roux-en-Y gastric
bypass are the most common weight loss surgeries in the United States,
and each presents its own short-term and long-term complications.
• Procedures common in the emergency department (ED), such as lumbar
puncture, venous access, CPR, and intubation, may be more difficult in the
obese patient.
• The increased body mass of the obese patient presents several challenges
to obtaining interpretable radiographic images.
1. Rates of obesity in the United States and around the world have b. Total body weight for the vast majority of drugs
been: c. A complex decision based on the properties of the drug and the
a. Increasing in the US and decreasing around the world. clinical picture of the patient
b. Increasing in the US and staying constant around the world. d. Total body weight if weight is less that 150 kg and ideal body
c. Staying about the same worldwide. weight if above that.
d. Increasing in the US and around the world. Answer: C. Drug dosing is a complex decision based on the properties
Answer: D. Increasing in the United States and around the world. of the drug and the clinical picture of the patient. The clinician faces
2. When lying supine, obese patients tend to: several challenges regarding proper medication dosing in the obese
a. Breathe normally because they have more developed chest mus- patient. The volume of distribution (Vd) of a drug is the principal factor
culature. involved in the loading dose whereas subsequent maintenance dosing
b. Have trouble breathing due to stomach distention. will primarily be governed by total body clearance (Cl).14 The Vd is
c. Have trouble breathing due to changes in lung mechanics. affected by many factors including drug lipophilicity, plasma binding,
d. Have trouble breathing due to chronic back pain. regional blood flow, body composition, molecular size, and degree of
Answer: C. Increased chest wall mass in conjunction with substantial ionization.
abdominal fat mass leads to reduced lung compliance and collapse of 5. Which of the following statements regarding internal hernias fol-
small airways resulting in increased airway resistance. These changes in lowing Roux-en-Y gastric bypass is true?
turn result in a decrease in functional residual capacity (FRC) leading a. They can be intermittent and may require exploratory surgery
to increased atelectasis. for diagnosis.
3. Challenges to obtaining high-quality CT scan images in the obese b. They are easy to diagnose based on history and physical exam-
patient include which of the following? ination.
a. Difficulty lying flat for the scan c. They are almost always visualized on CT studies.
b. Weight limits of the gantry table d. They are an exceedingly rare complication.
c. Girth greater than aperture size Answer: A. They can be intermittent and may require exploratory
d. Claustrophobia during the scan surgery for diagnosis Weight loss will predispose patients to internal
e. Area of interest laying outside the CT field of view (FOV) hernias as potential spaces open in the small bowel mesenteries. They
Answer: A, B, C, and E are all correct. Claustrophobia is often a diffi- can be particularly difficult to diagnose because they can be intermit-
culty during MRI, not CT. tent and CT scans are often falsely negative. Patients may complain
4. Drug dosing of obese patients is based on which of the following of intermittent pain without clear etiology which may localize to the
factors? left flank. Exploratory laparoscopy may be required to confirm the
a. Ideal body weight for the vast majority of drugs diagnosis.
The Combative and Difficult Patient 185
Answer: B. Antipsychotics can be divided into low potency (such as
KEY CONCEPTS
chlorpromazine and thioridazine), midrange potency (such as lox-
• T he emergency department (ED) should develop a written plan of action apine and molindone), and high potency (such as haloperidol and
to deal with violence that integrates the roles and activities of ED staff, droperidol). The incidence of extrapyramidal symptoms is greatest in
hospital administration, security, and local authorities. the high potency group.
• ED staff should be trained to recognize potentially violent individuals and to 185.4. Which of the following disorders is best defined as a pattern
intervene with verbal de-escalation techniques prior to the use of physical of instability of interpersonal relationships, self-image, affect,
or chemical restraint when possible. and marked impulsiveness that begins by early adulthood and
• The emergency clinician should be familiar with the use of physical and presents in a variety of contexts?
chemical restraints as well as the breadth of options for chemical sedation a. Antisocial personality disorder
and circumstances that may guide selection of particular medications. b. Borderline personality disorder
• For the undifferentiated, severely agitated patient requiring rapid tranquil- c. Dependent personality disorder
ization, we recommend a benzodiazepine (such as lorazepam) either alone d. Histrionic personality disorder
or with a first-generation antipsychotic (such as haloperidol). e. Paranoid personality disorder
• The possibility of an organic (medical) cause of aggressive behavior should Answer: B. Borderline personality disorder is a pattern of instability of
be considered in all violent patients, even those with known psychiatric interpersonal relationships, self-image, affect, and marked impulsive-
disease. ness beginning by early adulthood and present in a variety of contexts.
• Difficult patient encounters may result in undesirable implications for both It may be indicated by five or more of the following:
patients and their ED caregivers, including compromised patient care, com- 1. Frantic efforts to avoid real or imagined abandonment
passion fatigue, or professional burnout. 2. A pattern of unstable and intense interpersonal relation-
• Management of the difficult patient can be optimized by understanding the ships characterized by alternating extremes of idealization
multiple issues contributing to the impaired clinician-patient relationship, and devaluation
including factors of the ED setting (such as time constraints and lack of pri- 3. Identity disturbance: Markedly and persistently unstable
vacy), individual clinician influences (such as personal bias or poor commu- self-image or sense of self
nication), and patient contributions to the interaction, including behavioral, 4. Impulsiveness in at least two areas that are potentially self-
social, or substance use issues. damaging (e.g., spending, sex, substance abuse, reckless
• Pejorative stereotypes of difficult patients should be avoided. To aid in clini- driving, or binge eating)
cian strategies for challenging encounters, instead aim to characterize the
5. Recurrent suicidal behavior, gestures, threats, or self-
patient’s primary difficult behaviors.
mutilating behavior
• Understanding one’s own biases and reactions and optimizing communi-
6. Affective instability caused by a marked reactivity of mood
cation are helpful strategies in dealing with a suboptimal clinician-patient
(e.g., intense episodic dysphoria, irritability, or anxiety usu-
relationship.
ally lasting a few hours and rarely more than a few days)
185.1. Which of the following is a positive predictor of violent behav- 7. Chronic feelings of emptiness
ior in patients? 8. Inappropriate, intense anger, or difficulty controlling anger
a. Age (e.g., frequent displays of temper, constant anger, or recur-
b. Ethnicity rent physical fights)
c. Gender 9. Transient, stress-related paranoid ideation or severe disso-
d. Level of education ciative symptoms
e. Marital status 185.5. A patient demonstrates inappropriate sexually seductive behav-
Answer: C. Identification of potentially violent patients is difficult, ior while being interviewed by her physician. This characteris-
with male gender, prior history of violence, and drug or ethanol abuse tic is most consistent with which of the following personality
the only positive predictors. Ethnicity, diagnosis, age, marital status, disorders?
and education are not reliable identifiers. a. Antisocial personality disorder
185.2. Which of the following medications is associated with the b. Borderline personality disorder
highest incidence of sedation, hypotension, and anticholinergic c. Dependent personality disorder
symptoms? d. Histrionic personality disorder
a. Droperidol e. Paranoid personality disorder
b. Haloperidol Answer: D. Histrionic personality disorder is a pattern of excessive
c. Loxapine emotionality and attention seeking beginning by early adulthood as
d. Molindone indicated by five or more of the following:
e. Thioridazine 1. Discomfort in situations in which the center of attention is
Answer: E. Antipsychotics can be divided into low potency (such as someone else
chlorpromazine and thioridazine), midrange potency (such as loxap- 2. Interaction with others often characterized by inappropriate
ine and molindone), and high potency (such as haloperidol and dro- sexually seductive or provocative behavior
peridol). The incidence of sedation, hypotension, and anticholinergic 3. Displays rapidly shifting and shallow expression of emotions
symptoms is highest in the low potency group. 4. Consistently uses physical appearance to draw attention to self
185.3. Which of the following antipsychotic medications is associated 5. Has style of speech that is excessively impressionistic and lack-
with the highest incidence of extrapyramidal symptoms? ing in detail
a. Chlorpromazine 6. Shows self-dramatization, theatricality, and exaggerated expres-
b. Haloperidol sion of emotion
c. Loxapine 7. Is suggestible (i.e., easily influenced by others or by circum-
d. Molindone stances)
e. Thioridazine 8. Considers relationships to be more intimate than they actually are
SECTION FIVE Underserved Populations
1. Which of the following factors has not been identified as a health b. Family member
care disparity for minorities? c. Lip reading
a. High rates of no insurance or underinsurance d. Written communication on notepad
b. Lack of prenatal care Answer: A. Using a certified interpreter for the hard-of-hearing is the
c. Likelihood of living in an area with adequate physician concen- only reliable way of communicating with such patients.
tration 4. A helpful language translator can be essential in communication
d. Lower life expectancy when compared to white populations with a patient who is facing a language barrier. Which of the below
e. Relatively low number of minority providers methods are recommended for using an interpreter during a medi-
Answer: C. Minority patients are likely to live in impoverished areas cal examination?
with limited access to both primary and specialty medical care. As a a. Asking the patient to have friends or family translate discharge
result, they may present to the ED with previously undiagnosed medi- instructions for them
cal problems underlying their symptoms. b. Only obtaining an interpreter if the patient requests one
2. Patients’ cultural perceptions can have a significant impact on how c. Using a family member is appropriate as long as they are older
they perceive their provider and the care they are receiving. Which than 18 years
of the following methods can be used by providers to approach dif- d. Using a trained medical interpreter
ferent cultural perceptions so as to enhance the provider-patient e. Using health care staff members who are fluent in the patient’s
relationship? language
a. Being aware of one’s own cultural values Answer: D. Using a certified interpreter is the only consistently reliable
b. Developing assumptions of cultural perceptions base on patients’ way of communicating with patients with language barriers.
ethnicity or race 5. Which of the following organizations have suggested guidelines to
c. Limiting participation of alternative therapies and folk medicine address cultural competence in educational standards?
in the patient’s care plan a. American Association of Medical Colleges
d. Maintaining the role of the physician as the decision maker, even b. Liaison Committee on Medical Education
when it may conflict with the patient’s autonomy c. Accreditation Council on Graduate Education
Answer: A. Being aware of one’s own preconceived notions and biases d. All of the above
is the first step a health care provider can take on the path toward Answer: D. These are just some of the medical organizations that have
developing cultural sensitivity toward patients from minority groups. developed guidelines to assist medical schools and residency programs
3. Which method of communicating is adequate to utilize with a deaf in teaching physicians-in-training to become culturally competent in
or hard-of-hearing patient? their approach to patients from minority groups.
a. Certified interpreter
Human Trafficking
187
KEY CONCEPTS
• L abor and sex trafficking involve the exploitation of a person for labor or
commercial sex, respectively, affecting up to 25 million persons worldwide.
• Human smuggling, a crime in which a person contracts a smuggler to facil-
itate their illegal entry into a country, can evolve into trafficking during
transit or at the destination and under such circumstances the person is
considered a victim of human trafficking.
• Child victims of sex trafficking are considered victims of child abuse and
neglect under the law and thereby call into relevance state mandated
reporting statutes.
• Populations at greater risk for trafficking include persons with histories of
child abuse, family dysfunction, diverse sexual orientation or gender identity,
intellectual disability, homelessness, financial insecurity, and migration.
• Human trafficking often involves the use of abusive and violent tactics,
including forced substance use and psychological coercion to entrap and
exert control over trafficked persons, with profound implications for survi-
vors’ physical, reproductive, and mental health.
• Trafficked persons seek health care services during their exploitation for a
wide range of health conditions and the emergency department (ED) is the
most common access point for this patient population where indicators of
abuse, control, and the physical and psychosocial red flags of trafficking can
assist in the recognition of trafficked victims.
• Inquiry about forms of interpersonal violence, including trafficking, is fun-
damentally different from screening for medical issues with the goal of
providing a safe environment in which patients feel empowered to share
as much or as little as they choose, and where strengths and resilience are
recognized.
• Trafficked people may return to exploitative situations repeatedly before
exiting permanently; therefore, safety planning is critical for the discharged
trafficked patient.
• Trauma due to trafficking results in neurobiologic changes such that com-
monly occurring smells, sounds, sights, and procedures of the ED environ-
ment may be perceived as threats by those with trauma histories.
• The six principles of trauma-informed care include physical and psycholog-
ical safety; trustworthiness and transparency; peer support; collaboration
and mutuality; empowerment, voice, and choice; and cultural, historical,
and gender acknowledgment.
• Law enforcement involvement should be limited to patient request, in
state-specific mandated reporting scenarios, or when clinicians suspect
imminent danger to staff or the patient.
1. Which of the following statements regarding human trafficking delayed presentation for care of injuries or infections; signs of abuse or
inquiry is true? neglect (e.g., malnourishment, poor oral health); work injuries or expo-
a. It is imperative that trafficked victims recount their history sures that would be easily prevented by personal protective equipment
as often as possible to ensure consistency and to support law (PPE). Other potential indicators include patient communications that
enforcement in the investigation of trafficking networks. seem rehearsed or scripted, patient reluctance or inability to answer
b. The primary goal of inquiry is disclosure. simple questions about their living or working situation, conflicting
c. Inquiry is a series of protocol-based standard questions designed information, reported mechanism of injury inconsistent with the phys-
to efficiently obtain information from traumatized victims. ical evidence, and evidence of psychological distress (e.g., limited eye
d. The goal of inquiry is to provide a safe environment for patients contact, low trigger threshold, hypervigilance, hyperstartle reflex).
to share as much or as little as they choose. 4. Which of the following statements regarding the evaluation and
Answer: D. Rather than disclosure, the goal of inquiry is to provide a treatment of trafficked victims is true?
safe environment in which patients feel empowered to share as much a. Empiric STI treatment and emergency contraception is indi-
or as little as they choose where strengths and resilience are recognized. cated only for sex trafficked victims and not labor trafficked
Inquiry-based assessment is an active process that includes open- victims.
ended questions and dialogue. b. Emergency department evaluation should include addressing
2. Which of the following statements regarding health impacts on acute medical issues and focus on forensic examination but
trafficked victims is false? should exclude the evaluation of chronic medical conditions.
a. Traffickers employ abusive tactics that can cause physical, repro- c. Law enforcement involvement should be limited to patient
ductive, and mental health problems. request or when clinicians appreciate imminent danger to staff
b. Schizophrenia is not a diagnosis commonly associated with the or patients.
psychological and psychiatric sequelae of victims of trafficking. d. When a suspected human trafficking case is identified, foren-
c. Commonly occurring smells, sounds, sights, and procedures in sic examination should be performed by the original provider
the emergency department may be perceived as threats by traf- because including a sexual assault examination by forensic nurse
ficking victims who have experienced trauma but is not caused examiners can worsen victim traumatization.
by neurobiologic changes. Answer: C. For both labor and sex trafficking, empirical STI treat-
d. Substance abuse and drug use rates are common in trafficked ment and emergency contraception may be indicated. ED evaluation
victims under 18 years of age. may include addressing acute medical issues, evaluation of possible
Answer : C. Trauma due to trafficking results in neurobiologic changes untreated chronic medical problems, documentation of acute and
such that commonly occurring smells, sounds, sights, and procedures remote injuries, and STI testing and treatment, and consideration of
of the emergency department environment may be perceived as threats a sexual assault medical forensic examination and evidence collection.
by those with trauma histories. Research shows that the use of sexual assault/forensics nurse examiners
3. Which of the following statements regarding the indicators and red results in better patient outcomes in legal and emotional support.
flags used to identify victims of trafficking in the ED are true? 5. Which of the following statements regarding the trauma-informed
a. Hypervigilance and hyperstartle reflexes are pathognomonic of care of trafficking victims is false?
human trafficking. a. Mitigation of conscious and unconscious biases is important in
b. Accompanying individuals who are reluctant to leave the room, contributing to the traumatization of trafficked victims.
insist on answering questions or attempting to control encoun- b. Preventing retraumatization is an essential feature of trauma-
ters, or communications that may seem rehearsed may all be informed care in the emergency department.
indicators that a victim is being trafficked. c. The PEARR Tool provides a trauma-informed framework for
c. Physical red flags of human trafficking often include immediate emergency clinician assessment but is only validated for victims
presentation for care for easily preventable injuries. of human trafficking.
d. Evidence of psychological distress including limited eye contact, d. If a patient denies victimization or declines assistance, the clini-
low trigger threshold, hypervigilance, and hyperstartle reflex are cian may offer information about resources that can assist in the
uncommon presenting characteristics of trafficked victims and event of an emergency.
are not reliable enough indicators in emergency departments to Answer: C. The Privacy, Educate, Ask, Respect, and Respond (PEARR)
raise suspicion. tool provides a trauma-informed framework for the emergency clini-
Answer: B. There is no single characteristic that is pathognomonic cian assessing for any form of interpersonal violence, including human
of human trafficking. Physical red flags of human trafficking include trafficking.
Sexual Minority Populations (LGBTQ) 188
3. Which of the following statements regarding medical gender
KEY CONCEPTS affirming therapy is true?
a. Commonly used medications include hormone supplementa-
• S exual orientation, gender identity, and gender expression occur along tion with androgen blocking medications in conjunction with
a continuum and may be completely separate and discordant from one estrogen.
another. b. The most common gender affirming medical treatment for the
• Multiple health disparities exist in sexual minority populations, but many of transgender male is spironolactone in addition to 5-alpha reduc-
the contributing factors are likely unknown due to lack of studies. tase inhibitors and aromatase inhibitors.
• ED provider education about sexual minorities is often inadequate, limited, c. Transgender women who went through puberty and had expo-
or absent. sure to testosterone will usually have complete elimination of
• Transgender individuals may utilize a variety of medical and surgical inter- male patterned facial or body hair and change in pitch in voice
ventions in their transition to affirm their gender identity. Emergency clini- although little change in skeletal bone structure would occur
cians should be aware of complications related to these practices that may with hormone therapy alone.
require treatment in the ED. d. Finasteride is a 5-alpha reductase inhibitor used only in trans-
• Sexual minority patients at extremes of age, or who share another minority gender males for gender affirming therapy.
identity, have additional disparities that require consideration to deliver e. Both A and B are true.
equitable and safe care.
Answer: A. The most common gender affirming medical treatment
• Open, inclusive, and nonjudgmental patient-centered communication using
utilized by transgender men is testosterone. The most commonly
appropriate terminology is fundamental to fostering provider trust and min-
used androgen blocker in the United States is spironolactone. 5-Alpha
imizing health disparities in sexual minorities.
reductase inhibitors (finasteride, dutasteride) are alternative androgen
blocking medications. Transgender women who went through puberty
1. Which of the following statements regarding sexual minority popu- and had exposure to testosterone will not have complete elimination of
lations is true? male patterned facial or body hair, change in skeletal bone structure, or
a. Sexual orientation, gender identity, and gender expression change in pitch of their voice with just hormone therapy. These require
define psychological, cultural, and behavioral characteristics of specific interventions such as plastic surgery, laser hair removal or elec-
either being male or female. trolysis, and voice therapy.
b. Lesbian, gay, bisexual, and homosexual are all acceptable terms 4. Which of the following statements regarding surgical gender
when addressing the sexual minority community. affirming therapy is false?
c. The demographics of sexual minority populations indicate that a. Genital procedures in gender affirming surgeries for transgender
a majority of vast transgender individuals were born after 1994 women also include the removal of the prostate gland as well as
(Generation Z). orchiectomy and vaginoplasty.
d. In sexual minority populations, it is important to separate sexual b. Complications of metoidioplasty and phalloplasty include spray-
and gender identities from sexual behavior. ing or dribbling of urine and the development of urethral stric-
Answer: D. Sexual orientation, gender identity, and gender expression tures and fistulas.
occur along a continuum and may be completely separate and discor- c. In transgender males, pregnancy is not a consideration.
dant from one another whereas gender refers to the psychological, d. Both a and c are false.
behavioral, and cultural characteristics of being either male or female. e. None of the above are true.
The term homosexuality is associated with past bias and discrimination Answer: D. Genital procedures in gender affirming surgeries for trans-
and is not a preferred term when addressing sexual minority popu- gender women do not include the removal of the prostate gland, so
lations. Sexual minority individuals cross every demographic, and the risk for prostate cancer persists necessitating screening. Pregnancy
multiple cultural and societal influences can affect their visibility and should always be considered in any patient who still has a uterus and
disclosure of sexual and gender identity. ovaries.
2. Which of the following factors does not affect equitable care in sex- 5. Which of the following statements is false?
ual minority populations? a. There is an increased risk for mental health disorders among the
a. Parental barriers sexual minority population.
b. Same-sex marriage b. The population of older Americans who identify as LGBTQ is
c. Access to care likely to shrink by the year 2030 and experience less depression.
d. Provider education and knowledge c. African American and Latino sexual minorities experience
e. None of the above greater stigma relative to white counterparts.
Answer: E. Same-sex couples with adopted children can face diffi- d. Rates of HIV have risen among the Asian and Latino males who
culty in legal recognition of parenthood, resulting in barriers in med- have sex with males over the last decade.
ical decision making and obtaining appropriate health care. Although Answer: B. There are approximately 3 million Americans who identify
marriage equality provided the opportunity for stronger and automatic as LGBTQ over the age of 55, and by 2030 it is estimated there will be
protections previously denied to same-sex couples, only 51% of same- 6 million, with increasing burdens of mental health and lifetime phys-
sex couples’ households are legally married compared to over 90% of ical disabilities. Although overall rates of HIV have been stable in the
heterosexual households. Sexual minority patients have less access to last decade, rates rose 52% among Asian and 18% among Latino MSM.
health care and are less likely to have health insurance compared to The HIV infection rate among Black MSM was stable and the infection
majority counterparts. ED provider education about sexual minorities rate among white MSM decreased 16%.
is often inadequate, limited or absent, leading to lack of understanding
and poorly informed decision making or advocacy on behalf of sexual
minority populations.
189 Social Determinants
KEY CONCEPTS
• C hallenges with food, housing, transportation, and other social determi-
nants of health account for approximately 50% of poor health outcomes in
the United States.
• Emergency providers can connect patients to resources that can help
address challenges with different social determinants of health.
• Legal services attorneys provide low-cost or free services that can assist
with a number of health-harming legal needs.
• Health care providers and systems need to work with nonprofits, commu-
nity groups, community members, and community leaders to expand the
capacity to address social determinants of health.
1. Which of the following statements regarding social determinants of 4. Which of following health-related benefits available in the United
health (SDOH) is true? States can impact access to care and health outcomes?
a. Less than 20% of health outcomes can be attributed to SDOH. a. Children’s Health Insurance Program (CHIP) is a specific pro-
b. SDOH equally impacts all segments of the US population gram focused on providing access to care for children up to age
regardless of income. 26.
c. Immigration status is not considered a SDOH. b. The Affordable Care Act requires plans and issuers that offer
d. Health literacy is one of the most significant aspects of SDOH. dependent child coverage to make coverage available until the
Answer: D. Patients may have limited health literacy, which is defined child reaches the age of 21.
as limited capacity to obtain, process, and understand basic health c. The Hill-Burton Act of 1946 mandates charity care obligations
information and services needed to make appropriate health deci- for facilities that benefited from the funding.
sions may have poorer adherence to care plans and poorer health out- d. Part A Medicare benefits are essential to supporting prescription
comes. An estimated 20% of a patient’s health outcome is influenced drug coverage.
by medical care, whereas social and economic factors account for 50% Answer: C. Facilities built with assistance from the Hill-Burton Act of
of such outcomes. Social concerns disproportionately affect the poor 1946 have legally mandated charity care obligations. There are specific
and impede health. Immigration as an experience has profound effects programs focused on providing access to care for children under the
on individuals and their health. Immigration status has an important age of 18, including the Children’s Health Insurance Program (CHIP).
effect on patients for two reasons—immigration status is linked to abil- The Affordable Care Act requires plans and issuers that offer depen-
ity to work and to access public benefits, including health care. dent child coverage to make coverage available until the child reaches
2. Which of the following factors impact health outcomes? the age of 26. Medicare is administered and funded by the federal gov-
a. Access to transportation ernment and contains several parts including inpatient hospital cover-
b. Homelessness and housing age (Part A), outpatient medical coverage (Part B), alternative coverage
c. Food insecurity options including Medicare Advantage (Part C), and prescription drug
d. Income and insurance coverage (Part D).
e. All of the above 5. Which of the following statements regarding legal services available
Answer: E. Access to transportation affects an individual’s ability to to ED patients is true?
access care, adhere to appointments, and carry out treatment plans, a. Legal services are typically unhelpful in increasing access to
having a direct impact on health. These individuals have worse health health care coverage but are invaluable in supporting immigra-
outcomes and mortality compared with those who are housed. Delayed tion status adjustments.
care is a common issue that arises in the ED as uninsured and under- b. A public charge applies to anyone seeking health care services,
insured patients often are unable to access care prior to an ED visit. so referral to legal services is essential to evaluate individual
Lack of access to healthy foods leads to increased rates of mental health cases and public benefits.
problems, low birth weight, depression, hypertension, diabetes, and c. In managing family and domestic concerns, referral to legal ser-
hyperlipidemia. vices should only occur if there is evidence of intimate partner
3. Which of the following ED-based strategies can counteract SDOH? violence or elder abuse.
a. Systematic screening in the emergency department can be a sig- d. Heath-harming legal needs are defined as a group of legal factors
nificant step in recognizing patients with significant SDOH. that can adversely impact health outcomes.
b. Addressing SDOH must occur only within the health care sys- Answer: D. Legal services are typically helpful in a wide variety of cir-
tem for the strategy to be successful. cumstances and address what may be considered health-harming legal
c. Addressing concerns of health literacy first will substantively needs. The idea of becoming a “public charge,” or one who is reliant on
impact health outcomes by improving patients’ failure to com- public benefits, has caused fear and hesitation among immigrant com-
ply with follow-up care instructions and access to community munities to seek care. While applying for and receiving certain public
resources. benefit programs may make one a public charge, seeking and receiving
d. Violence prevention is not a reasonable strategy in combating health care services never does. However, for the undocumented indi-
poor outcomes due to SDOH. vidual, eligibility to receive public benefits is impermissible unless the
Answer: A. Most departments do not currently screen for social needs. specific exceptions are met. Family dynamics and relationships can be
Instead, social needs arise during the course of the patient encounter. a source of stress and lead to emergency department visits and poor
Options for systematic screening exist but require implementing a health. Addressing challenges can improve health for patients. Patients
framework to address social needs. may face challenges with marriage and children. If there is no inti-
mate partner violence, then a referral to the local legal services agency,
courthouse, and/or bar association can help patients access resources
to resolve these concerns.
190 Community Violence
times higher than that of their white counterparts3 and African Amer-
KEY CONCEPTS icans are 500 times more likely to die this way.5 African American
• It is important for prevention efforts, including those in hospitals and emer- young adults and adolescents only make up about 2.4% of the US pop-
gency departments, to consider unjust social conditions disproportionately ulation, but they are disproportionately victims of injuries from assault
experienced by minority communities, and especially by African Americans; (26%) and homicide (20.7%).3,6
these include generational poverty, residential segregation, and other 4. According to Rosenblatt et al.,42 several fundamental principles of
forms of racism that limit opportunities to grow up in healthy and violence- HVIPs include all of the following except:
free environments. a. Violence is not preventable, but it can be managed.
• Interventions should focus on altering the quality of life course for survi- b. Hospitals should use public health–based principles to
vors of violence through targeting their total wellness and thriving through approach violence prevention, including working to address
employment, education, financial stability and economic mobility, indepen- poor education, lack of job opportunities, injury and crimi-
dence, and stable emotional health. nal recidivism, socioeconomically deprived neighborhoods,
• Hospital based violence intervention programs (HVIPs) that incorporate substance abuse, complex posttraumatic stress disorder, and
at least some brief intervention and case management demonstrate HVIP lack of positive role models.
cost-effectiveness with decreases in injury recidivism. c. There is a “teachable” moment during which a patient may be
more receptive to psychological first aid and prevention princi-
1. Which of the following statements correctly describes a difference ples following a traumatic injury.
between uninsured African American (AA) patients and uninsured d. Intervention will begin in the hospital but must follow the
white patients with depression? patient and family into their community. Case management
a. AA patients are less likely to be prescribed antidepressants. principles are implemented, ideally including professionals with
b. AA patients are less likely to be diagnosed with depression. therapeutic skill sets and “credible messengers” with knowledge
c. AA patients are more likely to think about suicide. of the patient population.
d. AA patients are less likely to need ongoing therapy. Answer: A. Several fundamental principles of HVIPs include:
Answer: A. A study observed that African American parents were “1.Violence is preventable,
much less likely to seek mental health care services for their children 2. Hospitals should use public health-based principles to approach
if they themselves held high levels of stigma towards mental health.40 violence prevention, including working to address the following
Furthermore, uninsured African Americans with depression are less in the patients and communities they serve: poor education, lack
likely to be prescribed antidepressants compared to those who are of job opportunities, injury and criminal recidivism, socioeco-
insured, and those who are insured are still less likely to be prescribed nomically deprived neighborhoods, substance abuse, complex
medications compared to their white counterparts.41 posttraumatic stress disorder, lack of positive role models,
2. Which of the following statements is true regarding individuals 3. There is a ‘teachable’ moment during which a patient may be
who are survivors of interpersonal violence and are exposed to more receptive to psychological first aid and prevention princi-
community violence? ples following a traumatic injury, and
a. They are more at risk for reinjury/trauma recidivism. 4. Intervention will begin in the hospital but must follow the
b. They are less at risk for retaliation. patient and family into their community. Case management
c. They are more likely to receive follow-up care. principles are implemented, ideally including professionals with
d. They are less likely to seek psychological support. therapeutic skill sets and ‘credible messengers’ with knowledge
Answer: A and D. Victims of interpersonal violence may experience of the patient population.”42
traumatic physical injuries and may seek treatment from hospitals/ 5. Racism is not correlated with which of the following factors?
emergency departments/trauma centers. The vast majority of these a. Poor housing
centers focus exclusively on stabilizing patients and clinicians in these b. Inferior education
settings may not recognize the important of assessing psychological c. Longer life
and mental health issues victims may be experiencing post-injury. d. Access to health services
Most trauma centers are not equipped with the resources, staff, and Answer: C. African Americans in the United States are exposed to con-
systematic approach to identify and address the social factors that con- stant racism that is systemic (organized socially and culturally) through
tribute to persistent community violence among the patient population exclusion, prejudice, and discrimination.21 Racism is correlated with
they serve.42 Individuals who are victims of interpersonal violence and second-rate employment, housing, education, income, and access to
have exposure to community violence are more at risk for reinjury/ health services. It is well known that racism is a factor in health dis-
trauma recidivism.43 Furthermore, discrimination and limited access parities and inequities that is not explained by other demographic
to mental health facilities increase disparities in mental health for com- factors such as age, gender, and educational attainment.21 Also, there
munities of color.38 are many risks for individuals who are racial or ethnic minorities such
3. Which of the following young populations is most at risk for poor as occupational hazards, exposures to toxic substances and allergens
physical and mental health outcomes from exposure to physical in the home, low-quality schooling, and lack of availability of healthy
violence (e.g., homicide and aggravated assault)? foods.20 While these risk factors are not clinical in nature, they make up
a. Men the social determinants of health, which includes factors such as race/
b. Women racism, poverty, education, housing, access to healthy food, environ-
c. African Americans mental exposures, violence, and gender.
d. Latino Americans
Answer: C. African American youth in the United States dispropor-
tionately carry the largest burden of violence experiences in general.
The homicide rate for African Americans in the United States is eight
191 Sexual Assault
KEY CONCEPTS 2. Which of the following empirical antibiotic regimens is indicated
for sexual assault patients to prevent sexually transmitted infec-
• S exual assault is most common in women but can happen to gay and tions?
heterosexual men and to lesbian, gay, bisexual, transgender, and gender- a. Cefixime 400 mg PO
nonconforming individuals. b. Cefixime 400 mg PO once plus doxycycline 100 mg PO bid for
• Sexual assault often results in no physical signs of injury. 10 days
• Optimal care includes creating a safe confidential environment while c. Ceftriaxone 1 g IM (intramuscularly)
incorporating the principles of trauma-informed care. The patient should d. Ceftriaxone 1 g IM plus azithromycin 2 g orally (PO)
be included in decision making and ultimately decide treatment. Options e. Ceftriaxone 250 mg IM plus metronidazole 2 g PO plus azithro-
include injury evaluation, treatment to prevent pregnancy and sexually mycin 1 g PO
transmitted infections (STIs), support and trauma counseling, evidence col- Answer: E. Ceftriaxone is given to cover gonorrhea, azithromycin to
lection, and comprehensive toxicology testing if appropriate. cover chlamydia, and metronidazole (Flagyl) to cover Trichomonas.
• The sexual assault evidence collection examination is an intensive, Ceftriaxone is preferred over oral cefixime to cover incubating syph-
protocol-driven, multistep process, best performed by a certified sexual ilis and due to increasing gonorrhea resistance. Many providers opt to
assault examiner. give the metronidazole to take at home because it increases the risk
• Adult sexual assault patients should be treated empirically according to of nausea and vomiting, a common side effect of many of the med-
Center for Disease Control (CDC) guidelines to prevent STIs (e.g., gonor- ications (including emergency contraception and HIV postexposure
rhea, syphilis, chlamydia, trichomonas, human immunodeficiency virus prophylaxis).
[HIV], human papillomavirus [HPV], and hepatitis B). Children and adoles- 3. Which of the following statements best describes sexual assault in
cents should be tested and, if symptoms develop, treated for STIs. males?
• All adolescent and adult female sexual assault patients should be offered a. Ejaculation should not occur in the victim during male sexual
pregnancy prophylaxis. assault.
• HIV postexposure prophylaxis should be offered if the assailant is known to b. Males are more likely to overreport sexual assault.
be HIV-positive, multiple assailants are involved, or if the HIV status of the c. Males are more likely to require sexually transmitted infection
assailant is unknown. (STI) prophylaxis.
• Alcohol and drugs may have been ingested voluntarily or involuntarily by d. Males do not require referral to rape crisis centers.
the patient. If the patient consents, comprehensive toxicology testing may e. Males may require anoscopy to detect anogenital injuries.
be performed. Answer: E. Males may actually suffer more anogenital injuries than
• A strangulation attempt with loss of consciousness, bowel and bladder women; injury detection can be aided or enhanced by using an ano-
incontinence, persistent voice changes, difficulty swallowing, or shortness scope. Males underreport the crime, do not seek medical attention,
of breath should be comprehensively evaluated in the emergency depart- and absolutely need referral to a rape crisis center for post-rape care
ment (ED). Evaluation options include a chest x-ray, flexible laryngoscopy, and counseling. Males are not more likely to require STI prophylaxis
and CTA or MRI of the neck. We recommend prolonged observation or hos- because the risk of transmission per act does not change based on
pital admission for persistent symptoms. gender. Ejaculation may occur during sexual assault due to prostatic
• Many victims will not have obvious physical injuries; this does not imply stimulation and fear arousal. This should not be taken to infer that the
consent or refute a sexual assault. assault was consensual.
• The emergency clinician should record observations, statements, and find- 4. Which of the following factors reduces the likelihood of finding
ings that were gathered during the course of ED treatment objectively. genital injury during the sexual assault examination?
a. Digital penetration
b. Increased time since sexual assault occurred
1. Which of the following statements best describes hepatitis B infec- c. Penile penetration
tion prevention for victims of sexual assault? d. Use of foreign object during the assault
a. Give HBIG and hepatitis B vaccine if the patient has not been e. Victim sexual immaturity
immunized. Answer: B. The genital structures heal quickly, so the longer the time
b. Give HBIG only if the patient has not been immunized. since the sexual assault occurred, the less likelihood of finding evidence
c. Give hepatitis B vaccination if the patient is unimmunized or of injury on examination. All the other factors increase the likelihood
uncertain. of finding genital injury at the time of the sexual assault examination.
d. Give hepatitis B vaccine only if serologic testing shows that the
patient is not adequately immunized.
e. Serologic testing is always required, followed by hepatitis B
immunoglobulin (HBIG).
Answer: C. Give hepatitis B vaccination if the patient is unimmunized
or uncertain. Follow-up doses should be given at 1 to 2 months and 4 to
6 months (total of three doses). This strategy, which avoids the need for
serologic testing, has been shown to be effective. HBIG is not recom-
mended by the CDC after sexual assault unless the assailant is known
to be HBsAg-positive and the victim is unimmunized.
5. A 25-year-old woman presents 4 days after vaginal penetration. Her control pills are options. Ulipristal is more effective after 72 hours and
body mass index (BMI) is 33. Which of the following is true about in women with a BMI greater than 26. At a BMI above 35, both forms
emergency contraception (EC)? of oral EC are less effective, but should still be administered if there is
a. A pregnancy test is mandatory prior to offering EC. no alternative. IUD placement is the most effective form of EC; it can
b. She should be offered levonorgestrel because it is more effective be placed up to 5 days after assault. IUD placement allows for ongoing
in this situation. birth control in situations where there is likely to be loss of reproduc-
c. She should be offered ulipristal. tive control (intimate partner assault) but is often less desirable after
d. She should have an intrauterine device (IUD) inserted because assault. IUD placement is most often not available in a timely manner.
this is the most effective form of EC for her. A pregnancy test is not mandatory before giving EC because it will
e. She should not receive EC because it will likely be less effective not harm an existing pregnancy. A pregnancy test is suggested prior
due to her BMI. to ulipristal administration, given the lack of large studies in pregnant
Answer: C. Emergency contraception should be offered up to 5 days women.
after vaginal assault. Ulipristal, levonorgestrel, and high-dose birth
192 Intimate Partner Violence and Abuse
KEY CONCEPTS
• Intimate partner violence encompasses a pattern of controlling behaviors,
3. The best practice for inquiring about IPV is which of the following?
including intentional physical assault, sexual assault, psychological vio-
a. Use open-ended questions that describe actions and feelings.
lence, and financial control.
b. Use the term “abuse” when questioning the patient.
• Intimate partner violence (IPV) patients are best served by a coordinated
c. Ise the term “victim” to ensure the patient knows the clinician is
system response plan that includes staff training, social work, victim advo-
concerned.
cates, and a close relationship with area IPV service provider groups and
d. Question the patient’s partner about what happened.
law enforcement.
Answer: A. Partners should be asked to leave in a routine manner
• The emergency clinician’s role in caring for patients affected by intimate
before the patient is asked about IPV. Patients will rarely admit to
partner violence involves 4 basic steps.
abusive acts unless they are guaranteed privacy, and initiation of the
1. Identification (asking the patient)
topic is potentially dangerous in the presence of an abusive partner
2. Treatment (supporting messages and medical issues)
who should not be questioned by providers. Providers should query
3. Documentation (regarding violent actions and threats)
the patient with any reasonable options that provide an empathetic
4. Referral (e.g., community, social services, legal)
and open approach facilitating improved screening rates. Providers
• Sequelae of IPV include chronic pain, mental health issues (e.g., depres-
should ask about specific actions and avoid use of the terms “victim”
sion, posttraumatic stress syndrome [PTSD], substance abuse), sexually
and “abuse” because patients may not yes see the actions as abuse or
transmitted infections (STIs) and unintended pregnancy, and worsening of
themselves as a victims.
medical illnesses.
4. Which of the following is most suspicious for intentional injury
from IPV?
1. Intimate partner violence occurs in which of the following popula- a. Ecchymosis of lower extremity
tion of patients? b. Paronychia
a. Restricted to lower socioeconomic classes and is never seen in c. Pattern injuries
wealthy highly educated individuals. d. Infected earring
b. Found only amongst women in heterosexual relationships and Answer: C. Signs of an intentional injury include a central location
not in other individuals. (i.e., trunk and breasts), bilateral injuries (both arms or both legs),
c. Patient has injuries from a weapon used in the assault. defensive injuries (i.e., ecchymoses on the back of the hand as a result
d. Similar prevalence in same-sex and gender nonconforming indi- of protecting the face), and patterned injuries (such as carved words on
viduals as in heterosexual couples. the back, a cigarette burn or other pattern of an object,).
Answer: D. IPV occurs with at least as high a prevalence in same-sex and 5. By the state law where Dr. Jane works, emergency clinicians practic-
gender nonconforming individuals as in heterosexual couples. It also ing in the state are mandated reporters for suspicious injuries from
is found commonly in wealthy and highly educated individuals. Most assaultive or abusive conduct. After identifying a patient with such
patients who present to the ED with conditions related to IPV do not suspicious injuries from IPV Dr. Jane should do which of the fol-
have injuries at the time of the visit. Finally, IPV violence consists of acts lowing regarding reporting?
or threats of physical, psychological, or sexual violence between intimate a. Not report because the Health Insurance Portability and
or formerly intimate partners who need not ever have been married. Accountability Act (HIPAA) prohibits reporting of this patient’s
2. The emergency clinician after identifying a patient suffering from injuries.
IPV should employ which of the following strategies? b. Report without telling the patient so the patient cannot object.
a. Call the police and have the perpetrator immediately arrested. c. Report as required by state law because it supersedes patient’s
b. Validate the disclosure of abuse and communicate the importance HIPAA consent rights.
of leaving the situation immediately to ensure personal safety. d. Obtain a signed consent to make the report.
c. Provide kind and supportive messages and information. e. Call the legal department before reporting.
d. Report to the public health department. Answer: C. Some states have laws that require injury reporting to local
Answer: B. Emergency clinicians should validate the disclosure of authorities. Patients should be informed about state-specific report-
abuse, emphasize that the victim is not at fault, and encourage future ing requirements that may accompany disclosure of abusive injuries.
discussions with IPV community agencies or other health care pro- Mandatory reporting of health conditions required by local laws are
viders. Immediate safety should be assessed, but most patients will exempted from HIPAA regulations. In general, IPV should not be
not want to leave the abuser immediately; however, a positive initial reported to police without the consent of the survivor unless mandated
conversation may begin the process of ending the abusive relationship. by law (as in this case) or in cases of coexisting child, elder, or dis-
Reporting to law enforcement or health department should be done abled abuse or based on state-specific reporting statutes (e.g., burns or
only with victim consent or if mandated by law. A templated list will injuries inflicted by weapons). If reporting is mandated, the provider
allow the ED staff to create a basic safety plan with the patient; an indi- should make every effort to involve the patient. However, concerns for
vidualized plan is best done in conjunction with trained domestic vio- HIPAA violations do not apply in this circumstance; the privacy rule
lence advocates, typically in follow-up. Referral to couples counseling contains a provision allowing disclosure of protected health informa-
is inappropriate and ineffective in violent relationships and potentially tion to law enforcement in the case of reporting required by law.
dangerous.
Global Emergency Medicine e1
KEY CONCEPTS
• W orld Health Assembly resolution 72.16, “Emergency care systems for uni-
versal health coverage: ensuring timely care for the acutely ill and injured,”
was passed in 2019 and calls for the strengthening of emergency care
delivery through a detailed list of initiatives across the prehospital, facility,
public health, and disaster preparedness fields.
• In low-and middle-income countries (LMICs), substantial excess mortal-
ity amenable to emergency health care exists, and the development of
high-quality emergency care can be a significant systemic contribution to
improving mortality.
• Global emergency medicine (GEM) is an academic subspecialty of emer- Answer: A. The direct delivery of unsupervised medical care should
gency medicine focused on the strengthening of the specialty worldwide only be provided by a GEM physician with a current and valid license
and on improving international humanitarian response. issued by the appropriate regulatory body of the country within which
• GEM can be divided into development and humanitarian work. Develop- care will be delivered. Additional requirements, such as hospital cre-
ment work is typically focused on strengthening local systems in a sus- dentialing, are also generally required.
tainable manner, while humanitarian work involves an acute response to 3. The practice of global emergency medicine is an academic subspe-
large-scale crises in which local resources are overwhelmed. cialty that focuses on which areas of medicine and public health?
• The model curriculum for a GEM fellowship includes experiences in emer- a. All areas of medicine are represented except for surgical condi-
gency medicine systems, humanitarian relief, disaster medicine, public tions, as these are addressed by global surgeons.
health, travel and field medicine, program administration, academic skills, b. All areas of medicine for all age groups are represented, inclusive
and clinical skills. of trauma or surgical conditions, infectious and noninfectious
conditions, and pregnancy.
1. Research projects based in settings outside of the GEM physician’s c. All adult areas of medicine are represented, but not pediatrics as
home country require which of the following special considerations this field is addressed by global pediatricians.
with regard to research ethics? d. GEM focuses only on emergency conditions, excluding chronic
a. Ethical approval must be obtained only at the GEM physician’s disease as this generally does not contribute to the need for
home institution. GEM.
b. United States-based IRBs do not cover international settings, and Answer: B. Recognizing that patients present to the emergency depart-
as such the GEM physician does not need to seek ethical approval. ment (ED) without a diagnosis, but with a complaint, GEM focuses
c. Ethical approval is required from both the GEM physician’s on presentation-based solutions for conditions for all age groups, both
home institution and any partner institutions or research sites. trauma and medical, infectious and noninfectious, and pregnancy.
d. If the research is planned in a country that does not have an IRB, GEM seeks to ensure early access to quality emergency care for all peo-
no formal ethical approval is required. ple, regardless of gender, ethnicity, age, or ability to pay.
Answer: C. Ethical approval for any research project must be obtained 4. What is the relationship between humanitarian work and develop-
from the GEM physician’s home institution, any partner institutions, ment work as it pertains to the GEM physician?
and any research sites. In the event that a formal IRB does not exist for a. Humanitarian work and development work are best performed
a certain research site, efforts must be made to obtain and document separately to avoid confusion around roles, tasks, and funding
ethical approval from an appropriate site representative. commitments.
2. Which of the following is required when planning a clinical expe- b. GEM physicians provide only development work and do not
rience abroad in which the GEM physician will be providing direct focus on humanitarian response, which requires separate train-
clinical care without supervision? ing and expertise.
a. Current medical licensure from the country within which care c. Humanitarian and development work are the same thing, and
will be delivered. the difference in terminology is a function of who is performing
b. Current medical licensure from the GEM physician’s home the actual work.
country. d. Humanitarian work and development work complement and
c. Written agreement between the GEM physician and a licensed strengthen each other.
physician in the country within which care will be delivered. Answer: D. While GEM is broadly divided into humanitarian and devel-
d. No local medical license is required as long as the GEM phy- opment work, these two subfields are closely related and complement
sician is practicing within the same scope as allowed by home each other. An emergency care system strengthened through develop-
licensure. ment work should be able to surge during crises to enhance humani-
tarian efforts, and an effective humanitarian response will include
strengthening the local system to provide resilience against future crises.
e2 Humanitarian Aid in Disaster and Conflict
KEY CONCEPTS
• A key feature of humanitarian crises is the mass displacement of large
numbers of people from their homes. Displaced populations are designated
either as internally displaced persons (IDPs) or refugees.
• The international response community has developed practice guide-
lines that stem from international law, research, and expert consensus.
Responders should be familiar with these international standards and laws
to ensure that intervention is appropriate and ethical.
• The Sphere Handbook, a collaborative project of humanitarian response
experts, sets international standards for the provision of humanitarian aid
and is widely considered to define the gold standard of global humanitarian
response.
• Demand for humanitarian responders is likely to rise in the future as global
urbanization and an increase in climate-related disasters conspire to create
more frequent and severe crises.
1. Which of the following scenarios often creates ethical dilemmas for 3. What is the distinction between internally displaced persons and
those providing humanitarian assistance? refugees?
a. Availability of water a. Internally displaced persons may remain in country or reside in
b. Creating housing for patients another country.
c. Local government policies b. Refugees are persons who are displaced and reside in another
d. Triage of patients by local custom country.
Answer: D. Although all these issues are important, humanitarian c. Internally displaced persons have more rights and benefits than
workers often have ethical concerns about triage based on local cus- refugees.
toms versus severity of illness d. The refugee is dependent on the number of displaced persons.
2. Which of the following best describes the Sphere Handbook? Answer: B. Displaced populations are designated either as internally
a. A global compilation of nongovernmental agencies displaced persons (IDPs) or refugees. An IDP is someone who has
b. A description of legal decisions in international courts been forced from his or her home but who remains within the coun-
c. An educational guide to global relief efforts try of origin. If an IDP crosses an international border to seek help in
d. A guide that sets internal standards for humanitarian aid another country, she or he becomes a refugee. Although refugees enjoy
Answer: D. The Sphere Handbook, a collaborative project of humani- protections and rights guaranteed by international treaty, IDPs must
tarian response experts, sets international standards for the provision depend (in the absence of international aid) on their own government
of humanitarian aid and is widely considered to define the gold stan- for help, even though the actions—or inaction—of their government
dard of global humanitarian response. often caused their displacement,
Emergency Ultrasound e3
KEY CONCEPTS
• O ver the past 20 years, emergency ultrasound (EUS) has become an integral
part of emergency medical care in the United States and has become stan-
dard in the evaluation of emergency medical conditions.
• EUS answers specific, often binary questions, and is therefore neither suf-
ficient nor intended to diagnose all of the broad range of pathologic pro-
cesses encountered in emergency medicine.
• During cardiac arrest, ultrasound can be used to rapidly detect ventricu-
lar motion in asystole and pulseless electrical activity and confirm cardiac
standstill.
• The subcostal four-chamber view, as in the focused assessment of sonog-
raphy in trauma, is ideal for assessment of pericardial effusion and useful
during cardiac arrest because it does not interfere with chest compres-
sions.
• Although cardiac tamponade is a clinical diagnosis, there are several sug-
gestive echocardiographic features, including diastolic collapse of the RV,
loss of respiratory variation of the inferior vena cava (IVC), and transvalvu-
lar flow velocity paradoxus.
• EUS is more sensitive and specific compared with supine chest radiography
for the detection of pneumothorax, approaching that of CT.
• Pneumonia on EUS appears as dynamic air bronchograms, hyperechoic
areas within bronchi that move with respiration usually visualized within
the consolidated lung, and are highly specific for alveolar consolidation.
• Color Doppler can help differentiate mild hydronephrosis from renal vas-
culature, and possibly accentuate any renal stones by producing the renal
twinkle artifact.
• EUS has reduced the morbidity of ectopic pregnancy by shortening the time
to diagnosis and operating room treatment.
• In the ED, ultrasound-guided internal jugular cannulation is associated with
decreased time to vessel penetration and improved success in the difficult 3 Which type of US has the best resolution when the probe is perpen-
access patient, improved overall and first-attempt success rates, reduced dicular to the object of interest?
time to insertion, and reduced complication rate. a. Doppler US
b. Grayscale US
Answer: B. Grayscale or B-mode US produces best imaging when
1 Which frequency range defines US? perpendicular to the area of interest, whereas Doppler US is best per-
a. Less than 0.2 Hz formed when the angle of insonation is <60 degrees.
b. Over 2 Hz 4 Sensitivity is typically greater than specificity for FAST examina-
c. Over 200 Hz tion.
d. Over 20,000 Hz a. True
Answer: D. US is defined as sound with frequency over 20,000 Hz. b. False
Answer: B. The FAST examination is typically more specific than
2 Which of the following performance characteristics is true for sensitive.
higher-frequency probes?
a. Deeper penetration, higher resolution 5 Which of the following is the correct order of imaging for increas-
b. Deeper penetration, poorer resolution ing the sensitivity of detecting a pneumothorax?
c. Superficial penetration, higher resolution a. Chest x-ray (CXR) > CT > US
d. Superficial penetration, poorer resolution b. CT > CXR > US
Answer: C. Higher-frequency probes sacrifice penetration (depth of c. CT > US > CXR
scan) for improved resolution (image detail). d. US > CT > CXR
Answer: C. CT is only slightly more sensitive than US at detecting
pneumothorax, but US is significantly more sensitive than supine CXR.
The Geriatric Emergency Department e4
KEY CONCEPTS
• T he rapid increase in the number of older adults in the United States (US)
and around the world, as well as the unsustainable costs of the current US
health care system, mandates improved emergency care systems for these
vulnerable patients.
• Older adults are at high risk of experiencing harm from busy, crowded emer-
gency departments (ED). While in the ED, they may experience prolonged
lengths of stay, iatrogenic infections, misdiagnosis, delirium, bedsores and
other adverse consequences.
• Older adults who are discharged are at high risk of ED return visit, hospi-
talization, or death within three months. Because older adults admitted to
the hospital are at high risk of adverse events, care transition programs are
needed to reduce the risk of inpatient stays and improve post-ED care.
• The Geriatric ED Guidelines are an excellent resource to improve the phys-
ical and process design to better address the needs of older adults.
• The Geriatric ED Collaborative and Geriatric ED Accreditation programs are
innovative programs that facilitate the implementation of better emergency
care for older adults.
1. For which of the following outcomes are hospitalized elders at d. Pain management and palliative care
increased risk? e. Medication management
a. Infections Answer: C. See Box e4.1.
b. Delirium 4. ED consultation from which of the following interdisciplinary ser-
c. Functional decline vices has been shown to decrease the likelihood of an elder return-
d. Iatrogenic injuries ing to the ED with a fall within 30 days?
e. All of the above a. Pharmacy
Answer: E. Geriatric patients are particularly vulnerable to the dele- b. Social work
terious consequences of hospitalization, including nosocomial infec- c. Case management
tions, functional decline, increased rates of delirium, and iatrogenic d. Physical therapy
complications. e. Occupational therapy
2. The Geriatric ED (GED) Guidelines developed by ACEP, SAEM, Answer: D. Physical therapy. Although all of the above interdisciplin-
AGS, and ENA make recommendations in all of the following areas ary services likely improve care for older adults in the ED, being eval-
except: uated by a physical therapist in the ED after a fall is associated with
a. Education and patient care protocols lower likelihood of returning to an ED with another fall within 30 days.
b. Proper technique for central line placement 5. Which of the following core content areas is not a component of
c. Quality improvement measures geriatric emergency medicine education?
d. Staffing, supplies, and administration a. Elderly trauma, including falls and hip fracture
e. Follow-up and transitions of care b. Myocardial infarction
Answer: B. The GED guidelines consist of 40 specific recommen- c. Cognitive and behavioral disorders
dations in six general categories: (1) staffing and administration; d. Medication management
(2) equipment and supplies; (3) education; (4) policies, procedures, e. Transitions of care
and protocols; (5) follow-up and transitions of care; and (6) quality- Answer: B. Although myocardial infarction is a condition commonly
improvement measures. suffered by older adults, knowledge of myocardial infarction is not in
3. The GED guidelines recommend elder-specific educational content and of itself geriatric emergency medicine education. Geriatric emer-
in all of the following areas except: gency medicine education involves geriatric syndromes, conditions
a. Atypical presentations of disease underlying the acute presentation of disease such as polypharmacy or
b. Cognitive and behavioral disorders frailty, and management of conditions that are different for older adults
c. Psychotherapy for adjustment disorders than younger adults.
e5 End of Life
KEY CONCEPTS
• P alliative care is physical, spiritual, psychological, and social support pro-
vided to patients and families at any stage of serious illness.
• Palliative care teams are specialized interdisciplinary teams that should be
involved early in the course of illness.
• Hospice care is a care system for patients and families with a prognosis
of 6 months or less if the disease runs its usual course; referral from the
emergency department (ED) may be indicated.
• Patients at the end of life often follow one of four terminal illness trajecto-
ries: sudden death, organ failure, cancer, or frailty. Awareness about these
trajectories facilitates decision making.
• Outcomes are very poor for patients with advanced age or illness and car-
diac arrest or respiratory failure requiring mechanical ventilation. Decision
support tools should be incorporated into shared decision making.
• Goals-of-care conversations that outline patient and family hopes and
expectations of interventions should be attempted in the ED.
• Emergency clinicians should make recommendations for care plans, inter-
ventions, and treatment courses based on prognosis, goals of care, and
expected benefits of meeting the identified goals of care.
• Interpretation of an existing advance care plan or receipt of a prehospital
order (e.g., POLST—Physician Orders for Life-Sustaining Therapy) should
factor into ED treatment plans in the context of the patient’s illness trajec-
tory.
• Symptom management (e.g., dyspnea, pain, delirium, agitation, secretions)
is critical at the end of life.
1. Which of the following statements regarding patient confidentiality c. The costs and benefits of consequences in relation to a relatively
is FALSE? stable framework of personal values and priorities.
a. It describes the patient’s right to enough physical and auditory d. The court’s decision about their competency.
isolation so that they cannot be seen or heard by others during Answer: D. To be deemed competent an individual must understand A,
interactions with medical personnel. B, and C. Answer D is a legal issue and is not required for competency.
b. It imposes a duty on health care workers. 4. Which of the following statements regarding the withholding of
c. It may conflict with the law, especially public health statutes. treatment in the ED is TRUE?
d. It presumes that what the patient tells the physician will not be a. It legally differs from withdrawing treatment.
revealed to any other person or institution without the patient’s b. It morally differs from withdrawing treatment.
permission. c. It requires clinical information that is often unavailable immedi-
Answer: A. Although isolation of patients for confidentiality is a good ately.
goal, the limited space and hectic nature of an ED would sometimes d. It should never be done.
prevent timely and quality care. Answer: C. In an emergency not all relevant information is available at
2. Which of the following can emergency clinicians always rely on for the time important decisions must be made.
appropriate guidance when faced with ethical dilemmas? 5. When using the “rapid approach to ethical problems in the emer-
a. State laws and medical board policies gency department (ED)” to decide on a course of action:
b. The American College of Emergency Physicians’ (ACEP) Code a. Always consult with the bioethics committee/consultant before
of Ethics acting.
c. The Hippocratic Oath b. Assume that each ethical problem in the ED requires a unique
d. None of the above solution.
Answer: D. All these examples are great for guidance on ethical dilem- c. Test your chosen action against your religious values.
mas, but none of them can “always” have guidance for every situation d. When practicable and safe for the patient, buy time to consult on
that can occur. possible options.
3. To have adequate decision-making capacity in any circumstance, a Answer: D. Ethical problems can often take some time to work out, but
person must understand all of the following EXCEPT: a rapid approach rarely has enough time for a formal ethics consult.
a. The available options. Buying some time so a reasonable solution can be derived is often the
b. The consequences of acting on those options. best that can be done.
Emergency Medical Treatment and
e7 Labor Act and Medicolegal Issues
KEY CONCEPTS
• T he Emergency Medical Treatment and Labor Act (EMTALA) governs virtu-
ally every aspect of hospital-based emergency services.
• Centers for Medicare and Medicaid Services (CMS) deems anyone on
hospital property to have “come to the emergency department.” Hospital
property consists of the entire main hospital campus, including parking
lots, sidewalks, and driveways, and any ambulance owned and operated
by the hospital, even if the ambulance is not on hospital grounds.
• Any person who comes to an ED requesting examination or treatment
for a medical condition must be provided with an appropriate medical
screening examination (MSE). The purpose of the MSE is to determine
whether the patient has an emergency medical condition (EMC).
• Presentation to the ED or on hospital property is not sufficient to trig-
ger the hospital’s duty to provide an MSE; a request for examination or
treatment of a medical condition is also necessary. The request can be
made by the patient or by anyone on behalf of the patient (e.g., EMS per-
sonnel, a police officer, or a babysitter, family member, or legal guard-
ian).
• CMS regulations require that the screening examination be done by
“qualified medical personnel.” Typically, the designation applies to
physicians or advanced practice providers (nurse practitioners [NPs] or
physician assistants [PAs]).
• An appropriate MSE must be reasonably calculated to identify critical
medical conditions and must be uniformly provided to all patients who
present with substantially similar complaints.
• Hospitals need to have a policy and practice for left-without-being-seen
(LWBS) patients that adequately documents pertinent findings and pro-
tects the hospital from liability.
• Medicare-participating hospitals that have specialized capabilities or
facilities are required by EMTALA to accept appropriate transfers of
patients who require such capabilities or facilities if the hospital has the
capacity to treat the patient.
• Once a hospital admits the patient in good faith as an inpatient for fur-
ther treatment, the hospital’s obligation under EMTALA ends.
• Emergency Consent for Procedures
• The emergency clinician who proposes to undertake the procedure must
be the one to obtain the patient’s informed consent. The duty to obtain
consent cannot be delegated.
• Whenever emergency clinicians are in doubt about the legality of a situ-
ation related to consent, they should do and document what they believe
to be in the patient’s best interest. Delaying treatment in an emergency
to obtain informed consent is a much more serious and common medico-
legal problem than failure to obtain proper informed consent.
• Consent for minors in the ED is a nonissue. Consent is a creature of state
law; EMTALA preempts state law, and it requires the ED to provide an
MSE and stabilizing treatment to all minors presenting to the ED.
• A parent’s right to freedom of religion (e.g., Jehovah’s Witness) does
not include the right to deny life-sustaining medical intervention for that
person’s children.
• Leave Against Medical Advice
• Emergency clinicians should be cautious when patients the demonstrate
intention to leave the ED against medical advice. The capacity to make
reasonable medical decisions can easily be affected by alcohol, drugs,
pain, or any number of medical conditions. Careful evaluation and doc-
umentation of the patient’s decision-making capacity supports the deci-
sion to allow the patient to leave or to retain the patient against their
will. The courts (and public opinion) tend to support clinicians who act in
the best interests of the health and safety of their patients.
1. Which of the following is a common source of liability for failure Answer: B. The Emergency Medical Treatment and Labor Act
to comply with the Emergency Medical Treatment and Labor Act (EMTALA) defines the time of acceptance of responsibility as when
(EMTALA)? the patient comes to the ED. In its EMTALA regulations, the Centers
a. Failure to comply with written hospital policies and procedures for Medicare and Medicaid (CMS) defines the term comes to the ED
b. Failure to diagnose an emergency medical condition (EMC) to include anywhere on hospital property, so the hospital’s duty and
c. Failure to properly treat an admitted patient who is boarded in responsibility for the patient are triggered when the ambulance arrives
the emergency department (ED) while waiting for an intensive on hospital property.
care unit (ICU) bed to become available 4. A physician on-call for the hospital’s emergency department (ED)
d. Failure to stabilize a patient after admission to an inpatient unit has a duty under the Emergency Medical Treatment and Labor Act
Answer: A. Hospital policies become the hospital’s standard under (EMTALA) to come to the ED under which of the following cir-
EMTALA. Thus, failure to comply with written policies, a failure to cumstances?
follow your own rules, becomes an EMTALA violation when the pol- a. If asked to help stabilize a patient in the ED who has an emer-
icies are related to the medical screening and stabilization of patients gency medical condition (EMC)
in the ED. b. To help transfer a stable patient to another acute care hospital
2. Which of the following violates the Emergency Medical Treatment that has a specialist available that is not available at the transfer-
and Labor Act (EMTALA)? ring hospital
a. A 7-hour wait after triage to be medically screened by the emer- c. When asked to consult on a patient who has been stabilized in
gency clinician the ED and is being admitted by the patient’s primary care phy-
b. Boarding an admitted patient in the emergency department sician
(ED) for 72 hours after the patient’s physician has accepted the d. Whenever requested to come to the ED by the emergency clini-
patient for admission cian on-duty
c. Delaying the patient’s access to an ED bed by declining to Answer: A. Under EMTALA, the on-call physician only has a duty to
promptly accept the patient from an arriving emergency medical come to the ED if needed to help determine whether the patient has an
service (EMS) unit or by failing to immediately triage or evaluate EMC or to stabilize an EMC. Any other requirements concerning when
the patient to determine if the patient can wait on a stretcher the on-call physician must come to the ED are governed by state law
with the EMS folks until a bed becomes available and/or by the hospital’s medical staff bylaws, not by EMTALA.
d. Delaying the patient’s medical screening examination (MSE) in 5. According to the Centers for Medicare and Medicaid Services
the ED to triage the patient, and then for the emergency clini- (CMS), which of the following patients may a hospital with special-
cian to see the patients in the order based on the triage nurse’s ized capabilities and available capacity refuse to accept in transfer
perception of the patients’ acuity solely because the patient is uninsured?
Answer: C. Centers for Medicare and Medicaid Services (CMS) guide- a. A patient with an unstable emergency condition admitted to
lines require the hospital to immediately triage ambulance patients observation at another hospital that is unable to stabilize the
arriving at the hospital’s ED. patient’s condition
3. A 73-year-old woman is brought by private ambulance to the emer- b. Any patient the hospital chooses to reject
gency department (ED) for shortness of breath. Due to ED over- c. An emergency department (ED) patient with an unstable emer-
crowding, there are several ambulance stretchers in line ahead of gency condition who is at another hospital that is unable to sta-
her at triage. At what point does the hospital accept responsibility bilize the patient
for this patient? d. An inpatient with an unstable emergency condition at another
a. When she was placed in the ambulance hospital that is unable to stabilize the patient
b. When the ambulance arrived on hospital property Answer: D. CMS guidelines state that hospitals do not have to accept
c. When the patient is seen by the emergency clinician an inpatient, as defined by CMS, in transfer from another hospital
d. When the patient is seen by triage personnel under any circumstances, even if its decision to decline to accept the
patient in transfer will result in the patient’s death.
e8 Quality Improvement and Patient Safety
KEY CONCEPTS
• T he work of health care occurs within a complex socio-technical system so
that a change in even one component has an impact on other parts of the
system and ultimately on safety.
• Patient safety and mitigation of risk emerges from multifactorial interac-
tions within the clinical work system. Incidents of patient or staff harm are
most often the result of system failures, not individual human error.
• Process improvements to clinical care often have unintended consequences
elsewhere within the work system. Partnerships with information technol-
ogy; cognitive, behavioral, and social scientists; and engineers can help
reduce the likelihood of these events and improve adoption by workers.
• Don’t set up reporting systems without the resources to fully analyze the
reports.
1. Which of the following triage assessments are important contribu- the team does not feel empowered to voice a concern regarding patient
tors to error in emergency medicine? safety, it can potentially have adverse consequences.
a. Triage by age 3. Which of the following may ameliorate the effect of shift work?
b. Triage by nursing personnel a. Five consecutive night shifts
c. Inappropriate triage to a treatment area b. Assigning more senior staff to nights only
d. Triage based on comorbidities c. Sedatives
e. In-room patient triage and registration d. Circadian schedule
Answer: C. An inappropriate triage to a specific treatment area may Answer: D. A circadian schedule with forward rotating shifts has the
create a bias in the minds of the treating physician and staff, leading potential to improve sleep habits for those with a rotating shift sched-
to delays in care or misdiagnoses. This may be more pronounced with ule. Excessive amounts of consecutive nights, sedatives, and increased
undertriage versus overtriage. night shifts for senior attendings are all poor strategies to deal with the
2. A senior attending asks a resident to discharge a young patient burden of shift work.
with abdominal pain. The resident has noted lab abnormalities 4. Which of the following are more likely to cause error in emergency
she believes merits further work-up, but assumes the attending has medicine?
seen these results and defers to the attending’s judgment. These a. Low patient volumes, poor teamwork, electronic health record
abnormalities had in fact been overlooked by the attending and b. Inadequate supervision, high communication load, multiple
the patient gets sent home without appropriate work-up. This is an transitions of care
example of which potentially negative process? c. Overstaffing, multiple distractions, low numbers of transitions
a. Authority gradient in care
b. Anchoring d. Physician salaries, language barriers, quiet work environment
c. Countertransference Answer: B. Many factors affect an ED’s performance, making it more
d. Fatigue and shift work at risk for a medical error (see Table e8.1). Although each choice con-
e. Visceral bias tains at least one of these performance-affecting factors, only choice B
Answer: A. The hierarchical nature of medicine can lead to authority contains three of the factors.
gradients which may contribute to errors. If a more junior member of
e9 Patient Experience in the Emergency Department
KEY CONCEPTS
• F actors that can improve the patient experience in the emergency depart-
ment include:
• Communication that expresses empathy
• Working as a high-functioning team
• Setting realistic expectations regarding wait time
• Clear discharge instructions
• Improvement in the patient experience has been shown to impact com-
posite outcomes such as health-related quality of life to patients, reduced
malpractice risk, and improved staff satisfaction.
1. Which of the following outcomes are impacted by patient experi- Answer: A. Key drivers of patient experience are communication,
ence? empathy, expectation setting, and wait times.
a. Health-related quality of life 3. What is an evidence-based practice you can use to improve your
b. ED length of stay patient’s experience?
c. ED return visits (“bounce backs”) a. Using a computer on wheels
d. Staff salaries b. Sitting in the room
Answer: A. Patient-related outcomes (health-related quality of life, c. Bedside rounding
various pain scores, anxiety, depression, and specific disease conditions d. Direct bedding
such as asthma, blood pressure, diabetes, osteoarthritis, and weight Answer: B. Sitting is a straightforward technique that has been shown
loss) and staff-related outcomes (improve experience, satisfaction, and to increase patients’ perception of the provider’s skill at communica-
joy at work and decrease burnout, depersonalization, and emotional tion and listening, makes instructions easier to understand, and leaves
exhaustion). patients with a more positive evaluation of the experience. It has also
2. What is a key driver of patient experience? been shown to result in patients’ perception that emergency clinicians
a. Communication have spent more time at the bedside, without having actually spent
b. Years of staff experience more time, than those that stand. None of the other mentioned strate-
c. ED boarding gies have been studied in relationship to patient experience in the ED.
d. Nursing ratios
Wellness, Stress, and the Impaired Physician e10
KEY CONCEPTS
• P hysician Wellness: A quality of life that includes the presence of positive
physical, mental, social, and spiritual well-being experienced in connection
with activities and environments that allow physicians to develop their full
potentials across personal and work-life domains.1
• Physician Burnout: A work-related syndrome characterized by emotional
exhaustion, depersonalization, and a sense of reduced personal accomplish-
ment. It is frequently associated with chronic stress and emotionally intense
work demands for which resources are inadequate. Physicians exhibit a
higher rate of burnout than the general population, with emergency physi-
cians experiencing some of the highest rates in the medical profession.2-6
• Stress: A nonspecific response of the body to any demand that can have
both positive and negative effects. Without any demand, performance
suffers. With too much stress, anxiety and exhaustion lead to poor perfor-
mance.1 Chronic stress is associated with burnout.
• Compassion Fatigue: Resulting from exposure to a traumatized individual,
compassion fatigue has been described as the convergence of secondary
traumatic stress and burnout. It can lead to a reduced capacity and interest
in being empathetic toward future suffering.7
• Resilience: The ability of a person, community, or system to withstand,
adapt, recover, rebound, or even grow from adversity, stress, or trauma.
When considering individual resilience, personality traits such as optimism
and altruism are found in resilient people, but behaviors such as mindful-
ness and reflective practices also aid resilience. External factors such as
social support and good self-care are also important.8-10
• Impaired Physician: A physician who is unable to practice medicine with
reasonable skill and safety due to mental or physical illness; due to a con-
dition that adversely affects cognitive, motor, or perceptive skills; or due to
substance abuse. Substance abuse has been described as a direct conse-
quence of work stress and burnout for physicians.1,11
• A Systems Model of Physician Burnout: The process of physician burnout
is complex and occurs within a multilevel system that includes the individ-
ual, the front-line care delivery team, the health care organization, and the
external environment (governmental agencies, regulatory bodies, societal
norms, etc.). To effectively address burnout, interventions must be directed
at all levels of this complex system.10
1. Which of the following statements regarding factors that contribute b. Physician burnout is frequently associated with chronic stress
to occupational stress of the emergency clinician are true? and emotionally intense work demands for which resources are
a. Emergency clinicians with experience are not significantly inadequate.
impacted by end of life issues in the emergency department. c. Emergency physicians exhibit a lower rate of burnout com-
b. Personal safety is usually not a concern for the emergency phy- pared to physicians in other specialties and to the general
sician as there is often sufficient security to guard against the population.
impact of violence or verbal abuse in the ED. d. Physician burnout can often result in impairment that results
c. The expectation of error-free clinical practice is a major stressor in an inability to practice medicine and can lead to substance
for emergency clinicians. abuse.
d. Most emergency departments are not significantly stressful e. Both B and D are true.
environments because of the predictability of patient acuity and Answer: E. Physician burnout is a work-related syndrome charac-
patient arrival volumes. terized by emotional exhaustion, depersonalization, and a sense of
Answer: C. Personal safety is also a major issue while working in the reduced personal accomplishment. It is frequently associated with
ED environment, where exposure to acts of violence, verbal abuse, and chronic stress and emotionally intense work demands for which
risk of exposure to potentially life-threatening diseases is higher than resources are inadequate. Physicians exhibit a higher rate of burnout
in other practice settings. In addition to the aforementioned stresses is than the general population, with emergency physicians experiencing
the expectation of error-free practice and diagnostic certainty, with the some of the highest rates in the medical profession.
constant possibility of human error, a missed diagnosis, and the atten- 4. A systems model of physician burnout includes all of the following
dant risk of medical negligence as an ever-present stress for emergency features, except:
clinicians. The ED is a highly demanding environment because of 24/7 a. The external environment
patient care in settings where life-threatening illness and injury occur b. Organizational support and buy-in
with great frequency, and where arrival volumes, patient acuity, and c. Governmental agencies and regulatory bodies including licens-
nature of emergencies rapidly and unpredictably shift. ing boards
2. What specific strategies exist to impact individual wellness for d. Religious factors
emergency clinicians? Answer: D. The process of physician burnout is complex and occurs
a. A focus on individual well-being alone is usually sufficient to within a multilevel system that includes the individual, the front-line
prevent burnout. care delivery team, the health care organization, and the external envi-
b. Vigorous daily exercise has been shown to improve the ability to ronment (governmental agencies, regulatory bodies, societal norms,
cope with difficult situations but only improves mood less than etc.) (see Fig. e10.3). To effectively address burnout, interventions must
10% of the time. be directed at all levels of this complex system
c. Peer groups that allow for reflection and sharing on the meaning
inherent to the practice of medicine increase physician engage-
ment and decrease depersonalization. External • Health care industry
• Law, regulations, standards
d. Health care organizational engagement has little impact in indi- Environment • Societal values
vidual physician wellness but can improve efficiency of care in
Health care • Leadership/management
the work environment. • Governance
organization
e. All of the above. • Rewards and benefits
Answer: C. The current consensus suggests the importance of physi- Frontline care
• Organization conditions
• Technologies
cians focusing on their own well-being, but this alone is not enough delivery • Physicial environment
to prevent burnout. Physical exercise has been shown to improve the • Activities
mood of physicians and to help them cope with difficult situations. A Individual • Self-care
• Mindfulness
strong business case exists for organizations to invest in and promote practitioner • Meaning in medicine
engagement in combatting and reducing physician burnout, including
strategies of supporting executive leadership dedicated to improving
and sustaining professional well-being across organizations. Fig. e10.3 The four levels of the systems model of physician burnout.
3. Which of the following statements best describes physician burnout? (Modified from: National Academies of Sciences, Engineering, and
a. A non- work-related syndrome characterized by emotional Medicine. Taking Action Against Clinician Burnout: A Systems Approach
exhaustion, depersonalization, and a sense of reduced personal to Professional Well-Being. Washington DC: The National Academies
accomplishment. Press; 2019.)
Forensic Emergency Medicine e11
KEY CONCEPTS
• K nowledge of wound mechanics, production, and appearance can provide
practicing emergency clinicians with important clues regarding the forensic
interpretations of injuries.
• Wounds and injuries should be diagrammed and photographed.
• The medical record should accurately document objective findings associ-
ated with a patient’s wounds. Emergency clinicians should not speculate
about their mechanism or cause.
• For the chain of custody to be preserved, any evidence collected during the
course of treatment must be documented in the medical record, including to
whom the evidence was given.
1. When documenting a gunshot wound, it is important to do which from far enough away that only the bullet makes contact with the skin.
of the following? When an abrasion collar is the only visible superficial clinical finding
a. Describe the appearance, physical characteristics, and location present, the term indeterminate range describes the range of fire. Most
of wounds. entrance wounds will have an abrasion collar aside from those on the
b. Describe the bullet trajectory. palms and soles, which usually appear slit-like.
c. Determine the type of bullet used. 4. A 23-year-old woman presents after having been physically and
d. Estimate the caliber of the bullet. sexually assaulted by an unknown assailant. In addition to multiple
e. Interpret the wounds as “entrance” or “exit.” abrasions and brown contusions, she has a bite mark on her left
Answer: A. Emergency clinicians should not identify wounds as shoulder. Which of the following is not appropriate in your evi-
“entrance” or “exit.” Instead, their charting should include a detailed dence collection procedures and evaluation of this patient?
description, using appropriate forensic terminology, of the wound’s a. Estimating the age of the contusions based on their color
characteristics and location without speculating about its function or b. Measuring and documenting each visible injury on a diagram
the caliber or type of bullet (projectile) used to create it. c. Preserving the condition of the bite mark until after a forensic
2. A young man is brought to the emergency department following examination
an accidental rifle injury to his right leg. He has an obvious defor- d. Swabbing the bite mark with a sterile cotton-tipped applicator
mity to the extremity on examination with intact sensation and moistened with sterile water or saline
motor function, but pulses are diminished. Plain radiographs of the e. Taking photographs of the bite mark and each injury
extremity show comminuted tibial and fibular fractures. Which of Answer: A. When an acute bite mark is identified, take care that critical
the following is/are indicated in his management? evidence is not washed away. The skin surface should first be swabbed
a. Superficial débridement and wound closure with a sterile cotton-tipped applicator moistened with sterile saline.
b. CT angiography, prophylactic antibiotics, and operative man- Swabbing the area may reveal the assailant’s DNA present in dried
agement saliva. Remember, a contusion’s color is never a predictor of its age. The
c. Splinting and urgent orthopedic follow-up emergency clinician may be asked to render an opinion regarding the
d. Emergent fasciotomy age of a contusion. Because contusions develop as a result of multiple
e. Urgent surgical consultation variables, there is no reproducible standard for dating them.
Answer: B. High-energy gunshot wounds with associated fractures are 5. When managing a gunshot wound victim, which of the following is
managed as open fractures and are generally treated with prophylactic not important for forensic evidence collection?
antibiotics because contaminants can be sucked into the wound due to a. Do not clean the patient’s the hands with alcohol or Betadine.
cavitation. Operative intervention is required for patients with unsta- b. If soot is noted on the hands of the victim, cover with paper bags.
ble fractures, fractures with exposed bone, compartment syndrome, c. Maintain a chain of evidence.
or vascular injuries requiring repair. The latter is likely present in this d. Place each article of clothing in a separate paper bag.
patient given his diminished pulses and should be further evaluated e. Use metal forceps to handle a bullet to prevent contamination.
by CT angiography. These patients are also at risk for compartment Answer: E. Never remove a bullet with metal hemostats or pickups
syndrome and should be closely monitored. because metal tools can obliterate the microscopic markings (the tell-
3. An abrasion collar can be associated with which of the following tale fingerprint) of the gun from which it was shot. A victim’s clothing
types of wounds? may hold the answer to critical questions, such as “How far away was
a. Close-range wound the assailant who fired the weapon?” and “Which hole is the entrance
b. Contact wound and which the exit?” Articles of clothing removed from a wounded
c. Long-range or indeterminate-range wound patient should be placed in separate paper bags to preserve the trace
d. Intermediate-range wound evidence on them and to avoid accidentally transferring evidence from
e. Shotgun wound one article to another (cross-contamination). Always cover a patient’s
Answer: C. An abrasion collar (also referred to as an abrasion margin, hands with paper bags when the presence of soot is suspected. A
rim, or ring) results from skin indentation as the bullet penetrates the gunshot residue test may determine whether a victim or suspect has
surface. This collar is an abraded area of tissue that surrounds an entry been in close proximity to a weapon that has been fired. Factors that
wound as the result of friction between the bullet and epithelium. These decrease the sensitivity of gunshot residue tests include washing the
usually result from distant or long-range wounds, which are inflicted skin with alcohol or Betadine.
e12 Emergency Medical Services:
Overview and Ground Transport
KEY CONCEPTS
2. Which of the following best defines direct medical oversight in the
• P ublished in 1966, Accidental Death and Disability: The Neglected Dis-
prehospital setting?
ease of Modern Society by the National Academy of Sciences–National
a. Concurrent direction of EMTs providing patient care
Research Council was instrumental in emergency medical service (EMS)
b. Development of prehospital care policies
maturation in the United States.
c. Implement quality improvement program
• There are multiple models for EMS systems, including public and private
d. Use of prehospital patient care protocols
services, those operating at basic and advanced levels of care, and those
e. Use of standing field treatment protocols
including single or multiple tiers of response capability.
Answer: A. Direct medical oversight is the concurrent direction of
• There are four levels of prehospital providers recognized nationally—emer-
EMTs providing patient care, which can be at the scene or online over
gency medical responder (EMR), emergency medical technician (EMT),
radio or cell phone.
advanced emergency medical technician (AEMT), and paramedic, which is
3. Which of the following recognized the need to formally develop
the most advanced level.
EMS in the United States?
• The community paramedic provider focusing on population health issues
a. Highway Safety Act of 1973
such as access, chronic disease, and decreasing utilization and readmission
b. The EMS Agenda for the Future
is now being considered by many communities.
c. Accidental Death and Disability: The Neglected Disease of Modern
• Advances in emergency medical dispatching and positioning resources at
Society
locations and during specified times where expected call volumes are prev-
d. 1965 President’s Commission on Highway Safety
alent are innovations that are being implemented to decrease response
e. The Future of Emergency Care in United States Health System
times and improve outcomes.
Answer: C. The National Academy of Sciences–National Research
• Regulatory oversight for EMS systems lies at the individual state level,
Council published this document as a result of the growing concern for
and medical direction for individual public or private services resides at the
EMS and hospitals not being prepared to respond to the growing num-
local level.
ber of injuries resulting from traffic accidents on the nation’s highways.
• Direct medical oversight involves real-time interaction with the prehos-
pital providers via face-to-face or radio communications. Indirect medical 4. Which of the following prehospital interventions has been shown to
oversight involves off-line processes such as protocol development, quality improve patient outcome?
improvement, and education. a. Defibrillation
• Advances in prehospital care of medical patients have included analgesics b. Endotracheal intubation for children
and anxiolytics that can be administered intranasally, negating intravenous c. Endotracheal intubation in severe head trauma
routes, noninvasive measures to support patients in respiratory distress, d. Intraosseous needle placement
alternative adjuncts in place of endotracheal intubation for managing air- e. Needle cricothyrotomy
ways, and tourniquets for controlling hemorrhage. Answer: A. Rapid defibrillation has been shown to improve outcomes
• More advanced diagnostics (such as 12-lead electrocardiography and use for patients in cardiopulmonary arrest; other interventions have not
of stroke screens) have assisted in transporting patients to appropriate shown proven benefit in the prehospital setting.
facilities based on their illness and acuity. 5. The EMS Agenda 2050 describes a vision for EMS that is defined by
which of the following?
a. Availability of ambulance resources
1. Which of the following describes an EMS system that is contracted b. Centralized resources that can be deployed
to provide services? c. Community-based
a. Public utility d. People-centered
b. Third-service municipal Answer: D. The EMS Agenda 2050 describes a vision for EMS that is
c. Private people-centered along with six guiding qualities: inherently safe and
d. Fire-based effective, integrated and seamless, socially equitable, reliable and pre-
e. Hospital-based pared, sustainable and efficient, adaptable and innovative.
Answer A. A public utility model is a hybrid between private and pub-
lic design that allows local government to contract with a private or
public provider.
Air Medical Transport e13
KEY CONCEPTS
• A ir medical transport (AMT) is a critical component of a comprehensive
health care system and a vital link for rural communities and critical access
hospitals to distant emergency care.
• Boyle’s law and Dalton’s law have the greatest impact and explain the
development of hypoxia and most common altitude-related symptoms.
Other stresses of flight that can affect the patient or crew include tempera-
ture fluctuations, dehydration, noise, and vibration.
• Although most flight programs do both primary (scene flights) and second-
ary (interfacility) response, ground ambulance remains the primary means
of out-of-hospital and interfacility patient transport.
• The helicopter offers several advantages over other transport vehicles,
including reduction in travel time by up to 75%, ability to avoid common
ground delays (traffic, obstacles, and so on), and ability to fly into locations
that may be inaccessible to other modes of travel.
• All air medical services require involvement of a medical director responsi-
ble for supervising, evaluating, and ensuring the quality of medical care.
• As a general rule, helicopters are less useful in urban settings because of
the proximity of health care facilities.
• Helicopter emergency medical services (HEMS) represents the only modal- 3. A working knowledge of aviation physiology is vital to understand-
ity by which nearly 28% of United States residents have timely access ing the effects of AMT on pilots, medical personnel, and patients.
(within 1 hour) to level I or level II trauma centers. Which of the following statements is correct regarding the aviation
• HEMS may benefit in other time-critical situations, including ST-elevation physiology gas laws?
myocardial infarction (STEMI) patients going to a catheterization laboratory a. Boyle’s law explains the physiologic effects of expansion and
and acute stroke patients going to regional stroke centers. contraction of gases within the closed spaces of the body that
• Safety is the predominant concern of air medical operations. Helicopter may occur with altitude change.
shopping, the practice of a requesting EMS agency or hospital calling b. Charles’ law can be shown as Pt = P1 + P2 + P3…Pn.
numerous HEMS programs after other programs have declined the flight c. Dalton’s law accounts for the gas changes that result in decom-
because of bad weather, must be avoided. pression sickness.
d. Henry’s law explains why the ambient temperature decreases
with increased altitude.
1. Which of the following statements is true regarding air medical Answer: A.
transport (AMT)? 4. Which of the following statements is correct with regard to landing
a. AMT does little to improve outcomes when used for STEMI zone safety?
transport from rural areas. a. During night operations, spotlights should be toward the
b. AMT scene response for acute stroke is limited by emergency med- approaching aircraft.
ical service (EMS) providers’ inability to effectively triage and iden- b. If the aircraft is parked on a slope, approach and depart on the
tify patients who would likely benefit from HEMS scene response. downhill side.
c. Faster times to a trauma center are not required for outcomes c. In a rear-loading helicopter, it is safe to approach or depart from
benefit from AMT. the rear of the helicopter.
d. The emphasis for AMT of neonates is often more on the mode of d. Vehicles and personnel should be kept at least 50 ft from the
transport than on the transport team. landing zone.
Answer: C. Answer: B.
5. Determining which patient(s) might benefit from AMT is a critical
2. Safety is a critical consideration for all air medical operations.
decision for prehospital and interfacility personnel. Which of the
Which of the following statements is true?
following would represent a safe and appropriate request for AMT?
a. If the patient’s condition is severe, it is prudent to request service
a. Any burn victim being transferred to a regional burn center
from several AMT providers in the face of marginal weather.
b. Based on distance for pediatric trauma patients, regardless of the
b. Spectators should be kept at least 200 feet away from the landing
type and severity of injury
zone.
c. The preferred HEMS program has turned down the flight due
c. The use of a hospital helipad for emergency medical service
to weather conditions but there are other programs that can be
(EMS) rendezvous triggers the Emergency Medical Treatment
called
and Active Labor Act (EMTALA).
d. When the distance to the closest appropriate facility is too great
d. Vehicles and personnel should be kept at least 50 feet away from
for safe and timely transport by ground ambulance
the landing zone.
Answer: D.
Answer: B.
e14 Disaster Preparedness
KEY CONCEPTS
3. What is accurate advice that one would give to those planning a
• C omprehensive emergency management consists of four phases: mitiga disaster response?
tion, preparedness, response, and recovery. a. Critical incident stress debriefing is best conducted 1 or 2 weeks
• Mass casualty planning should account for the breakdown of traditional after the event.
transportation and communications systems during a disaster. b. Medical resupply systems are the most vulnerable component of
• Field personnel should be specifically trained in mass casualty triage and a disaster plan.
stabilization because austere field conditions change management strate c. One of the Department of Veterans Affairs (VA) system’s four
gies. legally mandated missions is emergency management.
• All plans must protect caregivers and rescue personnel. d. The Federal Emergency Management Agency (FEMA) has a
• Critical incident stress management is highly desirable after an event and coordinating responsibility for the entire spectrum of disasters.
should be planned for in advance. Psychological triage tools, such as Psy Answer: C. One of the VA health system’s four mandated missions is
START, may help. emergency management. Although controversy exists, critical incident
• Planners should establish and exercise a hospital-based incident manage stress debriefing, if used, should be implemented as early as possible.
ment system. The Department of Homeland Security (DHS) has full-spectrum disas-
• Disaster planning needs to include policies to address the needs of vulner ter coordinating responsibility. Communications systems are likely the
able populations such as children, disabled, and the elderly. most vulnerable systems.
• Multiple United States government agencies support disaster response,
4. Which of the following statements is not a feature of comprehensive
including the National Disaster Medical System, the Department of
emergency management?
Defense, the Department of Veterans Affairs, and the Centers for Disease
a. An incident management system consistent with NIMS should
Control and Prevention.
be implemented.
b. Health care facilities should perform a hazard vulnerability anal-
1. How can one characterize application of the Secondary Assessment ysis to assess community risks.
of Victim Endpoint (SAVE) triage guidelines? c. Most incident management systems are characterized by five
a. Triage decisions are based on patient outcome expectations. functional elements: incident command, operations, logistics,
b. It is applicable to day-to-day hospital operations. planning, and finance.
c. Patients are sorted immediately after the incident. d. The four phases of comprehensive emergency management
d. The guidelines provide detailed management of psychogenic (CEM) are pre-event, event, post-event, and recovery.
cases. Answer: D. The four phases of CEM are mitigation, preparedness,
Answer: A. The SAVE system is designed specifically for care in an response, and recovery. These phases represent a continuum over time,
austere environment. It triages patients into (1) those who will die and more than one phase may be in effect at the same time (e.g., even
regardless of treatment, (2) those who will live regardless of treatment, during the response phases, recovery actions may take place.)
and (3) those who would benefit from austere intervention. The triage 5. After an earthquake, what resource would be expected to provide
decisions are based on field outcome expectations from existing sur- the most rapid and effective search and rescue activity finding the
vival and morbidity statistics. most victims who would most likely survive?
2. Which of the following best describes the National Disaster Medi- a. Urban search and rescue teams
cal System (NDMS)? b. Private citizens
a. It does not include private sector institutions. c. Disaster medical assistance teams
b. It does not involve the Department of Veterans Affairs (VA). d. National Guard Civil Support Teams
c. The medical response element consists of Department of Defense Answer: B. Uninjured survivors perform the most rapid and effective
(DoD) personnel. search and rescue activity after an earthquake. These private citizens
d. The Department of Health and Human Services (HHS) has will recover most victims who ultimately survive. Although formal
oversight. urban search and rescue teams will eventually arrive, they do not find
Answer: D. The NDMS is a federally coordinated initiative that is a many victims and it is unknown how many survive. National Guard
cooperative program between the DoD, VA, DHHS, and Department Civilian Support Teams and disaster medical assistance teams do not
of Homeland Security (DHS) with oversight provided by DHHS. The perform search and rescue activities.
NDMS provides a system of coordinated mutual aid agreements among
federal, state, local, and private institutional entities for resource and
personnel provision in times of disaster. The medical response element
includes dozens of volunteer civilian medical teams that supplement
the local medical infrastructure.
Weapons of Mass Destruction e15
2. Which of the following statements best describes issues in manage-
KEY CONCEPTS
ment of anthrax infection?
• E mergency department (ED) preparedness for a radiation incident should a. Antibiotics do not change the mortality of cutaneous disease.
address decontamination (an external freestanding decontamination unit b. Cutaneous anthrax lesions are nontender.
is best), triage, staff safety, personal protective equipment (PPE), and diag- c. Doxycycline or ciprofloxacin are both options for single-agent
nostic procedures that emphasize radiation monitoring. It is important that treatment of symptomatic inhalational anthrax.
emergency personnel know their radiation safety officer. d. Sputum culture and Gram stain obtained early in the disease
• External decontamination of radioactive particles can be achieved by help differentiate inhalational anthrax.
removing clothing and washing the skin with soap and water. Answer: B. Cutaneous anthrax causes a severe, although nontender,
• Management of acute, life-threatening conditions (such as major traumatic peripheral vesicle that progresses to an eschar accompanied by
injuries) takes priority over radiation-associated issues. regional adenopathy. Antibiotics lower the mortality from cutaneous
• Aerosol dispersal is a likely route that terrorists may use to deploy biologic anthrax twenty-fold. For inhalational, gastrointestinal, and cutane-
weapons, so victims will present primarily with respiratory complaints. ous anthrax with toxicity, intravenous (IV) treatment is with cip-
• In addition to influenza-like illness (ILI) symptoms, anthrax typically causes rofloxacin or doxycycline plus at least two other agents. Regarding
mediastinal widening, pulmonary consolidation, and pleural effusions best inhalational anthrax, mechanical ventilation may not improve mor-
seen on chest computed tomography (CT) scans. tality and sputum cultures and Gram stains are not helpful until late
• Smallpox can spread in a hospital environment; thus, patients thought to in the disease.
have smallpox should be admitted to locations isolated from the rest of the 3. Several children ages 5 to 8 years old have definitely been exposed
hospital. to anthrax spores. Health department officials have brought these
• Decontamination is a key activity in the management of patients exposed children to the emergency department (ED). They are all ambula-
to chemical agents and frequently first occurs on emergency department tory with normal vital signs and without symptoms. Which of the
arrival. following is the most appropriate management?
• Nerve agents are organophosphates; patients exposed to these agents are a. Admission for parenteral penicillin G
treated with large doses of atropine (repeated frequently), pralidoxime, and b. Doxycycline for 5 days
benzodiazepines. c. Outpatient ciprofloxacin for 60 days or until the children have
• Patients with blast injuries can have subtle and delayed presentations of received three doses of vaccine
life-threatening conditions. d. Penicillin VK for 7 to 10 days
Answer: C. See Box e15.5. For children without toxicity, ciprofloxa-
1. Three patients arrive at triage simultaneously: One has received a 4 cin, doxycycline, or amoxicillin orally is indicated for a minimum of 60
Gy work-related irradiation exposure from a food sterilizer but no days or until the child has received three doses of vaccine. The vaccine
other injury; one is experiencing an acute ST elevation myocardial has not been approved for children but may be indicated to reduce the
infarction (MI); and one likely has urosepsis but with a stable blood long-term exposure to antibiotics. Note that weaponized anthrax may
pressure and heart rate of 105 beats/min. Which of these actions is be penicillin-resistant.
your first priority? 4. An individual is exposed to sarin vapor. She presents complaining
a. Activate the cardiac catheterization team. of difficulty with vision, salivation, vomiting, and the urge to defe-
b. Decontaminate the irradiated victim before placing him in a cate. The most appropriate treatment for this patient is which of the
room. following?
c. Initiate intravenous (IV) fluid bolus and prepare for intubation a. 5 mg diazepam intravenous (IV)
of the radiation-exposed patient. B. 6 mg atropine IV and 1 g 2-PAM IV every hour for a total dose
d. Place a central venous catheter in the urosepsis patient. of 3 g
Answer: A. Even patients who have received a lethal radiation C. 10 mg diazepam intramuscular (IM) via autoinjector
dose do not die quickly as a consequence of the ionizing exposure. D. One or two Mark 1 or the DuoDote autoinjector kits IM
Patients should be triaged according to severity of the medical Answer: D. The victim described would be characterized as a moderate
conditions and/or vital sign derangements. Here, the patient with exposure to sarin vapor. Treatment is indicated with one or two Mark 1
the MI is most acute and should be the highest priority necessitat- or DuoDote autoinjectors IM. Observation would not be appropriate.
ing activation of the cardiac catheterization team. The radiation- Diazepam is not indicated for moderate exposures. If an IV is estab-
exposed patient was not contaminated, just irradiated. As such, lished, the initial treatment is atropine 2 to 4 mg IV and 2-PAM 1 g IV.
decontamination and isolation are unnecessary. Although the vic- 5. Which of the following statements regarding blast injury is true?
tim has received the LD50 dose of ionizing radiation, no significant a. Classically, there are three different types of blast injury.
injury will result just after exposure, so this patient does not take b. Primary blast injury is caused by flying debris and bomb fragments.
priority over the MI patient at this point and intubation is not indi- c. Exacerbation of asthma is an example of tertiary blast injury.
cated. The septic patient is not critically ill and should not be treated d. Secondary blast injury is the most common cause of morbidity
ahead of the MI patient. and mortality following a blast event.
Answer: D. Of the four types of blast injury, secondary blast injury is
the most common cause of morbidity and mortality. These injuries are
caused by flying debris generated by the explosion. An asthma exacer-
bation could be considered a quaternary injury in the correct setting.
Tympanic membrane rupture is not an accurate predictor of internal
injuries in blast events.
Tactical Emergency Medical Support
e16 and Urban Search and Rescue
KEY CONCEPTS
• T actical emergency medical support (TEMS) facilitates the overall success
and safety of law enforcement missions during all phases of a tactical opera-
tion through the delivery of preventive, urgent, and emergency medical care.
• A fundamental principle in tactical medicine is that the medical mission
may be subordinate to the overall law enforcement mission.
• Tactical combat casualty care (TCCC) has adapted civilian advanced life
support principles to provide medical care during a hostile force encounter.
Its goals are to treat the casualty, prevent additional casualties, and com-
plete the mission.
• TCCC is divided into three phases of care: care under fire (CUF), tactical field
care (TFC), and combat casualty evacuation care (CASEVAC).
• Tactical emergency casualty care (TECC) addresses casualty management 3. The principles of TEMS have largely developed from which type of
during high-threat civilian tactical and rescue operations and is divided into incidents?
three phases; direct threat, indirect threat, and evacuation care. a. Civilian multicasualty events
• Urban search and rescue (USAR) encompasses responding to, locating, b. Military conflicts
reaching, medically treating, and safely extricating victims entrapped by c. Natural disasters
collapsed structures. The primary role of the emergency clinician is support d. Pandemics
of the health and welfare of the team members, including canines.
• USAR teams often treat crush syndrome, particulate inhalation, hazardous Answer: B. The principles of TEMS have largely developed from les-
materials exposures, and blast injuries. sons learned during military conflicts and most closely emulate the
• In crush syndrome, treatment with fluids begins prior to extrication to avoid medical support structure of military special operations units.
life-threatening complications once the patient is extricated. 4. Which of the following reflects an important distinction between
tactical medicine and stand EMS practice?
a. Airway compromise is treated before hemorrhage.
1. Which of the following is a priority for the tactical emergency med- b. Medical care may be delayed in order to accomplish the law
ical support (TEMS) provider in caring for a victim shot during a enforcement mission.
hostage incident? c. Care under fire requires the immediate movement of patients
a. Bag-valve-mask ventilation from the scene.
b. Cervical spine stabilization d. Tourniquet use is secondary to other forms of hemorrhage con-
c. Intravenous access and fluid resuscitation trol.
d. Tourniquet application to bleeding extremity Answer: B. A fundamental principle in tactical medicine is that the
Answer: D. This scenario describes care rendered by TEMS in the medical mission may be subordinate to the overall law enforcement
hot zone, or care under fire; thus, conventional care sequences do not mission. In contrast to conventional EMS and hospital practices, in
apply. Priorities include mitigation of threats, tourniquet use for seri- which the sole priority is usually the health and welfare of the patient,
ous bleeding, and evacuation to a safer location. the essential priority in a tactical mission is the success of the law
2. A 41-year-old man has been trapped under a piece of large crane enforcement objective. When a casualty occurs during a tactical oper-
that overturned during a partial construction site collapse. The ation, medical providers may be directed to delay or modify medical
patient has been trapped for over 7 hours and the rescue team is care until the tactical commander determines that rendering care will
actively working to extricate him. His right leg is both mangled and not jeopardize the overall mission. Bleeding control is emphasized
pinned up to the level of his pelvis. He is conscious, in pain, has a with rapid use of tourniquets. In care under fire patients should not be
systolic blood pressure of 140, a heart rate 112, Spo2 97%, RR 20. moved unless the danger is alleviated.
He received pain medication and fluid resuscitation. The monitor 5. Which of the following is the primary responsibility of the USAR
shows a widening QRS interval. Which of the following treatments team emergency clinician?
should be initiated? a. Ensuring a public health framework exists for the region
a. Blood products b. Ensuring that entering a collapsed building is safe
b. Calcium c. Medical treatment of entrapped victims
c. Epinephrine d. Supporting the health and welfare of team members
d. Hypertonic saline Answer: D. The USAR team emergency clinician must be familiar
Answer: B. Prolonged entrapment raises the concern for crush injury, with the types of medical conditions that may be encountered by the
which increases after 4 hours of immobilization. Continuous cardiac team. The USAR team emergency clinician has a primary responsibil-
monitoring during extrication is indicated to monitor the development ity to the team members should they require medical attention. Sec-
of arrhythmias which may occur in the setting of hyperkalemia. Evi- ondarily, treatment of entrapped patients can be done but is not their
dence of hyperkalemia should be treated and includes calcium. There primary mission. Ensuring broader elements of a public framework in
is no clinical evidence to suggest hemorrhagic shock or uncontrolled a disaster-affected area is typically beyond the conventional role of the
bleeding to warrant blood product administration. Fluid resuscita- USAR clinician, though this individual may liaison with other health
tion may help prevent the hypotension associated with release of the and medical personnel in the same geographic area of operations.
entrapped limb(s) and should be initiated before extrication.